You are on page 1of 347

Pan Malayan Insurance Corp. v. CA (Hilario) 2.

  Through the negligence of Erlinda Fabie’s (FABIE) unknown driver (he


April 3, 1990 | Cortes, J. | Subrogation was driving a pick-up), the insured car was hit and suffered damages worth
P42, 052.
PETITIONER: Pan Malayan Insurance Corp. 3.   Being the insurance company, Pan Malayan paid for the cost of repair of the
RESPONDENTS: Court of Appeals, Erlinda Fabie and her unkown driver insured car and was therefore subrogated to the rights of CANLUBANG
against the driver of the pick-up and his employer Erlind Fabie.
SUMMARY: Pan Malayan (PANMALAY) insured a Mitsubishi Colt Lancer. The 4.   Despite repeated demands, Fabie failed and refused to pay the claim of
car was registered under the name of Canlubang Automotive Resources Corporation PANMALAY, so PANMALAY filed a complaint for damages before the
(CANLUBANG). Through the negligence of Erlinda Fabie’s (FABIE) unknown RTC of Makati against Fabie and her driver.
driver (he was driving a pick-up), the insured car was hit and suffered damages 1.   PANMALAY said that the damage caused to the insured car was
worth P42, 052. Being the insurance company, Pan Malayan paid for the cost of settled uner the “own damage” coverage of the insurance policy,
repair of the insured car and was therefore subrogated to the rights of CANLUBANG and submitted a copy of the insurance policy and the Release of
against the driver of the pick-up and his employer Erlind Fabie. Despite repeated Claim and Subrogation Receipt executed by CANLUBANG in
demands, Fabie failed and refused to pay the claim of PANMALAY, so favor of PANMALAY.
PANMALAY filed a complaint for damages before the RTC of Makati against Fabie 5.   FABIE filed a Motion to Dismiss saying PANMALAY had no cause of
and her driver. PANMALAY said that the damage caused to the insured car was action against them, and claimed that the “own damage” clause of the
settled uner the “own damage” coverage of the insurance policy, and submitted a insurance policy precluded subrocation since indemnification in Art. 2207
copy of the insurance policy and the Release of Claim and Subrogation Receipt of the Civil Code1 was made on the assumption that there was no
executed by CANLUBANG in favor of PANMALAY. FABIE filed a Motion to wrongdoer or no third party at fault.
Dismiss saying PANMALAY had no cause of action against them, and claimed that 6.   RTC dismissed PANMALAY’s complaint for no cause of action, and this
the “own damage” clause of the insurance policy precluded subrogation since was upheld by the CA. So now, PANMALAY comes before the SC.
indemnification in Art. 2207 of the Civil Code was made on the assumption that
there was no wrongdoer or no third party at fault. RTC dismissed PANMALAY’s ISSUE/s:
complaint for no cause of action, and this was upheld by the CA. So now, 1.   WON PANMALAY (insurer) may institute an action to recover the amound
PANMALAY comes before the SC. Issue: WON PANMALAY (insurer) may it had paid CANLUBANG (assured) in settlement of an insurance claim
institute an action to recover the amound it had paid CANLUBANG (assured) in against FABIE? – YES, PANMALAY has cause of action because payment
settlement of an insurance claim against FABIE? YES, because payment by the by the insurer to the assured operates as an equitable assignment to the
insurer to the assured operates as an equitable assignment to the assured of all assured of all remedies with the latter may have against the third party
remedies with the latter may have against the third party whose negligence or whose negligence or wrongful act caused the loss.
wrongful act caused the loss. The right of subrogation is not dependent upon, nor
does it grow out of, any privity of contract or upon written assignment of claim-- it RULING: WHEREFORE, in view of the foregoing, the present petition is
accrues simply upon payment of the insurance claim by the insurer. See ratio 3 GRANTED. Petitioner's complaint for damages against private respondents is hereby
for exceptions to that rule (but none of them apply here J). REINSTATED. Let the case be remanded to the lower court for trial on the merits.

RATIO:
DOCTRINE: Payment by the insurer to the assured operates as an equitable 1.   Payment by the insurer (PANMALAYA) to the assured (CANLUBANG)
assignment to the assured of all remedies with the latter may have against the third operates as an equitable assignment to the assured (CANLUBANG) of
party whose negligence or wrongful act caused the loss. The right of subrogation all remedies which the latter may have against the third party whose
accrues simply upon payment of the insurance claim by the insurer. negligence or wrongful act caused the loss.

                                                                                                                       
FACTS: 1
 Art.  2207.  If  the  plaintiff's  property  has  been  insured,  and  he  has  received  indemnity  from  
1.   Pan Malayan (PANMALAY) insured a Mitsubishi Colt Lancer. The car was
the  insurance  company  for  the  injury  or  loss  arising  out  of  the  wrong  or  breach  of  contract  
registered under the name of Canlubang Automotive Resources Corporation complained   of,   the   insurance   company   shall   be   subrogated   to   the   rights   of   the   insured  
(CANLUBANG). against  the  wrongdoer  or  the  person  who  has  violated  the  contract.  If  the  amount  paid  by  
the   insurance   company   does   not   fully   cover   the   injury   or   loss,   the   aggrieved   party   shall   be  
entitled  to  recover  the  deficiency  from  the  person  causing  the  loss  or  injury.  
2.   The right of subrogation is not dependent upon, nor does it grow out of, any
privity of contract or upon written assignment of claim-- it accrues simply
upon payment of the insurance claim by the insurer.
3.   Exceptions to this rule:
1.   If the assured by his own act releases the wrongdoer or third party
liable for the loss or damage, from liability, the insurer's right of
subrogation is defeated; and
2.   Where the insurer pays the assured the value of the lost goods
without notifying the carrier who has in good faith settled the
assured's claim for loss, the settlement is binding on both the
assured and the insurer, and the latter cannot bring an action
against the carrier on his right of subrogation; and
3.   Where the insurer pays the assured for a loss which is not a risk
covered by the policy, thereby effecting "voluntary payment", the
former has no right of subrogation against the third party liable for
the loss. (But see ratio 5)
4.   (re: why the lower court is wrong in interpreting “own cause” clause; just in
case sir asks) It must be emphasized that the lower court's ruling that the
"own damage" coverage under the policy implies damage to the insured car
caused by the assured itself, instead of third parties, proceeds from an
incorrect comprehension of the phrase "own damage" as used by the
insurer. When PANMALAY utilized the phrase "own damage" — a phrase
which, incidentally, is not found in the insurance policy — to define the
basis for its settlement of CANLUBANG's claim under the policy, it simply
meant that it had assumed to reimburse the costs for repairing the
damage to the insured.
5.   Even if PANMALAY could not be deemed subrogated to the rights of its
assured under Article 2207 of the Civil Code, PANMALAY would still
have a cause of action against private respondents. In the pertinent case of
Sveriges Angfartygs Assurans Forening v. Qua Chee Gan, the Court ruled
that the insurer who may have no rights of subrogation due to
"voluntary" payment may nevertheless recover from the third party
responsible for the damage to the insured property under Article 1236
of the Civil Code.
002 FEDERAL EXPRESS CORPORATION v. AMERICAN HOME FACTS:
ASSURANCE COMPANY and PHILAM INSURANCE COMPANY, 1.   SMITHKLINE Beecham (SMITHKLINE) of Nebraska, USA delivered to
INC (HIRANG) Burlington Air Express (BURLINGTON), an agent of Federal Express
August 18, 2004 |Panganiban, J. | Subrogation Corporation, a shipment of 109 cartons of veterinary biologicals for
delivery to consignee SMITHKLINE and French Overseas Company in
Makati City, Metro Manila.
PETITIONER: FEDERAL EXPRESS CORPORATION 2.   The shipment reads REFRIGERATE WHEN NOT IN TRANSIT and
RESPONDENTS: AMERICAN HOME ASSURANCE COMPANY and PHILAM PERISHABLE stamp marked on its face
INSURANCE COMPANY, INC 3.   BURLINGTON insured the cargoes in the amount of $39,339.00 with
American Home Assurance Company (AHAC)
SUMMARY: SMITHKLINE, USA delivered to BURLINGTON (Agent of 4.   BURLINGTON turned over the custody of said cargoes to FEDERAL
FEDERAL EXPRESS) 109 cartons of veterinary biologicals for delivery to EXPRESS which transported the same to Manila.
consignee SMITHKLINE and French Overseas Company in Makati City, Metro 5.   They were shipped in two batches. The first one consist of 92 cartons and
Manila. The same was insured by BURLINGTON in the amount of $39,339.00 with the next one came 2 days after which consist of 17 cartons. These were
American Home Assurance Company (AHAC). BURLINGTON then subsequently immediately stored at Cargohaus Inc’s warehouse.
turned over the goods to FEDERAL EXPRESS which then shipped the cargoes to 6.   Twelve (12) days after the cargoes arrived in Manila, a non-licensed
the consignee SMITHKLINE and French Overseas Company in Makati City, Metro customs broker, DARIO C. DIONEDA (DIONEDA) who was assigned by
Manila. The cargoes was then stored in Cargohaus’ warehouse. The cargo reads GETC (Custom broker Corporation hired by SMITHKLINE) to facilitate
REFRIGERATE WHEN NOT IN TRANSIT and PERISHABLE stamp marked on the release of the subject cargoes, found out, that the goods were stored
its face. 12 days after the shipment, DARIO C. DIONEDA, the broker of the GETC only in a room with two (2) air conditioners running, to cool the place
(Custom Broker Corporation hired by SMITHKLINE to release the goods) found out instead of a refrigerator
that the goods were stored inside a room with only 2 aircondition units. Upon 7.   When he asked an employee of Cargohaus why the cargoes were stored in
instructions for GETC, Dioneda did not proceed with the withdrawal of the vaccines the cool room only, the latter told him that the cartons where the vaccines
but instead samples were taken to the Bureau of Animal Industry and there it was were contained specifically indicated therein that it should not be subjected
discovered that the ELISA reading of vaccinates sera are below the positive to hot or cold temperature.
reference serum. Because of the unusable vaccines, SMITHKLINE abandoned the 8.   Upon instructions from GETC, Dioneda did not proceed with the
shipment and filed a claim against AHAC thorught its representative in the withdrawal of the vaccines but instead samples were taken to the Bureau
Philippines, PHILAM. PHILAM paid SMITHKLINE and subsequently filed a case of Animal Industry and there it was discovered that the ELISA reading
for recovery of damages against FEDERAL EXPRESS and Cargohaus, imputing of vaccinates sera are below the positive reference serum.
negligence in handling the cargo. FEDERAL EXPRESS contends that PHILAM has 9.   As a result, SMITHKLINE abandoned the shipment and declared a total
no personality to sue and thus, it has no cause of action against it. The issue in this loss for the unusable shipment of vaccines. SMITHKLINE then filed a
case is WoN, PHILAM has the capacity to sue and the court held in the affirmative. claim against AHAC through its representative in the Philippines, Philam
The SC held that upon receipt of the insurance proceeds, SMITHKLINE executed a Insurance, the whole amount of the cargo.
subrogation Receipt in favor of PHILAM. PHILAM was thus authorized to file 10.   PHILAM paid SMITHKLINE the whole amount and subsequently it
claims and begin suit against any such carrier, vessel, person, corporation or (PHILAM) filed an action for damages against FEDERAL EXPRESS and
government. Upon payment to the consignee of an indemnity for the loss of or Cargohaus imputing negligence in handling the cargo.
damage to the insured goods, the insurers entitlement to subrogation pro tanto --
11.   The Trial Court ruled in favor of PHILAM. FEDERAL EXPRESS and
being of the highest equity -- equips it with a cause of action in case of a contractual
Cargohaus were held to be solidarily liable.
breach or negligence. Further, the insurers subrogatory right to sue for recovery
12.   The CA also ruled in favor of PHILAM stating that the goods were
under the bill of lading in case of loss of or damage to the cargo is jurisprudentially
delivered to FEDERAL EXPRESS in good condition as provided for in the
upheld.
shipping receipts. FEDERAL EXPRESS was not able to discharge the
presumption that the goods were indeed delivered to them in good condition
DOCTRINE: In the exercise of its subrogatory right, an insurer may proceed against
thus, creating a presumption that they were the one’s negligent in handling
an erring carrier. To all intents and purposes, it stands in the place and in substitution
the goods.
of the consignee. A fortiori, both the insurer and the consignee are bound by the
contractual stipulations under the bill of lading
13.   FEDERAL EXPRESS contends that PHILAM has no personality to Burlington. This document has thus been duly indorsed in blank and is
sue and thus, no cause of action against it - because the payment made deemed a bearer instrument.
to SMITHKLINE was erroneous.
2.   Since the Certificate was in the possession of Smithkline, the latter had the
ISSUE/s: right of collecting or of being indemnified for loss of or damage to the
-WoN payment made to SMITHKLINE was erroneous – NO, SMITHKLINE is insured shipment, as fully as if the property were covered by a special
the holder of the Certificate of Insurance and under this, it had the right of policy in the name of the holder. Hence, being the holder of the Certificate
collecting or of being indemnified for loss of or damage to the insured shipment and having an insurable interest in the goods, Smithkline was the proper
-WoN PHILAM had the personality to sue – YES, Upon receipt of the insurance payee of the insurance proceeds.
proceeds, Smithkline executed a subrogation Receipt in favor of respondents. The (CLAIM ALREADY PRESCRIBED)
latter were thus authorized to file claims and begin suit against any such carrier,
vessel, person, corporation or government. (SUBROGATION ISSUE) 1.   In this jurisdiction, the filing of a claim with the carrier within the time
-WoN Federal Express is liable for the damage to or loss of the insured limitation therefor actually constitutes a condition precedent to the accrual
products? – NO, SC stated that the reason they are not liable is because of the fact of a right of action against a carrier for loss of or damage to the goods. The
that the claim has already prescribed. The filing of the claim within the time limit is a shipper or consignee must allege and prove the fulfillment of the condition.
condition precedent to the accrual of a right of action against a carrier for loss of or If it fails to do so, no right of action against the carrier can accrue in favor
damage to the goods. of the former. The aforementioned requirement is a reasonable condition
precedent; it does not constitute a limitation of action
RULING: WHEREFORE, the Petition is GRANTED, and the assailed 2.   In this case, even from the very start of the trial, PHILAM never submitted
Decision REVERSED insofar as it pertains to Petitioner Federal Express any evidence to prove that it indeed made a complaint in writing when it
Corporation. No pronouncement as to costs. found out that the goods were damaged
3.   According to the Airway bill (contract of carriage), the person entitled
RATIO: to delivery must make a complaint in writing in case of visible and
(SUBROGATION ISSUE) other damages within 14 days from the receipt of the goods.
1.   Upon receipt of the insurance proceeds, Smithkline executed a subrogation 4.   Nothing from the records show that PHILAM indeed made a complaint.
Receipt in favor of PHILAM. The latter was thus authorized to file claims 5.   The requirement of giving notice of loss of or injury to the goods is not an
and begin suit against any such carrier, vessel, person, corporation or empty formalism. The fundamental reasons for such a stipulation are (1) to
government. inform the carrier that the cargo has been damaged, and that it is being
2.   Upon payment to the consignee of an indemnity for the loss of or damage to charged with liability therefor; and (2) to give it an opportunity to examine
the insured goods, the insurers entitlement to subrogation pro tanto -- being the nature and extent of the injury. This protects the carrier by affording it
of the highest equity -- equips it with a cause of action in case of a an opportunity to make an investigation of a claim while the matter is fresh
contractual breach or negligence. Further, the insurers subrogatory right to and easily investigated so as to safeguard itself from false and fraudulent
sue for recovery under the bill of lading in case of loss of or damage to the claims
cargo is jurisprudentially upheld 6.   When an airway bill -- or any contract of carriage for that matter -- has a
3.   In the exercise of its subrogatory right, an insurer may proceed against an stipulation that requires a notice of claim for loss of or damage to goods
erring carrier. To all intents and purposes, it stands in the place and in shipped and the stipulation is not complied with, its enforcement can be
substitution of the consignee. A fortiori, both the insurer and the consignee prevented and the liability cannot be imposed on the carrier. To stress,
are bound by the contractual stipulations under the bill of lading notice is a condition precedent, and the carrier is not liable if notice is not
given in accordance with the stipulation. Failure to comply with such a stipulation
OTHER ISSUES: bars recovery for the loss or damage suffered.
(ERRONEOUS PAYMENT TO SMITHKLINE)
1.   The Certificate specifies that loss of or damage to the insured cargo is
payable to order x x x upon surrender of this Certificate. Such wording
conveys the right of collecting on any such damage or loss, as fully as if the
property were covered by a special policy in the name of the holder itself.
At the back of the Certificate appears the signature of the representative of
03 FIREMAN’S FUND INSURANCE CO vs. JAMILA (LAGUILLES) 5.   The lower court dismissed the complaint as to Jamila on the ground that
April 7, 1976 | Aquino, J. | Art. 2207 there was no allegation that it had consented to the subrogation, and,
therefore, Fireman’s Fund had no cause of action against it.
PETITIONER: Fireman’s Fund Insurance Company and Firestone Tire and 6.   The lower court also dismissed the complaint as to First Quezon City
Rubber Company of the Philippines Insurance Co on the ground of res judicata. It appears that the same action
RESPONDENTS: Jamila & Company, Ic. and First Quezon City Insurance Co was previously filed which was dismissed because of the failure of the
Inc. plaintiffs to appear at pre-trial.
7.   Firestone and Fireman’s Fund moved for the reconsideration of the order of
SUMMARY: Jamila contracted to supply security guards to Firestone for dismissal, and the lower court set aside its order of dismissal. It sustained
which Jamila assumed responsibility for the acts of its security guards. With plaintiff’s contention that there was no res judicata as to First Quezon City
this, First Quezon City Insurance Co (surety) executed a bond in the sum of Insurance Co because the case was dismissed without prejudice.
P20,000 to guarantee Jamila’s obligations under the contract. Firestone 8.   Due to inadvertence, the lower court did not state why it set aside its prior
properties worth P19,000 were lost allegedly due to the acts of its employees order dismissing the complaint with respect to Jamila.
who connived with Jamila’s security guards. Fireman’s Fund as insurer, paid to 9.   Jamila, upon noticing that the order had obliterated its victory without any
Firestone the amount of the loss and Fireman’s Fund was subrogated reason therefore, filed a motion for reconsideration. It originally moved for
(substituted) to Firestone’s right to get reimbursed from Jamila. However, the dismissal of the complaint on the ground of lack of cause of action. Its
Jamila and its surety failed to pay. The lower court dismissed the complaint as contention was based on two grounds:
to Jamila on the ground that the latter never consented to the subrogation, hence a.   that the complaint did not allege that Firestone, pursuant to the
there is no cause of action. The issue is WoN the complaint states a cause of contractual stipulation quoted in the complaint, had investigated
action against Jamila. the loss and that Jamila was represented in the investigation
b.   That Jamila did not consent to the subrogation of Fireman’s Fund
The SC held that yes, there is a cause of action because as insurer, Fireman’s to Firestone’s right to get reimbursement from Jamila and its
Fund is entitled to go after the person or entity that violated its contractual surety.
commitment. Subrogation is a normal incident of indemnity insurance. Upon 10.   The lower court sustained the second ground. However, Jamila in its MR
payment of the loss, the insurer is entitled to be subrogated pro tanto to any invoked the first ground which was never passed upon by the lower court.
right of action which the insured may have against the third person whose However, the lower court completely ignored the first ground and reverted
negligence or wrongful act caused the loss. The loss in the first instance is that to the second ground which was relied upon in its order.
of the insured but after reimbursement, it becomes the loss of the insurer. 11.   The lower court reiterated that Fireman’s Fund had no cause of action
against Jamila because Jamila did not consent to the subrogation.
DOCTRINE: Upon payment of the loss, the insurer is entitled to be subrogated 12.   Plaintiffs filed a second motion for reconsideration and called the lower
pro tanto to any right of action which the insured may have against the third court’s attention to the fact that the issue of subrogation was of no moment
person whose negligence or wrongful act caused the loss. because Firestone, the subrogor, is a party-plaintiff and could sue directly
Jamila in its own right.
FACTS: 13.   The plaintiffs cited Art. 2207 of the Civil Code which provides:
1.   Jamila or the Veterans Philippine Scouts Security Agency contracted to “If the plaintiff’s property has been insured, and he has received
supply security guards to Firestone for which Jamila assumed responsibility indemnity from the insurance company for the injury or loss arising out
for the acts of its security guards. of the wrong or breach of contract complained of, the insurance
2.   First Quezon City Insurance Co., Inc. executed a bond in the sum of company shall be subrogated to the rights of the insured against the
P20,000.00 to guarantee Jamila’s obligations under the contract. wrongdoer or the person who has violated the contract.”
3.   However, properties of Firestone valued at P11,925.00 were lost allegedly 14.   Firestone and Fireman’s Fund contend that the trial court’s dismissal of
due to the acts of its employees who connived with Jamila’s security their complaint is contrary to the aforementioned provision which provides
guards. for legal subrogation.
4.   Fireman’s Fund, as insurer, paid to Firestone the amount of the loss and
Fireman’s Fund was subrogated to Firestone’s right to get reimbursement ISSUE/s:
from Jamila, and Jamila and its surety, First Quezon City Insurance Co, 1.   WoN the complaint of Firestone and Fireman’s Fund states a cause of
failed to pay the amount of the loss in spite of repeated demands. action against Jamila – YES, because as insurer, Fireman’s Fund is entitled
to go after the person or entity that violated its contractual commitment. equitable assignment to the insurer of the property and all remedies which
the insured may have for the recovery thereof.

RULING: Finding the trial court's order of dismissal to be legally untenable, the
same is SET ASIDE with costs against defendant-appellee Jamila & Co., Inc.

RATIO:
1.   Jamila is of the opinion that legal subrogation under Art. 2207 requires the
debtor’s consent; that legal subrogation takes place in the cases mentioned
in Art. 1302 of the Civil Code and the instant case is not among the three
cases enumerated in that article.
2.   The Court holds that Fireman’s Fund’s action against Jamila is sanctioned
by Art. 22071. As the insurer, Fireman’s Fund is entitled to go after the
person or entity that violated its contractual commitment to answer for
the loss insured against.
3.   The lower court erred in applying to this case the rules on novation. The
plaintiffs in alleging in their complaint that Fireman’s Fund became a party
in interest in this case by virtue of a subrogation right given in its favor by
Firestone, were not relying on the novation by change of creditors as
contemplated in Arts. 1291, and 1300 to 1303 of the Civil Code, but rather
on Art. 2207.
4.   Subrogation has been referred to as the doctrine of substitution. It is an arm
of equity that may guide or even force one to pay a debt for which an
obligation was incurred but which was in whole or in part paid by another.
5.   Subrogation is founded on principles of justice and equity, and its operation
is governed by principles of equity. It rests on the principle that substantial
justice should be attained regardless of form, that is, its basis is the doing of
complete, essential, and perfect justice between all the parties without
regard to form.
6.   Subrogation is a normal incident of indemnity insurance. Upon payment of
the loss, the insurer is entitled to be subrogated pro tanto to any right of
action which the insured may have against the third person whose
negligence or wrongful act caused the loss.
7.   The loss in the first instance is that of the insured but after reimbursement, it
becomes the loss of the insurer.
8.   Although many policies now provide for subrogation, and thus determine
the rights of the insurer in this respect, the equitable right of subrogation as
the legal effect of payment inures to the insurer without any formal
assignment or any express stipulation to that effect in the policy.
9.   When the insurance company pays for the loss, such payment operates as an

                                                                                                                       
1
  Art.  2207.  If  the  plaintiff’s  property  has  been  insured,  and  he  has  received  indemnity  from  
the  insurance  company  for  the  injury  or  loss  arising  out  of  the  wrong  or  breach  of  contract  
complained   of,   the   insurance   company   shall   be   subrogated   to   the   rights   of   the   insured  
against  the  wrongdoer  or  the  person  who  has  violated  the  contract.”  
004 FF. Cruz & Co., Inc v. CA (Marcos) the contents thereof.
Aug. 29, 1988 | Cortes, J. | Subrogation / Art. 2207 7.   Subsequently, the family filed an action for damages against FF. Cruz.
8.   The CFI ruled in favor of the Mable family, and later on the CA affirmed
PETITIONER: F.F. Cruz and Co., Inc. the decision of the trial court but reduced the award for damages.
RESPONDENTS: The Court of Appeals, Gregorio Mable as substituted by his 9.   Both Courts did not deduct the P35,000 from the award of damages.
wife Luz Almonte Mable and children Doming, Leonidas, Ligaya, Elena, 10.   FF. Cruz filed a petition for review with the SC after its MR with the CA
Gregorio, Jr., Salome, Antonio, and Bernardo all surnamed Mable. was dnid.
11.   FF. Cruz contends that the CA erred in not deducting the P35,000 recovered
SUMMARY: The manufacturing shop of FF. Cruz and the house of Mable was insurance from the award of damages, in awarding excessive and unproved
situated adjacent with each other. Mable repeatedly requested that a firewall be damages, and in applying the doctrine of res ipsa loquitur.
constructed between the shop and their house but such request fell on deaf ears.
A fire broke in the shop and it then spread to the residence. Both the shop and ISSUE/s:
the house were razed to the ground. The Mable family was able to recover 1.   WoN FF. Cruz is still liable to pay for the 35,000 recovered by the Mable
P35,000 from its insurance on the house. The family then filed an action for Family from its insurance – No, because Art. 2207 of the Civil Code
damages against FF. Cruz. The CFI ruled in favor of the family, and later on the provides that it should be deducted from the amount of award as damages.
CA affirmed the decision. Both Courts did not deduct the P35,000 from the
award of damages. The issue before the SC is WoN FF. Cruz is still liable to pay RULING: WHEREFORE, in view of the foregoing, the decision of the Court of
for the 35,000 recovered by the Mable Family from its insurance – No, because Appeals is hereby AFFIRMED with the following modifications as to the damages
Art. 2207 of the Civil Code provides that it should be deducted from the amount awarded for the loss of private respondents' house, considering their receipt of
of award as damages. Having been indemnified by their insurer, The Mable P35,000.00 from their insurer.
family are only entitled to recover the deficiency from FF. Cruz. The insurer, if it
is so minded, may seek reimbursement of the amount it indemnified the family RATIO:
from FF. Cruz. Under Article 2207, the real party in interest with regard to the 1.   Doctrine of res ipsa loquitur: Where the thing which caused the injury
indemnity received by the insured is the insurer. Whether or not the insurer complained of is shown to be under the management of the defendant or his
should exercise the rights of the insured to which it had been subrogated lies servants and the accident is such as in the ordinary course of things does not
solely within the former's sound discretion. happen if those who have its management or control use proper care, it
affords reasonable evidence, in the absence of explanation by the defendant,
DOCTRINE: The insurer, if it is so minded, may seek reimbursement of the that the accident arose from want of care.
amount it indemnified the family from FF. Cruz. This is the essence of its right 2.   The facts of the case likewise call for the application of the doctrine,
to be subrogated to the rights of the insured, as expressly provided in Article considering that in the normal course of operations of a furniture
2207. Upon payment of the loss incurred by the insured, the insurer is entitled to manufacturing shop, combustible material such as wood chips, sawdust,
be subrogated pro tanto to any right of action which the insured may have paint, varnish and fuel and lubricants for machinery may be found thereon
against the third person whose negligence or wrongful act caused the loss. 3.   It must also be noted that negligence or want of care on the part of
petitioner or its employees was not merely presumed.
4.   The Court of Appeals found that petitioner failed to construct a firewall
FACTS: between its shop and the residence of private respondents as required by a
1.   The furniture manufacturing shop of FF. Cruz in Caloocan was situated city ordinance; that the fire could have been caused by a heated motor or a
adjacent to the resident of respondent Mable family. lit cigarette; that gasoline and alcohol were used and stored in the shop; and
2.   Sometime in Aug 1971, Mable approached Cruz, the plant manager, to that workers sometimes smoked inside the shop
request that a firewall be constructed between the shop and the residence. 5.   In the instant case, with more reason should petitioner be found guilty of
3.   The request was repeated several times but they fell on deaf ears. negligence since it had failed to construct a firewall between its property
4.   In the morning of Sept. 6, 1974, fire broke in the shop which then spread to and private respondents' residence which sufficiently complies with the
the house of the Mable family. Both the shop and the house were razed to pertinent city ordinances. The failure to comply with an ordinance
the ground. The cause of the conflagration was never discovered. providing for safety regulations had been ruled by the Court as an act of
5.   The National Bureau of Investigation found specimens from the burned negligence
structures negative for the presence of inflammable substances. 6.   [INSURANCE TOPIC] While this Court finds that petitioner is liable for
6.   The Mable family collected P35,000.00 on the insurance on their house and
damages to private respondents as found by the Court of Appeals, the fact
that private respondents have been indemnified by their insurer in the
amount of P35,000.00 for the damage caused to their house and its
contents has not escaped the attention of the Court.
7.   Hence, the Court holds that in accordance with Article 22072 of the
Civil Code the amount of P35,000.00 should be deducted from the
amount awarded as damages.
8.   Having been indemnified by their insurer, The Mable family are only
entitled to recover the deficiency from FF. Cruz.
9.   The insurer, if it is so minded, may seek reimbursement of the amount
it indemnified the family from FF. Cruz. This is the essence of its right
to be subrogated to the rights of the insured, as expressly provided in
Article 2207. Upon payment of the loss incurred by the insured, the
insurer is entitled to be subrogated pro tanto to any right of action
which the insured may have against the third person whose negligence
or wrongful act caused the loss
10.   Under Article 2207, the real party in interest with regard to the
indemnity received by the insured is the insurer
11.   Whether or not the insurer should exercise the rights of the insured to
which it had been subrogated lies solely within the former's sound
discretion.
12.   Since the insurer is not a party to the case, its identity is not of record and
no claim is made on its behalf, the private respondent's insurer has to claim
his right to reimbursement of the P35,000.00 paid to the insured.

                                                                                                                       
2
  Art. 2207. If the plaintiffs property has been insured, and he has received indemnity from the insurance
company for the injury or loss arising out of the wrong or breach of contract complained of, the insurance
company is subrogated to the rights of the insured against the wrongdoer or the person who violated the
contract. If the amount paid by the insurance company does not fully cover the injury or loss, the
aggrieved party shall be entitled to recover the deficiency from the person causing the loss or injury.    
 
005 RIZAL SURETY & INSURANCE CO v. MANILA RAILROAD violated the contract.”
COMPANY AND MANILA PORT SERVICES (MATSUMURA) In this case, Rizal Surety cannot recover from Manila Railroad and Manila Port Services
April 25, 1968 | Fernando, J. | Subrogation an amount greater than that to which the consignee could lawfully lay claim.

PETITIONER: Rizal Surety & Insurance Company


FACTS
RESPONDENTS:Manila Railroad Company and Manila Port Service
1.   On or before November 29, 1960 the vessel, SS Flying Trader, loaded on
SUMMARY: On November 29, 1960, the SS Flying Trader loaded for shipment several Board at Genoa, Italy for shipment to Manila, Philippines.
cargo from Genoa Italy to Manila. Among the cargo was 6 cases of OMH containing 2.   Among the cargo was 6 cases of OMH, Special Single Colour Offset Press
Special Single Colour Offset Press Machines. When the vessel arrived in Manila, the Machine where Bill of Lading No. 1 was issued. Such machine was
cargo was discharged completely and in good order to the custody of Manila Port services consigned to Suter, Inc.
(Arrastre operator). While the cargo was being lifted and loaded into Manila Port 3.   When the vessel arrived at the Port of Manila on January 16, 1961, the
Services’ truck via crane, one of the cases was dropped by the crane resulting to the cargo was discharged complete and in good order into the custody of
damage of one of the Special Single Colour Offset Press Machines. Rizal Surety (insurer)
Manila Port Service as arrastre operator.
paid P16,500 to Suter, Inc (consignee) representing the cost of the replacement and repair
of the machine, plus P180.70 to the International Adjustment Bureau as adjuster’s fee. 4.   In the course of the handling, one of the six cases identified as Case No.
2143 containing the OMH, Special Single Colour Offset Press was dropped
Clause 15 of the management contract between Suter, and Manila Railroad Co and by the crane as it was being lifted and loaded of the Manila Port Service
Manila Port Services appeared in the delivery permit of the cargo. The permit stated that into the consignee's truck. Because of this, the machine was heavily
“presented subject to all the terms and conditions of the Management Contract between damaged.
the Bureau of Customs and Manila Port Service and amendments thereto or alterations 5.   Rizal Surety, as insurer, paid
thereof, particularly but not limited to paragraph 15 thereof limiting the Company
liability to P500.00 per package, unless the value of the goods is otherwise, specified, a.   P16,500.00 to consignee, Suter, Inc. — representing damages by
declared or manifested and the corresponding arrastre charges have been paid, . . .” way of costs of replacement parts and repairs to put the machine in
working condition,
Rizal Surety now filed a suit against Manila Railroad and Manila Port Services to recover b.   P180.70 to the International Adjustment Bureau — as adjuster's
the amount it paid to Suter arguing that as the insurance company it has been subrogated fee for the survey conducted on the damaged cargo
to the rights of the insured. The lower court ruled that Rizal Surety is entitled to recovery 6.   In total, Rizal Surety paid a total of P16,680.70 representing its liability
but only to the extent of P500 as stated in Clause 15. Hence, this appeal by Rizal Surety
under the insurance contract. The arrastre charges in this particular
arguing that it should be entitled to recover the full amount.
shipment was paid on the weight or measurement basis whichever is higher,
The SC ruled that Rizal Surety is only entitled to P500. The literal language of Article and not on the value thereof.
2207 does not warrant such an interpretation. It is clear that in the event that the property 7.   Clause 15 of the management contract appeared "at the dorsal part of the
has been insured and the Insurance Company has paid the indemnity for the injury or loss Delivery Permit" and was "used in taking delivery of the subject shipment
sustained, it "shall be subrogated to the rights of the insured against the wrong-doer or the from the Manila Port Service and Manila Railroad Co.’s custody and
person who has violated the contract.” In this case, the management contract is clear that control, issued in the name of consignee's broker," contained what was
the amount is limited to P500 per package. While yes the management contract stated
referred to as "an important notice."
that a greater value could be recovered when it stated “unless the value of the goods is
otherwise specified, declared, or manifested”, Suter (consignee) failed to do so. Hence, 8.   Such permit is “presented subject to all the terms and conditions of the
since the insurer is subrogated merely to the rights of the consignee its recovery Management Contract between the Bureau of Customs and Manila Port
necessarily should be limited to what was recoverable by the insured -in this case P500 Service and amendments thereto or alterations thereof, particularly but not
because no other value was declared. limited to paragraph 15 thereof limiting the Company liability to P500.00
per package, unless the value of the goods is otherwise, specified, declared
DOCTRINE: Under Article 2207, in the event that the property has been insured and the or manifested and the corresponding arrastre charges have been paid, . . .”
Insurance Company has paid the indemnity for the injury or loss sustained, it "shall be
subrogated to the rights of the insured against the wrong-doer or the person who has
9.   Rizal Surety filed a suit against Manila Railroad and Manila Port Services said payment — is not subject to the provisions of said stipulation, and that
to recover the amount it paid to consignee, Suter, Inc. based on the Civil the same is furthermore invalid. The lower court correctly rejected this
Code wherein it is tested that the Insurance Company "shall be subrogated pretense because, having taken delivery of the shipment aforementioned by
to the rights of the insured”. Rizal Surety argues that it is entitled to the virtue of a delivery permit, incorporating thereto, by reference, the
amount paid by it in full, by virtue of the insurance contract. provisions of said management contract, particularly paragraph 15 thereof,
10.   The lower court ruled that the Manila Railroad and Manila Port Services the gist of which was set forth in the permit, the consignee became bound
should jointly and severally pay Rizal Surety P500 with legal interest. by said provisions, and because it could have avoided the application of said
11.   Rizal Surety appealed the decision arguing that under the civil code, it could maximum limit of P500.00 per package by stating the true value thereof in
recover in full. Hence, this petition. (Note, the case didn’t mention what its claim for delivery of the goods in question, which admittedly, the
else happened in the lower court. It went straight to the SC decision) consignee failed to do.
ISSUE/S: 6.   Rizal Surety and Insurance Company having been subrogated merely to the
1.   W/N Rizal Surety is entitled to recover the full amount - NO because Rizal rights of the consignee its recovery necessarily should be limited to what
Surety having been subrogated merely to the rights of the consignee (Suter, was recoverable by the insured.
Inc) which is only P500 per package. Rizal Surety could not recover more 7.   The lower court therefore did not err when in the decision appealed from, it
because Suter as the consignee did not declare a higher amount which, limited the amount which defendants were jointly and severally to pay
under Management Contract, should have been done by Suter if it wishes to plaintiff-appellant to "Five Hundred Pesos (P500.00) with legal interest
be entitled to a greater amount. thereon from January 31, 1962, the date of the filing of the complaint, . . . "

RULING: WHEREFORE, the decision appealed from is affirmed. With costs


against Rizal Surety and Insurance Company.

RATIO:
1.   The literal language of Article 2207 does not warrant such an interpretation.
It is clear that in the event that the property has been insured and the
Insurance Company has paid the indemnity for the injury or loss sustained,
it "shall be subrogated to the rights of the insured against the wrong-doer or
the person who has violated the contract.”
2.   In this case, Rizal Surety cannot recover from Manila Railroad and Manila
Port Services an amount greater than that to which the consignee could
lawfully lay claim.
3.   The management contract is clear. The amount is limited to Five Hundred
Pesos (P500.00).
4.   Such a stipulation has invariably received the approval of this Court from
the leading case of Bernabe & Co. v. Delgado Bros., Inc. Such a decision
was quoted with approval in the following subsequent cases: Atlantic
Mutual Insurance Co. v. Manila Port Service, Insurance Service Co. of
North America v. Manila Port Service, Insurance Company of North
America v. U.S. Lines, Co., and Insurance Company of North America v.
Manila Port Service.
5.   Rizal Surety maintains that, not being a party to the management contract,
the consignee — into whose shoes plaintiff had stepped in consequence of
PIONEER INSURANCE V CA (ARMAND) cover the loss, then the aggrieved party is the one entitled to recover the
July 28, 1989| Gutierrez, J. | Subrogation deficiency. Evidently, under this legal provision, the real party in interest with regard
PETITIONER: Pioneer Insurance & Surety Corporation to the portion of the indemnity paid is the insurer and not the insured
RESPONDENTS: Hon. Court of Appeals, Border Machinery & Heavy Equipment FACTS:
Inc, Constancio M. Maglana and Jacob S. Lim 7.   This case involves consolidated petitions, one of which came from the
SUMMARY: Lim is an owner-operator of Southern Airlines (SAL). Japan Domestic decision of the CA which modified the decision of the then CFI of Manila.
Airlines (JDA) and Lim entered into a sales contract. Pioneer Insurance and Surety 8.   In 1965, Jacob S. Lim (petitioner in G.R. No. 84157) was engaged in the
Corp. as surety executed its surety bond in favor of JDA on behalf of its principal airline business as owner-operator of Southern Air Lines (SAL) a single
Lim. Border Machinery and Heacy Equipment Co, Inc., Francisco and Modesto proprietorship.
Cervantes, and Constancio Maglana contributed funds based on the misrepresentation 9.   On May 17, 1965, at Tokyo, Japan, Japan Domestic Airlines (JDA) and
of Lim that they will form a new corporation to expand his business. They executed Lim entered into and executed a sales contract for the sale and purchase of
two separate indemnity agreements in favor of Pioneer, one signed by Maglana and two (2) aircrafts and one (1) set of necessary spare parts for the total agreed
the other jointly signed by Lim for SAL, Bormaheco and the Cervanteses. The price of US $109,000.00 to be paid in installments.
indemnity agreements stipulated that the indemnitors principally agree and bind 10.   On May 22, 1965, Pioneer Insurance and Surety Corporation (Pioneer) as
themselves jointly and severally to indemnify and hold and save Pioneer from and surety executed and issued its Surety Bond No. 6639 in favor of JDA, in
against any/all damages, losses, etc. of whatever kind and nature may incur in behalf of its principal, Lim, for the balance price of the aircrafts and spare
consequence of having become surety. Lim executed in favor of Pioneer a deed of parts.
chattel mortgage as security. Upon default on the payments, Pioneer paid for him and 11.   It appears that Border Machinery and Heavy Equipment Company, Inc.
filed a petition for the foreclosure of chattel mortgage as security. Maglana, (Bormaheco), Francisco and Modesto Cervantes (Cervanteses) and
Bormaheco and the Cervantes’s filed cross-claims against Lim alleging that they Constancio Maglana contributed some funds used in the purchase of the
were not privies to the contracts signed by Lim and for recovery of the sum of money above aircrafts and spare parts. The funds were supposed to be their
they advanced to Lim for the purchase of the aircrafts. The decision was rendered contributions to a new corporation proposed by Lim to expand his airline
holding Lim liable to pay. The issues are (1) Whether or not the CA erred when it business.
dismissed the appeal of Pioneer on the sole ground that it had already collected 12.   They executed two (2) separate indemnity in favor of Pioneer, one signed
the proceeds of the reinsurance on its bond in favor of the JDA and that it by Maglana and the other jointly signed by Lim for SAL, Bormaheco and
cannot represent a reinsurer to recover the amount from herein respondents – the Cervanteses. The indemnity agreements stipulated that the indemnitors
NO. Pioneer was already paid by the reinsurers the amount of P295,000, and had principally agree and bind themselves jointly and severally to indemnify
already foreclosed the mortgaged chattels for the amount of 37,050, thus Pioneer is and hold and save harmless Pioneer from and against any/all damages,
already overpaid by P33,383, and has no more claim against the defendants. Pioneer, losses, costs, damages, taxes, penalties, charges and expenses of whatever
having foreclosed the chattel mortgage on the planes and spare parts, no longer has kind and nature which Pioneer may incur in consequence of having become
any further action against the defendants as indemnitors to recover any unpaid surety upon the bond/note and to pay, reimburse and make good to Pioneer,
balance. The indemnity agreement was ipso jure extinguished upon the foreclosure of its successors and assigns, all sums and amounts of money which it or its
the chattel mortgage. These defendants, as indemnitors, would be entitled to be representatives should or may pay or cause to be paid or become liable to
subrogated to the right of Pioneer should they make payments to the latter. pay on them of whatever kind and nature.
(2) Whether or not Maglana, Bormaheco and the Cervanteses must share in the 13.   On June 10, 1965, Lim doing business under the name and style of SAL
loss of the venture in proportion to their contribution - NO. There was no de facto executed in favor of Pioneer as deed of chattel mortgage as security for
partnership. Ordinarily, when co-investors agreed to do business through a the latter's suretyship in favor of the former. It was stipulated therein that
corporation but failed to incorporate, a de facto partnership would have been formed, Lim transfer and convey to the surety the two aircrafts. The deed was duly
and as such, all must share in the losses and/or gains of the venture in proportion to registered with the Office of the Register of Deeds of the City of Manila
their contribution. But in this case, it was shown that Lim did not have the intent to and with the Civil Aeronautics Administration pursuant to the Chattel
form a corporation with Maglana et al. Mortgage Law and the Civil Aeronautics Law (Republic Act No. 776).
DOCTRINE: In general a reinsurer, on payment of a loss, acquires the same rights 14.   Lim defaulted on his subsequent installment payments prompting JDA to
by subrogation as are acquired in similar cases where the original insurer pays a loss. request payments from the surety. Pioneer paid a total sum of P298,626.12.
If a property is insured and the owner receives the indemnity from the insurer, it is 15.   Pioneer then filed a petition for the extrajudicial foreclosure of the said
provided in said article that the insurer is deemed subrogated to the rights of the chattel mortgage before the Sheriff of Davao City. The Cervanteses and
insured against the wrongdoer and if the amount paid by the insurer does not fully
Maglana, however, filed a third party claim alleging that they are co-owners represent a reinsurer to recover the amount from herein respondents –
of the aircraft. NO. Pioneer was already paid by the reinsurers the amount of P295,000,
16.   On July 19, 1966, Pioneer filed an action for judicial foreclosure with an and had already foreclosed the mortgaged chattels for the amount of 37,050,
application for a writ of preliminary attachment against Lim and thus Pioneer is already overpaid by P33,383, and has no more claim against
respondents, the Cervanteses, Bormaheco and Maglana. the defendants.
17.   In their Answers, Maglana, Bormaheco and the Cervanteses filed cross- 3.   Whether or not Maglana, Bormaheco and the Cervanteses must share
claims against Lim alleging that they were not privies to the contracts in the loss of the venture in proportion to their contribution - NO. There
signed by Lim and, by way of counterclaim, sought for damages for being was no de facto partnership. Ordinarily, when co-investors agreed to do
exposed to litigation and for recovery of the sums of money they advanced business through a corporation but failed to incorporate, a de facto
to Lim for the purchase of the aircrafts in question. partnership would have been formed, and as such, all must share in the
18.   After trial, a decision was rendered holding Lim liable to pay Pioneer but losses and/or gains of the venture in proportion to their contribution. But in
dismissed Pioneer's complaint against all other defendants. this case, it was shown that Lim did not have the intent to form a
19.   As stated earlier, the appellate court modified the trial court's decision in corporation with Maglana et al.
that the plaintiffs complaint against all the defendants was dismissed. In all
other respects the trial court's decision was affirmed. RULING: WHEREFORE, the instant petitions are DISMISSED. The questioned
20.   The appellate court ruled that Pioneer's contention that it is representing the decision of the Court of Appeals is AFFIRMED.
reinsurer to recover the amount from defendants, hence, it instituted the
action is utterly devoid of merit. Plaintiff did not even present any evidence RATIO:
that it is the attorney-in-fact of the reinsurance company, authorized to 6.   As for the first case, the payment to the petitioner made by the reinsurers
institute an action for and in behalf of the latter. Moreover, the appellate was not disputed in the appellate court. Considering this admitted payment,
court ruled that Pioneer cannot be considered as the real party in interest as the only issue that cropped up was the effect of payment made by the
it has already been paid by the reinsurer the sum of P295,000.00 — the bulk reinsurers to the petitioner. Therefore, the petitioner's argument that the
of defendants' alleged obligation to Pioneer. It also ruled that in addition to respondents had no interest in the reinsurance contract as this is strictly
the said proceeds of the reinsurance received by plaintiff Pioneer from its between the petitioner as insured and the reinsuring company pursuant to
reinsurer, the former was able to foreclose extra-judicially one of the subject Section 91 (should be Section 98) of the Insurance Code has no basis.
airplanes and its spare engine, realizing the total amount of P37,050.00 7.   Hence the applicable law is Article 2207 of the new Civil Code, to wit:
from the sale of the mortgaged chattels. Adding the sum of P37,050.00, to Art. 2207. If the plaintiffs property has been insured, and he has received indemnity from the
insurance company for the injury or loss arising out of the wrong or breach of contract
the proceeds of the reinsurance amounting to P295,000.00, it is patent that
complained of, the insurance company shall be subrogated to the rights of the insured against
plaintiff has been overpaid in the amount of P33,383.72 considering that the the wrongdoer or the person who has violated the contract. If the amount paid by the insurance
total amount it had paid to JDA totals to only P298,666.28. To allow company does not fully cover the injury or loss, the aggrieved party shall be entitled to recover
plaintiff Pioneer to recover from defendants the amount in excess of the deficiency from the person causing the loss or injury.
P298,666.28 would be tantamount to unjust enrichment 8.   Interpreting the aforesaid provision, we ruled in the case of Phil. Air Lines,
21.   Pioneer contends that (1) it is at a loss where respondent court based its Inc. v. Heald Lumber Co. which we subsequently applied in Manila
finding that Pioneer was paid by its reinsurer, (2) even assuming that it was Mahogany Manufacturing Corporation v. Court of Appeals:
Note that if a property is insured and the owner receives the indemnity from the insurer, it is
paid by its reinsurer, none of the respondents had any interest in the matter
provided in said article that the insurer is deemed subrogated to the rights of the insured against
since reinsurance is between the petitioner and the reinsurer pursuant to the wrongdoer and if the amount paid by the insurer does not fully cover the loss, then the
Sec. 91 of the Insurance Code, (3) pursuant to the indemnity agreements, aggrieved party is the one entitled to recover the deficiency. Evidently, under this legal
the petitioner is entitled to recover from respondents Bormaheco and provision, the real party in interest with regard to the portion of the indemnity paid is the
insurer and not the insured. (Emphasis supplied)
Maglaa and (4) the principle of unjust enrichment is not applicable since
whatever amount he would recover from the co-indemnitor will be paid to 9.   It is clear from the records that Pioneer sued in its own name and not as an
the insurer. attorney-in-fact of the reinsurer. Accordingly, the appellate court did not
commit a reversible error in dismissing the petitioner's complaint as
ISSUE/s: against the respondents for the reason that the petitioner was not the
2.   Whether or not the CA erred when it dismissed the appeal of Pioneer real party in interest in the complaint and, therefore, has no cause of
on the sole ground that it had already collected the proceeds of the action against the respondents.
reinsurance on its bond in favor of the JDA and that it cannot 10.   Nevertheless, the Pioneer argues that the appeal as regards the counter
indemnitors should not have been dismissed on the premise that the
evidence on record shows that it is entitled to recover from the counter effect partners inter se, and their rights as members of the company to the
indemnitors. It does not, however, cite any grounds except its allegation that property acquired by the company will be recognized.
respondent "Maglanas defense and evidence are certainly incredible" 14.   A partnership relation between certain stockholders and other stockholders,
11.   On the other hand, we find the trial court's findings on the matter replete who were also directors, will not be implied in the absence of an agreement,
with evidence to substantiate its finding that the counter-indemnitors are so as to make the former liable to contribute for payment of debts illegally
not liable to the petitioner, to wit: contracted by the latter
Pioneer Insurance, knowing the value of the aircrafts and the spare parts involved, agreed to 15.   In his answer, the petitioner denied having received any amount from
issue the bond provided that the same would be mortgaged to it, but this was not possible respondents Bormaheco, the Cervanteses and Maglana but the trial and
because the planes were still in Japan and could not be mortgaged here in the Philippines. As
soon as the aircrafts were brought to the Philippines, they would be mortgaged to Pioneer appellat court found that petitioner received various amounts from
Insurance to cover the bond, and this indemnity agreement would be cancelled. Pioneer, Bormaheco and Magnlana in the ownership of subject airplanes.
having foreclosed the chattel mortgage on the planes and spare parts, no longer has any 16.   It is therefore clear that the petitioner never had the intention to form a
further action against the defendants as indemnitors to recover any unpaid balance of the
corporation with the respondents despite his representations to them. This
price. The indemnity agreement was ipso jure extinguished upon the foreclosure of the chattel
mortgage. These defendants, as indemnitors, would be entitled to be subrogated to the gives credence to the cross-claims of the respondents to the effect that they
right of Pioneer should they make payments to the latter. were induced and lured by the petitioner to make contributions to a
Pioneer's election of the remedy of foreclosure precludes any further action to recover any proposed corporation which was never formed because the petitioner
unpaid balance of the price. SAL or Lim, having failed to pay the second to the eight and last
reneged on their agreement.
installments to JDA and Pioneer as surety having made of the payments to JDA, the alternative
remedies open to Pioneer were as provided in Article 1484 of the New Civil Code, known as 17.   Applying therefore the principles of law earlier cited to the facts of the case,
the Recto Law. necessarily, no de facto partnership was created among the parties which
Pioneer exercised the remedy of foreclosure of the chattel mortgage both by extrajudicial would entitle the petitioner to a reimbursement of the supposed losses of the
foreclosure and the instant suit. Such being the case, as provided by the aforementioned proposed corporation. The record shows that the petitioner was acting on
provisions, Pioneer shall have no further action against the purchaser to recover any unpaid
balance and any agreement to the contrary is void. The restructuring of the obligations of SAL his own and not in behalf of his other would-be incorporators in transacting
or Lim, thru the change of their maturity dates discharged these defendants from any liability the sale of the airplanes and spare parts.
as alleged indemnitors. The change of the maturity dates of the obligations of Lim, or SAL
extinguish the original obligations thru novations thus discharging the indemnitors. These
restructuring of the obligations with regard to their maturity dates, effected twice, were done
without the knowledge, much less, would have it believed that these defendants Maglana (sic).
Pioneer's official Numeriano Carbonel would have it believed that these defendants and
defendant Maglana knew of and consented to the modification of the obligations. But if that
were so, there would have been the corresponding documents in the form of a written notice to
as well as written conformity of these defendants, and there are no such document. The
consequence of this was the extinguishment of the obligations and of the surety bond secured
by the indemnity agreement which was thereby also extinguished.
Pioneer's liability as surety to JDA had already prescribed when Pioneer paid the same.
Consequently, Pioneer has no more cause of action to recover from these defendants, as
supposed indemnitors, what it has paid to JDA. By virtue of an express stipulation in the
surety bond, the failure of JDA to present its claim to Pioneer within ten days from default of
Lim or SAL on every installment, released Pioneer from liability from the claim. Therefore,
Pioneer is not entitled to exact reimbursement from these defendants thru the indemnity
12.   As for the second case wherein the petioner is Jacob S. Lim, he theorizes
that as a result of the failure of respondents Bormaheco, Spouses Cervantes,
Constancio Maglana and petitioner Lim to incorporate, a de
facto partnership among them was created, and that as a consequence of
such relationship all must share in the losses and/or gains of the venture in
proportion to their contribution. Petitioner then questions the appellate
court’s findings ordering him to reimburse certain amounts given by
respondents as their contributions to the intended corporation.
13.   Where persons associate themselves together under articles to purchase
property to carry on a business, and their organization is so defective as to
come short of creating a corporation within the statute, they become in legal
007 ORIENTAL ASSURANCE v. ONG (MERILLES)
October 11, 2017 | Leonen, J. | Subrogation FACTS:
1.   JEA Steel Industries imported from South Korea 72 aluminum-zinc-alloy-
coated steel sheets in coils. These steel sheets were transported to Manila on
PETITIONER: Oriental Assurance Corporation
board the vessel M/V Dooyang Glory as evidenced by a Bill of Lading.
RESPONDENTS: Manuel Ong, doing business under the name of Western
2.   Upon arrival of the vessel at the Manila South Harbor on June 10, 2002, the
Pacific Transport Services and/or Asian Terminals, Inc.
72 coils were discharged and stored in Pier 9 under the custody of the
NOTE THE RELATIONSHIPS: Oriental = insurer; JEA = insured/consignee; Asian
arrastre contractor, Asian Terminals, Inc (respondent company in this case).
Terminals = consignor 3.   From the storage compound, the coils were loaded on the trucks on Manuel
Ong (respondent) and delivered to JEA Steel’s plant in Trece Martirez,
SUMMARY: JEA Steels imported from South Korea 72 coils. When it arrived Cavite.
at Manila, it was stored under the custody of Asian Terminals. From storage it 4.   Eleven (11) of the coils ''were found to be in damaged condition, dented or
was loaded on trucks of Manuel Ong to be delivered at the plant of JEA Steel. their normal round shape deformed.”
However, upon arrival at the plant, 11 coils were found to be in damaged 5.   JEA Steel filed a claim with Oriental for the value of the 11 damaged coils,
condition. pursuant to Marine Insurance Policy No. OAC/M-12292.
6.   Oriental paid JEA Steel the sum of ₱521,530.16 and subsequently
JEA claimed insurance from Oriental Assurance. Oriental paid JEA the sum of demanded indemnity from Ong and Asian Terminals (respondents), but
₱521,530.16. Oriental demanded indemnity form Ong and Asian Terminals, but they refused to pay.
they refused. Hence, a complaint was filed for collection of sum of money. Ong 7.   Oriental filed a complaint before the RTC of Manila for sum of money
alleged that the coils were already damaged when he received it. Asian argues against Ong and Asian Terminals.
hat it had exercised due diligence in caring for the consigned goods. The RTC 8.   Ong countered that the 1l coils were already damaged when they were
dismissed the case, which the CA affirmed. The CA stated that the action of loaded on board his trucks and transported to the consignee.
Oriental has already prescribed. 9.   Asian Terminals claimed that it exercised due diligence in handling the
cargo, that the cargo was released to the consignee's representative in the
The issue relevant to our topic is wether the claim against Asian Terminals, Inc. same condition as when received from the vessel, and that the damages
is barred by prescription. The court ruled that NO, the action is not barred. were sustained while in the custody of the vessel or the customs broker.
Under the Management Contract between JEA and Asian Terminal, claims must a.   Asian Terminals further argued that Oriental's claim was barred for
be filed within 15 days from the issuance of the Gate Pass. However, also under the latter's failure to file a notice of claim within the 15-day period
the Management Contract, the consignee (JEA) had thirty (30) days from provided in the Gate Pass and in Article VII, Section 7.01 of the
receipt of the cargo to request for a certificate of loss from Asian Terminals. Contract for Cargo Handling Services (Management Contract)
Oriental claimed damages 17 days after last receipt of the goods or within the 30 between the Philippine Ports Authority and Asian Terminals
day period. Although not a party to the contract, Oriental was subrogated to the b.   Asian Terminals added that its liability, if any, should not exceed
rights of JEA. ₱5,000.00, pursuant to said Section 7.01.
10.   RTC dismissed the complaint as it found no preponderance of evidence to
If the insured property is destroyed or damaged through the fault or establish that Ong and Asian Terminals were the ones responsible for the
negligence of a party other than the assured, then the insurer, upon damages. The MR was likewise denied.
payment to the assured, will be subrogated to the rights of the assured to 11.   The CA dismissed Oriental’s appeal on the ground that its claim had
recover from the wrongdoer to the extent that the insurer has been already prescribed.
obligated to pay. As subrogee, Oriental merely stepped into the shoes of the a.   CA also found that the coils were already damaged when it was
consignee and may only exercise those rights that the consignee may have loaded in Ong’s trucks
against the wrongdoer who caused the damage. b.   Hence, the legal presumption of negligence applies against Asian
Terminals unless it is able to prove that it exercised extraordinary
DOCTRINE: As subrogee, Oriental merely stepped into the shoes of the diligence in the handling of the cargo.
consignee and may only exercise those rights that the consignee may have c.   The CA held that as an arrastre operator, Asian Terminals was
against the wrongdoer who caused the damage. bound to observe the same degree of care required of common
carriers.
d.   The CA ultimately ruled that while Asian Terminals failed to rebut of Appeals in passing upon this issue.
the presumption of negligence against it, it cannot be held liable to 2.   [IMPORTANT] ISSUE #2 ON WHETHER THE CLAIM IS BARRED
pay the value of the damaged coils because Oriental's claim was BY PRESCRIPTION:
filed beyond the 15-day prescriptive period stated in the Gate Pass a.   Oriental contends that it was not aware of the provisions of the
e.   An MR was also denied. Gate Pass or the Management Contract, neither of which it was a
12.   Hence, this petition before the SC. party to. Consequently, it cannot be bound by the stipulation
limiting the liability of Asian Terminals.
ISSUE/s: b.   Asian Terminals counters that "the provisions of the Management
1.   Whether or not the Court of Appeals gravely erred in passing upon the issue Contract and the Gate Pass are binding on Oriental as insurer-
of prescription even though it was not an assigned error in the appeal – NO, subrogee and successor-in-interest of the consignee.”
the issue of prescription is closely related to, and determinant of, the c.   THE COURT AGREES WITH ASIAN TERMINALS. The fact
propriety of the lower court's ruling, absolving respondents from liability that Oriental is not a party to the Gate Pass and the Management
for the damaged goods and dismissing Oriental's complaint. Contract does not mean that it cannot be bound by their provisions.
2.   Whether or not the claim against Asian Terminals, Inc. is barred by Oriental is subrogated to the rights of the consignee simply upon
prescription – NO, the claim was received within the 30 day period to its payment of the insurance claim.
request for a certificate of loss, damage, or injury from the arrastre operator. d.   Citing GSIS v Manila Railroad: The SC has repeatedly held that,
3.   Whether or not the Court of Appeals gravely erred in ruling that Manuel by availing himself of the services of the arrastre operator and
Ong is not liable for the damage of the cargo – NO. Manuel Ong is not taking delivery therefrom in pursuance of a permit and a pass
liable, Asian Terminal knows of the defect prior to it being loaded in his issued by the latter, which were "subject to all the terms and
truck. conditions" of said management contract, including, inter alia, the
requirement thereof that "a claim is filed with the Company within
RULING: WHEREFORE, the Petition for Review is GRANTED. The February 19, 15 days from the date of arrival of the goods", the consignee - and,
2009 Decision and August 25, 2009 Resolution of the Court of Appeals in CA-G.R. hence, the insurer, or plaintiff herein, as successor to the rights of
CV No. 89311 are SET ASIDE. Respondent Asian Terminals, Inc. is ORDERED to the consignee - became bound by the provisions of said contract.
pay petitioner Oriental Assurance Corporation the amount of ₱55,000.00, with e.   Citing Summa Insurance v CA: However, a management contract,
interest at the legal rate of six percent (6%) per annum from the date of finality of which is a sort of a stipulation pour autrui within the meaning of
this judgment until fully paid. Article 1311 of the Civil Code, is also binding on a consignee
because it is incorporated in the gate pass and delivery receipt
RATIO: which must be presented by the consignee before delivery can be
1.   ISSUE #1 ON RULING ON PRESCRIPTION (can skip): effected to it. The insurer, as successor-in-interest of the consignee,
a.   Oriental submits that the "Court of Appeals cannot rule on the is likewise bound by the management contract.
issue of prescription as this was not included in the assignment of f.   The fact that Oriental is not a party to the Gate Pass and the
errors nor was this properly argued by any of the parties in their Management Contract does not mean that it cannot be bound by
respective briefs filed before the Court of Appeals. their provisions. Oriental is subrogated to the rights of the
b.   On the other hand, Asian Terminals counters that the Court of consignee simply upon its payment of the insurance claim.
Appeals properly reviewed the issue of prescription even though it g.   Art. 2207 of the Civil Code is founded on the well-settled
was not raised in Oriental's appeal brief. This issue is closely principle of subrogation. If the insured property is destroyed
related to the liability of Asian Terminals for the damaged or damaged through the fault or negligence of a party other
shipment, the first error in Oriental's appeal. Moreover, Asian than the assured, then the insurer, upon payment to the
Terminals asserts that it raised the issue of prescription before the assured, will be subrogated to the rights of the assured to
trial court, although it was not resolved. recover from the wrongdoer to the extent that the insurer has
c.   THE COURT AGREES WITH ASIAN TERMINALS. The issue been obligated to pay.
of prescription is closely related to, and determinant of, the i.   Payment by the insurer to the assured operates as an
propriety of the lower court's ruling, absolving respondents from equitable assignment to the former of all remedies which
liability for the damaged goods and dismissing Oriental's the latter may have against the third party whose
complaint. Thus, the Court finds no error on the part of the Court negligence or wrongful act caused the loss,
ii.   The right of subrogation is not dependent upon, nor does fact, Ong and Asian Terminals were sued in the alternative because
it grow out of, any privity of contract or upon written petitioner was uncertain against whom it was entitled for relief.
assignment of claim. d.   Furthermore, there was no proof of Ong's bad faith.
iii.   It accrues simply upon payment of the insurance claim e.   Mere allegation cannot take the place of evidence.
by the insurer f.   Besides, Ong's assertion that the loading of the cargo on the trucks
h.   As subrogee, Oriental merely stepped into the shoes of the was undertaken by Asian Terminals and the unloading of the same
consignee and may only exercise those rights that the consignee cargo was undertaken by the consignee at its warehouse remains
may have against the wrongdoer who caused the damage. unrequited
i.   Under the express terms of the Management Contract, the g.   Asian Terminals caused the inspection of the shipment before they
consignee had thirty (30) days from receipt of the cargo to request were loaded on Ong's trucks. At the consignee's warehouse, the
for a certificate of loss from the arrastre operator. inspection was done in the presence of the consignee's authorized
j.   Upon receipt of such request, the arrastre operator would have representative.
15 days to issue a certificate of loss, either actually or h.   Thus, Ong is not obliged to inform the consignee or Asian
constructively. Terminals about the damaged coils as they would have presumably
k.   From the date of issuance of the certificate of loss or where no known about them.
certificate was issued, from the expiration of the 15-day period, 5.   ASIAN TERMINALS IS FOUND LIABLE AND ORDERED TO PAY
the consignee has 15 days within which to file a formal claim ORIENTAL P55,000 with interests.
with the arrastre operator.
l.   In other words, the consignee had 45 to 60 days from the date of
last delivery of the goods within which to submit a formal claim to
the arrastre operator.
m.   The consignee's claim letter was received by Asian Terminals
on July 4, 2002, or 17 days from the last delivery of the goods,
still within the prescribed 30-day period to request a certificate
of loss, damage, or injury from the arrastre operator.
3.   LIABILITY OF ASIAN TERMINALS:
a.   Evidence showed that Upon Asian Terminal’s request, Ultraphil
conducted surveys which sowed that 11 coils were damaged before
it was loaded don Ong’s truck.
b.   Asian Terminals request for the cargo survey shows that it had
knowledge of the damage of the shipment while in its possession
and that the survey was sought specifically to ascertain the nature
and extent of the damage.
c.   Thus, Asian Cargos cannot escape liability for the damaged coils,
simply by its own act of not sending a representative, after it had
contracted for the survey of the shipment.
4.   ISSUE #3 ON LIABILITY OF MANUEL ONG:
a.   Both the Court of Appeals and the Regional Trial Court found that
the 11 coils were already damaged before the coils were loaded on
Ong's truck. Hence, Ong could not be responsible for the damaged
shipment.
b.   However, petitioner asserts that Ong should be held solidarily
liable with Asian Terminals for acting in bad faith when it did not
apprise the consignee or Asian Terminals about the damaged coils.
THE COURT FINDS THIS CONTENTION UNTENABLE.
c.   This issue was never raised by petitioner in the lower courts. In
001 UNITED DOCTORS MEDICAL CENTER v. BERNADAS 13.   On July 17, 1986, Cesario Bernadas (Cesario) was an orderly3 in United
(PELIÑO) Doctors Medical Center’s (UDMC) housekeeping department; he was
December 13, 2017 | Leonen, J. | Insurance and Retirement Benefits eventually promoted as a utility man.
14.   UDMC and the rank-and-file employees had a collective bargaining agreement
(CBA) where the rank-and-file employees were entitled to optional retirement
PETITIONER: United Doctors Medical Center benefits (ORB).
RESPONDENT: Cesario Bernadas, represented by Leonila Bernadas a.   The provision on the CBA reads: “Sec. 1. Retirement and Severance Pay.
The CENTER shall grant each employee retirement and severance pay in
SUMMARY: Cesario was an employee of UDMC. UDMC and the rank-and-file
accordance with law. It shall also continue its present policy on optional
employees had a CBA where these employees were entitled to ORB. Based on this
optional retirement policy, an employee who has rendered at least 20 years of service is retirement.”
entitled to optionally retire and the pay would be equal to a retiree’s salary for 11 days per b.   Based on this optional retirement policy, an employee who has rendered at
year of service. Cesario died of a freak accident. So Leonila, his wife, filed a Complaint least 20 years of service is entitled to optionally retire and the retirement
for payment of retirement benefits, damages, and atty’s fees with the NLRC. She was pay is equal to a retiree’s salary for 11 days per year of service.
already able to get 180k insurance proceeds based on the CBA. The LA dismissed her c.   Employees are also provided insurance where UDMC pays the
complaint, saying that since Cesario didn’t apply for ORB, then the beneficiaries are not premiums and the family members would be the beneficiaries.
entitled. She appealed to NLRC, and they reversed the LA. NLRC said that since the 15.   October 20, 2009, Cesario, 53 years at that time, died in a “freak accident”
optional retirement plan was never presented, it cast a doubt on whether an application is while working in a doctor’s residence.
required before an employee could be entitled, hence the doubt should be resolved in 16.   His wife, Leonila Bernadas (Leonila) filed a Complaint for payment of
favor of Cesario. On appeal to the CA, CA sustained the NLRC, saying that the retirement benefits, damages, and attorney’s fees with the NLRC. And Leonila
retirement benefits and insurance are 2 distinct benefits. Hence, this petition. The issue in and her son also claimed and were able to receive insurance proceeds of Php
this case is whether or not Cesario is entitled to receive his ORB despite receiving 180k under the CBA.
insurance benefits. The SC answered in the affirmative. The SC distinguished between 17.   Labor Arbiter: dismissed Leonila’s complaint.
insurance and retirement plans, wherein a contract of insurance – indemnity; agreement a.   Cesario should have applied for ORB during his lifetime, the benefits being
whereby one undertakes for a consideration to indemnify another against loss, optional. But since he didn’t, then his beneficiaries are not entitled to claim
damage or liability arising from an unknown or contingent event, while retirement his ORB.
plans are those which serve to secure loyalty and efficiency on the part of employees, 18.   Leonila appealed to the NLRC.
and to increase continuity of service and decrease the labor turnover, by giving to the a.   NLRC reversed the Labor Arbiter.
employees some assurance of security as they approach and reach the age at which b.   It found that the optional retirement plan was never presented so it cast a
earning ability and earnings are materially impaired or at an end. Hence, grant of doubt on whether or not the plan required an application for ORB before an
insurance proceeds will not necessarily bar the grant of retirement benefits; these are employee could become entitled to them. And considering the
2 separate and distinct benefits that an employer may provide to its employees. In “constitutional mandate to afford full protection to labor”, NLRC resolved
addition, Cesario is also entitled to ORB because CBA does not mandate that an the doubt in favor of Cesario.
application must first be filed before the benefits may vest, hence, this ambiguity c.   UDMC filed a MR but was denied.
should be resolved in favor of the retiree. 19.   UDMC subsequently filed a Petition for Certiorari with the CA.
a.   CA rendered a decision sustaining the ruling of the NLRC.
DOCTRINE: Contract of insurance – indemnity; agreement whereby one undertakes b.   CA held that the retirement plan and insurance are 2 separate and distinct
for a consideration to indemnify another against loss, damage or liability arising benefits that were granted to employees and that Leonila’s receipt of the
from an unknown or contingent event, while retirement plans are those which serve insurance proceeds did not bar her from being entitled to the retirement
to secure loyalty and efficiency on the part of employees, and to increase continuity benefits under the CBA.
of service and decrease the labor turnover, by giving to the employees some c.   UDMC filed a MR but was denied as well.
assurance of security as they approach and reach the age at which earning ability and 20.   Hence, this petition.
earnings are materially impaired or at an end. Hence, grant of insurance proceeds 21.   UDMC argues:
will not necessarily bar the grant of retirement benefits; these are 2 separate and a.   Cesario’s beneficiaries do not have legal capacity to apply for his ORB
distinct benefits that an employer may provide to its employees since Cesario never applied for it in his lifetime, and even assuming that he
was already qualified to apply for optional retirement 3 years prior to his
FACTS: death, he never did. Hence, no basis for Cesario’s beneficiaries to be
12.   This is a Petition for Review on Certiorari assailing the Decision and Resolution
of the CA, sustaining the NLRC’s finding.                                                                                                                        
3
Merriam-Webster defined this as a person who works in a hospital and does various jobs (such as
moving patients or cleaning)
entitled to his optional retirement benefits. b.   Voluntary4: by agreement between the employer and employee – This is
b.   To grant them these ORB on top of the life insurance benefits would usually embodied in a CBA.
amount to double compensation and unjust enrichment. c.   Voluntary: given by the employer – Announced company policy or
22.   Leonila argues: impliedly as in a failure to contest the employee’s claim for retirement
a.   Had her husband died under normal circumstances, he would have applied benefits.
for the ORB. 6.   These retirement plans are not meant to be a replacement to the compulsory
b.   The fact that he was unable to apply before his death is a procedural retirement scheme under social security laws.
technicality that should be set aside so that “full protection to labor” is
afforded and “the ends of social and compassionate justice” are met. On whether Cesario is entitled to receive his optional retirement benefits despite his
untimely death
ISSUE/s: 1.   Issue in this case concerns the second type of retirement plan.
2.   WON Leonila, as her husband’s representative, may claim his optional 2.   The terms and conditions of a CBA constitute the law between the parties, but
retirement benefits. – YES, she is a beneficiary of Cesario. the CBA involved in this case does not provide for the terms and conditions of
3.   WON Cesario can receive the ORB despite already receiving insurance the “present policy on optional retirement”.
benefits. – YES, insurance and retirement plans are two separate benefits and a.   Leonila merely alleged before the Labor Arbiter that UDMC grants an
receiving one is not a bar to receiving the other. employee a retirement or separation equivalent to 11 days per year of
4.   WON Cesario is entitled to receive his ORB despite his untimely death. – YES, service after serving for at least 20 years, which was not deisputed by
CBA does not mandate that an application must first be filed before the benefits UDMC. Therefore, doubt arises as to what UDMC’s optional retirement
may vest, hence, this ambiguity should be resolved in favor of the retiree. package actually entails.
3.   It is settled that doubts must be resolved in favor of labor.
RULING: WHEREFORE, the petition is DENIED. The Decision and Resolution of the a.   Retirement laws should be liberally construed and administered in favor of
CA are AFFIRMED. Petitioner UDMC is ordered to pay Cesario, through his beneficiary the persons intended to be benefited and all doubts as to the intent of the
Leonila, optional retirement benefits in the amount of Php 98,252.55 as provided by the law should be resolved in favor of the retiree to achieve its humanitarian
Labor Code. purpose.
4.   UDMC: insists that Cesario would not have been entitled to his ORB as he
RATIO: failed to exercise the option before his untimely death.
1.   Retirement – result of a bilateral act of the parties, a voluntary agreement a.   SC: Retirement encompasses even the concept of death and that the Court
between the employer and the employee whereby the latter, after reaching a has considered death as a form of disability retirement since there is no
certain age, agrees to sever his or her employment with the former. more permanent or total physicaly disability than death.
2.   Contract of Insurance – indemnity; agreement whereby one undertakes for i.   Compulsory retirement and death both involve events beyond the
a consideration to indemnify another against loss, damage or liability employee’s control.
arising from an unknown or contingent event. b.   UDMC also admits that Cesario was already qualified to receive retirement
3.   Retirement plans – Serve to secure loyalty and efficiency on the part of benefits since he was already working there for 23 years, and while the
employees, and to increase continuity of service and decrease the labor choice to retire was within Cesario’s control, his death foreclosed that
turnover, by giving to the employees some assurance of security as they possibility.
approach and reach the age at which earning ability and earnings are c.   UDMC’s optional retirement plan is based on length of service and not of
materially impaired or at an end. reaching a certain age. It rewards loyalty and continued service by granting
4.   Given the above, it is clear that there is a difference between insurance and an employee an earlier age to claim his or her retirement benefits even if
retirement benefits. Hence, grant of insurance proceeds will not necessarily the employee has not reached the twilight years. It would be iniquitous to
bar the grant of retirement benefits; these are 2 separate and distinct withhold it from he just because he died before he applied.
benefits that an employer may provide to its employees. d.   CBA does not mandate that an application must first be filed before
5.   Three types of retirement plans available to employees in the PH: the benefits may vest, hence, this ambiguity should be resolved in favor
a.   Compulsory and Contributory – RA 8282 for private (SSS) and RA 8291 of the retiree.
for government (GSIS). These laws provide mandatory contribution which e.   Retirement benefits are the property interests of the retiree and his or her
shall become a pension fund upon retirement. A mandatory contribution is beneficiaries. CBA doesn’t prohibit the beneficiaries from claiming
deducted from the employee’s monthly income; retirement packages are retirement benefits even if the retiree dies before the proceeds could be
usually crafted as ‘forced savings’. There is a vested right to the benefits released.
here, so the beneficiaries of retired employees are entitled to receive even
after the employee’s death.
                                                                                                                       
4
The last 2 retirement plans are governed by the Labor Code.  
damage through accident for an amount not exceeding P8,000 (Policy No.
02 CCC INSURANCE CORP. v. CA (PLEYTO) MC-1156)
January 30, 1970 | Reyes, J.B.L, J. | Insurance Contract Interpretation 24.   On June 25, 1961, and during the effectivity of the policy, the insured
vehicle, while being driven by the driver of Robes, became involved in a
vehicle collision along Rizal Avenue Extension, Potrero, Malabon, Rizal
PETITIONER: CCC Insurance Corporation
25.   The car was damaged and the repair was estimated to cost P5,300.
RESPONDENTS: Court of Appeals (Fourth Division) and Carlos F. Robes
26.   Robes instituted a civil case in the CFI of Rizal because the insurance
company refused either to pay for the repair or to cause the restoration of
SUMMARY: On Mar. 1, 1961, Robes took an insurance, with the CCC Insurance
the car to its original condition
Corp, on his Dodge Kingsway car against loss or damage through accident for an
27.   The civil case was for recovery not only of the amount necessary for the
amount not exceeding P8000 (policy). On June 25 (while policy was effective),
repair of the insured car but also of actual and moral damages, attys fees
Robes’ driver, while driving the insured car, got into a collision. The cost of damages
and costs
was estimated to cost P5,300. The insurance company refused to either pay for the
28.   Resisting plaintiff’s (Robes) claim, the insurance company disclaimed
repair or the restoration so Robes instituted a civil case for recovery of the amount
liability for payment, alleging that there had been violation of the insurance
necessary for the repair, actual and moral damages, and attys fees. CCC disclaimed
contract because the one driving the car at the time of the incident was not
liability, alleging that there had been a violation of the insurance contract because the
an “authorized driver”
one driving the car was not an authorized driver. The trial court ruled in favor of
29.   After due hearing, judgment was rendered for Robes and CCC was ordered
Robes, granting him the P5,031.28, for the cost of repair, actual and moral damages
to pay the cost of repair in the sum of P5,031.28; the sum of P150, for the
and attorneys fees. CA affirmed this except for actual damages, because it was said
hauling and impounding of the car at the repair shop; P2k as actual
to be too speculative. CCC was not content hence, this petition. The issue is WoN
damages; P1k as attys fees, plus costs
the damage to the insured car was not covered by the insurance policy because at the
30.   CCC went to the CA, raising inter alia the questions of the qualification of
time of the accident it was being driven by one who was not an authorized driver.
Robes’ driver to operate the insured vehicle and the correctness of the trial
CCC claims that the driver is not authorized because does not know how to read and
court’s award to Robes of the cost of repairs, actual and moral damages, and
write; that he was able to secure a driver’s license, without passing any examination
attys fees
thereof, by paying P25 to a certain woman; and that the Cavite agency of the Motor
31.   On Jan. 31, 1966, CA affirmed the ruling of the lower court except the
Vehicles Office has certified not having issued Reyes’ purported driver’s license. CA
award of actual damages in the sum of P2k, for being too speculative
argued that since Reyes' purported driver's license bears all the earmarks of a duly
32.   Not content, CCC filed the present petition of the decision of the CA
issued license, then it is a public document, and CCC then has the burden of
disproving its genuineness, which the latter has failed to do. Also, the fatal flaw in
ISSUE/s:
CCC’s argument is that it ignores the provisions of law existing at the time of the
4.   WoN the damage to the insured car was not covered by the insurance policy
mishap, which says that the issuance of a driving license without previous
because at the time of the accident it was being driven by one who was not
examination does not necessarily imply that the license issued is invalid. Considering
an authorized driver (main) – NO. The Court said that Reyes (the driver) is
that, as pointed out by the Court of Appeals the weight of authority is in favor of a
authorized and his license is genuine, as issued by the MVO. A
liberal interpretation of the insurance policy for the benefit of the party insured, and
5.   WoN the proceedings observed in the trial court were irregular and invalid
strictly against the insurer, SC finds no reason to diverge from the conclusion
– NO. There is nothing wrong with the practice of delegating to a
reached by the Court of Appeals that no breach was committed of the above- quoted
commissioner, usually the clerk of court, who is a duly sworn court officer,
provision of the policy.
the reception of both parties and for him to submit a report thereon to the
court. (not relevant)
DOCTRINE: The weight of authority is in favor of a liberal interpretation of the
insurance policy for the benefit of the party insured, and strictly against the insurer
RULING: the decision of the CA is affirmed, with costs against CCC

FACTS:
22.   This is a petition for review of the decision of the CA, affirming that of the
CFI of Rizal, QC, allowing insurance indemnification of plaintiff for his
damaged car and the payment of atty’s fees
23.   On March 1, 1961, Carlos F. Robes took an insurance, with the CCC
Insurance Corporation (CCC), on his Dodge Kingsway car against loss or
RATIO: license without previous examination does not necessarily imply that the
18.   It is vigorously urged by the insurer that the one driving the insured vehicle license issued is invalid.
at the time of the accident was not an authorized driver thereof within the 25.   As the law stood in 1961, when the claim arose, the examinations could be
5
purview of the provision of the insurance policy dispensed with in the discretion of the Motor Vehicles Office officials.
19.   It has been found as a fact by the CA that Domingo Reyes, the driver, does Whether discretion was abused in issuing the license without examination is
not know how to read and write; that he was able to secure a driver’s not a proper subject of inquiry in these proceedings, though, as a matter of
license, without passing any examination thereof, by paying P25 to a certain legislative policy, the discretion should be eliminated. There is no proof that
woman; and that the Cavite agency of the Motor Vehicles Office has the owner of the automobile knew that the circumstances surrounding such
certified not having issued Reyes’ purported driver’s license issuance showed that it was irregular.
20.   In holding that the damage sustained by the car comes within the coverage 26.   The issuance of the license is proof that the Motor Vehicles Office official
of the insurance policy, the CA argued that since Reyes' purported driver's considered Reyes, the driver of Robes, qualified to operate motor vehicles,
license bears all the earmarks of a duly issued license, then it is a public and the insured was entitled to rely upon such license.
document, and CCC then has the burden of disproving its genuineness, 27.   In this connection, it should be observed that the chauffeur, Reyes, had been
which the latter has failed to do driving since 1957, and without mishap, for all the record shows.
21.   CA ruled: 28.   Considering that, as pointed out by the Court of Appeals the weight of
a. the fact that the Cavite Agency of the Motor Vehicles Office authority is in favor of a liberal interpretation of the insurance policy
states that Drivers License was not issued by that office, does not for the benefit of the party insured, and strictly against the insurer, SC
remove the possibility that said office may have been mistaken or finds no reason to diverge from the conclusion reached by the Court of
that said license was issued by another agency. Appeals that no breach was committed of the above- quoted provision of
b. Indeed a certain Gloria Presa made the notation thereon "no the policy.
license issued" and which notation was the basis of the 1st
Indorsement, signed by the MVO Cavite City Agency's officer-in- 2nd issue
charge. 29.   The next issue is anchored on the fact that the decision of the trial court was
c. Neither Gloria Presa nor the officer-in-charge Marciano A. based on evidence presented to and received by the clerk of court who acted
Monzon was placed on the witness stand to be examined in order as commissioner, although allegedly, there was no written court order
to determine whether said license is indeed void. constituting him as such commissioner, no written request for his
d. thus, the fact remains that Domingo Reyes is in possession of a commission was made by the parties; he did not take an oath prior to
driver's license issued by the Motor Vehicles Office which on its entering into the discharge of his commission; no written report of his
face appears to have been regularly issued. findings was ever submitted to the court; and no notice thereof was sent to
22.   CA then found the license as genuine, that is, one really issued by the MVO the parties, contrary to the provisions of Rule 33 of the Rules of Court.
or its authorized deputy; and this finding of fact is now conclusive and may 30.   There is nothing wrong with the practice of delegating to a commissioner,
not be questioned in this appeal usually the clerk of court, who is a duly sworn court officer, the reception of
23.   Nevertheless, CCC insists that, under the established facts of this case, both parties and for him to submit a report thereon to the court.
Reyes, being admittedly one who cannot read and write, who has never 31.   In fact, this procedure is expressly sanctioned by Revised Rule 33 of the
passed any examination for drivers, and has not applied for a license from Rules of Court. CCC’s objection in this case, however, is directed not
the duly constituted government agency entrusted with the duty of licensing against its referral to the clerk of court but against the alleged non-
drivers, cannot be considered an authorized driver. observance of the prescribed steps in connection with such delegation.
24.   The fatal flaw in CCC’s argument is that it ignores the provisions of law 32.   SC finds no cause sufficient to invalidate the proceedings had in the trial
6 court. SC notes that this issue was brought up by CCC for the first time only
existing at the time of the mishap, which says that the issuance of a driving
in its motion for reconsideration filed in the Court of Appeals.
                                                                                                                                                                                                                                                                                                                                                                                                               
5
 "AUTHORIZED DRIVER: "Any of the following: 6
Under Section 24 of the revised Motor Vehicles Law:
(a)   The insured; “An examination or demonstration to show any applicant's ability to operate motor vehicles may also be
(b)   Any person driving on the Insured's order or with his permission, provided that the person required in the discretion of the Chief, Motor Vehicles Office or his deputies."
driving is permitted in accordance with licensing laws or regulations to drive the motor vehicle And reinforcing Section 24, Section 26 prescribes further:
covered by this Policy, or has been so permitted and is not disqualified by order of a court of "SEC. 26. Issuance of chauffeur's license; professional badge: If, after examination, or without the same,
law or by reason of any enactment or regulation from driving such Motor Vehicle." (Italics the Chief, Motor Vehicles Office or his deputies, believe the applicant to possess the necessary
ours) qualifications and knowledge, they shall issue to such applicant a license to operate as chauffeur . . ."
33.   It was not raised in the trial court, where the defect could still be remedied.
This circumstance precludes ventilation of the issue of validity of the
hearing at this stage; for, if such irregularity is to vitiate the proceeding, the
question should have been seasonably raised, i.e., either before the parties
proceeded with the hearing or before the court handed down its ruling. It is
a procedural point that can be waived by consent of the parties, express or
implied.
34.   For the same reason, appellant cannot insist now on the annulment of the
proceeding on the basis of alleged lack of written consent of the parties to
the commission, or of an order appointing the clerk as commissioner, or of
notice of the submission of his report to the court.
35.   Furthermore, appellant has presented no proof that the clerk of court
committed any mistake or abuse in the performance of the task entrusted to
him, or that the trial court was not able to properly appreciate the evidence
in the case because it was received by another person. If indeed there were
errors at all, they would be non-prejudicial and could not justify the holding
of a new trial, as urged by herein petitioner.
003 Assoc. of Baptists for World Evangelism, Inc. v. Fieldmen’s with Motor No. 032433272555 and Plate No. E-73317 covered by
Insurance (Punsalan) Registration Certificate No. 288141 Rizal, issued by the Davao Motor
Sept. 21, 1983 | Melencio-Herrera, J. | Joyride in Davao; Insurance Contract Vehicles Office Agency No. 20 and owned by Reverend Clinton Bonnel
Interpretation 25.   Association insured said vehicle with Fieldmen’s with Private Car
Comprehensive Policy No. 22 (Comprehensive Policy) against loss or
damage up to P5,000.00.
PETITIONER: Association of Baptists for World Evangelism, Inc.
26.   1961: Association’s representative Dr. Antonio Lim (Dr. Lim) placed the
RESPONDENTS: Fieldmen’s Insurance Co., Inc.
Chevrolet Carry-all at the Jones Monument Mobilgas Service Station at
Davao City, under care of station operator Rene Te (Te) so that the vehicle
SUMMARY: Association has an insurable interest in its Chervrolet Carry-all. It
could be displayed as being for sale and that the station boys would
insured the vehicle with Fieldmen’s with Private Car Comprehensive Policy No.
receive 2% commission should they sell the vehicle.
22 (Comprehensive Policy) against loss or damage up to P5,000. Assoc.’s
27.   1962: One night, Romeo Catiben (station boy in said service station) and a
representative, Dr. Lim, proceeded to Jones Monument Mobilgas Service Station
nephew (unnamed in the case) of the wife of Te, took the vehicle for a
at Davao City where he left the car so that it could be displayed for sale and the
joyrde in Toril, Davao City, without the prior permission, authority, or
station boys would be entitled to 2% commission for it. One night, Romeo
consent of the Association or its rep. Dr. Lim, or of Te, and on its way back
Catiben (one of the station boys) and the nephew of station operator Rene Te,
to Davao City, the vehicle experienced a mechanical defect which led
rode the Chevrolet Carry-all and took it for a joyride to Toril, Davao City, w/o
into an accident, bumping an electric post (actual damages at
the consent of Association, Dr. Lim, or Te. Upon returning, however, the vehicle
P5,518.61)
experienced a mechanical defect and they accidentally bumped into an electric
28.   When the matter was brought up with the Trial Court, it tackled the issue of
post (actual damage from it was P5,518.61). Thus, Association wants to claim
whether the there should be a prior criminal conviction of Catiben for theft
the P5k compensation for the damage sustained by the vehicle but Fieldmen’s
before Association can claim compensation for the actual damage caused to
argue that there was no prior conviction of Catiben for theft of the vehicle, so
its vehicle under the Comprehensive Policy.
they did not give the insurance claim.
29.   The trial court rendered judgment and ordered Fieldmen’s to pay
Association the amount of P5,000.00 as indemnity for the damage sustained
ISSUE: WoN Catiben’s acts constitutes theft under the Comprehensive Policy –
by the vehicle, P2k for atty’s fees, and costs.
YES
30.   Upon appeal, the CA elevated the matter to the SC on a question of law.
It is settled that the act of Catiben in taking the vehicle for a joy ride to Toril,
ISSUE/s:
Davao City, constitutes theft within the meaning of the insurance policy and
5.   WoN the acts of Catiben constitutes theft under the Comprehensive Policy
that recovery for damage to the car is not barred by the illegal use of the car
of Fieldmen’s, thus entitling Association to compensation – YES, because
by one of the station boys. Even without prior conviction, his intent to gain is
theft under the policy does not need a prior conviction; his intent to gain
evident since he derives therefrom utility, satisfaction, enjoyment, and pleasure.
was evident from the joyride. As long as it can be proven by preponderance
Thus, there is no need for prior conviction for the crime of theft to make an
of evidence in a civil action for recovery, the amount on an insurance policy
insurer liable under the theft clause of the policy. (proceed to doctrine)
may be recovered.
DOCTRINE: In a civil action for recovery on an automobile insurance, the
RULING: Judgment in question is affirmed.
question whether a person using a certain automobile at the time of the accident
stole it or not is to be determined by a fair preponderance of evidence and
RATIO:
NOT by the rule of criminal law requiring proof of guilt beyond reasonable
13.   The Comprehensive Policy issued by the insurance company includes loss
doubt.
of or damage to the motor vehicle by "burglary . . . or theft.
14.   It is settled that the act of Catiben in taking the vehicle for a joy ride to
FACTS: Toril, Davao City, constitutes theft within the meaning of the insurance
23.   Association of Baptists for World Evangelism, Inc. (Association) is a policy and that recovery for damage to the car is not barred by the illegal
religious corporation duly organized and registered in PH; Fieldmen’s use of the car by one of the station boys.
Insurance Co., Inc. (Fieldmen’s) is also a domestic corporation duly 15.   Where a car is unlawfully and wrongfully taken by some people, be they
organized and existing under the laws of the PH employees of the car shop or not to whom it had been entrusted, and taken
24.   Association has an insurable interest in a Chevrolet Carry-all, 1955 Model
on a long trip to Montalban without the owner’s consent or knowledge,
constitutes or partakes of the nature of theft as defined in Art. 308 of the
RPC:
(W)ho are liable for theft. — … any person who, with intent to
gain but without violence against or intimidation of persons nor
force upon things, shall take personal property of another without
the latter’s consent
16.   Court sustains that when a person, either with the object of going to a
certain place or learning how to drive or enjoying a free ride, takes
possession of a vehicle belonging to another, w/o the latter’s consent, he is
guilty of theft because his intent to gain is evident since he derives
therefrom utility, satisfaction, enjoyment, and pleasure.
17.   Thus, there is no need for prior conviction for the crime of theft to make an
insurer liable under the theft clause of the policy.
18.   The facts are stipulated where Catiben took the car for a joyride and
thereafter bumped it against an electric post resulting in damages. The act is
theft within the policy of insurance.
19.   In a civil action for recovery on an automobile insurance, the question
whether a person using a certain automobile at the time of the accident stole
it or not is to be determined by a fair preponderance of evidence and
NOT by the rule of criminal law requiring proof of guilt beyond
reasonable doubt. (NOTE: besides, there was no provision in the policy
requiring prior criminal conviction for theft)
004 Mayer Steel Pipe Corporation v. CA (Cristelle) evidenced by Invoice Nos. MSPC-1014, MSPC-1015, MSPC-1025, MSPC-
June 19, 1997 | Puno J. | Insurance Contract Interpretation 1020, MSPC-1017 and MSPC-1022.
3.   Prior to the shipping, petitioner Mayer insured the pipes and fittings against
PETITIONER: MAYER STEEL PIPE CORPORATION and HONGKONG all risks with private respondents South Sea Surety and Insurance Co., Inc.
GOVERNMENT SUPPLIES DEPARTMENT (South Sea) and Charter Insurance Corp. (Charter). The pipes and fittings
RESPONDENTS: COURT OF APPEALS, SOUTH SEA SURETY AND covered by Invoice Nos. MSPC-1014, 1015 and 1025 with a total amount of
INSURANCE CO., INC. and the CHARTER INSURANCE CORPORATION US$212,772.09 were insured with respondent South Sea, while those
SUMMARY: Hong Kong Government Supplies Department contracted Mayer Steel covered by Invoice Nos. 1020, 1017 and 1022 with a total amount of
Pipe Corporation to manufacture and supply various steel pipes and fittings. Prior to US$149,470.00 were insured with respondent Charter.
the shipping, Mayer insured these pipes and fittings against all risks with South Sea 4.   Petitioners Mayer and Hongkong jointly appointed Industrial Inspection
Surety and Insurance Co., Inc. and Charter Insurance Corp., with Industrial Inspection (International) Inc. as third-party inspector to examine whether the pipes
Inc. appointed as third-party inspector. After examining the pipes and fittings, and fittings are manufactured in accordance with the specifications in the
Industrial Inspection certified that they are in good order condition. However, when contract. Industrial Inspection certified all the pipes and fittings to be in
the goods reached Hong Kong, it was discovered that a substantial portion thereof was good order condition before they were loaded in the vessel. Nonetheless,
damaged. The trial court found in favor of the insured. However, when the case was when the goods reached Hongkong, it was discovered that a substantial
elevated to the CA, it set aside the decision of the trial court and dismissed the portion thereof was damaged.
complaint on the ground of prescription. It held that the action was barred under Sec. 5.   Hongkong & Mayer filed a claim against South Sea surety & charter
3(6) of the Carriage of Goods by Sea Act (COGSA) since it was filed only on April insurance corp. for indemnity under the insurance contract. Respondent
17, 1986, more than two years from the time the goods were unloaded from the Charter paid petitioner Hongkong the amount of HK$64,904.75. HK &
vessel. Issue: Whether or not the action is barred by prescription? The cause of Mayer demanded payment of the balance of HK$299,345.30 representing
action had not yet prescribed. Sec. 3(6) of the COGSA states that the carrier and the the cost of repair of the damaged pipes. South Sea & charter refused to pay
ship shall be discharged from all liability for loss or damage to the goods if no suit is because the insurance surveyor's report allegedly showed that the damage is
filed within one year after delivery of the goods or the date when they should have a factory defect.
been delivered. Under this provision, only the carrier’s liability is extinguished if no 6.   On April 17, 1986, petitioners filed an action against South Sea & charter to
suit is brought within one year. But the liability of the insurer is not extinguished recover the sum of HK$299,345.30 South Sea & charter averred that they
because the insurer’s liability is based not on the contract of carriage but on the have no obligation to pay the amount claimed by petitioners because the
contract of insurance. Thus, when private respondents issued the “all risks” policies to damage to the goods is due to factory defects which are not covered by the
Mayer, they bound themselves to indemnify the latter in case of loss or damage to the insurance policies.
goods insured. Such obligation prescribes in ten years, in accordance with Article 7.   TRIAL COURT - ruled in favor of petitioners. It found that the damage to
1144 of the New Civil Code. the goods is not due to manufacturing defects. It also noted that the
DOCTRINE: An insurance contract is a contract whereby one party, for a insurance contracts executed by petitioner Mayer and private respondents
consideration known as the premium, agrees to indemnify another for loss or damage are "all risks" policies which insure against all causes of conceivable loss or
which he may suffer from a specified peril. An “all risks” insurance policy covers all damage. The only exceptions are those excluded in the policy, or those
kinds of loss other than those due to willful and fraudulent act of the insured. sustained due to fraud or intentional misconduct on the part of the insured.
8.   South Sea & charter elevated the case to respondent Court of Appeals.
FACTS: 9.   CA - affirmed the finding of the trial court that the damage is not due to
1.   This is a petition for review on certiorari to annul and set aside the Decision factory defect and that it was covered by the "all risks" insurance policies
of respondent CA dated December 14, 1995 and its Resolution dated issued by South Sea & charter to petitioner Mayer. However, it set aside the
February 22, 1996 in CA-G.R. CV No. 45805 entitled Mayer Steel Pipe decision of the trial court and dismissed the complaint on the ground of
Corporation and Hongkong Government Supplies Department v. South Sea prescription. It held that the action is barred under Section 3(6) of the
Surety Insurance Co., Inc. and The Charter Insurance Corporation. Carriage of Goods by Sea Act since it was filed only on April 17, 1986,
2.   In 1983, petitioner Hongkong Government Supplies Department more than two years from the time the goods were unloaded from the
(Hongkong) contracted petitioner Mayer Steel Pipe Corporation (Mayer) to vessel.
manufacture and supply various steel pipes and fittings. From August to a.   Section 3(6) of the Carriage of Goods by Sea Act provides that
October, 1983, Mayer shipped the pipes and fittings to Hongkong as "the carrier and the ship shall be discharged from all liability in
respect of loss or damage unless suit is brought within one year
after delivery of the goods or the date when the goods should have third-party complaint against the carrier for reimbursement of the amount it
been delivered." Respondent court ruled that this provision applies paid to the shipper. The insurer filed the third-party complaint on January 9,
not only to the carrier but also to the insurer, citing Filipino 1978, more than one year after delivery of the goods on December 17,
Merchants Insurance Co., Inc. vs. Alejandro. 1977. The court held that the Insurer was already barred from filing a claim
10.   Hence this petition for review. against the carrier because under the Carriage of Goods by Sea Act, the suit
a.   CA erred in holding that petitioners' cause of action had already against the carrier must be filed within one year after delivery of the goods
prescribed on the mistaken application of the Carriage of Goods by or the date when the goods should have been delivered. The court said that
Sea Act and the doctrine of Filipino Merchants Co., Inc. v. "the coverage of the Act includes the insurer of the goods."
Alejandro (145 SCRA 42); and 4.   The Filipino Merchants case is different from the case at bar. In Filipino
b.   CA committed an error in dismissing the complaint. Merchants, it was the insurer which filed a claim against the carrier for
reimbursement of the amount it paid to the shipper. In the case at bar, it was
ISSUES: Whether or not the action is barred by prescription? NO. The petition is the shipper which filed a claim against the insurer. The basis of the shipper's
impressed with merit. Respondent court erred in applying Section 3(6) of the claim is the "all risks" insurance policies issued by private respondents to
Carriage of Goods by Sea Act. An insurance contract is a contract whereby one petitioner Mayer.
party, for a consideration known as the premium, agrees to indemnify another for 5.   The ruling in Filipino Merchants should apply only to suits against the
loss or damage which he may suffer from a specified peril. An "all risks" insurance carrier filed either by the shipper, the consignee or the insurer. When
policy covers all kinds of loss other than those due to willful and fraudulent act of the court said in Filipino Merchants that Section 3(6) of the Carriage of
the insured. Thus, when private respondents issued the "all risks" policies to Goods by Sea Act applies to the insurer, it meant that the insurer, like the
petitioner Mayer, they bound themselves to indemnify the latter in case of loss or shipper, may no longer file a claim against the carrier beyond the one-year
damage to the goods insured. Such obligation prescribes in ten years, in accordance period provided in the law. But it does not mean that the shipper may no
with Article 1144 of the New Civil Code. longer file a claim against the insurer because the basis of the insurer's
liability is the insurance contract.
RULING: IN VIEW WHEREOF, the petition is GRANTED. The Decision of
respondent Court of Appeals dated December 14, 1995 and its Resolution dated
February 22, 1996 are hereby SET ASIDE and the Decision of the Regional Trial
Court is hereby REINSTATED. No costs. SO ORDERED.
RATIO:
1.   Section 3(6) of the Carriage of Goods by Sea Act states that the carrier and
the ship shall be discharged from all liability for loss or damage to the
goods if no suit is filed within one year after delivery of the goods or the
date when they should have been delivered.
2.   Under this provision, only the carrier's liability is extinguished if no suit is
brought within one year. But the liability of the insurer is not extinguished
because the insurer's liability is based not on the contract of carriage but on
the contract of insurance. A close reading of the law reveals that the
Carriage of Goods by Sea Act governs the relationship between the carrier
on the one hand and the shipper, the consignee and/or the insurer on the
other hand. It defines the obligations of the carrier under the contract of
carriage. It does not, however, affect the relationship between the shipper
and the insurer. The latter case is governed by the Insurance Code.
3.   Our ruling in Filipino Merchants Insurance Co., Inc. v. Alejandro and the
other cases cited therein does not support respondent court's view that the
insurer's liability prescribes after one year if no action for indemnity is filed
against the carrier or the insurer. In that case, the shipper filed a complaint
against the insurer for recovery of a sum of money as indemnity for the loss
and damage sustained by the insured goods. The insurer, in turn, filed a
005 Great Pacific Life v. CA (Rosales) corresponding form in his own handwriting. Mondragon finally type-wrote
April 30, 1979 | De Castro, J. | Insurance Contract Interpretation the data on the application form which was signed by Ngo Hing.
2.   The Ngo Hing paid the annual premuim the sum of P1,077.75 going over to
PETITIONER: Great Pacific Life Assurance Company; Lapulapu D. the Company, but he retained the amount of P1,317.00 as his commission
Mondragon for being a duly authorized agent of Pacific Life. Upon the payment of the
RESPONDENTS: Court of Appeals; Ngo Hing insurance premuim, the binding deposit receipt was issued to Ngo Hing.
Likewise, Mondragon handwrote at the bottom of the back page of the
SUMMARY: Ngo Hing filed an application with the Great Pacific Life application form his strong recommendation for the approval of the
Assurance for a twenty-year endownment policy in the amount of P50,000.00 on insurance application. Then on April 30, 1957, Mondragon received a letter
the life of his one-year old daughter Helen Go. Pacific Life supplied the essential from Pacific Life disapproving the insurance application. The letter stated
data which Lapulapu D. Mondragon, Branch Manager of the Pacific Life in that the said life insurance application for 20-year endowment plan is not
Cebu City wrote on the corresponding form. Upon the payment of the insurance available for minors below seven years old, but Pacific Life can consider
premuim, the binding deposit receipt was issued to Ngo Hing. Likewise, the same under the Juvenile Triple Action Plan, and advised that if the offer
Mondragon handwrote at the bottom of the back page of the application form his is acceptable, the Juvenile Non-Medical Declaration be sent to the
strong recommendation for the approval of the insurance application. company.
Mondragon received a letter from Pacific Life disapproving the insurance 3.   The non-acceptance of the insurance plan by Pacific Life was allegedly not
application. The non-acceptance of the insurance plan by Pacific Life was communicated by Mondragon to Ngo Hing. Instead, on May 6, 1957,
allegedly not communicated by Mondragon to Ngo Hing. Instead, Mondragon Mondragon wrote back Pacific Life again strongly recommending the
wrote back Pacific Life again strongly recommending the approval of the 20- approval of the 20-year endowment insurance plan to children, pointing out
year endowment insurance plan. Helen Go died of influenza with complication that since 1954 the customers, especially the Chinese, were asking for such
of bronchopneumonia. Thereupon, Ngo Hing sought the payment of the coverage.
proceeds of the insurance, but having failed in his effort, he filed the action for 4.   It was when things were in such state that on May 28, 1957 Helen Go died
the recovery of the same. Issue is WoN the binding deposit receipt constituted a of influenza with complication of bronchopneumonia. Thereupon, Ngo
temporary contract of the life insurance in question? The SC held NO, because Hing sought the payment of the proceeds of the insurance, but having failed
there is an absence of a meeting of the minds between Pacific Life and Ngo Hing in his effort, he filed the action for the recovery of the same before the
and there is and non-compliance of the abovequoted conditions stated in the Court of First Instance of Cebu, which rendered the adverse decision as
disputed binding deposit receipt. As held by this Court, where an agreement is earlier refered to against both Pacific Life and Mondragon.
made between the applicant and the agent, no liability shall attach until the
principal approves the risk and a receipt is given by the agent. The acceptance is ISSUE/s:
merely conditional and is subordinated to the act of the company in approving or 6.   WoN the binding deposit receipt constituted a temporary contract of the life
rejecting the application. Thus, in life insurance, a "binding slip" or "binding insurance in question – NO, because there is an absence of a meeting of the
receipt" does not insure by itself. minds between Pacific Life and Ngo Hing and there is and non-compliance
of the abovequoted conditions stated in the disputed binding deposit receipt.
DOCTRINE: A contract of insurance, like other contracts, must be assented to
by both parties either in person or by their agents ... The contract, to be binding RULING: WHEREFORE, the decision appealed from is hereby set aside, and in
from the date of the application, must have been a completed contract, one that lieu thereof, one is hereby entered absolving petitioners Lapulapu D. Mondragon and
leaves nothing to be dione, nothing to be completed, nothing to be passed upon, Great Pacific Life Assurance Company from their civil liabilities as found by
or determined, before it shall take effect. There can be no contract of insurance respondent Court and ordering the aforesaid insurance company to reimburse the
unless the minds of the parties have met in agreement. amount of P1,077.75, without interest, to private respondent, Ngo Hing. Costs
against private respondent.
FACTS: RATIO:
1.   It appears that on March 14, 1957, Ngo Hing filed an application with the 1.   At the back of the binding receipt are condition precedents required before a
Great Pacific Life Assurance for a twenty-year endownment policy in the deposit is considered a BINDING RECEIPT. The aforequoted provisions
amount of P50,000.00 on the life of his one-year old daughter Helen Go. printed show that the binding deposit receipt is intended to be merely a
Pacific Life supplied the essential data which Lapulapu D. Mondragon,
Branch Manager of the Pacific Life in Cebu City wrote on the
provisional or temporary insurance contract and only upon compliance of application would not have any adverse effect on the allegedly perfected
the following conditions: temporary contract. There was no contract perfected between the parties
(1)   that the company shall be satisfied that the applicant who had no meeting of their minds. Ngo Hing, being an authorized
was insurable on standard rates; insurance agent of Pacific Life at Cebu branch office, is indubitably aware
(2)   that if the company does not accept the application and that said company does not offer the life insurance applied for. When he
offers to issue a policy for a different plan, the filed the insurance application in dispute, Ngo Hing was, therefore, only
insurance contract shall not be binding until the taking the chance that Pacific Life will approve the recommendation of
applicant accepts the policy offered; otherwise, the Mondragon for the acceptance and approval of the application in question
deposit shall be reftmded; and along with his proposal that the insurance company starts to offer the 20-
(3)   that if the applicant is not able according to the standard year endowment insurance plan for children less than seven years.
rates, and the company disapproves the application, the Nonetheless, the record discloses that Pacific Life had rejected the proposal
insurance applied for shall not be in force at any time, and recommendation.
and the premium paid shall be returned to the applicant. 7.   Also, having an insurable interest on the life of his one-year old daughter,
2.   Clearly implied from the aforesaid conditions is that the binding deposit aside from being an insurance agent and an offense associate of
receipt in question is merely an acknowledgment, on behalf of the company, Mondragon, Ngo Hing must have known and followed the progress on the
that the latter's branch office had received from the applicant the insurance processing of such application and could not pretend ignorance of the
premium and had accepted the application subject for processing by the Company's rejection of the 20-year endowment life insurance application.
insurance company; and that the latter will either approve or reject the same 8.   With regard to the issue that Ngo Hing concealed the state of health and
on the basis of whether or not the applicant is "insurable on standard rates." physical condition of Helen Go, which rendered void the aforesaid Binding
Since Pacific Life disapproved the insurance application of Ngo Hing, the Receipt, where he supplied the required essential data for the insurance
binding deposit receipt in question had never become in force at any time. application form, he was fully aware that his one-year old daughter is
3.   Upon this premise, the binding deposit receipt is, manifestly, merely typically a mongoloid child. Such a congenital physical defect could never
conditional and does not insure outright. As held by this Court, where an be ensconced nor disguished. Nonetheless, Ngo Hing, in apparent bad faith,
agreement is made between the applicant and the agent, no liability withheld the fact materal to the risk to be assumed by the insurance
shall attach until the principal approves the risk and a receipt is given compary. As an insurance agent of Pacific Life, he ought to know, as he
by the agent. The acceptance is merely conditional and is subordinated surely must have known his duty and responsibility to such a material fact.
to the act of the company in approving or rejecting the application. 9.   The contract of insurance is one of perfect good faith uberrima fides
Thus, in life insurance, a "binding slip" or "binding receipt" does not meaning good faith, absolute and perfect candor or openness and honesty;
insure by itself. the absence of any concealment or demotion, however slight, not for the
4.   In the absence of a meeting of the minds between Pacific Life and Ngo alone but equally so for the insurer. Concealment is a neglect to
Hing over the 20-year endowment life insurance in the amount of communicate that which a party knows and ought to communicate. Whether
P50,000.00 in favor of the latter's one-year old daughter, and with the non- intentional or unintentional the concealment entitles the insurer to rescind
compliance of the abovequoted conditions stated in the disputed binding the contract of insurance.
deposit receipt, there could have been no insurance contract duly perfected
between the, Accordingly, the deposit paid by Ngo Hing shall have to be
refunded by Pacific Life.
5.   As held in De Lim vs. Sun Life Assurance Company of Canada, supra, "a
contract of insurance, like other contracts, must be assented to by both
parties either in person or by their agents ... The contract, to be binding
from the date of the application, must have been a completed contract,
one that leaves nothing to be dione, nothing to be completed, nothing to
be passed upon, or determined, before it shall take effect. There can be
no contract of insurance unless the minds of the parties have met in
agreement."
6.   We are not impressed with Ngo Hing’s contention that failure of
Mondragon to communicate to him the rejection of the insurance
006 AMERICAN HOME ASSURANCE COMPANY v. TANTUCO
ENTERPRISES (Sabaupan) FACTS:
8 October 2001 | Puno, J. | Insurance Contract Interpretation 31.   Tantuco Enterprises, Inc., engaged in the coconut oil milling and refining
industry, owns two oil mills. Both are located at its factory compound in
Lucena City. Initially, it has only one oil mill. It started operating its second
PETITIONER: American Home Assurance Company
oil mill in 1988 and this oil mill is commonly referred to as the new oil mill.
RESPONDENTS: Tantuco Enterprises, Inc.
32.   The two oil mills were separately covered by fire insurance policies issued
by American Home Assurance Co., Philippine Branch. The first oil mill
SUMMARY: Tantuco Enterprises owns two oil mills, one of which was
was insured for P3 million for the period 1 March 1991 to 1992 while the
commonly referred to as the new oil mill. Both oil mills are covered by
new oil mill is insured for P6 million for the same term.
insurance policy from American Home Assurance for P3 million and P6 million.
33.   In September 1991, a fire broke out which gutted and consumed the new oil
A fire broke which gutted and consumed the new oil mill. Tantuco immediately
mill. Tantuco immediately notified American Home of the incident. After
informed the insurance company of the incident. However, the insurance
inspection, American Home rejected Tantuco’s claim for the insurance
company refused to pay on the ground that the building covered as specified by
proceeds on the ground that no policy was issued by it covering the burned
the boundaries stated in the policy is not the building that was gutted by fire.
oil mill. It stated that the description of the insured establishment referred to
Tantuco filed a claim for specific performance and damages with the RTC. The
another building. The policies allegedly extend insurance coverage to oil
RTC and CA both ruled that American Home Assurance is liable under the
mill under Building No. 5, whilst the affected oil mill is under Building No.
insurance policy. The issue is whether the insurance policy covers the new oil
14.
mill. The Supreme Court ruled in the affirmative. Notwithstanding the
34.   Tantuco instituted a complaint for specific performance and damages with
misdescription in the policy, it is beyond dispute that what the parties manifestly
the RTC. The lower court ruled that American Home is liable on the
intended to insure was the new oil mill. This is obvious from the categorical
insurance policy. On appeal, the CA affirmed. Hence, the instant petition.
statement embodied in the policy, extending its protection the equipment and
35.   American Home’s arguments:
accessories contained in the new oil mill building. If the parties really intended
a.   The burned oil mill is not covered by any insurance policy. The oil
to protect the first oil mill, then there is no need to specify it as new. It would be
mill insured is specifically described in the policy by its boundaries
absurd to assume the Tantuco would protect its first oil mill for different
and the oil mill gutted by fire was not the one described by the
amounts and leave uncovered the new oil mill. The imperfection in the
specific boundaries in the contested policy.
description of the insured oil mill’s boundaries can be attributed to a
b.   Tantuco did not have the supposed wrong description or mistake
misunderstanding between American Home’s general agent, Mr. Borja, and its
corrected despite the fact that the policy was issued 3 years before
policy issuing clerk, who made the error of copying the boundaries of the first oil
the dire and the despite the “Important Notice” in the policy that
mill when typing the policy to be issued for the new one. Mr. Borja testified that
states “Please read and examine the policy and if incorrect, return
the intention of Tantuco is to cover the new oil mill and the policy issuing clerk
it immediately for alteration.”
copied the description of the boundaries from the old policy, but she inserted
c.   Tantuco is barred by the parole evidence rule from presenting
covering the new oil mill, and to Mr. Borja, the important thing is that it covered
evidence other than the policy of its self-serving intention that it
the new oil mill because it is just within one compound and there are only two
intended really to insure the burned oil mill just as it is barred by
oil mills. It is thus clear that the source of the discrepancy happened during the
estoppel from claiming that the description of the insured oil mill
preparation of the written contract.
in the policy was wrong.
d.   Tantuco forfeited the renewal policy for its failure to pay the full
DOCTRINE: In view of the custom of insurance agents to examine buildings
amount of the premium and breach of the Fire Extinguishing
before writing policies upon them, and since a mistake as to the identity and
Appliances Warranty.
character of the building is extremely unlikely, the courts are inclined to consider
that the policy of insurance covers any building which the parties manifestly
ISSUE/s:
intended to insure, however inaccurate the description may be.
7.   Whether insurance policy covers the new oil mill. YES because the intent
was the parties was to cover the new oil mill and the courts are inclined to
In determining what the parties intended, the courts will read and construe the
consider that the policy of insurance covers any building which the parties
policy as a whole and if possible, give effect to all the parts of the contract,
manifestly intended to insure, however inaccurate the description may be.
keeping in mind always, however, the prime rule that in the event of doubt, this
doubt is to be resolved against the insurer.
RULING: Judgment in question is affirmed. Petition dismissed. witness to testify that Tantuco indeed failed to pay the full amount of the
premium.
RATIO: 26.   On the claim of violation of warranty, the SC held that the warranty did not
Insurance Issue require Tantuco to provide for all the fire extinguishing appliances (portable
20.   In construing the words used descriptive of a building insured, the greatest extinguishers, internal and external hydrants, fire pump, and 24-hour
liberality is shown by the courts in giving effect to the insurance. In view of security services) enumerated therein. Neither did it require that the
the custom of insurance agents to examine buildings before writing policies appliances are restricted to those mentioned in the warranty. What the
upon them, and since a mistake as to the identity and character of the warranty mandates is that Tantuco should maintain in efficient working
building is extremely unlikely, the courts are inclined to consider that the condition within the premises of the insured property, firefighting
policy of insurance covers any building which the parties manifestly equipment such as, but not limited to, those identified in the list. Tantuco
intended to insure, however inaccurate the description may be. was compliant with the warranty for within the vicinity of the new oil mill
21.   Notwithstanding the misdescription in the policy, it is beyond dispute that are the following: numerous portable fire extinguishers, two fire hoses, fire
what the parties manifestly intended to insure was the new oil mill. This is hydrant, and an emergency fire engine.
obvious from the categorical statement embodied in the policy, extending 27.   Not only are warranties strictly construed against the insurer, but they
its protection: should by themselves be reasonably interpreted. That reasonableness is to
“On machineries and equipment with complete accessories usual to a be ascertained in light of the factual conditions prevailing in each case. in
coconut oil mill including stocks of copra, copra cake and copra mills whilst this case, there is no more need for an internal hydrant considering that
contained in the new oil mill building x x x” inside the burned building were numerous portable fire extinguishers, an
22.   If the parties really intended to protect the first oil mill, then there is no emergency fire engine, and a fire hose which has a connection to one of the
need to specify it as new. It would be absurd to assume the Tantuco would external hydrants.
protect its first oil mill for different amounts and leave uncovered the new
oil mill. It is unthinkable for Tantuco to obtain the other policy from the Evidence issue
very same company considering that the first oil mill is already insured. The 28.   The present case falls within one of the recognized exceptions to the parole
latter ought to know that a second agreement over the same realty results in evidence rule. Under the Rules of Court, a party may present evidence to
its overinsurance. modify, explain or add to the terms of the written agreement if he puts in
23.   The imperfection in the description of the insured oil mill’s boundaries can issue in his pleading its failure to express the true intent and agreement of
be attributed to a misunderstanding between American Home’s general the parties thereto.
agent, Mr. Borja, and its policy issuing clerk, who made the error of 29.   In this case, the contractual intention of the parties cannot be understood
copying the boundaries of the first oil mill when typing the policy to be from a mere reading of the instrument. While the contract explicitly
issued for the new one. Mr. Borja testified that the intention of Tantuco is to stipulated that it was for the insurance of the new oil mill, the boundary
cover the new oil mill and the policy issuing clerk copied the description of description written on the policy concededly pertains to the first oil mill.
the boundaries from the old policy but she inserted covering the new oil This irreconcilable difference can only be clarified by admitting evidence
mill, and to Mr. Borja, the important thing is that it covered the new oil mill aliunde, which will explain the imperfection and clarify the intent of the
because it is just within one compound and there are only two oil mills. It is parties.
thus clear that the source of the discrepancy happened during the 30.   On the claim of estoppel, the Court ruled that evidence on record reveals
preparation of the written contract. that Tantuco’s operating manager notified Mr. Borja about the inaccurate
24.   The object of the court in construing a contract is to ascertain the intent of description in the policy. However, Mr. Borja assured Tantuco that the use
the parties and to enforce the agreement which the parties have entered into. of the adjective new will distinguish the insured property. The assurance
In determining what the parties intended, the courts will read and construe convinced Tantuco that despite the impreciseness in the specification of the
the policy as a whole and if possible, give effect to all the parts of the boundaries, the insurance will cover the new oil mill.
contract, keeping in mind always, however, the prime rule that in the event
of doubt, this doubt is to be resolved against the insurer.
25.   On the claim of non-payment of premiums, the CA refused to consider such
on the ground that said issue was raised for the first time on appeal, hence,
beyond its jurisdiction to resolve. The SC agrees. Moreover, American
Home fatally neglected to present, during the whole course of the trial, any
RIZAL SURETY v. CA (Saldua) Mar. 13, 1981. The buildings covered by the said fire insurance policy were
July 18, 2000 | Purisima J. | Art. 1377, New Civil Code also insured with New India Assurance Company, Ltd.
PETITIONER: Rizal Surety & Insurance Company 2.   On Jan. 12, 1981, a fire broke out in the compound of Transworld. The fire
RESPONDENTS: Court of Appeals and Transworld Knitting Mills, Inc. razed the middle portion of its four-span building (hereinafter referred to as
“Main Building”). It also partly gutted ( and thus was also destroyed by the
SUMMARY: fire) the left and right sections of the building (behind the said 4-span
Rizal Surety & Insurance Company issued Fire Insurance Policy No. 45727 in building) where fun & amusement machines and spareparts were stored
favor of Transworld Knitting Mills, Inc. initially for ₱1M, and eventually increased (hereinafter reffered to as “Annex or Annex Building”).
to ₱1.5M covering the period from August 14, 1980 to March 13, 1981. On January 3.   Transworld filed its insurance claims with Rizal Surety and New India
12, 1981, a fire broke out in the compound of Transworld razing the middle portion Assurance, but to no avail.
of its Main Building and partly gutting the left and right sections thereof. A 2- 4.   Thus, Transworld brought against them an action for collection of sum of
storey annex building (behind the four-span building) where fun and amusement money and damages praying for judgment ordering Rizal Insurance and
machines and spare parts were stored, was also destroyed by the fire. Transworld New India to pay the amount of ₱2,747,867.00 plus legal interest,
filed its insurance claims with Rizal Surety and New India Assurance Company, ₱400,000.00 as attorney's fees, exemplary damages, expenses of litigation
but to no avail. On May 26, 1982 Transworld brought against the said insurance of ₱50,000.00 and costs of suit.
companies an action for collection of sum of money and damages. Meanwhile, 5.   Meanwhile, Rizal Insurance countered that the subject fire insurance policy
Rizal Insurance countered that the subject fire insurance policy with Transworld with Transworld covered only the ocntents of the 4-span building (which
covered only the ocntents of the 4-span building (which was party burned), and was party burned), and does not cover the damage caused by the fire on the
does not cover the damage caused by the fire on the 2-storey annex building. 2-storey annex building.
Issue is WON the subject fire insurance policy protected only the contents of the 6.   Trial Court = Dismissed the case against New India. Ordered Rizal Surety
main building and did not include those stored in the annex building = NO. The to pay Transworld ₱826,500.00 representing the actual value of the losses
subject fire insurance policy protected both the contents of the main building and suffered by it.
the annex building. In this case, the loer courts found the so-called annex was 7.   CA = New India is ordered to pay ₱1,818,604.19 while Rizal Surety has to
actually an integral and inseparable part of the four-span building described in the pay Transworld ₱470,328.67, based on the actual losses sustained in the
policy. Consequently, the machines and spare parts stored in the annex were fire. Total amount of ₱2,790,376.00 is against the amounts of fire insurance
covered by the stipulations in the subject fire insurance policy. Verily, the 2-storey coverages respectively extended by New India (₱5,800,000.00) and Rizal
annex building involved a permanent structure which adjoins and Surety (₱1,500,000.00).
intercommunicates with the first right span of the lofty story main building, formed 8.   On Aug. 20, 1993, New India appealed the above order on the ground that
part thereof, and meets the requisites for compensability under the fire insurance Transworld could not be compensated for the loss of the Annex Building
policy sued upon. Moreover, the issue of whether or not private respondent has an because Transworld had no insurable interest in the goods/items stored
insurable interest in the fun and amusement machines and spare parts, which therein. However, CA denied this appeal with finality.
entitles it to be indemnified for the loss thereof, had been settled in the case of New
India Assurance Company vs. CA. Accordingly, the assailed decision and ISSUE/s:
resolution of the Court of Appeals were affirmed. 1.   WON the subject fire insurance policy protected only the contents of the
main building and did not include those stored in the annex building. NO.
DOCTRINE: The stipulation as to the coverage of the fire insurance policy under The subject fire insurance policy protected both the contents of the main
controversy has created a doubt regarding the portions of the building insured building and the annex building.
thereby. Article 1377 of the New Civil Code provides: "Art. 1377. The
interpretation of obscure words or stipulations in a contract shall not favor the party RULING: WHEREFORE, the Decision, dated July 15, 1993, and the Resolution,
who caused the obscurity" dated October 22, 1993, of the Court of Appeals in CA-G.R. CV NO. 28779 are
AFFIRMED in toto. No pronouncement as to costs. SO ORDERED.
FACTS: RATIO:
1.   On March 13, 1980, Rizal Insurance issued Fire Insurance Policy No. 1.   The resolution of this case depends upon the interpretation of the stipulation
45727 in favor of Transworld Knitting Mills Inc. (initialy for ₱1M and in the subject fire insurance policy with reference to the coverage reads:
eventually increased to ₱1.5M). It covered the period from Aug. 14, 1980 – ". . . contained and/or stored during the currency of
this Policy in the premises occupied by them
forming part of the buildings situate (sic) within
own Compound . . ."
2.   Thus, it can be understood from the stipulation that the subject fire
insurance policy did not limit its coverage to what were stored in the main
building only. As said by the RTC, 2 requirements must concur in order
that the Annex Building can be said to be protected by the subject fire
insurance policy:
A.   Said properties must be contained and/or stored in the areas
occupied by Transworld.
B.   Said areas must form part of the building described in the
policy.
3.   In this case, both the RTC and CA found that the Annex was actually an
integral and inseparable part of the main building described in the
policy. Consequently, the fun & amusement machines and spare parts
stored in the Annex Building are also covered by the subject fire
insurance policy.7
4.   The annex building (w/c is a 2-storey building) is a permanent structure w/c
adjoins and intercommunicates with the right span of the lofty storey
building formed part of said lofty storey building. Hence, it meets the
requisites for compensability under the subject fire insurance policy.
5.   Moreover, the 2-storey building (annex) was built in 1978, and the subject
fire insurance policy contract was entered into on Jan. 12, 1981. Given this,
Rizal Surety should have specifically excluded the said 2-storey annex
building from the coverage of the fire insurance.
6.   But Rizal Surety did not, and instead, it went on to provide that the said fire
insurance policy covers the products, raw materials and supplied that are
stored within the premises of Transworld which was an integral part of the
4-span main building occupied by Transworld. This is in spite knowing
fully well about the existence of the annex building that is adjoining and
intercommunicating with the right subject of the main building.
7.   Also, Article 1377, New Civil Code provides that the “interpretation of
obscure words or stipulations in a contract shall not favor the party
who caused the obscurity.” Thus, the doubt should be resolved against
Rizal Surety whose lawyer/managers drafted the subject fire insurance
policy.
8.   Considering that Transworld’s insurable interest in, and compensability for
the loss of subject fun and amusement machines and spare parts, had been
adjudicated, settled and sustained by the Court of Appeals (see fact #8) can
no longer be litigated, Rizal Surety is thus bound by this ruling.

                                                                                                                       
7
Annex Building - Two-storey building constructed of partly timber
and partly concrete hollow blocks under g.i. roof which is adjoining
and intercommunicating with the repair of the first right span of the
lofty storey building and thence by property fence wall.
008 PERLA COMPANIA v. CA (EMAR) d.   she sought reimbursement of said amounts from the Perla Compania, which
May 28, 1990 | Fernan, C.J. | Insurance Contract Interpretation notwithstanding the fact that her claim was within its contractual liability
PETITIONER: PERLA COMPANIA DE SEGUROS, INC. under the insurance policy, refused to make such re-imbursement;
RESPONDENTS: CA and MILAGRO CAYAS e.   she suffered moral damages as a consequence of such refusal, and
SUMMARY: Cayas’ bus, insured by Perla Compania, figured in an accident, injuring f.   she was constrained to secure the services of counsel to protect her rights;
people. 3 injured entered into a settlement with Cayas to which Cayas paid P4k/person and praying that judgment be rendered directing Perla Compania to pay:
while Perea filed a case against Cayas for damages in which the court ruled in favor of a.   P50k for compensation of the injured victims, such sum as the
Perea because Cayas failed to appear in the pre-trial and hence, she was declared as in court might approximate as damages, and
default. Cayas filed a complaint for a sum of money and damages against Perla b.   P6k as atty's fees.
Compania (for items in Fact #5a-f). The CFI ordered Perla Compania P50k for the 6.   Because of Cayas' failure to prosecute the case, the court motu propio ordered
injured passengers. CA affirmed. Perla Compania alleges that its liability should be dismissal without prejudice. Cayas filed an MR, alleging:
limited to a total of P12k, and that it should not be ordered to reimburse Cayas’ a.   She did not receive a copy of the answer to the complaint, and
payments to the 3 injured with whom she entered a settlement without Perla b.   "out of sportsmanship", she did not file a motion to hold Perla Compania in
Compania’s written consent. The SC ruled in favor of Perla Compania, on the grounds default.
that the insurance policy explicitly limits Perla Compania’s liability to P12k/person; 7.   March 1982: The MR was granted.
and required Perla Compania’s written consent before any payment in settlement 8.   2 months later: Cayas filed a motion to declare Perla Compania in default for
of any claim could be made. Cayas is precluded from seeking reimbursement of failure to file an answer. The motion was granted and Cayas was allowed to
the payments made to the 3 injured in view of her failure to comply with the adduce evidence ex-parte. The court rendered judgment by default and ordered
condition contained in the insurance policy Perla Compania to pay Cayas
DOCTRINE: The terms of the contract constitute the measure of the insurer's liability a.   P50k as compensation for the injured passengers,
and compliance therewith is a condition precedent to the insured's right of recovery b.   P5k as moral damages and
from the insurer. c.   P5k as atty’s fees.
9.   Said decision was set aside after the Perla Compania filed a motion therefor.
FACTS:
Trial of the case ensued.
1.   Cayas was the registered owner of a Mazda bus which was insured with Perla
a.   Perla Compania’s defense: before it can be made to pay, the liability must
Compania on February 3, 1978.
first be determined in an appropriate court action.
2.   December 1978: the bus figured in an accident in Naic, Cavite injuring several
b.   Despite CFI’s determination of liability, Perla Compania sought escape from
passengers including19-year old Perea who sued Cayas for damages in the CFI;
its obligation by saying that Cayas lost the case due to her negligence
and del Carmen, Magsarili and Antolin who agreed to a settlement of P4k each.
because of which, efforts exerted by Perla Compania’s lawyers in
3.   Cayas failed to appear in the pre-trial and hence, she was declared as in protecting Cayas' rights proved futile and rendered nugatory. It alleges
default. CFI decided in favor of Perea. that had Cayas cooperated fully with Perla Compania’s lawyers, the
4.   When the decision was about to be executed, Cayas filed a complaint against
latter could have won the suit and thus relieved of any obligation to
Perla Compania in the Office of the Insurance Commissioner praying that Perla
Perea. Even accepting Perla Compania’s postulate, it cannot be said, nor
Compania be ordered to pay P40k for all the claims against her arising from the
was it shown positively and convincingly, that if the case had proceeded
vehicular accident plus legal and other expenses. Realizing her procedural
on trial on the merits, a decision favorable to Cayas could have been
mistake, she later withdrew said complaint.
obtained. Nor was it definitely established that if the pre-trial was
5.   November 11, 1981: Cayas filed a complaint for a sum of money and damages
undertaken in that case, defendant's lawyers could have mitigated the claim
against Perla Compania in the CFI, alleging that:
for damages by Perea against Cayas.
a.   to satisfy the judgment in her previous case, her house and lot were sold at
c.   The court: There being proof that Cayas was compelled to engage the
public auction for P38k;
services of counsel to protect her rights under the insurance policy, the
b.   to avoid numerous suits and the "detention" of the insured vehicle, she paid
court granted P5k atty's fees.
P4k to each of the 4 injured passengers; 10.   May 8, 1987: CA affirmed in toto the decision. MR was denied.
c.   she could not have suffered said financial setback had the counsel for Perla
11.   Perla Compania seeks, before the SC, to limit its liability only to the payment
Compania, who also represented her, appeared at the trial and attended to the
made by Cayas to Perea and only up to the amount of P12K. It denies liability
claims of the 3 other victims;
for Cayas’ payments to the other 3 injured passengers for a total of P12k.
ISSUES: 7.   SC ruled as valid and binding upon Cayas the condition requiring her to secure
1.   WON Perla Compania should pay only P12k – YES, as limited by the contract. the written permission of petitioner before effecting any payment in
2.   WON Cayas is precluded from seeking reimbursement of the payments settlement of any claim against her. There is nothing unreasonable, arbitrary
made to the 3 injured– YES because in view of her failure to comply with the or objectionable in this stipulation as would warrant its nullification. The same
condition in the insurance policy to secure Perla Compania’s written consent was obviously designed to safeguard the insurer's interest against collusion
between the insured and the claimants.
RULING: CA decision is modified in that Perla Compania shall pay Cayas P12k 8.   In her cross-examination, Cayas admitted, thus:
plus legal interest from the promulgation of the decision of the lower court until it is Atty. Yabut:
fully paid and P5k atty's fees. No pronouncement as to costs. Q: With respect to the other injured passengers of your bus wherein you made payments you did not
secure the consent of Perla Compania when you made those payments?
A: I informed them about that.
RATIO: Q: But they did not give you the written authority that you were supposed to pay those claims?
1.   There is merit in La Compania’s assertions. A: No, sir.
2.   The insurance policy involved explicitly limits Perla Compania’s liability to 9.   It being specifically required that Perla Compania’s written consent be first
P12k/person and to P50k/accident. Pertinent provisions of the policy state: secured before any payment in settlement of any claim could be made,
SECTION I-Liability to the Public Cayas is precluded from seeking reimbursement of the payments made to
xxx xxx xxx the 3 injured in view of her failure to comply with the condition contained in the
3. The Limit of Liability stated in Schedule A as applicable
insurance policy.
(a) to 3rd PARTY is the limit of the Company's liability for all damages arising out of death, bodily injury
and damage to property combined so sustained as the result of any 1 accident; 10.   The principle that contracts are respected as the law between the contracting
(b) "per person" for PASSENGER liability is the limit of the Company's liability for all damages arising parties finds application in the present case. Thus, it was error on the part of
out of death or bodily injury sustained by one person as the result of any one accident: the TC and CA to disregard the stipulations of the parties and to have substituted
(c) "per accident" for PASSENGER liability is, subject to the above provisions respecting per person, the
their own interpretation of the insurance policy.
total limit of the Company's liability for all such damages arising out of death or bodily injury sustained
by two or more persons as the result of any one accident. 11.   Phil. American General Insurance vs. Mutuc: Contracts, private laws of the
Conditions Applicable to All Sections contracting parties, should be fulfilled according to the literal sense of their
xxx xxx xxx stipulations, if their terms are clear and leave no room for doubt as to the
5. No admission, offer, promise or payment shall be made by or on behalf of the insured without the intention of the contracting parties, for contracts are obligatory, no matter what
written consent of the Company which shall be entitled, if it so desires, to take over and conduct in his
name the defense or settlement of any claim, or to prosecute in his name for its own benefit any claim for form they may be, whenever the essential requisites for their validity are present.
indemnity or damages or otherwise, and shall have full discretion in the conduct of any proceedings in the 12.   Pacific Oxygen & Acetylene Co. vs. Central Bank: The first and fundamental
settlement of any claim, and the insured shall give all such information and assistance as the Company duty of the courts is the application of the law according to its express terms,
may require. If the Company shall make any payment in settlement of any claim, and such payment
interpretation being called for only when such literal application is
includes any amount not covered by this Policy, the Insured shall repay the Company the amount not so
covered. impossible.
3.   Stokes vs. Malayan Insurance Co.: The terms of the contract constitute the 13.   Patent error: Although Cayas was able to prove a total loss of only P44k,
measure of the insurer's liability and compliance therewith is a condition Perla Compania was made liable for the amount of P50k, the maximum
precedent to the insured's right of recovery from the insurer. liability per accident stipulated in the policy. An insurance indemnity, being
4.   Here, the insurance policy clearly and categorically placed Perla Compania’s merely an assistance or restitution insofar as can be fairly ascertained, cannot be
liability for all damages arising out of death/bodily injury sustained by 1 availed of by any accident victim or claimant as an instrument of enrichment by
person as a result of any 1 accident at P12k. reason of an accident.
5.   Said amount complied with the minimum fixed by the law then prevailing, 14.   SC found no reason to disturb the award of atty's fees.
Sec. 377, PD612 (which was retained by P.D. No. 1460, the Insurance Code of
1978), providing that the liability of land transportation vehicle operators for
bodily injuries sustained by a passenger arising out of the use of their
vehicles shall not be less than P12k, which means that the minimum liability is
P12k/passenger.
6.   Perla Compania’s liability under the insurance contract not being less than
P12k, and therefore not contrary to law, morals, good customs, public
order or public policy, said stipulation must be upheld as effective, valid
and binding as between the parties.
009 Zenith Insurance v. CA (Sarmiento) awarded as well.
May 14, 1990 | Medialeda, J. | Insurance—damages

PETITIONER: Zenith Insurance Corporation


FACTS:
RESPONDENTS: Court of Appeals and Lawrence Fernandez
1.   Private respondent Lawrence Fernandez insured his car for “own damage”
SUMMARY: Lawrence Fernandez insured his car for own damages under a under private car Policy No. 50459 with petitioner Zenith Insurance
private car with petitioner Zenith Insurance. The car had an accident and Corporation.
suffered actual damages in the amount of P3,640.00. Afer being given a run
around by Zenith for 2 months, Fernandez filed a complaint for sum of money 2.   The car figured in an accident and suffered actual damages in the amount of
with damages resulting from the refusal of Zenith to pay the amount claimed. A P3,640.00. After allegedly being given a run around by Zenith for two (2)
decision was rendered by the trial court A decision was rendered by the trial months, Fernandez filed a complaint with the Regional Trial Court
court in favor of private respondent Fernandez. Zenith Insurance was ordered to of Cebu for sum of money and damages resulting from the refusal of Zenith
pay Fernandez, the amount of P3,640.00 representing the damage incurred plus to pay the amount claimed.
interest at the rate of twice the prevailing interest rates; P20,000.00 by way of
moral damages; P20,000.00 by way of exemplary damages, amount of 3.   Zenith filed an answer alleging that it offered to pay the claim of Fernandez
P5,000.00 as attorney’s fees; amount of P3,000.00 as litigation expenses. Zenith pursuant to the terms and conditions of the contract which, the private
insurance filed an appeal. Zenith questions the award of moral, exemplary respondent rejected.
damages and attorney’s fees in an amount more than prayed for in the complaint. a.   Zenith wanted to settle the case, but it did not push through. Trial
The award of damages in case of unreasonable delay in the payment of insurance ensued, and Fernandez presented his evidence. Zenith, however
claims is governed by the Philippine Insurance Code. It is clear that under the failed to present its evidence, without any justifiable reason. The
Insurance Code, in case of unreasonable delay in the payment of the proceeds trial court issued an order submitting the case for decision without
of an insurance policy, the damages that may be awarded are: 1) attorney’s fees; Zenith’s evidence.
2) other expenses incurred by the insured person by reason of such unreasonable b.   Zenith filed a petition for certiorari with the Court of Appeals
denial or withholding of payment; 3) interest at twice the ceiling prescribed by assailing the order of the trial court submitting the case for
the Monetary Board of the amount of the claim due the injured; and 4) the decision without petitioner’s evidence. However, the petition was
amount of the claim. As regards the moral and exemplary damages, it governed denied
by the New Civil Code. The act of petitioner Zenith of delaying payment for two
months cannot be considered as so wanton or malevolent to justify an award of 4.   A decision was rendered by the trial court in favor of private respondent
P20,000.00 as moral damages, taking into consideration also the fact that the Fernandez. Zenith Insurance was ordered to pay Fernandez,
actual damage on the car was only P3,640. Moral damages of P10,000 is a.   The amount of P3,640.00 representing the damage incurred plus
equitable. On the other hand, exemplary or corrective damages are imposed, interest at the rate of twice the prevailing interest rates;
by way of example or correction for the public good. The amount of P5,000.00 b.   The amount of P20,000.00 by way of moral damages;
awarded as attorney’s fees is justified under the circumstances. As regards the c.   The amount of P20,000.00 by way of exemplary damages;
actual damages incurred by private respondent, the amount of P3,640.00 had d.   The amount of P5,000.00 as attorney’s fees;
been established. Policy does not also mention the policy, e.   The amount of P3,000.00 as litigation expenses; and
any deductible franchise. f.   Costs.
5.   Upon motion of Fernandez and before the expiration of the period to appeal,
DOCTRINE: It is clear that under the Insurance Code, in case of unreasonable the trial court, ordered the execution of the decision pending appeal.
delay in the payment of the proceeds of an insurance policy, the damages that a.   The order was assailed by petitioner in a petition
may be awarded are: 1) attorney’s fees; 2) other expenses incurred by the for certiorari with the Court of Appeals but which petition was
insured person by reason of such unreasonable denial or also dismissed.
withholding of payment; 3) interest at twice the ceiling prescribed by the b.   Petitioner filed a notice of appeal before the trial court.
Monetary Board of the amount of the claim due the injured; and 4) the amount of
the claim. And under the new civil code, moral and exemplary damages can be
i.   The notice of appeal was granted in the same order a.   “SEC. 244. In case of any litigation for the enforcement of any
granting private respondent’s motion for execution policy or contract of insurance, it shall be the duty of the
pending appeal. Commissioner or the Court, as the case may be, to make a finding
as to whether the payment of the claim of the insured has
ISSUE: Whehter or not the award of moral and exemplary damages is been unreasonably denied or withheld; and in the affirmative case,
proper—NO. Moral damages cannot be awarded because there was no bad faith on the insurance company shall be adjudged to pay damages which
the part of Zenith in delaying the payment of the proceeds. shall consist of attorney’s fees and other expenses incurred by the
insured person by reason of such unreasonable denial or
RULING: ACCORDINGLY, the appealed decision is MODIFIED as above stated. withholding of payment plus interest of twice the ceiling prescribed
by the Monetary Board of the amount of the claim due the insured,
RATIO: from the date following the time prescribed in section two hundred
forty-two or in section two hundred forty-three, as the case may be,
1.   Court of Appeals rendered its decision affirming in toto the decision of the until the claim is fully satisfied; Provided, That the failure to pay
trial court. It also ruled that the matter of the trial court’s denial of any such claim within the time prescribed in said sections shall be
Fernandez’ right to adduce evidence is a closed matter in view of its (CA) considered prima facie evidence of unreasonable delay in
ruling in AC-G.R. 04644 wherein Zenith’s petition questioning the trial payment.”
court’s order submitting the case for decision without Zenith’s evidence,
was dismissed 5.   It is clear that under the Insurance Code, in case of unreasonable delay in
a.   The Motion for Reconsideration of the decision of the Court of the payment of the proceeds of an insurance policy, the damages that may
Appeals was denied for lack of merit. be awarded are: 1) attorney’s fees; 2) other expenses incurred by the
b.   Hence, the instant petition was filed by Zenith on the allegation insured person by reason of such unreasonable denial or
that respondent Court of Appeals’ decision and resolution ran withholding of payment; 3) interest at twice the ceiling prescribed by the
counter to applicable decisions of this Court and that they were Monetary Board of the amount of the claim due the injured; and 4) the
rendered without or in excess of jurisdiction. amount of the claim.

2.   Zenith insurance questions the legal basis of the CA in awarding moral 6.   As regards the award of moral and exemplary damages, the rules under
damages, exemplary damages and attorney’s fees in an amount more than the New Civil Code of the Philippines shall govern.
that prayed for in the complaint. a.   “The purpose of moral damages is essentially indemnity or
a.   The award of actual damages of P3,640.00 instead of only reparation, not punishment or correction. Moral damages are
P1,927.50 which was arrived at after deducting P250.00 emphatically not intended to enrich a complainant at the expense
and P274.00 as deductible franchise and 20% depreciation on parts of a defendant, they are awarded only to enable the injured party to
as agreed upon in the contract of insurance. obtain means, diversions or amusements that will serve to alleviate
b.   Petitioner contends that while the complaint of private respondent the moral suffering he has undergone by reason of the defendant’s
prayed for P10,000.00 moral damages, the lower court awarded culpable action.”
twice the amount, or P20,000.00 without factual or legal basis; b.   While it is true that no proof of pecuniary loss is necessary in order
while private respondent prayed for P5,000.00 exemplary that moral damages may be adjudicated, the assessment of which is
damages, the trial court awarded P20,000.00; and while private left to the discretion of the court according to the circumstances of
respondent prayed for P3,000.00 attorney’s fees, the trial court each case (Art. 2216, New Civil Code), it is equally true that in
awarded P5,000.00. awarding moral damages in case of breach of contract, there must
be a showing that the breach was wanton and deliberately injurious
3.   The propriety of the award of moral damages, exemplary damages and or the one responsible acted fraudulently or in bad faith
attorney’s fees is the main issue raised herein by Zenith.
7.   In the instant case, there was a finding that private respondent was given a
4.   The award of damages in case of unreasonable delay in the payment of “run-around” for two months, which is the basis for the award
insurance claims is governed by the Philippine Insurance Code, which of the damages granted under the Insurance Code for unreasonable delay in
provides: the payment of the claim.
a.   However, the act of petitioner Zenith of delaying payment for two b.   2) P10,000.00 as moral damages;
months cannot be considered as so wanton or malevolent to justify c.   3) P5,000.00 as attorney’s fees;
an award of P20,000.00 as moral damages, taking into d.   4) P3,000.00 as litigation expenses; and
consideration also the fact that the actual damage on the car was e.   4) Costs.
only P3,640.
b.   The reason for Zenith’s failure to indemnify private respondent
Fernandez within the two-month period was that the parties could
not come to an agreement as regards the amount of the actual
damage on the car.
c.   The amount of P10,000.00 prayed for by private respondent as
moral damages is equitable.

8.   On the other hand, exemplary or corrective damages are imposed, by way


of example or correction for the public good (Art. 2229, New Civil Code of
the Philippines).
a.   In the case of Noda v. Cruz-Arnaldo, exemplary damages were not
awarded at the insurance company had not acted in wanton,
oppressive or malevolent manner.
b.   The same is true in the case at bar.

9.   The amount of P5,000.00 awarded as attorney’s fees is justified


under the circumstances of this case considering that there were other
petitions filed and defended by private respondent in connection with this
case.

10.   As regards the actual damages incurred by private respondent, the amount
of P3,640.00 had been established before the trial court and affirmed by the
appellate court.

11.   Respondent appellate court correctly ruled that the deductions of P250.00
and P274.00 as deductible franchise and 20% depreciation on parts,
respectively claimed by petitioners as agreed upon in the contract, had no
basis.
a.   Respondent court CA ruled: “Under its second assigned error,
defendant-appellant puts forward two arguments, both of which are
entirely without merit. It is contented that the amount recoverable
under the insurance policy defendant-appellant issued over the
car of plaintiff-appellee is subject to deductible franchise, and “the
policy, does not mention any deductible franchise.”

12.   Therefore, the award of moral damages is reduced to P10,000.00 and the
award of exemplary damages is hereby deleted. The awards due to private
respondent Fernandez are as follows:
a.   1) P3,640.00 as actual claim plus interest of twice the ceiling
prescribed by the Monetary Board computed from the time of
submission of proof of loss;
010 STRONGHOLD INSURANCE v. INTERPACIFIC CONTAINER 37.   While the policy was in effect, the truck figured into an accident along the
SERVICES (SEE) National highway in Brgy. Palihan, Hermosa, Bataan resulting in 4 deaths
July 1, 2015 | Perez, J. | Insurance Contract Interpretation and 3 serious injuries. 2 vehicles were also heavily damages.
38.   Chong then filed a claim for the recovery of P550,000.
PETITIONER: Stronghold Insurance Company Inc. a.   Comprehensive Third Party Liability (CTPL) --------- ₱50,000.00
b.   Own Damage (OD) --------------------------------------- ₱300,000.00
RESPONDENTS: Interpacific Container Services and Gloria Dee Chong
c.   Excess I Bodily Injury (BI)------------------------------ ₱100,000.00
d.   Third Party Liability (TPL) ----------------------------- ₱100,000.00
SUMMARY: Chong owned a Fuso truck which was insured with Stronghold e.   Total -------------------------------------------------------- ₱550,000.00
Insurance under a comprehensive motor car insurance policy which undertook to 39.   Stronghold denied on the ground that the driver was heavily drunk at the
indemnify the insured against loss or damage to the car and death or injury time of the accident as shown in the Pagpapatunay issued by the Barangay
caused to third persons by reason of accident. While the policy was in effect, the Chairman Rafael Torres and the Medico-Legal Certificate signed by Dr.
truck figured into an accident along the National highway in Brgy. Palihan, Bautista.
Hermosa, Bataan resulting in 4 deaths and 3 serious injuries. 2 vehicles were 40.   Chong and Interpacific Services (I assumed this is Chong’s company) then
also heavily damages. Chong then filed a claim for the recovery of P550,000. sued for recovery of sum of money before the RTC of Caloocan. They
Stronghold denied on the ground that the driver was heavily drunk at the time of argued that there was no sufficient proof to support the claim of the
the accident as shown in the Pagpapatunay issued by the Barangay Chairman Stronghold that the driver was drunk at the time of the incident
and the Medico-Legal Certificate. Chong and Interpacific Services then sued for underscoring the lack of mention of such crucial fact in the police blotter
recovery of sum of money before the RTC of Caloocan. The RTC and the CA report documenting the incident.
ruled for Chong and ordered Stronghold to pay. The issue in this case is WoN it 41.   Stronghold averred hat the intoxication of the driver of the insured vehicle
was proven during the trial that the driver of the insured vehicle was intoxicated legally avoided the liability of the insurance company under the policy.
at the time of the accident thereby precluding Chong and Interpacific from Stronghold further claimed that the insured violated Section 53 of the Land
claiming the proceeds of the insurance policy-NO, the evidences provided by Transportation and Traffic Code which prohibits driving of motor vehicles
Stronghold failed to prove intoxication. The RTC doubted the authenticity of the under the influence of alcohol. Since the driver of the insured vehicle was
Medico Legal Certificate because of the attendant alteration and tampering on found drunk at the time of the accident, the denial of the insurance claim of
the face of the document. The evident tampering of the medico legal certificate Chong is therefore justified under provisions of the insurance contract and
necessitated the presentation by Stronghold of additional evidence to buttress his the existing statutes.
claim but it did not present any other evidence. What further dampens 42.   The RTC ruled for Chong and Interpacific and ordered Stronghold to pay
Stronghold's position is the absence of the crucial fact of intoxication in the them. The CA affirmed but deleted the award for exemplary damages.
blotter report which officially documented the incident.
ISSUE/s:
DOCTRINE: This case involves a contract of insurance, the authenticity and 8.   WoN it was proven during the trial that the driver of the insured vehicle was
validity of which was uncontested. In exempting insurers from liability under intoxicated at the time of the accident thereby precluding Chong and
the contract, proof thereof must be clear, credible and convincing. Interpacific from claiming the proceeds of the insurance policy—NO, the
Fundamental is the rule that the contract is the law between the parties and, that evidences provided by Stronghold failed to prove intoxication.
absent any showing that its provisions are wholly or in part contrary to law,
morals, good customs, public order, or public policy, it shall be enforced to the RULING: WHEREFORE, premises considered, the instant petition is hereby
letter by the courts. DENIED. The assailed Decision of the Court of Appeals in CA-G.R. CV No. 80557
is hereby AFFIRMED.
FACTS:
36.   Gloria Dee Chong owned a Fuso truck with Plate number PWH 512. The RATIO:
truch was insured with Stronghold Insurance under a comprehensive motor 1.   Stronghold claims that the factual findings of the lower courts were fallible.
car insurance policy which undertook to indemnify the insured against loss However, the Supreme Court is not a trier of facs and as a rule, the factual
or damage to the car and death or injury caused to third persons by reason conclusion of the lower courts are rcognized by the Supreme Court, unless
of accident. of some facts and circumstances of weight and substance, having been
overlooked or misinterpreted, might materially affect the disposition of the
case. The exception has not been shown.
2.   Contrary to the claim of Stronghold; it miserably failed to prove the fact of
intoxication during the trial. Aside from the Medico Legal Certificate and
the Pagpapatunay, which were stripped of evidentiary value because of the
dubious circumstances under which they were obtained, the Stronghold did
not adduce other proof to justify the avoidance of the policy.
3.   The RTC doubted the authenticity of the Medico Legal Certificate because
of the attendant alteration and tampering on the face of the document.
In adopting the findings of the trial court, the appellate court reiterated the
evidentiary rule that the party alleging violation of the provision of the
contract bears the burden of proof to prove the same.
4.   The evident tampering of the medico legal certificate necessitated the
presentation by Stronghold of additional evidence to buttress his
claim. For instance, Stronghold could have adduced affidavits of witnesses
who were present at the scene of the accident to attest to the fact that the
driver was intoxicated. It did not. Upon the other hand, Chong and
Interpacific duly established their right to claim the proceeds of a validly
subsisting contract of insurance. Such contract was never denied.
5.   In civil cases, the party having the burden of proof must produce a
preponderance of evidence thereon, with plaintiff having to rely on the
strength of his own evidence and not upon the weakness of the defendant's.
The concept of "preponderance of evidence" refers to evidence which is of
greater weight or more convincing, than that which is offered in opposition
to it; at bottom, it means probability of truth.
6.   What further dampens Stronghold's position is the absence of the
crucial fact of intoxication in the blotter report which officially
documented the incident. Entries in police records made by a police
officer in the performance of the duty especially enjoined by law are prima
facie evidence of the fact therein stated, and their probative value may be
substantiated or nullified by other competent evidence. In this case, the lack
of statement to the effect that the driver was under the influence of alcohol
in the said report is too significant to escape the attention of this Court.
7.   This case involves a contract of insurance, the authenticity and validity of
which was uncontested. In exempting insurers from liability under the
contract, proof thereof must be clear, credible and convincing.
Fundamental is the rule that the contract is the law between the parties and,
that absent any showing that its provisions are wholly or in part contrary to
law, morals, good customs, public order, or public policy, it shall be
enforced to the letter by the courts.
White Gold Marine Services, Inc. vs. Pioneer Insurance and Surety Association (Bermuda) Limited (Steamship Mutual) through Pioneer Insurance
Corporation (Siapno) and Surety Corporation (Pioneer).
July 28, 2005 | Quisumbing, J. | Doing Insurance Business 2.   White Gold was issued a Certificate of Entry and Acceptance. Pioneer also issued
receipts evidencing payments for the coverage. When White Gold failed to fully
PETITIONER: White Gold Marine Services, Inc. pay its accounts, Steamship Mutual refused to renew the coverage.
RESPONDENT: Pioneer Insurance and Surety Corporation and the Steamship 3.   Steamship Mutual thereafter filed a case against White Gold for collection of sum
Mutual Underwriting Association (Bermuda) LTD. of money to recover the latter’s unpaid balance. White Gold on the other hand,
filed a complaint before the Insurance Commission claiming that Steamship
SUMMARY: White Gold Marine Services, Inc. (White Gold) procured a Mutual violated Sections 186 and 187 of the Insurance Code, while Pioneer
protection and indemnity coverage for its vessels from The Steamship Mutual violated Sections 299, 300 and 301 in relation to Sections 302 and 303, thereof
Underwriting Association (Bermuda) Limited (Steamship Mutual) through (see end of the digest, but these provisions talk about the requirements of
Pioneer Insurance and Surety Corporation (Pioneer). When White Gold failed to transacting insurance business in the Philippines)
fully pay its accounts, Steamship Mutual refused to renew the coverage. 4.   Insurance Commission: dismissed the complaint since was no need for Steamship
Steamship Mutual thereafter filed a case against White Gold for collection of Mutual to secure a license because it was not engaged in the insurance business.
sum of money to recover the latter’s unpaid balance. White Gold on the other It explained that Steamship Mutual was a Protection and Indemnity Club (P & I
hand, filed a complaint before the Insurance Commission claiming that Club). Likewise, Pioneer need not obtain another license as insurance agent
Steamship Mutual failed to secure the license needed for transacting insurance and/or a broker for Steamship Mutual because Steamship Mutual was not
businesses in the Philippines. Steamship Mutual claimed to be a Protection and engaged in the insurance business. Moreover, Pioneer was already licensed,
Indemnity Club and Pioneer claimed that it need not obtain another license as hence, a separate license solely as agent/broker of Steamship Mutual was already
insurance agent and/or a broker for Steamship Mutual because Steamship superfluous.
Mutual was not engaged in the insurance business. Issues are: 5.   CA: affirmed the decision of the Insurance Commissioner. In its decision, the
1.   WoN Steamship Mutual, a P & I Club, engaged in the insurance business in appellate court distinguished between P & I Clubs vis--vis conventional
the Philippines – YES The test to determine if a contract is an insurance insurance. The appellate court also held that Pioneer merely acted as a collection
contract or not, depends on the nature of the promise, the act required to be agent of Steamship Mutual.
performed, and the exact nature of the agreement in the light of the
occurrence, contingency, or circumstances under which the performance ISSUE:
becomes requisite. It is not by what it is called. A P & I Club is a form of 1.   WoN Steamship Mutual, a P & I Club, engaged in the insurance business in
insurance against third party liability, where the third party is anyone other the Philippines – YES, A P & I Club is a form of insurance against third party
than the P & I Club and the members. By definition then, Steamship Mutual liability, where the third party is anyone other than the P & I Club and the
as a P & I Club is a mutual insurance association engaged in the marine members. By definition then, Steamship Mutual as a P & I Club is a mutual
insurance business. insurance association engaged in the marine insurance business.
2.   WoN Pioneer needs a license as an insurance agent/broker for Steamship 2.   WoN Pioneer needs a license as an insurance agent/broker for Steamship
Mutual – YES. Although Pioneer is already licensed as an insurance Mutual – YES, Although Pioneer is already licensed as an insurance company, it
company, it needs a separate license to act as insurance agent for Steamship needs a separate license to act as insurance agent for Steamship Mutual as per
Mutual as per Section 299 of the Insurance Code. Section 299 of the Insurance Code

DOCTRINE: The test to determine if a contract is an insurance contract or not, RULING: The Steamship Mutual Underwriting Association (Bermuda) Ltd., and
depends on the nature of the promise, the act required to be performed, and the Pioneer Insurance and Surety Corporation are ORDERED to obtain licenses and to
exact nature of the agreement in the light of the occurrence, contingency, or secure proper authorizations to do business as insurer and insurance agent,
circumstances under which the performance becomes requisite. It is not by what respectively. The petitioners prayer for the revocation of Pioneers Certificate of
it is called. Authority and removal of its directors and officers, is DENIED. Costs against
respondents.
FACTS:
RATIO:
1.   White Gold Marine Services, Inc. (White Gold) procured a protection and
3.   The parties admit that Steamship Mutual is a P & I Club. Steamship Mutual
indemnity coverage for its vessels from The Steamship Mutual Underwriting
admits it does not have a license to do business in the Philippines although
Pioneer is its resident agent. This relationship is reflected in the certifications 13.  A P & I Club is a form of insurance against third party liability, where the
issued by the Insurance Commission. third party is anyone other than the P & I Club and the members. By
4.   Hyopsung Maritime Co., Ltd. v. CA: P & I Club is an association composed of definition then, Steamship Mutual as a P & I Club is a mutual insurance
shipowners in general who band together for the specific purpose of providing association engaged in the marine insurance business.
insurance cover on a mutual basis against liabilities incidental to shipowning that 14.  The records reveal Steamship Mutual is doing business in the country albeit
the members incur in favor of third parties. without the requisite certificate of authority mandated by Section 187 of the
5.   It stresses that as a P & I Club, Steamship Mutuals primary purpose is to solicit Insurance Code. It maintains a resident agent in the Philippines to solicit
and provide protection and indemnity coverage and for this purpose, it has insurance and to collect payments in its behalf. We note that Steamship Mutual
engaged the services of Pioneer to act as its agent. even renewed its P & I Club cover until it was cancelled due to non-payment of
6.   Pioneer and Steamship Mutual contend that although Steamship Mutual is a P & the calls. Thus, to continue doing business here, Steamship Mutual or
I Club, it is not engaged in the insurance business in the Philippines. It is merely through its agent Pioneer, must secure a license from the Insurance
an association of vessel owners who have come together to provide mutual Commission.
protection against liabilities incidental to shipowning. 15.  Since a contract of insurance involves public interest, regulation by the State
7.   Section 2(2) of the Insurance Code enumerates what constitutes doing an is necessary. Thus, no insurer or insurance company is allowed to engage in the
insurance business or transacting an insurance business. These are: insurance business without a license or a certificate of authority from the
(a) making or proposing to make, as insurer, any insurance contract; Insurance Commission.
(b) making, or proposing to make, as surety, any contract of suretyship as a vocation 16.  Pioneer is the resident agent of Steamship Mutual as evidenced by the certificate
and not as merely incidental to any other legitimate business or activity of the of registration issued by the Insurance Commission. It has been licensed to do or
surety; transact insurance business by virtue of the certificate of authority issued by the
(c) doing any kind of business, including a reinsurance business, specifically same agency. However, a Certification from the Commission states that
recognized as constituting the doing of an insurance business within the meaning Pioneer does not have a separate license to be an agent/broker of Steamship
of this Code; Mutual.
(d) doing or proposing to do any business in substance equivalent to any of the 17.  Although Pioneer is already licensed as an insurance company, it needs a
foregoing in a manner designed to evade the provisions of this Code. separate license to act as insurance agent for Steamship Mutual. Section 299
8.   The same provision also provides, the fact that no profit is derived from the of the Insurance Code clearly states:
making of insurance contracts, agreements or transactions, or that no separate or No person shall act as an insurance agent or as an insurance broker in the solicitation
direct consideration is received therefor, shall not preclude the existence of an or procurement of applications for insurance, or receive for services in obtaining
insurance business. insurance, any commission or other compensation from any insurance company
9.   The test to determine if a contract is an insurance contract or not, depends doing business in the Philippines or any agent thereof, without first procuring a
on the nature of the promise, the act required to be performed, and the exact license so to act from the Commissioner, which must be renewed annually on
nature of the agreement in the light of the occurrence, contingency, or the first day of January, or within six months thereafter.
circumstances under which the performance becomes requisite. It is not by 18.  Finally, White Gold seeks revocation of Pioneers certificate of authority and
what it is called. removal of its directors and officers. Regrettably, we are not the forum for these
10.  An insurance contract is a contract of indemnity. In it, one undertakes for a issues.
consideration to indemnify another against loss, damage or liability arising from
an unknown or contingent event. RELATED PROVISIONS:
SEC. 186. No person, partnership, or association of persons shall transact any insurance business in the
11.  In particular, a marine insurance undertakes to indemnify the assured against
Philippines except as agent of a person or corporation authorized to do the business of insurance in the
marine losses, such as the losses incident to a marine adventure. Section 99 of the Philippines, unless possessed of the capital and assets required of an insurance corporation doing the same
Insurance Code enumerates the coverage of marine insurance. kind of business in the Philippines and invested in the same manner; nor unless the Commissioner shall
12.  Relatedly, a mutual insurance company is a cooperative enterprise where the have granted to him or them a certificate to the effect that he or they have complied with all the provisions
of law which an insurance corporation doing business in the Philippines is required to observe.
members are both the insurer and insured. In it, the members all contribute, by a
Every person, partnership, or association receiving any such certificate of authority shall be subject to the
system of premiums or assessments, to the creation of a fund from which all insurance laws of the Philippines and to the jurisdiction and supervision of the Commissioner in the same
losses and liabilities are paid, and where the profits are divided among manner as if an insurance corporation authorized by the laws of the Philippines to engage in the business
themselves, in proportion to their interest. Additionally, mutual insurance of insurance specified in the certificate.
associations, or clubs, provide three types of coverage, namely, protection and SEC. 187. No Insurance Company shall transact any insurance business in the Philippines until after it
shall have obtained a certificate of authority for that purpose from the Commissioner upon application
indemnity, war risks, and defense costs. therefor and payment by the company concerned of the fees hereinafter prescribed.
SEC. 299. No insurance company doing business in the Philippines, nor any agent thereof, shall pay any
commission or other compensation to any person for services in obtaining insurance, unless such person
shall have first procured from the Commissioner a license to act as an insurance agent of such company or
as an insurance broker as hereinafter provided.
No person shall act as an insurance agent or as an insurance broker in the solicitation or procurement of
applications for insurance, or receive for services in obtaining insurance, any commission or other
compensation from any insurance company doing business in the Philippines or any agent thereof, without
first procuring a license so to act from the Commissioner, . . .
SEC. 300. Any person who for compensation solicits or obtains insurance on behalf of any insurance
company or transmits for a person other than himself an application for a policy or contract of insurance to
or from such company or offers or assumes to act in the negotiating of such insurance shall be an
insurance agent within the intent of this section and shall thereby become liable to all the duties,
requirements, liabilities and penalties to which an insurance agent is subject.
SEC. 301. Any person who for any compensation, commission or other thing of value acts or aids in any
manner in soliciting, negotiating or procuring the making of any insurance contract or in placing risk or
taking out insurance, on behalf of an insured other than himself, shall be an insurance broker within the
intent of this Code, and shall thereby become liable to all the duties, requirements, liabilities and penalties
to which an insurance broker is subject.
SEC. 99. Marine insurance includes:
(1) Insurance against loss of or damage to:
(a) Vessels, craft, aircraft, vehicles, goods, freights, cargoes, merchandise, effects, disbursements, profits,
moneys, securities, choses in action, evidences of debt, valuable papers, bottomry, and respondentia
interests and all other kinds of property and interests therein, in respect to, appertaining to or in connection
with any and all risks or perils of navigation, transit or transportation, or while being assembled, packed,
crated, baled, compressed or similarly prepared for shipment or while awaiting shipment, or during any
delays, storage, trasshipment, or reshipment incident thereto, including war risks, marine builders risks,
and all personal property floater risks.
(b) Person or property in connection with or appertaining to a marine, inland marine, transit or
transportation insurance, including liability for loss of or damage arising out of or in connection with the
construction, repair, operation, maintenance or use of the subject matter of such insurance (but not
including life insurance or surety bonds nor insurance against loss by reason of bodily injury to any person
arising out of the ownership, maintenance, or use of automobiles).
(c) Precious stones, jewels, jewelry, precious metals, whether in course of transportation or otherwise.
(d) Bridges, tunnels and other instrumentalities of transportation and communication (excluding buildings,
their furniture and furnishings, fixed contents and supplies held in storage); piers, wharves, docks and
slips, and other aids to navigation and transportation, including dry docks and marine railways, dams and
appurtenant facilities for the control of waterways.
(2) Marine protection and indemnity insurance, meaning insurance against, or against legal liability of the
insured for loss, damage, or expense incident to ownership, operation, chartering, maintenance, use,
repair, or construction of any vessel, craft or instrumentality in use in ocean or inland waterways,
including liability of the insured for personal injury, illness or death or for loss of or damage to the
property of another person.
001 GUINGON vs. DEL MONTE (SOLCO) accident caused by or arising out of the use of the Motor Vehicle/s or in
August 17, 1967 | BENGZON, J.P., J. | No Action Clause in Insurance Contracts connection with the loading or unloading of the Motor Vehicle/s, against all
sums including claimant's costs and expenses which the Insured shall
PETITIONER: DIONISIA GUINGON, et al. become legally liable to pay in respect of: (a.) death of or bodily injury to
RESPONDENTS: ILUMINADO DEL MONTE, et al. any person and (b.) damage to property.”
3.   Iluminado del Monte, one of the drivers of the jeepneys, while driving
SUMMARY: Julio Aguilar owned several Jeepneys which he insured with Capital along the intersection of Juan Luna and Moro streets in Manila, bumped
Insurance & Surety Co,. Inc. (CISC). The policy states that it will reimburse Aguilar, with the jeepney of Gervacio Guingon who had just alighted from another
subject to the limits of his liability, for all accidents caused by the use of the motor jeepney and as a consequence the latter died some days thereafter. A
vehicles covered against all sums including claimant's costs and expenses which corresponding information for homicide thru reckless imprudence was filed
the Insured shall become legally liable to pay in respect of: (a.) death of or bodily against del Monte, who pleaded guilty. A penalty of four months
injury to any person and (b.) damage to property. His jeepney driver, Iluminado imprisonment was imposed on him.
bumped another jeepney while Gervacio, a passenger, was alighting from it, which 4.   Corollary to the criminal action, the heirs of Gervacio Guingon filed an
caused Gervacio to die. So an action for homicide thru reckless imprudence was filed action for damages against Julio Aguilar, and the Capital Insurance &
against him for which he was convicted. Corollary to the action, the heirs filed an Surety Co., Inc. Julio Aguilar did not answer and was thus declared in
action for damages against Aguilar and CISC. CFI held both of them liable. CISC default.
appealed to SC stating that it isn’t liable for the death of Gervacio and also it cannot 5.   Capital Insurance & Surety Co., Inc answered, alleging that the plaintiff has
be impleaded in the case against Aguilar because of a certain “no action” clause in no cause of action against it.
their insurance contract which requires that a final judgement be rendered first 6.   CFI ordered Del Monte and Aguilar jointly and severally to pay P8,572.95
against the insured before they can be sued for the proceeds. SC ruled against CISC to plaintiffs as damages for the death of their father and P1,000 as
and stated that it is clear from the provisions of your contract with Aguilar that you attorney’s fees.
were insuring him against all damages which he may become legally liable for 7.   Capital Insurance and Surety Co., Inc. was also sentenced to pay the
arising out of any accident caused by his jeepneys which causes death or injury to a plaintiffs P5,000.00 and P500.00 as attorney's fees and costs which shall be
person. Secondly, even if the “no action” clause in the insurance contract states that applied in partial satisfaction of the judgment rendered against Iluminado
you cannot be sued or impleaded until a final judgement is rendered against against del Monte and Julio Aguilar in this case.
the insured, such clause cannot prevail against the Rules of Court which allows 8.   Hence this appeal by Capital Insurance & Surety Co., Inc.
permissive joinder of parties and was intended to avoid multiplicity of suits. Sec. 5
of Rule 2 on "Joinder of causes of action" and Sec. 6 of Rule 3 on "Permissive ISSUE/s:
joinder of parties" cannot be superseded, at least with respect to third persons not a 1.   Whether plaintiff can sue Capital Insurance & Surety Co., Inc.? If so, can
party to the contract, as herein, by a "no action" clause in the contract of insurance. plaintiffs sue the insurer jointly with the insured? YES to both, Firstly, the
insurance contract expressly states that it covers said liability.
DOCTRINE: 1. The "no action" clause in the policy of insurance cannot prevail Secondly, the “no action” clause in the contract cannot prevail over the
over the Rules of Court provision aimed at avoiding multiplicity of suits Rules of Court as provided for by jurisprudence.

2.  The right of the person injured to sue the insurer of the party at fault (insured), RULING: Wherefore, the judgment appealed from is affirmed in toto. Costs against
depends on whether the contract of insurance is intended to benefit third persons also appellant. So ordered.
or only the insured. The test to be applied is: Where the contract provides for
indemnity against liability to third persons, then third persons to whom the insured is RATIO:
liable,can sue the insurer. 1.   The right of the person injured to sue the insurer of the party at fault
(insured), depends on whether the contract of insurance is intended to
benefit third persons also or only the insured. The test to be applied is:
FACTS: Where the contract provides for indemnity against liability to third persons,
1.   In 1961, Julio Aguilar owned and operated several jeepneys in Manila. He then third persons to whom the insured is liable, can sue the insurer. Where
entered into a contract with the Capital Insurance &Surety Co., Inc. insuring the contract is for indemnity against actual loss or payment, then third
the operation of his jeepneys against accidents with third-party liability. persons cannot proceed against the insurer, the contract being solely to
2.   The insurance policy provides that “Capital Insurance & Surety Co., Inc. reimburse the insured for liability actually discharged by him thru payment
will, subject to the limits of liability, indemnify the Insured in the event of
to third persons, said third persons' recourse being thus limited to the
insured alone.
2.   The policy in the present case is one whereby the insurer agreed to
indemnify the insured "against all sums which the Insured shall
become legally liable to pay in respect of: a. death of or bodily injury to
any person." Clearly, therefore, it is one for indemnity against liability;
from the fact then that the insured is liable to the third person, such
third person is entitled to sue the insurer
3.   Appellant contends that the "no action" clause in the policy closes the
avenue to any third party which may be injured in an accident wherein the
jeepney of the insured might have been the cause of the injury of third
persons, alleging the freedom of contracts.

Action Against Company Clause:

1.   “No action shall lie against the Company unless, as a condition


precedent thereto, the Insured shall have fully complied with all of the
terms of this Policy, nor until the amount of the Insured's obligation to
pay shall have been finally determined either by judgment against the
Insured after actual trial or by written agreement of the Insured, the
claimant, and the Company.”
2.   “Any person or organization or the legal representative thereof who has
secured such judgment or written agreement shall thereafter be entitled to
recover under this policy to the extent of the insurance afforded by the
Policy. Nothing contained in this policy shall give any person or
organization any right to join the Company as a co-defendant in any
action against the Insured to determine the Insured's liability.”
3.   The "no action" clause in the policy of insurance cannot prevail over the
Rules of Court provision aimed at avoiding multiplicity of suits. In a case
squarely on the point, American Automobile Ins. Co. vs. Struwe, 218 SW
534 (Texas CCA), it was held that a "no action" clause in a policy of
insurance cannot override procedural rules aimed at avoidance of
multiplicity of suits.
4.   Similarly, in the instant suit, Sec. 5 of Rule 2 on "Joinder of causes of
action" and Sec. 6 of Rule 3 on "Permissive joinder of parties" cannot be
superseded, at least with respect to third persons not a party to the contract,
as herein, by a "no action" clause in the contract of insurance.
002 ETERNAL GARDENS MPC. v. PHILAM (STA. MARIA) person is insured as soon as he contracts a loan with Eternal, it cannot be then shown that
April 9, 2008| Velasco, Jr., J. | Covered by Insurance Philam would first need to approve his application for it to be valid; group life insurance
covers all those who contracted with Eternal, with or without the approval.
PETITIONERS: Eternal Gardens Memorial Park Corporation
RESPONDENTS: The Philippine American Life Insurance Company FACTS:
1.   Philamlife entered into an agreement denominated as Creditor Group Life
SUMMARY: Philamlife entered into an agreement denominated as Creditor Group Life Policy with petitioner Eternal Gardens Memorial Park Corporation
Policy with Eternal. Under the policy, the clients of Eternal who purchased burial lots
(Eternal).
from it on installment basis would be insured by Philamlife. The amount of insurance
coverage depended upon the existing balance of the purchased burial lots. The most 2.   Under the policy, the clients of Eternal who purchased burial lots from it on
relevan provision of the policy is EFFECTIVE DATE OF BENEFIT. The insurance of installment basis would be insured by Philamlife. The amount of insurance
any eligible Lot Purchaser shall be effective on the date he contracts a loan with the coverage depended upon the existing balance of the purchased burial lots.
Assured. However, there shall be no insurance if the application of the Lot Purchaser is The policy was to be effective for a period of one year, renewable on a
not approved by the Company.Eternal was required under the policy to submit to yearly basis.
Philamlife a list of all new lot purchasers, together with a copy of the application of each 3.   The relevant provisions8 of the policy are: (MOST RELEVANT)
purchaser, and the amounts of the respective unpaid balances of all insured lot purchasers. EFFECTIVE DATE OF BENEFIT. The insurance of any eligible Lot
Eternal complied by submitting a letter containing a list of insurable balances of its lot Purchaser shall be effective on the date he contracts a loan with the
buyers. One of those included in the list as “new business” John Chuang. His balance of
Assured. However, there shall be no insurance if the application of the Lot
payments was 100K. Later on, Chuang died. Eternal sent a letter dated to Philamlife,
which served as an insurance claim for Chuang’s death. Attached to the claim were Purchaser is not approved by the Company.
certain documents. In reply, Philamlife wrote Eternal a letter requiring Eternal to submit 4.   Eternal was required under the policy to submit to Philamlife a list of all
the additional documents relative to its insurance claim for Chuang’s death. Eternal new lot purchasers, together with a copy of the application of each
transmitted the required documents through a letter which was received by purchaser, and the amounts of the respective unpaid balances of all insured
Philamlife.After more than a year, Philamlife had not furnished Eternal with any reply to lot purchasers.
the latter’s insurance claim. This prompted Eternal to demand from Philamlife the 5.   Eternal complied by submitting a letter containing a list of insurable
payment of the claim for PhP 100,000. Philamlife denied Eternal’s insurance claim as balances of its lot buyers. One of those included in the list as "new
there was no application for Group Insurance was submitted in our office prior to business" was John Chuang. His balance of payments was PhP 100,000. On
Chuang’s death and hence, this was not covered by the insurance. Eternal filed a case
August 2, 1984, Chuang died.
with the RTC for a sum of money against Philamlife, which decided in favor of Eternal,
ordering Philamlife to pay the former 100K representing the proceeds of the policy. CA 6.   Eternal sent a letter to Philamlife, which served as an insurance claim for
reversed. The issue in this case is WON Chuang’s death was covered by the insurance Chuang’s death. Attached to the claim were the following documents: (1)
policy?– Yes. An examination of the provision of the policy under effective date of Chuang’s Certificate of Death; (2) Identification Certificate stating that
benefit, would show ambiguity between its two sentences. The first sentence appears to Chuang is a naturalized Filipino Citizen; (3) Certificate of Claimant; (4)
state that the insurance coverage of the clients of Eternal already became effective upon Certificate of Attending Physician; and (5) Assured’s Certificate.
contracting a loan with Eternal while the second sentence appears to require Philamlife to 7.   Philamlife wrote Eternal a letter requiring Eternal to submit the following
approve the insurance contract before the same can become effective. The SC ruled that documents relative to its insurance claim for Chuang’s death: (1) Certificate
since, an insurance contract is a contract of adhesion this must be construed liberally in of Claimant (with form attached); (2) Assured’s Certificate (with form
favor of the insured and strictly against the insurer. The SC harmonized the sentences to
mean that upon a party’s purchase of a memorial lot on installment from Eternal, an                                                                                                                        
8
insurance contract covering the lot purchaser is created and the same is effective, valid,  ELIGIBILITY.  Any  Lot  Purchaser  of  the  Assured  who  is  at  least  18  but  not  more  than  65  years  of  age,  is  
and binding until terminated by Philamlife by disapproving the insurance application. indebted   to   the   Assured   for   the   unpaid   balance   of   his   loan   with   the   Assured,   and   is   accepted   for   Life  
(Applied to this case, the purchase of the burial lot on installment basis by Chuang made Insurance  coverage  by  the  Company  on  its  effective  date  is  eligible  for  insurance  under  the  Policy.  
his death part of Philamlife’s insurance coverage.) EVIDENCE   OF   INSURABILITY.No   medical   examination   shall   be   required   for   amounts   of   insurance   up   to  
P50,000.00.   However,   a   declaration   of   good   health   shall   be   required   for   all   Lot   Purchasers   as   part   of   the  
application.  The  Company  reserves  the  right  to  require  further  evidence  of  insurability  satisfactory  to  the  
DOCTRINE: (No specific doctrine with respect to the syllabus topic which is what may Company  in  respect  of  the  following:  (1).  Any  amount  of  insurance  in  excess  of  P50,000.00.  (2).  Any  lot  
be insured/insured against? – this can be seen in the application of the facts but the court purchaser  who  is  more  than  55  years  of  age.  
used the rule on interpretation of a contract of adhesion in order to determine the coverage LIFE   INSURANCE   BENEFIT.   The   Life   Insurance   coverage   of   any   Lot   Purchaser   at   any   time   shall   be   the  
of the insurance policy) An insurance contract is a contract of adhesion which must be amount   of   the   unpaid   balance   of   his   loan   (including   arrears   up   to   but   not   exceeding   2   months)   as  
construed liberally in favor of the insured and strictly against the insurer in order to reported  by  the  Assured  to  the  Company  or  the  sum  of  P100,000.00,  whichever  is  smaller.  Such  benefit  
safeguard the latter’s interest. (As an application to the facts) When the contract states the shall  be  paid  to  the  Assured  if  the  Lot  Purchaser  dies  while  insured  under  the  Policy.  
 
attached); (3) Application for Insurance accomplished and signed by the memorial lot on installment from Eternal, an insurance contract covering
insured, Chuang, while still living; and (4) Statement of Account showing the lot purchaser is created and the same is effective, valid, and binding
the unpaid balance of Chuang before his death. until terminated by Philamlife by disapproving the insurance application.
8.   Eternal transmitted the required documents through a letter which was (Applied to this case, the purchase of the burial lot on installment basis by
received by Philamlife. Chuang made his death part of Philamlife’s insurance coverage.)
9.   After more than a year, Philamlife had not furnished Eternal with any reply
to the latter’s insurance claim. This prompted Eternal to demand from RULING: WHEREFORE, we GRANT the petition. The November 26, 2004 CA Decision
Philamlife the payment of the claim for PhP 100,000. in CA-G.R. CV No. 57810 is REVERSED and SET ASIDE. The May 29, 1996 Decision of
10.   In response to Eternal’s demand, Philamlife denied Eternal’s insurance the Makati City RTC, Branch 138 is MODIFIED. Philamlife is hereby ORDERED:
claim in a letter a portion of which reads: (1) To pay Eternal the amount of PhP 100,000 representing the proceeds of the Life Insurance
Policy of Chuang; (2) To pay Eternal legal interest at the rate of six percent (6%) per annum of
PhP 100,000 from the time of extra-judicial demand by Eternal until Philamlife’s receipt of
The deceased was 59 years old when he entered into Contract #9558 and 9529 with Eternal the May 29, 1996 RTC Decision on June 17, 1996; (3) To pay Eternal legal interest at the rate
Gardens Memorial Park in October 1982 for the total maximum insurable amount of of twelve percent (12%) per annum of PhP 100,000 from June 17, 1996 until full payment of
P100,000.00 each. No application for Group Insurance was submitted in our office prior this award; and (4) To pay Eternal attorney’s fees in the amount of PhP 10,000.
to his death. In accordance with our Creditor’s Group Life, under Evidence of Insurability
provision, "a declaration of good health shall be required for all Lot Purchasers as party of the RATIO:
application." We cite further the provision on Effective Date of Coverage under the policy
1.   As a general rule, this Court is not a trier of facts and will not re-examine
which states that "there shall be no insurance if the application is not approved by the
Company." Since no application had been submitted by the Insured/Assured, prior to his factual issues raised before the CA and first level courts, considering their
death, for our approval but was submitted instead after his death, Mr. John Uy Chuang findings of facts are conclusive and binding on this Court. However, such
was not covered under the Policy. We wish to point out that Eternal Gardens being the rule is subject to exceptions. One exception is when the findings of the CA
Assured was a party to the Contract and was therefore aware of these pertinent provisions. xxx are contrary to the trial court which exception is present in the instant case.
Thus, the SC can review the factual findings.
11.   Eternal filed a case before the RTC for a sum of money against Philamlife. 2.   Eternal claims that the evidence that it presented before the trial court
The RTC ruled in favor of Eternal, ordering Philamlife to pay P100,000 supports its contention that it submitted a copy of the insurance application
representing the proceeds of the Policy of John Uy Chuang. of Chuang before his death. In Eternal’s letter, a list of insurable interests of
12.   The RTC found that Eternal submitted Chuang’s application for insurance buyers for October 1982 was attached, including Chuang in the list of new
which he accomplished before his death, as testified to by Eternal’s witness businesses. Eternal added it was noted at the bottom of said letter that the
and evidenced by a lettr. It further ruled that due to Philamlife’s inaction corresponding "Phil-Am Life Insurance Application Forms & Cert." were
from the submission of the requirements of the group insurance to Chuang’s enclosed in the letter that was apparently received by Philamlife. Finally,
death, as well as Philamlife’s acceptance of the premiums during the same Eternal alleged that it provided a copy of the insurance application which
period, Philamlife was deemed to have approved Chuang’s application. The was signed by Chuang himself and executed before his death.
RTC said that since the contract is a group life insurance, once proof of 3.   Philamlife claims that the evidence presented by Eternal is insufficient,
death is submitted, payment must follow. arguing that Eternal must present evidence showing that Philamlife received
13.   Philamlife appealed to the CA which reversed the RTC’s order and the a copy of Chuang’s insurance application.
complaint was dismissed. It based its Decision on the factual finding that 4.   The evidence on record supports Eternal’s position.
Chuang’s application was not enclosed in Eternal’s letter. Thus, the CA 5.   The fact of the matter is, the letter, which Philamlife stamped as received,
concluded, there being no application form, Chuang was not covered by states that the insurance forms for the attached list of burial lot buyers were
Philamlife’s insurance. attached to the letter. Such stamp of receipt has the effect of acknowledging
receipt of the letter together with the attachments. Such receipt is an
ISSUE: admission by Philamlife against its own interest. The burden of evidence
1.   WON Chuang’s death was covered by the insurance policy? – Yes. An has shifted to Philamlife, which must prove that the letter did not contain
examination of the provision of the policy under effective date of benefit, Chuang’s insurance application. However, Philamlife failed to do so; thus,
would show ambiguity between its two sentences. The SC ruled that since, Philamlife is deemed to have received Chuang’s insurance application.
an insurance contract is a contract of adhesion this must be construed 6.   To reiterate, it was Philamlife’s bounden duty to make sure that before a
liberally in favor of the insured and strictly against the insurer. The SC transmittal letter is stamped as received, the contents of the letter are correct
harmonized the sentences to mean that upon a party’s purchase of a and accounted for.
7.   Philamlife’s allegation that Eternal’s witnesses ran out of credibility and it should be construed liberally in favor of the insured and strictly against
reliability due to inconsistencies is groundless. the insurer.
8.   An examination of the testimonies of the witnesses mentioned by 17.   The vague contractual provision, in Creditor Group Life Policy must
Philamlife, however, reveals no overlooked facts of substance and value. be construed in favor of the insured and in favor of the effectivity of the
9.   Philamlife primarily claims that Eternal did not even know where the insurance contract.
original insurance application of Chuang was, as shown by the testimony of 18.   On the other hand, the seemingly conflicting provisions must be
Edilberto Mendoza, when asked where the original was he said: “As far as harmonized to mean that upon a party’s purchase of a memorial lot on
I remember I do not know where the original but when I submitted with that installment from Eternal, an insurance contract covering the lot
payment together with the new clients all the originals I see to it before I purchaser is created and the same is effective, valid, and binding until
sign the transmittal letter the originals are attached therein.” terminated by Philamlife by disapproving the insurance application.
10.   In other words, the witness admitted not knowing where the original 19.   The second sentence of Creditor Group Life Policy on the Effective Date of
insurance application was, but believed that the application was transmitted Benefit is in the nature of a resolutory condition which would lead to the
to Philamlife as an attachment to a transmittal letter. cessation of the insurance contract. Moreover, the mere inaction of the
11.   As to the seeming inconsistencies between the testimony of Manuel Cortez insurer on the insurance application must not work to prejudice the insured;
on whether one or two insurance application forms were accomplished and it cannot be interpreted as a termination of the insurance contract. The
the testimony of Mendoza on who actually filled out the application form, termination of the insurance contract by the insurer must be explicit and
these are minor inconsistencies that do not affect the credibility of the unambiguous.
witnesses. 20.   To characterize the insurer and the insured as contracting parties on equal
12.   In the present case, the number of copies of the insurance application that footing is inaccurate at best. Insurance contracts are wholly prepared by
Chuang executed is not at issue, neither is whether the insurance application the insurer with vast amounts of experience in the industry
presented by Eternal has been falsified. Thus, the inconsistencies pointed purposefully used to its advantage. More often than not, insurance
out by Philamlife are minor and do not affect the credibility of Eternal’s contracts are contracts of adhesion containing technical terms and
witnesses. However, the question arises as to whether Philamlife assumed conditions of the industry, confusing if at all understandable to laypersons,
the risk of loss without approving the application. This question must that are imposed on those who wish to avail of insurance.
be answered in the affirmative. 21.   As such, insurance contracts are imbued with public interest that must be
13.   Philamlife and Eternal entered into an agreement denominated as Creditor considered whenever the rights and obligations of the insurer and the
Group Life Policy. In the policy, it is provided that: EFFECTIVE DATE insured are to be delineated. Hence, in order to protect the interest of
OF BENEFIT.The insurance of any eligible Lot Purchaser shall be insurance applicants, insurance companies must be obligated to act
effective on the date he contracts a loan with the Assured. However, there with haste upon insurance applications, to either deny or approve the
shall be no insurance if the application of the Lot Purchaser is not same, or otherwise be bound to honor the application as a valid,
approved by the Company. binding, and effective insurance contract.
14.   An examination of the above provision would show ambiguity between
its two sentences. The first sentence appears to state that the insurance
coverage of the clients of Eternal already became effective upon contracting
a loan with Eternal while the second sentence appears to require Philamlife
to approve the insurance contract before the same can become effective.
15.   It must be remembered that an insurance contract is a contract of
adhesion which must be construed liberally in favor of the insured and
strictly against the insurer in order to safeguard the latter’s interest.
16.   In Malayan Insurance Corporation v. Court of Appeals, this Court held
that: Indemnity and liability insurance policies are construed in accordance
with the general rule of resolving any ambiguity therein in favor of the
insured, where the contract or policy is prepared by the insurer. A contract
of insurance, being a contract of adhesion, par excellence, any
ambiguity therein should be resolved against the insurer; in other words,
003 Blue Cross Health Care v. Olivares (Soriano) service of limitless consultations for additional p1,000. She paid those
February 12, 2008 | Corona, J. | The Contract of Insurance amounts in full on oct 17, 2002. The application was approved on October
22, 2002.
PETITIONER: Clue Ceoss Health Care Inc. 44.   In the heath care agreement, ailments due to pre-existing conditions were
RESPONDENTS: Neomi and Danilo Olivares excludedfrom the coverage.
45.   On Nov 30, 2002, 38 days from the effectivity of her health insurance,
SUMMARY: Neomi Olivares applied for a health care program with Blue Neomi suffered stroke and was admitted at the Medical City hich was one
Cross. In the health care agreement, ailments due to pre-existing conditions were of the accredited hospitals by Blue Cross. During her confinement, she
excluded from the coverage. Neomi suffered stroke and was admitted to Medical underwent several lab tests.
City, one of the accredited hospitals. She incurred hospital exenses amounting to 46.   She incurred hospital expenses amounting tp P34,217.20. Consequently, she
P34,217.20. She requested from the representative of petitioner at Medical City a requested from the representative of petitioner at Medical City a letter of
letter of authorization in order to settle her medical bills. But petitioner refused authorization in order to settle her medical bills. But petitioner refused to
to issue the letter and suspended payment pending the submission of a issue the letter and suspended payment pending the submission of a
certification from her attending physician that the stroke she suffered was not certification from her attending physician that the stroke she suffered was
caused by a pre-existing condition. She demanded petitioner Blue Cross to pay not caused by a pre-existing condition.
the medical bill. When the petitioner refused, the spouses were constrained to 47.   When she was discharged, she demanded the petitioner to pay her mdical
settle the bill. Thereafter, they filed a complaint in the MeTC. Petitioner said that bill. hen petitioner still refused, she and her husband, respondent Danilo
it had not yet denied the claim, s it was awaiting for the attending physician’s Olivares, were constrained to settle the bill.
report. The Dr. Saniel, the attending physician, stated that as oer the request of 48.   Thereafter, they filed a complaint for collection of sum of money against
Neomi, he willnot release any medical information concerning neomi;s petitioner in the MeTC.
neurologic status to anyone without her approval. MeTC then denied the 49.   Blue Cross maintained that it had not yet denied respondents’ claim as it
complaint. RTC reversed the MeTC ruling and held that it was the burden of was still waiting Dr. Saniel’s report.
petitioner to prove that the stroke of Neomi was excluded. CA affirmed RTC 50.   Dr. Saniel in his letter to the petitioner state that Neomi called and stated
decision. Petitioner argues that respondents prevented Dr. Saniel from that she no longer has any relationship with Blue Cross, and that Dr. Saniel
submitting the medical report. Hence, the presumption that evidence willfully should not release any medical information concerning Neomi’s neurologic
suppressed would be adverse if produced should apply in its favor. Respondents status to anyone without her approval.
counter that the burden was on petitioner to prove that Neomi's stroke was 51.   MeTC dismissed the complaint for lack of cause of action. Stating that the
excluded from the coverage of their agreement because it was due to a pre- evidence (letter) on record reveals that it was no less than [respondent
existing condition. It failed to prove this. Issue: 1. Whther petitioner Blue Neomi] herself who prevented her attending physician from issuing the
Cross was able to prove that Neomi’s stroke was caused by pre-existing required certification, petitioner Blue Cross cannot be faulted from
condition and therefore was excluded from the coverage of the health agreement suspending payment of her claim, for until and unless it can be shown from
–No. Petitioner never presented any evidence to prove that respondent Neomi's the findings made by her attending physician that the stroke she suffered
stroke was due to a pre-existing condition. It merely speculated that Dr. Saniel's was not due to pre-existing conditions could she demand entitlement to the
report would be adverse to Neomi, based on her invocation of the doctor-patient benefits of her policy
privilege. 52.   RTC reversed the decision of the MeTC, ordered Blue Cross to pay the
medical bill, moral, exemplary damages, and attorney’s fees, and held that
DOCTRINE: It is an established rule in insurance contracts that when their that it was the burden of petitioner to prove that the stroke of respondent
terms contain limitations on liability, they should be construed strictly against Neomi was excluded from the coverage of the health care program for being
the insurer. These are contracts of adhesion the terms of which must be caused by a pre-existing condition. It was not able to discharge that burden.
interpreted and enforced stringently against the insurer which prepared the 53.   CA affirned RTC decision. Hence, this petition.
contract. This doctrine is equally applicable to health care agreements. 54.   Petitioner argues that respondents prevented Dr. Saniel from submitting the
medical report. Hence, the presumption that evidence willfully suppressed
would be adverse if produced should apply in its favor.
FACTS: 55.   Respondents counter that the burden was on petitioner to prove that Neomi's
43.   Respondent Neomi Olivares applied for a health care program with Blue stroke was excluded from the coverage of their agreement because it was
Cross, a health maintenance firm. For the period Oct 16, 2002 to Oct 15, due to a pre-existing condition. It failed to prove this.
2003, she paid P11,117. For the same period, she also availed of additional
of the agreement are excluded from its coverage if they become manifest
within one year from its effectivity. Stated otherwise, petitioner is not
ISSUE/s: liable for pre-existing conditions if they occur within one year from the
9.   Whther petitioner Blue Cross was able to prove that Neomi’s stroke was time the agreement takes effect.
caused by pre-existing condition and therefore was excluded from the 3.   The court ruled in a previous case that a health care agreement is in the
coverage of the health agreement –No. Petitioner never presented any nature of a non-life insurance.
evidence to prove that respondent Neomi's stroke was due to a pre-existing 4.   It is an established rule in insurance contracts that when their terms contain
condition. It merely speculated that Dr. Saniel's report would be adverse to limitations on liability, they should be construed strictly against the insurer.
Neomi, based on her invocation of the doctor-patient privilege These are contracts of adhesion the terms of which must be interpreted and
enforced stringently against the insurer which prepared the contract. This
RULING: WHEREFORE, the petition is hereby DENIED. The July 29, 2005 doctrine is equally applicable to health care agreements.
decision and September 21, 2005 resolution of the Court of Appeals in CA-G.R. SP 5.   Petitioner never presented any evidence to prove that respondent Neomi's
No. 84163 are AFFIRMED. stroke was due to a pre-existing condition. It merely speculated that Dr.
RATIO: Saniel's report would be adverse to Neomi, based on her invocation of the
1.   The court agrees with the respondents. doctor-patient privilege. This was a disputable presumption at best.
2.   The health care agreement defined a pre-existing condition as: 6.   Disputable presumption does not apply if (a) the evidence is at the disposal
a.   Disability which existed before the commencement date of membership of both parties; (b) the suppression was not willful; (c) it is merely
whose natural history can be clinically determined, whether the Member corroborative or cumulative and (d) the suppression is an exercise of a
was aware of such illness or condition. Such conditions also include privilege.
disabilities existing prior to reinstatement date in the case of lapse of an 7.   Here, respondents' refusal to present or allow the presentation of Dr.
Agreement. Notwithstanding, the following disabilities but not to the Saniel's report was justified. It was privileged communication between
exclusion of others are considered pre-existing conditions including their physician and patient.
complications when occurring during the first year of a Members 8.   Furthermore, as already stated, limitations of liability on the part of the
coverage: insurer or health care provider must be construed in such a way as to
I. Tumor of Internal Organs
preclude it from evading its obligations. Accordingly, they should be
II. Hemorrhoids/Anal Fistula scrutinized by the courts with extreme jealousy and care and with
III. Diseased tonsils and sinus conditions requiring surgery a jaundiced eye.
IV. Cataract/Glaucoma 9.   Since petitioner had the burden of proving exception to liability, it should
V. Pathological Abnormalities of nasal septum or turbinates
VI. Goiter and other thyroid disorders
have made its own assessment of whether respondent Neomi had a pre-
VII. Hernia/Benign prostatic hypertrophy existing condition when it failed to obtain the attending physician's report. It
VIII. Endometriosis could not just passively wait for Dr. Saniel's report to bail it out. The mere
IX. Asthma/Chronic Obstructive Lung disease reliance on a disputable presumption does not meet the strict standard
X. Epilepsy
XI. Scholiosis/Herniated disc and other Spinal column
required under our jurisprudence.
abnormalities 10.   On the issue on the grant of exemplary and moral damages: The RTC and
XII. Tuberculosis CA found that there was a factual basis for the damages adjudged against
XIII. Cholecysitis
petitioner. They found that it was guilty of bad faith in denying a claim
XIV. Gastric or Duodenal ulcer
XV. Hallux valgus
based merely on its own perception that there was a pre-existing condition
XVI. Hypertension and other Cardiovascular diseases
XVII. Calculi
XVIII. Tumors of skin, muscular tissue, bone or any form of
blood dyscracias
XIX. Diabetes Mellitus
XX. Collagen/Auto-Immune disease

After the Member has been continuously covered for 12


months, this pre-existing provision shall no longer be
applicable except for illnesses specifically excluded by an
endorsement and made part of this Agreement
b.   Under this provision, disabilities which existed before the commencement
004 MALAYAN INSURANCE v. REGIS BROKERS (Paul) 4.   When the shipment arrived at ABB Koppels warehouse, it was discovered
Nov. 23, ’07 | Tinga, J. | What may be insured – has to be proven that only 65 of the 120 pieces of motors were actually delivered and that
the remaining 55 motors, valued at US$2,374.35, could not be accounted
for.
PETITIONER: Malayan Insurance Co., Inc. 5.   The shipment was insured by Malayan. Malayan paid ABB Koppel Php
RESPONDENTS: Regis Brokerage Corp. 156,549.55 and got subrogated to the right of ABB Koppel v. Regis and
Paircargo.
SUMMARY: On Feb, 1, ’95, Fasco Motors Group loaded 120 pieces of motors 6.   Malayan then filed a complaint for damages against Regis and Paircargo
on board board China Airlines Flight 621 bound for Manila from the United with the Manila MeTC. In the course of the trial, Malayan presented
States. The cargo was to be delivered to consignee ABB Koppel, Inc. (ABB Marine Risk Note No. RN-0001-19832 (Marine Risk Note) dated 21
Koppel). When the cargo arrived at NAIA, it was cleared and forwarded to March 1995 as proof that the cargo was insured by Malayan.
Peoples Aircargo & Warehousing Corp’s (Paircargos) warehouse for temporary 7.   MeTC ruled that Regis alone was solely liable. The RTC affirmed this.
storage pending release by the Bureau of Customs. Paircargo remained in 8.   Regis brought it to the CA which reversed the RTC judgment and dismissed
possession of the cargo until 7 March 1995, at which point respondent Regis Malayan’s complaint. It held that the Marine Risk Note presented as proof
Brokerage Corp. (Regis) withdrew the cargo and delivered the same to ABB that the cargo was insured was invalid.
Koppel at its warehouse. When the shipment arrived at ABB Koppels warehouse, 9.   It was observed that the Marine Risk Note was procured from Malayan only
it was discovered that only 65 of the 120 pieces of motors were actually on 21 March 1995, when in fact the insured, ABB Koppel, had learned of
delivered. Malayan paid and filed a complaint for damages against Regis the partial loss of the motors as early as 7 March 1995.
and Paircargo with the MeTCMalayan presented the Marine Risk Note 10.   The CA noted that under Section 3 of the Insurance Code, the past event
dated 21 March 1995 as proof that the cargo was insured by Malayan. The which may be insured against must be unknown to the parties and so for
MeTC and the RTC ruled in Malayan’s favor. However, the CA reversed it, that reason the insurance contract in this case violated Section 3.
saying that the Marine Risk Note was procured from Malayan only on 21 11.   Also, the CA held that due execution and authenticity of the subrogation
March 1995, but the loss was discovered on March 7. The SC ruled that receipt presented before the trial court by Malayan were not duly proven
Malayan failed to establish its cause of action because it never presented the since the signatories thereto were not presented by Malayan before the trial
Marine Insurance Policy. Because Malayan's right to recovery derives from court to identify their signatures thereon, and neither was evidence
contractual subrogation as an incident to an insurance relationship, and not presented to establish the genuineness of such signatures
from any proximate injury to it inflicted by the Regis, it is critical that 12.   Malayan filed an MR and said that the Marine Risk Note is an open policy
Malayan establish the legal basis of such right to subrogation by presenting the per Marine Open Cargo Policy No. OPEN POLICY-0001-00410
contract constitutive of the insurance relationship between it and ABB Koppel. issued before February 1, 1995.
ISSUES:
DOCTRINE: The particular date as to when such insurance contract was 1.   W/N Malayan was able to establish its cause of action against Regis? No
constituted cannot be established with certainty without the contract itself, because it wasn’t able to conclusively prove the existence of an insurance
and that point is crucial since there can be no insurance on a risk that had contract.
already occurred by the time the contract was executed.
RULING: WHEREFORE, the petition is DENIED. Costs against petitioner.
FACTS:
1.   On Feb, 1, ’95, Fasco Motors Group loaded 120 pieces of motors on board RATIO:
board China Airlines Flight 621 bound for Manila from the United States. 1.   Malayan was arguing that the lost cargo was covered by not only the Marine
The cargo was to be delivered to consignee ABB Koppel, Inc. (ABB Risk Note but also by the Marine Insurance Policy (Fact 12) which was
Koppel). issued days before the cargo arrived in Manila.
2.   When the cargo arrived at NAIA, it was cleared and forwarded to Peoples 2.   The Marine Policy was attached to the petition of the SC, but it was not
Aircargo & Warehousing Corp’s (Paircargos) warehouse for temporary presented in trial.
storage pending release by the Bureau of Customs 3.   Malayan tried to showed that it had an “open policy” with ABB Koppel
3.   Paircargo remained in possession of the cargo until 7 March 1995, at which covered by Section 60 of the Insurance Code, wherein the value of the thing
point respondent Regis Brokerage Corp. (Regis) withdrew the cargo and insured is not agreed upon but left to be ascertained in case of loss, and that
delivered the same to ABB Koppel at its warehouse. the Marine Risk Note was nothing but a determination of the value of the
thing insured pursuant to the open policy as established by the Marine 13.   What the Marine Risk Note bears, as a matter of evidence, is that it is not
Insurance Policy. apparently the contract of insurance by itself, but merely a complementary
4.   Unfortunately for Malayan, the Court could not attribute any evidentiary or supplementary document to the contract of insurance that may have
weight to the Marine Insurance Policy. existed as between Malayan and ABB Koppel.
5.   The SC is not a trier of facts. Therefore, it relied on CA’s conclusion that at 14.   But again, Malayan never introduced the Marine Insurance Policy as the
the very instance the Marine Risk Note was offered in evidence, Regis main insurance contract nor even referred to it in the original complaint.
already posed its objection to the admission of said document on the ground Therefore, it failed to establish its cause of action.
that such was "immaterial, impertinent and irrelevant to this case because 15.   Because Malayan's right to recovery derives from contractual subrogation
the same was issued on March 21, 1995 which is after the occurrence of as an incident to an insurance relationship, and not from any proximate
the loss on February 1, 1995." injury to it inflicted by the Regis, it is critical that Malayan establish the
6.   Since no insurance policy was presented at the trial by Malayan, or even legal basis of such right to subrogation by presenting the contract
before the CA, there is no basis for the SC to consider it, notwithstanding constitutive of the insurance relationship between it and ABB Koppel.
Malayans attempt to submit such document to us along with its present 16.   his should have been accomplished from the moment it filed the complaint.
petition, even it may have reflected the existence of an insurance contract Since the Marine Insurance Policy was constitutive of the insurer-insured
between Malayan and ABB Koppel prior to the loss of the motors. relationship from which Malayan draws its right to subrogation, such
7.   Malayan’s theory of the case it pursued before the trial court was that the document should have been attached to the complaint itself.
perfected insurance contract which it relied upon as basis for its right to 17.   It is an actionable document and Sec. 710, Rule 9 of the Rules of Civil
subrogation was not the Marine Insurance Policy but the Marine Risk Procedure governs it.
Note which, unlike the former, was actually presented at the trial and 18.   It was incumbent on Malayan, whose right of subrogation derived from the
offered in evidence. Marine Insurance Policy, to set forth the substance of such contract in its
8.   The Claims Processor of Malayan who testified in court in behalf of his complaint and to attach an original or a copy of such contract in the
employer actually acknowledged that the "proof that ABB Koppel insured complaint.
the shipment to Malayan" was the Marine Risk Note, and not the Marine 19.   Not doing so actually violates Regis’ right to due proves because the latter
Insurance Policy. cannot object to it.
9.   Even the very complaint filed by Malayan before the MeTC stated that "the 20.   It cannot be denied from the only established facts that Malayan and ABB
subject shipment was insured by Malayan under Risk Note No. 0001- Koppel comported as if there was an insurance relationship between them
19832," and not by the Marine Insurance Policy, which was not adverted to and documents exist that evince the presence of such legal relationship.
at all in the complaint. 21.   But under these premises, the very insurance contract emerges as the white
10.   Therefore, the SC can only consider the Marine Risk Note in determining elephant in the room — an obdurate presence which everybody reacts to,
the existence of an insurance contract. However, again, it is dated Mar. 21, yet legally invisible as a matter of evidence since no attempt had been made
1995 – after the loss9. However, the motors insured were already to prove its corporeal existence in the court of law.
compromised by then. 22.   Malayan would have us effectuate an insurance contract without having to
11.   The Court, in Aboitiz c. Philam Gen. has already held that a Marine Risk consider its particular terms and conditions, and on a blind leap of faith that
Note is not an insurance policy on its own. It is only an acknowledgment or such contract is indeed valid and subsisting.
declaration of the private respondent confirming the specific shipment 23.   The conclusion further works to the utter prejudice of Regis since it would
covered by its Marine Open Policy, the evaluation of the cargo, and the be deprived the opportunity to examine the document that gives rise to the
chargeable premium, a description that is reflective as well of the present Malayan’s right to recover against them, or to raise arguments or objections
Marine Risk Note, if not of marine risk notes in this country in general. against the validity or admissibility of such document.
12.   Malayan correctly points out that the Marine Risk Note itself adverts to
"Marine Cargo Policy Number Open Policy-0001-00410" as well as to "the                                                                                                                        
10
standard Marine Cargo Policy and the Company's Marine Open Policy."  SEC.  7.  Action  or  defense  based  on  document.   —  Whenever  an  action  or  defense  is  based  
                                                                                                                        upon  a  written  instrument  or  document,  the  substance  of  such  instrument  or  document  shall  
9
 "Had  this  day  noted  the  above-­‐mentioned  risk  in  your  favor  and  hereby  guarantees  that  this   be   set   forth   in   the   pleading,   and   the   original   or   a   copy   thereof   shall   be   attached   to   the  
document   has   all   the   force   and   effect   of   the   terms   and   conditions   in   the   Corporation's   pleading  as  an  exhibit,  which  shall  be  deemed  to  be  a  part  of  the  pleading,  or  said  copy  may  
printed  form  of  the  standard  Marine  Cargo  Policy  and  the  Company's  Marine  Open  Policy."     with  like  effect  be  set  forth  in  the  pleading.    
   
24.   If a legal claim is irrefragably sourced from an actionable document, the
defendants cannot be deprived of the right to examine or utilize such
document in order to intelligently raise a defense.
25.   The inability or refusal of the plaintiff to submit such document into
evidence constitutes an effective denial of that right of the defendant which
is ultimately rooted in due process of law.
26.   The particular date as to when such insurance contract was constituted
cannot be established with certainty without the contract itself, and that
point is crucial since there can be no insurance on a risk that had
already occurred by the time the contract was executed.
27.   Since the documents in evidence and testimonies allude to "marine
insurance" or "marine risk note," it also is a legitimate question whether
the particular marine insurance relationship between Malayan and ABB
Koppel also covers cargo delivered not by ships at sea but by airplane
flights, as had occurred in this case. Only the actual policy itself could
definitively settle such a question.
005 EASTERN SHIPPING v. PRUDENTIAL (TIMBOL)
September 11, 2009 | Del Castillo, J. | What can be insured/insured against FACTS:
33.   56 cases of completely knock-down auto parts of Nissan motor vehicles
PETITIONER: Eastern Shipping Lines, Inc. (cargoes) were loaded on board M/V Apollo Tujuh (carrier) at Nagoya,
RESPONDENTS: Prudential Guarantee and Assurance, Inc. Japan, to be shipped to Manila
1.   The shipment was consigned to Nissan Motor Philippines, Inc.
SUMMARY: 56 cases of completely knock-down auto parts of Nissan Motor (Nissan) and was covered by Bill of Lading No. NMA-1
vehicles were loaded on board M/V Apollo Tujuh, owned by Eastern Shipping, at 2.   The carrier was owned and operated by Eastern Shipping Lines,
Japan to be shipped to Manila. When it arrived to Manila, it was discharged to ATI Inc.
in good condition, except for the four cases. From ATI, they were then brought to 34.   The carrier arrived at the port of Manila. The shipment was then discharged
Nissan’s warehouse, and after conducting a survey, the four cases were found to from the vessel onto the custody of the arrastre operator, Asian Terminals,
have shortage and damage due to the pilferage and improper handling of the vessel Inc. (ATI), complete and in good condition, except for four cases
and/or the Arrastre contractors. Thus, Nissan demanded for reimbursement from 35.   The shipment was withdrawn by Saefront Customs and Brokerage from the
Eastern Shipping and ATI, but failed. After which, Nissan claimed reimbursement pier and delivered to the warehouse of Nissan in Quezon City
from Prudential as the insurer based on all risks per Marine Open Policy and Marine 36.   A survey of the shipment was then conducted by Tan-Gaute Adjustment
Cargo Risk Notice. Upon payment of Prudential to Nissan, the former filed a claim Company, Inc. (surveyor) at Nissan’s warehouse
against Eastern and ATI. The RTC ruled in favor of Prudential finding Eastern and 1.   The surveyor submitted its report with a finding that there were
ATI solidarily liable. However, on appeal, the CA exonerated ATI and held Eastern “short (missing)” items in Cases Nos. 10/A26/T3K and 10/A26/7K
Shipping solely responsible. And that the presentation of insurance policy is and “broken/scratched” and “broken” items in Case No.
dispensible. 10/A26/70K”;
2.   And that in its opinion, the shortage and darnage sustained by the
Hence the issue before the Court being whether the Marine Risk Note and the shipment were due to pilferage and improper handling,
Subrogation Receipt, without the presentation of the Marine Insurance Policy, are respectively while in the custody of the vessel and/or Arrastre
sufficient to prove Prudential’s right to subrogation. Contractors
37.   Nissan demanded the sum of P1,047,298.34 representing the cost of the
The SC held that NO, presentation in this case is required for Prudential to claim the damages sustained by the shipment from Eastern Shipping, the owner of the
right of subrogration. It must first be noted that a Marine Risk Note is not an vessel, and ATI, the arrastre operated
insurance policy. Rather, it is an acknowledgement or declaration of the insurer 1.   However, the demands were not heeded
confirming the specific shipment covered by its marine open policy, the evaluation 38.   As insurer of the shipment against all risks per Marine Open Policy No. 86-
of the cargo and the chargeable premium. Furthermore, the Marine Risk Note was 168 and Marine Cargo Risk Note No. 3921/95, Prudential Guarantee and
questionable since the date of the such note is the same as the time the goods arrived Assurance Inc. paid Nissan the sum of P1,047,298.34
the port. Meaning that the goods were not specifically covered by any particular 39.   Prudential sued Eastern and ATI for reimbursement of the amount it paid to
insurance at the time of transit. And without the presentation of the Marine Nissan before the RTC of Makati, and claimed that it was subrogated to the
Insurance Policy, it would be impossible to know the specifics of the Policy as rights of Nissan by virtue of said payment
written in the Note, as well as the terms and conditions were complied before 40.   RTC rendered a decision in favor of Prudential and held that Eastern and
Prudential paid Nissan’s claim. Moreover, the reading of the transcript of records ATI are solidarily liable
show that Eastern Shipping already objected to the non-presentation of the Marine 41.   On appeal, the CA exonerated ATI and ruled that Eastern was solely
Insurance Policy. And since the claim of insurance is based on an actionable responsible for the damages caused to the cargoes.
document, it was incumbent upon Prudential to present the policy based on Sec 7, 1.   Moreover, the CA relying on Delsan Transport Lines, Inc. v. CA,
Rule 9 of the Rules of Civil Procedure. ruled that the right of subrogation accrues upon payment by the
insurance company of the insurance claim and that the presentation
DOCTRINE: It is significant that the date when the alleged insurance contract was of the insurance policy is not indispensable before the appellee
constituted cannot be established with certainty without the contract itself. Said point may recover in the exercise of its subrogatory right
is crucial because there can be no insurance on a risk that had already occurred by 42.   As such, Eastern argues that Prudential was not properly subrograted
the time the contract was executed. Surely, the Marine Risk Note on its face does not because of the non-presentation of the marine insurance policy. In the
specify when the insurance was constituted
case at bar, in order to prove its claim, Prudential presented a marine cargo guarantees that this document has all the force and effect of the
risk note and a subrogation receipt. terms and conditions in the Corporation’s printed form of the
standard Marine Cargo Policy and the Company’s Marine Open
ISSUE/s: Policy “
6.   WoN the Marine Risk Note and the Subrogation Receipt, without the 41.   Likewise, the date of the issuance of the Marine Risk Note also caught the
Marine Insurance Policy, are sufficient to prove Prudential’s right of attention of Eastern Shipping, stating in it’s comment that:
subrogration – NO, because a Mairne Risk Note is only an 1.   Exh. B, Marine Cargo Risk Note No. 39821 dated Nov. 16, 1995 is
acknowledgement or declaration of the insurer confirming the specific being objected to for being irrelevant and immaterial as it was
shipment covered by its marine open policy. The presentation of the marine executed on Nov.16, 1995. The cargoes arrived in Manila on Nov.
open policy indicating therein the extent of the note, as well as the terms 16, 1995.
and conditions needed to be complied with. 2.   This means that the cargoes are not specifically covered by any
particular insurance at the time of transit
RULING: WHEREFORE, premises considered, the petition is GRANTED. The 3.   The alleged Marine Open Policy was not presented. Marine Open
April 26, 2006 Decision and August 15, 2006 Resolution of the Court of Appeals in Policy may be subject to Institute Cargo Clauses which may
CA G.R. CV No. 68165 are hereby REVERSED and SET ASIDE. The Complaint require arbitration prior to the filing of an action in court.
in Civil Case No. 96 1665 is DISMISSED. 42.   The Marine Risk Note relied upon by Prudential as the basis for its
claim for subrogation is insufficient to prove said claim
RATIO: 1.   Hence, without a copy of the marine insurance policy, it would be
36.   Before anything else, it must be emphasized that a marine risk note is not impossible and simply guesswork to know whether the cargo was
an insurance policy. It is only an acknowledgement or declaration of the inusred during the voyage which started on Nov. 8, 1995
insurer confirming the specific shipment covered by its marine open policy, 2.   Again, without the marine insurance policy, it would be
the evaluation of the cargo and the chargeable premim impossible for this Court to know the following:
37.   The nature of a marine cargo risk note was explained in ICTSI v. FGU 1.   The specifics of the Institute Cargo Clauses A and
Insurance Corporation: other terms and conditions per Marine Open Policy-
1.   It is the marine open polic which is the main insurance contract. In 86-168 as alluded to in the Marine Risk Note;
other words, the marine open policy is the blanket insurance ot be 2.   If the said terms and conditions were actually
undertaken by FGU on all goods to be shipped by RAGC during complied with before Prudential paid Nissan’s claim
the existence of the contract, while the marine risk note specifies 43.   Furthermore, a reading of the transcript of the records clearly show that, at
the particular goods/shipment insured by FGU on that specific the RTC, Eastern Shipping had already objected to the non-presentation of
transaction, including the sum insured, the shipment the marine insurance policy
particulars as well as the premium paid for such shipment 1.   Clearly, Eastern Shipping was not remis when it openly objected to
38.   It is undisputed that the cargoes were already on board the carrier as early as the non-presentation oft eh Marine Insurance Policy
Nov. 8, 1995 and that the same arrived at the port of Manila on Nov. 16, 44.   In addition, it was Prudential’s burden to present the evidence necessary to
1995 substantiate its claim
39.   It is, however, very apparent that the Marine Cargo Risk Note was issued 1.   In its Complaint, Prudential alleged: “That the above-described
only on November 16, 1995. The same, therefore, should have raised a red shipment was insured for P14,173,042.91 against all risks under
flag, as it would be impossible to know whether said goods were actually Marine Cargo Risk Note No. 39821/Marine Open Policy No. 86-
insured while the same were in transit from Japan to Manila. 168.”
40.   Malayan Insurance Co., Inc. v. Regis Brokerage Corp: 2.   Therefore, other than the marine cargo risk note, Prudential should
1.   Thus, we can only consider the Marine Risk Note in determining have also presented the marine insurance policy, as the same also
whether there existed a contract of insurance between ABB Koppel served as the basis for its complaint
and Malayan at the time of the loss of the motors 45.   Sec. 7, Rule 9 of the 1997 Rules of Civil Procedure, provide:
2.   However, the very terms of the Marine Risk Note itself are 1.   Action or defense based on document – Whenever an action or
quite damning. It is dated 21 Marhc 1995, or after the defense is based upon a written instrument or document, the
occurrence of the loss, and specifically states that Malayan “had substance of such instrument or document shall be set forth in
this day noted the above-mentioned risk in your favor and hereby the pleading, and the original or a copy thereof shall be
attached to the pleading as an exhibit, which shall be deemed vessel, unlike in Home Insurance in which the cargo passed
to be a part of the pleading, or said copy may, with like effect, through several statges with different parties and it could not be
be set forth in the pleading. determined when the damage to the cargo occurred, such that the
46.   Malayan Insurance Co., Inc. v. Regis Brokerage Corp is instructive: insurer should be liable for it
1.   xxx Our procedural rules make plain how easily Malayan could 51.   Although the CA may have ruled that the damage to the cargo
have adduced the Marine Insurance Policy. Ideally, this should occurred while the same was in Eastern Shipping’s custody, this Court
have been accomplished from the momen tit filed the coplaint. cannot apply the ruling in ICTSI to the case at bar
Since the Marine Insurance Policy was constitute of the 1.   In contrast, unlike in ICTSI where there was no issue as
insurer-insured relationship from which Malayan draws its regrads the provisions of the marine insurance policy, such
right to subrogation, such document should have been attached that the presentation of the contract itself is necessary for
to the complaint itself, as provided for in Section 7, Rule 9 of the perusal, herein Eastern Shipping had repeatedly objected to
1997 Ruels of Civil Procedure x x x the non-presentation of the marine insurance policy and had
47.   Therefore, since Prudential alluded to the actionable document in its manifested its desire to know the specific provisions thereof
complaint, the contract of insurance between it and Nissan, as integral to its 2.   Moreover, and the same is critical, the marine risk note in the
cause of action against Eastern Shipping, the Marine Insurance Policy case at bar is questionable, because:
should have been attached to the Complaint 1.   It is dated on the same day the cargoes arrived at the
48.   It is significant that the date when the alleged insurance contract was port of Manila and not during the duration of the
constituted cannot be established with certainty without the contract voyage
itself. Said point is crucial because there can be no insurance on a risk 2.   Wihtout the marine insurance policy to elucidate on
that had already occurred by the timte the contract was executed. the specifics of the terms and conditions allueded to in
Surely, the Marine Risk Note on its face does not specify when the the marine risk note, it would be simply guesswork to
insurance was constituted know if the same were complied with
49.   The importance of the presentation of the Marine Insurance Policy was also 52.   The Court further recognizes the danger as precedent should we
emphasized in Wallemn Philippines Shipping v. Prudential Guarantee & sustain Malayan’s position, and not only because such a ruling would
Assurance: formally violate the rule on actionable documents
1.   The contract of insurance must be presented in evidence to indicate 53.   Malayan would have us effectuate an insurance contract without
the extent of its coverage. As there was no determination of rights having to consider its particular terms and conditions, and on a blind
under the insurance contract, this Court’s ruling in Home Insurance leap of faith that such contract is indeed valid and subsisting
Corporation v. CA is applicable: 54.   The conclusion further works to the utter prejudice of defendants such
1.   The insurance contract has not been presented. It may be as Regis or Paircargo since they would be deprived the opportunity to
assumed for the sake of argument that the subrogration examine the document that gives rise to the plaintiff’s right to recover
receipt may nevertheless be used to establish the against them, or to raise arguments or objections against the validity or
relationship between Home Insurance and Nestle Phil admissibility of such document
(consignee) and teha moutn paid to settle the claim. But 55.   If a legal claim is irrefragably sourced from an actionable document,
this is all the document can do. By itself alone, the the defendant cannot be deprived of the right to examine or utilize such
subrogation receipt is not sufficient to prove the document in order to intelligently raise a defense
petitioner’s (Home Insurance) claim holding the 56.   The inability or refusal of the plaintiff (insurer) to submit such
respondent (Mabuhay Brokerage) liable for the damage to document into evidence constitutes an effective denial of that right of
the engine the defendant which is ultimately rooted in due process of law, to say
50.   There are however, cases where the Court ruled that the non-presentation of nothing on how such failure fatally diminishes the plaintiff’s
the marine insurance policy is not fatal, as can be gleaned in the case of substantiation of its own cause of action
ICTSI: 57.   In conclusion, this Court rules that based on the applicable
1.   However, as in every general rule, there are admitted exceptions. jurisprudence, because of the inadequacy of the Marine Cargo Risk
In Delsan Transport Lines, Inc. v. CA, the Court stated that the Note for the reasons already stated, it was incumbent on respondent to
presentation of the insurance policy was not fatal because the loss present in evidence the Marine Insurance Policy, and having failed in
of the cargo undoubtedly occurred while on board the petitioner’s doing so, its claim of subrogation must necessarily fail
006 PARAMOUNT v. SPOUSES REMONDEULAZ (CHIQUI) improvements thereon, however, Sales failed to return the subject vehicle
November 28, 2012 | Peralta, J. | What May Be Insured within the agreed three-day period.
4.   As a result, Spouses notified Paramount Insurance to claim for the
PETITIONER: Paramount Insurance Corporation reimbursement of their lost vehicle. However, Paramount Insurance refused
RESPONDENTS: Spouses Yves and Maria Teresa Remondeulaz to pay.
5.   The Spouse lodged a complaint for a sum of money against Paramount
Insurance before the RTC praying for the payment of the insured value of
SUMMARY: Spouses Remondeulaz insured their car under Paramount Insurance
their car plus damages. After presentation of the Spouses’ evidence,
for one year for own damage, theft, third-party property damage, and third-party
Paramount Insurance filed a Demurrer to Evidence.
personal injury. The car was unlawfully taken by Sales who was supposed to add
6.   RTC dismissed the complaint filed by the Spouses.
accessories and improvements on it but failed to return the vehicle within the agreed
a.   Paramount Insurance had successfully prosecuted and had been
period. The Spouses sought for reimbursement from Paramount Insurance but the
awarded the amount claimed in this action, in another action which
latter refused to pay. Spouses then filed a case for a sum of money in the RTC. RTC
involved the loss of the same vehicle under the same circumstances
sided with Paramount Insurance stating that the Spouses already had reimbursement
although under a different policy and insurance company (Standard
for the same subject vehicle but from another insurance company. On the other
Insurance Company, Inc). This, considered with the principle that
hand, CA reversed the decision saying that that car is different from the one insured
an insured may not recover more than its interest in any property
under Paramount Insurance. W/N Paramount Insurance is liable under the insurance
subject of an insurance, leads the court to dismiss this action.
policy for the loss of the Spouses’ vehicle – YES, the Spouses’ policy clearly
7.   CA reversed and set aside the Order issued by the trial court the subject car
undertook to indemnify the insured against loss of or damage to the scheduled
is different from the one insured with another insurance company.
vehicle when caused by theft. (see doctrine) Sales did not have juridical possession
over the vehicle. Records would show that the Spouses entrusted possession of their
vehicle only to the extent that Sales will introduce repairs and improvements ISSUE/s:
thereon, and not to permanently deprive them of possession thereof. Since, theft can 1.   W/N Paramount Insurance is liable under the insurance policy for the loss
also be committed through misappropriation, the fact that Sales failed to return the of the Spouses’ vehicle – YES, the Spouses’ policy clearly undertook to
subject vehicle to the Spouses constitutes Qualified Theft. Hence, since the indemnify the insured against loss of or damage to the scheduled vehicle
Spouses’ car is undeniably covered by a Comprehensive Motor Vehicle Insurance when caused by theft.
Policy that allows for recovery in cases of theft, Paramount Insurance is liable under
the policy for the loss of the Spouses’ vehicle under the "theft clause."
RULING: WHEREFORE, the instant petition is DENIED. The Decision dated April
DOCTRINE: In interpreting the theft clause, SC explained that when one takes the 12, 2005 and Resolution dated July 20, 2006 of the Court of Appeals are hereby
motor vehicle of another without the latter's consent even if the motor vehicle is AFFIRMED in toto. SO ORDERED.
later returned, there is theft — there being intent to gain as the use of the thing
unlawfully taken constitutes gain. RATIO:
58.   Paramount Insurance argues that the loss of the Spouses vehicle is not a
FACTS: peril covered by the policy. It maintains that it is not liable for the loss,
since the car cannot be classified as stolen as the Spouses entrusted the
possession thereof to another person.
1.   Spouses Remondeulaz insured with Paramount Insurance their 1994 Toyota 59.   Adverse to Paramount Insurance’s claim, the Spouses’ policy clearly
Corolla Sedan under a Comprehensive Motor Vehicle Insurance Policy for
undertook to indemnify the insured against loss of or damage to the
one year for Own Damage, Theft, Third-Party Property Damage and Third-
scheduled vehicle when caused by theft.
Party Personal Injury, for the period commencing 26 May 1994 to 26 May
a.   SECTION III LOSS OR DAMAGE: The Company will, subject to
1995.
the Limits of Liability, indemnify the insured against loss of or
2.   During the effectivity of said insurance, Spouses’ car was unlawfully taken.
damage to the Scheduled Vehicle and its accessories and spare
Hence, they immediately reported the theft to the Traffic Management
parts whilst thereon: —
Command of the PNP who made them accomplish a complaint sheet.
b.   xxx (b) by fire, external explosion, self-ignition or lightning or
3.   In said complaint sheet, the Spouses alleged that a certain Ricardo Sales burglary, housebreaking or theft;
(Sales) took possession of the subject vehicle to add accessories and
60.   In People v. Bustinera, this Court had the occasion to interpret the "theft
clause" of an insurance policy. In this case, the Court explained that when
one takes the motor vehicle of another without the latter's consent even if
the motor vehicle is later returned, there is theft — there being intent to gain
as the use of the thing unlawfully taken constitutes gain.
61.   Also, in Malayan Insurance Co., Inc. v. Court of Appeals, this Court held
that the taking of a vehicle by another person without the permission or
authority from the owner thereof is sufficient to place it within the
ambit of the word theft as contemplated in the policy, and is therefore,
compensable.
62.   Moreover, the case of Santos v. People is worthy of note. Similarly in
Santos, the owner of a car entrusted his vehicle to therein petitioner Lauro
Santos who owns a repair shop for carburetor repair and repainting.
However, when the owner tried to retrieve her car, she was not able to do so
since Santos had abandoned his shop. In the said case, the crime that was
actually committed was Qualified Theft. However, the Court held that
because of the fact that it was not alleged in the information that the object
of the crime was a car, which is a qualifying circumstance, the Court found
that Santos was only guilty of the crime of Theft and merely considered the
qualifying circumstance as an aggravating circumstance in the imposition of
the appropriate penalty.
63.   In the instant case, Sales did not have juridical possession over the vehicle.
Here, it is apparent that the taking of the Spouses’ vehicle by Sales is
without any consent or authority from the former.
64.   Records would show that the Spouses entrusted possession of their
vehicle only to the extent that Sales will introduce repairs and
improvements thereon, and not to permanently deprive them of
possession thereof. Since, theft can also be committed through
misappropriation, the fact that Sales failed to return the subject vehicle
to the Spouses constitutes Qualified Theft.
65.   Hence, since the Spouses’ car is undeniably covered by a Comprehensive
Motor Vehicle Insurance Policy that allows for recovery in cases of theft,
Paramount Insurance is liable under the policy for the loss of the Spouses’
vehicle under the "theft clause."
66.   All told, Sales' act of depriving the Spouses of their motor vehicle at, or
soon after the transfer of physical possession of the movable property,
constitutes theft under the insurance policy, which is compensable.
007 United Merchants v. Country Bankers (Valle)
11 july 2012 | Carpio, J. | What may be insured 58.   UMC and Country executed Endorsement F/96-154 and Fire Invoice No.
16583A to form part of the Insurance Policy. Endorsement F/96-154
PETITIONER: United Merchants Corporation provides that UMCs stocks in trade were insured against additional
RESPONDENTS: Country Bankers Insurance Corporation perils, to wit: typhoon, flood, ext. cover, and full earthquake. The sum
insured was also increased to P50,000,000.00 effective 7 May 1996 to 10
SUMMARY: January 1997. On 9 May 1996, CBIC issued Endorsement F/96-157 where
UMC owns a warehouse where UMC stored its products (Christmas lights). the name of the assured was changed from Alfredo Tan to UMC.
UMC insured the Christmas lights with Country. This was later changed into 59.   A fire gutted the warehouse. Country designated CRM Adjustment
insure stocks in trade. A fire gutted the warehouse. UMC is claiming Php 50M Corporation (CRM) to investigate and evaluate the loss. Country’s insurer,
from Country but Country refused to pay, alleging that the fire was done through Central Surety, also requested the NBI to conduct an investigation.
arson and that UMC committed fraud. Country claimed that UMC was declaring 60.   UMC submitted to Country its Sworn Statement of Formal claim with
loss and claiming for loss that wasn’t covered under the policy. proofs of the loss. UMC demanded for at least 50% payment of its claim.
The issue in this case is whether or not UMC can validly claim for the Php 50M Country rejected the claim due to breach of the Insurance Policy which
it allegedly lost due to the fire. states:
a.   Condition No 15. If the claim be in any respect fraudulent, or if
The SC held that UMC cannot claim from Country. The Insurance Policy any false declaration be made or used in support thereof, or if any
provides that Country agreed to insure UMCs stocks in trade. UMC defined fraudulent means or devices are used by the Insured or anyone
stock in trade as tangible personal property kept for sale or traffic. UMC was acting in his behalf to obtain any benefit under this Policy; or if the
showing invoices that were suspicious, the Certificate from the Bureau of Fire loss or damage be occasioned by the willful act, or with the
Protection showed that the 50M it was claiming from Country included both the connivance of the Insured, all the benefits under this Policy shall
building and the contents of the building. But the Insurance Policy covered the be forfeited
stocks in trade. Basically, UMC was trying to claim 50M from Country by 61.   UMC filed a complaint in the RTC against Country. Country raised the
presenting imvoices and documents which were fraudulent. The Supreme Court following defenses:
refused the claim of UMC since submisisons of false invoices is a clear case of a.   No cause of action
fraud and fraud voids the policy. b.   Already prescribed
c.   UMC’s fire claim is tainted with fraud because the Statement of
DOCTRINE: Inventory showed that it had no stocks in trade and that UMC’s
The submission of false invoices to the adjusters establishes a clear case of fraud suspicious purchases for the year didn’t amount to 25M. The GIS
and misrepresentation which voids the insurer’s liability as per condition of the and Financial reports of UMC revealed that it had insufficient
policy. capital, which meant that UMC couldn’t have afforded the alleged
50M worth of stocks in trade.
What can be insured is only what is covered by the Policy. 62.   UMC presented five witnesses. The first witness was Josie Ebora (Ebora),
UMCs disbursing officer. Ebora testified that UMCs stocks in trade, at the
time of the fire, consisted of:
FACTS: a.   raw materials for its Christmas lights;
56.   United Merchants Corporation (UMC) is engaged in buying, selling, and b.   Christmas lights already assembled; and
manufacturing Christmas lights. It leased a warehouse where UMC c.   Christmas lights purchased from local suppliers.
assembled and stored its products.
57.   UMC’s general manager, Alfredo Tan insured UMC’s stocks in trade                                                                                                                                                                                                                                                                                        
Of Christmas lights against fire with Country Bankers Inusrance (Country) responsible  in  the  event  of  loss  and/or  damage  during  the  currency  of  this  
for P 15M. The policy was valid until Sept 6 1996.11
policy,   whilst   contained   in   the   building   of   one   lofty   storey   in   height,  
                                                                                                                        constructed   of   concrete   and/or   hollow   blocks   with   portion   of   galvanized  
11
  PROPERTY   INSURED:   On   stocks   in   trade   only,   consisting   of   Christmas   iron   sheets,   under   galvanized   iron   rood,   occupied   as   Christmas   lights  
Lights,   the   properties   of   the   Assured   or   held   by   them   in   trust,   on   storage.  
commissions,   or   on   joint   account   with   others   and/or   for   which   they   are    
d.   These stocks in trade were delivered from August 1995 to May shifts to the insurer to controvert the insureds prima facie case.
1996. She stated that Straight Cargo Commercial Forwarders 32.   Here, UMC established prima facie case against Country. Country doesn’t
delivered the imported materials to the warehouse, evidenced by dispute that UMC’s stocks were insured against fire and that the warehouse
delivery receipts. However, for the year 1996, UMC had no was gutted with fire. But since Country alleged an excepted risk, the burden
importations and only bought from its local suppliers. is shifted to Country to prove such exception.
e.   Ebora identified the suppliers as Fiber Technology Corporation 33.   Country failed to discharge its primordial burden of establishing that the
from which UMC bought stocks worth P1,800,000.00 on 20 May damage or loss was caused by arson, a limitation in the policy. Country’s
1996; Fuze Industries Manufacturer Philippines from which evidence did not prove that the fire was intentionally caused by UMC.
UMC bought stocks worth P19,500,000.00 from 20 January 1996 34.   First, the findings of Country’s witnesses, Cabrera and Lazaro, were based
to 23 February 1996; and Tomco Commercial Press from which on an investigation conducted more than four months after the fire. The
UMC bought several Christmas boxes. testimonies of Cabrera and Lazaro, as to the boxes doused with kerosene as
f.   Ebora testified that all these deliveries were not yet paid. Ebora told to them by barangay officials, are hearsay because the barangay
also presented UMCs Balance Sheet, Income Statement and officials were not presented in court.
Statement of Cash Flow. Per her testimony, UMCs purchases 35.   Cabrera and Lazaro even admitted that they did not conduct a forensic
amounted to P608,986.00 in 1994; P827,670.00 in 1995; and investigation of the warehouse nor did they file a case for arson.
P20,000,000.00 in 1996. Ebora also claimed that UMC had sales 36.   Second, the Sworn Statement of Formal Claim submitted by UMC, through
only from its fruits business but no sales from its Christmas lights CRM, states that the cause of the fire was faulty electrical wiring/accidental
for the year 1995. in nature. Country is bound by this evidence because in its Answer, it
63.   Annie Pabustan testified that the company provided 25 workers to assemble admitted that it designated CRM to evaluate UMCs loss.
and pack. 37.   Third, the Certification by the Bureau of Fire Protection states that the fire
64.   The third witness is Cesar Martinez, Officer of Metrobank, who said that was accidental in origin. This Certification enjoys the presumption of
UMC opened letters of credit with it for 1995 only regularity, which Country failed to rebut.
65.   The fourth witness presented was Ernesto Luna (Luna), the delivery 38.   In Uy Hu & Co. v. The Prudential Assurance Co., Ltd.,
checker of Straight Commercial Cargo Forwarders. Luna affirmed the a.   the Court held that where a fire insurance policy provides that if
delivery of UMCs goods to its warehouse the claim be in any respect fraudulent, or if any false declaration be
66.   . Lastly, CRMs adjuster Dominador Victorio testified that he inspected made or used in support thereof, or if any fraudulent means or
UMCs warehouse and prepared preliminary reports in this connection. devices are used by the Insured or anyone acting on his behalf
67.   Country rpesented Cabrera and Lazar who testified that they were hired to to obtain any benefit under this Policy, and the evidence is
investigate and found that arson was committed based from the interview conclusive that the proof of claim which the insured submitted
with barangay officials and pictures that showed blackened surfaces were was false and fraudulent both as to the kind, quality and
present at different parts of the warehouse. amount of the goods and their value destroyed by the fire, such
68.   RTC ruled in favor of UMC. The CA ruled in favor of Country. The CA a proof of claim is a bar against the insured from recovering on
said that the claim under the policy was void and that the fire was the policy even for the amount of his actual loss.
intentional in origin and that the claim was overvalued through fraudulent 39.   In this case, proof of its loss of stocks in trade amounting to
transactions. P50,000,000.00, UMC submitted its Sworn Statement of Formal Claim
together with the following documents:
ISSUE/s: a.   letters of credit and invoices for raw materials, Christmas lights
10.   WoN UMC can claim payment from Country– NO, because it committed and cartons purchased;
fraud which is a breach of the Policy. Commission of fraud voids the policy. b.   charges for assembling the Christmas lights; and
c.   delivery receipts of the raw materials.
RULING: Judgment in question is affirmed. 40.   However, the charges for assembling the Christmas lights and delivery
receipts could not support its insurance claim. The Insurance Policy
RATIO: provides that Country agreed to insure UMCs stocks in trade. UMC
31.   The party who asserts affirmative of the ossue has the burden of proof to defined stock in trade as tangible personal property kept for sale or
obtain a favorable judgment. Particularly, in insurance cases, once an traffic.Applying UMCs definition, only the letters of credit and invoices
insured makes out a prima facie case in its favor, the burden of evidence for raw materials, Christmas lights and cartons may be considered.
41.   The invoices, however, cannot be taken as genuine. The invoices reveal that
the stocks in trade purchased for 1996 amounts to P20,000,000.00 which
were purchased in one month. Thus, UMC needs to prove purchases
amounting to P30,000,000.00 worth of stocks in trade for 1995 and prior
years.
42.   However, in the Statement of Inventory it submitted to the BIR, UMC
stated that it had no stocks in trade as of 31 December 1995. In its
defense, UMC alleged that it did not include as stocks in trade the raw
materials to be assembled as Christmas lights, which it had on 31
December 1995. However, as proof of its loss, UMC submitted invoices
for raw materials, knowing that the insurance covers only stocks in
trade.
43.   Plus, the invoices from Fuze Industries were suspicious. The purchases
amounted to Php 19, 550 400 worth of Christmas lights. But the testimony
of Cabrera revealed that there was no Fuze Industries located at the business
address appearing in the invoices.
44.   In UMC’s Income Statement, it says that UMC has 1050862.71as assets
and 167, 058.47 as liabilities. It is claiming for more.
45.   Plus, in the certification from the Bureau of Fire Protection, it says that it
estimated a damage of Php 55M to the building and its contents.
008 VALENZUELA v. CA (VARGAS) commission of 32%. However, Valenzuela did not receive his commission
October 19, 1990 | Gutierrez, Jr., J. | Parties to the Contract amounting to P1.6M. During 1976-1978, premium payments were paid
directly to Philamgen.
PETITIONER: Arturo P. Valenzuela and Hospitalita N. Valenzuela 4.   In 1977, Philamgen started to become interested in and expressed its intent
RESPONDENTS: Hon. Court of Appeals, Bienvenido M. Aragon, Robert E. to share in the commission due Valenzuela on a fifty-fifty basis.
Parnell, Carlos K. Catolico, and the Philippine American General Insurance Valenzuela refused. Philamgen insisted on the sharing of commission with
Company, Inc. Valenzuela. Valenzuela firmly reiterated his objection to the proposals of
SUMMARY: Valenzuela is the General Agent for Philamgen (insurance Philamgen.
company). Valenzuela solicited marine insurance from Delta Motors from which 5.   Because of the refusal of Valenzuela, Philamgen took drastic action against
he was entitled to a commission of 32.5% from Philamgen. In 1977, Philamgen Valenzuela:
started to become interested in and expressed its intent to share in the a.   Reversed the commission due him by not crediting in his account the
commission due Valenzuela on a 50-50 basis thus giving Valenzuela an agent’s commission earned from the Delta Motors, Inc.
commission of 16.25%. Despite several attempts to convince Valenzuela, he still b.   Placed agency transactions on a cash and carry basis
refused to accept such proposals to share commission with Philamgen. c.   Threatened the cancellation of policies issued by his agency
Consequently, Philamgen did some acts of harassment against Valenzuela which d.   Started to leak out news that Valenzuela has a substantial account with
led to the decline of Valenzuela’s business as insurance agent. Eventually,
Philamgen.
Philamgen terminated the General Agency Agreement with Valenzuela. The
issues in this case are All of these acts resulted in the decline of his business as insurance agent.
1.   WON Philamgen and/or its officers can be held liable for damages Then on December 27, 1978, Philamgen terminated the General Agency
due to the termination of the General Agency Agreement it entered Agreement of Valenzuela.
into with Valenzuela – YES. The agency involving Valenzuela and 6.   Vaenzuela filed a complaint against Philamgen.
Philamgen is one "coupled with an interest," and, therefore, should 7.   Trial Court ruled in favor of Valenzuela stating that the principal cause of
not be freely revocable at the unilateral will of the latter. The principal the termination of Valenzuela as General Agent of Philamgen was based on
cause of the termination of Valenzuela as General Agent of Philamgen
his refusal to share his Delta Commission. The acts of harassment done by
arose from his refusal to share his Delta Commission. Such termination,
as held by the Trial Court, is tainted with bad faith. As such, Philamgen to force Valenzuela to agree to the sharing of his Delata
2.   WON Valenzuela is liable to Philamgen for the unpaid and commission do not justify the termination of the General Agency
uncollected premiums – NO. The remedy for the non-payment of Agreement entered into by Philamgen and Valenzuela.
premiums is to put an end to and render the insurance policy not 8.   CA ruled in favor of Philamgen stating that the principal’s power to revoke
binding. an agency at will is so pervasive that termination may be effected even if
DOCTRINE: Valenzuela, being a mere agent of Philamgen, is not a party to the
the principal acts in bad faith, subject only to the principal’s liability for
contract of insurance. The contract is essentially between Philamgen and Delta
Motors. Consequently, Valenzuela is under no obligation to pay Philamgen for damages.
the unpaid and uncollected premiums. The remedy for the non-payment of
premiums is to put an end to and render the insurance policy not binding. ISSUE:
FACTS: 1.   WON Philamgen and/or its officers can be held liable for damages due
1.   Characters: to the termination of the General Agency Agreement it entered into
Valenzuela – General Agent of Philamgen with Valenzuela – YES. The agency involving Valenzuela and Philamgen
Philamgen – Insurance Company is one "coupled with an interest," and, therefore, should not be freely
Delta Motors – Insured revocable at the unilateral will of the latter. The principal cause of the
2.   Valenzuela is a General Agent of Philamgen since 1965. As such, he was termination of Valenzuela as General Agent of Philamgen arose from his
authorized to solicit and sell in behalf of Philamgen all kinds of non-life refusal to share his Delta Commission. Such termination, as held by the
insurance, and in sconsideration of services rendered was entitled to receive Trial Court, is tainted with bad faith.
the full agent’s commission of 32.5% from Philamgen. 2.   WON Valenzuela is liable to Philamgen for the unpaid and uncollected
3.   From 1973-1975, Valenzuela solicited marine insurance from one of his premiums – NO. The remedy for the non-payment of premiums is to put an
clients Delta Motors amounting to P4.4M from which he was entitled to a end to and render the insurance policy not binding.
embarrassment, difficulties, and oftentimes frustrations in the solicitation
RULING: ACCORDINGLY, the petition is GRANTED. The impugned decision of and procurement of the insurance policies. To sell policies, an agent exerts
January 29, 1988 and resolution of April 27, 1988 of respondent court are hereby great effort, patience, perseverance, ingenuity, tact, imagination, time and
SET ASIDE. The decision of the trial court dated January 23, 1986 in Civil Case No.
money.
121126 is REINSTATED with the MODIFICATIONS that the amount of FIVE
HUNDRED TWENTY ONE THOUSAND NINE HUNDRED SIXTY-FOUR AND 51.   In the case of Valenzuela, he was able to build up an Agency from scratch
16/100 PESOS (P521,964.16) representing the petitioners Delta commission shall in 1965 to a highly productive enterprise. The records sustain the finding
earn only legal interests without any adjustments under Article 1250 of the Civil that the Philamgen started to covet a share of the insurance business that
Code and that the contractual relationship between Arturo P. Valenzuela and Valenzuela had built up, developed and nurtured to profitability through
Philippine American General Insurance Company shall be deemed terminated upon over thirteen (13) years of patient work and perseverance. When Valenzuela
the satisfaction of the judgment as modified. refused to share his commission in the Delta account, the boom suddenly
fell on him.
RATIO:
46.   The principal cause of the termination of Valenzuela as General Agent of 52.   With the termination of the General Agency Agreement, Valenzuela would
Philamgen arose from his refusal to share his Delta commission. The no longer be entitled to commission on the renewal of insurance policies of
records sustain the conclusions of the trial court on the apparent bad faith of clients sourced from his agency. Worse, despite the termination of the
the Philamgen in terminating the General Agency Agreement of agency, Philamgen continued to hold Valenzuela jointly and severally
Valenzuela. In the case at bar, the records show that the findings and liable with the insured for unpaid premiums. Under these circumstances,
conclusions of the trial court are supported by substantial evidence and it is clear that Valenzuela had an interest in the continuation of the
there appears to be no cogent reason to disturb them. agency when it was unceremoniously terminated not only because of the
47.   As early as September 30,1977, Philamgen told the petitioners of its desire commissions he should continue to receive from the insurance business he
to share the Delta Commission with them. It stated that should Delta back has solicited and procured but also for the fact that by the very acts of the
out from the agreement, Valenzuela would be charged interests through a respondents, he was made liable to Philamgen in the event the insured fail
reduced commission after full payment by Delta. to pay the premiums due. They are estopped by their own positive
48.   On January 23, 1978 Philamgen proposed reducing the Valenzuela’s averments and claims for damages. Therefore, the respondents cannot state
commissions by 50% thus giving them an agent's commission of 16.25%. that the agency relationship between Valenzuela and Philamgen is not
On February 8, 1978, Philamgen insisted on the reduction scheme. There coupled with interest.
were other pressures:
a.   The petitioners were told that the Delta commissions would not be As to Valenzuela’s liability to pay Philamgen for the unpaid and uncollected
credited to their account premiums
b.   They were informed that the Valenzuela agency would be placed on a 53.   We rule that the respondent court erred in holding Valenzuela liable. We
cash and carry basis thus removing the 60-day credit for premiums due find no factual and legal basis for the award. Under Section 7712 of the
c.   Existing policies were threatened to be cancelled Insurance Code, the remedy for the non-payment of premiums is to put an
d.   The Valenzuela business was threatened with diversion to other agencies end to and render the insurance policy not binding.
e.   Rumors were also spread about alleged accounts of the Valenzuela 54.   Philippine Phoenix Surety and Insurance, Inc. v. Woodworks, Inc.: Non-
agency payment of premium does not merely suspend but puts an end to an
49.   It is also evident from the records that the agency involving Valenzuela and
                                                                                                                       
Philamgen is one "coupled with an interest," and, therefore, should not be 12
Sec. 77 ... [N]otwithstanding any agreement to the contrary, no policy or contract of insurance is valid
freely revocable at the unilateral will of the latter. and binding unless and until the premiums thereof have been paid except in the case of a life or industrial
50.   In the insurance business in the Philippines, the most difficult and life policy whenever the grace period provision applies (P.D. 612, as amended otherwise known as the
Insurance Code of 1974)
frustrating period is the solicitation and persuasion of the prospective clients
to buy insurance policies. Normally, agents would encounter much
 
insurance contract since the time of the payment is peculiarly of the essence
of the contract.
55.   Since admittedly the premiums have not been paid, the policies issued have
lapsed. The insurance coverage did not go into effect or did not continue
and the obligation of Philamgen as insurer ceased. Hence, for Philamgen
which had no more liability under the lapsed and inexistent policies to
demand, much less sue Valenzuela for the unpaid premiums would be the
height of injustice and unfair dealing. In this instance, with the lapsing of
the policies through the nonpayment of premiums by the insured there
were no more insurance contracts to speak of.
009 RCBC v. CA (VICENCIO) FACTS:
April 20, 1998 | Melo, J. | Parties to a Contract 43.   Parties: Insured: GOYU; Insurer: MICO thru its agent Alchester;
Mortgagor: GOYU; Mortgagee: RCBC
PETITIONER: Rizal Commercial Banking Corporation (RCBC), Uy Chun Bing, Eli D. Law 44.   Goyu & Sons (GOYU) applied for credit facilities and accomodations with
RESPONDENTS: Court of Appeals and Goyu & Sons Rizal Commercial Banking Corporation (RCBC) at its Binondo Branch.
45.   After due evaluation, RCBC, through its key officers, petitioner Uy Chin
SUMMARY: Goyu and Sons (GOYU) applied for credit facilities with RCBC. A credit Bing and Eli D. Lao, recommended GOYU’s application for approval with
amount of P30M was granted, which was incrased to P117M. As security for its credit, GOYU RCBC’s executive committee.
executed two real estate mortagages and two chattel mortgages, in favor of RCBC. Under each 46.   A credit facilitiy in the amount of P30 million was initially granted. Upon
of these contracts, GOYU committed itself to insure the mortgages property with an insurance GOYU’s application and Uy’s and Lao’s recommendation, RCBCs
company approved by RCBC. GOYU obtained a total of 10 insurance policies from Malayan
Insurance Compan (MICO). Alchester Insurance Company (Alchester), MICO’s insurance
executive committee increased GOYUs credit facility to P50 million, then
agent, issued nine endorsements in favor of RCBC. On April 27, 1992, one of GOYU’s to P90 million, and finally to P117 million.
factory buildings in Valenzuela was gutted by fire. GOYU submitted its claim for indemnity 47.   As security for its credit facilities with RCBC, GOYU executed two real
against MICO. MICO however denied the claim on the ground that the insurance policies estate mortgages and two chattel mortgages in favor of RCBC.
were already subject to attachments by other courts because the insurance proceeds were also 48.   Under each of these four mortgage contracts, GOYU committed itself to
claimed by other creditors of GOYU. insure the mortgaged property with an insurance company approved by
RCBC, and subsequently, to endorse and deliver the insurance policies to
RCBC, one of GOYU’s creditors, also filed with MICO its formal claim over the proceeds of RCBC.
the insurance policies. RCBC here is claiming that since they are the mortgagee of security 49.   GOYU obtained in its name a total of ten insurance policies from Malayan
provided by GOYU, the insured, they also have the right to claim the insurance proceeds from
MICO, the insurer. Its claim was also denied by MICO, however. The RTC ruled in favor of
Insurance Company (MICO). Alchester Insurance Agency, Inc. (Alchester),
GOYU, ordering MICO to pay GOYU its fire loss claims, but that GOYU should also pay the insurance agent where GOYU obtained the MICO policies, issued nine
RCBC for its loan obligations with the latter. CA partly affirmed. endorsements in favor of RCBC seemingly upon instructions of GOYU.
50.   On April 27, 1992, one of GOYU’s factory buildings in Valenzuela was
The issue before the SC is WoN RCBC, as mortgagee, has any right over the insurance gutted by fire.
policies by GOYU, as mortgagor, in case of occurrence of loss. The SC held that YES, but 51.   Consequently, GOYU submitted its claim for indemnity on account of the
only because of the peculiarity of the circumstances obtaining in the instant case which loss insured against MICO.
presented a justification to take exception to the strict application of Sec. 53 of the Insurance 52.   MICO denied the claim on the ground that the insurance policies were
Code. GOYU voluntarily procured insurance policies to cover the mortgaged property from
either attached pursuant to writs of attachments/garnishments issued by
MICO, no less than a sister company of RCBC. Endorsement documents were prepared by
MICO’s underwriter, Alchester Insurance Agency, Inc., and copies thereof were sent to
various courts or that the insurance proceeds were also claimed by other
GOYU, MICO, and RCBC. GOYU did not assail, until of late, the validity of said creditors of GOYU alleging better rights to the proceeds than the insured.
endorsements. GOYU continued until the occurrence of the fire, to enjoy the benefits of the 53.   GOYU filed a complaint for specific performance and damages with the
credit facilities extended by RCBC. GOYU is bound by the principle of estoppel. The RTC.
intention of the parties was also plain: RCBC is truly the person or entity for whose benefit the 54.   RCBC, one of GOYU’s creditors, also filed with MICO its formal claim
policies were clearly intended. over the proceeds of the insurance policies, but said claims were also denied
for the same reasons that MICO denied GOYU’s claims.
Being exclusively payable to RCBC by reason of the endorsement by Alchester to RCBC, 1.   Personal note: RCBC here is claiming that since they are the
these 8 policies (of 10) can not be attached by GOYU’s other creditors up to the extent of the mortgagee of security provided by GOYU, the insured, they also
GOYUs outstanding obligation in RCBC’s favor. The other 2 policies, since they have not
been proven to be endorsed and delivered by Alchester, can be subject of attachments by other
have the right to claim the insurance proceeds from MICO, the
creditors. insurer
55.   The RTC confirmed that GOYU’s other creditors, namely, Urban Bank,
DOCTRINE: Section 53 of the Insurance Code ordains that the insurance proceeds of the Alfredo Sebastian, and Philippine Trust Company obtained their respective
endorsed policies shall be applied exclusively to the proper interest of the person for whose writs of attachments form various cours, covering an aggregate ammoung of
benefit it was made. P14,938,080, and ordered that the proceeds of the ten insurance policies be
deposited with the said court minus the aforementioned 14M.
56.   Accordingly, MICO deposited the amoun gog P50,505,594 with the RTC.
57.   The RTC rendered judgment in favor of GOYU:
1.   For defendant MICO to pay GOYU its fire loss claims in the total of P74,040,518, 69.   In the present case, although it appears that GOYU obtained the subject
less the amount of P50M which is deposited with this Court.
insurance policies naming itself as the sole payee, the intentions of the
2.   To pay MICO damages by way of interest for the delay
3.   For RCBC to pay GOYU actual damages of 2PM parties as shown by their contemporaneous acts, must be given due
4.   For the both MICO and RCBC to pay GOYU exemplary and attorney’s fees. consideration in order to better serve the interest of justice and equity.
5.   And on the counterclaim of RCBC: ordering GOYU to pay its loan obligations with 70.   It is to be noted that nine endorsement documents were prepared by
RCBC in the amoung of P68,785,069, with interest.
Alchester in favor of RCBC. The Court is in a quandary how Alchester
58.   All parties appealed. GOYU was unsatisfied with the amounts awarded in could arrive at the idea of endorsing any specific insurance policy in
its favor. MICO and RCBC disputed the trial courts findings of liability on favor of any particular beneficiary or payee (RCBC) other than the
their part. insured (GOYU) had not such named payee or beneficiary been
59.   The CA partly granted GOYUs appeal (it additionally ordered MICO to pay specifically disclosed by the insured itself.
GOYU damages by way of interest until GOYU receives the said amount), 71.   It is also significant that GOYU voluntarily and purposely took the
but sustained the findings of the trial court with respect to MICO and insurance policies from MICO, a sister company of RCBC, and not just
RCBCs liabilities. from any other insurance company.
60.   Hence this appeal by RCBC and MICO. 72.   Had it not been for GOYU, Alchester would not have known of GOYU’s
intention of obtaining insurance coverage in compliance with its
ISSUE/s: undertaking in the mortgage contracts with RCBC, and verily, Alchester
7.   WoN RCBC, as mortgagee, has any right over the insurance policies taken would not have endorsed the policies to RCBC had it not been so directed
by GOYU, the mortgagor, in case of the occurrence of loss – YES, but only by GOYU.
because of the peculiarity of the circumstances obtaining in the instant case 73.   On equitable principles, particularly on the ground of estoppel, the Court is
which presented a justification to take exception to the strict application of constrained to rule in favor of mortgagor RCBC.
Sec. 53 of the Insurance Code, it having been sufficiently established that it 74.   Evelyn Lozada of Alchester testified that upon instructions of Mr. Go,
was the intention of the parties to designate RCBC as the party for whose through a certain Mr. Yam, she prepared in quadruplicate on February 11,
benefit the insurance policies were taken out. 1992 the nine endorsement documents for GOYUs nine insurance policies
in favor of RCBC.
RULING: Petitions are hereby GRANTED and the decision and resolution of 75.   RCBC, in good faith, relied upon the endorsement documents sent to it
December 16, 1996 and April 3, 1997 in CA-G.R. CV No. 46162 are hereby as this was only pursuant to the stipulation in the mortgage contracts.
REVERSED and SET ASIDE. A new one entered: The Court finds such reliance to be justified under the circumstances of
1.   Dismissing the Complaint of private respondent GOYU in Civil Case No. 93-65442 before
Branch 3 of the Manila Regional Trial Court for lack of merit; the case.
2.   Ordering Malayan Insurance Company, Inc. to deliver to Rizal Commercial Banking 76.   GOYU failed to seasonably repudiate the authority of the person or
Corporation the proceeds of the insurance policies in the amount of P51,862,390.94 (per report persons who prepared such endorsements.
of adjuster Toplis & Harding (Far East), Inc., Exhibits 2 and 2-1), less the amount of 77.   Over and above this, GOYU continued, in the meantime, to enjoy the
P50,505,594.60 (per O.R. No. 3649285);
3.   Ordering the Clerk of Court to release the amount of P50,505,594.60 including the interests benefits of the credit facilities extended to it by RCBC.
earned to Rizal Commercial Banking Corporation; 78.   After the occurrence of the loss insured against, it was too late for
4.   Ordering Goyu & Sons, Inc. to pay its loan obligation with Rizal Commercial Banking GOYU to disown the endorsements for any imagined or contrived lack
Corporation in the principal amount of P107,246,887.90, with interest at the respective rates
of authority of Alchester to prepare and issue said endorsements.
stipulated in each promissory note from January 21, 1993 until finality of this judgment, and
surcharges at 2% and penalties at 3% from January 21, 1993 to March 9, 1993, minus 79.   If there had not been actually an implied ratification of said endorsements
payments made by Malayan Insurance Company, Inc. and the proceeds of the amount by virtue of GOYUs inaction in this case, GOYU is at the very least
deposited with the trial court and its earned interest. The total amount due RCBC at the time of estopped from assailing their operative effects.
the finality of this judgment shall earn interest at the legal rate of 12% in lieu of all other
80.   To permit GOYU to capitalize on its non-confirmation of these
stipulated interests and charges until fully paid
endorsements while it continued to enjoy the benefits of the credit
RATIO: facilities of RCBC which believed in good faith that there was due
67.   It is settled that a mortgagor and a mortgagee have separate and distinct endorsement pursuant to their mortgage contracts, is to countenance
insurable interests in the same mortgaged property, such that each one of grave contravention of public policy, fair dealing, good faith, and
them may insure the same property for his own sole benefit. justice.
68.   There is no question that GOYU could insure the mortgaged property for its 81.   Under the peculiar circumstances obtaining in this case, the Court is bound
own exclusive benefit. to recognize RCBC’s right to the proceeds of the insurance policies if not
for the actual endorsement of the policies, at least on the basis of the use, with the declarations, amplifications and limitations established by law,
whether the estate remains in the possession of the mortgagor, or it passes into the
equitable principle of estoppel.
hands of a third person.
82.   GOYU cannot seek relief under Section 53 of the Insurance Code which 88.   Significantly, the Court notes that out of the 10 insurance policies subject of
provides that the proceeds of insurance shall exclusively apply to the this case, only 8 of them appear to have been subject of the endorsements
interest of the person in whose name or for whose benefit it is made. The prepared and delivered by Alchester for and upon instructions of GOYU.
peculiarity of the circumstances obtaining in the instant case presents a 1.   Policy Number F-114-07795 has not been endorsed. This fact was
justification to take exception to the strict application of said provision, it admitted by MICOs witness, Atty. Farolan.
having been sufficiently established that it was the intention of the parties to 2.   Likewise, the record shows no endorsement for Policy Number
designate RCBC as the party for whose benefit the insurance policies were CI/F-128-03341.
taken out. 89.   Being exclusively payable to RCBC by reason of the endorsement by
83.   The Court considered the following: Alchester to RCBC, which we already ruled to have the force and effect
1.   It is undisputed that the insured pieces of property were the subject of
of an endorsement by GOYU itself, these 8 policies can not be attached
mortgage contracts entered into between RCBC and GOYU in
consideration of and for securing GOYUs credit facilities from by GOYU’s other creditors up to the extent of the GOYUs outstanding
RCBC. The mortgage contracts contained common provisions whereby obligation in RCBC’s favor.
GOYU, as mortgagor, undertook to have the mortgaged property properly 90.   Section 53 of the Insurance Code ordains that the insurance proceeds of
covered against any loss by an insurance company acceptable to RCBC. the endorsed policies shall be applied exclusively to the proper interest
2.   GOYU voluntarily procured insurance policies to cover the mortgaged of the person for whose benefit it was made.
property from MICO, no less than a sister company of RCBC and 91.   In this case, to the extent of GOYUs obligation with RCBC, the interest of
definitely an acceptable insurance company to RCBC. GOYU in the subject policies had been transferred to RCBC effective
3.   Endorsement documents were prepared by MICOs underwriter, Alchester as of the time of the endorsement.
Insurance Agency, Inc., and copies thereof were sent to GOYU, MICO,
92.   These policies may no longer be attached by the other creditors of GOYU,
and RCBC. GOYU did not assail, until of late, the validity of said
endorsements. like Alfredo Sebastian, which may nonetheless forthwith be dismissed for
4.   GOYU continued until the occurrence of the fire, to enjoy the benefits of being moot and academic in view of the results reached herein.
the credit facilities extended by RCBC which was conditioned upon the 93.   Only the two other policies may be validly attached, garnished, and levied
endorsement of the insurance policies to be taken by GOYU to cover the upon by GOYUs other creditors.
mortgaged properties. 94.   To the extent of GOYUs outstanding obligation with RCBC, all the rest of
84.   This Court can not over stress the fact that upon receiving its copies of the the other insurance policies above-listed which were endorsed to RCBC,
endorsement documents prepared by Alchester, GOYU, despite the are, therefore, to be released from attachment, garnishment, and levy by the
absence of its written conformity thereto, obviously considered said other creditors of GOYU.
endorsement to be sufficient compliance with its obligation under the 95.   Given the factual milieu spread hereover, the Court also rulse that it was
mortgage contracts since RCBC accordingly continued to extend the error to hold MICO liable in damages for denying or withholding the
benefits of its credit facilities and GOYU continued to benefit proceeds of the insurance claim to GOYU:
therefrom. 1.   Firstly, by virtue of the mortgage contracts as well as the endorsements of
85.   Just as plain too is the intention of the parties to constitute RCBC as the the insurance policies, RCBC has the right to claim the insurance
beneficiary of the various insurance policies obtained by GOYU. proceeds, in substitution of the property lost in the fire. Having assigned
86.   The intention of the parties will have to be given full force and effect in this its rights, GOYU lost its standing as the beneficiary of the said insurance
particular case. The insurance proceeds may, therefore, be exclusively policies.
applied to RCBC, which under the factual circumstances of the case, is 2.   Secondly, for an insurance company to be held liable for unreasonably
delaying and withholding payment of insurance proceeds, the delay must
truly the person or entity for whose benefit the policies were clearly be wanton, oppressive, or malevolent. The case at bar does not show that
intended. MICO wantonly and in bad faith delayed the release of the proceeds. The
87.   Moreover, the laws evident intention to protect the interests of the problem in the determination of who is the actual beneficiary of the
mortgagee upon the mortgaged property is expressed in Article 2127 of the insurance policies, aggravated by the claim of various creditors who
Civil Code which states: wanted to partake of the insurance proceeds, not to mention the
1.   ART. 2127. The mortgage extends to the natural accessions, to the improvements, importance of the endorsement to RCBC, to our mind, and as now borne
growing fruits, and the rents or income not yet received when the obligation out by the outcome herein, justified MICO in withholding payment to
becomes due, and to the amount of the indemnity granted or owing to the proprietor GOYU.
from the insurers of the property mortgaged, or in virtue of expropriation for public
010 PALILEO v. COSIO (VILLAVICENCIO) 4.   This document did not express the true intention of the parties which was merely
November 28, 1955 | Bautista Angelo, J. | Parties to the contract to place said property as security for the payment of the loan.
5.   Cosio insured the building against fire with the Associated Insurance & Surety
PETITIONER: Cherie Palileo Co., Inc. for the sum of P15,000, the insurance policy having been issued in the
RESPONDENTS: Beatriz Cosio name of Cosio.
6.   The building was partly destroyed by fire and, after proper demand, Cosio
SUMMARY: Palileo obtained a loan from Cosio. Cosio required Palileo to sign a collected from the insurance company an indemnity of P13,107.00.
“Conditional sale of residential building” to secure the payment of the loan. This 7.   Palileo demanded from Cosio that she be credited with the necessary amount to
document did not express the true intention of the parties that was merely to place pay her obligation out of the insurance proceeds but Cosio refused to do so.
said property as security for the payment of the loan. Cosio insured the building with 8.   Trial court rendered a decision ordering Cosio to credit the 13,107 to Palileo’s
Associated Insurance & Surety Co., Inc. The building was party destroyed by fire. payment to her.
Cosio then collected 13,107 from Associated Insurance. Palileo demanded from
Cosio that she be credited with the necessary amount to pay her obligation out of the ISSUE/s:
insurance proceeds but Cosio refused to do so. Trial court rendered a decision 1.   WoN the obligation of Palileo can be fully compensated by the insurance
ordering Cosio to credit the 13,107 to Palileo’s payment to her. WoN the obligation amount – NO, because it runs counter to the rule governing an insurance taken
of Palileo can be fully compensated by the insurance amount – NO, because it runs by a mortgagee independently of the mortgagor.
counter to the rule governing an insurance taken by a mortgagee independently of the
mortgagor. In that event, upon the destruction of the property the insurance money RULING: Consistent with the foregoing pronouncement, we therefore modify the
paid to the mortgagee will not inure to the benefit of the mortgagor, and the amount judgment of the lower court as follows:(1) the transaction had between the plaintiff
due under the mortgage debt remains unchanged. The mortgagee, however, is not and defendant as shown in Exhibit A is merely an equitable mortgage intended to
allowed to retain his claim against the mortgagor, but it passes by subrogation to the secure the payment of the loan of P12,000;(2) that the proceeds of the insurance
insurer, to the extent of the insurance money paid. amounting to P13,107.00 was properly collected by defendant who is not required to
account for it to the plaintiff; (3) that the collection of said insurance proceeds shall
DOCTRINE: The rule is that "where a mortgagee, independently of the mortgagor, not be deemed to have compensated the obligation of the plaintiff to the defendant,
insures the mortgaged property in his own name and for his own interest, he is but bars the latter from claiming its payment from the former; and (4) defendant shall
entitled to the insurance proceeds in case of loss, but in such case, he is not allowed pay to the plaintiff the sum of P810.00 representing the overpayment made by
to retain his claim against the mortgagor, but is passed by subrogation to the insurer plaintiff by way of interest on the loan. No pronouncement as to costs.
to the extent of the money paid." Or, stated in another way, "the mortgagee may
insure his interest in the property independently of the mortgagor. RATIO:
96.   The rule is that "where a mortgagee, independently of the mortgagor, insures the
mortgaged property in his own name and for his own interest, he is entitled to
the insurance proceeds in case of loss, but in such case, he is not allowed to
FACTS: retain his claim against the mortgagor, but is passed by subrogation to the
1.   Palileo obtained from Cosio a loan in the sum of P12,000 subject to the
insurer to the extent of the money paid."
following conditions: (a) that Palileo shall pay to Cosio an interest in the amount
97.   Or, stated in another way, "the mortgagee may insure his interest in the property
of P250 a month; (b) that Cosio shall deduct from the loan certain obligations of
independently of the mortgagor.
Palileo to third persons amounting to P4,550, plus the sum of P250 as interest
98.   In that event, upon the destruction of the property the insurance money paid to
for the first month; and (c) that after making the above deductions, Cosio shall
the mortgagee will not inure to the benefit of the mortgagor, and the amount due
deliver to Palileo only the balance of the loan of P12,000.
under the mortgage debt remains unchanged.
2.   Pursuant to their agreement, Palileo paid to Cosio as interest on the loan a total
99.   The mortgagee, however, is not allowed to retain his claim against the
of P2,250.00 corresponding to nine months from December 18, 1951, on the
mortgagor, but it passes by subrogation to the insurer, to the extent of the
basis of P250.00 a month, which is more than the maximum interest authorized
insurance money paid."
by law.
100.  This is the same rule upheld by this Court in a case that arose in this jurisdiction.
3.   To secure the payment of the aforesaid loan, Cosio required Palileo to sign a In the case mentioned, an insurance contract was taken out by the mortgagee
document known as "Conditional Sale of Residential Building", purporting to
upon his own interest, it being stipulated that the proceeds would be paid to him
convey to Cosio, with right to repurchase, a two-story building of strong
only and when the case came up for decision, this Court held that the mortgagee,
materials belonging to Palileo.
in case of loss, may only recover upon the policy to the extent of his credit at the
time of the loss. It was declared that the mortgaged had no right of action against
the mortgagee on the policy. (San Miguel Brewery vs. Law Union, 40 Phil.,
674.)
101.  The general rule and the weight of authority is, that the insurer is thereupon
subrogated to the rights of the mortgagee under the mortgage.
102.  the correct solution should be that the proceeds of the insurance should be
delivered to Cosio but that her claim against Palileo should be considered
assigned to the insurance company who is deemed subrogated to the rights of
Cosio to the extent of the money paid as indemnity.
011 GREAT PACIFIC LIFE ASSURANCE v. CA (Yap) (Grepalife).
October 13, 1999 | Quisumbing, J. | Parties to the contract

PETITIONER: Great Pacific Life Assurance Corp. (Grepalife) FACTS:


RESPONDENT: Court of Appeals and Medarda V. Leuterio 69.   Great Pacific Life Assurance (Petitioner / Grepalife) and the Development
Bank of the Philippines (DBP) entered into a Group Life Insurance to
SUMMARY: Grepalife and DBP entered into a group life insurance to insure insure the lives of eligible housing loan mortgagors of DBP (a.k.a.,
the lives of eligivle housing loan mortgagors of DBP (a.k.a., mortgage Mortgage Redemption Insurance).
redemption insurance). When Dr. Wilfredo Leuterio applied, he indicated that he 70.   Dr. Wilfredo Leuterio, a physicial and a housing debtor of DBP, applied for
had never consulted a physician for a heart condition, high blood pressure, etc. membership in the group life insurance.
and that to the best of his knowledge, he is in good health. Thereafter, he died a.   In his application, he answered that (1) “He had NEVER consulted
due to massive cerebral hemorrhage. When DBP submitted its claim, from the a physician for a heart condition, high blood pressure, cancer,
insurance, to Grepalife, the latter denied on the ground that it is entitled to diabetes, lung, kidney or stomach disorder or any other physical
rescind the contract since Wilfredo disclosed his hypertension, as attested by Dr. impairment;” and (2) “He was, to the best of his knowledge, in
Hernando Mejia that “the cerebral hemorrhage was probably secondary to good health.”
hypertension.” Because of the denial, DBP proceeded to claim from Wilfredo b.   Grepalife granted Wilfredo an insurance coverage of PHP86,200
and even foreclosed on the mortgaged property. Medarda Leuterio, the widow, over his DBP mortgage indebtedness.
filed a complaint for specific performance and damages against Grepalife to 71.   Wilfredo died due to massive cerebral hemorrhage.
fulfill its obligation under the insurance contract. Grepalife now argues that 72.   DBP submitted a death claim to Grepalife.
Medarda is not the real party-in-interest, which should be DBP the mortgagee. a.   Grepalife denied the claim on the ground that Wilfredo was not
physically healthy when he applied for an insurance coverage, and
The pertinent issue is whether Medarda is the real party-in-interest. that he concealed the fact that he ws suffering from hypertension,
which caused his death.
The SC ruled in the affirmative. In Gonzales v. Yek Tong Lin, the SC ruled that b.   According to Grepalife, this was attested to the attending
the insured may be regarded as the real party-in-interest, ALTHOUGH he has physician, Dr. Hernando Mejia, when he said that Wilfredo “died
assigned the policy for the purpose of collection, or has assigned as collateral of cerebral hemorrhage, PROBABLY secondary to hypertension.”
security any judgment he may obtain. In this case, since a policy of insurance c.   Thus, DBP collected the debt from the mortgagor (Wilfredo) and
upon life or health MAY PASS by transfer, will or succession to any person, took the necessary action of foreclosure on the residential lot of the
whether he has an insurable interest or not, and such person may recover it Leuterios.
whatever the insured might have recovered, the widow (Medarda) of the 73.   Medarda Leuterio, Wilfredo’s widow, filed this complaint against Grepalife
decedent may file the suit against the insurer (Grepalife). (To simplify, Wilfredo for specific performance with damages, alleging that Grepalife’s claim
remained as the real party-in-interest even though he assigned the DBP based on Dr. Mejia’s testimony was not a conclusive inference because
mortgage indebtedness to Grepalife. By virtue of Wilfredo’s death, his heirs Wilfredo was not autopsied, hence other causes could not be ruled out.
succeeded to his rights in the insurance contract). Hence, Medarda has the a.   Trial court ruled in favor of Medarda. CA sustained.
proper standing to claim from Grepalife, which erred in rescinding the insurance 74.   Grepalife now alleges that this complaint was not filed by the real party-in-
contract on the ground of Wilfredo’s alleged concealment – there was no interest which is DBP.
concealment since the testimony of Dr. Mejia, as to the “probability” that the
cerebral hemorrhage was secondary to hypertension, was not conclusive proof ISSUE/s:
that Wilfredo was not in good health when he applied for the insurance policy. 11.   Whether Medarda is a real party-in-interest – YES, because Wilfredo
There was no fraudulent intent established. Now, the rightful owners of the remained the real party-in-interest despite the mortgage, and when he died,
proceeds are Wilfredo’s heirs. his heirs succeeded thereto.
12.   Whether Grepalife may rescind the group life insurance of Wilfredo – NO,
DOCTRINE: Since a policy of insurance upon life or health MAY PASS by because there is no conclusive proof of fraudulent intent since the testimony
transfer, will or succession to any person, whether he has an insurable interest or of Dr. Mejia merely mentioned that his cerebral hemorrhage was
not, and such person may recover it whatever the insured might have recovered, “probably” secaondary to hypertension.
the widow (Medarda) of the decedent may file the suit against the insurer
RULING: Petition DENIED. CA decision AFFIRMED with MODIFICATION that c.   Misrepresentation as a defense of the insurer to avoid liability is an
Grepalife is ORDERED to pay PHP86,200 insurance proceeds to the heirs of the affirmative defense.
insured. i.   The duty to establish such defense by satisfactory and
RATIO: convincing evidence rests upon the insurer.
56.   Medarda is the real party-in-interest. 58.   Since there was no evidence on record as to the amount of Wilfredo’s
a.   Nature of a Mortgage Redemption Insurance contract: outstanding indebtedness to DBP at the time of his death, the measure of
i.   In favor of the mortgagee (DBP) – in the event of the indemnity under a policy of insurance upon life or health is the sum fixed in the
unexpected demise of the mortgagor during the substinence of policy (here, it’s PHP86,200).
the mortgage contract, the proceeds from such insurance will 59.   Considering that DBP already foreclosed on the residential lot subject of the
be applied to the payment of the mortgage debt, thereby mortgage, it cannot unjustly enrich itself by collecting the insurance proceeds.
relieving the heirs of the mortgagor from paying the a.   The proceeds now rightly belong to Wilfredo’s heirs.
obligation.
ii.   In favor of the mortgagor (Wilfredo) – in the event of death,
the mortgage obligation will be extinguished by the
application of the insurance proceeds to the mortgage
indebtedness.
iii.   Where the mortgagor pays the insurance premium under the
group insurance policy, making the loss payable to the
mortgagee, the insurance is on the mortgagor’s interest, and
the mortgagor continues to be a party to the contract. Here the
mortgagee is simply an appointee of the insurance fund,
such loss-payable clause does not make the mortgagee a
party to the contract.
b.   §8, Insurance Code – In the event of the debtor’s (Wilfredo’s) death
before his indebtedness with the creditor (DBP) shall have been fully
paid, an amount to pay the outstanding indebtedness shall first be paid
to the creditor and the balance of sum assured, if there is any, shall then
be paid to the beneficiary/ies designated by the debtor.
c.   Gonzales La O v. Yek Tong Lin – the insured may be regarded as the
real party-in-interest, ALTHOUGH he has assigned the policy for the
purpose of collection, or has assigned as collateral security any
judgment he may obtain.
i.   Since a policy of insurance upon life or health MAY PASS by
transfer, will or succession to any person, whether he has an
insurable interest or not, and such person may recover it
whatever the insured might have recovered, the widow
(Medarda) of the decedent may file the suit against the insurer
(Grepalife).
57.   Grepalife may not rescind the group life insurance since Wilfredo did not
conceal the status of his health.
a.   The certification of Dr. Mejia was that Wilfredo “died of cerebral
hemorrhage, PROBABLY secondary to hypertension.” Hence, contrary
to Grepalife’s allegations, there was no sufficient proof that the
Wilfredo had suffered from hypertension.
b.   To entitle Grepalife to rescind the contract, the fraudulent intent on the
part of Wilfredo must be established.
001 EL ORIENTE FABRICA DE TABACOS, INC. v. POSADAS (Adrias) 3.   El Oriente, in order to protect itself against the loss that it might suffer by
September 21, 1931 | Malcolm, J. | Life Insurance Proceeds – Taxable or Not reason of the death of its manager A. Velhagen, who had more than 35
years of experience in the manufacture of cigars in the Philippines and
PLAINTIFF-APPELLANT: El Oriente Fabrica De Tabacos, Inc. whose death would be a serious loss to El Oriente, procured from
DEFENDANT-APPELLEE: Juan Posadas, Collector of Internal Revenue Manufacturers Life Insurance Co. of Toronto, Canada through its local
agent R.R. Elser (Manulife) an insurance policy on the life of A. Velhagen
Insurer: Manufacturers Life Insurance Co., of Toronto, Canada, thru its local agent for the sum of 50,000 USD.
E.E. Elser 4.   El Oriente designated itself as the sole beneficiary. The insured has no
Insured: A. Velhagen (Manager of El Oriente) interest or participation in the proceeds of said life insurance policy.
Beneficiary: El Oriente Fabrica de Tabacos, Inc. 5.   El Oriente charged as expenses of its business all the said premiums and
deducted the same from its gross incomes as reported in its annual income
SUMMARY: El Oriente, in order to protect itself against the loss that it might tax returns, which deductions were allowed by Posadas upon showing by El
suffer by reason of the death of its manager A. Velhagen, who had more than 35 Oriente that such premiums were legitimate expenses of the business.
years of experience in the manufacture of cigars in the Philippines and whose death 6.   Upon the death of Manager A. Velhagen, El Oriente received all the
would be a serious loss to El Oriente, procured from Manufacturers Life Insurance proceeds of the life insurance policy together with the interest and the
Co. of Toronto, Canada through its local agent R.R. Elser (Manulife) an insurance dividends accruing thereon, aggregating P104,957.88.
policy on the life of A. Velhagen for the sum of 50,000 USD. El Oriente designated 7.   Posadas assessed and levied the sum of P3,148.74 as income tax on the
itself as the sole beneficiary. El Oriente charged as expenses of its business all the proceeds of the insurance policy, which was paid by El Oriente under
said premiums and deducted the same from its gross incomes as reported in its protest.
annual income tax returns. Upon the death of Manager A. Velhagen, El Oriente 8.   RTC Decision: Ruled in favor of Posadas
received all the proceeds of the life insurance policy. Posadas assessed and levied the 9.   El Oriente’s contentions:
sum of P3,148.74 as income tax on the proceeds of the insurance policy, which was a.   That trial court erred in holding that section 4 of the Income Tax
paid by El Oriente under protest. The issue in this case is WoN the proceeds of Law (Act No. 2833) is not applicable to the present case.
insurance taken by a corporation on the life of an important official to indemnify it b.   The trial court erred in reading into the law certain exceptions and
against loss in case of his death, are taxable as income under the Philippine Income distinctions not warranted by its clear and unequivocal provisions.
Tax Law? The Court ruled in the negative. Act No. 2833 “Income Tax Law for the c.   The trial court erred in assuming that the proceeds of the life
Philippines” provides in Section 4, Chapter I on Individuals that life insurance insurance policy in question represented a net profit to El Oriente
proceeds are exempt from Income Tax. However, no provisions in Chapter II on when, as a matter of fact, it merely represented an indemnity, for
Corporations explicitly provide for such exemption. Considering, however, the the loss suffered by it thru the death of its manager, the insured.
purport of the stipulated facts, considering the uncertainty of Philippine law, and d.   The trial court erred in refusing to hold that the proceeds of the life
considering the lack of express legislative intention to tax the proceeds of life insurance policy in question is not taxable income.
insurance policies paid to corporate beneficiaries, particularly when in the exemption
in favor of individual beneficiaries in the chapter on this subject, the clause is ISSUE/s:
inserted "exempt from the provisions of this law," we deem it reasonable to hold 1.   WoN the proceeds of insurance taken by a corporation on the life of an
the proceeds of the life insurance policy in question as representing an important official to indemnify it against loss in case of his death, are
indemnity and not taxable income. taxable as income under the Philippine Income Tax Law? – NO.
Considering, therefore, the purport of the stipulated facts, considering
DOCTRINE: El Oriente, as a corporation, can purchase a life insurance policy on an the uncertainty of Philippine law, and considering the lack of express
employee and it can designate itself as the sole beneficiary. The corporation will not legislative intention to tax the proceeds of life insurance policies paid to
be taxed on such proceeds, since it is considered as an indemnity and not income. corporate beneficiaries, particularly when in the exemption in favor of
individual beneficiaries in the chapter on this subject, the clause is
inserted "exempt from the provisions of this law," we deem it
FACTS:
reasonable to hold the proceeds of the life insurance policy in
1.   El Oriente is a domestic corporation and existing under and by virtue of the
question as representing an indemnity and not taxable income
laws of the Philippine Islands.
2.   Juan Posadas (Posadas) is the duly appointed, qualified and acting Collector
of Internal Revenue of the Philippine Islands.
RULING: WHEREFORE, The foregoing pronouncement will result in the judgment 9.   In reality, what El Oriente received was in the nature of an indemnity for
being reversed and in another judgment being rendered in favor of El Oriente and the loss, which it actually suffered because of the death of its manager A.
against Posadas for the sum of P3,148.74. So ordered, without costs in either Velhagen.
instance. 10.   Considering, therefore, the purport of the stipulated facts, considering the
uncertainty of Philippine law, and considering the lack of express legislative
RATIO: intention to tax the proceeds of life insurance policies paid to corporate
1.   The Income Tax Law for the Philippines is Act No. 2833, as amended. beneficiaries, particularly when in the exemption in favor of individual
a.   In Chapter I On Individuals, is to be found Section 4 which beneficiaries in the chapter on this subject, the clause is inserted "exempt
provides that, from the provisions of this law," we deem it reasonable to hold the
"The following incomes shall be exempt from the proceeds of the life insurance policy in question as representing an
provisions of this law: (a) The proceeds of life insurance indemnity and not taxable income.
policies paid to beneficiaries upon the death of the
insured ... ."  
b.   In Chapter II on Corporations does not provide as above.
i.   Section 10, as amended, in Chapter II On Corporations,
provides that, There shall be levied, assessed, collected,
and paid annually upon the total net income received in
the preceding calendar year from all sources by every
corporation ... a tax of three per centum upon such
income ... ."
ii.   Section 11 in the same chapter, provides the exemptions
under the law, but neither here nor in any other section is
reference made to the provisions of section 4 in Chapter I.
2.   It is certain that the proceeds of life insurance policies are exempt. It is
not so certain that the proceeds of life insurance policies paid to corporate
beneficiaries upon the death of the insured are likewise exempt.
3.   The situation will be better elucidated by a brief reference to laws on the
same subject in the United States.
4.   The Income Tax Law of 1916 extended to the Philippine Legislature, when
it came to enact Act No. 2833, to copy the American statute.
5.   Subsequently, the Congress of the United States enacted its Income Tax
Law of 1919, in which certain doubtful subjects were clarified.
6.   Thus, as to the point before us, it was made clear, when not only in the part
of the law concerning individuals were exemptions provided for
beneficiaries, but also in the part concerning corporations, specific reference
was made to the exemptions in favor of individuals, thereby making the
same applicable to corporations.
7.   This was authoritatively pointed out and decided by the United States
Supreme Court in the case of United States vs. Supplee-Biddle Hardware
Co., which involved facts quite similar to those before us.
8.   To quote the exact words in the cited case of Chief Justice Taft delivering
the opinion of the court:
“It is earnestly pressed upon us that proceeds of life insurance paid
on the death of the insured are in fact capital, and cannot be taxed
as income … that proceeds of a life insurance policy paid on the
death of the insured are not usually classed as income.”
002 INSULAR LIFE V. EBRADO (ALCAZAR) FACTS:
October 28,1977 | Martin, J. | Designation of Beneficiary 1.   Buenaventura Ebrado was issued by The Life Assurance Co., Ltd. a
policy on a whole-life insurance for P5,882.00 with a, rider for
PETITIONER: The Insular Life Assurance Company, Ltd. Accidental Death for the same amount. He designated Carponia T.
RESPONDENTS: Carponia T. Ebrado and Pascuala Vda. De Ebrado Ebrado as the revocable beneficiary in his policy. He referred to her
as his wife.
SUMMARY: Buenaventura Ebrado was insured by The Life Assurance Co., 2.   Ebrado diedwhen he was hit by a falling branch of a tree.
Ltd. a policy on a whole-life insurance with a rider on accidental death with 3.   As the policy was in force, The Insular Life Assurance Co., Ltd.
Carponia Ebrada as his beneficiary. He refered to her as his wife. He died liable to pay the coverage in the total amount of P11,745.73,
when a falling branch of a tree fell on him. Carponia (Common law wife) representing the face value of the policy in the amount of P5,882.00
filed a claim for the proceeds of the insurance as the designated beneficiary. plus the additional benefits for accidental death also in the amount of
While Pascual Vda. De Ebrado (Legal wife) also filed her claim as the P5,882.00 and the refund of P18.00 paid for the premium minus the
widow of the deceased insured. The Insurer filed an interpleader before the unpaid premiums and interest thereon.
CFI to determine who to give the insurance proceeds to. The CFI rendered a 4.   Carponia T. Ebrado filed with the insurer a claim for the proceeds of
decision in favor Pascuala. The CA affirmed this decision and ruled that the Policy as the designated beneficiary therein, although she admits
Carponia was disqualified from becoming a beneficiary of Ebrado. that she and the insured Buenaventura C. Ebrado were merely living
The issue is: Whether a common-law wife named as beneficiary in the life as husband and wife without the benefit of marriage.
insurance policy of a legally married man may claim the proceeds thereof in 5.   Pascuala Vda. de Ebrado also filed her claim as the widow of the
case of death of the latter. NO. Common-law spouses are barred from deceased insured. She asserts that she is the one entitled to the
receiving donations from each other. insurance proceeds, not the common-law wife, Carponia T. Ebrado.
In essence, a life insurance policy is no different from a civil donation insofar 6.   In doubt as to whom the insurance proceeds shall be paid, the
as the beneficiary is concerned. Both are founded upon the same insurer, The Insular Life Assurance Co., Ltd. commenced an action
consideration: liberality. A beneficiary is like a donee, because from the for Interpleader before the Court of First Instance
premiums of the policy, which the insured pays out of liberality, the 7.   CFI: Found that Pascuala had 6 legitimate children with Ebrado and
beneficiary will receive the proceeds or profits of said insurance. Article 739 Carponia had 2 illegitimate children with Ebrado. The court ruled
of the new Civil Code provides: The following donations shall be void: (1) that Carponia T. Ebrado disqualified from becoming beneficiary of
Those made between persons who were guilty of adultery or concubinage at the insured Buenaventura Ebrado and directing the payment of the
the time of donation. As a consequence, the proscription in Article 739 of the insurance proceeds to the estate of the deceased insured.
new Civil Code should equally operate in life insurance contracts. The 8.   CA: Affirmed the judgment of the lower court. It ruled that:
mandate of Article 2012 should be followed: any person who cannot receive •   Section 50 of the Insurance Act which provides that "(t)he
a donation cannot be named as beneficiary in the life insurance policy of the insurance shall be applied exclusively to the proper interest of
person who cannot make the donation. Therefore, Carponia Ebrada as the the person in whose name it is made".
common law wife is disqualified to be a beneficiary. As a consequence, the •   The word "interest" highly suggests that the provision refers
proceeds of the policy are hereby held payable to the estate of the deceased only to the "insured" and not to the beneficiary, since a contract
insured. of insurance is personal in character. Otherwise, the prohibitory
laws against illicit relationships especially on property and
DOCTRINE: Any person who is forbidden from receiving any donation descent will be rendered nugatory, as the same could easily be
under Article 739 cannot be named a beneficiary of a life insurance policy by circumvented by modes of insurance.
the person who cannot make a donation to him. Common-law spouses are, •   Rather, the general rules of civil law should be applied to resolve
definitely, barred from receiving donations from each other. this void in the Insurance Law. Article 2011 of the New Civil
Code states: "The contract of insurance is governed by special
laws. Matters not expressly provided for in such special laws
shall be regulated by this Code." When not otherwise specifically donation cannot be named as beneficiary in the life insurance policy
provided for by the Insurance Law, the contract of life insurance of the person who cannot make the donation.
is governed by the general rules of the civil law regulating 3.   Family laws, reason and morality dictate that the impediments
contracts. imposed upon married couple should likewise be imposed upon
•   Article 739 of the new Civil Code provides: The following extra-marital relationship. If a legitimate relationship is
donations shall be void: (1) Those made between persons who circumscribed by these legal disabilities, with more reason should an
were guilty of adultery or concubinage at the time of donation; illicit relationship be restricted by these disabilities.
x x x 4.   A conviction for adultery or concubinage is not required before the
In the case referred to in No. 1, the action for declaration of disabilities mentioned in Article 739 may effectuate. The article says
nullity may be brought by the spouse of the donor or donee; and that in the case referred to in No. 1, the action for declaration of
the guilt of the donee may be proved by preponderance of nullity may be brought by the spouse of the donor or donee; and the
evidence in the same action. guilty of the donee may be proved by preponderance of evidence in
•   Under Article 2012 of the same Code, "any person who is the same action.
forbidden from receiving any donation under Article 739 5.   The underscored clause neatly conveys that no criminal conviction
cannot be named beneficiary of a life insurance policy by the for the offense is a condition precedent. The law plainly states that
person who cannot make a donation to him. Common-law the guilt of the party may be proved “in the same acting for
spouses are, definitely, barred from receiving donations from declaration of nullity of donation.” And, it would be sufficient if
each other. evidence preponderates.
6.   The requisite proof of common-law relationship between the insured
ISSUE/s: and the beneficiary has been conveniently supplied by the
1.   Whether a common-law wife named as beneficiary in the life stipulations between the parties in the pre-trial conference of the
insurance policy of a legally married man may claim the case. The judicial admissions are sufficient and are considered valid.
proceeds thereof in case of death of the latter. NO. Common-law 7.   Carponia Ebrada as the common law wife is disqualified to be a
spouses are barred from receiving donations from each other. beneficiary. As a consequence, the proceeds of the policy are hereby
held payable to the estate of the deceased insured.
RULING: ACCORDINGLY, the appealed judgment of the lower court is
hereby affirmed. Carponia T. Ebrado is hereby declared disqualified to be the  
beneficiary of the late Buenaventura C. Ebrado in his life insurance policy.
As a consequence, the proceeds of the policy are hereby held payable to the
estate of the deceased insured.

RATIO:
1.   In essence, a life insurance policy is no different from a civil
donation insofar as the beneficiary is concerned. Both are founded
upon the same consideration: liberality. A beneficiary is like a
donee, because from the premiums of the policy, which the
insured pays out of liberality, the beneficiary will receive the
proceeds or profits of said insurance.
2.   As a consequence, the proscription in Article 739 of the new Civil
Code should equally operate in life insurance contracts. The mandate
of Article 2012 should be followed: any person who cannot receive a
003 PHILAM v. PINEDA (APASAN)
July, 19, 1989 | Paras, J. | Designation of beneficiary ISSUE/s:
8.   WoN the designation of the irrevocable beneficiaries could be changed or
PETITIONER: Philippine American Insurance Company amended without the consent of all the irrevocable beneficiaries – No, this
RESPONDENTS: Honorable Gregorio G. Pineda and Rodolfo C. Dimayuga was expressly prohibited by the Insurance Act, which was the law
applicable at that time.
SUMMARY: Rodolfo Dimayuga (Dimayuga) procured an ordinary life insurance 9.   WoN the irrevocable beneficiaries herein, who are all minors (NOTE: wife,
policy from Philippine American Insurance Company (Philam) designating his wife who was also a beneficiary, predeceased Dimayuga), could validly give
and children as irrevocable beneficiaries. Subsequently, Dimayuga filed a petition consent to the change or amendment in the designation of the irrevocable
with the CFI to amend the designation in the policy from irrevocable to recovable. beneficiaries – No, because being minors, this would not be an effective
Philam opposed, but the lower court denied such opposition and granted the petition. ratification.
Issues are: 1.) WoN the designation of irrevocable beneficiaries could be
amended without the consent of all the beneficiaries – No, under the Insurance RULING: Order of the lower court was nullified and set aside.
Act, which was the law applicable at that time, the beneficiary designated in a life
insurance contract cannot be changed without the consent of the beneficiary because RATIO:
he has a vested interest in the policy. Also, the provisions of the Insurance Contract First Issue
did not provide for any exception regarding its irrevocability. Both the law and the 1.   Needless to say, the applicable law in the instant case is the Insurance Act,
policy do not provide for any other exception, thus, abrogating the contention of the otherwise known as Act No. 2427 as amended, the policy having been
Dimayuga that said designation can be amended if the Court finds a just, reasonable procured in 1968.
ground to do so 2.) WoN the irrevocable beneficiaries herein who are all minors a.   Under the said law, the beneficiary designated in a life insurance
(NOTE: wife, who was also a beneficiary, predeceased Dimayuga), could validly contract cannot be changed without the consent of the
give consent to the change or amendment in the designation of the irrevocable beneficiary because he has a vested interest in the policy.
beneficiaries – No. First, being minors, their ratification would be ineffective. 2.   In this regard, it is worth noting that the Beneficiary Designation
Second, neither could they act through their father insured since their interests are Indorsement in the policy which forms part of Insurance Policy in the name
quite divergent from one another. Lastly, parent-insured (Dimayuga) cannot exercise of Rodolfo Dimayuga states that the designation of the beneficiaries is
rights and/or privileges pertaining to the insurance contract, for otherwise, the vested irrevocable, to wit:
rights of the irrevocable beneficiaries would be rendered inconsequential. a.   It is hereby understood and agreed that, notwithstanding the
provisions of this policy to the contrary, inasmuch as the
DOCTRINE: Under the Insurance Act, otherwise known as Act No. 2427, the designation of the primary/contingent beneficiary/beneficiaries
beneficiary designated in a life insurance contract cannot be changed without the in this Policy has been made without reserving the right to
consent of the beneficiary because he has a vested interest in the policy. Further, change said beneficiary/ beneficiaries, such designation may not
minor-beneficiaries cannot validly give their consent to such amendment. be surrendered to the Company, released or assigned; and no right
or privilege under the Policy may be exercised, or agreement
made with the Company to any change in or amendment to the
FACTS: Policy, without the consent of the said
1.   On January 15, 1968, Rodolfo Dimayuga (Dimayuga) procured an ordinary beneficiary/beneficiaries.
life insurance policy from Philippine American Insurance Company 3.   Be it noted that the foregoing is a fact which the private respondent did not
(Philam) and designated his wife and children as irrevocable beneficiaries bother to disprove.
of said policy. 4.   Inevitably therefore, based on the aforequoted provision of the contract,
2.   On February 22, 1980 Dimayuga filed a petition before the Court of First it is only with the consent of all the beneficiaries that any change or
Instance of Rizal to amend the designation of the beneficiaries in his life amendment in the policy concerning the irrevocable beneficiaries may
policy from irrevocable to revocable. be legally and validly effected. Both the law and the policy do NOT
3.   Philam filed its Comment and/or Opposition to Petition. provide for any other exception, thus, abrogating the contention of the
a.   Respondent Judge Gregorio G. Pineda (Judge Pineda) denied this Dimayuga that said designation can be amended if the Court finds a
opposition, and the consequence of which was the issuance of the just, reasonable ground to do so.
questioned Order granting the petition.
4.   Hence, this petition.
Second Issue 10.   Finally, the fact that the contract of insurance does not contain a
5.   Similarly, the alleged acquiescence of the six (6) children beneficiaries of contingency when the change in the designation of beneficiaries could
the policy (the beneficiary-wife predeceased the insured) CANNOT be be validly effected means that it was never within the contemplation of
considered an effective ratification to the change of the beneficiaries the parties.
from irrevocable to revocable. a.   The lower court, in gratuitously providing for such contingency,
6.   Indubitable is the fact that all the six (6) children named as beneficiaries made a new contract for them, a proceeding which the Supreme
were minors at the time, for which reason, they could NOT validly give Court cannot tolerate.
their consent. Neither could they act through their father insured since b.   Ergo, the Supreme Court cannot help but conclude that the lower
their interests are quite divergent from one another. In point is an court acted in excess of its authority when it issued the Order dated
excerpt from the Notes and Cases on Insurance Law by Campos and March 19, 1980 amending the designation of the beneficiaries from
Campos, 1960, reading- "irrevocable" to "revocable" over the disapprobation of the
a.   The insured ... can do nothing to divest the beneficiary of his rights petitioner insurance company.
without the latter’s consent. He cannot assign his policy, nor even
take its cash surrender value without the consent of the beneficiary.  
Neither can the insured's creditors seize the policy or any right
thereunder. The insured may not even add another beneficiary
because by doing so, he diminishes the amount which the
beneficiary may recover and this he cannot do without the
beneficiary's consent.
7.   Therefore, the parent-insured cannot exercise rights and/or privileges
pertaining to the insurance contract, for otherwise, the vested rights of
the irrevocable beneficiaries would be rendered inconsequential.
8.   Of equal importance is the well-settled rule that the contract between the
parties is the law binding on both of them and for so many times, this court
has consistently issued pronouncements upholding the validity and
effectivity of contracts.
a.   Where there is nothing in the contract which is contrary to law,
good morals, good customs, public policy or public order the
validity of the contract must be sustained.
b.   Likewise, contracts which are the private laws of the contracting
parties should be fulfilled according to the literal sense of their
stipulations, if their terms are clear and leave no room for doubt as
to the intention of the contracting parties, for contracts are
obligatory, no matter in what form they may be, whenever the
essential requisites for their validity are present.
9.   In the recent case of Francisco Herrera vs. Petrophil Corporation, 146
SCRA 385, this Court ruled that: ... it is settled that the parties may
establish such stipulations, clauses, terms, and conditions as they may want
to include; and as long as such agreements are not contrary to law, good
morals, good customs, public policy or public order, they shall have the
force of law between them.
a.   Undeniably, the contract in the case at bar, contains the
indispensable elements for its validity and does not in any way
violate the law, morals, customs, orders, etc. leaving no reason for
the Court to deny sanction thereto.
004 HEIRS OF LORETO MARAMAG v. MARAMAG (Arcenas) 1.   Vicenta Maramag (legal wife), representing the heirs of Loreto, filed a case
June 5, 2009| Nachura, C. J. | Designation of beneficiary against Eva Maramag et al (kabit and anak ng kabit) with the RTC for
revocation and/or reduction of insurance proceeds for being void and/or
PETITIONERS: Heirs Of Loreto C. Maramag, Represented By Surviving Spouse inofficious, with prayer for a temporary restraining order (TRO) and a writ
Vicenta Pangilinan Maramag of preliminary injunction, alleging that
RESPONDENTS: Eva Verna De Guzman Maramag, Odessa De Guzman Maramag, a.   Heirs were the legitimate wife and children of Loreto Maramag,
Karl Brian De Guzman Maramag, Trisha Angelie Maramag, The Insular Life while Eva Maramag et al were Loretos illegitimate family;
Assurance Company, Ltd., And Great Pacific Life Assurance Corporation b.   Eva de Guzman Maramag (Eva) was a concubine of Loreto and a
suspect in the killing of the latter, and thus disqualified to receive
SUMMARY: Petitioners in this case are the legitimate heirs of Loreto, represented any proceeds from his insurance policies from Insular Life
by the legitimate wife, Vienta. They were not named as beneficiaries in the insurance Assurance Company, Ltd. (Insular) and Great Pacific Life
policies issued by Insular and Grepalife, instead, Loreto designated his concubine, Assurance Corporation (Grepalife);
Eva, and their illegitimate children as his beneficiaries under policies with Insular c.   the illegitimate children of Loreto: Odessa, Karl Brian, and Trisha
Life and Grepalife. Vicenta et al are seeking the revocation and reduction of Angelie were entitled only to one-half of the legitime of the
insurance proceeds in their favor. The basis of the claim is that Eva, being a legitimate children, thus, the proceeds released to Odessa and those
concubine of Loreto and a suspect in Loreto’s murder, is disqualified from being to be released to Karl Brian and Trisha Angelie were inofficious
designated as beneficiary of the insurance policies, and that Eva’s children with and should be reduced; and
Loreto, being illegitimate children, are entitled to a lesser share of the proceeds of d.   Heirs could not be deprived of their legitimes, which should be
the policies. They also argued that pursuant to Section 12 of the Insurance Code, satisfied first.
Evas share in the proceeds should be forfeited in their favor, the former having e.   That part of the insurance proceeds had already been released in
brought about the death of Loreto. Thus, they prayed that the share of Eva and favor of Odessa, while the rest of the proceeds are to be released in
portions of the shares of Loretos illegitimate children should be awarded to them, favor of Karl Brian and Trisha Angelie, both minors, upon the
being the legitimate heirs of Loreto entitled to their respective legitimes. RTC appointment of their legal guardian.
initially ruled that the claim be dismissed against the heirs and sustained the action f.   Prayer for the total amount of P320,000.00 as actual litigation
against Eva, Insular and Grepalife. However, reversed itself by granting the motion expenses and attorneys fees.
for reconsideration of the same parties. On appeal, the same was denied for lack of 2.   INSULAR’S ANSWER: Insular admitted that Loreto misrepresented Eva
jurisdiction and was filed beyond reglementary period. Hence, this petition. The as his legitimate wife and Odessa, Karl Brian, and Trisha Angelie as his
issue is whether or not the legitimate children can claim Eva’s share since she was legitimate children, and that they filed their claims for the insurance
disqualified as beneficiary under the insurance policy of the deceased, and that the proceeds of the insurance policies;
same should not have been distributed among the illegitimate children. SC held that a.   that when it ascertained that Eva was not the legal wife of Loreto,
they cannot since the legitimate children are third parties to the insurance contracts, it disqualified her as a beneficiary and divided the proceeds among
and not entitled to the proceeds despite disqualification of Eva as beneficiary illegitimate children, as the remaining designated beneficiaries;
because the designation of the illegitimate children as beneficiaries remained valid. and
Also, Eva’s share whether forfeited by the court in view of the prohibition on b.   that it released Odessas share as she was of age, but withheld the
donations under Article 739 of the Civil Code or by the insurers themselves for release of the shares of minors Karl Brian and Trisha Angelie
reasons based on the insurance contracts, must be awarded to the said illegitimate pending submission of letters of guardianship.
children, the designated beneficiaries, to the exclusion of the legitimate children. c.   Insular alleged that the complaint or petition failed to state a cause
of action: when it sought to declare as void the designation of Eva
DOCTRINE: The only persons entitled to claim the insurance proceeds are either as beneficiary, because Loreto already revoked Eva’s designation
the insured, if still alive; or the beneficiary, if the insured is already deceased, upon as such in one policy and it disqualified her in another and when it
the maturation of the policy. It is only in cases where the insured has not designated sought to declare as inofficious the shares of Odessa, Karl Brian,
any beneficiary, or when the designated beneficiary is disqualified by law to receive and Trisha Angelie, considering that no settlement of Loretos
the proceeds, that the insurance policy proceeds shall redound to the benefit of the estate had been filed nor had the respective shares of the heirs been
estate of the insured. determined.

FACTS:
d.   Insular further claimed that it was bound to honor the insurance 8.   MR GRANTED: dismissed prosecution order against eva, insular and
policies designating the children of Loreto with Eva as grepalife.
beneficiaries pursuant to Section 53 of the Insurance Code. a.   RTC ruled it is only in cases where there are no beneficiaries
3.   GREPALIFE’s ANSWER: with compulsory counterclaim designated, or when the only designated beneficiary is disqualified,
a.   Grepalife alleged that Eva was not designated as an insurance that the proceeds should be paid to the estate of the insured.
policy beneficiary; b.   As to the claim that the proceeds to be paid to Loreto’s illegitimate
b.   that the claims filed by Odessa, Karl Brian, and Trisha Angelie children should be reduced based on the rules on legitime, the trial
were denied because Loreto was ineligible for insurance due to a court held that the distribution of the insurance proceeds is
misrepresentation in his application form that he was born on governed primarily by the Insurance Code, and the provisions of
December 10, 1936 and, thus, not more than 65 years old when he the Civil Code are irrelevant and inapplicable.
signed it in September 2001; c.   Grepalife policy: Eva was never designated as a beneficiary, but
c.   that the case was premature, there being no claim filed by the only the illegitimate children; thus, it upheld the dismissal of the
legitimate family of Loreto; and case as to them, matter of Loretos misrepresentation was premature
d.   that the law on succession does not apply where the designation of 9.   CA RULING: dismissed Vicenta’s appeal for lack of jurisdiction, holding
insurance beneficiaries is clear that the decision of the trial court dismissing the complaint for failure to
4.   As the whereabouts of Eva et al were not known to Vicenta et al, summons state a cause of action involved a pure question of law, and that the appeal
by publication was resorted to. Eva et al failed to answer, and declared in was filed beyond the reglementary period. Hence, this petition
default by the trial court. ISSUE/s:
5.   VICENTA ET AL’s comment: alleged that the issue raised by Insular and 1.   W/N the legitimate children can claim Eva’s share since she was
Grepalife was purely legal whether the complaint itself was proper or not disqualified as beneficiary under the insurance policy of the deceased, and
and that the designation of a beneficiary is an act of liberality or a donation that the same should not have been distributed among the illegitimate
and, therefore, subject to the provisions of Articles 752 and 772 of the Civil children – NO, legitimate children are third parties to the insurance
Code. contracts, and not entitled to the proceeds despite disqualification of Eva as
6.   REPLY: Both insurance companies countered that the insurance proceeds beneficiary because the designation of the illegitimate children as
belong exclusively to the designated beneficiaries in the policies, not to the beneficiaries remained valid. Also, Eva’s share whether forfeited by the
estate or to the heirs of the insured. court in view of the prohibition on donations under Article 739 of the Civil
a.   Grepalife also reiterated that it had disqualified Eva as a Code or by the insurers themselves for reasons based on the insurance
beneficiary when it ascertained that Loreto was legally married to contracts, must be awarded to the said illegitimate children, the designated
Vicenta Pangilinan Maramag. beneficiaries, to the exclusion of the legitimate children.
7.   RTC RULING: dismissed the complaint illegitimate children for the
reduction and/or declaration of inofficiousness of donation as primary RULING: WHEREFORE, the petition is DENIED for lack of merit. Costs against
beneficiary in the insurances of the late Loreto C. Maramag, but it proceeds petitioners.
against Eva, Insular and Grepalife.
a.   Can proceed against eva because she is the concubine of the RATIO:
deceased: any person who is forbidden from receiving any 1.   In this case, it is clear from the petition filed before the trial court that,
donation under Article 739 cannot be named beneficiary of a life although Vicenta et al are the legitimate heirs of Loreto, they were not
insurance policy of the person who cannot make any donation to named as beneficiaries in the insurance policies issued by Insular and
him, according to said article (Art. 2012, Civil Code). If a Grepalife.
concubine is made the beneficiary, it is believed that the insurance a.   The basis of the claim is that Eva, being a concubine of Loreto and
contract will still remain valid, but the indemnity must go to the a suspect in his murder, is disqualified from being designated as
legal heirs and not to the concubine, for evidently, what is beneficiary of the insurance policies, and that Evas children with
prohibited under Art. 2012 is the naming of the improper Loreto, being illegitimate children, are entitled to a lesser share of
beneficiary the proceeds of the policies.
b.   Since designation to Eva is void, the insurance indemnity paid to b.   They also argued that pursuant to Section 12 of the Insurance
her must go to the legal heirs Code, Evas share in the proceeds should be forfeited in their favor,
the former having brought about the death of Loreto.
c.   Thus, they prayed that the share of Eva and portions of the shares
of Loretos illegitimate children should be awarded to them, being
the legitimate heirs of Loreto entitled to their respective legitimes.
2.   It is evident from the face of the complaint that Vicente et al are not entitled
to a favorable judgment in light of Article 2011 of the Civil Code which
expressly provides that insurance contracts shall be governed by special
laws, i.e., the Insurance Code, specifically Section 53.13
a.   Pursuant thereto, it is obvious that the only persons entitled to
claim the insurance proceeds are either the insured, if still
alive; or the beneficiary, if the insured is already deceased,
upon the maturation of the policy.
b.   The exception to this rule is a situation where the insurance
contract was intended to benefit third persons who are not parties
to the same in the form of favorable stipulations or indemnity. In
such a case, third parties may directly sue and claim from the
insurer.
3.   Vicenta et al are third parties to the insurance contracts with Insular
and Grepalife and, thus, are not entitled to the proceeds.
a.   Accordingly, Insular and Grepalife have no legal obligation to turn
over the insurance proceeds to Vicenta et al.
b.   The revocation of Eva as a beneficiary in one policy and her
disqualification as such in another are of no moment considering
that the designation of the illegitimate children as beneficiaries in
Loretos insurance policies remains valid.
c.   Because no legal proscription exists in naming as beneficiaries
the children of illicit relationships by the insured, the shares of
Eva in the insurance proceeds, whether forfeited by the court in
view of the prohibition on donations under Article 739 of the Civil
Code or by the insurers themselves for reasons based on the
insurance contracts, must be awarded to the said illegitimate
children, the designated beneficiaries, to the exclusion of Vicenta
et al.
d.   It is only in cases where the insured has not designated any
beneficiary, or when the designated beneficiary is disqualified by
law to receive the proceeds, that the insurance policy proceeds
shall redound to the benefit of the estate of the insured.

                                                                                                                       
13  
SECTION 53. The insurance proceeds shall be applied exclusively to the proper interest of the person in
whose name or for whose benefit it is made unless otherwise specified in the policy.
 
Lampano v. Jose, et. al. (Lindon) sum of P2,000, evidenced by a promissory note, and Placida A. Jose still
March 30, 1915 | Trent, J. | Insurable Interest owed Mariano R. Barretto on the cost of the construction the sum of
P2,000.
PETITIONER: Antonina Lampano 5.   After the completion of the house and sometime before it was destroyed,
RESPONDENTS: Placida Jose Mariano R. Barretto took out an insurance policy upon it in his own name,
with the consent of Placida A. Jose, for the sum of P4,000.
SUMMARY: Barretto built a house for Jose in Manila. Then, Jose sold the same 6.   After its destruction, he collected P3,600 from the insurance company,
to Lampano for 6k. A few months later, house was gutted by fire. At that time, having paid in premiums the sum of P301.50.
Lampano still owed Jose 2k, evidenced by a promissory note, and Jose still owed
Barretto the cost of construction (2k as well). After the completion of the house Arguments before the lower courts
and before it was destroyed, Barretto took out an insurance policy in his name Petitioner’s Side
with consent of Jose for 4k. Barretto was able to collect 3.6k after paying 300 as 7.   Antonina Lampano alleged in her complaint that there was a verbal
premiums. agreement between her and Placida A. Jose, at the time of the purchase and
sale of the house, to the effect that the latter14 agreed to deliver to her the
Lampano sued Jose and Barretto for the amount of the insurance collected. Jose insurance policy on the building;
denied the transfer of the policy to Lampano since it was Barretto’s. Jose filed a 8.   She15 did not learn that the policy was in the name of Barretto16 until after
counterclaim against Lamoano for the balance of the purchase price. Trial court the fire; and the neither Placida A. Jose17 nor Mariano R. Barretto18 has any
ruled in favor of Jose against Barretto for the difference between insurance right to the insurance or to the money received therefrom.
proceeds and the amount yet due him. TC did not rule on Barretto v. Lampano 9.   She19 prayed for judgment against each20 of them for the sum of P3,600, the
for lack of privity. Hence, review before SC as Barretto and Lampano appealed. amount of the insurance collected.

In the case at bar Barretto assumed the responsibility for the insurance. Respondents’ Side
The premiums, as we have indicated, were paid by him without any 10.   To this complaint the defendant, Placida A. Jose,21 answered, denying that
agreement or right to recoup the amount paid therefor should no loss result she agreed to transfer the policy of insurance to the plaintiff22 and alleging:
to the property. It would not, therefore, be in accordance with the law and his a.   that the insurance was taken out and paid for by Barretto before the
contractual obligations to compel him to account for the insurance money, or any sale of the house to the plaintiff;
par thereof, to Antonina Lampano, who assumed no risk whatever. b.   that Barretto did this because he had constructed the house and she
was owing him therefor; and
DOCTRINE: Where different persons have different interests in the same c.   that the insurance was entirely for the personal account and in the
property, the insurance taken by one in his own right and in his own interest does exclusive interest of Barretto.
not in any way insure to the benefit of another. This is the general rule prevailing 23
in the United States and we find nothing different in this jurisdiction. 11.   In her cross-complaint she asked for judgment against Antonina

                                                                                                                       
14
N.B. The alleged oral transfer of the policy was not proven in the court below.  PLACIDA  A  JOSE  
15
 ANTONINA  LAMPANO  
  16
 THE  CO-­‐DEFENDANT  WHO  CONSTRUCTED  THE  HOUSE  
FACTS: 17
 THE  DEFENDANT  WHOM  BARRETTO  BUILT  THE  HOUSE  FOR  
1.   Mariano R. Barretto constructed a house for the other defendant, Placida 18
A. Jose on land described as No. 72, plot F. Estate of Nagtahan, district of  THE  CO-­‐DEFENDANT  WHO  CONSTRUCTED  THE  HOUSE  
19
Sampaloc, city of Manila, for the agreed price of P6,000.  PLACIDA  A  JOSE  
20
2.   Subsequent thereto and on November 12, 1912, Placida A. Jose sold the  PLACIDA  A  JOSE  and  MARIANO  R.  BARRETTO  
house to the plaintiff, Antonina Lampano, for the sum of P6,000. 21
 THE  PERSON  WHOM  THE  HOUSE  WAS  CONSTRUCTED  FOR  
3.   On March 22, 1913, the house was destroyed by fire. 22
 ANTONINA  LAMPANO  
4.   At the time of the fire Antonina Lampano still owed Placida A. Jose the 23
 I  think  this  should  have  been  a  counter-­‐claim  
Lampano for the sum of P2,000, the balance due on the purchase price. 2.   If Barretto had an insurable interest in the house, he could insure this
Barretto answered, reciting the facts giving rise to his taking out the interest for his sole protection. The policy was in the name of Barretto
insurance on the house and denying any obligation to Antonina Lampano in alone. It was, therefore, a personal contract between him and the
connection therewith. company and not a contract which ran with the property. According to
this personal contract the insurance policy was payable to the insured
Judgment of the lower courts without regard to the nature and extent of his interest in the property,
12.   Judgment was entered against Barretto and in favor of Placida A. Jose for provided that he had, as we have said, an insurable interest at the time of the
the sum of P1,298.50, being the difference between the amount collected by making of the contract, and also at the time of the fire. Where different
Barretto on the insurance and the amount yet due him for the construction persons have different interests in the same property, the insurance
of the house, including the premiums paid. taken by one in his own right and in his own interest does not in any
13.   Judgment was also entered in favor of the defendant, Placida A. Jose, way insure to the benefit of another. This is the general rule prevailing
against the plaintiff for the sum of P2,000, being the balance of the purchase in the United States and we find nothing different in this jurisdiction.
price of the house. 3.   In the case of Shadgett vs. Phillips and Crew Co., reported in 56 L. R. A.,
14.   The plaintiff was authorized to offset this judgment against her for P2,000 461, Mrs. Shagett received a piano as a gift from her husband and insured it.
by the P2,000 which the court declared had been paid the defendant, Placida She knew that it was the obligation of her husband to insure he piano for the
A. Jose, by Barretto out of the insurance money. A final judgment was benefit of the vendor. The court held, however, that the vendor (mortgagee)
entered in favor of the plaintiff against the defendant, Placida A. Jose, for was not entitled to the proceeds of the insurance as "there was no
the sum of P1,298.50, being the amount of the judgment against Barretto. undertaking on the part of Mrs. Shadgett to either insure for complainant's
15.   Barretto appealed the judgment in the CROSS-CLAIM. benefit, or to assume her husband's obligation to so insure, and mere
16.   No judgment was rendered in favor of the Antonina Lampano since they knowledge of that obligation did not impose it upon her."
have no juridical tie. Antonina Lampano appealed as well. a.   The contract of insurance was wholly between the defendant and
the insurance company, and was personal, in the sense that the
ISSUE/s: money agreed to be paid in case of loss was not to stand in the
1.   Has Antonina Lampano any right to recover from Barretto any portion of place of the piano itself, but was a mere indemnity against the loss
the insurance money? (This was the wording in the case) – No. Barretto of defendant's interest therein. If her interest was small, on account
alone assumed the responsibility for the insurance. of incumbrances existing in favor of the complainant, that fact was
RULING: For the foregoing reasons the judgment appealed from, in so far as it for the consideration only of the insurer and defendant, for
affects the appellant, is reversed and he is absolved. Without costs. So ordered. complaint has no concern with the adjustment of the loss between
them. We know of no principle, either of law or equity, which
RATIO: would bind defendant to carry out her donor's contract to insure, in
the absence of any agreement on her part to do so, even though the
Complications property in her hands was subject to complainant's rights therein as
1.   The plaintiff does not contend that Barretto participated in this sale, or even a conditional vendor.
had any knowledge of it, until sometime after it was consummated. Placida b.   A contract of insurance made for the insurer's (insured) indemnity
A. Jose denies that she agreed to transfer the policy to the plaintiff, and the only, as where there is no agreement, express or implied, that it
deed of purchase and sale makes no mention of such an agreement. The shall be for the benefit of a third person, does not attach to or run
policy is not mentioned in this document, although it was agreed that the with the title to the insured property on a transfer thereof personal
vendor would transfer to the vendee all of the former's right, title, and as between the insurer and the insured. In such case strangers to
interest in the leasehold to the land upon which the house was built. It the contract cannot require in their own right any interest in the
would seem that if the vendor agreed to transfer the policy, this agreement insurance money, except through an assignment or some contract
would have been inserted in the document of purchase and sale, the same as with which they are connected.
that with reference to the lease. The trial court did not find that such an 4.   In Vandergraf vs. Medlock (3 Porter, 389; 29 Am. Dec., 256), it was held
agreement existed and we think the plaintiff has failed to establish this that the mortgage is not entitled to the proceeds of an insurance policy
verbal agreement. procured by the mortgages, there being no agreement that such insurance
should be effected by the latter for the benefit of the former. The court says:
Ruling Proper "It is well settled that a policy of insurance is a distinct independent contract
between the insured and insurers, and third person have no right either in a
court of equity, or in a court of law, to the proceeds of it, unless there be
some contract or trust, expressed or implied, between the insured and third
persons."
5.   In Burlingane vs. Goodspeed (10 L. R. A., 495), the court says that where a
mortgage at his own expense and without any agreement or understanding
with he mortgagor obtains insurance upon his interest as a mortgage and
collects the money from the insurer after a loss, he is not bound to account
for it to the mortgagor.
6.   In the case at bar Barretto assumed the responsibility for the
insurance. The premiums, as we have indicated, were paid by him
without any agreement or right to recoup the amount paid therefor
should no loss result to the property. It would not, therefore, be in
accordance with t he law and his contractual obligations to compel him to
account for the insurance money, or any par thereof, to Antonina Lampano,
who assumed no risk whatever.
7.   That Barretto had an insurable interest in the house, we think there can be
no question. He construed the building, furnishing all the materials and
supplies, and insured it after it had been completed

 
006 TAI TONG CHUACHE & CO. v. INSURANCE COMMISSION Building 50,000 24,918.79
(BALISONG) SSS Building 25,000 5,938.50
29 Feb. 1988 | Gancayco, J. | Insurable Interest; Fire Insurance Accredited
Traveller’s Contents 70,000 16,628.00
PETITIONER: Tai Tong Chuache & Co. Building 30,000 14,467.31
RESPONDENTS: Insurance Commission and Traveller’s Multi-Indemnity Zenith, Phil. British, and SSS paid the whole or a portion of the claim. However,
Corporation Traveller’s disclaimed liability, prompting Palomo to claim from the other insurers
Traveller’s share based on the adjusted computation:
SUMMARY: Tai Tong loaned P100,000 to Palomo. The loan was secured by a
mortgage over a land and a building owned by Palomo. The building and its contents COMPANY RISK INSURES PAYS
were insured by Palomo with SSS, Zenith, and Phil. British. Tai Tong insured its Zenith Building P50,000 17,610.93
interest over the building and its contents with Traveller’s through Arsenio Chua. Phil. British Household 70,000 24,655.31
The building was gutted by fire. Claims were made by Palomo. Traveller’s refused to Building 50,000 39,186.10
pay on the ground that Palomo was not the beneficiary of the Policy, but Arsenio SSS Building 25,000 8,805.47
Chua. It also denied Tai Tong’s claim, saying that the loan and mortgage was Accredited
between Palomo and Chua. The issue is whether Tai tong has an insurable interest
over the building and the contents thereof. The Court held in the affirmative.
Arsenio, in contracting with the parties here, was acting as an agent of Tai Tong,
4.   The Zenith and Phil. British denied the claim on the ground that the claim
being its managing partner. Further, Palomo admitted that the mortgage remained
effective at the time of the fire as she has yet to pay Tai Tong. The Insurance had already been waived, extinguished, or paid. SSS paid the balance in
Commission was reversed. full.

DOCTRINE: The mortgagee has insurable interest over the subject of the mortgage 5.   Traveller’s reason for non-payment was that the Fire Insurance Policy over
is it can be shown that the loan secured by the same has not yet been paid at the time the building and the furniture inside was taken by Arsenio Chua as
the risk happened. mortgage creditor, and that it was Arsenio Chua who paid the premiums,
and that Palomo endorsed the Policy to Chua to protect the latter’s interest
FACTS: therein. Tai Tong filed a complaint in intervention, but was opposed by
1.   Palomo acquired a parcel of land and a building under an assumption of Traveller’s arguing that Tai Tong did not have insurable interest as it is not
mortgage arrangement. The building was insured with S.S.S. Accredited a party to the contract, the real party being Arsenio Chua.
Group of Insurers for P25,000.00.
6.   The Insurance Commission ruled in favor of Traveller’s. It held that Palomo
2.   Palomo obtained a P100,000-loan from Tai Tong Chuache, Inc. (Tai Tong), had no interest on the proceeds of the policy because it was taken out by the
secured by a mortgage over the land and the building. Aresenio Chua, in mortgagee. It also denied Tai Tong’s intervention on the strength of a
representation of Tai Tong, insured the partnership’s interest with certification from the CFI of Davao that a case between Arsenio Chua and
Traveller’s Multi-Indemnity Corporation for P100,000. Palomo secured a Palomo is pending before said court, which case, the Commission surmised,
Fire Insurance Policy over the building for P50,000 with Zenith was grounded on the mortgage contract between Arsenio Chua and Palomo,
Corporation, and for P50,000 (building) and P70,000 (contents) with NOT Tai Tong and Palomo, hence Tai Tong cannot claim under the policy
Philippine British Assurance Corporation. The building was razed by fire. as it had no insurable interest on the building, its contents, and the land over
which it stands.
3.   Palomo filed claims based on the following:
ISSUE/s:
COMPANY RISK INSURES PAYS
1.   Whether Tai Tong had insurable interest on the building and its contents.
Zenith Building P50,000 11,877.14
YES — The fact that Palomo admitted that she was indebted to Tai Tong
Phil. British Household 70,000 16,628.00
should have settled the matter. Further to this, Arsenio Chua is the
managing partner of Tai Tong, and was acting in representation of the
partnership in the above actions.

RULING: IN VIEW OF THE FOREGOING, the decision appealed from is hereby


SET ASIDE and ANOTHER judgment is rendered ordering private respondent
Travellers Multi-Indemnity Corporation to pay petitioner the face value of Insurance
Policy No. 599-DV in the amount of P100,000.00. Costs against said private
respondent.

RATIO:
1.   Traveller’s did not assail the validity of the insurance policy taken out by
petitioner over the mortgaged property. Neither did it deny that the said
property was totally razed by fire within the period covered by the
insurance.

2.   The record of the case shows that Tai Tong, to support its claim for the
insurance proceeds, offered as evidence the contract of mortgage which has
not been cancelled nor released. It has been held in a long line of cases that
when the creditor is in possession of the document of credit, he need not
prove non-payment for it is presumed.

3.   Tai Tong being a partnership may sue and be sued in its name or by its duly
authorized representative. The fact that Arsenio Lopez Chua is the
representative of Tai Tong is not questioned. Tai Tong’s declaration that
Arsenio Lopez Chua acts as the managing partner of the partnership was
corroborated by Traveller’s.

 
007 SPOUSES CHA v. CA (Buenaventura) FACTS:
August 18, 1997 | Padilla, J. | Lack of insurable interest
1.   Petitioner-spouses Nilo Cha and Stella Uy-Cha, as lessees, entered into a
lease contract with private respondent CKS Development Corporation
PETITIONER: Spouses Nilo and Stella Cha, and United Insurance Co., Inc
(hereinafter CKS), as lessor, on 5 October 1988.
RESPONDENTS: CA and CKS Development Corporation
2.   One of the stipulations of the one (1) year lease contract states:
SUMMARY:
a.   18. x x x. The LESSEE shall not insure against fire the chattels,
Spouses Nilo Cha and Stella Uy-Cha and CKS Development Corporation
merchandise, textiles, goods and effects placed at any stall or store
entered a 1 year lease contract with a stipulation not to insure against fire the
or space in the leased premises without first obtaining the written
chattels, merchandise, textiles, goods and effects placed at any stall or store or
consent and approval of the LESSOR. If the LESSEE obtain(s) the
space in the leased premises without first obtaining the written consent and
insurance thereof without the consent of the LESSOR then the
approval of the lessor. But it insured against loss by fire their merchandise
policy is deemed assigned and transferred to the LESSOR for its
inside the leased premises for P500,000 with the United Insurance Co., Inc.
own benefit; x x x
without the written consent of CKS
3.   Notwithstanding the above stipulation in the lease contract, the Cha spouses
On the day the lease contract was to expire, fire broke out inside the leased
insured against loss by fire their merchandise inside the leased premises for
premises and CKS learning that the spouses procured an insurance wrote to
Five Hundred Thousand (P500,000.00) with the United Insurance Co., Inc.
United to have the proceeds be paid directly to them. But United refused so CKS
(hereinafter United) without the written consent of private respondents
filed against Spouses Cha and United.
CKS.
RTC: United to pay CKS the amount of P335,063.11 and Spouses Cha to
4.   On the day that the lease contract was to expire, fire broke out inside the
pay P50,000 as exemplary damages, P20,000 as attorney’s fees and costs of suit
leased premises.
CA: deleted exemplary damages and attorney’s fees
5.   When CKS learned of the insurance earlier procured by the Cha spouses
The issue is whether or not the paragraph 18 of the lease contract entered
without its consent, it wrote the insurer (United) a demand letter asking that
into between CKS and the Cha spouses is valid insofar as it provides that any
the proceeds of the insurance contract (between the Cha spouses and
fire insurance policy obtained by the lessee Cha spouses over their merchandise
United) be paid directly to CKS, based on its lease contract with Cha
inside the leased premises is deemed assigned or transferred to the lessor CKS.
spouses.
SC held that NO, the provision is void, as against public policy. CKS has
6.   United refused to pay CKS. Hence, the latter filed a complaint against the
no insurable interest over the merchandise.
Cha spouses and United.
A non-life insurance policy such as the fire insurance policy taken by
7.   RTC Manila, rendered a decision* ordering therein defendant United to pay
petitioner-spouses over their merchandise is primarily a contract of
CKS the amount of P335,063.11 and defendant Cha spouses to
indemnity. Insurable interest in the property insured must exist at the time the
pay P50,000.00 as exemplary damages, P20,000.00 as attorney’s fees and
insurance takes effect and at the time the loss occurs.
costs of suit.
The automatic assignment of the policy to CKS under the provision of the
8.   CA rendered a decision affirming the trial court decision, deleting however
lease contract previously quoted is void for being contrary to law and/or public
the awards for exemplary damages and attorneys fees. A motion for
policy. The proceeds of the fire insurance policy thus rightfully belong to the
reconsideration by United was denied on 29 March 1996.
spouses.
ISSUE:
DOCTRINE:
1.   Whether or not the paragraph 18 of the lease contract entered into between
The basis of such requirement of insurable interest in property insured is based
CKS and the Cha spouses is valid insofar as it provides that any fire
on sound public policy: to prevent a person from taking out an insurance policy
insurance policy obtained by the lessee Cha spouses over their merchandise
on property upon which he has no insurable interest and collecting the proceeds
inside the leased premises is deemed assigned or transferred to the lessor
of said policy in case of loss of the property.
CKS – NO, the provision is void, as against public policy. CKS has no
insurable interest over the merchandise.

RULING: WHEREFORE, the decision of the Court of Appeals in CA-G.R. CV


No. 39328 is SET ASIDE and a new decision is hereby entered, awarding the
proceeds of the fire insurance policy to petitioners Nilo Cha and Stella Uy-Cha. SO merchandise, without the consent of CKS, is a separate and distinct issue
ORDERED. which we do not resolve in this case.

RATIO:
 
1.   It is, of course, basic in the law on contracts that the stipulations contained
in a contract cannot be contrary to law, morals, good customs, public order
or public policy.
2.   Sec. 18 of the Insurance Code provides:
Sec. 18. No contract or policy of insurance on property shall be
enforceable except for the benefit of some person having an insurable
interest in the property insured.
3.   A non-life insurance policy such as the fire insurance policy taken by
petitioner-spouses over their merchandise is primarily a contract of
indemnity. Insurable interest in the property insured must exist at the time
the insurance takes effect and at the time the loss occurs. The basis of such
requirement of insurable interest in property insured is based on sound
public policy: to prevent a person from taking out an insurance policy
on property upon which he has no insurable interest and collecting the
proceeds of said policy in case of loss of the property.
4.   In such a case, the contract of insurance is a mere wager which is void
under Section 25 of the Insurance Code, which provides:
SECTION 25. Every stipulation in a policy of Insurance for the
payment of loss, whether the person insured has or has not any interest
in the property insured, or that the policy shall be received as proof of
such interest, and every policy executed by way of gaming or
wagering, is void.
5.   In the present case, it cannot be denied that CKS has no insurable interest in
the goods and merchandise inside the leased premises under the provisions
of Section 17 of the Insurance Code which provide.
Section 17. The measure of an insurable interest in property is the
extent to which the insured might be damnified by loss of injury
thereof."
6.   Therefore, respondent CKS cannot, under the Insurance Code a special law
be validly a beneficiary of the fire insurance policy taken by the petitioner-
spouses over their merchandise. This insurable interest over said
merchandise remains with the insured, the Cha spouses. The automatic
assignment of the policy to CKS under the provision of the lease
contract previously quoted is void for being contrary to law and/or
public policy. The proceeds of the fire insurance policy thus rightfully
belong to the spouses. The insurer (United) cannot be compelled to pay the
proceeds of the fire insurance policy to a person (CKS) who has no
insurable interest in the property insured.
7.   The liability of the Cha spouses to CKS for violating their lease contract in
that Cha spouses obtained a fire insurance policy over their own
008 DELSAN TRANSPORT LINES, INC. v. CA (CABUSORA) 61.   Caltex Philippines (Caltex) entered into a contract of affreightment with
November 15, 2001 | De Leon, Jr., J. | Measure of Indemnity the petitioner Delsan Transport Lines, Inc. (common carrier), for a one
year where Delsan would transport Caltex’s industrial fuel oil from the
PETITIONER: Delsan Transport Lines, Inc. Batangas-Bataan Refinery to different parts of the country.
RESPONDENTS: The Hon. Court Of Appeals and American Home Assurance 62.   Under the contract, Delsan took on board its vessel (MT Maysun)
Corporation 2,277.314 kiloliters of said oil to be transported to Caltex Oil Terminal in
Zamboanga City. Said shipment was insured with private respondent
SUMMARY: Caltex and Delsan entered into a contract of affreightment where American Home Assurance Corporation (AHAC).
Delsan would transport industrial fuel oil from the Batangas-Bataan Refinery to 63.   (14 Aug 1986) MT Maysun set sail to Zamboanga City.
Caltex Oil Terminal in Zamboanga City. Said shipment was insured by Caltex with 64.   (16 Aug 1986) MT Maysun sank near Panay Gulf and Cuyo East Pass in
AHAC. While in transit, Delsan’s ship (MT Maysun) sunk allegedly due to bad Visayas.
weather. AHAC paid Caltex for the lost shipment. When AHAC went after Delsan 65.   AHAC subsequently paid Caltex P5,096,635.57 (amount of insured cargo).
for reimbursement, the latter refused. Thus AHAC filed a complaint for complaint 66.   AHAC later demanded Delsan the same amount as an exercise of its right
for sum of money. The TC dismissed the case holding that Delsan was liable due to of subrogation under Art. 2207, New Civil Code.
force majeure. On appeal, the CA reversed the decision stating that there was no 67.   Delsan refused to pay, thus AHAC filed a complaint for collection of sum
force majeure. of money with the RTC, Makati.
The issues were (1) WON payment of AHAC to Caltex amounted to a and admission 68.   Trial Court: Dismissed the complaint reasoning that MT Maysun was
that MT Maysun was seaworthy and thus precluding any action AHAC may have seaworthy as determined by the Philippine Coast Guard per Survey
against Delsan (NO) and (2) WON the marine insurance policy in this case was Certificate Report No. M5-016-MH upon inspection during its annual
necessary (NO). dry-docking and that the incident was caused by unexpected inclement
Payment by AHAC to Caltex was only a waiver of AHAC’s right to enforce the weather condition or force majeure, thus exempting the common carrier
term of the implied warranty against Caltex under the marine insurance policy but (Delsan) from liability for the loss of its cargo.
does not mean admission of the vessel’s seaworthiness by AHAC as to foreclose 69.   CA: Reversed the TC Decision. CA gave credence to the weather report
recourse against Delsan for any liability under its contractual obligation as a issued by PAGASA which which showed that from 2-8 a.m. on 16 Aug
common carrier. The marine insurance policy is not necessary in this case and that 1986, the wind speed remained at 10-20 knots per hour while the waves
the subrogation receipt was already sufficient having showed both (1) relationship measured from 0.7-2 meters in height only in contrast to herein Delsan’s
of AHAC and Caltex, as insurer and assured, respectively and (2) amount paid to allegation that the waves were 20 ft. high.
settle the insurance claim. 70.   Delsan filed an MR which was however denied and hence this petition.
71.   Delsan invokes/alleges:
1.   Sec. 113, Insurance Code states that in every marine insurance upon a
DOCTRINE24: Payment by insurer to insured under a marine insurance policy ship or freight, or freightage, or upon any thing which is the subject of
means the waiver of the insurer’s right to enforce the term of the implied warranty marine insurance there is an implied warranty by the shipper that the
against the insured but does not mean waiver of the insurer’s right to subrogation. ship is seaworthy. Consequently, the insurer will not be liable to the
The presentation in evidence of the insurance policy is not indispensable before the assured for any loss under the policy in case the vessel would later on be
insurer may recover. The subrogation receipt, by itself, is sufficient to establish not found as not seaworthy at the inception of the insurance. The act of
only the relationship of the insurer and the insured, but also the amount paid to settle AHAC of paying Caltex the value of the lost cargo amounted to a tacit
the insurance. recognition that MT Maysun was seaworthy, otherwise, AHAC was not
legally liable to Caltex due to the latter’s breach of implied warranty under
the marine insurance policy that the vessel was seaworthy.
FACTS: 2.   CA erred in finding that MT Maysun was not seaworthy on the ground
                                                                                                                        that Francisco Berina, chief mate of the vessel, was not qualified. Under
24
 Contracts  of  marine  insurance  are  contracts  of  indemnity  which  means  its  purpose  is,  in  case  of  loss,  to   Sec. 116, Insurance Code, the implied warranty of seaworthiness of the
place   the   insured   in   the   same   situation   in   which   he   was   before   the   loss   subject   to   the   terms   and   vessel, which AHAC admitted as having been fulfilled by its payment of
conditions  of  the  policy.  Amount  of  indemnity  may  be  determined  after  the  loss  or  is  previously  fixed  in   the insurance proceeds to Caltex of its lost cargo, extends to the vessel’s
the   contract.   The   amount   of   insurance   fixed   in   the   policy   of   a   marine   insurance   is   not   the   exact   measure   complement. Despite Berina having merely a 2nd officer’s license, he was
of  indemnity  to  which  the  insured  is  entitled,  but  the  maximum  indemnity  which  he  might  obtain.  The   qualified to act as the vessel’s chief officer under Chapter IV(403),
insured  cannot  recover  in  excess  of  his  actual  loss.  (The  case  makes  no  mention  of  the  topic  this  case  was   Category III(a)(3)(ii)(aa) of the Philippine Merchant Marine Rules and
assigned   –   Measure   of   Indemnity.   This   doctrine   is   from   De   Leon,   and   it’s   the   closest   I   can   relate   the   case   Regulations. In fact, all the crew and officers of MT Maysun were
to.  Apologies.)  
exonerated in the administrative investigation conducted by the Board of company of the insurance claim. Consequently, the payment made by the
Marine Inquiry after the subject accident. AHAC (insurer) to Caltex (assured) operates as an equitable assignment to
3.   The doctrine in Home Insurance Corporation v. CA (i.e. failure of the the former of all the remedies which the latter may have against Delsan.
private respondent to present the insurance policy in evidence is allegedly 105.  From the nature of their business and for reasons of public policy, common
fatal to its claim inasmuch as there is no way to determine the rights of the
carriers are bound to observe extraordinary diligence in the vigilance over
parties thereto), should have been applied herein, since AHAC failed to
present the subject marine cargo insurance policy as evidence. the goods and for the safety of passengers transported by them, according to
all the circumstances of each case.
ISSUE/s: 1.   GR: In the event of loss, destruction or deterioration of the insured
10.   WON the payment made by the AHAC to Caltex for the insured value of goods, common carriers shall be responsible.
the lost cargo amounted to an admission that the vessel was seaworthy, thus Exception: If the same is brought about, among others, by flood,
precluding any action for recovery against the petitioner. – NO. Payment by storm, earthquake, lightning or other natural disaster or calamity.
AHAC to Caltex was only a waiver of AHAC’s right to enforce the term 2.   GR: If the goods are lost, destroyed or deteriorated, common
of the implied warranty against Caltex under the marine insurance policy carriers are presumed to have been at fault or to have acted
but does not mean admission of the vessel’s seaworthiness by AHAC as negligently.
to foreclose recourse against Delsan for any liability under its contractual Exception: If the common carrier proves that they observed
obligation as a common carrier. extraordinary diligence.
11.   WON the non-presentation of the marine insurance policy bars the 106.  Jaime Jarabe, captain of MT Maysun, and Berina testified that at around
complaint for recovery of sum of money for lack of cause of action. – NO. 3:15 a.m. a squall ("unos") carrying strong winds with an approximate
In this case, said insurance policy is not necessary. The subrogation receipt velocity of 30 knots per hour and big waves averaging 18-20 ft. high caused
was already sufficient and it showed both (1) relationship of AHAC and the vessel to sink. However, this tale was effectively rebutted PAGASA’s
Caltex, as insurer and assured, respectively and (2) amount paid to settle weather report. (See Fact #9) Thus, Delsan’s defense of force majeure
the insurance claim. cannot be sustained. Also, said witnesses as employees of Delsan, cannot be
expected to testify against the latter. (See Fact #11.2) Exoneration of MT
RULING: WHEREFORE, the instant petition is DENIED. The Decision dated June Maysun’s officers and crew to liability is only with respect to administrative
17, 1996 of the Court of Appeals in CA-G.R. CV No. 39836 is AFFIRMED. Costs liabilities and does not operate to absolve Delsan’s civil liabilities arising
against the petitioner. SO ORDERED. from its negligence as a common carrier. (See Fact #11.2)
107.  Evidence certificates that MT Maysun was fit for voyage at the time of
RATIO: dry-docking and inspection by the Philippine Coast Guard, was also not
103.  (First Issue) The payment made by the AHAC for the insured value of the credited by the court to prove the vessel’s fitness for voyage at the time of
lost cargo operates as waiver of its (AHAC) right to enforce the term of the the commencement of the voyage. “Seaworthiness relates to a vessel's
implied warranty against Caltex under the marine insurance policy. actual condition. Neither the granting of classification or the issuance of
However, the same cannot be validly interpreted as an automatic certificates establishes seaworthiness.”
admission of the vessel's seaworthiness by the AHAC as to foreclose 108.  (Second Issue) The marine insurance policy is not necessary in this case.
recourse against the Delsan for any liability under its contractual obligation The subrogation receipt, by itself, is sufficient to establish (1) the
as a common carrier. Said payment grants AHAC subrogatory right to relationship of AHAC, as insurer, and Caltex, as the assured shipper of
exercise legal remedies available to Caltex against Delsan, the common the lost cargo and (2) the amount paid to settle the insurance claim. The
carrier by virtue of Art. 2207, Civil Code25. right of subrogation accrues simply upon payment by the insurance
104.  The right of subrogation has its roots in equity. It is not dependent upon, company of the insu rance claim.
nor does it grow out of, any privity of contract or upon written 109.  The presentation of the insurance policy was necessary in the case of Home
assignment of claim. It accrues simply upon payment by the insurance Insurance Corporation v. CA because the shipment therein passed through
several stages with different parties involved in each stage. In that case, in
                                                                                                                        the absence of proof of stipulations to the contrary, the hauler can be liable
25
  Art.   2207.   If   the   plaintiff's   property   has   been   insured,   and   he   has   received   indemnity   from   the   only for any damage that occurred from the time it received the cargo until
insurance  company  for  the  injury  or  loss  arising  out  of  the  wrong  or  breach  of  contract  complained  of,  
the   insurance   company   shall   be   subrogated   to   the   rights   of   the   insured   against   the   wrongdoer   or   the  
it finally delivered it to the consignee. It cannot be held responsible for the
person  who  has  violated  the  contract.  If  the  amount  paid  by  the  insurance  company  does  not  fully  cover   handling of the cargo before it actually received it. The insurance contract,
the  injury  or  loss,  the  aggrieved  party  shall  be  entitled  to  recover  the  deficiency  from  the  person  causing   which was not presented in evidence in that case would have indicated the
the  loss  or  injury.  
scope of the insurer's liability, if any, since no evidence was adduced
indicating at what stage in the handling process the damage to the
cargo was sustained.
110.  In this case, there is no doubt that the cargo of oil belonging to Caltex was
lost while on board MT Maysun, which was owned by Delsan. Thus,
Delsan cannot deny its liability.
 
009 GAISANO CAGAYAN, INC. v. INSURANCE COMPANY OF Insurance Company failed to present sufficient evidence to prove its cause of
NORTH AMERICA (Callueng) action. Hence, the claim of subrogation to Gaisano Cagayan's case for recovery
June 8, 2006 | Austria-Martinez, J. | Measure of Indemnity of the amount of P535,613.00 for LSPI is fatal.

PETITIONER: Gaisano Cagayan, Inc. DOCTRINE: A vendor or seller retains an insurable interest in the property sold
RESPONDENTS: Insurance Company of North America so long as he has any interest therein, in other words, so long as he would suffer
by its destruction, as where he has a vendor's lien.
SUMMARY: IMC is the maker of Wrangler Blue Jeans. Levi Strauss (Phils.)  
Inc. LSPI is the local distributor of products bearing trademarks owned by Levi FACTS:
Strauss & Co.. IMC and LSPI separately obtained from Insurance Company of 1.   Intercapitol Marketing Corporation (IMC) is the maker of Wrangler Blue
North America fire insurance policies with book debt endorsements. The Jeans. Levi Strauss (Phils.) Inc. LSPI is the local distributor of products
insurance policies provide for coverage on "book debts in connection with bearing trademarks owned by Levi Strauss & Co.. IMC and LSPI separately
ready-made clothing materials which have been sold or delivered to various
obtained from Insurance Company of North America (Insurance Company)
customers and dealers of the Insured anywhere in the Philippines." The policies
fire insurance policies with book debt endorsements.
defined book debts as the "unpaid account still appearing in the Book of Account
2.   The insurance policies provide for coverage on "book debts in connection
of the Insured 45 days after the time of the loss covered under this Policy." with ready-made clothing materials which have been sold or delivered to
Gaisano Cagayan is a customer and dealer of the products of IMC and LSPI. The various customers and dealers of the Insured anywhere in the Philippines."
Gaisano Superstore Complex in Cagayan de Oro City, owned by Gaisano, was 3.   The policies defined book debts as the "unpaid account still appearing in the
consumed by fire. Included in the items lost or destroyed in the fire were stocks Book of Account of the Insured 45 days after the time of the loss covered
of ready-made clothing materials sold and delivered by IMC and under this Policy."
LSPI. Insurance Company filed a complaint for damages against Gaisano. It 4.   The policies also provide for the following conditions:
alleges that IMC and LSPI filed with Insurance Company their claims under 1.   Warranted that the Company shall not be liable for any unpaid account in respect of
their respective fire insurance policies with book debt endorsements; that there the merchandise sold and delivered by the Insured which are outstanding at the date
are unpaid accounts of Gaisano with IMC for P2,119,205.00 while with LSPI of loss for a period in excess of six (6) months from the date of the covering invoice
for P535,613.00; Insurance Company paid the claims of IMC and LSPI and, by or actual delivery of the merchandise whichever shall first occur.
2.   Warranted that the Insured shall submit to the Company within twelve (12) days
virtue thereof, , Insurance Company was subrogated to their rights against after the close of every calendar month all amount shown in their books of accounts
Gaisano. Gaisano Cagayan contends that it could not be held liable because the as unpaid and thus become receivable item from their customers and dealers.
property covered by the insurance policies were destroyed due to fortuities event 5.   Gaisano Cagayan, Inc. (Gaisano Cagayan) is a customer and dealer of the
or force majeure. The RTC dismissed Insurance Company’s complaint. But, the products of IMC and LSPI. On February 25, 1991, the Gaisano Superstore
CA rendered its decision setting aside the decision of the RTC. ISSUE: WoN Complex in Cagayan de Oro City, owned by Gaisano, was consumed by
Gaisano Cagayan is liable for the unpaid accounts. YES, because the insurance fire. Included in the items lost or destroyed in the fire were stocks of ready-
in this case is not for loss of goods by fire but for Gaisano Cagayan's accounts made clothing materials sold and delivered by IMC and LSPI.
with IMC and LSPI that remained unpaid 45 days after the fire. Gaisano 6.   On February 4, 1992, Insurance Company filed a complaint for damages
Cagayan's argument that it is not liable because the fire is a fortuitous event against Gaisano. It alleges that IMC and LSPI filed with , Insurance
under Article 1174 of the Civil Code is misplaced. As held earlier, Gaisano Company their claims under their respective fire insurance policies with
Cagayan bears the loss under Article 1504 (1) of the Civil Code. Moreover, it book debt endorsements; that as of February 25, 1991, the unpaid accounts
must be stressed that the insurance in this case is not for loss of goods by fire but of Gaisano on the sale and delivery of ready-made clothing materials with
for Gaisano Cagayan's accounts with IMC and LSPI that remained unpaid 45 IMC was P2,119,205.00 while with LSPI it was P535,613.00; that ,
days after the fire. Accordingly, Gaisano Cagayan's obligation is for the payment Insurance Company paid the claims of IMC and LSPI and, by virtue
of money. As correctly stated by the CA, where the obligation consists in the thereof, , Insurance Company was subrogated to their rights against
payment of money, the failure of the debtor to make the payment even by reason Gaisano; that , Insurance Company made several demands for payment
of a fortuitous event shall not relieve him of his liability. Thus, whether fire is a upon Gaisano but these went unheeded.
fortuitous event or Gaisano Cagayan was negligent are matters immaterial to this 7.   In its Answer with Counter Claim, Gaisano Cagayan contends that it could
case. What is relevant here is whether it has been established that Gaisano not be held liable because the property covered by the insurance policies
Cagayan has outstanding accounts with IMC and LSPI. With respect to IMC, the were destroyed due to fortuities event or force majeure; that , Insurance
Insurance Company has adequately established its claim. However, as to LSPI, Company right of subrogation has no basis inasmuch as there was no breach
of contract committed by it since the loss was due to fire which it could not deemed to be over credit since an insurance "on credit" belies not only the
prevent or foresee; that IMC and LSPI never communicated to it that they nature of fire insurance but the express terms of the policies; that it was not
insured their properties; that it never consented to paying the claim of the credit that was insured since Insurance Company paid on the occasion of
insured. the loss of the insured goods to fire and not because of the non-payment by
8.   The parties failed to arrive at an amicable settlement. Gaisano Cagayan of any obligation; that, even if the insurance is deemed as
9.   The RTC rendered its decision dismissing, Insurance Company’s one over credit, there was no loss as the accounts were not yet due since no
complaint. It held that the fire was purely accidental; that the cause of the prior demands were made by IMC and LSPI against Gaisano Cagayan for
fire was not attributable to the negligence of the Gaisano Cagayan; that it payment of the debt and such demands came from Insurance Company only
has not been established that Gaisano Cagayan is the debtor of IMC and after it had already paid IMC and LSPI under the fire insurance policies.
LSPI; that since the sales invoices state that "it is further agreed that merely 2.   Gaisano Cagayan avers that despite delivery of the goods, Gaisano
for purpose of securing the payment of purchase price, the above-described Cagayan-buyer IMC and LSPI assumed the risk of loss when they secured
merchandise remains the property of the vendor until the purchase price is fire insurance policies over the goods.
fully paid", IMC and LSPI retained ownership of the delivered goods and 3.   Gaisano Cagayan submits that there is no subrogation in favor of Insurance
must bear the loss. Company as no valid insurance could be maintained thereon by IMC and
10.   Dissatisfied, Gaisano Cagayan appealed to the CA. The CA rendered its LSPI since all risk had transferred to Gaisano Cagayan upon delivery of the
decision setting aside the decision of the RTC. goods; that Gaisano Cagayan was not privy to the insurance contract or the
11.   The CA held that the sales invoices are proofs of sale, being detailed payment between Insurance Company and its insured nor was its consent or
statements of the nature, quantity and cost of the thing sold; that loss of the approval ever secured; that this lack of privity forecloses any real interest on
goods in the fire must be borne by Gaisano Cagayan since the part of Insurance Company in the obligation to pay, limiting its interest
the proviso contained in the sales invoices is an exception under Article to keeping the insured goods safe from fire.
1504 (1) of the Civil Code, to the general rule that if the thing is lost by a 4.   Insurance Company counters that while ownership over the ready- made
fortuitous event, the risk is borne by the owner of the thing at the time the clothing materials was transferred upon delivery to Gaisano Cagayan, IMC
loss under the principle of res perit domino; that Gaisano Cagayan's and LSPI have insurable interest over said goods as creditors who stand to
obligation to IMC and LSPI is not the delivery of the lost goods but the suffer direct pecuniary loss from its destruction by fire; that Gaisano
payment of its unpaid account and as such the obligation to pay is not Cagayan is liable for loss of the ready-made clothing materials since it
extinguished, even if the fire is considered a fortuitous event; that by failed to overcome the presumption of liability under Article 1265 of the
subrogation, the insurer has the right to go against Gaisano Cagayan; that, Civil Code; that the fire was caused through Gaisano Cagayan's negligence
being a fire insurance with book debt endorsements, what was insured was in failing to provide stringent measures of caution, care and maintenance on
the vendor's interest as a creditor. its property because electric wires do not usually short circuit unless there
12.   Gaisano Cagayan filed a motion for reconsideration but it was denied by the are defects in their installation or when there is lack of proper maintenance
CA. and supervision of the property; that Gaisano Cagayan is guilty of gross and
evident bad faith in refusing to pay Insurance Company's valid claim and
ISSUE/s: should be liable to Insurance Company for contracted lawyer's fees,
1.   WoN Gaisano Cagayan is liable for the unpaid accounts. YES, because the litigation expenses and cost of suit.
insurance in this case is not for loss of goods by fire but for Gaisano 5.   At issue is the proper interpretation of the questioned insurance policy.
Cagayan's accounts with IMC and LSPI that remained unpaid 45 days Gaisano Cagayan claims that the CA erred in construing a fire insurance
after the fire. policy on book debts as one covering the unpaid accounts of IMC and LSPI
since such insurance applies to loss of the ready-made clothing materials
sold and delivered to Gaisano Cagayan.
RULING: WHEREFORE, the petition is partly GRANTED. The assailed Decision
dated October 11, 2000 and Resolution dated April 11, 2001 of the Court of Appeals 6.   Indeed, when the terms of the agreement are clear and explicit that they do
in CA-G.R. CV No. 61848 are AFFIRMED with the MODIFICATION that the not justify an attempt to read into it any alleged intention of the parties, the
terms are to be understood literally just as they appear on the face of the
order to pay the amount of P535,613.00 to Insurance Company is DELETED for
contract. Thus, what were insured against were the accounts of IMC and
lack of factual basis.
LSPI with Gaisano Cagayan which remained unpaid 45 days after the loss
through fire, and not the loss or destruction of the goods delivered.
RATIO:
1.   Gaisano Cagayan contends that the insurance in the present case cannot be
7.   Gaisano Cagayan argues that IMC bears the risk of loss because it expressly where the obligation consists in the payment of money, the failure of the
reserved ownership of the goods by stipulating in the sales invoices that debtor to make the payment even by reason of a fortuitous event shall not
"[i]t is further agreed that merely for purpose of securing the payment of the relieve him of his liability. The rationale for this is that the rule that an
purchase price the above described merchandise remains the property of the obligor should be held exempt from liability when the loss occurs thru a
vendor until the purchase price thereof is fully paid." fortuitous event only holds true when the obligation consists in the delivery
8.   The Court is not persuaded. of a determinate thing and there is no stipulation holding him liable even in
9.   The present case clearly falls under paragraph (1), Article 1504 of the Civil case of fortuitous event. It does not apply when the obligation is pecuniary
Code: in nature.
(1)   ART. 1504. Unless otherwise agreed, the goods remain at the seller's risk until the 16.   Thus, whether fire is a fortuitous event or Gaisano Cagayan was negligent
ownership therein is transferred to the buyer, but when the ownership therein is are matters immaterial to this case. What is relevant here is whether it has
transferred to the buyer the goods are at the buyer's risk whether actual delivery has
been made or not, except that: been established that Gaisano Cagayan has outstanding accounts with IMC
i.   Where delivery of the goods has been made to the buyer or to a bailee and LSPI.
for the buyer, in pursuance of the contract and the ownership in the 17.   With respect to IMC, the Insurance Company has adequately established its
goods has been retained by the seller merely to secure performance by
claim. Exhibits "C" to "C-22"show that Gaisano Cagayan has an
the buyer of his obligations under the contract, the goods are at the
buyer's risk from the time of such delivery; outstanding account with IMC in the amount of P2,119,205.00. Exhibit
10.   Thus, when the seller retains ownership only to insure that the buyer will "E"is the check voucher evidencing payment to IMC. Exhibit "F"is the
pay its debt, the risk of loss is borne by the buyer. Accordingly, Gaisano subrogation receipt executed by IMC in favor of Insurance Company upon
Cagayan bears the risk of loss of the goods delivered. receipt of the insurance proceeds. All these documents have been properly
11.   IMC and LSPI did not lose complete interest over the goods. They have an identified, presented and marked as exhibits in court. The subrogation
insurable interest until full payment of the value of the delivered goods. receipt, by itself, is sufficient to establish not only the relationship of
Unlike the civil law concept of res perit domino, where ownership is the Insurance Company as insurer and IMC as the insured, but also the amount
basis for consideration of who bears the risk of loss, in property insurance, paid to settle the insurance claim. The right of subrogation accrues simply
one's interest is not determined by concept of title, but whether insured has upon payment by the insurance company of the insurance claim. Insurance
substantial economic interest in the property. Company's action against Gaisano Cagayan is squarely sanctioned by
12.   Section 13 of our Insurance Code defines insurable interest as "every Article 2207 of the Civil Code which provides:
interest in property, whether real or personal, or any relation thereto, or (1)   Art. 2207. If the plaintiff's property has been insured, and he has received indemnity
from the insurance company for the injury or loss arising out of the wrong or breach
liability in respect thereof, of such nature that a contemplated peril might of contract complained of, the insurance company shall be subrogated to the rights
directly damnify the insured." Parenthetically, under Section 14 of the same of the insured against the wrongdoer or the person who has violated the contract.
Code, an insurable interest in property may consist in: (a) an existing 18.   Gaisano Cagayan failed to refute Insurance Company's evidence.
interest; (b) an inchoate interest founded on existing interest; or (c) an 19.   As to LSPI, Insurance Company failed to present sufficient evidence to
expectancy, coupled with an existing interest in that out of which the prove its cause of action. No evidentiary weight can be given to Exhibit "F
expectancy arises. Levi Strauss", a letter dated April 23, 1991 from Gaisano Cagayan's
13.   Indeed, a vendor or seller retains an insurable interest in the property sold General Manager, Stephen S. Gaisano, Jr., since it is not an admission of
so long as he has any interest therein, in other words, so long as he would Gaisano Cagayan's unpaid account with LSPI. It only confirms the loss of
suffer by its destruction, as where he has a vendor's lien. In this case, the Levi's products in the amount of P535,613.00 in the fire that razed Gaisano
insurable interest of IMC and LSPI pertain to the unpaid accounts appearing Cagayan's building on February 25, 1991.
in their Books of Account 45 days after the time of the loss covered by the 20.   Moreover, there is no proof of full settlement of the insurance claim of
policies. LSPI; no subrogation receipt was offered in evidence. Thus, there is no
14.   Gaisano Cagayan's argument that it is not liable because the fire is a evidence that Insurance Company has been subrogated to any right which
fortuitous event under Article 1174 of the Civil Code is misplaced. As held LSPI may have against Gaisano Cagayan. Failure to substantiate the claim
earlier, Gaisano Cagayan bears the loss under Article 1504 (1) of the Civil of subrogation is fatal to Gaisano Cagayan's case for recovery of the amount
Code. of P535,613.00.
15.   Moreover, it must be stressed that the insurance in this case is not for loss of
goods by fire but for Gaisano Cagayan's accounts with IMC and LSPI that
remained unpaid 45 days after the fire. Accordingly, Gaisano Cagayan's
obligation is for the payment of money. As correctly stated by the CA,
010 GLARAGA V. SUN LIFE ASSURANCE (CANDELARIA) 74.   Glaraga alleges that defendant O.O. Hanson, the insurance agent (Hanson)
December 14, 1926 | J. Johns | Premium Payment who insured Juares, was the same person who collected the first premiums.
75.   Juares, during his life, defaulted in paying his second premium, upon the
PETITIONER: SUSANA GLARAGA allegation that Hanson allegedly made written instructions not to send the
RESPONDENTS: SUN LIFE ASSURANCE COMPANY OF CANADA and O.O. premium because Hanson would pay the premium, subject to Juares’
HANSON reimbursement.
76.   Sunlife alleges that the terms of the policy provided that the premiums
SUMMARY: Sunlife executed a life insurance policy in favor of Juares. The terms should be paid on the 1st of June and December of each year, or within a
of the insurance policy provided specific powers and limitations which shall be grace period of not more than 30 days. However, Juares defaulted with the
complied with in order to bind Sunlife (see FACTS, par. 6). Juares paid the first second premium because no payment was made on the due date and
premium, but defaulted in the second premium because Hanson, the insurance agent, within the grace period. By reason of the default, the policy elapsed and
sent a letter informing Juares that Hanson will pay the second premium, subject to was of no legal force or effect at the time of death of Juares.
77.   * RELEVANT PROVISION: “No persons, except the President,
reimbursement from Juares. However, no payment was made. When Juares died,
Managing-Director or Secretary has power to alter this contract, to extend
Glaraga, as the administratrix, made a demand for the proceeds of the policy. Sunlife
the time for paying a premium, to bind the Company by making any
refused, alleging that the policy was no longer in effect because of the default in the
promise or by receiving any representation or information not contained
second premium. The trial court ruled in favor of Glaraga.
in the application for this policy. No payment made to any person, except
in exchange for the Company’s official receipt, will be recognized by the
The issue is whether or not the policy was still in effect upon Glaraga’s demand.
Company. This policy does not take effect until the first premium has been
actually paid, during the life and good health of the insured.”
The Supreme Court stated that the policy was no longer in effect, and ruled in favor
78.   Sunlife alleges that Hanson was not authorized or empowered to modify the
of Sunlife. The Court emphasized that when an insurance policy specifically defines
terms of the policy, and that Hanson is not an official of Sunlife. Neither did
and limits the powers and duties of an agent, it shall bind the parties. Non-
the company know that a promise was made by Hanson, and it never
compliance, as stated in this case, shall render the policy ineffective. When Hanson
received from him any payment for the second premium.
sent the letter to Juares assuming the obligation to pay the premium was contrary to
79.   The trial court ruled in favor of Glaraga. Hence, this appeal by Sunlife.
the terms of the policy. Adding to the fact that no one paid the second premium, the
policy officially lapsed after the 30-day grace period. Hanson’s promise bound only
ISSUE/s:
him and the insured party. Sunlife is absolved from liability, as agreed in the
1.   W/N the insurance policy was still in effect upon Glaraga’s demand – NO.
contract, and Hanson will be held liable to Glaraga.
When an insurance policy specifically defines and limits the powers and
duties of an agent, it shall bind the parties. Non-compliance, as stated in
DOCTRINE: Where a life insurance company issued and delivered one of its
this case, shall render the policy ineffective. Under the relevant provisions
policies to the insured in and by which the powers of its pecial agents were limited
of the contract, payment made to any person, except in exchange for the
and defined, and the policy specified how and in what manner the premiums were to
company’s official receipt would not be recognized by the company, and
be paid and to whom they were to paid, in the absence of allegation and proof of an
that no person except those provided, had any power to alter the contract or
established custom or another rule of conduct ratified and approved by the company
extend the time for the premium, or bind the company itself by making of
the premiums on the policy must be paid at the time and in the way and manner
any promise.
specified in the policy and be forfeited by its own terms.
RULING: The SC reversed the trial court’s decision, but affirmed against Hanson.
FACTS: There was a written contract, in the form of the policy, duly signed and accepted by
72.   Sun Life Assurance (Sunlife), a domestic corporation, issued a life both parties, which expressed the manner the premiums are to be paid, and the terms
insurance policy on the life of Juares by which it promised to pay Php 5K to which specifies and limits the powers and dutiesof the agent to the delivery of the
the legal representatives of the insured, upon the condition that the policy official receipt of the company upon the payment to him of the amount of the
was in legal force and effect at the time of death of the insured. Juares died. premium. Without compliance with such terms, the subsequent acts after the
73.   Glaraga, the insured, was appointed as the administratrix of his estate, and perfection of the contract will have no effect; and non-compliance with the grace
she made a demand against Sunlife. Sunlife refused to pay, which led to period, as expressly stated, extinguished the conract.
Glaraga filing for collection and damages.
method by which Hanston collects premiums which would be binding upon
RATIO: the company.  
1.   It is conclusive that the second premium was never paid to Sunlife.   10.   Failure to comply with the contract after the default, the insurance policy
2.   The written promise by Hanson stated that Hanson assumed payment of the was no longer binding and effective against the Sunlife. However, Hanson
second premium, subject to the reimbursement from Juares.   is bound because he promised and agreed to premium. It is an established in
3.   On the other hand, the insurance policy, in writing and issued to Juares, the law on agency that when an agent acts beyond the authority given to
provided for 30 days grace period. If the insured party defaults and fails to him by the principal, it shall be binding on him as if he had entered into the
pay within the grace period, or if the balance due at the end of the term of contract by himself. Therefore, damage resulting to a third party, such as
the policy exceeds the coverage of the premium, then the policy shall lapse Glaraga, must be enforceable against him.  
and become void.    
4.   By its express terms, the non-payment of any premium when due or within  
the 30-day grace period, ipso facto causes the policy to lapse, and relieves    
the insurance company from all liability.  
5.   Under the relevant provision (see FACTS, par. 6), a payment made to any
person, except in exchange for the company’s official receipt would not be
recognized by the company, and that no person except those provided, had
any power to alter the contract or extend the time for the premium, or bind
the company itself by making of any promise.  
6.   In this case, there is no claim that in the writing of Hanson, he claimed that
he was acting for, or representing, the company, either as its agent or
otherwise. Juares relied on Hanson’s promise to pay the premium. But that
promise of Hanson, and of Hanson only, and was made by him to Juares,
and was made by him only.  
7.   There is no evidence that Sunlife ever ratified or approved Hanson’s letter,
or that it ever knew that it was written. It was not proven that Hanson or
anyone else paid the premium, or that Sunlife extended the time for its
payment. The letter was not the act of company or even that of its agent,
and even if it purported to be the act of the agent of the company, it
might well be doubted whether it would be binding upon the company,
in the absence of ratification or approval.  
8.   There is a marked distinction between the legal force and effect of the
powers and duties of a life insurance agent in soliciting insurance, and what
he says and does before the policy is issued, and what he says and does after
the policy is issued.  
a.   No written contract between the insured and insurer – insurance is
stopped to deny the authority of its agent as to acts previous to and
upon issuance of the policy; or  
b.   With written contract – specifically defined and limiting the
powers the duties of the agent, agent must comply to bind the
insurer  
9.   If Hansons was vested with all the powers of a general agent of the
company, such contract made by him would then be binding upon the
company, but there is no claim or pretense that Hanson was anything more
than a special agent with limited powers and dutes as the receipt of
premiums which are specifically defined by the express provisions of
the policy. Neither is there proof of an established custom as to manner or
011 ARCE v. CAPITAL INSURANCE (Castillo) 3.   November 27, 1965 - COMPANY sent to the INSURED Renewal
September 30, 1982 | Abad Santos, J. | Payment of premiums necessary for Certificate No. 47302 to cover the period December 5, 1965 to December 5,
effectivity of policy 1966. The COMPANY also requested payment of the corresponding
premium in the amount of P38.10.
4.   Anticipating that the premium could not be paid on time, the INSURED,
PLAINTIFF-APPELLEE: Pedro Arce thru his wife, promised to pay it on January 4, 1966. The COMPANY
DEFENDANT-APPELLANT: The Capital Insurance & Surety Co., Inc. accepted the promise but the premium was not paid on January 4, 1966. On
January 8, 1966, the house of the INSURED was totally destroyed by fire.
SUMMARY: The appellee owned a residential house which was insured with 5.   January 10, 1966 - NSURED's wife presented a claim for indemnity to the
the appellant COMPANY since 1961. In November 1965, the COMPANY sent COMPANY. She was told that no indemnity was due because the premium
to the INSURED a Renewal Certificate to cover the period from December 5, on the policy was not paid.
1965 to December 5,1966, and requested payment of the corresponding 6.   Nonetheless the COMPANY tendered a check for P300.00 as financial aid
premium. Anticipating that the premium could not be paid on time, the which was received by the INSURED's daughter, Evelina R. Arce. The
INSURED asked for an extension which was granted by the COMPANY. After voucher for the check which Evelina signed stated that it was "in full
the lapse of the requested extension, INSURED still failed to pay the premium. settlement (ex gratia) of the fire loss under Claim No. F-554 Policy No. F-
Thereafter, the house of the INSURED was totally destroyed by fire. Upon 24202.”
INSURED's presentation of claim for indemnity, he was told that no indemnity 7.   INSURED and his wife went to the office of the COMPANY to have his
was due because the premium was not paid. The INSURED sued the signature on the check identified preparatory to encashment.
COMPANY for indemnity. The trial court held the COMPANY liable to 8.   At that time the COMPANY reiterated that the check was given "not as
indemnify the INSURED on the ground that since the COMPANY could have an obligation, but as a concession" because the renewal premium had
demanded payment of the premium, mutuality of obligation required that it not been paid. The INSURED cashed the check but then sued the
should be liable on the policy. Issue is WoN the Company is liable to indemnify COMPANY on the policy.
the Insured even if no premium was paid. The Court ruled in the negative 9.   The court a quo held that since the COMPANY could have demanded
stating that Section 72 of the Insurance Act as amended by R.A.. 3540 states payment of the premium, mutuality of obligation requires that it should
that "no policy issued by an insurance company is valid and binding unless and also be liable on its policy. The court a quo also held that the INSURED
until the premium thereof has been paid.” Because of this amendment, the legal was not bound by the signature of Evelina on the check voucher because he
regime changed in that unless the premium is paid there is no insurance. did not authorize her to sign the waiver.
DOCTRINE: It is obvious from both the Insurance Act, as amended, and the
ISSUES:
stipulation of the parties that time is of the essence in respect to the payment of
1.   WoN the Company is liable to indemnify the Insured even if no premium
the insurance premium so that if it is not paid the contract does not take effect
was paid – NO because Section 72 of the Insurance Act as amended by
unless there is still another stipulation to the contrary. In the instant case, the
R.A.. 3540 states that "no policy issued by an insurance company is valid
INSURED was given a grace period to pay the premium but the period
and binding unless and until the premium thereof has been paid."
having expired with no payment made, he cannot insist that the
COMPANY is nonetheless obligated to him.  
RULING: WHEREFORE, the decision of the court a quo is reversed; the appellee's
complaint is dismissed. No special pronouncement as to costs.
 
FACTS:
1.   In Civil Case No. 66466 of the Court of First Instance of Manila, the RATIO:
Capital Insurance and Surety Co., Inc., (COMPANY) was ordered to pay 1.   Capital Insurance and Surety Co., Inc. vs. Delgado: appellants failed to pay
Pedro Arce (INSURED) the proceeds of a fire insurance policy. Not a balance of P583.95 on the premium charges due, notwithstanding
satisfied with the decision, the company appealed to this Court on questions demands made upon them. As with the issuance of the policy to appellants
of law. the same became effective and binding upon the contracting parties, the
2.   The INSURED was the owner of a residential house in Tondo, Manila, latter can not avoid the obligation of paying the premiums agreed upon. It is
which had been insured with the COMPANY since 1961 under Fire Policy clear from the foregoing that appellants are under obligation to pay the
No. 24204 amount sued upon.
2.   Sec. 72 of the Insurance Act, as amended by R.A. No. 3540: SEC. 72. An
insurer is entitled to payment of premium as soon as the thing insured is
exposed to the perils insured against, unless there is clear agreement to
grant credit extension for the premium due. No policy issued by an
insurance company is valid and binding unless and until the premium
thereof has been paid.
3.   Parties in this case had stipulated: notwithstanding anything to the
contrary contained in the within policy, this insurance will be deemed valid
and binding upon the Company only when the premium and documentary
stamps therefor have actually been paid in full and duly acknowledged in an
official receipt signed by an authorized official/representative of the
Company.
4.   It is obvious from both the Insurance Act, as amended, and the stipulation
of the parties that time is of the essence in respect of the payment of the
insurance premium so that if it is not paid the contract does not take effect
unless there is still another stipulation to the contrary.
5.   In the instant case, the INSURED was given a grace period to pay the
premium but the period having expired with no payment made, he cannot
insist that the COMPANY is nonetheless obligated to him.
6.   Delgado was decided in the light of the Insurance Act before Sec. 72 was
amended. Prior to the amendment, an insurance contract was effective even
if the premium had not been paid so that an insurer was obligated to pay
indemnity in case of loss and correlatively he had also the right to sue for
payment of the premium.
7.   But the amendment to Sec. 72 has radically changed the legal regime in
that unless the premium is paid there is no insurance.
8.   With the foregoing, it is not necessary to dwell at length on the trial court's
second proposition that the INSURED had not authorized his daughter
Evelina to make a waiver because the INSURED had nothing to waive; his
policy ceased to have effect when he failed to pay the premium.
   
012 MAKATI TUSCANY CONDOMINIUM v. CA (CASTRO) March 1983, with a total premium of P466,103.05.
November 6, 1992 | Bellosillo, J. | Insurance Premium
2.   The premium was paid on installments on 12 March 1982, 20 May 1982, 21
PETITIONER: MAKATI TUSCANY CONDOMINIUM CORPORATION June 1982 and 16 November 1982, all of which were accepted by AHAC
RESPONDENTS: THE COURT OF APPEALS, AMERICAN HOME
ASSURANCE CO., represented by American International Underwriters 3.   On 10 February 1983, AHAC issued to Makati Tuscany an insurance policy
(Phils.), Inc., which replaced and renewed the previous policy, for a term covering
INSURANCE POLICY 2: 1 March 1983 to 1 March 1984. The premium
SUMMARY: Basically, this case involves 3 insurance policies (1982-1983, in the amount of P466,103.05 was again paid on installments on 13 April
1983-1984, and the source of the controversy 1984-1985). In these insurance 1983, 13 July 1983, 3 August 1983, 9 September 1983, and 21 November
policies, Makati Tuscany pays its premiums by installments which were 1983. All payments were likewise accepted
accepted by American Home Assuance. For the 3rd insurance policy, Makati
Tuscany refused to complete its payment of premium by claiming that because 4.   On 20 January 1984, the policy was again renewed for the period
of the arrangement of paying in installments, then the insurance policy was no INSURANCE POLICY 3: 1 March 1984 to 1 March 1985. On this
valid and binding because the said arrangement is proscribed by the insurance renewed policy, Makati Tuscany made two installment payments
code. Hence, American Home filed a collection case against Makati Tuscany for (amounting to 152,000), both accepted by AHAC. After the two
the payment of balance while Makati filed for a counterclaim refunding the said installments, Makati Tuscany refused to pay the balance of the premium
premiums paid. RTC ruled that he insurance policy is valid and that Makati
Tuscany should pay the balance which was affirmed by the CA with some 5.   Consequently, AHAC filed an action to recover the unpaid balance
modifications. CA further stated that Makati Tuscany cannot deny liability on amounting 314,103.05 (arrived by deducting the installments paid which is
the payment of other premiums while AHAC cannot also deny liability on the 152,000 from the amount of the policy which is 466,103.05)
insurance because it agreed with the set-up and it accepted the installment 6.   In its answer, Makati Tuscany admitted the issuance of the insurance policy.
payments. The issue now before the SC is W/N an arrangement to pay the It explained that it discontinued the payment premiums because the policy
premiums in installment will invalidate the contract of insurance and SC said did not contain a credit clause in its favor and the receipts of the installment
NO, because what merely the law precludes is the stipulation making the payments covering the policy for 1984-1985, as well as the two previous
contract valid despite non-payment of premiums at all. The installment policies, stated the following reservations:
arrangement is not contrary to law, morals, good customs, or public policy. a.   Acceptance of this payment shall not waive any of the company
rights to deny liability on any claim under the policy arising
DOCTRINE: Section 77 merely precludes the parties from stipulating that the before such payments or after the expiration of the credit clause of
policy is valid even if premiums are not paid, but does not expressly prohibit an the policy; and
agreement granting credit extension, and such an agreement is not contrary to b.   Subject to no loss prior to premium payment. If there be any
morals, good customs, public order or public policy). So is an understanding to loss such is not covered.
allow insured to pay premiums in installments not so proscribed. At the very
least, both parties should be deemed in estoppel to question the arrangement they 7.   Makati Tuscany claimed that the policy was never binding and valid, and no
have voluntarily accepted. risk attached to the policy. It then pleaded a counterclaim for 152,000
refund for the premiums already paid for 1984-1985. Later on it amended
  the counterclaim, sought the refund of 924,206 representing the premium
FACTS: payments for 1982-1985.
INSURED: (Petitioner in the SC but the Defendant in the lower court case):
MANILA TUSCANY CONDOMINIUM 8.   After some incidents, Makati Tuscany and AHAC moved for summary
INSURER: AHAC through American Underwriters (Plaintiff in the lower judgment
court)
1.   Sometime in early 1982, private respondent American Home Assurance Co. 9.   RTC dismissed the complaint and the counterclaim upon the following
(AHAC), represented by American International Underwriters (Phils.), Inc., fiindings:
issued in favor of petitioner Makati Tuscany Condominium Corporation a.   While it is true that the receipts issued to Makati Tuscany
(TUSCANY) an insurance policy on the latter's building and premises, for a contained the aforementioned reservations, it is equally true that
period beginning INSURANCE POLICY 1: 1 March 1982 and ending 1
payment of the premiums of the three aforementioned policies
(being sought to be refunded) were made during the lifetime or 12.   Makati Tuscany now asserts that its payment by installment of the
term of said policies, hence, it could not be said, inspite of the premiums for the insurance policies for 1982, 1983 and 1984 invalidated
reservations, that no risk attached under the policies. said policies because of the provisions of Sec. 77 of the Insurance Code26,
Consequently, Makati Tuscany's counterclaim for refund is not as amended, and by the conditions stipulated by the insurer in its receipts,
justified. disclaiming liability for loss for occurring before payment of premiums.
b.   As regards the unpaid premiums, in view of the reservation in the
receipts ordinarily issued by AHAC on premium payments the 13.   Hence this petition
only plausible conclusion is that AHAC has no right to demand
their payment after the lapse of the term of said policy on
March 1, 1985. Therefore, the defendant was justified in refusing ISSUE/s:
to pay the same. 1.   W/N the payment by installments of the premiums due on an insurance
policy invalidates a contract of insurance in view of Sec. 77 of PD 612 –
10.   Both parties appealed from the judgment of the trial court. Thereafter, the No,, Sec 77 merely precludes the parties from stipulating that the policy is
Court of Appeals rendered a decision modifying that of the trial court by valid even if premiums are not paid, but does not expreslly prohibit
ordering Makati Tuscany to pay the balance of the premiums due on granting credit extension in a form of payment by installments.
policy plus legal interest until fully paid, and affirming the denial of the
counterclaim. RULING: WHEREFORE, finding no reversible error in the judgment appealed
from, the same is AFFIRMED. Costs against petitioner.
11.   CA explained the decision by saying:
a.   The obligation to pay premiums when due is ordinarily as SO ORDERED.
indivisible obligation to pay the entire premium. Here, the parties
herein agreed to make the premiums payable in installments, and
there is no pretense that the parties never envisioned to make the RATIO:
insurance contract binding between them. It was renewed for two SUBJECT POLICIES ARE VALID DESPITE INSTALLMENT PAYMENTS
succeeding years, the second and third policies being a 1.   The records clearly show that Makati Tuscany and AHAC intended subject
renewal/replacement for the previous one. And the insured never insurance policies to be binding and effective notwithstanding the staggered
informed the insurer that it was terminating the policy because the payment of the premiums. The initial insurance contract entered into in
terms were unacceptable. 1982 was renewed in 1983, then in 1984. In those three (3) years, the
insurer accepted all the installment payments. Such acceptance of
b.   While it may be true that under Section 77 of the Insurance Code, payments speaks loudly of the insurer's intention to honor the policies
the parties may not agree to make the insurance contract valid it issued to Makati Tuscany. Certainly, basic principles of equity and
and binding without payment of premiums, there is nothing in fairness would not allow the insurer to continue collecting and
said section which suggests that the parties may not agree to accepting the premiums, although paid on installments, and later deny
allow payment of the premiums in installment, or to consider liability on the lame excuse that the premiums were not prepared in
the contract as valid and binding upon payment of the first full.
premium. Otherwise, the court would allow the insurer to renege
on its liability under the contract, had a loss incurred (sic) before                                                                                                                        
completion of payment of the entire premium, despite its voluntary 26
  Sec.   77.   An   insurer   is   entitled   to   the   payment   of   the   premium   as  
acceptance of partial payments, a result eschewed by a basic
considerations of fairness and equity.
soon   as   the   thing   is   exposed   to   the   peril   insured   against.  
Notwithstanding  any  agreement  to  the  contrary,  no  policy  or  contract  
c.   the insurance contract became valid and binding upon payment of of   insurance   issued   by   an   insurance   company   is   valid   and   binding  
the first premium, and AHAC could not have denied liability on unless   and   until   the   premium   thereof   has   been   paid,   except   in   the  
the ground that payment was not made in full, for the reason
that it agreed to accept installment payment. case   of   a   life   or   an   industrial   life   policy   whenever   the   grace   period  
provision  applies  
2.   While the import of Section 77 is that prepayment of premiums is strictly
required as a condition to the validity of the contract, The court is not
prepared to rule that the request to make installment payments duly
approved by the insurer, would prevent the entire contract of insurance
from going into effect despite payment and acceptance of the initial
premium or first installment. Section 78 of the Insurance Code in effect
allows waiver by the insurer of the condition of prepayment by making an
acknowledgment in the insurance policy of receipt of premium as
conclusive evidence of payment so far as to make the policy binding despite
the fact that premium is actually unpaid.
3.   Section 77 merely precludes the parties from stipulating that the policy
is valid even if premiums are not paid, but does not expressly prohibit
an agreement granting credit extension, and such an agreement is not
contrary to morals, good customs, public order or public policy). So is
an understanding to allow insured to pay premiums in installments not
so proscribed. At the very least, both parties should be deemed in
estoppel to question the arrangement they have voluntarily accepted.
4.   Lastly, Makati Tuscany’s reliance on Arce v. Capital Surety is unavailing,
for the facts therein are substantially different because there was no
premium paid at all in Arce. While in this case Makati Tuscany paid the
initial installment and thereafter made staggered payments.
5.   It appearing from the peculiar circumstances that the parties actually
intended to make three (3) insurance contracts valid, effective and
binding, Makati Tuscany may not be allowed to renege on its obligation
to pay the balance of the premium after the expiration of the whole
term of the third policy (No. AH-CPP-9210651) in March 1985.
6.   Moreover, as correctly observed by the appellate court, where the risk
is entire and the contract is indivisible, the insured is not entitled to a
refund of the premiums paid if the insurer was exposed to the risk
insured for any period, however brief or momentary.
   
013 Areola v. CA (COSCOLLUELA) same credited “a return premium of P1,609.65 plus documentary stamps
September 22, 1994 | Romero, J. | Premium Payment and premium tax” to the account of Areola.
84.   Shocked, Areola confronted the representative of Prudential, Carlito Ang.
PETITIONER: Santos B. Areola and Lydia D. Areola Areola still failed to receive any official receipt from Prudential
RESPONDENTS: Court of Appeals, Prudential Guarantee and Assurance, Inc. 85.   Areola sent a letter demanding from Prudential that he be insured under the
same terms and conditions of his previous policy commencing upon the
receipt of his letter, or that the current commercial rate of increase on the
SUMMARY: 7 months after the issuance of Santos Areola’s Personal Accident payment he had made under the provisional receipt be return within 5 days.
Insurance Policy, Prudential unilaterally cancelled the same since company records 86.   Areola received a letter from production manager Malapit, informing him of
revealed that Areola failed to pay his premiums evidenced by the lack of Official the “partial payment” of P1,000 he had made on the policy had been
Receipt which was supposed to be issued within 7 days after the Provisional Receipt. exhausted pursuant to the provisions of the Short Period Rate Scale. He was
Prudential offered to reinstate same policy it had previously cancelled and even also warned that failure to pay the balance, the company’s liability would
proposed to extend its lifetime, upon a finding that the cancellation was erroneous cease to operate.
and that the premiums were paid in full by Areola but were not remitted by Teofilo 87.   Prudential, in response to Areola’s letter, wrote a letter to the latter stating
M. Malapit, Prudential’s Baguio City branch manager. Areola and his wife filed a that the company was verifying whether the payment had in fact been
complaint with the trial court which rendered a decision in favor of them but this was remitted to the company and why no official receipt had been issued
reversed by the CA. therefor. Asst. Vice-President Mariano Ampil III emphasized that the OR
should have been issued 7 days from the issuance of the provisional receipt,
The issue is WoN Prudential should be held liable for the fraudulent act of Malapit. and because there was no such OR, there was reason to believe that no
payment had been made.
Yes, because a corporation acts solely thru its employees. The latter’s acts are 1.   Ampil apologized for the inconvenience and said they will hold Areola
considered as its own for which it can be held to account. It is beyond doubt that covered under the terms of the policy until such time that the matter is
Malapit represented the interests of Prudential and acted in its behalf. His act of cleared.
receiving the premiums collected is well within the provice of his authority. The fact 2.   Ampil wrote another letter confirming the total amount was received and
that Prudential was itself defrauded due to the anomalies that took place in its Baguio that Prudential was amenable to extending the policy up to December 17,
branch office, such as the non-accrual of said premiums to its account, does not free 1985.
the same from its obligation to Areola. Consequently, Prudential is liable by way of 88.   Unfortunately, as early as August 6, 1985 Areola and his wife already filed
damages for the fraudulent acts committed by Malapit that gave occasion to the a breach of contract with damages.
erroneous cancellation of subject insurance policy. Its earlier act of reinstating the 89.   Prudential admitted that the cancellation was due to Malapit’s failure to turn
insurance policy also can not obliterate the injury. over the premiums. However, it argued that by acknowledging the
inconvenience and taking steps to rectify its omission, Prudential had
DOCTRINE: A corporation acts solely thru its employees. The latter’s acts are complied with its obligation. Hence, Areola has no longer a cause of action.
considered as its own for which it can be held to account. 90.   Trial court ruled in favor of Areola ruling that Prudential was in bad faith in
unilaterally cancelling the insurance policy. Had Areola met an accident,
Prudential would certainly have disclaimed liability. It held that Prudential
FACTS:
breached its contract with Areola.
80.   Santos Areola (Areola), a lawyer from Dagupan bought from Prudential
91.   CA reversed the trial court. It held that Prudential was not motivated by bad
Guarantee and Assurance, Inc. (Prudential) a personal accident insurance
faith or malice and that the cancellation was based on what the existing
policy covering the one-year period between noon of November 28, 1984
records merely showed. In fact, it’s subsequent acts of rectifying the wrong
and noon of November 28, 1985.
indicate that Prudential did not ct precipitately or willfully to inflict a wrong
81.   Areola was allegedly not able to pay the total amount of P1,609.65 which
on Areola.
included the premium, documentary stamp and premium tax.
92.   Hence, this appeal.
82.   On December 17, 1984, Prudential issued a collector’s provisional receipt
93.   Areola argues that it was Malapit’s misappropriation of the premiums
to Areola for the total amount. It noted thereon that the collector’s
which was the proximate cause of the cancellation of the insurance policy.
provisional receipt will be confirmed by their official receipt.
Malapit’s act of signing and even sending the notice of cancellation himself,
83.   The Branch Manager of Baguio City sent Areola an Endorsement which
notwithstanding his personal knowledge of Areola’s full payment of
“cancelled flat” his insurance policy for non-payment of premium. The
premiums, further reinforces the allegation of bad faith. Subsequent and a creditor of the other, such that the obligation of one is dependent upon
reinstatement could not possibly absolve Prudential from liability, there the obligation of the other.
being an obvious breach of contract. 118.  Under the law governing reciprocal obligations the injured party is given a
94.   Prudential however argues that where reinstatement, the equitable relief choice between fulfillment or rescission of the obligation. The law also
sought by Areola was granted at an opportune moment i.e. prior to the filing entitles Areola to payment of damages, regardless of whether he demands
of the complaint, he was left without a cause of action fulfillment or rescission.
119.  The nature of the damages is in the form of nominal damages contrary to
ISSUE/s: what was awarded by the trial court because o actual or substantial damage
111.  WoN Prudential should be held liable for its agent’s fraudulent act of or injury was inflicted on Areola at the time the insurance policy was
misappropriating the premiums paid by Areola – YES, because a cancelled. (Areola was awarded actual, moral, exemplary, and atty’s fees)
corporation acts solely thru its employees. The latter’s acts are considered
as its own for which it can be held to account.

RULING: WHEREFORE, the petition for review on certiorari is


hereby GRANTED and the decision of the Court of Appeals in CA-G.R. No. 16902  
on May 31, 1990, REVERSED. The decision of Branch 40, RTC Dagupan City, in    
Civil Case No. D-7972 rendered on June 30, 1987 is hereby REINSTATED subject
to the following modifications: (a) that nominal damages amounting to P30,000.00
be awarded petitioner in lieu of the damages adjudicated by court a quo; and (b) that
in the satisfaction of the damages awarded therein, respondent insurance company is
ORDERED to pay the legal rate of interest computed from date of filing of
complaint until final payment thereof.

SO ORDERED.

RATIO:
112.  Malapit’s fraudulent act of misappropriating the premiums paid by Areola
is beyond doubt directly imputable to Prudential. A corporation acts solely
thru its employees. The latter’s acts are considered as its own for which it
can be held to account.
113.  It is beyond doubt that Malapit represented the interests of Prudential and
acted in its behalf. His act of receiving the premiums collected is well
within the provice of his authority.
114.  Malapit’s failure to remit the premiums he received cannot constitute as a
defense for Prudential.
115.  The fact that Prudential was itself defrauded due to the anomalies that took
place in its Baguio branch office, such as the non-accrual of said premiums
to its account, does not free the same from its obligation to Areola.
Consequently, Prudential is liable by way of damages for the fraudulent acts
committed by Malapit that gave occasion to the erroneous cancellation of
subject insurance policy.
116.  Its earlier act of reinstating the insurance policy also can not obliterate the
injury.
117.  Prudential should be reminded that a contract of insurance creates reciprocal
obligations for both insurer and insured. Reciprocal obligations are those
which arise from the same cause and in which each party is both a debtor
014 TIBAY v. CA (Cruz) FACTS:
May 24, 1996 | Bellosillo, J. | Partial Payment of premiums 75.   Fortune Life and General Insurance Co., Inc. (FORTUNE) issued
Petitioners: Sps. Antonio A. Tibay & Violeta R. Tibay and Ofelia Roraldo, Fire Insurance Policy No. 136171 in favor of Violeta R. Tibay (Violeta)
Victorina Roraldo, Virgilio Roraldo, Myrna Roraldo and Rosabella Roraldo and/or Nicolas Roraldo (Nicolas) on their two-storey residential building
Respondent: CA and Fortune Life & General Insurance Co., Inc., located at 5855 Zobel Street, Makati City, together with all their personal
effects therein. The insurance was for P600,000.00 covering the period from
SUMMARY: FORTUNE issued Fire Insurance Policy in favor of Violeta 23 January 1987 to 23 January 1988. On 23 January 1987, of the total
and/or Nicolas on their two-storey residential building. The insurance was for premium of P2,983.50, Violeta Tibay only paid P600.00 thus leaving a
P600,000.00. Violeta only paid P600.00 out of the total premium of P2,983.50, considerable balance unpaid.
thus leaving a considerable balance unpaid. The insured building was completely 76.   The insured building was completely destroyed by fire. Two days later
destroyed by fire. Two days later Violeta paid the balance of the premium. On Violeta Tibay paid the balance of the premium. On the same day, she filed
the same day, she filed with FORTUNE a claim on the fire insurance policy In a with FORTUNE a claim on the fire insurance policy.
letter FORTUNE denied the claim of Violeta for violation of Policy Condition a.   Her claim was accordingly referred to its adjuster, Goodwill
No. 2 and of Sec. 77 of the Insurance Code. Violeta et al sued FORTUNE for Adjustment Services, Inc. (GASI), which immediately wrote
damages. The trial court ruled for Violeta et al and adjudged FORTUNE liable Violeta requesting her to furnish it with the necessary documents
for the total value of the insured building and personal properties The CA for the investigation and processing of her claim. To which Violeta
reversed this. The issue in this case is WoN fire insurance policy is valid, forthwith complied.
binding and enforceable upon mere partial payment of premium. The SC 77.   On 28 March 1987 she signed a non-waiver agreement with GASI to the
ruled in the negative. The Fire Insurance Policy provides for payment of effect that any action taken by the companies or their representatives in
premium in full. Accordingly, where the premium has only been partially paid investigating the claim made by the claimant for his loss which occurred at
and the balance paid only after the peril insured against has occurred, the 5855 Zobel Roxas, Makati on March 8, 1987, or in the investigating or
insurance contract did not take effect and the insured cannot collect at all on the ascertainment of the amount of actual cash value and loss, shall not waive
policy. The stipulations in the policy explicitly worded and taken in conjunction or invalidate any condition of the policies of such companies held by said
with Sec. 77 of the Insurance Code the payment of partial premium by the claimant, nor the rights of either or any of the parties to this agreement,
assured in this particular instance should not be considered the payment required and such action shall not be, or be claimed to be, an admission of liability
by the law and the stipulation of the parties. Rather, it must be taken in the on the part of said companies or any of them.
concept of a deposit to be held in trust by the insurer until such time that the full 78.   In a letter FORTUNE denied the claim of Violeta for violation of Policy
amount has been tendered and duly receipted for. In other words, as expressly Condition No. 2 and of Sec. 77 of the Insurance Code. Efforts to settle the
agreed upon in the contract, full payment must be made before the risk occurs case before the Insurance Commission proved futile.
for the policy to be considered effective and in force. Moreover, The case 79.   Violeta and the other petitioners sued FORTUNE for damages in the
of South Sea Surety and Insurance Company, Inc. v. Court of Appeals, speaks amount of P600,000.00 representing the total coverage of the fire insurance
only of two (2) statutory exceptions to the requirement of payment of the entire policy plus 12% interest per annum, P 100,000.00 moral damages, and
premium as a prerequisite to the validity of the insurance contract. These attorneys fees equivalent to 20% of the total claim.
exceptions are: a) in case the insurance coverage relates to life or industrial life 80.   The trial court ruled for Sps Tibay and adjudged FORTUNE liable for the
(health) insurance when a grace period applies, and b) when the insurer makes a total value of the insured building and personal properties in the amount of
written acknowledgment of the receipt of premium, this acknowledgment being P600,000.00 plus interest at the legal rate of 6% per annum from the filing
declared by law to, be then conclusive evidence of the premium payment. under of the complaint until full payment, and attorneys fees equivalent to 20% of
Sec. 77, as well as Sec. 78, until the premium is paid, and the law has not the total amount claimed plus costs of suit.
expressly excepted partial payments, there is no valid and binding 81.   The CA reversed the court a quo by declaring FORTUNE not to be liable
contract. Hence, in the absence of clear waiver of prepayment in full by the therein but ordering the return of the premium of P2,983.50 plus 12%
insurer, the insured cannot collect on the proceeds of the policy interest from when Violeta paid the balance of the premium (2 days after
DOCTRINE: For as long as the current Insurance Code remains unchanged and the fire) until full payment.
partial payment of premiums is not mentioned at all as among the exceptions 82.   Hence this petition for review.
provided in Secs. 77 and 78, no policy of insurance can ever pretend to be
efficacious or effective until premium has been fully paid. ISSUE/s:
13.   WoN fire insurance policy is valid, binding and enforceable upon mere
 
partial payment of premium – NO where the premium has only been an insurance contract with the insured. In the case before
partially paid and the balance paid only after the peril insured against has us, there is, quite unlike in Phoenix, a specific stipulation
occurred, the insurance contract did not take effect and the insured cannot that (t)his policy xxx is not in force until the premium has
collect at all on the policy been fully paid and duly receipted by the Company x x x.
ii.   It is correct to say that in Phoenix a contract was
RULING: WHEREFORE, the petition is DENIED and the assailed Decision of the perfected upon partial payment of the premium since the
Court of Appeals dated 24 March 1995 is AFFIRMED. parties had not otherwise stipulated that prepayment of
SO ORDERED. the premium in full was a condition precedent to the
existence of a contract.
RATIO: b.   In Phoenix, by accepting the initial payment of P3,000.00 and then
1.   Insurance is a contract whereby one undertakes for a consideration to later demanding the remainder of the premium without any other
indemnify another against loss, damage or liability arising from an precondition to its enforceability as in the instant case, the insurer
unknown or contingent event. in effect had shown its intention to continue with the existing
a.   The consideration is the premium, which must be paid at the time contract of insurance, as in fact it was enforcing its right to collect
and in the way and manner specified in the policy, and if not so premium, or exact specific performance from the insured. This is
paid, the policy will lapse and be forfeited by its own terms. not so here. By express agreement of the parties, no vinculum
2.   the Policy provides for payment of premium in full. Accordingly, where the juris or bond of law was to be established until full payment was
premium has only been partially paid and the balance paid only after the effected prior to the occurrence of the risk insured against.
peril insured against has occurred, the insurance contract did not take effect 5.   In Makati Tuscany Condominium Corp. v. Court of Appeals the parties
and the insured cannot collect at all on the policy. This is fully supported by mutually agreed that the premiums could be paid in installments, which in
Sec. 7727 of the Insurance Code fact they did for three (3) years, hence, this Court refused to invalidate the
3.   Apparently the crux of the controversy lies in the phrase unless and until the insurance policy.
premium thereof has been paid. For whatever judicial construction may be 6.   While it maybe true that under Section 77 of the Insurance Code, the parties
accorded the disputed phrase must ultimately yield to the clear mandate of may not agree to make the insurance contract valid and binding without
the law. The principle that where the law does not distinguish the court payment of premiums, there is nothing in said section which suggests that
should neither distinguish assumes that the legislature made no qualification the parties may not agree to allow payment of the premiums in installment,
on the use of a general word or expression. or to consider the contract as valid and binding upon payment of the first
4.   Violeta et al insists that FORTUNE is liable on the policy despite partial premium. Otherwise we would allow the insurer to renege on its liability
payment of the premium due and the express stipulation thereof to the under the contract, had a loss incurred (sic) before completion of payment
contrary. They rely heavily on the 1967 case of Philippine Phoenix and of the entire premium, despite its voluntary acceptance of partial payments,
Insurance Co., Inc. v. Woodworks, Inc. where the Court sustained the ruling a result eschewed by basic considerations of fairness and equity.
of the trial court that partial payment of the premium made the policy 7.   These two (2) cases, Phoenix and Tuscany, adequately demonstrate the
effective during the whole period of the policy. In that case, the insurance waiver, either express or implied, of prepayment in full by the insurer:
company commenced action against the insured for the unpaid balance on a impliedly, by suing for the balance of the premium as inPhoenix, and
fire insurance policy. In its defense the insured claimed that nonpayment of expressly, by agreeing to make premiums payable in installments as
premium produced the cancellation of the insurance contract. in Tuscany.
a.   The 1967 Phoenix case is not persuasive; neither is it decisive of 8.   But contrary to the stance taken by Violeta et al, there is no waiver express
the instant dispute. or implied in the case at bench. Precisely, the insurer and the insured
i.   The factual scenario is different.In Phoenix it was the expressly stipulated that (t)his policy including any renewal thereof and/or
insurance company that sued for the balance of the any indorsement thereon is not in force until the premium has been fully
premium, i.e., it recognized and admitted the existence of paid to and duly receipted by the Company x x x and that this policy shall
be deemed effective, valid and binding upon the Company only when the
                                                                                                                        premiums therefor have actually been paid in full and duly acknowledged.
27
 SEC.  77.  An  insurer  is  entitled  to  payment  of  the  premium  as  soon  as  the  thing  insured  is  exposed  to  
the   peril   insured   against.Notwithstanding   any   agreement   to   the   contrary,   no   policy   or   contract   of  
9.   Conformably with the aforesaid stipulations explicitly worded and taken in
insurance  issued  by  an  insurance  company  is  valid  and  binding  unless  and  until  the  premium  thereof  has   conjunction with Sec. 77 of the Insurance Code the payment of partial
been   paid,  except   in   the   case   of   a   life   or   an   industrial   life   policy   whenever   the   grace   period   provision   premium by the assured in this particular instance should not be considered
applies  (Italics  supplied).  
the payment required by the law and the stipulation of the parties. Rather, it 15.   In the desire to safeguard the interest of the assured, it must not be ignored
must be taken in the concept of a deposit to be held in trust by the insurer that the contract of insurance is primarily a risk-distributing device, a
until such time that the full amount has been tendered and duly receipted mechanism by which all members of a group exposed to a particular risk
for. In other words, as expressly agreed upon in the contract, full payment contribute premiums to an insurer. To ensure payment for losses, the law
must be made before the risk occurs for the policy to be considered mandates all insurance companies to maintain a legal reserve fund in favor
effective and in force. of those claiming under their policies.
10.   Thus, no vinculum juris whereby the insurer bound itself to indemnify the a.   It should be understood that the integrity of this fund cannot be
assured according to law ever resulted from the fractional payment of secured and maintained if by judicial fiat partial offerings of
premium. The insurance contract itself expressly provided that the policy premiums were to be construed as a legal nexus between the
would be effective only when the premium was paid in full. It would have applicant and the insurer despite an express agreement to the
been altogether different were it not so stipulated. Ergo, petitioners had contrary.
absolute freedom of choice whether or not to be insured by FORTUNE b.   What could prevent the insurance applicant from deliberately or
under the terms of its policy and they freely opted to adhere thereto. wilfully holding back full premium payment and wait for the risk
11.   In the construction of an insurance contract is the intention of the parties as insured against to transpire and then conveniently pass on the
expressed in the policy. Courts have no other function but to enforce the balance of the premium to be deducted from the proceeds of the
same. The rule that contracts of insurance will be construed in favor of the insurance?
insured and most strongly against the insurer should not be permitted to 16.   For as long as the current Insurance Code remains unchanged and
have the effect of making a plain agreement ambiguous and then construe it partial payment of premiums is not mentioned at all as among the
in favor of the insured. exceptions provided in Secs. 77 and 78, no policy of insurance can ever
a.   Verily, it is elemental law that the payment of premium is requisite pretend to be efficacious or effective until premium has been fully paid.
to keep the policy of insurance in force. If the premium is not paid 17.   Premium is the elixir vitae of the insurance business because by law the
in the manner prescribed in the policy as intended by the parties insurer must maintain a legal reserve fund to meet its contingent obligations
the policy is ineffective. Partial payment even when accepted as a to the public, hence, the imperative need for its prompt payment and full
partial payment will not keep the policy alive even for such satisfaction
fractional part of the year as the part payment bears to the whole 18.   Once payment of premium is left to the whim and caprice of the insured, as
payment. when the courts tolerate the payment of a mere P600.00 as partial
12.   Applying further the rules of statutory construction, the position maintained undertaking out of the stipulated total premium of P2,983.50 and the
by Violeta et al becomes even more untenable. balance to be paid even after the risk insured against has occurred, as
13.   The case of South Sea Surety and Insurance Company, Inc. v. Court of Violeta have done in this case, on the principle that the strength of
Appeals, speaks only of two (2) statutory exceptions to the requirement the vinculumjuris is not measured by any specific amount of premium
of payment of the entire premium as a prerequisite to the validity of the payment, we will surely wreak havoc on the business and set to naught what
insurance contract. These exceptions are: has taken actuarians centuries to devise to arrive at a fair and equitable
a.   in case the insurance coverage relates to life or industrial life distribution of risks and benefits between the insurer and the insured.
(health) insurance when a grace period applies, and 19.   The terms of the insurance policy constitute the measure of the insurers
b.   when the insurer makes a written acknowledgment of the receipt of liability. In the absence of statutory prohibition to the contrary, insurance
premium, this acknowledgment being declared by law to, be then companies have the same rights as individuals to limit their liability and to
conclusive evidence of the premium payment. impose whatever conditions they deem best upon their obligations not
14.   A maxim of recognized practicality is the rule that the expressed exception inconsistent with public policy. The validity of these limitations is by law
or exemption excludes others. Exceptio firm at regulim in casibus non passed upon by the Insurance Commissioner who is empowered to approve
exceptis. all forms of policies, certificates or contracts of insurance which insurers
a.   Thus, under Sec. 77, as well as Sec. 78, until the premium is paid, intend to issue or deliver. That the policy contract in the case at bench was
and the law has not expressly excepted partial payments, there is approved and allowed issuance simply reaffirms the validity of such policy,
no valid and binding contract. Hence, in the absence of clear particularly the provision in question.
waiver of prepayment in full by the insurer, the insured cannot
collect on the proceeds of the policy.    
015 American Home Assurance Co. v. Chua (Daguman) behalf payment of any premium which is due on such policy or contract of
June 28,1999 | Davide,Jr, J. | Premiums insurance at the time of its issuance or delivery or which becomes due thereon.
Section 78 of the Insurance Code explicitly provides: “An acknowledgment in a
PETITIONER: American Home Assurance Company policy or contract of insurance of the receipt of premium is conclusive evidence
RESPONDENT: Antonio Chua of its payment, so far as to make the policy binding, notwithstanding any
stipulation therein that it shall not be binding until the premium is actually paid.”
SUMMARY: On April 5, 1990, Antonio Chua renewed the fire insurance for
its stock-in-trade of his business, Moonlight Enterprises with American Home  
Assurance Company by issuing a check to its agent James Uy who delivered the FACTS:
Renewal Certificate to him. On April 6, 1990, Moonlight Enterprises was 83.   American Home Assurance Company (AHAC) is a domestic corporation
completely razed by fire. On April 10, 1990, an official receipt was issued and engaged in the insurance business. In 1990, Chua obtained a fire insurance
subsequently, a policy was issued covering March 25 1990 to March 25 1991 from AHAC covering the stock-in trade of his business, Moonlight
Antonio Chua filed an insurance claim with American Home and 4 other co- Enterprises. The insurance was to expire on 25 March 1990.
insurers (Pioneer Insurance and Surety Corporation, Prudential Guarantee and 84.   5 April 1990, Chua issued a PCIBank Check to AHAC’s agent, for the
Assurance, Inc. and Filipino Merchants Insurance Co) American Home refused renewal of the policy. The check was drawn and deposited in AHAC’s bank
alleging the no premium was paid RTC: favored Antonio Chua for paying by account.
way of check a day before the fire occurred CA: Affirmed in toto. Won there 85.   A corresponding official receipt was issued on April 10.
was a valid payment of premium, considering that Chua’s check was cashed 86.   Subsequently, a new insurance policy was issued, whereby AHAC
after the occurrence of the fire? YES. the renewal certificate issued to Chua undertook t indemnify Chua for any damage or loss arising from fire for the
contained the acknowledgment that premium had been paid. It is not disputed period March 25 1990- March 25 1991
that the check drawn by Chua in favor of AHAC and delivered to its agent was 87.   6 April 1990, Moonlight Enterprises was completely razed by fire. Chua
honored when presented and AHAC forthwith issued its official receipt to Chua filed an insurance claim with AHAC and 4 other co-insurers (Pioneer,
on 10 April 1990. Section 306 of the Insurance Code provides that any insurance Prudential, Filipino Merchants and Domestic Insurance- Insurance
company which delivers a policy or contract of insurance to an insurance agent Companies)
or insurance broker shall be deemed to have authorized such agent or broker to 88.   AHAC refused to honor the insurance despite demands. Chua filed an
receive on its behalf payment of any premium which is due on such policy or action before the trial court against AHAC.
contract of insurance at the time of its issuance or delivery or which becomes 89.   AHAC argued that there was no existing insurance contract when the fire
due thereon. In the instant case, the best evidence of such authority is the fact occurred since Chua did not pay the premium. And even assuming that
that AHAC accepted the check and issued the official receipt for the payment. It there is a contract, Chua violated several conditions: 1. submission of
is, as well, bound by its agent’s acknowledgment of receipt of payment. WoN fraudulent income tax returns and financial statements. 2. Failure to
Chua violated the policy by his submission of fraudulent documents and non- establish actual loss 3. Failure to notify AHAC of any insurance already
disclosure of the other existing insurance contracts; and finally? NO. The effected to cover the insured goods.
submission of the alleged fraudulent documents pertained to Chua’s income tax 90.   The trial court ruled in favor of Chua. It found Chua paid by way of check a
returns for 1987 to 1989. Chua, however, presented a BIR certification that he day before the fire occurred, which was deposited in AHAC’s account.
had paid the proper taxes for the said years. AHAC’s loss adjuster had known all There was even an acknowledged certificate issued by AHAC’s agent. The
along of the other existing insurance contracts, yet, he did not use that as basis fraudulent documents were limited to a disparity of official receipts inssued
for his recommendation of denial. The loss adjuster, being an employee of by the BIR and the income tax returns for years 1987 to 1989. As to Chua’s
AHAC, is deemed a representative of the latter whose awareness of the other failure to notify AHAC of other insurance contracts, it held that AHAC
insurance contracts binds AHAC failed to show such fraudulent omission was intentional and fraudulent.
91.   CA affirmed in toto, MR was denied.
DOCTRINE: The general rule in insurance laws is that unless the premium is 92.   [AHAC’s Argument]
paid the insurance policy is not valid and binding. The only exceptions are life 93.   AHAC cites the case of Arce v. Capital Insurance & Surety Co., Inc., where
and industrial life insurance we ruled that unless and until the premium is paid there is no insurance.
Section 306 of the Insurance Code provides that any insurance company which AHAC emphasizes that when the fire occurred on 6 April 1990 the
delivers a policy or contract of insurance to an insurance agent or insurance insurance contract was not yet subsisting pursuant to Article 1249 of the
broker shall be deemed to have authorized such agent or broker to receive on its Civil Code, which recognizes that a check can only effect payment once it
has been cashed. Although Chua testified that he gave the check on 5 April AHAC’s loss adjuster had known all along of the other existing insurance
to a certain James Uy, the check, drawn against a Manila bank and contracts, yet, he did not use that as basis for his recommendation of denial.
deposited in a Cagayan de Oro City bank, could not have been cleared by 6 The loss adjuster, being an employee of AHAC, is deemed a representative
April, the date of the fire. In fact, the official receipt issued for Chua’s of the latter whose awareness of the other insurance contracts binds AHAC
check payment was dated 10 April 1990, four days after the fire occurred.
94.   Citing jurisprudence, AHAC also contends that Chua’s non-disclosure of RULING: Whereby, petition is denied.
the other insurance contracts rendered the policy void. It underscores the
trial court’s neglect in considering the Commission on Audit’s certification RATIO:
that the BIR receipts submitted by Chua were, in effect, fake since they 1.   The general rule in insurance laws is that unless the premium is paid the
were issued to other persons insurance policy is not valid and binding. The only exceptions are life
95.   [Chua’s Argument] and industrial life insurance. Whether payment was indeed made is a
96.   Chua refutes the reason for AHAC’s denial of his claim. As found by the question of fact which is best determined by the trial court
trial court, AHAC’s loss adjuster admitted prior knowledge of Chua’s 2.   According to the trial court the renewal certificate issued to Chua contained
existing insurance contracts with the other insurance companies. the acknowledgment that premium had been paid. It is not disputed that the
Nonetheless, the loss adjuster recommended the denial of the claim, not check drawn by Chua in favor of AHAC and delivered to its agent was
because of the said contracts, but because he was suspicious of the honored when presented and AHAC forthwith issued its official receipt to
authenticity of certain documents which Chua submitted in filing his claim. Chua on 10 April 1990. Section 306 of the Insurance Code provides that
97.   To bolster his argument, Chua cites Section 66 of the Insurance Code, any insurance company which delivers a policy or contract of insurance
which requires the insurer to give a notice to the insured of its intention to to an insurance agent or insurance broker shall be deemed to have
terminate the policy fortyfive days before the policy period ends. In the authorized such agent or broker to receive on its behalf payment of any
instant case, AHAC opted not to terminate the policy. Instead, it renewed premium which is due on such policy or contract of insurance at the
the policy by sending its agent to Chua, who was issued a renewal time of its issuance or delivery or which becomes due thereon.
certificate upon delivery of his check payment for the renewal of premium. 3.   In the instant case, the best evidence of such authority is the fact that
At this precise moment the contract of insurance was executed and already AHAC accepted the check and issued the official receipt for the
in effect. He also claims that it is standard operating procedure in the payment. It is, as well, bound by its agent’s acknowledgment of receipt
provinces to pay insurance premiums by check when collected by insurance of payment. Section 78 of the Insurance Code explicitly provides:
agents.
98.   Hence, these issues. “An acknowledgment in a policy or contract of insurance of the
receipt of premium is conclusive evidence of its payment, so far as
ISSUE/s: to make the policy binding, notwithstanding any stipulation
1.   Won there was a valid payment of premium, considering that Chua’s check therein that it shall not be binding until the premium is actually
was cashed after the occurrence of the fire? YES. Section 306 of the paid.”
Insurance Code provides that any insurance company which delivers a
policy or contract of insurance to an insurance agent or insurance broker 4.   This Section establishes a legal fiction of payment and should be interpreted
shall be deemed to have authorized such agent or broker to receive on its as an exception to Section 77. Is Chua guilty of the policy violations
behalf payment of any premium which is due on such policy or contract of imputed against him? We are not convinced by AHAC’s arguments. The
insurance at the time of its issuance or delivery or which becomes due submission of the alleged fraudulent documents pertained to Chua’s income
thereon. In the instant case, the best evidence of such authority is the fact tax returns for 1987 to 1989. Chua, however, presented a BIR certification
that AHAC accepted the check and issued the official receipt for the that he had paid the proper taxes for the said years. The trial court and the
payment. It is, as well, bound by its agent’s acknowledgment of receipt of Court of Appeals gave credence to the certification and it being a question
payment. of fact, we hold that said finding is conclusive.
2.   WoN Chua violated the policy by his submission of fraudulent documents 5.   Ordinarily, where the insurance policy specifies as a condition the
and non-disclosure of the other existing insurance contracts; and finally? disclosure of existing co-insurers, nondisclosure thereof is a violation
NO. The submission of the alleged fraudulent documents pertained to that entitles the insurer to avoid the policy. This condition is common in
Chua’s income tax returns for 1987 to 1989. Chua, however, presented a fire insurance policies and is known as the “other insurance clause.”
BIR certification that he had paid the proper taxes for the said years. The purpose for the inclusion of this clause is to prevent an increase in the
moral hazard. We have ruled on its validity and the case of Geagonia v.    
Court of Appeals clearly illustrates such principle. However, we see an
exception in the instant case.
6.   Citing Section 29 of the Insurance Code, the trial court reasoned that Chua’s
failure to disclose was not intentional and fraudulent. The application of
Section 29 is misplaced. Section 29 concerns concealment which is
intentional. The relevant provision is Section 75, which provides that:

“A policy may declare that a violation of specified provisions


thereof shall avoid it, otherwise the breach of an immaterial
provision does not avoid the policy. contracts must be upon the
same subject matter and with the same interest and risk.”

7.   Indeed, Chua acquired several co-insurers and he failed to disclose this


information to AHAC. Nonetheless, AHAC is estopped from invoking
this argument. The trial court cited the testimony of AHAC’s loss
adjuster who admitted previous knowledge of the coinsurers.

Thus, COURT:
Q The matter of additional insurance of other companies,
was that ever discussed in your investigation?
A Yes, sir.
Q In other words, from the start, you were aware the
insured was insured with other companies like Pioneer
and so on?
A Yes, Your Honor.
Q But in your report you never recommended the denial of
the claim simply because of the non-disclosure of other
insurance? [sic]
A Yes, Your Honor.
Q In other words, to be emphatic about this, the only
reason you recommended the denial of the claim, you
found three documents to be spurious. That is your only
basis?
A Yes, Your Honor. [Emphasis supplied]

8.   Indubitably, it cannot be said that AHAC was deceived by Chua by the


latter’s non-disclosure of the other insurance contracts when AHAC
actually had prior knowledge thereof. AHAC’s loss adjuster had known
all along of the other existing insurance contracts, yet, he did not use
that as basis for his recommendation of denial. The loss adjuster, being
an employee of AHAC, is deemed a representative of the latter whose
awareness of the other insurance contracts binds AHAC. We, therefore,
hold that there was no violation of the “other insurance” clause by Chua.

 
016 UCPB GENERAL INSURANCE v. MASAGANA TELAMART (Dim) policies. Written notice was given to the addresses stated in the policies.
June 15, 1999 | J. Pardo | Actual payment of premium is needed 101.  On June 13, 1992, a fire razed Masagana's property covered by three of
the insurance policies UCPB issued. On the same day, Masagana
PETITIONER: UCPB General Insurance Co Inc. presented 5 manager’s checks to the cashier of UCPB worth
RESPONDENT: Masagana Telamart, Inc. P225,753.95, representing the premium for the renewal of the policies.
No notice of loss was filed by Masagana under the policies at this time
SUMMARY: UCPB issued 5 insurance policies covering Masagana Telamart’s 102.  On July 14, 1992, Masagana filed with UCPB its formal claim for
properties against fire from May 22, 1991 to May 22, 1992. UCPB decided not indemnification of the insured property. On the same day, UCPB returned
to renew the policies upon their expiration and sent notice to Masagana’s to Masagana the five (5) manager's checks that it tendered, and at the
insurance broker. On June 13, 1992, a fire razed the properties of Masagana same time rejected Masagana's claim for the reasons (a) that the
covered by those insurance policies. On the same day, Masagana presented 5 policies had expired and were not renewed, and (b) that the fire
checks to UCPB representing premiums for the renewal of the policies. A month occurred on June 13, 1992, before Masagana's tender of premium
later, Masagana filed a formal claim for indemnification for the properties razed payment.
by the fire. UCPB returned the 5 checks and rejected the claim of Masagana
103.  Masagana filed with the RTC of Makati a civil complaint against UCPB for
because the policies had expired and were not renewed when the fire occurred.
recovery of P18,645,000.00, representing the face value of the policies
Masagana filed a civil complaint against UCPB for the recovery of the value of
covering Masagana's insured property razed by fire, and for attorney's fees
the razed properties. The RTC and CA ruled in favor of Masagana, first stating
that in previous practice, Masagana was allowed a 60-90 day credit term for the 104.  UCPB filed an answer, stating that the complaint fails to state a cause of
renewal of its policies, second, that the acceptance of the late premium payment action, and that UCPB was not liable for insurance proceeds under the
suggested an understanding that payment could be made later, and third, that policies because at the time of the loss of the property due to fire, the
notice to the broker was not notice to Masagana.The issue before the SC is WoN policies had long expired and were not renewed.
the fire insurance polcies had been renewed upon payment of premiums on a
date later than the expiration of the policies. The Court ruled in the negative. 105.  The RTC ruled in favor of Masagana. Upon appeal, the CA affirmed the
Any insurance policy, other than life, issued originally or on renewal, is not valid ruling, holding that following previous practise, Masagana was allowed
and binding until actual payment of the premium. Any agreement to the contrary a sixty (60) to ninety (90) day credit term for the renewal of its policies,
is void. The parties may not agree expressly or impliedly on the extension of and that the acceptance of the late premium payment suggested an
credit or time to pay the premium and consider the policy binding before actual understanding that payment could be made later. Hence the appeal to the
payment. SC.

DOCTRINE: Notwithstanding any agreement to the contrary, no policy or 106.  Masagana submits that the CA correctly ruled that no timely notice of non-
contract of insurance issued by an insurance company is valid and binding unless renewal was sent. The notice of non-renewal sent to broker Zuellig which
and until the premium thereof has been paid, except in the case of a life or an claimed that it verbally notified the insurance agency but not Masagana
industrial life policy. itself did not suffice.

FACTS: [this case is shorter than my attention span] ISSUE/s:


WoN the fire insurance policies had been extended or renewed by an implied
99.   UCPB General Insurance Company Inc. issued five (5) insurance policies credit arrangement though actual payment of premium was tendered on a later
covering Masagana Telamart Inc.'s various property described therein date after the occurrence of the risk insured against. – NO. This is against
against fire, for the period from May 22, 1991 to May 22, 1992. Section 7728 of the Insurance Code.
100.  UCPB evaluated the policies and decided not to renew them upon                                                                                                                        
expiration of their terms on May 22, 1992. UCPB advised Masagana's 28
    Section   77.   An   insurer   is   entitled   to   payment   of   the   premium   as  
broker, Zuellig Insurance Brokers, Inc. of its intention not to renew the
soon   as   the   thing   insured   is   exposed   to   the   peril   insured   against.  
RULING: WHEREFORE, the Court hereby REVERSES and SETS ASIDE the
decision of the Court of Appeals. In lieu thereof, the Court renders judgment
dismissing respondent's complaint and petitioner's counterclaims thereto filed with
the Regional Trial Court, Branch 58, Makati City. Without costs.

RATIO:
60.   The answer is easily found in the Insurance Code. Any insurance policy,
other than life, issued originally or on renewal, is not valid and binding
until actual payment of the premium. Any agreement to the contrary is
void. The parties may not agree expressly or impliedly on the extension of
credit or time to pay the premium and consider the policy binding before
actual payment.

61.   The case of Malayan Insurance Co., Inc. vs. Cruz-Arnaldo, cited by the CA,
is not applicable. In that case, payment of the premium was in fact actually
made on December 24, 1981, and the fire occurred on January 18, 1982.

62.   In this case, the payment of the premium for renewal of the policies was
tendered on July 13, 1992, a month after the fire occurred on June 13,
1992. The assured did not even give the insurer a notice of loss within a
reasonable time after occurrence of the fire.

 
   

                                                                                                                                                                                                                                                                                       
Notwithstanding  any  agreement  to  the  contrary,  no  policy  or  contract  
of   insurance   issued   by   an   insurance   company   is   valid   and   binding  
unless   and   until   the   premium   thereof   has   been   paid,   except   in   the  
case   of   a   life   or   an   industrial   life   policy   whenever   the   grace   period  
provision   applies,   or   whenever   under   the   broker   and   agency  
agreements  with  duly  licensed  intermediaries,  a  ninety  (90)-­‐day  credit  
extension  is  given.  No  credit  extension  to  a  duly  licensed  intermediary  
should  exceed  ninety  (90)  days  from  date  of  issuance  of  the  policy.  
017 UCPB GENERAL INSURANCE vs. MASAGANA TELEMART (Eleazar) notwithstanding any stipulation therein that it shall not be binding until premium
April 4, 2001 | Davide, C.J. | Premium Payment is actually paid. Makati Tuscany v Court of Appeals- Section 77 may not apply
if the parties have agreed to the payment in installments of the premium and
PETITIONERS: UCPB GENERAL INSURANCE CO. INC. partial payment has been made at the time of loss. Section 78 allows waiver by
RESPONDENTS: MASAGANA TELAMART, INC. the insurer of the condition of prepayment and makes the policy binding despite
the fact that premium is actually unpaid.
SUMMARY: Masagana obtained from UCPB five (5) insurance policies on its Section 77 does not expressly prohibit an agreement granting credit extension.
Manila properties. The policies were effective from May 22, 1991 to May 22, At the very least, both parties should be deemed in estoppel to question the
1992. On June 13, 1992, Masagana’s properties were razed by fire. On July 13, arrangement they have voluntarily accepted.
1992, Masagana tendered five checks for as renewal premium payments. A The Tuscany case has provided another exception to Section 77 that the insurer
receipt was issued. On July 14, 1992, Masagana made its formal demand for may grant credit extension for the payment of the premium. If the insurer has
indemnification for the burned insured properties. UCPB then rejected granted the insured a credit term for the payment of the premium and loss occurs
Masagana’s claims under the argument that the fire took place before the tender before the expiration of the term, recovery on the policy should be allowed even
of payment. Hence Masagana filed a case. The Court of Appeals disagreed with though the premium is paid after the loss but within the credit term.
UCPB’s argument that Masagana’s tender of payment of the premiums on 13 Moreover, there is nothing in Section 77 which prohibits the parties in an
July 1992 did not result in the renewal of the policies, having been made within insurance contract to provide a credit term within which to pay the premiums.
the effective date of renewal as provided under Policy Condition No. 26, which That agreement is not against the law, morals, good customs, public order or
states: 26. Renewal Clause. -- Unless the company at least 45 days in advance of public policy. The agreement binds the parties.
the end of the policy period mails or delivers to the assured at the address shown It would be unjust if recovery on the policy would not be permitted against
in the policy notice of its intention not to renew the policy or to condition its UCPB, which had consistently granted a 60- to 90-day credit term for the
renewal upon reduction of limits or elimination of coverages, the assured shall payment of premiums. Estoppel bars it from taking refuge since Masagana
be entitled to renew the policy upon payment of the premium due on the relied in good faith on such practice. Estoppel then is the fifth exception.
effective date of renewal.
DOCTRINE: There are exceptions to IC Sec. 77.
Both the Court of Appeals and the trial court found that sufficient proof exists 1.   In case of a life or industrial life policy whenever the grace period
that Masagana, which had procured insurance coverage from UCPB for a provision applies [Sec. 77]
number of years, had been granted a 60 to 90-day credit term for the renewal of 2.   Any acknowledgment of the receipt of premiumis conclusive evidence
the policies. of payment [Sec. 78]
3.   If the parties have agreed to the payment ininstallments of the premium
The Supreme Court ruled against UCPB in the first case on the issue of whether and partial payment has been made at the time of loss [Makati Tuscany
the fire insurance policies issued by UCPB to the Masagana covering the period Condominium v. CA]
from May 22, 1991 to May 22, 1992 had been extended or renewed by an 4.   The insurer may grant credit extension for the payment of the premium
implied credit arrangement though actual payment of premium was tendered on [Makati Tuscany Condominium]
a later date and after the occurrence of the risk insured against. UCPB filed a 5.   Estoppel
motion for reconsideration. The Supreme Court, upon observing the facts,
affirmed that there was no valid notice of non-renewal of the policies in Sec. 77 merely precludes the parties from stipulating that the policy is valid even
question, as there is no proof at all that the notice sent by ordinary mail was if premiums are not paid, but does not expressly prohibit an agreement granting
received by Masagana. Also, the premiums were paid within the grace period. credit extension, and such an agreement is not contrary to morals, good customs,
The issue in this case is whether Section 77 of the Insurance Code of 1978 must public order or public policy.
be strictly applied to UCPB’s advantage despite its practice of granting a 60- to
90-day credit term for the payment of premiums? The SC said no. Section 77 of FACTS:
the Insurance Code provides: No policy or contract of insurance issued by an 1.   In the SC’s decision of 15 June 1999 in this case, they reversed and set
insurance company is valid and binding unless and until the premium thereof has aside the assailed decision of the Court of Appeals, which affirmed with
been paid. An exception to this section is Section 78 which provides: Any modification the judgment of the trial court
acknowledgment in a policy or contract of insurance of the receipt of premium is a.   (a) allowing Masagana to consign the sum of P225,753.95 as full
conclusive evidence of its payment, so far as to make the policy binding, payment of the premiums for the renewal of the five insurance
policies on Respondents properties; 5.   Both the Court of Appeals and the trial court found that sufficient proof
b.   (b) declaring the replacement-renewal policies effective and exists that Masagana, which had procured insurance coverage from UCPB
binding from 22 May 1992 until 22 May 1993; and for a number of years, had been granted a 60 to 90-day credit term for the
c.   (c) ordering Petitioner to pay Respondent P18,645,000.00 as renewal of the policies. Such a practice had existed up to the time the claims
indemnity for the burned properties covered by the renewal- were filed.
replacement policies. The modification consisted in the (1) 6.   In the decision of 15 June 1999, the SC defined the main issue to be
deletion of the trial courts declaration that three of the policies whether the fire insurance policies issued by UCPB to the Masagana
were in force from August 1991 to August 1992; and (2) reduction covering the period from May 22, 1991 to May 22, 1992 had been extended
of the award of the attorneys fees from 25% to 10% of the total or renewed by an implied credit arrangement though actual payment of
amount due the Respondent. premium was tendered on a later date and after the occurrence of the (fire)
2.   The material operative facts upon which the appealed judgment was based risk insured against.
are summarized by the Court of Appeals in its assailed decision as follows. 7.   The Court resolved this issue in the negative in view of Section 77 of the
a.   Masagana obtained from UCPB five (5) insurance policies on its Insurance Code and the decisions in Valenzuela v. Court of Appeals; South
properties in Pasay City and Manila. Sea Surety and Insurance Co., Inc. v. Court of Appeals; and Tibay v. Court
b.   All 5 policies reflect on their face the effectivity term: "from 4:00 of Appeals. Accordingly, we reversed and set aside the decision of the Court
P.M. of 22 May 1991 to 4:00 P.M. of 22 May 1992." of Appeals.
c.   On June 13, 1992, Masagana's properties located at 2410-2432 and 8.   Masagana seasonably filed a motion for the reconsideration of the adverse
2442-2450 Taft Avenue, Pasay City were razed by fire. verdict. It alleges in the motion that the Court had made in the decision their
d.   On July 13, 1992, Masagana tendered, and UCPB accepted, five own findings of facts, which are not in accord with those of the trial court
(5) Equitable Bank Manager's Checks in the total amount of and the Court of Appeals.
P225,753.45 as renewal premium payments for which Official 9.   The courts below correctly found that no notice of non-renewal was made
Receipt Direct Premium No. 62926 was issued by UCPB. within 45 days before 22 May 1992, or before the expiration date of the fire
e.   On July 14, 1992, Masagana made its formal demand for insurance policies. Thus, the policies in question were renewed by operation
indemnification for the burned insured properties. On the same of law and were effective and valid on 30 June 1992 when the fire
day, UCPB returned the five (5) manager's checks stating in its occurred, since the premiums were paid within the 60- to 90-day credit
letter that it was rejecting Masagana's claim on the following term.
grounds: 10.   Masagana likewise disagrees with the Court’s ruling that parties may
i.   "a) Said policies expired last May 22, 1992 and were not neither agree expressly or impliedly on the extension of credit or time to pay
renewed for another term; the premium nor consider a policy binding before actual payment. It urges
ii.   b) Defendant had put plaintiff and its alleged broker on the Court to take judicial notice of the fact that despite the express provision
notice of non-renewal earlier; and of Section 77 of the Insurance Code, extension of credit terms in premium
iii.   c) The properties covered by the said policies were payment has been the prevalent practice in the insurance industry.
burned in a fire that took place last June 13, 1992, or 11.   Most insurance companies, including UCPB, extend credit terms because
before tender of premium payment.” Section 77 of the Insurance Code is not a prohibitive injunction but is
3.   Hence Masagana filed this case. merely designed for the protection of the parties to an insurance
4.   The Court of Appeals disagreed with UCPB’s stand that Masagana’s tender contract. The Code itself, in Section 78, authorizes the validity of a policy
of payment of the premiums on 13 July 1992 did not result in the renewal of notwithstanding non-payment of premiums.
the policies, having been made beyond the effective date of renewal as 12.   Masagana also asserts that the principle of estoppel applies to UCPB.
provided under Policy Condition No. 26, which states: Despite its awareness of Section 77 UCPB persuaded and induced
a.   26. Renewal Clause. -- Unless the company at least forty five days Masagana to believe that payment of premium on the 60- to 90-day credit
in advance of the end of the policy period mails or delivers to the term was perfectly alright; in fact it accepted payments within 60 to 90 days
assured at the address shown in the policy notice of its intention after the due dates.
not to renew the policy or to condition its renewal upon reduction 13.   By extending credit and habitually accepting payments 60 to 90 days from
of limits or elimination of coverages, the assured shall be entitled the effective dates of the policies, UCPB has implicitly agreed to modify the
to renew the policy upon payment of the premium due on the tenor of the insurance policy and in effect waived the provision therein that
effective date of renewal. it would pay only for the loss or damage in case the same occurred after
payment of the premium. except in the case of a life or an industrial life policy whenever the
14.   UCPB filed an opposition to Masagana’s motion for reconsideration. It grace period provision applies.
argues that both the trial court and the Court of Appeals overlooked the fact 2.   This Section is a reproduction of Section 77 of P.D. No. 612 (The Insurance
that on 6 April 1992 UCPB sent by ordinary mail to Masagana a notice of Code) promulgated on 18 December 1974. In turn, this Section has its
non-renewal and sent by personal delivery a copy thereof to Masagana’s source in Section 72 of Act No. 2427 otherwise known as the Insurance Act
broker, Zuellig. Both courts likewise ignored the fact that Masagana was as amended by R.A. No. 3540, approved on 21 June 1963, which reads:
fully aware of the notice of non-renewal. A reading of Section 66 of the a.   SEC. 72. An insurer is entitled to payment of premium as soon as
Insurance Code readily shows that in order for an insured to be entitled to a the thing insured is exposed to the peril insured against, unless
renewal of a non-life policy, payment of the premium due on the effective there is clear agreement to grant the insured credit extension of the
date of renewal should first be made. Masagana’s argument that Section 77 premium due. No policy issued by an insurance company is valid
is not a prohibitive provision finds no authoritative support. and binding unless and until the premium thereof has been paid.
15.   Upon a meticulous review of the records and reevaluation of the issues (Underscoring supplied)
raised in the motion for reconsideration and the pleadings filed thereafter by 3.   It can be seen at once that Section 77 does not restate the portion of Section
the parties, the Court resolved to grant the motion for reconsideration. 72 expressly permitting an agreement to extend the period to pay the
16.   The following facts, as found by the trial court and the Court of Appeals, premium. But there exceptions to Sec. 77.
are indeed duly established: 4.   The first exception is provided by Section 77 itself, and that is, in case of a
a.   For years, UCPB had been issuing fire policies to the Masagana, life or industrial life policy whenever the grace period provision applies.
and these policies were annually renewed. 5.   The second is that covered by Section 78 of the Insurance Code, which
b.   UCPB had been granting Masagana a 60- to 90-day credit term provides:
within which to pay the premiums on the renewed policies. a.   SEC. 78. Any acknowledgment in a policy or contract of insurance
c.   There was no valid notice of non-renewal of the policies in of the receipt of premium is conclusive evidence of its payment, so
question, as there is no proof at all that the notice sent by ordinary far as to make the policy binding, notwithstanding any stipulation
mail was received by Masagana, and the copy thereof allegedly therein that it shall not be binding until premium is actually paid.
sent to Zuellig was ever transmitted to Masagana. 6.   A third exception was laid down in Makati Tuscany
d.   The premiums for the policies in question in the aggregate amount Condominium Corporation vs. Court of Appeals, wherein the Court ruled
of P225,753.95 were paid by Masagana within the 60- to 90-day that Section 77 may not apply if the parties have agreed to the payment in
credit term and were duly accepted and received by UCPB cashier. installments of the premium and partial payment has been made at the time
of loss. Thus:
ISSUE/s a.   We hold that the subject policies are valid even if the premiums
WoN Section 77 of the Insurance Code must be strictly applied to UCPB’s were paid on installments. The records clearly show that the
advantage despite its practice of granting a 60- to 90-day credit term for the payment petitioners and private respondent intended subject insurance
of premiums. policies to be binding and effective notwithstanding the staggered
payment of the premiums. The initial insurance contract entered
RULING: WHEREFORE, the Decision in this case of 15 June 1999 is into in 1982 was renewed in 1983, then in 1984. In those three
RECONSIDERED and SET ASIDE, and a new one is hereby entered DENYING the years, the insurer accepted all the installment payments. Such
instant petition for failure of UCPB to sufficiently show that a reversible error was acceptance of payments speaks loudly of the insurers intention to
committed by the Court of Appeals in its challenged decision, which is hereby honor the policies it issued to petitioner. Certainly, basic principles
AFFIRMED in toto. of equity and fairness would not allow the insurer to continue
collecting and accepting the premiums, although paid on
RATIO: installments, and later deny liability on the lame excuse that the
1.   Section 77 of the Insurance Code of 1978 provides: premiums were not prepaid in full.
a.   SEC. 77. An insurer is entitled to payment of the premium as soon 7.   Not only that. In Tuscany, the Court also quoted with approval the
as the thing insured is exposed to the peril insured following pronouncement of the Court of Appeals in its Resolution denying
against. Notwithstanding any agreement to the contrary, no policy the motion for reconsideration of its decision:
or contract of insurance issued by an insurance company is valid a.   While the import of Section 77 is that prepayment of premiums is
and binding unless and until the premium thereof has been paid, strictly required as a condition to the validity of the contract, We
are not prepared to rule that the request to make installment
payments duly approved by the insurer would prevent the entire
contract of insurance from going into effect despite payment and
acceptance of the initial premium or first installment. Section 78 of
the Insurance Code in effect allows waiver by the insurer of the
condition of prepayment by making an acknowledgment in the
insurance policy of receipt of premium as conclusive evidence of
payment so far as to make the policy binding despite the fact that
premium is actually unpaid. Section 77 merely precludes the
parties from stipulating that the policy is valid even if premiums
are not paid, but does not expressly prohibit an agreement granting
credit extension, and such an agreement is not contrary to morals,
good customs, public order or public policy. So is an
understanding to allow insured to pay premiums in installments not
so prescribed. At the very least, both parties should be deemed in
estoppel to question the arrangement they have voluntarily
accepted.
8.   By the approval of the aforequoted findings and conclusion of the Court of
Appeals, Tuscany has provided a fourth exception to Section 77, namely,
that the insurer may grant credit extension for the payment of the premium.
9.   This simply means that if the insurer has granted the insured a credit term
for the payment of the premium and loss occurs before the expiration of the
term, recovery on the policy should be allowed even though the premium is
paid after the loss but within the credit term.
10.   Moreover, there is nothing in Section 77 which prohibits the parties in an
insurance contract to provide a credit term within which to pay the
premiums. That agreement is not against the law, morals, good customs,
public order or public policy. The agreement binds the parties. Article 1306
of the Civil Code provides:
a.   ART. 1306. The contracting parties may establish such stipulations
clauses, terms and conditions as they may deem convenient,
provided they are not contrary to law, morals, good customs,
public order, or public policy.
11.   Finally in the instant case, it would be unjust and inequitable if recovery on
the policy would not be permitted against UCPB, which had consistently
granted a 60- to 90-day credit term for the payment of premiums despite its
full awareness of Section 77. Estoppel bars it from taking refuge under said
Section, since Masagana relied in good faith on such practice. Estoppel then
is the fifth exception to Section 77.

   
18 SOUTH SEA SURETY AND INSURANCE COMPANY INC. v. CA insurance policy for the plaintiffs logs, he is deemed to have been authorized by
(HORTALEZA) the South Sea Surety and Insurance Co., Inc. to receive the premium which is
June, 02, 1995 | VITUG, J. | PREMIUM PAYMENT due on its behalf. When therefore the insured logs were lost, the insured had
already paid the premium to an agent of the South Sea Surety and Insurance
PETITIONER: South Sea Surety and Insurance Company Inc. Co., Inc., which is consequently liable to pay the insurance proceeds under the
RESPONDENTS: Hon.Court of Appeals and Valenzuela Hardwood and Industrial policy it issued to the insured.
Supply Inc. DOCTRINE: Sec. 306. . . . Any insurance company which delivers to an insurance
SUMMARY: (take note of the dates!) Valenzuela Hardwood and Industrial Supply agent or insurance broker a policy or contract of insurance shall be deemed to have
Inc. (Hardwood) undertook to board lauan round logs numbering 940 on January 16 authorized such agent or broker to receive on its behalf payment of any
1984 with Seven Brothers. On January 20 1984 the logs were then insured with premium which is due on such policy of contract of insurance at the time of its
South Sea Surety and Insurance Company Inc. (South Sea) for 2 million pesos. The issuance or delivery or which becomes due thereon.
logs were then boarded into vessel M/V Seven Ambassador owned by Seven (the doctrine in my own words) When an insurer delivers the insurance policy to
Brothers Shipping Corporation. On January 24 1984, Hardwood gave a check to an individual or a person he/she is deemed to have been authorized by the
Victorio Chua who works as an administrative assistant for the past ten years and an insurer to receive the premium which is due on its behalf. Thus when the
agent for less than ten years of the Columbia Insurance Brokers, Ltd. The M/V payment is made to the agent it is deemed as if made to the insurer himself.
Seven Ambassador ultimately sank on January 25 1984, and led to Hardwood
claiming upon the obligations of South Sea to pay the value of the loss logs. South FACTS:
Sea denies the claim on the premise that they only received on January 30 1984 a 1.   Two issues on the subject of insurance are raised in this petition, that assails
check for P5,625.00 serving as the Payment for the Premium agreed upon. The the decision, that assails the decision of the Court of Appeals. (in CA G.R.
refusal of South Sea to indemnify Hardwood led to the filing of the case. Hardwood NO. CV- 20156), the first dealing on the requirement of premium
and South Sea argue on whether Chua was acting as an agent for the insured payment and the second relating to the agency relationship of parties
(Hardwood) or the insurer (South Sea). South Sea claims that since Chua received under that contract.
the insurance premiums as an agent of the Columbia Insurance Broker, acted as an 2.   It appears that on 16 January 1984, Valenzuela Hardwood and Industrial
agent of the insured under Section 301 of the insurance code: Sec. 301.”Any person Supply, Inc. entered into an agreement with the defendant Seven Brothers
who for any compensation, commission or other thing of value, acts, or aids in whereby the latter undertook to load on board its vessel M/V Seven
soliciting, negotiating or procuring the making of any insurance contract or in Ambassador the former's lauan round logs numbering 940 at the port of
placing risk or taking out insurance, on behalf of an insured other than himself, shall Maconacon, Isabela for shipment to Manila.
be an insurance broker within the intent of this Code, and shall thereby become liable 3.   On 20 January 1984, plaintiff insured the logs, against loss and/or,
to all the duties requirements, liabilities and penalties to which an insurance broker is damage with defendant South Sea Surety and Insurance Co., Inc. for
subject.” while Hardwood claims under the second paragraph of Sec 306 of the P2,000,000.00 end the latter issued its Marine Cargo Insurance Policy
insurance code Sec. 306. “. . . Any insurance company which delivers to an No. 84/24229 for P2,000,000.00 on said date.
insurance agent or insurance broker a policy or contract of insurance shall be 4.   On 24 January 1984, the plaintiff gave the check in payment of the
deemed to have authorized such agent or broker to receive on its behalf premium on the insurance policy to Mr. Victorio Chua.
payment of any premium which is due on such policy of contract of insurance at 5.   In the meantime, the said vessel M/V Seven Ambassador sank on 25
the time of its issuance or delivery or which becomes due thereon”. In this January 1984 resulting in the loss of the plaintiffs insured logs.
petition for review on certiorari brought by South Sea Surety and Insurance Co., Inc 6.   On 30 January 1984, a check for P5,625.00 to cover payment of the
it argues that it likewise should have been freed from any liability to Hardwood. The premium and documentary stamps due on the policy was tendered to the
Issue was WoN the appellate court erred (a) for having Supposedly disregarded insurer but was not accepted. Instead, the South Sea Surety and Insurance
Section 7729 of the insurance Code and (b) for holding Victorio Chua to have been an Co., Inc. cancelled the insurance policy it issued as of the date of inception
authorized representative of the insurer.? NO to both,When the appellant South for non payment of the premium due in accordance with Section 77 of the
Sea Surety and Insurance Co., Inc. delivered to Mr. Chua the marine cargo Insurance Code.
7.   On 2 February 1984, plaintiff demanded from defendant South Sea Surety
                                                                                                                       
29 and Insurance Co., Inc. the payment of the proceeds of the policy but the
 Sec. 77. An insurer is entitled to payment of the premium as soon as the thing insured is exposed to the latter denied liability under the policy. Plaintiff likewise filed a formal
peril insured against. Notwithstanding any agreement to the contrary, no policy or contract of insurance
issued by an insurance company is valid and binding unless and until the premium thereof has been paid,
claim with defendant Seven Brothers Shipping Corporation for the value of
except in the case of a life or an industrial life policy whenever the grace period provision applies.  
the lost logs but the latter denied the claim.
8.   In its decision, dated 11 May 1988, the trial court rendered judgment in insurance, on behalf of an insured other than himself,
favor of plaintiff Hardwood. shall be an insurance broker within the intent of this
9.   In the decision the court dismissed the claims against the Seven Brothers Code, and shall thereby become liable to all the duties
Shipping corporation because there was a stipulation exempting them from requirements, liabilities and penalties to which an
liability and they were deemed here to be a private carrier (only relevant insurance broker is subject.
information about the shipping corporation is this) c.   The appellees, upon the other hand, claim that the second
10.   In this petition for review on certiorari brought by South Sea Surety and paragraph of Section 306 of the Insurance Code provide as
Insurance Co., Inc., argues that it likewise should have been freed from any follows:
liability to Hardwood. It faults the appellate court (a) for having Supposedly i.   Sec. 306. . . . Any insurance company which
disregarded Section 77 of the insurance Code and (b) for holding Victorio delivers to an insurance agent or insurance broker a policy
Chua to have been an authorized representative of the insurer. or contract of insurance shall be deemed to have
a.   Sec. 77. An insurer is entitled to payment of the premium as soon authorized such agent or broker to receive on its behalf
as the thing insured is exposed to the peril insured against. payment of any premium which is due on such policy of
Notwithstanding any agreement to the contrary, no policy or contract of insurance at the time of its issuance or delivery
contract of insurance issued by an insurance company is valid and or which becomes due thereon.
binding unless and until the premium thereof has been paid, except
in the case of a life or an industrial life policy whenever the grace ISSUE/s:
period provision applies. 1.   WoN the appellate court erred (a) for having Supposedly disregarded
11.   No attempt becloud the issues can disguise the fact that the sole question Section 77 of the insurance Code and (b) for holding Victorio Chua to have
raised in the instant petition is really evidentiary in nature, i.e., whether been an authorized representative of the insurer.? No to both,When the
or not Victorio Chua, in receiving the check for the insurance premium appellant South Sea Surety and Insurance Co., Inc. delivered to Mr.
prior to the occurrence of the risk insured against has so acted as an Chua the marine cargo insurance policy for the plaintiffs logs, he is
agent of South Sea. The appellate court, like the trial court, has found in deemed to have been authorized by the South Sea Surety and Insurance
the affirmative. Said the appellate court: Co., Inc. to receive the premium which is due on its behalf. When
a.   In the instant case, the Marine Cargo Insurance Policy No. therefore the insured logs were lost, the insured had already paid the
84/24229 was issued by defendant insurance company on 20 premium to an agent of the South Sea Surety and Insurance Co., Inc.,
January 1984. At the time the vessel sank on 25 January 1984 which is consequently liable to pay the insurance proceeds under the
resulting in the loss of the insured logs, the insured had already policy it issued to the insured.
delivered to Victorio Chua the check in payment of premium.
But, as Victorio Chua testified, it was only in the morning of 30 RULING: WHEREFORE, the resolution, dated 01 February 1993, granting due
January 1984 or 5 days after the vessel sank when his course to the petition is RECALLED, and the petition is DENIED. Costs against
messenger tendered the check to defendant South Sea Surety petitioner.
and Insurance Co., Inc.
b.   Appellant surety company insists that Mr. Chua is an RATIO:
administrative assistant for the past ten years and an agent for less 1.   Undoubtedly, the payment of the premium is a condition precedent to, and
than ten years of the Columbia Insurance Brokers, Ltd. He is paid a essential for, the efficaciousness of the contract. The only two statutorily
salary as a administrative assistant and a commission as agent provided exceptions are (a) in case the insurance coverage relates to life or
based on the premiums he turns over to the broker. Appellant industrial life (health) insurance when a grace period applies and (b) when
therefore argues that Mr. Chua, having received the insurance the insurer makes a written acknowledgment of the receipt of premium, this
premiums as an agent of the Columbia Insurance Broker, acted as acknowledgment being declared by law to be then conclusive evidence of
an agent of the insured under Section 301 of the Insurance Code the premium payment (Secs. 77-78, Insurance Code). The appellate court,
which provides as follows: contrary to what the petition suggests, did not make any pronouncement to
i.   Sec. 301. Any person who for any compensation, the contrary. Indeed, it has said:
commission or other thing of value, acts, or aids in a.   Concerning the issue as to whether there is a valid contract of
soliciting, negotiating or procuring the making of any insurance between plaintiff-appellee and defendant-appellant
insurance contract or in placing risk or taking out South Sea Surety and Insurance Co., Inc., Section 77 of the
Insurance Code explicitly provides that notwithstanding any
agreement to the contrary, no policy issued by an insurance
company is valid and binding unless and until premium thereof has
been paid. It is therefore important to determine whether at the
time of the loss, the premium was already paid
2.   On cross-examination in behalf of South Sea Surety and Insurance Co., Inc.
Mr. Chua testified that the marine cargo insurance policy for the plaintiff's
logs was delivered to him on 21 January 1984 at his office to be
delivered to the plaintiff.
3.   When the appellant South Sea Surety and Insurance Co., Inc. delivered to
Mr. Chua the marine cargo insurance policy for the plaintiffs logs, he is
deemed to have been authorized by the South Sea Surety and Insurance
Co., Inc. to receive the premium which is due on its behalf.
4.   When therefore the insured logs were lost, the insured had already paid
the premium to an agent of the South Sea Surety and Insurance Co.,
Inc., which is consequently liable to pay the insurance proceeds under
the policy it issued to the insured
 
   
019 GAISANO v. DEVELOPMENT INSURANCE (Fordan) at the time of loss on Sept. 27. It does not also falls in the 4th and 5th exceptions
Feb. 27, 2017 | Jardeleza, J. | Payment of insurance premiums since Gaisano failed to establish the fact of a grant by Development Insurance of
a credit term in his favor, or that the grant has been consistent. While there was
PETITIONER: Jaime T. Gaisano mention of a credit agreement between Trans-Pacific and Development
RESPONDENTS: Development Insurance and Surety Corporation Insurance, such arrangement was not proven and was internal between agent and
principal. Under the principle of relativity of contracts, contracts bind the parties
SUMMARY: 2. On Sept. 27, 1996, Development Insurance issued a who entered into it. It cannot favor or prejudice a third person, even if he is
comprehensive commercial vehicle policy to Gaisano of P1.5M over his vehicle, aware of the contract and has acted with knowledge. Moreover, the policy itself
Mitsubishi Montero for 1 year commencing on said date. It also issued 2 other states that the insured's application for the insurance is subject to the payment of
commercial vehicle policies to Gaisano covering 2 other motor vehicles for the the premium. There is no waiver of pre-payment, in full or in installment, of the
same period. A statement of account was sent to Noah’s Ark, Gaisano’s premiums under the policy. Consequently, Development Insurance cannot be
company, wherein it immediately processed and issued a check for the payment placed in estoppel. Thus, SC find that Gaisano is not entitled to the insurance
of the premiums. However, Trans-Pacific, agent of Development Insurance, proceeds because no insurance policy became effective for lack of premium
informed Noah’s Ark that it would pick-up the check the following day since payment.
there was a celebration in the company. Meanwhile, on the evening of Sept. 27,
while under the custody of Pacquing, Noah’s Ark Marketing Manager, the DOCTRINE: The general rule in insurance laws is that unless the premium is
vehicle was stolen in SM Megamall. The incident was reported to the PNP and paid, the insurance policy is not valid and binding pursuant to Sec. 77 of the
despite the search and retrieval efforts, the vehicle was not recovered. Oblivious Insurance Code. However, UCPB General Insurance Co., Inc. v. Masagana
of the incident, Trans-Pacific did picked-up the check on Sept. 28, issued an OR, Telamart, Inc., summarized the following exceptions:
and deposited it to Metrobank on Oct. 1. Upon notice from Pacquing, Gaisano 1.  in case of life or industrial life policy, whenever the grace period provision
reported the loss and and filed a claim with Development Insurance for the applies, as expressly provided by Sec. 77 itself;
insurance proceeds of P1.5M. However, Development Insurance denied the 2.  where the insurer acknowledged in the policy or contract of insurance itself
claim on the ground that there was no insurance contract since non-payment of the receipt of premium, even if premium has not been actually paid, as
the premium rendered the policy ineffective. The premium was received only on expressly provided by Sec. 78 itself;
Oct. 2, 1996, and there was no known loss covered by the policy to which the 3.  where the parties agreed that premium payment shall be in installments and
payment could be applied.. Gaisano then filed a complaint for collection of sum partial payment has been made at the time of loss, as held in Makati
of money and damages with the RTC. RTC ruled in favor of Gaisano. However, Tuscany Condominium Corp. v. Court of Appeals;
the CA reversed the decision. Hence, the current petition. 4.  where the insurer granted the insured a credit term for the payment of the
premium, and loss occurs before the expiration of the term, as held in
The issue is whether or not there is a binding insurance contract between Makati Tuscany Condominium Corp.; and
Gaisano and Development Insurance. NO. The SC held that just like any other 5.  where the insurer is in estoppel as when it has consistently granted a 60-to
contract, insurance contract requires a cause or consideration. The consideration 90-day credit term for the payment of premiums.
is the premium, which must be paid at the time and in the way and manner
specified in the policy. If not so paid, the policy will lapse and be forfeited by its FACTS:
own terms. The general rule in insurance laws is that unless the premium is paid, 107.  Petitioner Jaime T. Gaisano (Gaisano) was the registered owner of a 1992
the insurance policy is not valid and binding pursuant to Sec. 77 of the Insurance Mitsubishi Montero with plate number GTJ 777 (vehicle) while
Code. 4. In this case, there is no dispute that the check was delivered to and was respondent Development Insurance and Surety Corporation (Development
accepted by Trans-Pacific, only on Sept. 28. No payment of premium had thus Insurance) is a domestic corporation engaged in the insurance business.
been made at the time of the loss of the vehicle on Sept. 27. However, there are 108.  On Sept. 27, 1996, Development Insurance issued a comprehensive
exceptions to the rule that no insurance contract takes effect unless premium is commercial vehicle policy to Gaisano of P1.5M over the vehicle for 1 year
paid as summarized in UCPB General Insurance Co., Inc. v. Masagana commencing on Sept. 27, 1996. It also issued 2 other commercial vehicle
Telamart, Inc. (kindly see doctrine). In this case, the insurance policy in question policies to Gaisano covering 2 other motor vehicles for the same period.
does not fall under the 1st to 3rd exceptions laid out in UCPB General Insurance 109.  To collect the premiums, Trans-Pacific Underwriters Agency (Trans-
Co., Inc.: (1) the policy is not a life or industrial life policy; (2) the policy does Pacific), agent of Development Insurance, issued a statement of account to
not contain an acknowledgment of the receipt of premium but merely a Gaisano’s company, Noah’s Ark Merchandising (Noah’s Ark).
statement of account on its face; and (3) no payment of an installment was made 110.  Noah's Ark immediately processed the payments and issued a Far East Bank
check (of P140,893.50) dated Sept. 27, 1996 payable to Trans Pacific on 120.  Hence, the current petition. Gaisano argues that:
the same day. The check represents payment for the 3 insurance policies, a.   there was a valid and binding insurance contract between him and
with P55,620.60 for the premium and other charges over the vehicle. Development Insurance;
111.  However, nobody from Trans Pacific picked up the check that day (Sept. b.   it comes within the exceptions to the rule in Sec. 77 of the Insurance
27) because its president and general manager, Rolando Herradura, was Code that no contract of insurance becomes binding unless and until
celebrating his birthday. Trans Pacific informed Noah's Ark that its the premium thereof has been paid;
messenger would get the check the next day, Sept. 28. c.   the prohibitive tenor of Sec. 77 does not apply because the parties
112.  Meanwhile, in the evening of Sept. 27, while under the official custody of stipulated for the payment of premiums wherein the parties intended
Noah's Ark marketing manager Achilles Pacquing (Pacquing) as a service the contract of insurance to be immediately effective upon issuance,
company vehicle, the vehicle was stolen in the vicinity of SM Megamall at despite nonpayment of the premium, because Development Insurance
Ortigas, Mandaluyong City. trusted him;
113.  Pacquing reported the loss to the PNP Traffic Management Command at d.   Development Insurance waived its right to a prepayment in full of the
Camp Crame in Quezon City. Despite search and retrieval efforts, the terms of the policy, and is in estoppel;
vehicle was not recovered. e.   assuming he is not entitled to recover insurance proceeds but only to
114.  Oblivious of the incident, Trans-Pacific picked up the check the next day the return of the premiums paid, then he should be able to recover the
(Sept. 28). It issued an OR (dated Sept. 28) acknowledging the receipt of full P140,893.50 and not merely P55,620.60 since the insurance policy
the premium and other charges over the vehicle. The check was then covered 3 vehicles yet Development Insurance's intention was merely
deposited with Metrobank for encashment on Oct. 1 which was the same to disregard the contract for only the lost vehicle. Thus, the principle
day that Pacquing informed Gaisano of the vehicle's loss. of mutuality of contracts is violated, at his expense.
115.  Thereafter, Gaisano reported the loss and filed a claim with Development
Insurance for the insurance proceeds of P1.5M. However, Development ISSUE: Whether or not there is a binding insurance contract between Gaisano
Insurance denied the claim on the ground that there was no insurance and Development Insurance would be binding. – NO, since there was no payment
contract. of premium that has been made at the time of the loss of the vehicle on Sept. 27 and
116.  Gaisano, through counsel, sent a final demand but Development Insurance the circumstances does not fall into any of the exceptions noted in the case of UCPB
still refused to pay the insurance proceeds or return the premium paid on the General Insurance Co., Inc.
vehicle. Thus, Gaisano filed a complaint for collection of sum of money and
damages with the RTC. RULING: The petition is denied. The assailed decision of the CA and the
117.  In its Answer, Development Insurance asserted that the nonpayment of the Resolution are affirmed with the modification that Development Insurance should
premium rendered the policy ineffective. The premium was received only return the P55,620.60 with the legal interest computed at the rate of 6% per annum
on Oct. 2, 1996, and there was no known loss covered by the policy to reckoned from July 7, 1997 until finality of this judgment. Thereafter, the total
which the payment could be applied. amount shall earn interest at the rate of 6% per annum from the finality of this
118.  The RTC ruled in favor of Gaisano It considered the premium paid as of judgment until its full satisfaction.
Sept. 27, even if the check was received only on Sept. 28 because (1) Trans-
Pacific, agent, acknowledged payment of the premium on that date (Sept. RATIO:
27), and (2) the check issued was honored in acknowledgment of the 63.   Insurance is a contract whereby one undertakes for a consideration to
authority of the agent to receive it. indemnify another against loss, damage or liability arising from an
119.  On appeal, the CA reversed the decision of the RTC. It upheld unknown or contingent event. Just like any other contract, it requires a
Development Insurance’s position that an insurance contract becomes valid cause or consideration. The consideration is the premium, which must
and binding only after the premium is paid pursuant to Sec. 77 of the be paid at the time and in the way and manner specified in the policy. If
Insurance Code. It found that the premium was not yet paid at the time of not so paid, the policy will lapse and be forfeited by its own terms.
the loss on Sept. 27, but only a day after, when the check was picked up by 64.   The law, however, limits the parties' autonomy as to when payment of
TransPacific. It also found that none of the exceptions to Sec. 77 obtains in premium may be made for the contract to take effect. The general rule in
this case. Nevertheless, the CA ordered Development Insurance to return insurance laws is that unless the premium is paid, the insurance policy
the premium it received in of P55,620.60, with interest at the rate of 6% per is not valid and binding. Section 77 of the Insurance Code, applicable at
annum from the date of the denial of the claim on Oct. 9, 1996 until the time of the issuance of the policy, provides:
payment. Sec. 77. An insurer is entitled to payment of the premium as soon as the
thing insured is exposed to the peril insured against. Notwithstanding any
agreement to the contrary, no policy or contract of insurance issued by an 60-to 90-day credit term for the payment of premiums.
insurance company is valid and binding unless and until the premium 69.   In this case, the insurance policy in question does not fall under the 1st
thereof has been paid, except in the case of a life or an industrial life to 3rd exceptions laid out in UCPB General Insurance Co., Inc.: (1) the
policy whenever the grace period provision applies. policy is not a life or industrial life policy; (2) the policy does not
65.   In Tibay v. Court of Appeals, we explained that in an insurance contract, contain an acknowledgment of the receipt of premium but merely a
both the insured and insurer undertake risks. On one hand, there is the statement of account on its face; and (3) no payment of an installment
insured, a member of a group exposed to a particular peril, who contributes was made at the time of loss on Sept. 27.
premiums under the risk of receiving nothing in return in case the 70.   Furthermore, the SC does not agree with Gaisano that his case falls under
contingency does not happen; on the other, there is the insurer, who the 4th and 5th exceptions.
undertakes to pay the entire sum agreed upon in case the contingency 71.   The 4th and 5th exceptions to Sec. 77 operate under the facts obtaining in
happens. This risk-distributing mechanism operates under a system where, Makati Tuscany Condominium Corp. and UCPB General Insurance Co.,
by prompt payment of the premiums, the insurer is able to meet its legal Inc. Both contemplate situations where the insurers have consistently
obligation to maintain a legal reserve fund needed to meet its contingent granted the insured a credit extension or term for the payment of the
obligations to the public. The premium, therefore, is the elixir vitae or premium.
source of life of the insurance business. 72.   Here, Gaisano failed to establish the fact of a grant by Development
66.   In this case, there is no dispute that the check was delivered to and was Insurance of a credit term in his favor, or that the grant has been
accepted by Trans-Pacific, only on Sept. 28. No payment of premium consistent. While there was mention of a credit agreement between
had thus been made at the time of the loss of the vehicle on Sept. 27. Trans-Pacific and Development Insurance, such arrangement was not
67.   While Gaisano claims that Trans-Pacific was informed that the check was proven and was internal between agent and principal. Under the
ready for pick-up on Sept. 27, the notice of the availability of the check, by principle of relativity of contracts, contracts bind the parties who
itself, does not produce the effect of payment of the premium. Trans-Pacific entered into it. It cannot favor or prejudice a third person, even if he is
could not be considered in delay in accepting the check because when it aware of the contract and has acted with knowledge.
informed Gaisano that it will only be able to pick-up the check the next day, 73.   Furthermore, the SC cannot sustain Gaisano’s claim that the parties agreed
the latter did not protest to this, but instead allowed it to do so. Thus, at the that the insurance contract is immediately effective upon issuance despite
time of loss, there was no payment of premium yet to make the insurance non-payment of the premiums.
policy effective. 74.   Even if there is a waiver of prepayment of premiums, that in itself does not
68.   There are, of course, exceptions to the rule that no insurance contract become an exception to Sec. 77, unless the insured clearly gave a credit
takes effect unless premium is paid. In UCPB General Insurance Co., term or extension. This is the clear import of the 4th exception in the UCPB
Inc. v. Masagana Telamart, Inc., we summarized the exceptions as General Insurance Co., Inc. To rule otherwise would render nugatory the
follows: requirement in Sec. 77 that "[n]otwithstanding any agreement to the
a.   in case of life or industrial life policy, whenever the grace period contrary, no policy or contract of insurance issued by an insurance company
provision applies, as expressly provided by Sec. 77 itself; is valid and binding unless and until the premium thereof has been paid, x x
b.   where the insurer acknowledged in the policy or contract of x."
insurance itself the receipt of premium, even if premium has not 75.   Moreover, the policy itself states:
been actually paid, as expressly provided by Sec. 7830 itself; WHEREAS THE INSURED, by his corresponding proposal and
c.   where the parties agreed that premium payment shall be in declaration, and which shall be the basis of this Contract and deemed
installments and partial payment has been made at the time of loss, incorporated herein, has applied to the company for the insurance
as held in Makati Tuscany Condominium Corp. v. Court of Appeals; hereinafter contained, subject to the payment of the Premium as
d.   where the insurer granted the insured a credit term for the payment consideration for such insurance. (Emphasis supplied.)
of the premium, and loss occurs before the expiration of the term, as 76.   The policy states that the insured's application for the insurance is
held in Makati Tuscany Condominium Corp.; and subject to the payment of the premium. There is no waiver of pre-
e.   where the insurer is in estoppel as when it has consistently granted a payment, in full or in installment, of the premiums under the policy.
Consequently, Development Insurance cannot be placed in estoppel.
                                                                                                                        77.   Thus, SC find that Gaisano is not entitled to the insurance proceeds
30
SEC. 78. Any acknowledgment in a policy or contract of insurance of the receipt of because no insurance policy became effective for lack of premium
premium is conclusive evidence of its payment, so far as to make the policy binding, payment.
notwithstanding any stipulation therein that it shall not be binding until premium is actually
78.   The consequence of this declaration is that Gaisano is entitled to a return of
paid.
the premium paid for the vehicle of P55,620.60 under the principle of unjust
enrichment.
79.   There is unjust enrichment when a person unjustly retains a benefit to the
loss of another, or when a person retains money or property of another
against the fundamental principles of justice, equity and good conscience.
80.   Gaisano cannot claim the full P140,893.50, which includes the payment of
premiums for the 2 other vehicles. These two policies are not affected by
the ruling on the policy subject of this case because they were issued as
separate and independent contracts of insurance.
 
   
020 PHILIPPINE PHOENIX SURETY & INSURANCE v. b.   Phoenix Surety issued to Woodworks on April 1, 1960 Fire Policy
WOODWORKS (GALINDEZ) No. 9652 for P300,000 under the terms and conditions set forth in
August 31, 1967 | Dizon, J. | Return of Premium the policy
c.   The premiums of the policy amounted to P6,051.95, the margin fee
amounted to P363.72 and documentary stamp tax, P96.42
PETITIONER: Philippine Phoenix Surety & Insurance, Inc.
d.   Woodworks paid P3,000 on September 22, 1960
RESPONDENTS: Woodworks, Inc. e.   Phoenix Surety made several demands on Woodworks to pay
P3,522.09.
SUMMARY: Phoenix Surety issued to Woodworks on April 1, 1960 Fire
Policy No. 9652 for P300,000 under the terms and conditions set forth in the ISSUE/s:
policy. The premiums of the policy amounted to P6,051.95, the margin fee 1.   WoN the nonpayment of premiums produced the cancellation of the
amounted to P363.72 and documentary stamp tax, P96.42. Woodworks paid
insurance contract – NO. The correct view would seem to be this: as the
P3,000 on September 22, 1960. Phoenix Surety made several demands on
contract had become perfected, the parties could demand from each other
Woodworks to pay P3,522.09.
the performance of whatever obligations they had assumed.
Phoenix Surety instituted an action to recover from Woodworks the balance of
RULING: Wherefore, the appealed decision being in accordance with law and the
P3,522.09 for the premium of the fire policy. Woodworks refused to pay and the
evidence, the same is hereby affirmed, with costs.
case reached the SC. Woodworks’ theory: non-payment by it of the premium
due produced the cancellation of the contract of insurance. However, such RATIO:
theory would place exclusively in the hands of one of the contracting parties the 1.   There is no doubt that between the insurer and insured, there was not only a
right to decide whether the contract should stand or not. perfected contract of insurance but a partially performed one as far as the
payment of the agreed premium was concerned.
ISSUE: WoN the nonpayment of premiums produced the cancellation of the
2.   After, the obligation of the insurer to pay the insured the amount for which
insurance contract – NO. (See Doctrine)
the policy was issued in case the conditions had been complied with, arose
and became binding upon it, while the obligation of the insured to pay the
In the case of the insurer, it is obvious that it had the right to demand from the
remainder of the total amount of the premium became due and demandable.
insured the completion of the payment of the premium due or sue for the
3.   The Court cannot agree with Woodworks’ theory that non-payment by it of
rescission of the contract. As it chose to demand specific performance of the
the premium due produced the cancellation of the contract of insurance.
insured's obligation to pay the balance of the premium, the latter's duty to pay is
Such theory would place exclusively in the hands of one of the contracting
indeed indubitable. parties the right to decide whether the contract should stand or not.
4.   Rather the correct view would seem to be this: as the contract had become
DOCTRINE: The correct view would seem to be this: as the contract had perfected, the parties could demand from each other the performance of
become perfected, the parties could demand from each other the performance of whatever obligations they had assumed.
whatever obligations they had assumed.
5.   In the case of the insurer, it is obvious that it had the right to demand from
the insured the completion of the payment of the premium due or sue for the
This is more or less the whole case J rescission of the contract.
6.   As it chose to demand specific performance of the insured's obligation to
FACTS: pay the balance of the premium, the latter's duty to pay is indeed
1.   Appellee Phoenix Surety commenced this action in the Municipal Court of indubitable.
Manila to recover from appellant Woodworks, Inc. P3,522.09 representing    
the unpaid balance of the premiums on a fire insurance policy issued by
Phoenix Surety in favor of Woodworks for a term of one year, from April 1
1960 to 1961.
2.   From an adverse decision of said court, Woodworks appealed to the CFI of
Manila where the parties submitted the following stipulation of facts:
a.   Both parties are corporations duly organized and existing by virtue
of Philippine laws
021 CAPITAL INSURANCE v. PLASTIC ERA (Gonzales) 3.   When the policy was delivered, Plastic Era failed to pay the corresponding
July 18, 1975 | Martin, J. | Return of Premium insurance premium. However, through its duly authorized representative, it
executed acknowledgment receipt.31
PETITIONER: Capital Insurance & Surety Co., Inc. 4.   In partial payment, Plastic Era delivered to Capital Insurance, a check for
RESPONDENT: Plastic Era Co., Inc. and Court of Appeals the amount of P1,000.00 postdated January 16, 1961.
5.   However, Capital Insurance tried to deposit the check only on February 20,
SUMMARY: When Capital Insurance delivered to Plastic Era its open fire policy 1961 and the same was dishonored for lack of funds. The records show that
whereby the former undertook to insure the latter's property, the latter did not pay the as of January 19, 1961 Plastic Era had a balance of P1,193.41 with the bank
premium, but instead executed an acknowledgment receipt with a promise to pay the 6.   On January 18, 1961 or two days after the insurance premium became due,
premium within 30 days from December 17, 1960, the effectivity date of the policy. at about 4:00 to 5:00 o'clock in the morning, the property insured by Plastic
Capital Insurance duly accepted the receipt. On January 8, 1961, Plastic Era, in Era was destroyed by fire.
partial payment of the premium, delivered a check of P1,000, postdated January 16, 7.   The loss and/or damage was estimated to be P283,875. However, according
1961. Capital Insurance tried to deposit the check only on February 20, 1961, and the to the records the same property has been insured by Plastic Era with the
same was dishonored for lack of funds. But the records show that as of January 19, Philamgen Insurance Company for P200,000.00.
1961, Plastic Era had sufficient funds with the bank to cover the check. On January 8.   Plastic Era demanded from Capital Insurance the payment of the sum of
18, 1961, or two days after the insurance premium became due, the insured P100,000.00 but the latter refused because of Plastic Era’s failure to pay the
properties were destroyed by fire. Plastic Era demanded payment of indemnity but insurance premium. Plastic Era thus filed a complaint for the recover of the
Capital Insurance refused on the ground of non-payment of premium. Plastic Era sum of P100,000 plus attorney’s fees and additional expenses.
sued Capital Insurance and the trial court rendered judgment in favor of the former. 9.   The trial court ordered Capital Insurance to pay Plastic Era P88,325.63.
The Court of Appeals sustained the trial court. The issue is WoN a contract of 10.   CA affirmed the decision of the trial court.
insurance has been duly perfected between Capital Insurance and Plastic Era – YES.
By accepting the Plastic Era’s promise to pay the insurance premium, Capital ISSUE:
Insurance, in effect, had extended credit to the former and did not have the right to 1.   WoN a contract of insurance has been duly perfected between Capital
cancel the policy except by putting Plastic Era in default by giving it personal notice Insurance and Plastic Era – YES. By accepting the promise of Plastic Era to
to that effect. pay the insurance premium within thirty (30) days from the effective date of
policy, Capital Insurance has implicitly agreed to modify the tenor of the
DOCTRINE: Where the insurer accepted the promise of the insured to pay the insurance policy and in effect, waived the provision therein that it would
insurance premium within 30 days from the effectivity date of the policy, the former only pay for the loss or damage in case the same occurs after the payment of
implicitly agreed to modify the tenor of the insurance contract and in effect waived the premium.
the provisions therein that it would only pay for the loss or damage in case the same
occurs after the payment of the premium; and where the policy is silent as to the RULING: Petition denied. CA decision affirmed.
mode of payment, the insurer is deemed to have accepted the promissory note in
payment of the premium. This rendered, the policy is immediately operative. RATIO:
1.   In clear and unequivocal terms the insurance policy32 provides that it is only
upon payment of the premiums by Plastic Era that Capital Insurance agrees
FACTS:
1.   Capital Insurance delivered to Plastic Era its open Fire Policy No.                                                                                                                        
31
22760 wherein the former undertook to insure the latter's building, "This acknowledged receipt of Fire Policy) NO. 22760 Premium. . .) (I promise to pay)
(P2,220.00) (has been paid) THIRTY DAYS AFTER on effective date ______ (Date)”
equipments, raw materials, products and accessories located at Sheridan 32
"This Policy of Insurance Witnesseth That in consideration of PLASTIC ERA hereinafter called the
Street, Mandaluyong, Rizal. Insured, paying to the Capital Insurance hereinafter called the Company, the sum of PESOS TWO
2.   The policy provides that if the property insured would be destroyed or THOUSAND ONE HUNDRED EIGHTY EIGHT the premium for the first period hereinafter mentioned,
damaged by fire after the payment of the premiums, at any time between the for insuring against Loss or Damage by only Fire or Lightning, as hereinafter appears, the Property
hereinafter described and contained, or described herein and not elsewhere, in the several sums following
15th day of December 1960 and one o'clock in the afternoon of the 15th day namely: PESOS ONE HUNDRED THOUSAND ONLY, PHILIPPINE CURRENCY; . . . THE
of December 1961, the insurance company shall make good all such loss or COMPANY HEREBY AGREES with the Insured but subject to the terms and conditions endorsed or
damage in an amount not exceeding P100,000.00. otherwise expressed hereon, which are to be taken as part of this Policy), that if the Property described, or
any part thereof, shall be destroyed or damaged by Fire or Lightning after payment of the Premiums, at
any time between the 15th day of December One Thousand Nine Hundred and Sixty and 1 o'clock in the
afternoon of the 15th day of December One Thousand Nine Hundred and Sixty-One of the last day of any
to insure the properties of the former against loss or damage in an amount claiming a forfeiture of its policy for non-payment even if the check had
not exceeding P100,000.00. been dishonored later.
2.   The crux of the problem then is whether at the time the insurance policy 9.   Finally, it is submitted by Capital Insurance that: "We are here concerned
was delivered to Plastic Era on December 17, 1960, the latter was able to with a case of reciprocal obligations, and Plastic Era having failed to
pay the stipulated premium. comply with its obligation to pay the insurance premium due on the policy
3.   On the day the insurance policy was delivered, Plastic Era did not pay the within thirty days from December 17, 1960, Capital Insurace was relieved
Capital Insurance, but instead executed an acknowledgment receipt wherein of its obligation to pay anything under the policy, without the necessity of
it promised to pay the premium within thirty (30) days from the effectivity first instituting an action for rescission of the contract of ensured entered
date of the policy on December 17, 1960 and Capital Insurance accepted it. into by the parties."
4.   Under Article 124933 of the Civil Code, the mere delivery of a bill of 10.   But precisely in this case, Plastic Era has complied with its obligation to
exchange in payment of a debt does not immediately effect payment. pay the insurance premium and therefore Capital Insurance is obliged to
5.   Here, Capital Insurance accepted the promise of Plastic Era to pay the make good its undertaking to Plastic Era.
insurance premium within thirty (30) days from the effective date of policy. |
By so doing, it has implicitly agreed to modify the tenor of the
insurance policy and in effect, waived the provision therein that it  
would only pay for the loss or damage in case the same occurs after the
payment of the premium. Considering that the insurance policy is silent as
to the mode of payment, Capital Insurance is deemed to have accepted the
promissory note in payment of the premium.
6.   Hence, when the damage of the insured property occurred, the insurance
policy was in full force and effect. The fact that the check issued by
Plastic Era in partial payment of the promissory note was later on
dishonored did not in any way operate as a forfeiture of its rights under
the policy, there being no express stipulation therein to that effect.
7.   The payment of the premium on the insurance policy became an
independent obligation the non-fulfillment of which would entitle Capital
Insurance to recover. It could just deduct the premium due and unpaid upon
the satisfaction of the loss under the policy. It did not have the right to
cancel the policy for nonpayment of the premium except by putting Plastic
Era in default and giving it personal notice to that effect.
8.   Although the check was due for payment on January 16, 1961 and Plastic
Era had sufficient funds to cover it as of January 19, 1961, Capital
Insurance decided to hold the same for thirty-five (35) days before
presenting it for payment. Having held the check for such an
unreasonable period of time, Capital Insurance was estopped from

                                                                                                                                                                                                                                                                                       
subsequent period in respect of which the insured, or a successor in interest to whom the insurance is by
an endorsement hereon declared to be or is otherwise continued, shall pay to the Company and the
Company shall accept the sum required for the renewal of this Policy, the Company will pay or make
good all such loss or Damage, to an amount not exceeding during any one period of the insurance in
respect of the several matters specified, the sum; set opposite thereto respectively, and not exceeding the
whole sum of PESOS, ONE HUNDRED THOUSAND ONLY, PHIL. CUR. . . ."
33
"The delivery of promissory notes payable to order, or bills of exchange or other mercantile documents
shall produce the effect of payment only when they have been cashed, or when through the fault of the
creditor they have been impaired." xxx "In the meantime, the action derived from the original obligation
shall be held in abeyance."
 
001 GULF RESORT INC. v. PHIL. CHARTER (Escalona) 1.   Gulf Resort is the owner of the Plaza Resort situated at Agoo,
May 16, 2005 | Puno, J. | The Policy La Union and had its properties in said resort insured originally
with the American Home Assurance Company (AHAC). In the
PETITIONER: Gulf Resort Inc. first four insurance policies issued by AHAC, the risk of loss
RESPONDENTS: Philippine Charter Insurance Corporation
from earthquake shock was extended only to Gulf Resort’s two
SUMMARY: Gulf Resort, Inc at Agoo, La Union was insured with American swimming pools. Item 5 in those policies referred to the two
Home Assurance Company, which includes loss or damage to shock to any of (2) swimming pools only; that subsequently AHAC issued in
the property insured by this Policy occasioned by or through or in consequence Gulf Resort’s favor Policy No. 206-4182383-0 and in said
of earthquake. Gulf Resort then insured its properties with PCIC based on the policy the earthquake endorsement clause was deleted and
same insurance contract they had with AHAC. Later on, an earthquake struck
Central Luzon and Northern Luzon so the properties and 2 swimming pools in its the entry under Endorsements/Warranties at the time of issue
Agoo Playa Resort were damaged. Gulf then claimed against PCIC for the read that Gulf Resort renewed its policy with AHAC for the
damages the earthquake created on its properties. Gulf's claim was denied on the period of March 14, 1989 to March 14, 1990 under Policy No.
ground that its insurance policy only afforded earthquake shock coverage to the 206-4568061-9, which carried the entry under
two swimming pools of the resort. Gulf contends that pursuant to this rider, no Endorsement/Warranties at Time of Issue, which read
qualifications were placed on the scope of the earthquake shock coverage, thus
the policy extended earthquake shock coverage to all of the insured properties. Endorsement to Include Earthquake Shock in the amount
The RTC ruled in favor of PCIC that the policy only covered the two swimming of P10,700.00 and paid P42,658.14 as premium thereof.
pools. The CA affirmed the decision of the RTC. 2.   Gulf then insured its properties with PCIC using the exact same
contract it used to have with AHAC.
The issue is WoN the insurance policy covers the other properties beyond the 3.   On July 16, 1990, an earthquake struck Central Luzon and
two swimming pools.
Northern Luzon and Gulf Resort’s properties covered by Policy
The Court ruled that the insurance policy did not cover the other properties. The No. 31944 issued by PCIC, including the two swimming pools
Court first ruled that a stipulation in the contract cannot be construed piecemeal, in its Agoo Playa Resort were damaged.
such that the contract should be construed as a whole, including the provisions 4.   After the earthquake, Gulf Resort advised PCIC that it would
and riders. They should all be construed together in order to evidence the be making a claim under its Insurance Policy No. 31944 for
intention of the parties of whether they wanted to insure the other properties
against earthquake shocks. The Court held that there was no intention of the damages on its properties. PCIC instructed Gulf Resort to file a
parties to include such because there were no premiums paid for the other formal claim, then assigned the investigation of the claim to an
properties against earthquake shocks, apart from the two swimming pools. This independent claims adjuster, Bayne Adjusters and Surveyors,
is in consonance with the history of the deletion of the other properties from Inc. On July 30, 1990, PCIC, through its adjuster, requested
insurance against earthquake shocks in the insurance contracts that Gulf had with Gulf Resort to submit various documents in support of its
AHAC.
claim. On August 7, 1990, Bayne Adjusters and Surveyors,
DOCTRINE: All provisions of the insurance policy should be examined and Inc., through its Vice-President A.R. de Leon, rendered a
interpreted in consonance with each other. All its parts are reflective of the true preliminary report finding extensive damage caused by the
intent of the parties. The policy cannot be construed piecemeal. Certain earthquake to the clubhouse and to the two swimming
stipulations cannot be segregated and then made to control; neither do particular pools. Mr. de Leon stated that except for the swimming
words or phrases necessarily determine its character.
pools, all affected items have no coverage for earthquake
FACTS:
shocks. On August 11, 1990, Gulf Resort filed its form
demand for settlement of the damage to all its properties in the
Agoo Playa Resort. On August 23, 1990, PCIC denied Gulf
Resort’s claim on the ground that its insurance policy only
afforded earthquake shock coverage to the two swimming Resort had with PCIC and upon which the subject
pools of the resort. Gulf Resort and PCIC failed to arrive at a insurance contract with Philippine Charter
settlement. Thus, on January 24, 1991, Gulf Resort filed a Insurance Corporation is said to have been based
complaint with the regional trial court of Pasig. and copied, covered an extended earthquake shock
5.   The lower court after trial ruled in favor of the PCIC, stating: insurance on all the insured properties.
a.   The above schedule clearly shows that plaintiff paid ISSUES:
only a premium of P393.00 against the peril of 1.   WoN the insurance policy covered only the two swimming pools, rather
than all the properties insured, against the risk of an earthquake shock –
earthquake shock, the same premium it paid against YES. Only the two swimming pools are covered because the contract that
earthquake shock only on the two swimming pools in reflected the intent of the parties evidenced that only the two swimming
all the policies issued by PCIC. From this fact the Court pools were insured against earthquake shocks. This was proven through the
must consequently agree with the position of PCIC that lack of payment of premiums and testimonies of witnesses.
the endorsement rider means that only the two
RULING: The judgment of the Court of Appeals is affirmed. The petition
swimming pools were insured against earthquake for certiorari is dismissed.
shock.
b.   Gulf Resort correctly points out that a policy of RATIO:
insurance is a contract of adhesion hence, where the 81.  The policy stated:
language used in an insurance contract or application is ANNUAL PAYMENT AGREEMENT ON
such as to create ambiguity the same should be resolved LONG TERM POLICIES
against the party responsible therefor, i.e., the insurance THE INSURED UNDER THIS POLICY HAVING
company which prepared the contract. To the mind of ESTABLISHED AGGREGATE SUMS INSURED IN
the Court, the language used in the policy in litigation is EXCESS OF FIVE MILLION PESOS, IN CONSIDERATION
clear and unambiguous hence there is no need for OF A DISCOUNT OF 5% OR 7 % OF THE NET PREMIUM
interpretation or construction but only application of the x x x POLICY HEREBY UNDERTAKES TO CONTINUE
provisions therein. THE INSURANCE UNDER THE ABOVE NAMED x x x
c.   From the above observations the Court finds that only AND TO PAY THE PREMIUM.
the two (2) swimming pools had earthquake shock Earthquake Endorsement
coverage and were heavily damaged by the earthquake, In consideration of the payment by the Insured to the Company
which struck on July 16, 1990. PCIC having admitted of the sum of P. . . . . . . . . . . . . . . . . additional premium the
that the damage to the swimming pools was appraised Company agrees, notwithstanding what is stated in the printed
by PCIC’s adjuster at P386,000.00, PCIC must, by conditions of this Policy to the contrary, that this insurance
virtue of the contract of insurance, pay Gulf Resort said covers loss or damage (including loss or damage by fire) to
amount. any of the property insured by this Policy occasioned by or
6.   Gulf Resort’s Motion for Reconsideration was denied. Thus, through or in consequence of Earthquake.
Gulf Resort filed an appeal with the Court of Appeals. After
review, the appellate court affirmed the decision of the trial Provided always that all the conditions of this Policy shall
court and ruled, thus: apply (except in so far as they may be hereby expressly varied)
a.   However, after carefully perusing the documentary and that any reference therein to loss or damage by fire should
evidence of both parties, We are not convinced that be deemed to apply also to loss or damage occasioned by or
the last two (2) insurance contracts, which the Gulf through or in consequence of Earthquake.
swimming pools. There is no mention of any premium
82.  Gulf Resort contends that pursuant to this rider, no payable for the other resort properties with regard to
qualifications were placed on the scope of the earthquake earthquake shock. This is consistent with the history of Gulf
shock coverage. Thus, the policy extended earthquake Resort’s previous insurance policies from AHAC.
shock coverage to all of the insured properties. 86.  PCIC, in compliance with the condition set by Gulf Resort,
83.  It is basic that all the provisions of the insurance policy copied AIU Policy No. 206-4568061-9 in drafting its Insurance
should be examined and interpreted in consonance with Policy No. 31944. It is true that there was variance in some
each other. All its parts are reflective of the true intent of terms, specifically in the replacement cost endorsement, but the
the parties. The policy cannot be construed piecemeal. principal provisions of the policy remained essentially similar
Certain stipulations cannot be segregated and then made to to AHAC’s policy. Consequently, we cannot apply the "fine
control; neither do particular words or phrases necessarily print" or "contract of adhesion" rule in this case as the parties
determine its character. Gulf Resort cannot focus on the intent to limit the coverage of the policy to the two swimming
earthquake shock endorsement to the exclusion of the other pools only is not ambiguous.
provisions. All the provisions and riders, taken and interpreted
together, indubitably show the intention of the parties to extend There were a lot of testimonies used as evidence of showing the intent
earthquake shock coverage to the two swimming pools only. of the parties to exclude the other properties from being insured
84.  A careful examination of the premium recapitulation will against earthquake shocks. Here’s the transcript of those testimonies:
show that it is the clear intent of the parties to extend
earthquake shock coverage only to the two swimming 87.   CROSS EXAMINATION OF LEOPOLDO MANTOHAC TSN, November 25, 1991
88.   pp. 12-13
pools. Section 2(1) of the Insurance Code defines a contract of 89.   Q. Now Mr. Mantohac, will it be correct to state also that insofar as your insurance
insurance as an agreement whereby one undertakes for a policy during the period from March 4, 1984 to March 4, 1985 the coverage on
earthquake shock was limited to the two swimming pools only?
consideration to indemnify another against loss, damage or 90.   A. Yes, sir. It is limited to the two swimming pools, specifically shown in the warranty,
liability arising from an unknown or contingent event. Thus, an there is a provision here that it was only for item 5.
91.   Q. More specifically Item 5 states the amount of P393,000.00 corresponding to the two
insurance contract exists where the following elements concur: swimming pools only?
a.   The insured has an insurable interest; 92.   A. Yes, sir.
93.   CROSS EXAMINATION OF LEOPOLDO MANTOHAC TSN, November 25, 1991
b.   The insured is subject to a risk of loss by the happening 94.   pp. 23-26
of the designated peril; 95.   Q. For the period from March 14, 1988 up to March 14, 1989, did you personally
arrange for the procurement of this policy?
c.   The insurer assumes the risk; 96.   A. Yes, sir.
d.   Such assumption of risk is part of a general scheme to 97.   Q. Did you also do this through your insurance agency?
98.   A. If you are referring to Forte Insurance Agency, yes.
distribute actual losses among a large group of persons 99.   Q. Is Forte Insurance Agency a department or division of your company?
bearing a similar risk; 100.   A. No, sir. They are our insurance agency.
101.   Q. And they are independent of your company insofar as operations are concerned?
e.   In consideration of the insurer's promise, the 102.   A. Yes, sir, they are separate entity.
insured pays a premium. 103.   Q. But insofar as the procurement of the insurance policy is concerned they are of
course subject to your instruction, is that not correct?
85.  An insurance premium is the consideration paid an insurer for 104.   A. Yes, sir. The final action is still with us although they can recommend what insurance
undertaking to indemnify the insured against a specified to take.
105.   Q. In the procurement of the insurance police (sic) from March 14, 1988 to March 14,
peril. In fire, casualty, and marine insurance, the premium 1989, did you give written instruction to Forte Insurance Agency advising it that the
payable becomes a debt as soon as the risk attaches. In the earthquake shock coverage must extend to all properties of Agoo Playa Resort in La
Union?
subject policy, no premium payments were made with
regard to earthquake shock coverage, except on the two
106.   A. No, sir. We did not make any written instruction, although we made an oral 137.   WITNESS:
instruction to that effect of extending the coverage on (sic) the other properties of the 138.   We do not normally cover earthquake shock endorsement on stand alone basis. For
company. swimming pools we do cover earthquake shock. For building we covered it for full
107.   Q. And that instruction, according to you, was very important because in April 1987 earthquake coverage which includes earthquake shock
there was an earthquake tremor in La Union? 139.   COURT:
108.   A. Yes, sir. 140.   As far as earthquake shock endorsement you do not have a specific coverage for other
109.   Q. And you wanted to protect all your properties against similar tremors in the [future], things other than swimming pool? You are covering building? They are covered by a
is that correct? general insurance?
110.   A. Yes, sir. 141.   WITNESS:
111.   Q. Now, after this policy was delivered to you did you bother to check the provisions 142.   Earthquake shock coverage could not stand alone. If we are covering building or another
with respect to your instructions that all properties must be covered again by earthquake we can issue earthquake shock solely but that the moment I see this, the thing that
shock endorsement? comes to my mind is either insuring a swimming pool, foundations, they are normally
112.   A. Are you referring to the insurance policy issued by American Home Assurance affected by earthquake but not by fire, sir.
Company marked Exhibit G? 143.   DIRECT EXAMINATION OF JUAN BARANDA III
113.   Atty. Mejia: Yes. 144.   TSN, August 11, 1992
114.   Witness: 145.   pp. 23-25
115.   A. I examined the policy and seeing that the warranty on the earthquake shock 146.   Q. Plaintiffs witness, Mr. Mantohac testified and he alleged that only Exhibits C, D, E
endorsement has no more limitation referring to the two swimming pools only, I was and F inclusive [remained] its coverage against earthquake shock to two (2) swimming
contented already that the previous limitation pertaining to the two swimming pools was pools only but that Exhibits G and H respectively entend the coverage against
already removed. earthquake shock to all the properties indicated in the respective schedules attached to
116.   Petitioner also cited and relies on the attachment of the phrase Subject to: Other Insurance said policies, what can you say about that testimony of plaintiffs witness?
Clause, Typhoon Endorsement, Earthquake Shock Endorsement, Extended Coverage 147.   WITNESS:
Endorsement, FEA Warranty & Annual Payment Agreement on Long Term Policies to 148.   As I have mentioned earlier, earthquake shock cannot stand alone without the other half
the insurance policy as proof of the intent of the parties to extend the coverage for earthquake of it. I assure you that this one covers the two swimming pools with respect to
shock. However, this phrase is merely an enumeration of the descriptive titles of the riders, earthquake shock endorsement. Based on it, if we are going to look at the premium there
clauses, warranties or endorsements to which the policy is subject, as required under Section has been no change with respect to the rates. Everytime (sic) there is a renewal if the
50, paragraph 2 of the Insurance Code. intention of the insurer was to include the earthquake shock, I think there is a substantial
117.   We also hold that no significance can be placed on the deletion of the qualification limiting the increase in the premium. We are not only going to consider the two (2) swimming pools
coverage to the two swimming pools. The earthquake shock endorsement cannot stand alone. of the other as stated in the policy. As I see, there is no increase in the amount of the
As explained by the testimony of Juan Baranda III, underwriter for AHAC-AIU: premium. I must say that the coverage was not broaden (sic) to include the other items.
118.   DIRECT EXAMINATION OF JUAN BARANDA III 149.   COURT:
119.   TSN, August 11, 1992 150.   They are the same, the premium rates?
120.   pp. 9-12 151.   WITNESS:
121.   Atty. Mejia: 152.   They are the same in the sence (sic), in the amount of the coverage. If you are going to
122.   We respectfully manifest that the same exhibits C to H inclusive have been previously do some computation based on the rates you will arrive at the same premiums, your
marked by counsel for defendant as Exhibit[s] 1-6 inclusive. Did you have occasion to Honor.
review of (sic) these six (6) policies issued by your company [in favor] of Agoo Playa 153.   CROSS-EXAMINATION OF JUAN BARANDA III
Resort? 154.   TSN, September 7, 1992
123.   WITNESS: 155.   pp. 4-6
124.   Yes[,] I remember having gone over these policies at one point of time, sir. 156.   ATTY. ANDRES:
125.   Q. Now, wach (sic) of these six (6) policies marked in evidence as Exhibits C to H 157.   Would you as a matter of practice [insure] swimming pools for fire insurance?
respectively carries an earthquake shock endorsement[?] My question to you is, on the 158.   WITNESS:
basis on (sic) the wordings indicated in Exhibits C to H respectively what was the extent 159.   No, we dont, sir.
of the coverage [against] the peril of earthquake shock as provided for in each of the six 160.   Q. That is why the phrase earthquake shock to the two (2) swimming pools only was
(6) policies? placed, is it not?
126.   x x x 161.   A. Yes, sir.
127.   WITNESS: 162.   ATTY. ANDRES:
128.   The extent of the coverage is only up to the two (2) swimming pools, sir. 163.   Will you not also agree with me that these exhibits, Exhibits G and H which you have
129.   Q. Is that for each of the six (6) policies namely: Exhibits C, D, E, F, G and H? pointed to during your direct-examination, the phrase Item no. 5 only meaning to (sic)
130.   A. Yes, sir. the two (2) swimming pools was deleted from the policies issued by AIU, is it not?
131.   ATTY. MEJIA: 164.   x x x
132.   What is your basis for stating that the coverage against earthquake shock as provided for 165.   ATTY. ANDRES:
in each of the six (6) policies extend to the two (2) swimming pools only? 166.   As an insurance executive will you not attach any significance to the deletion of the
133.   WITNESS: qualifying phrase for the policies?
134.   Because it says here in the policies, in the enumeration Earthquake Shock Endorsement, 167.   WITNESS:
in the Clauses and Warranties: Item 5 only (Earthquake Shock Endorsement), sir. 168.   My answer to that would be, the deletion of that particular phrase is inadvertent. Being a
135.   ATTY. MEJIA: company underwriter, we do not cover. . it was inadvertent because of the previous
136.   Witness referring to Exhibit C-1, your Honor.
policies that we have issued with no specific attachments, premium rates and so on. It indeed only Item 3 which were the two swimming pools have coverage for earthquake
was inadvertent, sir. shock.
169.   The Court also rejects petitioners contention that respondents contemporaneous and subsequent 197.   x x x
acts to the issuance of the insurance policy falsely gave the petitioner assurance that the 198.   Q. Now, may we know from you Engr. de Leon your basis, if any, for stating that except
coverage of the earthquake shock endorsement included all its properties in the resort. for the swimming pools all affected items have no coverage for earthquake shock?
Respondent only insured the properties as intended by the petitioner. Petitioners own witness 199.   x x x
testified to this agreement, viz: 200.   A. I based my statement on my findings, because upon my examination of the policy I
170.   CROSS EXAMINATION OF LEOPOLDO MANTOHAC found out that under Item 3 it was specific on the wordings that on the two swimming
171.   TSN, January 14, 1992 pools only, then enclosed in parenthesis (against the peril[s] of earthquake shock only),
172.   pp. 4-5 and secondly, when I examined the summary of premium payment only Item 3 which
173.   Q. Just to be clear about this particular answer of yours Mr. Witness, what exactly did refers to the swimming pools have a computation for premium payment for earthquake
you tell Atty. Omlas (sic) to copy from Exhibit H for purposes of procuring the policy shock and all the other items have no computation for payment of premiums.
from Philippine Charter Insurance Corporation? 201.   In sum, there is no ambiguity in the terms of the contract and its riders. Petitioner cannot rely
174.   A. I told him that the insurance that they will have to get will have the same provisions on the general rule that insurance contracts are contracts of adhesion which should be liberally
as this American Home Insurance Policy No. 206-4568061-9. construed in favor of the insured and strictly against the insurer company which usually
175.   Q. You are referring to Exhibit H of course? prepares it. A contract of adhesion is one wherein a party, usually a corporation, prepares the
176.   A. Yes, sir, to Exhibit H. stipulations in the contract, while the other party merely affixes his signature or his "adhesion"
177.   Q. So, all the provisions here will be the same except that of the premium rates? thereto. Through the years, the courts have held that in these type of contracts, the parties do
178.   A. Yes, sir. He assured me that with regards to the insurance premium rates that they not bargain on equal footing, the weaker party's participation being reduced to the alternative to
will be charging will be limited to this one. I (sic) can even be lesser. take it or leave it. Thus, these contracts are viewed as traps for the weaker party whom the
179.   CROSS EXAMINATION OF LEOPOLDO MANTOHAC courts of justice must protect. Consequently, any ambiguity therein is resolved against the
180.   TSN, January 14, 1992 insurer, or construed liberally in favor of the insured.
181.   pp. 12-14 202.   The case law will show that this Court will only rule out blind adherence to terms where facts
182.   Atty. Mejia: and circumstances will show that they are basically one-sided. Thus, we have called on lower
183.   Q. Will it be correct to state[,] Mr. Witness, that you made a comparison of the courts to remain careful in scrutinizing the factual circumstances behind each case to determine
provisions and scope of coverage of Exhibits I and H sometime in the third week of the efficacy of the claims of contending parties. In Development Bank of the Philippines v.
March, 1990 or thereabout? National Merchandising Corporation, et al., the parties, who were acute businessmen of
184.   A. Yes, sir, about that time. experience, were presumed to have assented to the assailed documents with full knowledge.
185.   Q. And at that time did you notice any discrepancy or difference between the policy 203.   We cannot apply the general rule on contracts of adhesion to the case at bar. Petitioner cannot
wordings as well as scope of coverage of Exhibits I and H respectively? claim it did not know the provisions of the policy. From the inception of the policy, petitioner
186.   A. No, sir, I did not discover any difference inasmuch (sic) as I was assured already that had required the respondent to copy verbatim the provisions and terms of its latest insurance
the policy wordings and rates were copied from the insurance policy I sent them but it policy from AHAC-AIU. The testimony of Mr. Leopoldo Mantohac, a direct participant in
was only when this case erupted that we discovered some discrepancies. securing the insurance policy of petitioner, is reflective of petitioners knowledge, viz:
187.   Q. With respect to the items declared for insurance coverage did you notice any 204.   DIRECT EXAMINATION OF LEOPOLDO MANTOHAC
discrepancy at any time between those indicated in Exhibit I and those indicated in 205.   TSN, September 23, 1991
Exhibit H respectively? 206.   pp. 20-21
188.   A. With regard to the wordings I did not notice any difference because it was exactly the 207.   Q. Did you indicate to Atty. Omlas (sic) what kind of policy you would want for those
same P393,000.00 on the two (2) swimming pools only against the peril of earthquake facilities in Agoo Playa?
shock which I understood before that this provision will have to be placed here because 208.   A. Yes, sir. I told him that I will agree to that renewal of this policy under Philippine
this particular provision under the peril of earthquake shock only is requested because Charter Insurance Corporation as long as it will follow the same or exact provisions of
this is an insurance policy and therefore cannot be insured against fire, so this has to be the previous insurance policy we had with American Home Assurance Corporation.
placed. 209.   Q. Did you take any step Mr. Witness to ensure that the provisions which you wanted in
189.   The verbal assurances allegedly given by respondents representative Atty. Umlas were not the American Home Insurance policy are to be incorporated in the PCIC policy?
proved. Atty. Umlas categorically denied having given such assurances. 210.   A. Yes, sir.
190.   Finally, petitioner puts much stress on the letter of respondents independent claims adjuster, 211.   Q. What steps did you take?
Bayne Adjusters and Surveyors, Inc. But as testified to by the representative of Bayne 212.   A. When I examined the policy of the Philippine Charter Insurance Corporation I specifically
Adjusters and Surveyors, Inc., respondent never meant to lead petitioner to believe that the told him that the policy and wordings shall be copied from the AIU Policy No. 206-4568061-9.
endorsement for earthquake shock covered properties other than the two swimming pools, viz:
191.   DIRECT EXAMINATION OF ALBERTO DE LEON (Bayne
192.   Adjusters and Surveyors, Inc.)
193.   TSN, January 26, 1993
194.   pp. 22-26
195.   Q. Do you recall the circumstances that led to your discussion regarding the extent of
coverage of the policy issued by Philippine Charter Insurance Corporation?
196.   A. I remember that when I returned to the office after the inspection, I got a photocopy
of the insurance coverage policy and it was indicated under Item 3 specifically that the
coverage is only for earthquake shock. Then, I remember I had a talk with Atty. Umlas
(sic), and I relayed to him what I had found out in the policy and he confirmed to me
002 Pineda v. CA (for Gueco c/o Daguman) reduced. The most persuasive rationale for adopting the view that the employer
September 27, 1993 | Davide Jr., J. | Group Policy acts as the agent of the insurer, however, is that the employee has no knowledge
of or control over the employer's actions in handling the policy or its
PETITIONER: LUZ PINEDA, MARILOU MONTENEGRO, VIRGINIA administration. An agency relationship is based upon consent by one person that
ALARCON, DINA LORENA AYO, CELIA CALUMBAG and LUCIA another shall act in his behalf and be subject to his control. It is clear from the
LONTOK, evidence regarding procedural techniques here that the insurer-employer
RESPONDENTS: HON. COURT OF APPEALS and THE INSULAR LIFE relationship meets this agency test with regard to the administration of the
ASSURANCE COMPANY, LIMITED policy, whereas that between the employer and its employees fails to reflect true
agency. The insurer directs the performance of the employer's administrative
SUMMARY: Prime Marine Services, Inc. (PMSI), a crewing/manning outfit, acts, and if these duties are not undertaken properly the insurer is in a position to
procured a Group Policy from Insular Life Assurance Co., Ltd. to provide life exercise more constricted control over the employer's conduct. In the light of the
insurance coverage to its sea-based employees. During the effectivity of the above disquisitions and after an examination of the facts of this case, we hold
policy, six covered employees perished at sea when their vessel sunk. They were that PMSI, through its President and General Manager, Capt. Nuval, acted as the
survived by the complainants-appellees, the beneficiaries under the policy. The agent of Insular Life. The latter is thus bound by the misconduct of its agent.
beneficiaries, except the spouses Alarcon, executed special powers of attorney
authorizing Capt. Nuval, President and General Manager of PMSI, to, among DOCTRINE: Group insurance is a comparatively new form of insurance. In the
others, “follow-up, ask, demand, collect and receive” for their benefit United States, the first modern group insurance policies appear to have been
indemnities of sums of money due them relative to the sinking of the vessel. By issued in 1911 by the Equitable Life Assurance Society. Group insurance is
virtue of these written powers of attorney, complainants-appellees were able to essentially a single insurance contract that provides coverage for many
receive their respective death benefits. Unknown to them, however, PMSI, in its individuals. In its original and most common form, group insurance provides life
capacity as employer and policyholder of the life insurance of its deceased or health insurance coverage for the employees of one employer.
workers, filed with Insular Life formal claims for and in behalf of the
beneficiaries, through Capt. Nuval. On the basis of the five special powers of We are convinced that the employer is the agent of the insurer in performing the
attorney, Insular Life drew against its account six (6) checks, four for duties of administering group insurance policies. It cannot be said that the
P200,000.00 each, one for P50,000.00 and another for P40,000.00 payable to the employer acts entirely for its own benefit or for the benefit of its employees in
order of complainants-appellees. Capt. Nuval, upon receipt of these checks undertaking administrative functions
endorsed and deposited them in his own account.
When Pineda et.al. learned that they were entitled, as beneficiaries, to life FACTS:
insurance benefits under a group policy, they sought to recover these benefits 121.  September 1983, Prime Marine Services Inc. (PMSI), a manning agency,
from Insular Life but the latter denied their claim on the ground that their procured a Group Policy from Insular Life Assurance Co. (Insular Life) to
liability was already extinguished. WoN the CA erred in the appreciation of the provide life insurance coverage to its sea-based employees enrolled under
SPA to include granting of authority to Capt. Nuval to obtain the insurance the plan.
proceeds from Insular Life? YES, because the special powers of attorney "do not
122.  During the effectivity of the policy, six covered employees of the PMSI
contain in unequivocal and clear terms authority to Capt. Nuval to obtain,
perished at sea when their vessel, M/V Nemos sunk in Morocco. They were
receive, receipt from respondent company insurance proceeds arising from the
death of the seaman-insured. On the contrary, the said powers of attorney are survived by Pineda et.al., the beneficiaries under the policy.
couched in terms which could easily arouse suspicion of an ordinary man. We 123.  Pineda et. al, sought to claim death benefits, they approached the President
are convinced that the employer is the agent of the insurer in performing the and General Manager of PMSI, Capt. Nuval. The latter evinced willingness
duties of administering group insurance policies. It cannot be said that the to assist Pineda to recover Overseas Workers Welfare Administration
employer acts entirely for its own benefit or for the benefit of its employees in (OWWA) benefits from the POEA arising frim the deaths of their
undertaking administrative functions. While a reduced premium may result if the husbands/sons.
employer relieves the insurer of these tasks, and this, of course, is advantageous 124.  They executed, with the exception of spouses Alarcon, a special power of
to both the employer and the employees, the insurer also enjoys significant attorney authorizing Capt. Nuval to “follow up, ask, demand, collect and
advantages from the arrangement. The reduction in the premium which results receive” for the benefit indemnities of sums due to them relative to the
from employer-administration permits the insurer to realize a larger volume of sinking of M/V Nemos. By virtue of this authority, they were able to
sales, and at the same time the insurer's own administrative costs are markedly
receive death benefits.
125.  Unknown to them, PMSI, in its capacity as employer and policyholder of released two checks in the amount of P150,000.00 for the three minor
the life insurance of the deceased workers, filed formal claims with Insular children (P50,000.00 each) of complainant, Dina Ayo and another check of
Life in behalf of the beneficiaries through Capt. Nuval. Among the P40,000.00 for minor beneficiary Marissa Lontok, daughter of another
documents for the processing of the claims were the five SPAs executed by complainant Lucia Lontok, there being no showing of any court
Pineda et.al. authorization presented or the requisite bond posted.
126.  Insular Life drew against its bank six checks four for 200k each, one for 50k 130.  On the other hand, the CA ruled that the powers of attorney, Exhibits "1" to
and another for 40k, payable to the order of Pineda et al. These checks were "5," relied upon by Insular Life were sufficient to authorize Capt. Nuval to
released to the treasurer of PMSI, who happened to be Capt. Nuval’s son- receive the proceeds of the insurance pertaining to the beneficiaries.
in-law deposited them in his account with Boston Bank. 131.  Hence, this issue.
127.  Pineda et.al. learned that they were entitled, as beneficiaries under the group
policy with Insular Life sought to recover these but the latter denied their
claim on the ground that the liability of Insular Life was already
ISSUE/s:
extinguished upon deliver to PMSI the six checks issued under their names.
1.   WoN the CA erred in the appreciation of the SPA to include granting of
128.  Pineda et.al. filed an administrative complaint against private respondent
authority to Capt. Nuval to obtain the insurance proceeds from Insular Life?
Insular Life which was filed with the Insurance Commission on 20
YES, because the special powers of attorney "do not contain in unequivocal
September 1989. They prayed therein that after due proceedings, Insular
and clear terms authority to Capt. Nuval to obtain, receive, receipt from
Life "be ordered to pay the claimants their insurance claims" and that
respondent company insurance proceeds arising from the death of the
"proper sanctions/penalties be imposed on" it "for its deliberate, feckless
seaman-insured. On the contrary, the said powers of attorney are couched in
violation of its contractual obligations to the complainants, and of the
terms which could easily arouse suspicion of an ordinary man.
Insurance Code." 5 Insular Life's motion to dismiss the complaint on the
ground that "the claims of complainants are all respectively beyond the
jurisdiction of the Insurance Commission as provided in Section 416 of the RULING: Petition is hereby affirmed.
Insurance Code.
129.  The Insurance Commission made the following conclusions: After taking RATIO:
into consideration the evidences [sic], testimonial and documentary for the
complainants and the respondent, the Commission finds that; First: The 1.   We rule for the petitioners Pineda et. al.
respondent erred in appreciating that the powers of attorney executed by 2.   We have carefully examined the specific powers of attorney, Exhibits "1" to
five (5) of the several beneficiaries convey absolute authority to Capt. "5," which were executed by petitioners Luz Pineda, Lucia B. Lontok, Dina
Nuval, to demand, receive, receipt and take delivery of insurance proceeds Ayo, Celia Calumag, and Marilyn Montenegro, respectively, on 14 May
from respondent Insular Life. A cursory reading of the questioned powers 1986 and uniformly granted to Capt. Rosendo Nuval the following powers:
of authority would disclosed [sic] that they do not contain in unequivocal
and clear terms authority to Capt. Nuval to obtain, receive, receipt from To follow-up, ask, demand, collect and receipt for my benefit
indemnities or sum of money due me relative to the sinking of
respondent company insurance proceeds arising from the death of the
M.V. NEMOS in the vicinity of El Jadida, Casablanca,
seaman-insured. On the contrary, the said powers of attorney are couched in
Morocco on the evening of February 17, 1986; and
terms which could easily arouse suspicion of an ordinary man. Insular Life
did not observe Section 18034 of the Insurance Code, when it issued or
To sign receipts, documents, pertinent waivers of indemnities
or other writings of whatsoever nature with any and all third
                                                                                                                       
                                                                                                                                                                                                                                                                                       
34 court authority or the giving of a bond where the interest of the minor in the
Section 180 is quotes [sic] partly as follows: In the absence of a judicial guardian,
particular act involved does not exceed twenty thousand pesos . . . .
the father, or in the latter's absence or incapacity, the mother of any minor, who is an
insured or a beneficiary under a contract of life, health or accident insurance, may
exercise, in behalf of said minor, any right, under the policy, without necessity of  
persons, concerns and entities, upon terms and conditions Q: What is the reason why policyholders are the ones who file the claim and
acceptable to my said attorney. not the designated beneficiaries of the employees of the policyholders?

3. We agree with the Insurance Commission that the special powers of attorney "do A: Yes because group insurance is normally taken by the employer as an
not contain in unequivocal and clear terms authority to Capt. Nuval to obtain, employee-benefit program and as such, the benefit should be awarded by the
receive, receipt from respondent company insurance proceeds arising from the death policyholder to make it appear that the benefit really is given by the employer
of the seaman-insured. On the contrary, the said powers of attorney are couched in
terms which could easily arouse suspicion of an ordinary man." The holding of the 6. On cross-examination, Urbano further elaborated that even payments, among other
CA to the contrary is principally premised on its opinion that: things, are coursed through the policyholder:

[t]here is nothing in the law which mandates a specific or special Q: What is the corporate concept of group insurance insofar as Insular Life is
power of attorney to be executed to collect insurance proceeds. concerned?
Such authority is not included in the enumeration of art. 1878 of
the New Civil Code. Neither do we perceive collection of
WITNESS:
insurance claims as an act of strict dominion as to require a special
power of attorney.
A: Group insurance is a contract where a group of individuals are covered
under one master contract. The individual underwriting characteristics of each
4. If this be so, then they could not have been meant to be a general power of
individual is not considered in the determination of whether the individual is
attorney since Exhibits "1" to "5" are special powers of attorney. The execution by
insurable or not. The contract is between the policyholder and the insurance
the principals of special powers of attorney, which clearly appeared to be in prepared company. In our case, it is Prime Marine and Insular Life. We do not have
forms and only had to be filled up with their names, residences, dates of execution, contractual obligations with the individual employees; it is between Prime
dates of acknowledgment and others, excludes any intent to grant a general power of
Marine and Insular Life.
attorney or to constitute a universal agency. Being special powers of attorney, they
must be strictly construed.
Q: And so it is part of that concept that all inquiries, follow-up, payment of
claims, premium billings, etc. should always be coursed thru the policyholder?
5. Certainly, it would be highly imprudent to read into the special powers of attorney
in question the power to collect and receive the insurance proceeds due to Pineda
et.al from Group Policy No. G-004694. Insular Life knew that a power of attorney in A: Yes that is our practice.
favor of Capt. Nuval for the collection and receipt of such proceeds was a deviation
from its practice with respect to group policies. Such practice was testified to by Mr. Q: And when you say claim payments should always be coursed thru the
Marciano Urbano, Insular Life's Assistant Manager of the Group Administrative policyholder, do you require a power of attorney to be presented by the
Department policyholder or not?

ATTY. CAGUIOA: a Not necessarily.

Can you explain to us why in this case, the claim was filed by a certain Capt. 7. This practice is usual in the group insurance business and is consistent with the
Noval [sic]? jurisprudence thereon in the State of California — from whose laws our Insurance
Code has been mainly patterned — which holds that the employer-policyholder is
WITNESS: the agent of the insurer.

A: The practice of our company in claim pertaining to group insurance, the 8. Group insurance is a comparatively new form of insurance. In the United States,
policyholder is the one who files the claim for the beneficiaries of the deceased. the first modern group insurance policies appear to have been issued in 1911 by the
At that time, Capt. Noval [sic] is the President and General Manager of Prime Equitable Life Assurance Society. Group insurance is essentially a single insurance
Marine. contract that provides coverage for many individuals. In its original and most
common form, group insurance provides life or health insurance coverage for the sales, and at the same time the insurer’s own administrative costs
employees of one employer. are markedly reduced.

9. The coverage terms for group insurance are usually stated in a master agreement Xxx xxx xxx
or policy that is issued by the insurer to a representative of the group or to an
administrator of the insurance program, such as an employer. The employer acts as a The most persuasive rationale for adopting the view that the
functionary in the collection and payment of premiums and in performing related employer acts as the agent of the insurer, however, is that the
duties. Likewise falling within the ambit of administration of a group policy is the employee has no knowledge of or control over the employer’s
disbursement of insurance payments by the employer to the employees. Most actions in handling the policy or its administration. An agency
policies, such as the one in this case, require an employee to pay a portion of the relationship is based upon consent by one person that another shall
premium, which the employer deducts from wages while the remainder is paid by the act in his behalf and be subject to his control. It is clear from the
employer. This is known as a contributory plan as compared to a non-contributory evidence regarding procedural techniques here that the insurer-
plan where the premiums are solely paid by the employer. employer relationship meets this agency test with regard to the
administration of the policy, whereas that between the employer
10. Although the employer may be the titular or named insured, the insurance is and its employees fails to reflect true agency. The insurer directs
actually related to the life and health of the employee. Indeed, the employee is in the the performance of the employer’s administrative acts, and if these
position of a real party to the master policy, and even in a non-contributory plan, the duties are not undertaken properly the insurer is in a position to
payment by the employer of the entire premium is a part of the total compensation exercise more constricted control over the employer’s conduct.
paid for the services of the employee. Put differently, the labor of the employees is
the true source of the benefits, which are a form of additional compensation to them. 14.   In the light of the above disquisitions and after an examination of the facts
of this case, we hold that PMSI, through its President and General Manager,
11. It has been stated that every problem concerning group insurance presented to a Capt. Nuval, acted as the agent of Insular Life. The latter is thus bound by
court should be approached with the purpose of giving to it every legitimate the misconduct of its agent.
opportunity of becoming a social agency of real consequence considering that the 15.   Insular Life, however, likewise recognized Capt. Nuval as the attorney-in-
primary aim is to provide the employer with a means of procuring insurance fact of the petitioners. Unfortunately, through its official, Mr. Urbano, it
protection for his employees and their families at the lowest possible cost, and in so acted imprudently and negligently in the premises by relying without
doing, the employer creates goodwill with his employees, enables the employees to question on the special power of attorney. In Strong vs. Repide, this Court
carry a larger amount of insurance than they could otherwise, and helps to attract and ruled that it is among the established principles in the civil law of Europe as
hold a permanent class of employees. well as the common law of American that third persons deal with agents at
their peril and are bound to inquire as to the extent of the power of the agent
12. In Elfstrom vs. New York Life Insurance Company, 27 the California Supreme with whom they contract
Court explicitly ruled that in group insurance policies, the employer is the agent of 16.   Even granting for the sake of argument that the special powers of attorney
the insurer. Thus: were in due form, Insular Life was grossly negligent in delivering the
checks, drawn in favor of the petitioners, to a party who is not the agent
We are convinced that the employer is the agent of the insurer in mentioned in the special power of attorney.
performing the duties of administering group insurance policies. It
cannot be said that the employer acts entirely for its own benefit or MINOR ISSUE
for the benefit of its employees in undertaking administrative
functions. While a reduced premium may result if the employer 17.   Nor can we agree with the opinion of the public respondent that since the
relieves the insurer of these tasks, and this, of course, is shares of the minors in the insurance proceeds are less than P50,000.00,
advantageous to both the employer and the employees, the insurer then under Article 225 of the Family Code their mothers could receive such
also enjoys significant advantages from the arrangement. The shares without need of either court appointments as guardian or the posting
reduction in the premium which results from employer- of a bond. It is of the view that said Article had repealed the third paragraph
administration permits the insurer to realize a larger volume of of Section 180 of the Insurance Code. The pertinent portion of Article 225
of the Family Code reads as follows:
Art. 225. The father and the mother shall jointly exercise legal
guardianship over the property of their unemancipated common
child without the necessity of a court appointment. In case of
disagreement, the father's decision shall prevail, unless there is
judicial order to the contrary.

Where the market value of the property or the annual income of the
child exceeds P50,000, the parent concerned shall be required to
furnish a bond in such amount as the court may determine, but not
less than ten per centum (10%) of the value of the property or
annual income, to guarantee the performance of the obligations
prescribed for general guardians.

18.   It is clear from the said Article that regardless of the value of the
unemancipated common child's property, the father and mother ipso
jure become the legal guardian of the child's property. However, if the
market value of the property or the annual income of the child exceeds
P50,000.00, a bond has to be posted by the parents concerned to guarantee
the performance of the obligations of a general guardian.
003 Vda. De Sindayen v. Insular (GUSTILO) DOCTRINE: When the policy is issued and delivered, in the absence of fraud or
September 4, 1935| Butte, J | Delivery of the Policy other grounds for rescission, it is plainly not within the intention of the parties that
there should be any questions held in abeyance or reserved for future determination
PETITIONER: Fortunata Lucero Viuda De Sindayen that leave the very existence of the contract in suspense and doubt. It is therefore in
RESPONDENTS: Insular Life Assurance Co. Ltd the public interest, for the public is profoundly and generally interested in life
insurance, as well as in the interest of the insurance companies themselves by giving
SUMMARY: A. Sindayen was employed as a linotype operator in the Bureau of certainly and security to their policies, that we are constrained to hold, as we, do, that
Printing at Manila.He was in Camiling Tarlac with his wife to spend Christmas the delivery of the policy to the insured by an agent of the company who is
vacation with his aunt, Felicidad Estrada. While there he made a written application authorized to make delivery or without delivery is the final act which binds the
to Insular for a policy of insurance on his life in the sum of P1,000 and he paid to the company and the insured as well in the absence of fraud or other legal ground for
agent Mendoza P15 cash as part of the first premium. It was agreed with the agent, rescission.
Mendoza that the policy, when and if issued, should be delivered to his aunt. Estrada, FACTS:
with whom A. Sindayen left the sum of P26.06 to complete the payment of the first 95.   Arturo Sindayen (A. Sindayen), up to the time of his death on January 19,
annual premium of P40.06. On January 1, 1933, A. Sindayen, was examined by 1933, was employed as a linotype operator in the Bureau of Printing at
Insular’'s doctor who made a favorable report, to Insular. The next day, January 2, A. Manila and had been such for eleven years prior thereto. He and his wife
Sindayen returned to Manila and resumed his work as a linotype operator in the went to Camiling, Tarlac, to spend the Christmas vacation with his aunt,
Bureau of Printing. On January 11, Insular accepted the risk and issued policy No. Felicidad Estrada (Estrada).
47710 dated back to December 1, 1932, and mailed the same to its agent, Mendoza, 96.   While there he made a written application on December 26, 1932, to Insular
for delivery to A Sindayen which was eventually received by Estrada upon her Life Assurance Co., Ltd. (Insular) through its agent, Cristobal Mendoza
payment of the balance of the 1st year premium. (Mendoza), for a policy of insurance on his life in the sum of P1,000 and he
A.  Sindayen died on January 19 and on January 20 Mendoza, learned of the death of paid to the agent Mendoza P15 cash as part of the first premium. It was
A. Sindayen and called on Estrada and asked her to return the policy but Mendoza agreed with the agent, Mendoza that the policy, when and if issued, should
did not return or offer to return the premium paid. Estrada on his aforesaid statement be delivered to his aunt. Estrada, with whom A. Sindayen left the sum of
gave him the policy. Insular obtained from the beneficiary, the widow of A, P26.06 to complete the payment of the first annual premium of P40.06.
Sindayen, her signature to a legal document entitled "ACCORD, SATISFACTION 97.   On January 1, 1933, A. Sindayen, who was then twenty-nine years of age,
AND RELEASE" whereby in consideration of the sum of P40.06 paid to her by a was examined by Insular’'s doctor who made a favorable report, to Insular.
check of Insular, she "assigns, releases and forever discharges said Insular, its The next day, January 2, A. Sindayen returned to Manila and resumed his
successors and assigns, of all claims, obligation in or indebtedness which she, as work as a linotype operator in the Bureau of Printing. On January 11, 1933,
such beneficiary ever had. The said check for P40.06 was never cashed but returned Insular accepted the risk and issued policy No. 47710 dated back to
to Insular. Thereupon this action was brought to enforce payment of the policy. The December 1, 1932, and mailed the same to its agent, Mendoza, for delivery
issue is WoN the policy took effect?- YES because Mendoza was an agent of to A Sindayen. On January 11, 1933, A. Sindayen was at work in the
Insular and his decision to deliver the policy is binding on the company. Bureau of Printing. On January 12, he complained of a severe headache and
The main defense of Insular is that the said policy never took effect because of remained at home. On January 15, he called a physician who found that he
paragraph 3 of the application above quoted, for at the time of its delivery by the was suffering from acute nephritis and uremia. His illness did not yield to
agent as aforesaid the insured was not in good health. The SC held that Mendoza was treatment and on January 19, he died.
duly licensed by the Insurance Commissioner to act as the agent of the Insular. The 98.   The policy which Insular issued and mailed in Manila on January 11 was
well known custom of the insurance business and the evidence in this case prove that received by its agent, Mendoza, in Camiling, Tarlac, on January 16. On
Mendoza was not regarded by Insular as a mere conduit or automaton for the January 18, the agent, Mendoza, in accordance with his agreement with the
performance of the physical act of placing the policy in the hands of the insured. insured, delivered the policy to Estrada upon her payment of the balance of
Further, when the policy is issued and delivered, in the absence of fraud or other the first year's annual premium. The agent, Mendoza, asked Estrada if her
grounds for rescission, it is plainly not within the intention of the parties that there nephew was in good health and she replied that she believed so because she
should be any questions held in abeyance or reserved for future determination that had no information that he was sick and he thereupon delivered to her the
leave the very existence of the contract in suspense and doubt. If this were not so, the policy.
entire business world which deals so voluminously in insurance would be affected by 99.   On January 20 the agent, Mendoza, learned of the death of A. Sindayen and
this uncertainly. called on Estrada and asked her to return the policy but Mendoza did not
return or offer to return the premium paid. Estrada on his aforesaid 3.  WoN there was delivery of the policy even if it was not delivered to the
statement gave him the policy. insured himself? YES because delivery to the insured in person is not necessary.
100.  On February 4, Insular obtained from the beneficiary, the widow of A, Delivery may be made by mail or to a duly constituted agent.
Sindayen, her signature to a legal document entitled "ACCORD,
SATISFACTION AND RELEASE" whereby in consideration of the sum of RULING :In view of the premises, we hold that Insular assumed the risk covered by
P40.06 paid to her by a check of Insular, she "assigns, releases and forever policy No. 47710 on the life of A. Sindayen on January 18, 1933, the date when the
discharges said Insular, its successors and assigns, of all claims, obligation policy was delivered to A Sindayen. The judgment appealed from is therefore
in or indebtedness which she, as such beneficiary ever had or now has, reversed with directions to enter judgment against Insular in the sum of P1,000
hereafter shall, or may have, for, upon, or by reason of said policy of life together with interest at the legal rate from and after May 4, 1933, with costs in both
insurance numbered 47710 upon the life of said A. Sindayen, the latter now instances against Insular.
deceased, or arising therefrom or connected therewith in any manner", RATIO:
101.  The said check for P40.06 was never cashed but returned to Insular. Main issue:
Thereupon this action was brought to enforce payment of the policy. By the 104.  The main defense of Insular is that the said policy never took effect because
terms of the policy, an annual premium of P40.06 is due on the first day of of paragraph 3 of the application above quoted, for at the time of its
December of each year, the first premium already paid by the insured delivery by the agent as aforesaid the insured was not in good health. We
covering the period from December 1, 1932 which is to December 1, 1933. have not heretofore been called upon to interpret and apply this clause in
It is to be noted that the policy was not issued and Insular assumed no actual life insurance application, but identical or substantially identical clauses
risk prior to January 11, 1933. have been construed and applied in a number of cases in the United States
102.  The policy contains the following paragraph: THE CONTRACT. This and the decisions thereon are far from uniform or harmonious. We do not
Policy and the application herefor constitute the entire contract between the find it practicable to attempt to determine where the weight of the authority
parties hereto. All statements made by the Insured shall, in the absence lies and propose to resolve this case on its own facts.
of fraud, be deemed representations and not warranties, and no such 105.  There is one line of cases which holds that the stipulation contained in
statement shall void the Policy unless it is contained in the written paragraph 3 is in the nature of a condition precedent, that is to say, that
application, a copy of which is attached to this Policy. Only the there can be no valid delivery to the insured unless he is in good health at
President, or the Manager, acting jointly with the Secretary or Assistant the time; that this condition precedent goes to the very essence of the
Secretary (and then only in writing signed by them) have power in behalf of contract and cannot be waived by the agent making delivery of the policy.
the Company to issue permits, or to modify this or any contract, or to On the other hand, a number of American decisions hold that an agent to
extend the time for making any premium payment, and the Company shall whom a life insurance policy similar to the one here involved was sent with
bound by any promise or representation heretofore hereafter given by any instructions to deliver it to the insured has authority to bind the company by
person other than the above-named officials, and by them only in writing making such delivery, although the insured was not in good health at the
and signed conjointly as stated. time of delivery, on the theory that the delivery of the policy being the final
103.  The application which A. Sindayen signed in Camiling, Tarlac, on act to the consummation of the contract, the condition as to the insurer's
December 26, 1932, contained among others the following provisions: (a) good health was waived by the company.
That if this application is accepted and a policy issued in my favor, I bind 106.  A number of these cases go to the of holding that the delivery of the policy
myself to accept the same and to pay at least the first year's premium by the agent to the insured consummates the contract even though the agent
thereon in the City of Manila; (b) That the said policy shall not take effect knew that the insured was not in good health at the time, the theory being
until the first premium has been paid and the policy has been delivered to that his knowledge is the company's knowledge and his delivery of the
and accepted by me, while I am in good health; (c) That the agent taking policy is the company's delivery; that when the delivery is made
this application has no authority to make, modify or discharge contracts, or notwithstanding this knowledge of the defect, the company is deemed to
to waive any of the Company's right or requirements. have waived the defect. Although that appears to be the prevailing view in
ISSUE/s: the American decisions and leads to the same conclusion, namely, that the
1.  WoN the policy took effect?- YES because Mendoza was an agent of Insular and act of delivery of the policy in the absence of fraud or other ground for
his decision to deliver the policy is binding on the company recission consummates the insurance, we are inclined to the view that it is
Other issues: more consonant with the well known practice of life insurance companies
2.  WoN the Accord, Satisfaction & Releae made by Insular is fraudulent?-YES and the evidence in the present case to rest our decision on the proposition
it is inequitable that Mendoza was authorized by the company to make the delivery of the
policy when he received the payment of the first premium and he was absence of fraud or other grounds for rescission, it is plainly not within
satisfied that the insured was in good health. the intention of the parties that there should be any questions held in
107.  Mendoza was duly licensed by the Insurance Commissioner to act as abeyance or reserved for future determination that leave the very
the agent of the Insular. The well known custom of the insurance existence of the contract in suspense and doubt. If this were not so, the
business and the evidence in this case prove that Mendoza was not entire business world which deals so voluminously in insurance would be
regarded by Insular as a mere conduit or automaton for the affected by this uncertainly.
performance of the physical act of placing the policy in the hands of the 111.  It is therefore in the public interest, for the public is profoundly and
insured. If Mendoza were only an automaton then the legally effective generally interested in life insurance, as well as in the interest of the
delivery of the policy and the consummation of the contract occurred when insurance companies themselves by giving certainly and security to
Insular expressed its will to release the policy by mailing it to Mendoza, their policies, that we are constrained to hold, as we, do, that the
namely, on January 11, 1933. In such a case Mendoza would perform a delivery of the policy to the insured by an agent of the company who is
purely ministerial act and have no discretion. He could do nothing but make authorized to make delivery or without delivery is the final act which
unconditional delivery. The legal result would be the same as if Insular had binds the company and the insured as well in the absence of fraud or
mailed the policy on January 11, to the insured directly using the post-office other legal ground for rescission. The fact that the agent to whom it has
as its conduit for delivery. On January 11, A. Sindayen was in good health entrusted this duty and corporation can only act through agents is derelict or
performing his regular duties in the Bureau of Printing. negligent or even dishonest in the performance of the duty which has been
108.  But we are not inclined to take such a restrictive view of the agent's entrusted to him would create a liability of the agent to the company but
authority because the evidence in the record shows that Mendoza had the does not resolve the company's obligation based upon the authorized acts of
authority, given him by Insular, to withhold the delivery of the policy to the agent toward a third party who was not in collusion with the agent.
the insured "until the first premium has been paid and the policy has 112.  Paragraph 4 of the application to the effect "that the agent taking this
been delivered to and accepted by the insured while I am in good application has no authority to make, modify or discharge contracts or to
health". Whether that condition had been met or not plainly calls for waive any of the company's rights or requirements" is not in point.
the exercise of discretion. Granted that Mendoza's decision that the Mendoza neither waived nor pretended to waive any right or requirement of
condition had been met by the insured and that it was proper to make a Insular. In fact, his inquiry as to the state of health of the insured discloses
delivery of the policy to him is just as binding on the company as if the that he was endeavoring to assure himself that this requirement of Insular
decision had been made by its board of directors. Granted that Mendoza had been satisfied. In doing so, he acted within the authority conferred
made a mistake of judgement because he acted on insufficient evidence as on him by his agency and his acts within that authority bind the
to the state of health of the insured. But it is not charged that the mistake company. Insular therefore having decided that all the conditions
was induced by any misconduct or omission of duty of the insured. precedent to the taking effect of the policy had been complied with and
109.  It is the interest not only the applicant but of all insurance companies having accepted the premium and delivered the policy thereafter to the
as well that there should be some act which gives the applicant the insured, Insular is now estopped to assert that it never intended that
definite assurance that the contract has been consummated. This sense the policy should take effect.
of security and of peace of mind that one's defendants are provided for Accord, Satisfaction & Release is Fraudulent
without risk either of loss or of litigation is the bedrock of life 113.  Insular does not set up any defense of fraud, misconduct or omission of
insurance. A cloud will be thrown over the entire insurance business if the duty of A. Sindayen or his agent, Estrada or of the beneficiary. In its answer
condition of health of the the insured at the time of delivery of the policy it pleads the "ACCORD, SATISFACTION AND RELEASE" signed by the
may be required into years afterwards with the view to avoiding the policy widow of A. Sindayen.
on the ground that it never took effect because of an alleged lack of good 114.  With respect to the Accord, Satisfaction & Release, it suffices to say
health, at the time of delivery. that this release is so inequitable, not to say fraudulent.
110.  It is difficult to imagine that the insurance company would take such a Delivery of the Policy
position in the face of the common belief of the insuring public that when 115.  It is suggested in Insular’'s brief that fhere was no delivery of the policy in
the policy is delivered, in the absence of fraud or other grounds for this case because the policy was not delivered to and accepted by A.
rescission, the contract of insurance is consummated. The insured rests and Sindayen in person.
acts on that faith. So does the insurance company, for that matter, for from 116.  Delivery to the insured in person is not necessary. Delivery may be made
the date of delivery of the policy it appropriates to its own use the premium by mail or to a duly constituted agent. Insular cites no authorities to support
paid by the insured. When the policy is issued and delivered, in the its proposition and none need be cited to refute it.
Dissenting of Imperial, J.:
117.   The stipulation that the insurance contract shall produce no effect unless the payment of the first premium
and the delivery of the policy be made when the insured is in good health, is not in conflict with any
provision of the Insurance Law now in force, nor with any other law of a general character; neither is said
stipulation contrary to morals or public order, and therefore the same is valid and binding upon the parties.
118.   The majority opinion states that the delivery of the policy by the agent after he has made use of the
discretion conferred upon him by Insular to deliver or to withhold said policy, is binding upon Insular and
the latter cannot evade the consequences thereof. This same legal question has been raised before various
appellate courts of several states of the Union, which made a distinction between agents whose only power
consisted in soliciting insurance and in delivering policies and those who, in addition to such power, were
authorized to issue policies and accept risks on behalf of insurance companies. In the first case the doctrine
is uniform that the acts of agents with limited powers are not binding upon the insurance companies, whereas
in the second case the acts of the agents bind and prejudice the insurance companies represented by them.
119.   It is clear, therefore, that the delivery of the policy by Mendoza does not bind Insular nor is Insulart estopped
from alleging its defense, for the simple reason that Mendoza was not an agent with authority to issue
policies or to accept risks in the name of his principle.
120.   However, it it said that Insular ikewise waived the defense which has hereinbefore been extensively
considered, because it failed to return the first premium collected, and this alleged failure is predicated upon
the statement contained in the penultimate paragraph of the instrument stating that the check for P40.06 was
returned to the widow in consideration of her waiver of any claim whatsoever. A careful reading of the
instrument will convince the mind that what was really meant is that the delivery of the check was another
consideration of the widow's waiver, it being self-evident that said check constituted, in effect, a refund of
the first premium paid by insured and received by the insurer. It is ridiculous to think that such a negligible
amount has been the only consideration of the widow’s waiver of any right or benefit accurring to her from
the policy. A careful perusal of the instrument will show that the real consideration of the widow's waiver
was the unenforceability of the policy due to her husband's illness and the mutual desire of the plaintiff of
the insurer to settle amicably the cases instead of resorting to courts.
121.   In conclusion it is my opinion: (1) That the policy has not produced any effect from which the widow may
derive any right, and (2) that she has expressly waived any all rights accurring from the policy; and for these
reasons I dissent from the majority opinion.
004 Argente v. West Coast Life Insurance Co. (Hilario) 1.   The wife signed a like application for the same policy as her
March 19, 1928 | Malcolm, J. | Concealment husband. Both applications (except names and signatures of the
spouses applicants) were written by Jose Geronimo del Rosario,
PETITIONER: Bernardo Argente an agent for West Coast.
RESPONDENTS: West Coast Life Insurance 2.   But all information contained in applications were furnished to
agent del Rosario by Argente.
SUMMARY: Bernardo Argente and his wife, Vicenta de Ocampo, applied for a 123.  Argente was examined by a medical examiner for Wesst Coast, and the
joint life insurance policy with West Coast Life Insurance Co. They were medically results were recorded in the Medical Examiner’s Report. This was all in the
examined by the company’s doctor, and they supplied the answers to the questions in hand writing of the doctor, but information and answers to questions on the
the Medical Examiner’s Report. Vicenta died of cerebral apoplexy a few months Report were furnished to the doctor by Argente. The same happened with
after the issuance of the policy, prompting Argente to claim on the policy (he was Vicenta, and she gave the answers to the questions to the doctor.
asking for P 15000). Upon the investigation of West Coast, it was discovered that the 124.  The spouses increased the amount of the insurance from P2000 to P15000,
spouses made several fraudulent declarations regarding the condition of their health and asked that the policy be dated May 15, 1925 (they asked this on May 9).
for the last five years, concealing the time the husband was hospitalized and that the 125.  A temporary policy for P 15 000 was issued in favor of the spouses, b ut it
wife had several neurological/phsycological problems (like she was diagnosed with was not delivered to Argente until the first quarterly premium on the policy
alcoholism, then later manic depressive psychosis, then a final diagnosis of psycho- was paid.
neurosis). Thus, West Coast refused to honor Argente’s claim, prompting him to file 1.   Since 30 days had elapsed since the applicants were examined,
an action before the Trial Court. The Trial Court ruled in favor of West Coast, and each of them was required to file a certificate of health before the
the Supreme Court upheld the Trial Court’s decision. policy was delivered on July 2.
126.  On November 18 of the same year (4 months after delivery of policy)
In an action on a life insurance policy where the evidence conclusively shows that Vicenta died of cerebral apoplexy, and her husband Argente presented a
the answers to questions concerning diseases were untrue, the truth of falsity of the claim for P 15 000.
answers become the determining factor. In the policy was procured by fraudulent 1.   Following investigation by West Coast, it was discovered that the
representations, the contract of insurance apparently set forth therein was answers given by the insured in their medical examinations
never legally existent. It can fairly be assumed that had the true facts been with regard to their health and previous illness and medical
disclosed by the assured, the insurance would never have been granted. Assurer in attendance were untrue.
assuming a risk is entitled to know every material fact of which the assured has 127.  For that reason, the West Coast Life Insurance Co. refused to pay the
exclusive or peculiar knowledge, as well as all material facts which directly tend to claim of Bernardo Argente, and on May 25, 1926, wrote him to the effect
increase the hazard or risk which are known by the assured, or which ought to be or that the claim was rejected because the insurance was obtained through
are presumed to be known by him. And a concealment of such facts vitiates the fraud and misrepresentation. This prompted Argente to file an action in the
policy. trial court.
128.  What they concealed (I don’t think you have to read this in detail, but just in
case sir asks!): In the Medical Examiner's Report that Bernardo Argente, in
DOCTRINE: One ground for the rescission of a contract of insurance under the response to the question asked by the medical examiner, "Have you ever
Insurance Act is "a concealment," which in section 25 is defined as "A neglect to consulted a physician for, or have you ever suffered from any ailment or
communicate that which a party knows and ought to communicate." disease of, the brain or nervous system?" answered "No." To the question,
The principal question, therefore, must be, Was the assurer misled or deceived into "Have you consulted a physician for any ailment or disease not included in
entering a contract obligation or in fixing the premium of insurance by a your above answer," answered "Yes. Nature of Ailment, Disease or Injury.
withholding of material information of facts within the assured's knowledge or Scabies, Number of attacks 1, Date 1911. Duration 1 month, Severity Fair,
presumed knowledge? results and, if within five years, name and address of every physician
consulted. Dr. P. Guazon. Cured. Dr. Guazon is dead now." And to the
question, "What physician or physicians, if any, not named above, have you
consulted or been treated by, within the last five years and for what illness
or ailment? (If none, so state)" answered "No." It is, however, not disputed
FACTS: that on January 10, 11, and 13, 1923, Bernardo Argente was confined in
122.  West Coast issued a joint life insurance policy in favor of Bernardo Argente
and his wife, Vicenta de Ocampo.
the Philippine General Hospital where he was treated by Dr. Agerico B. neglect to communicate that which a party knows and ought to
M. Sison for cerebral congestion and Bell's Palsy. communicate."
129.  What they concealed (I don’t think you have to read this in detail, but just in 121.  Argente claims that the alleged concealment was immaterial and
case sir asks!): Vicenta de Ocampo, in response to the question asked by the insufficient to avoid the policy.
medical examiner, "How frequently, if at all, and in what quantity do you 122.  The SC does not buy this.
use beer, wine, spirits or other intoxicants?" answered "Beer only in small 123.  In an action on a life insurance policy where the evidence conclusively
quantities occasionally." To the question, "Have you ever consulted a shows that the answers to questions concerning diseases were untrue, the
physician for or have you ever suffered from any ailment or disease of the truth of falsity of the answers become the determining factor. In the policy
brain or nervous system?" answered "No." To the question, "What was procured by fraudulent representations, the contract of insurance
physician or physicians, if any, not named above, have you consulted or apparently set forth therein was never legally existent. It can fairly be
been treated by, within the last five years and for what illness or ailment? assumed that had the true facts been disclosed by the assured, the
(If none, so state)" answered "None." And to the question, "Are you in good insurance would never have been granted.
health as far as you know and believe?" answered "Yes." 124.  In Joyce, The Law of Insurance, second edition, volume 3, Chapter LV, is
130.  It is, however, not disputed that Vicenta de Ocampo was taken by a found the following: Concealment exists where the assured has
patrolman, at the request of her husband, Bernardo Argente, on May 19, knowledge of a fact material to the risk, and honesty, good faith, and
1924, to the Meisic police station, and from there was transferred to the fair dealing requires that he should communicate it to the assured, but
San Lazaro Hospital. In San Lazaro Hospital, her case was diagnosed by he designated and intentionally with holds the same. Another rule is that
the admitting physician as "alcoholism," but later Doctor Domingo made if the assured undertakes to state all the circumstances affecting the
a diagnosis of probable "manic-depressive psychosis," and still, later in risk, a full and fair statement of all is required.
Mary Chiles Hospital, made a final diagnosis of "phycho-neurosis." 125.  It is also held that the concealment must, in the absence of inquiries, be
131.  Argente concedes and admits most of these facts, yet alleges that both he not only material, but fraudulent, or the fact must have been
and his wife revealed their medical conditions to the doctor who intentionally withheld.
examined them from the insurance company, but that the doctor, 1.   if no inquiries are made and no fraud or design to conceal
presumably acting in collusion with the insurance agent, failed to enters into the concealment the contract is not avoided.
record them in the medical reports. 126.  But it would seem that if a material fact is actually known to the assured, its
132.  The Trial Court ruled in favor of West Coast, and disregarded the concealment must of itself necessarily be a fraud, and if the fact is one
testimonies presented by Argente (his own testimony, his clerk’s, the which the assured ought to know, or is presumed to know, the
doctor’s, and the agent’s). presumption of knowledge ought to place the assured in the same
1.   There appears no motive on the part of the doctor to fasify the position as in the former case with relation to material facts;
Medical Examiner’s Reports. 127.  The basis of the rule vitiating the contract in case of concealment is that it
2.   Further, from the evidence and representations in their applications misleads or deceives the insurer into accepting the risk, or accepting it at the
regarding their health for the last 5 years before they applied for rate of premium agreed upon. The insurer, relying upon the belief that the
insurance, they were totally false. assured will disclose every material within his actual or presumed
133.  He now questions the trial court’s ruling before the SC. knowledge, is misled into a belief that the circumstance withheld does not
exist, and he is thereby induced to estimate the risk upon a false basis that it
ISSUE/s: does not exist.
12.   WoN the Trial Court was correct in saying that West Coast correctly 128.  The principal question, therefore, must be, Was the assurer misled or
refused to pay Argente’s claim – YES, because Argente is guilty of deceived into entering a contract obligation or in fixing the premium of
concealment, on of the grounds for rescission of a contract of insurance insurance by a withholding of material information of facts within the
under the Insurance Act. assured's knowledge or presumed knowledge?
129.  Assurer in assuming a risk is entitled to know every material fact of which
RULING: SC affirmed the lower courts decision. Pwede rin wherefore. the assured has exclusive or peculiar knowledge, as well as all material
facts which directly tend to increase the hazard or risk which are known by
RATIO: the assured, or which ought to be or are presumed to be known by him. And
120.  One ground for the rescission of a contract of insurance under the a concealment of such facts vitiates the policy.
Insurance Act is "a concealment," which in section 25 is defined as "A  
005 EDILLON VS MANILA BANKERS LIFE (HIRANG) FACTS:
30 September 1982 |Vasquez, J. | Concealment 132.  In April 1969, Carmen O. Lapuz applied with Manila Bankers Life (MBL)
for insurance coverage against accident and injuries. She filled up the blank
PETITIONER: REGINA EDILLON application form given to her and filed the same with MBL
RESPONDENTS: MANILA BANKERS LIFE INSURANCE CORPORATION 133.  In the said application form which was dated April 15, 1969, she gave the
date of her birth as July 11, 1904. On the same date, she paid the sum of
SUMMARY: P20.00 representing the premium for which she was issued the
Insured: Carmen Lapuz corresponding receipt signed by an authorized agent of the MBL
Insurer: Manila Bankers Life Insurance Corporation (MBL) 134.  During the effectivity of the Insurance Policy, Carmen Lapuz died, and as
Beneficiary: Regina Edillon such, Regina L. Edillon, the sister of the insured and who was the named
Policy: Insurance against accident and injuries beneficiary in the policy, filed her claim for the proceeds of the insurance
In 1969, Lapuz applied with MBL for a life insurance coverage against accident and 135.  MBL denied the claim of Edillon stating that it is not liable to pay claims
injuries. When she applied for the insurance she was already over 60 years of age under the policy in behalf of "persons who are under the age of sixteen (16)
but despite this, her application was accepted. Under such policy, her beneficiary years of age or over the age of sixty (60) years x x x." It is to note that when
was her sister, Regina Edillon. During the effectivy of the policy, Lapuz died due to Lapuz applied for the policy, she was already over 60 years old
an accident and as such, Edillon, as beneficiary filed a claim to collect the insurance 136.  The trial court ruled in favor of MBL stating that:
proceeds but was denied by MBL contending that in the Insurance Certicifacate a.   The policy of insurance being a contract of adhesion, it was the
provided for a provision which states that it is not liable to pay claims under the duty of the insured to know the terms of the contract he or she is
policy in behalf of "persons who are under the age of sixteen (16) years of age or entering into; the insured in this case, upon learning from its terms
over the age of sixty (60) years x x x." It is to note that when Lapuz applied for the that she could not have been qualified under the conditions stated
policy, she was already over 60 years old. The issue in this case is WoN MBL is in said contract, what she should have done is simply to ask for a
liable and the court ruled in the affirmative. The age of Lapuz was never concealed refund of the premium that she paid
to MBL. In fact, the application clearly showed that at the time of application Lapuz ISSUE/s: WoN Manila Bankers Life is liable – Yes, the court ruled that the fact
was almost 65 yrs old. The accident which resulted in the death of the insured, a that MBL accepted the application, knowing that the age of the applicant is an
risk covered by the policy, occurred on May 31, 1969 - 45 DAYS after the important element, showed that MBL either waived such criteria or it’s employees
insurance coverage was applied for. There was sufficient time for the private are negligence or incompetence for overlooking such fact
respondent to process the application and to notice that the applicant was over 60
years of age and thereby cancel the policy on that ground if it was minded to do RULING: WHEREFORE, the judgment appealed from is hereby REVERSED and
so. If MBL failed to act, it is either because it was willing to waive such SET ASIDE.
disqualification; or, through the negligence or incompetence of its employees for
which it has only itself to blame, it simply overlooked such fact. Under the RATIO:
circumstances, the insurance corporation is already deemed in estoppel 213.  The age of the insured Carmen O. Lapuz was not concealed to the insurance
company. Her application for insurance coverage which was on a printed
DOCTRINE: It is usually held that where the insurer, at the time of the issuance of form furnished by private respondent and which contained very few items
a policy of insurance, has knowledge of existing facts which, if insisted on, would of information clearly indicated her age at the time of filing the same to be
invalidate the contract from its very inception, such knowledge constitutes a waiver almost 65 years of age
of conditions in the contract inconsistent with the known facts, and the insurer is 214.  Despite such information which would hardly be overlooked in the
stopped thereafter from asserting the breach of such conditions. The law is application form, considering its prominence thereon and its materiality to
charitable enough to assume, in the absence of any showing to the contrary, that an the coverage applied for, the respondent insurance corporation received her
insurance company intends to execute a valid contract in return for the premium payment of premium and issued the corresponding certificate of insurance
received; and when the policy contains a condition which renders it voidable at its without question
inception, and this result is known to the insurer, it will be presumed to have 215.  The accident which resulted in the death of the insured, a risk covered by
intended to waive the conditions and to execute a binding contract, rather than to the policy, occurred on May 31, 1969 - 45 DAYS after the insurance
have deceived the insured into thinking he is insured when in fact he is not, and to coverage was applied for. There was sufficient time for the private
have taken his money without consideration respondent to process the application and to notice that the applicant was
over 60 years of age and thereby cancel the policy on that ground if it was
minded to do so. If MBL failed to act, it is either because it was willing to
waive such disqualification; or, through the negligence or incompetence of
its employees for which it has only itself to blame, it simply overlooked
such fact. Under the circumstances, the insurance corporation is already
deemed in estoppel
216.  Its inaction to revoke the policy despite a departure from the exclusionary
condition contained in the said policy constituted a waiver of such
condition. This doctrine was upheld by several cases:
a.   " Que Chee Gan vs. Law Union Rock Insurance Co., Ltd." - In this
case, the requirement of the insurance policy was to have 11 fire
hydrants however, the insurance company in this case, from the
very beginning, knew that the insured only had 2 but nevertheless
accepted the insurance policy and never revoked such policy. The
court in this case said that: “It is usually held that where the
insurer, at the time of the issuance of a policy of insurance, has
knowledge of existing facts which, if insisted on, would invalidate
the contract from its very inception, such knowledge constitutes a
waiver of conditions in the contract inconsistent with the known
facts, and the insurer is stopped thereafter from asserting the
breach of such conditions. The law is charitable enough to
assume, in the absence of any showing to the contrary, that an
insurance company intends to execute a valid contract in return for
the premium received; and when the policy contains a condition
which renders it voidable at its inception, and this result is known
to the insurer, it will be presumed to have intended to waive the
conditions and to execute a binding contract, rather than to have
deceived the insured into thinking he is insured when in fact he is
not, and to have taken his money without consideration”

b.   Capital Insurance & Surety Co., Inc. vs. Plastic Era Co., Inc, -
The company issued the policy upon the execution of a promissory
note for the payment of the premium. A check given subsequently
by the insured as partial payment of the premium was dishonored
for lack of funds. Despite such deviation from the terms of the
policy, the insurer was held liable. The insurance corporation
impliedly agreed to modify the tenor of the insurance policy and in
effect, waived the provision therein that it would only pay for the
loss or damage in case the same occurs after the payment of the
premium. Considering that the insurance policy is silent as to the
mode of payment, Capital Insurance is deemed to have accepted
the promissory note in payment of the premium
006 CANILANG v. CA (LAGUILLES) 17.   On August 5, Jaime Canilang died of congestive heart failure, anemia, and
June 7, 1993 | Feliciano, J. | Concealment chronic anemia. Thelma filed a claim with Great Pacific which the latter
denied, upon the ground that Jaime had concealed material information
PETITIONER: Thelma vda. De Canilang from it.
RESPONDENTS: CA and Great Pacific Life Insurance Corporation 18.   Thelma then filed a complaint against Great Pacific for the recovery of the
insurance proceeds. During the hearing called by the Insurance
SUMMARY: Jaime consulted a doctor and he was diagnosed with sinus Commissioner, Thelma testified that she was not aware of any serious
tachycardia and later on acute bronchitis. He then applied for a non-medical illness suffered by her late husband and that, as far as she knew, her
insurance policy with Great Pacific Life Assurance Company, naming his wife, husband died because of a kidney disorder.
Thelma as his beneficiary. Subsequently, Jaime died of congestive heart failure and 19.   A deposition was given by Dr. Claudio, stating that he previously treated
anemia. Thelma then filed a claim with Great Pacific but the latter refused because Jaime for sinus tachycardia and acute bronchitis. Great Pacific presented Dr.
Jaime allegedly concealed material information from it. Thelma filed a complaint Quismorio, a physician and a medical underwriter working for Great
against Great Pacific for the recovery of the insurance proceeds. Thelma testified Pacific. She testified that as a rule, medical examinations are required only
that she was not aware of any serious illness suffered by Jaime. The Insurance in cases where the applicant has indicated in his application that he has
Commissioner ruled in favor of Thelma on the ground that the ailment of Jaime was previously undergone medical consultation and hospitalization.
not so serious that even if it had been disclosed, it would not have affected Great 20.   The Insurance Commissioner ordered Great Pacific to pay P19,700 plus
Pacific’s decision to insure him. On appeal, the CA reversed the IC’s decision and
legal interest after holding that:
found that the failure to disclose previous medical consultation and treatment a.   The ailment of Jaime was not so serious that even if it had been disclosed, it would
constituted material information which should have been communicated to Great not have affected Great Pacific’s decision to insure him;
Pacific. The issue is WoN Jaime concealed his true state of health. b.   Great Pacific waived its rights to inquire into the health condition of the applicant
by the issuance of the policy despite the lack of answers to some of the pertinent
The SC held in the affirmative. In Jaime’s application for the insurance, he left questions in the insurance application;
c.   There was no intentional concealment on the part of the insured as he had thought
blank the portion for the exceptions that he consulted Dr. Claudio who found him to that he was merely suffering from a minor ailment and simple cold;
be suffering from sinus tachycardia and acute bronchitis. The Court agrees with the d.   BP 874 which voids an insurance contract whether or not concealment was
CA that the information which Jaime Canilang failed to disclose was material to the intentionally made, was not applicable to Jaime’s case because the law only became
ability of Great Pacific to estimate the probable risk he presented as subject of life effective in 1985.
insurance. Had he disclosed his visits to his doctor, the diagnosis made and the 21.   On appeal, the CA reversed and set aside the decision of the Insurance
medicines prescribed, in the insurance application, it may be reasonably assumed Commissioner:
that Great Pacific would have made further inquiries and would have probably a.   It found that the use of the word “intentionally” by the Insurance Commissioner in
refused to issue a non-medical insurance policy or at the very least, required a defining and resolving the issue was not supported by evidence;
b.   That the issue agreed upon by the parties had been whether the deceased insured
higher premium. Sec. 27 of the 1978 Insurance Code is properly read as referring to
made a material concealment as to the state of his health at the time of the filing of
any concealment without regard to whether such is intentional or unintentional. the application.
c.   The CA also held that the failure to disclose previous medical consultation and
DOCTRINE: It is immaterial whether the concealment is intentional or treatment constituted material information which should have been communicated
unintentional. to Great Pacific to enable the latter to make proper inquiries.
22.   Hence, this petition by Thelma Canilang.

ISSUE:
FACTS: WoN Jaime Canilang concealed his true state of health at the time of the filing of the
15.   On June 18, 1982, Jaime Canilang consulted Dr. Claudio and was insurance application – YES, because he did not disclose that he had consulted a
diagnosed as suffering from sinus tachycardia. The doctor prescribed doctor prior to the application for the insurance policy, and that he suffered from an
trazepam, a tranquilizer, and aptin, a beta-blocker drug. Canilang consulted illness.
the doctor again after two months after, and he was found to have acute
bronchitis. RULING: WHEREFORE, the Petition for Review is DENIED for lack of merit and the
16.   Jaime Canilang then applied for a non-medical insurance policy with Great Decision of the Court of Appeals dated 16 October 1989 in C.A-G.R. SP No. 08696 is hereby
Pacific Life Assurance Company naming his wife, petitioner Thelma AFFIRMED. No pronouncement as to costs.
Canilang, as his beneficiary. Jaime Canilang was issued an ordinary life
insurance effective as of August 9, 1982. RATIO:
1.   In addition to the negative statements made by Jaime in paragraphs 1 and 2 certain information to the insurer was not intentional in nature, for the
of the medical declaration, he failed to disclose in the appropriate space, reason that Jaime believed that he was suffering from a common cold.
under the caption “Exceptions,” that he had twice consulted Dr. Claudio 10.   The unspoken theory of the Insurance Commissioner appears to have been
who found him to be suffering from sinus tachycardia and acute bronchitis. that by deleting the phrase “intentional or unintentional,” the Insurance
2.   The relevant statutory provisions as they stood at the time are set out in PD Code of 1978 intended to limit the kinds of concealment which generate a
1460 or the Insurance Code of 1978: right to rescind on the part of the injured party to “intentional
a.   Sec. 26. A neglect to communicate that which a party knows and ought to concealments.” This argument is not persuasive. As a simple matter of
communicate, is called a concealment. grammar, it may be noted that “intentional and unintentional” cancel each
b.   Sec. 28. Each party to a contract of insurance must communicate to the other, in
good faith, all factors within his knowledge which are material to the contract and other out. The net result therefore of the phrase is precisely to leave
as to which he makes no warranty, and which the other has not the means of unqualified the term “concealment.”
ascertaining. 11.   Thus, Sec. 272 of the Insurance Code of 1978 is properly read as
3.   Under those provisions, the information concealed must be information referring to any concealment without regard to whether such is
which the concealing party knew and ought to have communicated, that is intentional or unintentional.
to say, information which was material to the contract. The test of 12.   In the case at bar, the nature of the facts not conveyed to the insurer was
materiality is contained in Sec. 31.1 such that the failure to communicate must have been intentional rather than
4.   Sinus tachycardia is considered present when the heart rate exceeds 100 merely inadvertent. For Jaime Canilang could not have been unaware that
beats per minute. The symptoms include pounding in the chest and his heart beat would at times rise to high and alarming levels and that he
sometimes faintness and weakness. had consulted a doctor twice in the two months before applying for the
5.   The Court agrees with the CA that the information which Jaime insurance.
Canilang failed to disclose was material to the ability of Great Pacific to 13.   The Court finds it difficult to take seriously the argument that Great Pacific
estimate the probable risk he presented as subject of life insurance. Had had waived inquiry into the concealment by issuing the insurance policy
he disclosed his visits to his doctor, the diagnosis made and the medicines notwithstanding Jaime’s failure to set out answers to some of the questions
prescribed, in the insurance application, it may be reasonably assumed that in the insurance application. Such failure precisely constituted concealment
Great Pacific would have made further inquiries and would have probably on the part of Jaime.
refused to issue a non-medical insurance policy or at the very least, required
a higher premium. The medical declaration which was set out in the application for insurance executed by Jaime
6.   The materiality of the information withheld by Great Pacific did not depend Canilang read as follows:
upon the state of mind of Jaime. A man’s state of mind is not capable of "MEDICAL DECLARATION `I hereby declare that:
(1) I have not been confined in any hospital, sanitarium or infirmary, nor received any medical or
proof in our judicial process, except through proof of external acts from
surgical advice/attention within the last five (5) years.
which inferences as to his subjective belief may be reasonably drawn. (2) I have never been treated nor consulted a physician for a heart condition, high blood pressure,
7.   Neither does materiality depend upon the actual or physical events which cancer, diabetes, lung, kidney, stomach disorder, or any other physical impairment.
ensue. Materiality relates rather to the “probable and reasonable influence (3) I am, to the best of my knowledge, in good health.
of the fact” upon the party to whom the communication should have been EXCEPTIONS:
made, in assessing the risk involved in making or omitting to make further
inquiries and in accepting the application for insurance.
8.   The insurance Great Pacific applied for was a non-medical insurance. It was GENERAL DECLARATION
I hereby declare that all the foregoing answers and statements are complete, true and correct. I
held that the waiver of medical examination in a non-medical insurance hereby agree that if there be any fraud or misrepresentation in the above statements material to the
contract renders even more material the information required of the risk, the INSURANCE COMPANY upon discovery within two (2) years from the effective
applicant concerning previous condition of health and diseases suffered, for date of insurance shall have the right to declare such insurance null and void. That the
such information constitutes an important factor which the insurer takes into liabilities of the Company under the said Policy/TA/Certificate shall accrue and begin only from the
consideration in deciding whether to issue the policy or not. date of commencement of risk stated in the Policy/TA/Certificate, provided that the first premium is
paid and the Policy/TA/Certificate is delivered to, and accepted by me in person, when I am in
9.   The Insurance Commissioner also ruled that the failure Jaime to convey
actual good health.
                                                                                                                       
1
  Sec.   31.   Materiality   is   to   be   determined   not   by   the   event,   but   solely   by   the   probable   and  
reasonable  influence  of  the  facts  upon  the  party  to  whom  the   communication  is  due,  in  forming  his                                                                                                                          
2
estimate  of  the  disadvantages  of  the  proposed  contract,  or  in  making  his  inquiries.      Sec.  27.  A  concealment  entitles  the  injured  party  to  rescind  a  contract  of  insurance.    
   
007 SUNLIFE v. CA (Marcos) Sunlife.
June 22, 1995 | Quiason, J. | Concealment 138.  He was issued issued Policy No. 3-903-766-X valued P100,000.00, with
double indemnity in case of accidental death
PETITIONER: Sunlife Assurance Company of Canada 139.  The designated beneficiary was his mother, Bernarda Bacani
RESPONDENTS: The Hon. Court of Appeals and Sps. Rolando and Bernarda 140.  Robert died in a plane crash. Subsequently, Bernarda filed a claim with
Bacani Sunlife, seeking the benefits of the insurance policy taken by her son.
141.  Sunlife conducted an investigation and its findings prompted it to reject the
SUMMARY: Robert Bacani procured a life insurance from Sunlife for himself, claim.
making his mother, Bernarda, the beneficiary. Robert died in a plane crash. 142.  In its letter, Sunlife informed Bernarda that Robert did not disclose material
Bernarda filed a claim with Sunlife, seeking the benefits of the insurance policy. facts relevant to the issuance of the policy, thus rendering the contract
Sunlife, however, rejected the claim of Bernarda given that after conducting an voidable. Attached to the letter was a check representing the total premiums
investigation, it found that Robert did not disclose material facts relevant to the paid.
issuance of the policy. Sunlife claimed that Robert gave false statements 143.  Sunlife claimed that the insured gave false statements in his application
regarding his medical history (See Fact 7 for the questions asked). It found that 2 when he answered the ff. questions:
5. Within the past 5 years have you:
weeks prior to his application, he was examined and confined at the Lung Center a) consulted any doctor or other health practitioner? 

for renal failure and was subject to different kinds of tests. Bernarda filed an b) submitted to: 
ECG? X-rays? blood tests? other tests? 

action for specific performance against Sunlife. The RTC and CA ruled in favor c) attended or been admitted to any hospital or other medical facility?
of Bernards contending that the facts concealed where made in good faith and 6. Have you ever had or sought advice for:
under the belief that they need not be disclosed, further the cause of death was xxx xxx xxx
b) urine, kidney or bladder disorder?"
unrelated to the facts concealed by the insured. The issue is WoN there was
concealment or misrepresentation by Robert that would warrant the rejection of
144.  Robert answered 5(a) in the affirmative but limited his answer to a
Bernarda’s claim in the insurance policy – YES, because Section 26 of the
consultation for flu and cough complications, while the other questions
Insurance Code is explicit in requiring a party to a contract of insurance to
were answered in the negative.
communicate to the other, in good faith, all facts within his knowledge which are
145.  However, it was discovered that two weeks prior to his application for
material to the contract and as to which he makes no warranty, and which the
insurance, Robert was examined and confined at the Lung Center of the
other has no means of ascertaining. Materiality is to be determined not by the
Philippines, where he was diagnosed for renal failure. During his
event, but solely by the probable and reasonable influence of the facts upon the
confinement, he was subject to urinalysis, ultra-sonography and hematology
party to whom communication is due, in forming his estimate of the
tests.
disadvantages of the proposed contract or in making his inquiries. The matters
146.  Bernarda and her husband filed an action for specific performance against
concealed would have definitely affected Sunlife’s action on his application,
Sunlife with the RTC of Valenzuela.
either by approving it with the corresponding adjustment for a higher premium
147.  Sunflife filed its answer with counterclaim and a list of exibits consisting of
or rejecting the same. "Good faith" is no defense in concealment. The insured's
medical records furnished by Lung Center.
failure to disclose the fact that he was hospitalized for two weeks prior to ling his
148.  The RTC ruled in favor of Bernarda and concluded that the facts concealed
application for insurance, raises grave doubts about his bonafides. It appears that
were made in good faith and under the belief that they need not be
such concealment was deliberate on his part.
disclosed. Moreover, it held that the health history of the insured was
immaterial since the insurance policy was “non-medical”.
DOCTRINE: Section 26 of The Insurance Code is explicit in requiring a party
149.  An appeal was made to the CA, which affirmed the decision of the RTC.
to a contract of insurance to communicate to the other, in good faith, all facts
150.  The CA held that the cause of death was unrelated to the facts concealed by
within his knowledge which are material to the contract and as to which he
the insured.
makes no warranty, and which the other has no means of ascertaining. Said
151.  Sunlife’s MR was denied, hence a petition was made to the SC.
Section provides: "A neglect to communicate that which a party knows and
ought to communicate, is called concealment."
ISSUE/s: WoN there was concealment or misrepresentation by Robert that would
warrant the rejection of Bernarda’s claim in the insurance policy – YES, because
Section 26 of the Insurance Code is explicit in requiring a party to a contract of
FACTS: insurance to communicate to the other, in good faith, all facts within his knowledge
137.  Robert John B. Bacani procured a life insurance contract for himself from which are material to the contract and as to which he makes no warranty, and which
the other has no means of ascertaining

RULING: Judgment in question is affirmed.

RATIO:
217.  Section 26 of the Insurance Code is explicit in requiring a party to a
contract of insurance to communicate to the other, in good faith, all facts
within his knowledge which are material to the contract and as to which he
makes no warranty, and which the other has no means of ascertaining.
218.  Said Section provides: "A neglect to communicate that which a party knows
and ought to communicate, is called concealment."
219.  Materiality is to be determined not by the event, but solely by the probable
and reasonable influence of the facts upon the party to whom
communication is due, in forming his estimate of the disadvantages of the
proposed contract or in making his inquiries.
220.  The terms of the contract are clear. The insured is specifically required to
disclose to the insurer matters relating to his health.
221.  The information which the insured failed to disclose were material and
relevant to the approval and the issuance of the insurance policy.
222.  The matters concealed would have definitely affected Sunlife’s action on
his application, either by approving it with the corresponding adjustment for
a higher premium or rejecting the same.
223.  Moreover, a disclosure may have warranted a medical examination of the
insured by Sunlife in order for it to reasonably assess the risk involved in
accepting the application.
224.  "Good faith" is no defense in concealment.
225.  The insured's failure to disclose the fact that he was hospitalized for two
weeks prior to ling his application for insurance, raises grave doubts about
his bonafides. It appears that such concealment was deliberate on his part.
226.  The argument, that Sunlife's waiver of the medical examination of the
insured debunks the materiality of the facts concealed, is untenable.
227.  We reiterate our ruling in Saturnino v. Philippine American Life Insurance
Company, that ". . . the waiver of a medical examination [in a non-medical
insurance contract] renders even more material the information required of
the applicant concerning previous condition of health and diseases suffered,
for such information necessarily constitutes an important factor which the
insurer takes into consideration in deciding whether to issue the policy or
not . . . ."
228.  Anent the finding that the facts concealed had no bearing to the cause of
death of the insured, it is well settled that the insured need not die of the
disease he had failed to disclose to the insurer. It is sufficient that his non-
disclosure misled the insurer in forming his estimates of the risks of the
proposed insurance policy or in making inquiries.
008 Saturnino v. Philam (MATSUMURA) detailed information regarding the applicant’s health and medical
February 28, 1963 | Makalintal, J. | Concealment history is needed.
2.   The policy was issued on the same day upon Saturnino’s payment of the
first year’s premium of P339.25.
PETITIONER: Ignacio Saturnino, in his own behalf and as the judicial guardian of
Carlos Saturnino, minor 3.   On September 19, 1958, Saturnino died of pneumonia, secondly to
RESPONDENTS: The Philippine American Life Insurance Policy influenza.
4.   Estefania’s husband, Ignacio Saturnino, and their minor child Carlos
SUMMARY: Estefania Saturnino (saturnino) applied for a non-medical insurance Saturnino demanded payment of the face value of the policy which is
with Philam. Here, Saturnino was asked to detail her medical history wherein she P5,000.
stated that she never had any of the ailments listen in the application, cancer, or other
tumors. Moreover, that she had not consulted any physician, undergone any operation 5.   It appears that the months prior to the issuance of the policy,
or suffered any injury within the last five years. The insurance policy was issued after Saturnino was operated on for cancer, involving complete removal of
Saturnino paid the first year’s premium. Less than a year later, Saturnino died of the right breast, including the pectoral muscles and the glands found in
pneumonia secondly to influenza. Her husband and child claimed from Philam the the right armpit.
insurance proceeds of P5,000 but was denied. Apparently, months before the issuance
6.   She stayed in the hospital for a period of eight days, after which she was
of the policy, Saturnino was operated on for cancer (her right break was removed) and
discharged, although according to the surgeon who operated on her she
that the doctor said she could not be considered definitely cured since her cancer was
could not be considered definitely cured, her ailment being of the
malignant. Thus, the saturnino family filed this case to recover the insurance.
malignant type.
The SC rueld that first, Saturnino did make false representation of material facts which 7.   Notwithstanding the fact of her operation Saturnino did not disclose this
would avoid the policy. While the Saturnino family argue that the misrepresentation information in her application for insurance.
was not material because the insurance was “non-medical”, the court ruled that the
8.   On the contrary, she stated therein that she did not have, nor had she ever
waiver of the medical examination renders even more material the meedical
had, among other ailments listed in the application, cancer or other
information that was asked from Saturnino. Second, the argument that Philam’s agent,
tumors; that she had not consulted any physician, undergone any
Santos, was aware of Saturnino’s ailment was rejected because it was already settled in
operation or suffered any injury within the preceding five years; and that
the trial court. Such being a question of fact will not be disrupted by the court since
she had never been treated for, nor did she ever have any illness or
what is assailed are pure questions of law. Third, as to the argument that Saturnino was
disease peculiar to her sex, particularly of the breast, ovaries, uterus, and
not aware that she had cancer (that the doctor allegedly didn’t tell her it was cancer),
menstrual disorders.
the court ruled that concealment of the fact of the operation itself was fraudulent.
Moreover, that in avoiding a policy, it is not necessary to show actual fraud. In this 9.   The application also recites that the foregoing declarations constituted "a
jurisdiction, a concealment, whether intentional or unintentional, entitles the insurer to further basis for the issuance of the policy."
rescind the contract of insurance, concealment being defined as "negligence to
10.   Thus, the insurance claim was rejected by Philam., and so the Saturnino
communicate that which a party knows and ought to communicate"
family filed an action to recover the P5,000.
DOCTRINE: A concealment, whether intentional or unintentional, entitles the insurer 11.   The CFI of Manila dismissed the case, but the Saturnino family were
to rescind the contract of insurance, concealment being defined as "negligence to declared to be entitled to the return of the premium already paid plus
communicate that which a party knows and ought to communicate. interest.
12.   Hence, this appeal.
FACTS
1.   Estefania Saturnino (Saturnino) applied for a non-medical insurance with ISSUE/S:
Philippine American Life Insurance Policy (Philam) on November 16, 1.   W/N Saturnino made false representations of material facts which would
1957, witnessed by Edward Santos (Philam’s agent) avoid the policy – YES because Saturnino misrepresented herself as
a.   This kind of policy dispenses with the medical examination of someone who’s never had any cancel or undergone any operation within the
the applicant usually required in ordinary life policies. However, a past 5 years when in fact she did.
9.   In the application for insurance signed by the Saturnino in this case, she
RULING: The judgment appealed from, dismissing the complaint and awarding the agreed to submit to a medical examination by a duly appointed examiner
return to appellants of the premium already paid, with interest at 6% up to January of Philam if in the latter's opinion such examination was necessary as
29, 1959, is affirmed, with costs against appellants. further evidence of insurability. In not asking her to submit to a medical
examination, the Saturnino family argue that Philam was negligent which
RATIO: precluded it from finding about her actual state of health.
1.   There can be no dispute that the information given by Saturnino in 10.   The SC rueld thst No such negligence can be imputed to Philam. It was
her application for insurance was false, namely, that she had never had precisely because the insured had given herself a clean bill of health that
cancer or tumors, or consulted any physician or undergone any operation Philam no longer considered an actual medical checkup necessary.
within the preceding period of five years.
11.   The Saturnino family also contend there was no fraudulent
2.   Are the facts thus falsely represented material? Section 30 of The concealment of the truth as much as Saturnino herself did not know,
Insurance Law provides that "materiality is to be determined not by the since her doctor never told her, that the disease for which she had
event, but solely by the probable and reasonable influence of informing been operated on was cancer.
his estimate of the proposed contract, or in facts upon the party to whom
12.   In the first place the concealment of the fact of the operation itself was
the communication is due, making his inquiries."
fraudulent, as there could not have been any mistake about it, no matter
3.   The Saturnino family argue that the facts subject of the representations what the ailment.
was not material in view of the "non-medical" nature of the insurance
13.   Secondly, in order to avoid a policy, it is not necessary to show actual
applied for, which does away with the usual requirement of medical
fraud on the part of the insured.
examination before the policy is issued.
14.   According to Kasprzyk vs. Metropolitan Insurance Co.,: "Moreover, if it
4.   However, such is without merit. If anything, the waiver of medical
were the law that an insurance company could not defend a policy on the
examination renders even more material the information required of
ground of misrepresentation, unless it could show actual knowledge on
the applicant concerning previous condition of health and diseases
the part of the applicant that the statements were false, then it is plain
suffered, for such information necessarily constitutes an important
that it would be impossible for it to protect itself and its honest
factor which the insurer takes into consideration in deciding whether
policyholders against fraudulent and improper claims. It would be
to issue the policy or not.
wholly at the mercy of any one who wished to apply for insurance, as it
5.   It is logical to assume that if Philam had been properly apprised of the would be impossible to show actual fraud except in the extremest cases.
insured's medical history Saturnino would at least have been made to It could not rely on an application as containing information on which it
undergo medical examination in order to determine her insurability. could act. There would be no incentive to an applicant to tell the truth."
6.   The Saturnino family also argue that due information concerning 15.   In this jurisdiction, a concealment, whether intentional or
Saturnino's previous illness and operation had been given Philam’s unintentional, entitles the insurer to rescind the contract of
agenct, Edward Santos, who filed the application form after it was signed insurance, concealment being defined as "negligence to communicate
in blank by Saturnino. that which a party knows and ought to communicate"
7.   This was denied by Santos in his testimony, and the trial court found such 16.   In Argente vs. West Coast Life Insurance Co, the court said: "'The basis
testimony to be true. This is a finding of fact which is binding upon the of the rule vitiating the contract in cases of concealment is that it misleads
SC, this appeal having been taken upon questions of law alone. or deceives the insurer into accepting the risk, or accepting it at the rate of
8.   The SC did not deem it necessary to consider Philam's additional premium agreed upon. The insurer, relying upon the belief that the
argument, which was upheld by the trial court, that in signing the assured will disclose every material fact within his actual or presumed
application form in blank and leaving it to Santos to fill (assuming that to knowledge, is misled into a belief that the circumstance withheld does not
be the truth) the insured in effect made Santos her agent for that purpose exist, and he is thereby induced to estimate the risk upon a false basis that
and consequently was responsible for the errors in the entries made by it does not exist.'"
him in that capacity.
009 FLORENDO v. PHILAM PLANS (MERILLES) 3.   Ma. Celeste Abcede, Perla’s duaghter, signed tha application as sales
February 22, 2012 | Abad, J. | Concealment counselor
4.   Aside from pension benefits, the comprehensive pension plan also provided
life insurance coverage to Florendo
PETITIONER: Ma. Lourdes S. Florendo
5.   This was covered by a Group Master Policy that Philippine American Life
RESPONDENTS: Philam Plans Inc., Perla Abcede. and Ma. Celeste Abcede
Insurance Company (Philam Life) issued to Philam Plans
a.   Under the master policy, Philam Life was to automatically provide
SUMMARY: Manuel filed an application for a comprehensive pension plan
life insurance coverage, including accidental death, to all who
with Philam Plans through Perla Abcede. Manuel signed the forms and left to
signed up for Philam Plans' comprehensive pension plan
Perla the accomplishment the contents. The pension plan had with it a life
b.   If the plan holder died before the maturity of the plan, his
insurance coverage. Philam Plans subsequently issued a Pension Plan
beneficiary was to instead receive the proceeds of the life
Agreement to Manuel. Manuel paid his quarterly premiums.
insurance, equivalent to the pre-need price. F
c.   urther, the life insurance was to take care of any unpaid premium
11 months after acquiring the pension, Manuel died. His beneficiary, Lourdes
until the pension plan matured, entitling the beneficiary to the
Florendo (his wife), claims for the payment of the benefits but was denied.
maturity value of the pension plan
Philam Plans alleges that Manuel concealed material facts as to his health when
6.   Philam Plans issued Pension Plan Agreement PP 43005584 to Manuel, with
he applied for the pension plans, therefore cannot recover from the insurer.
Ma. Lourdes S. Florendo, his wife, as beneficiary. Manuel paid his
quarterly premiums
Lourdes field an action against Philam plans. The RTC ruled that Manuel was
7.   11 months later, Manuel died of blood poisoning.
not guilty of concealment. The CA reversed saying that nsurance policies are
8.   Lourdes filed a claim with Philam Plans for the payment of the benefits
traditionally contracts uberrimae fidae or contracts of utmost good faith. As
under her husband’s plan.
such, it required Manuel to disclose to Philam Plans conditions affecting the risk
9.   Because Manuel died before his pension plan matured and his wife was to
of which he was aware or material facts that he knew or ought to know.
get only the benefits of his life insurance, Philam Plans forwarded her claim
to Philam Life
The issue before the SC is whether Manuel is guilty of concealing his illness in
10.   On May 3, 1999, Philam Plans wrote Lourdes a letter, declining her claim.
his application forms. The Court ruled that YES, he is guilty. When Manuel
a.   Philam Life found that Manuel was on maintenance medicine for
signed the pension plan application, he adopted as his own the written
his heart and had an implanted pacemaker.
representations and declarations embodied in it. Assuming that it was Perla
b.   Further, he suffered from diabetes mellitus and was taking insulin.
who filled up the application form, Manuel is still bound by what it
11.   Lourdes renewed her demand for payment under the plan
contains since he certified that he authorized her action. Philam Plans had
but Philam Plansrejected it, prompting her to file the present action against
every right to act on the faith of that certification
the pension plan company before the Regional Trial Court (RTC) of
Quezon City
DOCTRINE: (repetition of above) When Manuel signed the pension plan
12.   RTC rendered judgment ordering Philam PLans, Perla and Celeste
application, he adopted as his own the written representations and declarations
solidarity to pay lourdes all the benefits form her husband’s pension plan.
embodied in it. Assuming that it was Perla who filled up the application form,
a.   RTC ruled that Manuel was not guilty of concealing the state of his
Manuel is still bound by what it contains since he certified that he authorized
health from his pension plan application
her action. Philam Plans had every right to act on the faith of that certification
13.   CA reversed the RTC decision holding that insurance policies are
traditionally contracts uberrimae fidae or contracts of utmost good faith. As
such, it required Manuel to disclose to Philam Plans conditions affecting the
FACTS: risk of which he was aware or material facts that he knew or ought to know
1.   Manuel Florendo filed an application for comprehensive pension plan with 14.   Hence, this case.
respondent Philam Plans, Inc. (Philam Plans) after some convincing by ISSUE/s:
respondent Perla Abcede 1.   Whether the CA erred in finding Manuel guilty of concealing his illness
a.   The plan had a pre-need price of P997,050.00, payable in 10 years, when he kept blank and did not answer questions ins pension plan
and had a maturity value of P2,890,000.00 after 20 years application regarding the ailments he suffered - YES. In signing the forms
2.   Manuel signed the application and left to Perla the task of supplying the he attests to the completeness and truthfulness of its contents.
information needed in the application.
5.   Lourdes next points out that it made no difference if Manuel failed to reveal
RULING: WHEREFORE, the Court AFFIRMS in its entirety the decision of the the fact that he had a pacemaker implant in the early 70s since this did not
Court of Appeals in CA-G.R. CV 87085 dated December 18, 2007 fall within the five-year timeframe that the disclosure contemplated
a.   But a pacemaker is an electronic device implanted into the body
RATIO: and connected to the wall of the heart, designed to provide regular,
1.   Lourdes points out that, seeing the unfilled spaces in Manuel's pension plan mild, electric shock that stimulates the contraction of the heart
application relating to his medical history, Philam Plans should have muscles and restores normalcy to the heartbeat
returned it to him for completion. 6.   Assuming that it was Perla who filled up the application form, Manuel
a.   Since Philam Planschose to approve the application just as it was, is still bound by what it contains since he certified that he authorized
it cannot cry concealment on Manuel's part. her action. Philam Plans had every right to act on the faith of that
b.   Further, Lourdes adds that Philam Plans never queried Manuel certification
directly regarding the state of his health. Consequently, it could not 7.   In a final attempt to defend her claim for benefits under Manuel's pension
blame him for not mentioning it plan, Lourdes points out that any defect or insufficiency in the information
2.   However, the Court says that Lourdes forgets that provided by his pension plan application should be deemed waived after the
since Philam Plans waived medical examination for Manuel, it had to rely same has been approved, the policy has been issued, and the premiums have
largely on his stating the truth regarding his health in his application. been collected
a.   For, after all, he knew more than anyone that he had been under a.   The Court cannot agree. The comprehensive pension plan
treatment for heart condition and diabetes for more than five years that Philam Plans issued contains a one-year incontestability period
preceding his submission of that application. b.   The incontestability clause precludes the insurer from disowning
b.   But he kept those crucial facts from Philam Plans liability under the policy it issued on the ground of concealment or
c.   Besides, when Manuel signed the pension plan application, he misrepresentation regarding the health of the insured after a year of
adopted as his own the written representations and its issuance
declarations embodied in it. 8.   Since Manuel died on the eleventh month following the issuance of his
d.   It is clear from these representations that he concealed his chronic plan, the one year incontestability period has not yet set in.
heart ailment and diabetes from PhilamPlans Consequently, Philam Plans was not barred from questioning Lourdes'
3.   Since Manuel signed the application without filling in the details regarding entitlement to the benefits of her husband's pension plan.
his continuing treatments for heart condition and diabetes, the assumption is
that he has never been treated for the said illnesses in the last five years
preceding his application.
a.   This is implicit from the phrase "If your answer to any of the
statements above (specifically, the statement: I have never been
treated for heart condition or diabetes) reveal otherwise, please
give details in the space provided for."
b.   But this is untrue since he had been on "Coumadin," a treatment
for venous thrombosis, and insulin, a drug used in the treatment of
diabetes mellitus, at that time.
4.   Lourdes insists that Manuel had concealed nothing since Perla, the
soliciting agent, knew that Manuel had a pacemaker implanted on his chest
in the 70s or about 20 years before he signed up for the pension plan
a.   Nothing in it implies that someone else may provide the
information that Philam Plans needed.
b.   Manuel cannot sign the application and disown the
responsibility for having it filled up.
c.   If he furnished Perla the needed information and delegated to
her the filling up of the application, then she acted on his
instruction, not on Philam Plans' instruction
010 NG GAN ZEE V ASIAN CRUSADER LIFE INSURANCE (ARMAND) FACTS:
May 30, 1983 | Escolin, J. | Concealment 134.  On May 12, 1962, Kwong Nam applied for a 20-year endowment insurance
PETITIONER: Ng Gan Zee on his life for the sum of P20,000.00, with his wife, appellee Ng Gan Zee as
RESPONDENTS: Asian Crusader Life Assurance Corporation beneficiary. On the same date, Asian Crusader (Asian), upon receipt of the
SUMMARY: In May 1962, Kwong Nam applied for a 20-year policy with Asian required premium from the insured, approved the application and issued the
Crusader Life Assurance Corporation. Asian Crusader asked the following question: corresponding policy. On December 6, 1963, Kwong Nam died of cancer
Has any life insurance company ever refused your application for insurance or for of the liver with metastasis. All premiums had been religiously paid at the
reinstatement of a lapsed policy or offered you a policy different from that applied time of his death.
for? If, so, name company and date. – to which Kwong Nam answered “No”. Kwong 135.  On January 10, 1964, his widow Ng Gan Zee presented a claim in due form
Nam was also examined by Asian Crusader’s medical examiner to whom he to the Asian for payment of the face value of the policy. On the same date,
disclosed that he was once operated and a tumor was removed from his stomach and she submitted the required proof of death of the insured. Asian denied the
such was “associated with ulcer of the stomach.” Kwong Nam’s application was claim on the ground that the answers given by the insured to the questions
approved. In May 1963, he died. His widow, Ng Gan Zee, filed an insurance claim appealing in his application for life insurance were untrue.
but Asian Crusader refused her claim as it insisted that Kwong Nam concealed 136.  Ng Gan Zee brought the matter to the attention of the Insurance
material facts from them when he was applying for the insurance; that he Commissioner, the Hon. Francisco Y. Mandamus, and the latter, after
misrepresented the fact that he was actually denied application by Insular Life when conducting an investigation, wrote that Asian that he had found no material
he was renewing his application with them; that Kwong Nam was actually operated concealment on the part of the insured and that, therefore, Ng Gan Zee
for peptic ulcer. The issue is WoN because of the aforementioned representation, should be paid the full face value of the policy. This opinion of the
the insured has misled, or deceived the insurer in entering the contract or in Insurance Commissioner notwithstanding, appellant refused to settle its
accepting the risk at the rate of premium agreed upon – NO. The representation obligation.
was not material, nor fraudulent, and there was no proof to show that it was 137.  Asian alleged that the insured was guilty of misrepresentation when he
intentionally withheld. Asian Crusader was not able to prove that Kwong Nam’s answeres “No” to the following question: “Has any life insurance company
statement that Insular Life did not deny his insurance renewal with them is untrue. In ever refused your application for insurance or for reinstatement of a lapsed
fact, evidence showed that in April 1962, Insular Life approved Kwong Nam’s policy or offered you a policy different from that applied for? If, so, name
request of reinstatement only with the condition that Kwong Nam’s plan will be company and date.”
lowered from P50,000.00 to P20,000.00 considering his medical history. Kwong 138.  In its brief, Ng Gan Zee stated: “... As pointed out in the foregoing
Nam did not conceal anything from Asian Crusader. His statement that his operation, summary of the essential facts in this case, the insured had in January, 1962,
in which a tumor the size of a hen’s egg was removed from his stomach, was only applied for reinstatement of his lapsed life insurance policy with the Insular
“associated with ulcer of the stomach” and not peptic ulcer can be considered as an Life Insurance Co., Ltd, but this was declined by the insurance company,
expression made in good faith of his belief as to the nature of his ailment and although later on approved for reinstatement with a very high premium as a
operation. Indeed, such statement must be presumed to have been made by him result of his medical examination. Thus notwithstanding the said insured
without knowledge of its incorrectness and without any deliberate intent on his part to answered 'No' to the [above] question propounded to him.”
mislead Asian Crusader.While it may be conceded that, from the viewpoint of a 139.  Lower court found the argument bereft of factual basis, to wit: “On the first
medical expert, the information communicated was imperfect, the same was question there is no evidence that the Insular Life Assurance Co., Ltd. ever
nevertheless sufficient to have induced Asian Crusader to make further inquiries refused any application of Kwong Nam for insurance. Neither is there any
about the ailment and operation of Kwong Nam. It has been held that where, upon the evidence that any other insurance company has refused any application of
face of the application, a question appears to be not answered at all or to be Kwong Nam for insurance… No new policy was issued by the Insular Life
imperfectly answered, and the insurers issue a policy without any further inquiry, Assurance Co., Ltd. to Kwong Nam in connection with said application for
they waive the imperfection of the answer and render the omission to answer more reinstatement and amendment. Such being the case, the Court finds that
fully immaterial. there is no misrepresentation on this matter.”
DOCTRINE: The duty to establish such affirmative defense of fraudulent intent by 140.  Asian further maintains that when the insured was examined in connection
satisfactory and convincing evidence rests upon the defendant. with his application for life insurance, he gave the appellant's medical
It has been held that where, upon the face of the application, a question appears to be examiner false and misleading information as to his ailment and previous
not answered at all or to be imperfectly answered, and the insurers issue a policy operation. The alleged false statements given by Kwong Nam was that that
without any further inquiry, they waive the imperfection of the answer and render the he was operated on for a tumor in the stomach and that it was associated
omission to answer more fully immaterial.
with ulcer of the stomach, operated on at Chinese General Hospital 2 years with ulcer of the stomach, " should be construed as an expression made in
ago. good faith of his belief as to the nature of his ailment and operation. Indeed,
141.  Asian alleges that the physician who treated Kwong Nam at the Chinese such statement must be presumed to have been made by him without
Gen Hospital had diagnosed the patient’s ailment as peptic ulcer. Asian knowledge of its incorrectness and without any deliberate intent on his part
argues that the insured’s statement in his application that a tumor was to mislead the insurer.
removed during said operation constituted material concealment. (So 134.  While it may be conceded that, from the viewpoint of a medical expert, the
basically a discrepancy between “peptic ulcer” and “tumor of hen’s egg information communicated was imperfect, the same was nevertheless
size” sufficient to have induced insurer to make further inquiries about the
ailment and operation of the insured.
ISSUE/s: 135.  Section 32 of the Insurance Law provides:
13.   Whether or not because of the aforementioned representation, the Section 32. The right to information of material facts maybe waived either
insured has misled, or deceived the insurer in entering the contract or by the terms of insurance or by neglect to make inquiries as to such facts
in accepting the risk at the rate of premium agreed upon – NO. The where they are distinctly implied in other facts of which information is
representation was not material, nor fraudulent, and there was no proof to communicated.
show that it was intentionally withheld. 136.  It has been held that where, upon the face of the application, a question
appears to be not answered at all or to be imperfectly answered, and
RULING: Finding no reversible error committed by the trial court, the judgment the insurers issue a policy without any further inquiry, they waive the
appealed from is hereby affirmed, with costs against appellant Asian-Crusader life imperfection of the answer and render the omission to answer more
Assurance Corporation fully immaterial.
137.  As aptly noted by the lower court, "if the ailment and operation of Kwong
RATIO: Nam had such an important bearing on the question of whether the
130.  Section 27 of the Insurance Law (Act 2427) provides: defendant would undertake the insurance or not, the court cannot
Sec. 27. Such party a contract of insurance must communicate to the other, understand why the defendant or its medical examiner did not make any
in good faith, all facts within his knowledge which are material to the further inquiries on such matters from the Chinese General Hospital or
contract, and which the other has not the means of ascertaining, and as to require copies of the hospital records from the appellant before acting on the
which he makes no warranty. application for insurance. The fact of the matter is that the defendant was
131.  Thus, "concealment exists where the assured had knowledge of a fact too eager to accept the application and receive the insured's premium. It
material to the risk, and honesty, good faith, and fair dealing requires that would be inequitable now to allow the defendant to avoid liability under the
he should communicate it to the assurer, but he designedly and intentionally circumstances."
withholds the same." It has also been held "that the concealment must, in
the absence of inquiries, be not only material, but fraudulent, or the
fact must have been intentionally withheld."
132.  Assuming that the aforesaid answer given by the insured is false, as
claimed by the appellant. Sec. 27 of the Insurance Law, above-quoted,
nevertheless requires that fraudulent intent on the part of the insured
be established to entitle the insurer to rescind the contract. And as
correctly observed by the lower court, "misrepresentation as a defense of
the insurer to avoid liability is an 'affirmative' defense. The duty to
establish such a defense by satisfactory and convincing evidence rests
upon the defendant. The evidence before the Court does not clearly and
satisfactorily establish that defense."
133.  It bears emphasis that Kwong Nam had informed the insurer’s medical
examiner that the tumor for which he was operated on was "associated with
ulcer of the stomach." In the absence of evidence that the insured had
sufficient medical knowledge as to enable him to distinguish between
"peptic ulcer" and "a tumor", his statement that said tumor was "associated
011 FIELDMEN’S INSURANCE v. VDA. DE SONGCO (PELIÑO) a.   This time, the jeep had a different Plate No: J-68136
September 23, 1968 | Fernando, J. | Misrepresentation35 3.   On October 29, 1961 (policy was still effective), the jeep was being driven
by Rodolfo, son of Federico and a duly licensed driver.
PETITIONER: Fieldmen’s Insurance Co. a.   The jeep collided with a car in Calumpit, Bulacan, and as a result,
RESPONDENTS: Mercedes Vargas Vda. De Songco, et. al. and Court of Appeals Federico and Rodolfo died, while Carlos (another son), Angelita (wife
of Carlos), and a family friend sustained physical injuries.
SUMMARY: Federico, a person who only reached grade 1, owned a private jeepney. He 4.   The injured parties or the heirs wanted to claim from Fieldmen’s, but
was induced by Fieldmen’s agent, Sambat, to apply for a Common Carrier’s Liability Fieldmen’s denied, saying that what was insured was a private vehicle and
Insurance Policy to cover his motor vehicle. This was insured twice under the same
not a common carrier.
policy. While the policy was still effective, the jeep collided with a car, resulting to the
death of Federico and a son, and injuries to the other passengers who were members of 5.   During trial in the CFI, another son of Federico, Amor, testified that when
his family. The heirs wanted to claim on the insurance policy but Fieldmen’s said that Sambat was inducing Federico to insure the vehicle, he butted in saying that
they can’t since what was insured was a private vehicle and not a common carrier. When they can’t insure the vehicle since it is an “owner” private vehicle, but then
the heirs filed in the CFI, the surviving son, Amor, testified that Sambat induced Sambat said that whether the vehicle was an “owner” type or for
Federico to insure the vehicle despite knowing that the vehicle wasn’t a common carrier. passengers, it could be insured because their company is not owned by the
Lower court ruled in favor of the heirs. On appeal to the CA, the CA also affirmed the government, so they could do as they please whenever they believe that the
lower court. Hence, this petition. The issue in this case is whether or not Fieldmen’s is vehicle is insurable.
liable on the policy despite the fact that the vehicle insured is a private one and not a a.   Fieldmen’s didn’t rebut the testimony of Amor.
common carrier. The SC held in the affirmative. Where inequitable conduct is shown by
b.   Lower court ruled in their favor. Fieldmen’s appealed to the CA, and
an insurance firm, it is estopped from enforcing forfeitures in its favor in order to
forestall fraud or imposition on the insured. After Fieldmen’s led Federico to believe that
the also affirmed the lower court.
he could qualify under the common carrier liability insurance, it could not be permitted 6.   Hence, this case.
to change its stand to the detriment of the heirs of the insured. Estoppel is primarily 7.   Fieldmen’s claims that:
based on the doctrine of good faith and the avoidance of harm that will befall the a.   Estoppel cannot be invoked by the heirs as a bar to the alleged breach
innocent party due to its injurious reliance. A contract of insurance is one of perfect good of warranty and condition in the policy
faith not for the insured alone, but equally so for the insurer; in fact, it is more so for the b.   No legal liability was incurred under the policy by Fieldmen’s.
latter, since its dominant bargaining position carries with it stricter responsibility.
ISSUE/s:
DOCTRINE: Where inequitable conduct is shown by an insurance firm, it is estopped
1.   WON Fieldmen’s is liable on the policy despite the fact that the vehicle is a
from enforcing forfeitures in its favor in order to forestall fraud or imposition on the
insured. Contractual duty of disclosure imposed by utmost good faith is not required of private one and not a common carrier. – YES, after Fieldmen’s led Federico
the insured alone, but it is imposed with equal stringency upon the insurer, moreso upon to believe that he could qualify under the common carrier liability
the latter since his dominant bargaining position carries with it stricter responsibility. insurance, it could not be permitted to change its stand to the detriment of
  the heirs of the insured.
FACTS:
1.   Federico Songco (Federico) was a man of scant education, since he only RULING: WHEREFORE, the decision of the CA is AFFIRMED in its entirety.
reached first grade.
a.   He owned a private jeepney with Plate No. 41-289 for the year 1960. RATIO:
b.   On September 15, 1960, he was induced by Fieldmen’s Insurance On whether Fieldmen’s is liable on the policy
Co. (Fieldmen’s) Pampanga agent, Benjamin Sambat (Sambat) to 1.   Where inequitable conduct is shown by an insurance firm, it is
apply for a Common Carrier’s Liability Insurance Policy covering estopped from enforcing forfeitures in its favor in order to forestall
his motor vehicle upon paying annual premium of P16.50. fraud or imposition on the insured.
c.   4 days after, Fieldmen’s issued the Common Carriers Accident 2.   After Fieldmen’s led Federico to believe that he could qualify under the
Insurance Policy which will be effective for 1 year (Sept. 15, 1960- common carrier liability insurance, it could not be permitted to change its
Sept. 15, 1961). stand to the detriment of the heirs of the insured.
2.   On September 22, 1961, Fieldmen’s renewed the policy upon payment of 3.   Estoppel is primarily based on the doctrine of good faith and the
the premium, this time effective from October 15, 1961-October 15, 1962. avoidance of harm that will befall the innocent party due to its
injurious reliance. The failure to apply in this case would result in a gross
                                                                                                                        travesty of justice.
35
The case never mentioned the words concealment or misrepresentation.
4.   As to the issue on breach, CA said (which the SC also agreed with) that,
some of the conditions contained in the policy were impossible to comply
with under the existing conditions at the time and inconsistent with the
known facts, so the insurer (Fieldmen’s) is estopped from asserting
breach of such conditions.
5.   Even it be assumed that there was an ambiguity, based on Qua Chee Gan v.
Law Union and Rock Insurance, taking into account the well-known rule
that ambiguities or obscurities must be strictly interpreted against the party
that caused them.
a.   Nowadays, monopolies, cartels, those with overwhelming economic
power, manage to impose upon parties dealing with them contracts of
adhesion, in contrast to those entered by parties bargaining on equal
footing. So these kinds of contracts call for greater strictness and
vigilance on the part of courts of justice with a view to protecting the
weaker party.
6.   Contract of insurance is one of perfect good faith (uberima fides);
contractual duty of disclosure imposed by utmost good faith is not for
the insured alone, but equally so for the insurer; in fact, it is more so
for the latter, since its dominant bargaining position carries with it
stricter responsibility.
012 YU PANG CHENG v. CA (PLEYTO)
May 29, 1959 | Bautista Angelo, J. | Misrepresentation FACTS:
1.   On Sept. 5, 1950, Yu Pand Eng submitted parts II and III of his application
PETITIONER: Yu Pang Cheng alias Yu Pang Ching for insurance consisting of the medical declaration made by him to the
RESPONDENTS: The Court of Appeals, et. al. medical examiner of defendant (SORRY, HINDI TALAGA NAKALAGAY SA
CASE SINO SIYA so not sure who kasi may “et. al” so defendant-insurer
SUMMARY: Yu Pand Eng submitted his application for insurance of Sept. 1950. na lang sa next parts), and the medical examiner’s report.
He was issued a policy 3 days after. He entered St. Lukes on Dec. 29, 1950 but he 2.   On Sept. 7, he submitted part I of his application which is the declaration
eventually died of “infiltrating medullary carcinoma, Grade 4, advanced catdiac and made by him to an agent of defendant-insurer
lesser curvature, stomach metastases spleen” on Feb. 1951. Cheng, brother and 3.   Sept. 8: based on said application, and upon payment of the first premium in
beneficiary of Yu Pand Eng, demanded from the defendant-insurer the proceeds of the sum of P591.70, defendant-insurer issued to the insured Policy No.
the insurance but it was denied. Hence, this petition. Defendant-insurer in its answer, 812858
set up the defense that Yu Pand Eng was guilty of misrepresentation and 4.   Dec. 27, 1950: Yu Pand Eng entered St. Luke’s Hospital for medical
concealment of material facts in that he gave false and untruthful answers to certain treatment but he died on Feb. 27, 1951. According to the death certificate,
questions asked him in his application for insurance which were material to the risk he died of “infiltrating medullary carcinoma, Grade 4, advanced catdiac and
against and have the effect of avoiding the insurance policy. Trial court ruled in lesser curvature, stomach metastases spleen.”
favor of Cheng but this was reversed by the CA. The issue in this case is WoN the 5.   Yu Pang Cheng (Cheng), brother and beneficiary of the insured, demanded
insured is guilty of concealment of some facts material to the risk insured against from defendant-insurer the payment of the proceeds of the insurance polic.
which has the effect of avoiding the policy as found by the CA. It appears that Yu 6.   Defendant-insurer refused. Thus, Cheng brought this action to collect from
Pand Eng entered the Chinese General Hospital for medical treatment on January 29, the defendant-insurer the sum of P10,000, value of an insurance policy
1950 having stayed there up to February 11, 1950. Upon entering the hospital, he taken upon the life of one Yu Pand Eng, plus interest thereon at the legal
complained of dizziness, anemia, abdominal paids and tarry stools, and in the rate, the sum of P10,000 as moral damages, P3000 as attorney’s fees, and
evening of his admission he had several abdominal pains and his discharges were the costs of action
with black tarry stools and felt dizzy and weak. Yu Pand Eng's confinement in the 7.   Defendant-insurer in its answer, set up the defense that Yu Pand Eng was
Chinese General Hospital took place from January 29, 1950 to February 11, 1950, guilty of misrepresentation and concealment of material facts in that he
whereas his application for insurance wherein he stated his answers to the questions gave false and untruthful answers to certain questions asked him in his
propounded to him by the examining physician of defendant-insurer was submitted application for insurance which were material to the risk against and
to defendant-insurer on September 5, 1950. (check ratio #1) 5. It is apparent that have the effect of avoiding the insurance policy
when the insured gave his answers regarding his previous ailment, particularly with 8.   After trial, the court rendered judgment ordering defendant-insurer to pay
regard to "Gaztritis, Ulcer of the Stomach or any disease of that organ" and "Vertigo, Cheng the sum of P10k, with legal interest from the filing of the complaint,
Dizziness, Fainting-spells or Unconsciousness", he concealed the ailment of which plus P2k as attorney’s fees, and the costs of the suit
he was treated in the Chinese General Hospital which precisely has direct connection 9.   On appeal, the CA reversed the decision of the trial court, holding that the
with the subject of the questions propounded. This deprived defendant-insurer of the insured was guilty from liability.
opportunity to make the necessary inquiry as to the nature of his past illness so that it 10.   Hence, the present petition for review.
may form its estimate relative to the approval of his application. Had defendant-
insurer been given such opportunity, considering the previous illness of the insured ISSUE/s:
as disclosed by the records of the Chinese General Hospital, defendant-insurer would 1.   WoN the insured is guilty of concealment of some facts material to the risk
probably had never consented to the issuance of the policy in question. (Doctrine) insured against which has the effect of avoiding the policy as found by the
CA – YES. Eng lied on the application despite actually knowing his
DOCTRINE: Concealment is the neglect to communicate that which a party knows sickness.
and ought to communicate. Whenever intentional, the concealment entitles the
insurer to rescind the contract of insurance. The insurance law requires the insured to RULING: the decision appealed from is affirmed, with costs against Cheng
communicate to the insurer all facts within his knowledge which are material to the
contract and which the other party has no means of ascertaining, and the materiality RATIO:
is to be determined not by the event, but solely by the probable and reasonable
influence of the facts upon the party to whom the communication is due.
1.   The insured, in his application for insurance, particularly in his declarations stomach metastases spleen", which may have a direct connection with his
to the examining physician, stated the following in answering the questions previous illness.
propounded to him: 9.   Our Insurance Law provides that A neglect to communicate that which
"14. Have you ever had any of the following diseases or symptoms? Each question a party knows and ought to communicate, is called concealment"
must be read and answered "Yes" or "No." (Section 25, Act No. 2427 or the Insurance Act (not Code)).
xxx xxx xxx 10.   Whether intentional or unintentional, the concealment entitles the
"Gastritis, Ulcer of the Stomach or any disease of that organ? No. insurer to rescind the contract of insurance (Section 26).
"Vertigo, Dizziness, Fainting-spells or Unconsciousness? No. 11.   Our law even requires the insured to communicate to the insurer all
"Cancer, Tumors or Ulcers of any kind? No. facts within his knowledge which are material to the contract and
"15. Have you ever consulted any physician not included in any of the above which the other partty has not the means of ascertaining (Section 27)
answers? Give names and address or physicians list ailments or accidents and date. 12.   the materiality is to be determined not by the event but solely by the
No." probable and reasonable in uence of the facts upon the party to whom
2.   It appears that Yu Pand Eng entered the Chinese General Hospital for the communication is due (Section 30).
medical treatment on January 29, 1950 having stayed there up to February 13.   In the case of Argente vs. West Coast Life Insurance Co., 51 Phil., 725, this
11, 1950. Upon entering the hospital, he complained of dizziness, anemia, Court said:
abdominal paids and tarry stools, and in the evening of his admission he had "One ground for the rescission of a contract of insurance under the
several abdominal pains and his discharges were with black tarry stools and Insurance Act is 'a concealment', which in section 25 is defined 'A neglect
felt dizzy and weak. to communicate that which a party knows and ought to communicate.'
3.   The history of his illness shows that the same "started a year ago as frequent Appellant argues that the concealment was immaterial and isu ceint to avoid
dizziness." An X-ray picture of his stomach was taken and the diagnosis the policy. We cannot agree. In an action on a life insurance policy where
made of him by his doctors showed that his illness was "peptic ulcer, the evidence conclusively shows that the answers to questions concerning
bleeding." diseases were untrue, the truth or falsity of the answers become the
4.   Note: Yu Pand Eng's confinement in the Chinese General Hospital took determining factor. If the policy was procured by fraudulent representations,
place from January 29, 1950 to February 11, 1950, whereas his the contract of insurance apparently set forth therein was never legally
application for insurance wherein he stated his answers to the questions existent. It can fairly be assumed become that had the true facts been
propounded to him by the examining physician of defendant-insurer disclosed by he assured, the insurance would never have been granted."
was submitted to defendant-insurer on September 5, 1950. 14.   CA did not err in declaring the policy ineffective on the ground of
5.   It is apparent that when the insured gave his answers regarding his concealment and in relieving appellee from liability
previous ailment, particularly with regard to "Gaztritis, Ulcer of the
Stomach or any disease of that organ" and "Vertigo, Dizziness,
Fainting-spells or Unconsciousness", he concealed the ailment of which
he was treated in the Chinese General Hospital which precisely has
direct connection with the subject of the questions propounded.
6.   The negative answers given by the insured regarding his previous ailment,
or his concealment of the fact that he was hospitalized and treated for
sometime of peptic ulcer and had suffered form "dizziness, anemia,
abdominal pains and tarry stools", deprived defendant-insurer of the
opportunity to make the necessary inquiry as to the nature of his past
illness so that it may form its estimate relative to the approval of his
application.
7.   Had defendant-insurer been given such opportunity, considering the
previous illness of the insured as disclosed by the records of the Chinese
General Hospital, defendant-insurer would probably had never
consented to the issuance of the policy in question.
8.   In fact, according to the death certificate, the insured died of "infiltrating
medullary carcinoma, Grade 4, advanced cardiac and of lesser curvature,
013 EGUARAS v. GREAT EASTERN (Punsalan) the favorable medical examination made by Dr. Jose Vidal, Great Eastern
Jan. 24, 1916 | Torres, J. | Misrepresentation agreed to the life insurance sought.
3.   The contract of life insurance executed between The Great Eastern Life Assurance
Company, Ltd., and Dominador Albay is set forth in the policy itself and in the
PETITIONER: Francisca Eguaras
original and supplementary applications signed by Albay, it appeared to stipulate
RESPONDENTS: The Great Eastern Life Assurance Co., Ltd., West Smith
that:
a.   "This insurance is granted in consideration of the foregoing statements and
SUMMARY: Great Eastern Life Assurance, Co., Ltd. accepted Dominador Albay’s
agreement in the application presented to obtain this policy, which
application for insurance where P5,000.00 is to be paid by the insurer to Albay’s
application forms a part of the present contract."
beneficiary, Francisca Eguaras (mother-in-law). In perfection of the contract thereof,
4.   Such condition is repeated in Clause VIII of the conditions and privileges
Albay, who was not in good health and by connivance with insurance agent Remigio,
granted to the insured, that:
presented Castor Garcia (fake Albay) to Dr. Vidal who was commissioned by Great
a.   "This policy and the application presented to secure it, taken together,
Eastern to examine applicants for life insurance. In view of the favorable report of Dr.
constitute the whole contract, which cannot be altered except in writing by
Vidal that Albay (who was actually Garcia) was in good health and possessed the
the general manager or some person expressly appointed therefor by the
qualifications required by Great Eastern for perfecting the contract, the company freely
board of directors."
and willingly consented to the execution thereof. Such medical examination and of the
5.   (IMPT) In the supplementary application presented by Albay to the question “Do
favorable professional report induced Great Eastern to indeed believe that Albay was in
you think that you are free from disease and that you have a good
good health but who in fact was Castor Garcia. Actual insured Albay died 1 month and 23
constitution?” He answered “Yes.”
days in Sta. Cruz, Laguna after the insurance had been granted.
6.   (IMPT) Furthermore, in another question “Have you suffered from any affection
Issue: (1) WoN the life insurance obtained by Albay, with the assistance of Remigio of … (c) Chest – Cough, asthma, spitting blood, pleurisy?” he answered “No.”
(insurance agent), is legal and valid – NO 7.   Dr. Vidal, the physician of the insurance company in charge of the physical
The person who was actually examined was not Dominador Albay, but was another examination of the person applying for insurance in Laguna, did the check-up on
(Castor Garcia), in order to get a favorable recommendation on the health of the insured Albay and recorded that the development, expansion, percussion, and auscultation of
which induced Great Eastern to approve of the application and thus issue the contested the applicant's chest were "normal" and recommended to the company that it could
life insurance policy of Albay where Eguaras is the beneficiary. Furthermore, the "take the risk" of insuring the applicant Dominador Albay
signatures that appear on the papers referring to the insurance are so different from a.   The report also had medical officer Lunn placing “O.K.” on it.
those which appear on the other documents which unquestionably bear the 8.   A month after the issuance of the insurance policy, Albay allegedly died of
signature of the real Dominador Albay that SC can do no less than reach the intestinal occlusion (intestinal blockage which causes blockage of bowel) in Sta.
conclusion that there was a person who passed himself off as Dominador Albay and Cruz, Laguna, as proven in Dr. Kamatoy’s certificate.
said person was the one who went on signing the documents relating to the alleged 9.   Great Eastern, despite having received satisfactory proofs of its insured/Albay’s
insurance of Dominador Albay. death, refused to pay the amount of the insurance and alleged that the policy was
(2) WoN Great Eastern Life Assurance is under obligation to pay the value to Francisca secured through fraud and deceit and was therefore illegal and void.
Eguaras – NO
Having proven the misrepresentation by the insurance agent, serious deceit indeed NOTE: this recounts how the insurance policy was applied for and how the misrepresentation went about
occurred in perfecting the insurance contract. The company would not have granted 10.   It appears that Albay had knowledge of the false replies contained in the two
insurance applied for had misrepresentation and deception not occurred. Therefore, it is applications for insurance and knowingly permitted fraud to be practiced upon
improper nor is it permitted by the law to order collection of the P5,000.00 Great Eastern
a.   For in his acknowledgment and consent his mother-in-law was designated
DOCTRINE: Misrepresentation in applying for life insurance, also in connivance with as the beneficiary of the insurance, despite the fact that he had children
an insurance agent, constitutes serious deceit which cannot entitle a party to the proceeds and his mother was still living
of the policy despite being previously issued by the insurance company. 11.   The fraud consisted in the fact that a healthy and robust person was substituted in
  place of the Albay when Dr. Vidal made the physical exam of the one who was
FACTS: seeking to be insured, for Albay was in bad health on and before the date of
1.   Characters: executing the insurance contract, of which facts the insured Albay and the
Francisca Eguaras (Eguaras) – beneficiary; mother-in-law insurance agent Remigio had full knowledge.
Dominador Albay (Albay) – insured a.   Such person was presented to him by insurance agent Remigio who told
The Great Eastern Life Assurance Co., Ltd. (Great Eastern) – insurer him that such was Albay
Ponciano Remigio (Remigio) – insurance agent b.   But in truth, the person was Castor Garcia who presented himself to
2.   Oct. 14, 1912: Albay, through the efforts of Great Eastern’s agent, Ponciano be Dominador Albay.
Remigio, got to insure his life for the sum of P5,000.00, naming Eguaras (who 12.   The insurance company endeavored to prove by means of cross-examination of
was his mother-in-law) as the beneficiary in case of his death. Ponciano Remigioand by means of the declaration of another insurance agent, Jose
a.   That through Albay’s representations and statements in his application and D. Arce, that Remigio had always been in the habit of securing the insurance of
sick persons, who died shortly after it was issued, in fraud and to the serious 2.   But if the real Albay wrote in the Spanish style in the months of Jan-March
injury of the defendant company. 1912, as demonstrated by the signatures affixed to certain documents, it is
a.   Court overruled this attempt of Great Eastern and did not permit proof of impossible to believe that he should have radically changed his form of writing
specific fraudulent acts performed by its agent Remigio 2 months later by adopting a different handwriting as can be seen in the alleged
b.   It is observed that Remigio has already been convicted of the crime of signature in the electors.
estafa and was to restitute The Insular Life Insurance Co. an amount of 3.   The signatures that appear on the papers referring to the insurance are so
P20. different from those which appear on the other documents which
13.   There was also a sworn declaration by Atty. O’Brien indicating that agent Remigio unquestionably bear the signature of the real Dominador Albay that, in
interviewed him in his office telling him that the signatures affixed to the original consideration of the short time which elapsed between the last genuine signature in
and supplemental application for insurance signed before Dr. Vidal were false. March, 1912, when he sold a tract of land, and his oath as elector in May of the
a.   Remigio further told him that the former was disgusted with his same year, 1912, and the great difference that exists between the two signatures, we
accomplices because they could not reach an agreement regarding the can do no less than reach the conclusion already stated that there was a person
distribution among them of the amount of the policy. (scammaz) who passed himself off as Dominador Albay and said person was the one who
14.   Remigio denied all this in his testimony. went on signing the documents relating to the alleged insurance of Dominador
15.   Dr. Reyes also testified and said that Albay consulted him regarding the cough he Albay, who died on December 6, 1912
had and after a medical examination, the conclusion was that Albay was actually 4.   Moreover, Dominador Albay's age, according to the application and the insurance
suffering from tuberculosis in the first stage. policy, was 40 years in 1912, while according to his personal cedulas he was only 32
16.   There is circumstantial evidence in the case that Albay died of tuberculosis because years of age in 1911, so that when he was insured he must have been only 33.
his own mother, Manuela Flores, so affirmed in the affidavit. However, it was not 5.   It is therefore proven that the signatures on the insurance applications reading
presented as evidence because Flores repudiated it in court. "Dominador Albay" are false and forged
a.   It was repudiated because Great Eastern was unwilling to give her money 6.   Under Art. 126936(OLD CIVIL CODE), it is essential to the nature of the deceit that
for the statements they would make in the court in the event that West said deceit be prior to or contemporaneous with the consent that is a necessary
Smith, Great Eastern’s agent, should win the case, and accordingly to prerequisite for perfecting the contract, but not that it may have occurred or
execute “an instrument we can hold to.” happened thereafter.
a.   A contract is therefore deceitful, for the execution whereof the consent
NOTE: at this point, I’ll go back to what happened in the courts; though I think you can skip this sa recit of one of the parties has been secured by means of fraud, because he
17.   CFI rendered judgment where Great Eastern was to pay P5,000.00 to Eguaras with was persuaded by words or insidious machinations, statements or false
legal interest. promises, and a defective consent wrung from him, even though such do
18.   Great Eastern moved for a reopening of the case and a new trial which was denied not constitute estafa or any other criminal act subject to the penal
19.   Great Eastern filed a Bill of exceptions, approved and forwarded to the clerk of the law.
SC. 7.   This fraud consisted in the substitution at the examination of Castor Garcia in
place of the insured Dominador Albay, and as the deceit practiced in the said
ISSUE/s: contract is of a serious nature, the same is ipso facto void and ineffective, in
1.   WoN the life insurance obtained by Albay, with the assistance of Remigio accordance with the provisions of article 1270 of the [old] Civil Code.
(insurance agent), is legal and valid – NO, the person who was examined was not 8.   Serious deceit occurred in perfecting the insurance contract, for had the agent of the
Dominador Albay, but was another (Castor Garcia), in order to get a favorable company not been deceived it would not have granted the insurance applied for by
recommendation on the health of the insured which induced Great Eastern to Albay, nor would it have executed the contract by virtue whereof payment is
approve of the application and thus issue the contested life insurance policy of claimed of the value of the policy obtained through fraud; and consequently on such
Albay where Eguaras is the beneficiary. (misrepresentation) assumptions it is improper, nor is it permitted by the law, to order collection of the
2.   WoN Great Eastern Life Assurance is under obligation to pay the value (P5,000) to amount claimed.
Francisca Eguaras – NO. Having proven the misrepresentation by the insurance
agent, serious deceit indeed occurred in perfecting the insurance contract.

RULING: 1st part of judgment appealed from (regarding payment of P5000) is reversed. The
Great Eastern Life Assurance Company, Ltd. is absolved from the complaint. 2nd part of said
judgment in so far as it absolves West Smith is affirmed and petition for damages dismissed.

RATIO:
1.   Eguaras’ claim is based on the genuineness of the signature of “Dominador Albay”                                                                                                                        
in the elector’s oath on the contention that if the signature on said election oath is 36
Art. 1269: "There is deceit when by words or insidious machinations on the part of one of the
genuine, those which appear on the insurance applications, and that affixed to the contracting parties the other is induced to execute a contract which without them he would not have
Albay’s letter to Great Eastern, must also be true and genuine. made."
014 Soliman v. US (Cristelle)
1958 | N/A | Misrepresentation

Dean Hofi can’t find the original copy of this case, ito lang nasa internet
and super short digest pa ito.

DOCTRINE: One who solicits insurance is an underwriter and not an agent


of the insurance company. If insurer appoints a general agent, then such
agent can bind the company by virtue of the written appointment. On the
other hand, when an underwriter fills up a policy with false answers and
later the insured signs the policy, the false answers become the insured’s
own answers because he signed the policy.

FACTS:
1.   US Life issued a 20 year endowment life policy on the joint lives of
Patricio Soliman and his wife Rosario, each of them being the
beneficiary of the other.
2.   In March 1949, the spouses were informed that the premium for
January 1949 was still unpaid notwithstanding that the 31-day grace
period has already expired, and they were furnished at the same time
long-form health certificates for the reinstatement of the policies.
3.   In April 1949, they submitted the certificates and paid the premiums.
4.   In January 1950, Rosario died of acute dilation of the heart, and
thereafter, Patricio filed a claim for the proceeds of the insurance.
5.   US Life denied the claim and filed action for the rescission of the
contract on the ground that the certificates failed to disclose that
Rosario had been suffering from bronchial asthma for 3 years prior
to their submission.

ISSUES: Whether or not the contract can still be rescinded? Yes, it can
be rescinded.

RULING:

RATIO:
1.   The insurer is once again given two years from the date of
reinstatement to investigate into the veracity of the facts represented
by the insured in the application for reinstatement. When US life
sought to rescind the contract on the ground of
concealment/misrepresentation, two years had not yet
elapsed. Hence, the contract can still be rescinded.
015 Pacific Banking Corp v. CA (Rosales) the foundation does not exist, the superstructure does not arise. Falsehood in
November 28, 1988 | Paras, J. | Misrepresentation such representations is not shown to vary or add to the contract, or to terminate a
contract which has once been made, but to show that no contract has ever
PETITIONER: Pacific Banking Corporation existed. A void or inexistent contract is one which has no force and effect from
RESPONDENTS: Court of Appeals and Oriental Assurance Corporation the very beginning, as if it had never been entered into, and which cannot be
validated either by time or by ratification.
SUMMARY: On October 21,1963, Fire Policy No. F-3770, an open policy, was
issued to the Paramount Shirt Manufacturing Co., by which Oriental Assurance FACTS:
Corporation bound itself to indemnify the insured for any loss or damage, not 1.   On October 21,1963, Fire Policy No. F-3770, an open policy, was issued to
exceeding P61,000.00, caused by fire to its property for a period of one year. the Paramount Shirt Manufacturing Co., by which Oriental Assurance
Paramount Shirt is a debtor of Pacific Banking in the amount of not less than Corporation bound itself to indemnify the insured for any loss or damage,
P800,000.00 and the goods described in the policy were held in trust by the not exceeding P61,000.00, caused by fire to its property consisting of
insured for Pacific Banking under thrust receipts. A fire broke out on the subject stocks, materials and supplies usual to a shirt factory, including furniture,
premises destroying the goods contained in its ground and second floors. Pacific fixtures, machinery and equipment while contained in the ground, second
Banking sent a letter of demand to Oriental Assurance for indemnity due to the and third floors of the building situated at number 256 Jaboneros St., San
loss of property by fire. Oriental Assurance said it is not yet ready to accede to Nicolas, Manila, for a period of one year commencing from that date to
the latter's demand as it is awaiting the final report of the insurance adjuster, October 21, 1964.
H.H. Bayne Adjustment Company. The insurance adjuster notified Pacific 2.   Paramount Shirt was at the time of the issuance of the policy and is up to
Banking that Paramount Shirt under the policy had not filed any claim with it, this time, a debtor of Pacific Banking in the amount of not less than
nor submitted proof of loss which is a clear violation of Policy Condition No.11, P800,000.00 and the goods described in the policy were held in trust by the
and for which reason, determination of the liability of Oriental Assurance could insured for Pacific Banking under thrust receipts.
not be had. For failure of the insurance company to pay the loss as demanded, 3.   Said policy was duly endorsed to Pacific Banking as mortgagee/ trustor of
Pacific Banking on filed in the court an action for a sum of money. Pacific the properties insured, with the knowledge and consent of Oriental
Banking presented in evidence a communication revealing undeclared co- Assurance to the effect that "loss if any under this policy is payable to the
insurances undertaken by Paramount. RTC rendered a decision adjudging Pacific Banking Corporation".
Oriental Assurance liable to Pacific Banking under the said contract of 4.   On January 4, 1964, while the aforesaid policy was in full force and effect,
insurance. The CA reversed. a fire broke out on the subject premises destroying the goods contained in
its ground and second floors.
WoN Paramount Shirt, the insured, was guilty of fraud which rendered the 5.   On January 24, 1964, counsel for Pacific Banking sent a letter of demand to
contract of insurance void? – YES, because Paramount Shirt failed to reveal Oriental Assurance for indemnity due to the loss of property by fire under
before the loss three other insurances. By reason of said unrevealed insurances, the endorsement of said policy.
Paramount Shirt had been guilty of a false declaration; a clear misrepresentation 6.   On January 28, 1964, Oriental Assurance informed counsel for Pacific
and a vital one because where the insured had been asked to reveal but did not, Baking that it was not yet ready to accede to the latter's demand as the
that was deception. Concrete evidence of fraud or false declaration by former is awaiting the final report of the insurance adjuster, H.H. Bayne
Paramount Shirt was furnished by Pacific Banking itself when the facts alleged Adjustment Company.
in the policy under clauses "Co-Insurances Declared" and "Other Insurance 7.   On March 25, 1964, the said insurance adjuster notified counsel for Pacific
Clause" are materially different from the actual number of co-insurances taken Banking that Paramount Shirt under the policy had not filed any claim with
over the subject property. As the insurance policy against fire expressly required it, nor submitted proof of loss which is a clear violation of Policy Condition
that notice should be given by the insured of other insurance upon the same No.11, and for which reason, determination of the liability of Oriental
property, the total absence of such notice nullifies the policy. Undoubtedly, it is Assurance could not be had.
but fair and just that where the insured who is primarily entitled to receive the 8.   On April 24, 1964, Pacific Banking’s counsel replied to aforesaid letter
proceeds of the policy has by its fraud and/or misrepresentation, forfeited said asking the insurance adjuster to verify from the records of the Bureau of
right, with more reason Pacific Banking which is merely claiming as indorsee of Customs the entries of merchandise taken into the customs bonded
said insured, cannot be entitled to such proceeds. warehouse razed by fire as a reliable proof of loss.
9.   For failure of the insurance company to pay the loss as demanded, Pacific
DOCTRINE: Representations of facts are the foundation of the contract and if Banking on filed in the court an action for a sum of money against Oriental
Assurance Corporation in the principal sum of P61,000.00 issued in favor of merit, and the decision appealed from is AFFIRMED. No costs.
Paramount Shirt Manufacturing Co.
10.   Oriental Assurance raised the following defenses in its answer to wit: (a) RATIO:
lack of formal claim by insured over the loss and (b) premature filing of the 1.   Policy Condition No. 3 explicitly provides:
suit as neither Pacific Banking nor Paramount Shirt had submitted any proof The Insured shall give notice to the Company of any insurance
of loss on the basis of which Oriental Assurance would determine its already effected, or which may subsequently be effected, covering
liability and the amount thereof, either to Oriental Assurance or its adjuster any of the property hereby insured, and unless such notice be given
H.H. Bayne Adjustment Co., both in violation of Policy Condition No.11. and the particulars of such insurance or insurances be stated in or
11.   At the trial, Pacific Banking presented in evidence a communication dated endorsed on this Policy by or on behalf of the Company before the
December 22, 1965 of the insurance adjuster, H.H. Bayne Adjustment Co. occurrence of any loss or damage, all benefit under this policy
to Asian Surety Insurance Co., Inc., revealing undeclared co-insurances shall be forfeited.
with the following: P30,000.00 with Wellington Insurance; P25,000. 00 2.   It is not disputed that Paramount Shirt failed to reveal before the loss
with Empire Surety and P250,000.00 with Asian Surety; undertaken by three other insurances. As found by the Court of Appeals, by reason of
insured Paramount on the same property covered by its policy with said unrevealed insurances, the insured had been guilty of a false
Oriental Assurance whereas the only co-insurances declared in the declaration; a clear misrepresentation and a vital one because where
subject policy are those of P30,000.00 with Malayan P50,000.00 with the insured had been asked to reveal but did not, that was deception.
South Sea and P25.000.00 with Victory. Otherwise stated, had Paramount Shirt have known that there were
12.   It will be noted that the defense of fraud and/or violation of Condition No. 3 many co-insurances, it could have hesitated or plainly desisted from
in the Policy, in the form of non-declaration of co-insurances which was entering into such contract. Hence, the Paramount Shirt was guilty of
not pleaded in the answer was also not pleaded in the Motion to clear fraud.
Dismiss. 3.   Concrete evidence of fraud or false declaration by Paramount Shirt was
13.   At any rate the RTC denied Oriental Assurance’s motion on the ground that furnished by Pacific Banking itself when the facts alleged in the policy
the defense of lack of proof of loss or defects therein was raised for the under clauses "Co-Insurances Declared" and "Other Insurance Clause" are
first time after the commencement of the suit and that it must be materially different from the actual number of co-insurances taken over the
deemed to have waived the requirement of proof of loss. subject property.
14.   The case was considered submitted for decision from which order Oriental 4.   Consequently, the whole foundation of the contract fails, the risk does
Assurance filed a motion for reconsideration to set the case or further not attach and the policy never becomes a contract between the parties.
reception of Oriental Assurance’s additional evidence, “in order to prove Representations of facts are the foundation of the contract and if the
that insured has committed a violation of condition No. 3 of the policy in foundation does not exist, the superstructure does not arise. Falsehood
relation to the other Insurance Clause.” in such representations is not shown to vary or add to the contract, or
15.   The case was set for the continuation of the hearing for the reception merely to terminate a contract which has once been made, but to show that no
of the testimony of Alejandro Tan Gatue, Manager of the Adjustment Co., contract has ever existed. A void or inexistent contract is one which has
over the vehement opposition of Pacific Banking. no force and effect from the very beginning, as if it had never been
16.   RTC rendered a decision adjudging Oriental Assurance liable to Pacific entered into, and which cannot be validated either by time or by
Banking under the said contract of insurance. The CA reversed the decision ratification.
of the trial court. 5.   As the insurance policy against fire expressly required that notice
should be given by the insured of other insurance upon the same
ISSUE/s: property, the total absence of such notice nullifies the policy.
3.   WoN Paramount Shirt, the insured, was guilty of fraud which rendered the 6.   The argument that notice of co-insurances may be made orally negates
contract of insurance void? – YES, because Paramount Shirt failed to reveal policy condition No. 20 which requires every notice and other
before the loss three other insurances. By reason of said unrevealed communications to the insurer to be written or printed.
insurances, Paramount Shirt had been guilty of a false declaration; a clear 7.   Particularly referring to the mortgage clause of the policy, Pacific Banking
misrepresentation and a vital one because where the insured had been asked argues that considering the purpose for which the endorsement or
to reveal but did not, that was deception. assignment was made, that is, to protect the mortgagee/assignee against any
untoward act or omission of the insured, it would be absurd to hold that
RULING: PREMISES CONSIDERED, the petition is DISMISSED for lack of
Pacific Banking is barred from recovering the insurance on account of the necessary information to ascertain the particular account of the articles
alleged violation committed by the insured. destroyed by fire as well as the amount of loss. It is noteworthy that
8.   It is obvious that Pacific Banking has missed all together the import of Oriental Assurance and its adjuster notified Pacific Banking that insured
subject mortgage clause which specifically provides: had not yet filed a written claim nor submitted the supporting documents in
Mortgage Clause compliance with the requirements set forth in the policy. Despite the notice,
Loss, if any, under this policy, shall be payable to the PACIFIC the latter remained unheedful. Since the required claim by insured, together
BANKING CORPORATION Manila mortgagee/trustor as its with the preliminary submittal of relevant documents had not been
interest may appear, it being hereby understood and agreed that complied with, it follows that Oriental Assurance could not be deemed to
this insurance as to the interest of the mortgagee/trustor only have finally rejected Pacific Banking’s claim and therefore the latter's cause
herein, shall not be invalidated by any act or neglect—except of action had not yet arisen.
fraud or misrepresentation, or arson—of the mortgagor or 15.   Contracts of insurance are contracts of indemnity upon the terms and
owner/trustee of the property insured; provided, that in case the conditions specified in the policy. The parties have a right to impose
mortgagor or owner/ trustee neglects or refuses to pay any such reasonable conditions at the time of the making of the contract as
premium, the mortgagee/ trustor shall, on demand pay the same. they may deem wise and necessary. The agreement has the force of law
9.   The paragraph clearly states the exceptions to the general rule that between the parties. The terms of the policy constitute the measure of
insurance as to the interest of the mortgagee, cannot be invalidated; the insurer's liability, and in order to recover, the insured must show
namely: fraud, or misrepresentation or arson. As correctly found by the himself within those terms. The compliance of the insured with the
Court of Appeals, concealment of the aforecited co-insurances can easily be terms of the policy is a condition precedent to the light of recovery.
fraud, or in the very least, misrepresentation.
10.   Undoubtedly, it is but fair and just that where the insured who is
primarily entitled to receive the proceeds of the policy has by its fraud
and/or misrepresentation, forfeited said right, with more reason Pacific
Banking which is merely claiming as indorsee of said insured, cannot
be entitled to such proceeds.
11.   Pacific Banking further stressed that fraud which was not pleaded as a
defense in Oriental Assurance’s answer or motion to dismiss, should be
deemed to have been waived.
12.   It will be noted that the fact of fraud was tried by express or at least implied
consent of the parties. Pacific Banking did not only object to the
introduction of evidence but on the contrary, presented the very
evidence that proved its existence.
13.   In the case at bar, policy condition No. 11 specifically provides that the
insured shall on the happening of any loss or damage give notice to the
company and shall within fifteen days after such loss or damage deliver to
Oriental Assurance (a) a claim in writing giving particular account as to the
articles or goods destroyed and the amount of the loss or damage and (b)
particulars of all other insurances, if any. Likewise, insured was required "at
his own expense to produce, procure and give to the company all such
further particulars, plans, specifications, books, vouchers, invoices,
duplicates or copies thereof, documents, proofs and information with
respect to the claim".
14.   The evidence adduced shows that twenty-four days after the fire, Pacific
Banking merely wrote letters to Oriental Assurance to serve as a notice of
loss, thereafter, the former did not furnish the latter whatever pertinent
documents were necessary to prove and estimate its loss. Instead, Pacific
Banking shifted upon Oriental Assurance the burden of fishing out the
016 QUA CHEE GAN v. LAW UNION (Sabaupan) law is charitable enough to assume, in the absence of any showing to the
17 December 1955 | Reyes, J.B.L, J. | Breach of Warranty contrary, that an insurance company intends to execute a valid contract in return
for the premium received; and when the policy contains a condition which
PETITIONER: Qua Chee Gan renders it voidable at its inception, and this result is known to the insurer, it will
RESPONDENTS: Law Union and Rock Insurance Co., Ltd., represented by its be presumed to have intended to waive the conditions and to execute a binding
agent, Warner, Barnes and Co., Ltd. contract, rather than to have deceived the insured into thinking he is insured
when in fact he is not, and to have taken his money without consideration.”
SUMMARY: Qua Chee Gan owned 4 warehouses used for storage of stocks of
copra and of hemp. Bodegas 1, 3, and 4 were gutted by fire of unknown origin. It is well settled that the keeping of inflammable oils on the premises, though
Qua Chee Gan submitted fore claims to the insurance company but the latter prohibited by the policy, does not void it if such keeping is incidental to the
resisted payment claiming that the fire had been deliberately caused by the business.
insured. Qua Chee Gan then instituted a civil action to collect the insurance
proceeds. The insurance company argues among others that the insurance polices  
were avoided for breach of warranty. The issue is whether there was a breach of FACTS:
warranty, specifically the fire hydrant warranty and the hemp warranty. The 20.   Qua Chee Gan, a merchant from Albay, owned four warehouses or bodegas
Supreme Court ruled in the negative. The insurance company is barred by waiver (designated as Bodegas nos. 1-4) used for the storage of stocks of copra and
(or rather estoppel) to claim violation of the fire hydrants warranty, because of hemp, baled and loose. The bodegas, with their contents, were insured
knowing fully all the number of hydrants demanded therein never existed from with the Law Union (insurance company) since 1937, and the loss made
the very beginning, the insurance company nevertheless issued the policies in payable to the Philippine National Bank (PNB) as mortgage of the hemp
question subject to such warranty and received the corresponding premiums. The and copra, to the extent of its interest.
insurance company was aware, even before the policies were issued, that in the 21.   Fire of undetermined origin broke out in July 1940 and lasted for almost
premises insured, there were only 2 fire hydrants installed and 2 others nearby, one week. It gutted and completely destroyed Bodegas Nos. 1, 3, and 4,
owned by the municipality of Tabaco, contrary to the requirements of the with the merchandise stored therein. Qua Chee Gan informed the insurer on
warranty in question. For the hemp warranty, the Court noted that gasoline is not the same day of the fire and the next day, fire adjusters engaged by the
specifically mentioned among the prohibited articles listed in the “hemp insurance company arrived and proceeded to conduct an extensive
warranty.” The cause relied upon by the insurer speaks of “oils” (animal and/or investigation.
vegetable and/or mineral and/or their liquid products having a flash point of 22.   Qua Chee Gan submitted fire claims amounting to P398,562 (but reduced to
below 300°F). This is ambiguous and uncertain, for in ordinary parlance, “oils” the full amount of the insurance, P370,000). The insurance company
mean lubricants and not gasoline or kerosene. And how many insured are in a resisted payment, claiming violation of warranties and conditions, filing of
position to understand or determine “flash point below 300°F.” Here, again, by fraudulent claims, and that the fire had been deliberately caused by the
reason of the exclusive control of the insurance company over the terms and insured or by other persons in connivance with him.
phraseology of the contract, the ambiguity must be held strictly against the 23.   Qua Chee Gan and his brother, Qua Chee Pao, and some employees of his,
insurer and liberally in favor of the insured, specially to avoid forfeiture. were indicted and tried in 1940 for the crime of arson, it being claimed that
Another point that is in favor of the insured is that the gasoline kept in Bodega they had set fire to the destroyed warehouses to collect the insurance. They
No. 2 was only incidental to his business. It should also be noted that the "Hemp were acquitted.
Warranty" forbade storage only "in the building to which this insurance applies 24.   Thereafter, a civil suit to collect the insurance money was filed before the
and/or in any building communicating therewith", and it is undisputed that no CFI to recover the proceeds of certain life insurance policies. The CFI ruled
gasoline was stored in the burned bodegas, and that "Bodega No. 2" which was in favor of Qua Chee Gan. Hence, the instant petition.
not burned and where the gasoline was found, stood isolated from the other 25.   Insurance Company’s arguments:
insured bodegas. a.   The trial court should have held that the policies were avoided for
breach of warranty, specifically the one appearing on a rider pasted
DOCTRINE: It is usually held that where the insurer, at the time of the issuance on the face of the policies. These riders were attached for the first
of a policy of insurance, has knowledge of existing facts which, if insisted on, time in 1939, and the pertinent portions read as follows:
would invalidate the contract from its very inception, each knowledge constitutes
a waiver of conditions in the contract inconsistent with the known facts, and the “Memo. Of Warranty. – The undernoted Appliances for the
insurer is stopped thereafter from asserting the breach of such conditions. The extinction of fire being kept on the premises insured hereby, and it
being declared and understood that there is an ample end constant RATIO:
water supply with sufficient pressure available at all seasons for On the violation of fire hydrant warranty
the same, it is hereby warranted that the said appliances shall be 9.   The insurance company is barred by waiver (or rather estoppel) to claim
maintained in efficient working order during the currency of this violation of the fire hydrants warranty, because knowing fully all the
policy, by reason whereof a discount of 2.5% is allowed on the number of hydrants demanded therein never existed from the very
premium chargeable under this policy. beginning, the insurance company nevertheless issued the policies in
question subject to such warranty and received the corresponding
Hydrants in the compound, not less in number than one for each premiums.
150 feet of external wall measurement of buildings, protected, with 10.   It would perilously close to conniving at fraud upon the insured to allow the
not less than 100 feet of hose piping and nozzles for every 2 insurance company to claim now as void ab initio the policies that it had
hydrants kept under cover in convenient places, the hydrants being issued to Qua Chee Gan without warning of their fatal defect, of which it
supplied with water pressure by a pumping engine, or from some was informed, and after it had misled Qua Chee Gan into believing that the
other source, capable of discharging at the rate of not less than 200 policies were effective.
gallons of water per minute into the upper story of the highest 11.   The insurance company was aware, even before the policies were issued,
building protected, and a trained brigade of not less than 20 mean that in the premises insured, there were only 2 fire hydrants installed and 2
to work the same.” others nearby, owned by the municipality of Tabaco, contrary to the
b.   Given the size of the external wall of the bodegas, Qua Chee Gan requirements of the warranty in question. This fact was established by a
should have 11 fire hydrants in the compound, and that he actually positive testimony of the insurance company’s agents that they inspected
had only two, with a further pair nearby, belonging to the the premises. That such inspection was made is rendered probable by its
municipality of Tabaco. being a prerequisite for the fixing of the discount on the premium to which
c.   The insured violated the “Hemp Warranty” provisions of the the insured was entitled, since the discount depended on the number of
policy covering Bodega No. 2 against the storage of gasoline, since hydrants, and the firefighting equipment available.
Qua Chee Gan admitted that there were 36 cans of gasoline 12.   The law, supported by a long line of cases, is expressed by American
Bodega No. 2. Jurisprudence: “It is usually held that where the insurer, at the time of
d.   There was fraudulent overvaluation and the insured caused the fire the issuance of a policy of insurance, has knowledge of existing facts
so that he can pay the mortgagee bank. which, if insisted on, would invalidate the contract from its very
e.   The burned bodegas could not have contained the quantities of inception, each knowledge constitutes a waiver of conditions in the
copra and hemp stated in the fire claims. contract inconsistent with the known facts, and the insurer is stopped
f.   The insured made false and fraudulent statements that voids the thereafter from asserting the breach of such conditions. The law is
policy. charitable enough to assume, in the absence of any showing to the contrary,
that an insurance company intends to execute a valid contract in return for
ISSUE/s: the premium received; and when the policy contains a condition which
4.   Whether there was a violation of the fire hydrant warranty. – NO, the renders it voidable at its inception, and this result is known to the insurer, it
insurer is barred by estoppel to claim such violation. will be presumed to have intended to waive the conditions and to execute a
5.   Whether there was a violation of the hemp warranty. – NO, gasoline is not binding contract, rather than to have deceived the insured into thinking he is
specifically mentioned among the prohibited articles listed in the “hemp insured when in fact he is not, and to have taken his money without
warranty.” consideration.”
6.   Whether the insured made false and fraudulent statements that voids the 13.   The reason for this rule is as follows: “x x x To allow a company to accept
policy. – NO, the discrepancies were a result of the insured’s erroneous one's money for a policy of insurance which it then knows to be void and of
interpretation of the provisions of the insurance policies and claim forms, no effect, though it knows as it must, that the assured believes it to be valid
caused by his imperfect knowledge of English, and that the misstatements and binding, is so contrary to the dictates of honesty and fair dealing, and so
were innocently made and without intent to defraud. closely related to positive fraud, as to be abhorrent to fairminded men. It
would be to allow the company to treat the policy as valid long enough to
RULING: We find no reversible error in the judgment appealed from, wherefore the get the premium on it and leave it at liberty to repudiate it the next moment.
same is hereby affirmed. x x x”
14.   The inequitableness of the conduct observed by the insurance company in
this case is heightened by the fact that after the insured had incurred the would be unreasonable to expect the insured to maintain for his compound
expense of installing the two hydrants, the company collected the premiums alone a firefighting force that many municipalities in the islands do not even
and issued him a policy so worded that it gave the insured a discount much possess.
smaller than that he was normally entitled to. According to the "Scale of
Allowances," a policy subject to a warranty of the existence of one re On the violation of hemp warranty
hydrant for every 150 feet of external wall entitled the insured to a discount 21.   The Court noted that gasoline is not specifically mentioned among the
of 7.5% of the premium; while the existence of "hydrants, in compound" prohibited articles listed in the “hemp warranty.” The cause relied upon by
(regardless of number) reduced the allowance on the premium to a mere the insurer speaks of “oils” (animal and/or vegetable and/or mineral and/or
2.5%. their liquid products having a flash point of below 300°F). This is
15.   But the insurance company in this case, so worded the policies that while ambiguous and uncertain, for in ordinary parlance, “oils” mean lubricants
exacting the greater number of fire hydrants and appliances, it kept the and not gasoline or kerosene. And how many insured are in a position to
premium discount at the minimum of 2.5%, thereby giving the insurance understand or determine “flash point below 300°F.” Here, again, by reason
company a double benefit. No reason is shown why Qua Chee Gan’s of the exclusive control of the insurance company over the terms and
premises, that had been insured with the insurance company for several phraseology of the contract, the ambiguity must be held strictly against the
years past, should suddenly be regarded in 1939 as so hazardous as to be insurer and liberally in favor of the insured, specially to avoid forfeiture.
accorded a treatment beyond the limits of the insurance company’s own 22.   There is no reason why the prohibition of keeping gasoline in the premises
scale of allowances. Such abnormal treatment of the insured strongly points could not be expressed clearly and unmistakably, in the language and terms
at an abuse of the insurance company’s selection of the words and terms of that the general public can readily understand, without resort to obscure
the contract, over which it had absolute control. esoteric expression. If intended to rely upon a condition of that character, it
16.   Parol evidence is not applicable in the instant case. It is a well settled rule ought to have been plainly expressed in the policy.
of law that an insurer which with knowledge of facts entitling it to treat 23.   Another point that is in favor of the insured is that the gasoline kept in
a policy as no longer in force, receives and accepts a premium on the Bodega No. 2 was only incidental to his business, being no more than a
policy, estopped to take advantage of the forfeiture. x x x It would be customary 2 day's supply for the five or six motor vehicles used for
unconscionable to permit a company to issue a policy under circumstances transporting of the stored merchandise. It is well settled that the keeping
which it knew rendered the policy void and then to accept and retain of inflammable oils on the premises, though prohibited by the policy,
premiums under such a void policy. does not void it if such keeping is incidental to the business. According
17.   Moreover, taking into account the well-known rule that ambiguities or to the weight of authority, even though there are printed prohibitions against
obscurities must be strictly interpreted against the party that caused them, keeping certain articles on the insured premises the policy will not be
the "memo of warranty" invoked by the insurance company bars the latter avoided by a violation of these prohibitions, if the prohibited articles are
from questioning the existence of the appliances called for in the insured necessary or in customary use in carrying on the trade or business
premises, since its initial expression, "the undernoted appliances for the conducted on the premises.
extinction of fire being kept on the premises insured hereby, . . . it is hereby 24.   It should also be noted that the "Hemp Warranty" forbade storage only "in
warranted . . . ", admits of interpretation as an admission of the existence of the building to which this insurance applies and/or in any building
such appliances which appellant cannot now contradict, should the parol communicating therewith", and it is undisputed that no gasoline was stored
evidence rule apply. in the burned bodegas, and that "Bodega No. 2" which was not burned and
18.   The alleged violation of the firehose warranty must be equally rejected, where the gasoline was found, stood isolated from the other insured
since the insurance company’s argument thereon is based on the assumption bodegas.
that the insured was bound to maintain no less than eleven hydrants (one per
150 feet of wall), which requirement the insurance company is estopped On the charge of fraudulent overvaluation
from enforcing. 25.   This charge was found unsubstantiated by the trial court, and no reason has
19.   On the breach of water pressure condition, the court found such claim been shown to alter this finding. The insured gave the insurance examiner
worthless because the witness repeatedly refused and professed inability to all the data he asked for and the examiner even kept and photograph some
estimate the rate of discharge of the water. of the examined books in his possession.
20.   As to the maintenance of a trained fire brigade of 20 mean, the records
shows that the same was organized, and drilled, although not maintained as On the insured’s connivance at the loss
a permanently separate unit, which the warranty did not require. Anyway, it 26.   This defense is predicated on the assumption that the insured was n
financial difficulties and set the fire to defraud the insurance company, under the terms of the policies, and the trial court correctly deducted the
presumably in order to pay off PNB, to which most of the insured hemp and same from its award.
copra was pledged.
27.   This defense is fatally undermined by the established fact that,
notwithstanding the insurer’s refusal to pay the value of the policies,
insured was able to pay off PNB in a short time, and if he was able to do so,
no motive appears for attempt to defraud the insurer.
28.   While the acquittal of the insured in the arson case is not res judicata on the
present civil action, the insurer's evidence, to judge from the decision in the
criminal case, is practically identical in both cases and must lead to the
same result, since the proof to establish the defense of connivance at the re
in order to defraud the insurer "cannot be materially less convincing than
that required in order to convict the insured of the crime of arson.”

On the claim that the burned bodegas could not have contained the quantities of
copra and hemp stated in the fire claims
29.   The basis of the claim was the testimony of the insurer’s adjuster
investigator who examined the premises during and after the fire. His
testimony was however based on inferences from photographs and traces
found after the fire, and must yield to the contradictory testimony of the
engineer, the Chief of the loan department of PNB Legazpi branch, and of
the bank appraiser, who actually saw the contents of the bodegas shortly
before the fire, while inspecting them for the mortgagee bank.

On the claim of false and fraudulent statements by the insured


30.   The trial court found that the discrepancies were a result of the insured’s
erroneous interpretation of the provisions of the insurance policies and
claim forms, caused by his imperfect knowledge of English, and that the
misstatements were innocently made and without intent to defraud.
31.   Considering that all these claims were submitted to the same agent, and that
this same agent had paid the loss caused by the 1939 fire, the Court found
no error in the trial Court's acceptance of the insured's explanation that the
omission in certain policies was due to inadvertance, for the insured could
hardly expect under such circumstances, that the 1939 would pass unnoticed
by the insurance agents.
32.   Moreover, the 20% overclaim on 70% of the hemp stock, was explained by
the insured as caused by his belief that he was entitled to include in the
claim his expected profit on the 70 per cent of the hemp, because the same
as already contracted for and sold to other parties before the fire occurred.
The insured’s overclaim of 20% in the case at bar cannot be regarded as
“more than misstatement, more than inadvertence of mistake, more than a
mere error in opinion, more than a slight exaggeration” that would entitle
the insurer to avoid the policy.
33.   The rule is that to avoid a policy, the false swearing must be willful and
with intent to defraud which was not the cause. Of course, the lack of
fraudulent intent would not authorize the collection of the expected profit
017 FILIPINAS COMPAÑIA v. CA (Saldua) FACTS:
Jan. 31, 1950 | Padilla J. | Breach of Waranty 1.   Filipinas Compania de Seguros is a domestic insurance corp. licensed to
engage in the insurance business in PH. Tan Chauco is the owner of a
PETITIONER: Filipinas Compania de Seguros
building located in Lucena (then w/in Tayabas province). Tan Chauco
RESPONDENTS: Tan Chauco
insured the building for P20,000 and P10,000 in 2 policies issued by
Filipinas Compania.
SUMMARY:
2.   On Jan. 5, 1942, during the term of said 2 policies, the building insured
Tan Chauco insured his building under 2 insurance policies with Filipinas Compania.
was burned and completely destroyed. Notice of proof & loss had been
During the term of the 2 policies, a fire occurred which completely destroyed the
duly made, but since Filipinas Compania refused to pay, Tan Chauco filed
building. Tan Chauco claimed compensation under the policies but Filipinas Compania
an action to recover on the policies.
denied alleging that there was breach of warranties of the insurance policies rendering
3.   After trial, judgment was rendered against Filipinas Compania for the
it not compensable. The breach of warranties alleged are: 1) Sealing of the building by
amount of the 2 policies w/ legal interest from the date of filing of the
the Japanese Forces changed the nature of the building insured which increased the risk
complaint. CA affirmed this.
of loss/damage violating Art. 8 of the policies; 2) The fire occurred was an indirect
4.   Meanwhile, Filipinas Compania went up to the SC raised 3 contentions:
result of the abnormal conditions at that time i.e. Japanese occupation and as per Art. 6
A.   That Art. 8 of the policies provides that if the circumstances
of the policies, since the fire was connected with the abnormal conditions, it is not
affecting the building insured (or its contents) are changed in such a
compensable; and 3) Tan Chauco cannot recover bec. he had made a fraudulent
way as to increase the risk of loss/damage by fire, the insurance
declaration in the policy in w/c he denied a previous fire while in the trial, he admitted
ceases to attach as regards the property affected; UNLESS the
that there had been a previous fire on the land on which the insured building was built,
insured obtains the permission of the company before the occurrence
which was also had been destroyed by fire from neighboring buildings, violating Art.
through an endorsement upon the policy by or on behalf of the
13. Issue is WON there was a breach of warranty. SC said NO. As to the 1st allegation,
company.
the building was actually sealed as a form of precaution since looting was rampant and
the stores were abandoned by the owners, not because the contents of the building were •   Filipinas Compania alleges that the sealing of the subject
owned by enemy nationals. The sealing alone could not have increased the risk/hazard property by the Japanese Forces on Dec. 28, 1941 (before
to which a building is exposed of. As to the 2nd, Abnormal conditions are not just the fire occurred) had the effect of changing the nature of
disorders but also situations which deviated from the normal/ordinary. In this case, it the occupation thereof in a manner which increased the risk
came in the form of the absence of a regularly organized government with its police, of loss. It allegedly resulted to its conversion into an arsenal
health & fire depts. Meanwhile, the fire actually originated from the kitchen of a of war materials that it became the bounden duty of all loyal
neighboring Panciteria. They were next to each other such that while they were forces, whether the regular USAFFE or the guerrilla
separated by a narrow alley, the roofs of both houses almost touched each other and elements to destroy, and so increasing the risk of
loss/damage to the building insured.
covered the alley. Tan Chauco has in fact admitted that the fire could have occurred
even if there was a functioning fire dept. Thus, the risk caused by the proximity of the •   Thus, in accordance w/ the provisions of Art. 8, the
building to the Panciteria is not a new one, and must have already been considered at insurance ceased to attach as of the Dec. 28, 1941.
the time of issuing the policy. The fire could have occurred just as well in times of B.   That under Art. 6, the inferential finding that the fire of Jan. 5, 1942
peace and under normal conditions. Thus, the fire occurred from causes independent of was of accidental origin, and nothing more, could not make Tan
the abnormal conditions. As to the 3rd, SC held that the previous fire is irrelevant as far Chauco’s loss compensable since the contracts of insurance between
as the subject Jan. 5, 1942 fire is concerned. Hence, there was no breach of warranty. the parties specifically required that Tan Chauco prove that the loss
happened independently of the abnormal conditions.Thus, the fire,
DOCTRINE: being an indirect result of the abnormal conditions, is not
As the words of the policy are those of the company, they should be taken most compensable.
strongly against it. Interpretation should be adopted which is most favorable to the C.   That Art. 13 provides for the grounds of forfeiture of the benefits
insured, if such interpretation be not inconsistent with the words used. under the policies among which is: If any false declaration be made
It should mean that the policy covered loss by fire occurring during the existence of or used in support thereof.
any invasion, foreign enemy, rebellion, insurrection, riot, civil commotion, •   Filipinas Compania is alleging that Tan Chauco cannot
military/usurped power, or martial law, in the general locality where the property recover because he had made fraudulent declaration in the
insured was located (so long as the fire was not connected with the said situations). insurance claim that he submitted to the former i.e. that Tan
Chauco denied that there had been a previous fire in the
premises in which he was interested, while at the trial, Tan RE. ART. 6 OF THE POLICY: THE FIRE ACTUALLY OCCURRED FROM
Chauco admitted that there had been a previous fire on the CAUSES INDEPENDENT OF THE ABNORMAL CONDITIONS AT THE
land on which the insured building was built, which had TIME AND IS THUS COMPENSABLE.
also been destroyed by fire from neighboring buildings. 1.   Abnormal Conditions, accdg. to the SC, should be interpreted as to mean “a
ISSUE/s: situation, a condition of things deviating from the normal/ordinary, and
1.   WON there was a breach of warranty in the insurance policies? = NO. produced only by war of invasion, etc. Abnormal conditions do not
First, building was actually sealed as a form of precaution since looting was necessarily imply disorders, fighting, looting, etc. The existence of a
rampant, not because the contents of the building were owned by enemy regularly organized government with its police, health & fire depts. is a sign
nationals. The sealing alone could not have increased the risk/hazard to of normalcy in a community. The absence thereof is a sign of abnormal
which a building is exposed of. The fire actually originated from the kitchen conditions.
of a neighboring Panciteria. They were next to each other such that while 2.   Thus, the fact that the Lucena gov’t. ceased to function even before the
they were separated by a narrow alley, roofs of both houses almost touched entry of the Japs, and that there was no police dept. (Jap. Soldiers kept
each other and covered the alley. Tan Chauco has in fact admitted that the peace & order), the fire dept. had not yet been organized (no equipment as
fire could have occurred even if there was a functioning fire dept.. Thus, the well except for the hose), electrical service was suspended until Feb. 3,
risk caused by the prixmity of the building to the Panciteria is not a new 1942, and that there was a curfew imposed even after the date of the fire on
one, and must have already been considered at the time of issuing the Jan 5, 1942, shows there were abnormal conditions in Lucena on and before
policy. The fire could have occurred just as well in times of peace and under the fire occurred.
normal conditions. 3.   The fire that consumed Tan Chauco’s building was actually caused by a fire
that originated in the kitchen of Perrera’s Panciteria. They were next to each
RULING: The petition for a writ of certiorari is dismissed, with costs against the other such that while they were separated by a narrow alley, the roofs of
petitioner. both houses almost touched each other and covered the alley.
4.   Tan Chauco has in fact admitted that the fire could have occurred even if
RATIO: there was a functioning fire department. Thus, the risk caused by the
RE. ART. 8 OF THE POLICY: SEALING OF THE BUILDING BY THE JAP. prixmity of the building to the Panciteria is not a new one, and must have
FORCES DID NOT CHANGE THE NATURE OF THE BUILDING INSURED already been considered at the time of issuing the policy. The fire could
AND SO IT DID INCREASE THE RISK OF LOSS/DAMAGE have occurred just as well in times of peace and under normal conditions.
1.   At the time that the Japanese forces entered Lucena on Dec. 27, 1941, said
building was closed. On Dec. 28, 1941, all the stores including the stores in RE. ART. 13 OF THE POLICY: FRADULENT CLAIM NOT RELEVANT
Tan Chauco’s building were sealed by the Japanese Forces (except those 1.   SC held that the the previous fire that Tan Chauco failed to mention is
that were open). immaterial and irrelevant so far as the subject Jan. 5, 1942 fire is
2.   SC affirmed the finding of the trial court that the reason for the sealing concerned.
of the building is a form of precaution since looting was rampant and INTERPRETATION OF THE INSURANCE POLICIES/CONTRACTS
the stores were abandoned by the owners, not because the contents of 1.   As the words of the policy are those of the company, they should be taken
the building were owned by enemy nationals. most strongly against it. Interpretation should be adopted which is most
3.   As to the supposed increase in the risk, there were only 3 possible sources favorable to the insured, if such interpretation be not inconsistent with the
of danger to which the building insured could have been exposed to either words used.
by the acts of USAFFE, guerilla or civilian saboteurs. 2.   Thus, it should mean that the policy covered loss by fire occurring during
4.   However, there was no danger. The operations between the Jap. Army and the existence of any invasion, foreign enemy, rebellion, insurrection, riot,
USAFFE Forces shifted to the fortifications around Bataan & Corregidor. civil commotion, military/usurped power, or martial law, in the general
The guerilla units also only begun organizing only after the fall of Bataan locality where the property insured was located (so long as the fire was not
in April 1942 (well after the date of the fire). There was also no acts of connected with the said situations).
sabotage by civilians in Lucena or in Tayabas after the Jap. occupied 3.   In this case, the lower court has correctly found that the loss was occasion
Lucena on Dec. 27, 1941. Thus, except for looting, there was peace & quiet by causes that are independent of the existence of any invasion, foreign
in Lucena upon the Jap. Occupation. enemy, rebellion, insurrection, etc.
5.   Hence, sealing alone could not have increased the risk/hazard to which a
building is exposed of. Besides, the sealing was an act of the enemy over
which Tan Chauco had no influence or control.
018 YOUNG V. MIDLAND TEXTILE INSURANCE (EMAR) 7. Midland Ins alleged that the fireworks were "stored" within the meaning of
March 31, 1915 | Johnson, J. | Breach of Warranty “Warranty B”.
PLAINTIFF-APPELLEE: K.S. Young 8. Young contends that under all the facts and circumstances of the case, they were
DEFENDANT-APPELLANT: The Midland Textile Insurance Company not "stored" in said building, and that placing them in the building was not a
SUMMARY: Young and Midland Insurance entered into an insurance contract violation of the terms of the contract.
where Young would pay P60 for premium and Midland Ins. promised to pay 9. Both parties agree that if they were "hazardous goods," and were "stored," then the
Young P3k, in case the residence, bodega and its contents should be destroyed by Young’s act was a violation of the terms of the contract of Ins. and the Midland Ins
fire. The insurance contract has a condition under “Warranty B” that “during the was justified in repudiating its liability.
pendency of the policy, no hazardous goods [shall be] stored in the building”. 10. Young sought to recover P3k upon the policy.
Young was given 3 boxes of fireworks, intended to be used for the Chinese New 11. The lower court rendered a judgment in favor of the Young; against Midland Ins
Year but the city authorities prohibited the use of such so Young kept them in the for P2,708.78, and costs.
insured premises. The insured residence, bodega and contents were partly 12. Midland Ins appealed to this court.
destroyed by fire but the part where the fireworks were kept was not destroyed.
The issue in this case is WON placing of said fireworks in the building insured is ISSUE: WON the placing of the fireworks in the building insured, under the
a violation of the terms of the insurance contract – YES. The "hazardous goods" conditions enumerated, they being "hazardous goods," is a violation of the terms of
in question were "stored" in the bodega, as that word is generally defined. the contract of Ins. and especially of "warranty B." – YES. The "hazardous goods"
Suppose Midland Ins made an examination of the premises, even in the absence of were "stored" in the bodega, as that word is generally defined.
a fire, and had found he "hazardous goods" there, under the conditions above
described, it would be justified in declaring the policy null and of no effect by RULING: Judgment of the lower court is revoked and Midland Ins. is relieved from
reason of Young’s violation of its terms. Young paid a premium based upon the any responsibility under said complaint, and, without any finding as to costs.
risk at the time the policy was issued. Placing of the firecrackers in the building
insured increased the risk. Young did not pay a premium based upon the increased RATIO:
risk, neither had Midland Ins issued a policy upon the theory of a different risk. 1.   WON a particular article is "stored" must depend upon the intention of the
DOCTRINE: If the "warranty" is a term of the contract, its violation causes a parties.
breach and justify noncompliance or a repudiation. 2.   Most cases cited by the lower court are cases where the article was being put to
FACTS: some reasonable and actual use, which might easily have been permitted by
1. Young has a candy and fruit store in Manila, and occupied a building at Calle the terms of the policy, and within the intent of the parties, and excepted
Claveria, as a residence and bodega (storehouse). from the operation of the warranty, like the present. Said decision are upon cases
2. May 1912: The Midland Textile Insurance Company (Midland Ins), in like:
consideration of the payment of a premium of P60, entered into a contract of Ins. a. Where merchants have had or kept the "hazardous" articles in small
with the plaintiff (policy No. 509105) where Midland Ins. promised to pay Young quantities, and for actual daily use, for safe, such as gasoline, gunpowder, etc.;
P3k, in case the residence, bodega and contents should be destroyed by fire. b. Where such articles have been brought on the premises for actual use thereon,
3. A condition of the insurance contract states: "Waranty B. — It is agreed that and in small quantities, such as oil, paints, etc; and
during the pendency of this policy no hazardous goods stored or kept for sale, and c. Where such articles or goods were used for lighting purpose, and in small
no hazardous trade or process be carried on, in the building to which this Ins. applies, quantities.
or in any building connected therewith." 3.   Century and Standard Dictionaries define “Store”: to be a deposit in a store or
4. Feb 1913: Young was given fireworks, intended to be used for the Chinese New warehouse for preservation/safe keeping; place in a warehouse/other place of
Year but since City authorities prohibited its use, the 3 boxes of fireworks were deposit for safe keeping.
placed in the residence and bodega, in order that Young might later send them to a 4.   The definitions do not include a deposit in a store, in small quantities, for daily
friend in the provinces. use. "Daily use" precludes the idea of a deposit for preservation or safe keeping,
5. March 1913: The residence, bodega and its contents were partially destroyed by as well as a deposit for future consumption, or safe keeping.
fire but the part of the building 5.   In this case, no claim is made that the "hazardous goods" were placed in the
6. Young and Midland Ins agree that fireworks come within the phrase "hazardous bodega for present/daily use. They were placed in the bodega for sake keeping,
goods," mentioned in "warranty B" of the policy and that fireworks were found in a future use or consumption. Young did not claim that he deposited them there
part of the building not destroyed by the fire; that they did not contribute to the fire, with any other idea than "for future use" — for future consumption.
or to the loss occasioned thereby.
6.   The "hazardous goods" in question were "stored" in the bodega, as that word is the terms of the contract. The violation of the terms of the contract, by virtue
generally defined. of the provisions of the policy itself, terminated, at the election of either party,
7.   If Midland Ins examined the premises, even in the absence of a fire, and found he contractual relations. (Kyte vs. Commercial Union Assurance)
"hazardous goods" there, under the conditions above described, it would be 15.   Young paid a premium based upon the risk at the time the policy was issued.
justified, then and there, in declaring the policy null and of no effect because of Placing of the firecrackers in the building insured increased the risk, for which
Young’s violation of its terms. Young did not pay a premium based upon and, neither had Midland Ins issued a
8.   Midland Ins may repudiate is liability, even after the fire. If the "warranty" is a policy upon the theory of a different risk.
term of the contract, its violation causes a breach and justify 16.   Young was enjoying, if his contention may be allowed may be allowed, the
noncompliance/repudiation. benefits of an insurance policy upon one risk, whereas, as a matter of fact, it was
9.   Insurance Contracts are contracts of indemnity upon the terms and conditions issued upon an entirely different risk.
specified in the policy. The parties have a right to impose reasonable conditions 17.   Midland Ins had neither been paid nor had issued a policy to cover the increased
at the time of the making of the contract as they may deem wise and necessary. risk.
10.   The rate of premium is measured by the character of the risk assumed. The 18.   A substantial increase of risk and which is continued for a considerable period
insurance company, for a comparatively small consideration, undertakes to of time, is a direct and certain injury to the insurer, and changes the basis upon
guarantee the insured against loss or damage, upon the terms and conditions which the insurance contract rests. (Kyte vs. Commercial Union Assurance Co;
agreed upon, and upon no other, and when called upon to pay, in case of loss, Frost's Detroit Lumber vs. Millers' Mutual Ins. Co; Moore vs. Phoenix Ins.;
the insurer, therefore, may justly insist upon a fulfillment of these terms. Ferree vs. Oxford Fire & Life Ins.)
11.   If the insured cannot bring himself within the conditions of the policy, he is
not entitled to recover for the loss. Terms of the policy constitute the measure
of the insurer's liability, and in order to recover the insured must show himself
within those terms; and if it appears that the contract has been terminated by a
violation, on the part of the insured, of its conditions, then there can be no right
of recovery.
12.   Insured’s compliance with the terms of the contract is a condition precedent to
the right of recovery. If the insured has violated or failed to perform the
conditions of the contract, and such a violation or want of performance has
not been waived by the insurer, then the insured cannot recover. Courts are
not permitted to make contracts for the parties. The function and duty of the
courts consist simply in enforcing and carrying out he contracts actually made.
13.   GR: Insurance contracts of are construed most favorably to the insured, yet
contracts of Ins., like other contracts, are to be construed according to the sense
and meaning of the terms which the parties themselves have used. If such terms
are clear and unambiguous they must be taken and understood in their
plain, ordinary and popular sense. (Imperial Fire Ins. v. County of Coos; Kyte
v. Commercial Union Assurance) Conditions of insurance contracts, when
plainly expressed in a policy, are binding upon the parties and should be
enforced by the courts, if the evidence brings the case clearly within their
meaning and intent. It tends to bring the law itself into disrepute when, by astute
and subtle distinctions, a plain case is attempted to be taken without the
operation of a clear, reasonable, and material obligation of the contract. (Mack v.
Rochester German Ins.)
14.   Young argues that in view of the fact that the "storing" of the fireworks on the
premises of the insured did not contribute in any way to the damage occasioned
by the fire, he should be permitted to recover — that the "storing" of the
"hazardous goods" in no way caused injury to the defendant company. That
argument, however, is beside the question, if the "storing" was a violation of
001 Saura Import v. Phil International (Sarmiento) FACTS:
May 31, 1963 | Paredes J. | Cancellation
1.   the Saura Import & Expert Co., Inc., mortgaged to the Phil. National Bank,
PETITIONER: Saura Import and Export Co. Inc a parcel of land to secure the payment of a promissory note of P27,000.00
a.   the mortgage was amended to guarantee an increased amount,
RESPONDENTS: Philippine International Surety bringing the total mortgaged debt to P37,000.00
b.   The provisions of the mortgage contract, pertinent to the resolution
SUMMARY: Saura mortgaged to PNB a parcel of land to secure a payment of of the present case, provide as follows that Saura shall insure the
the loan. There was a building erected on the property, and it was agreed that the mortgage property at all times against fire and earthquake for an
Saura must insure the property against fire and earthquake. Thus, Saura insured amount and with such company satisfactory to the mortgagee,
it with Philippines International Surety for a a period of one year from Oct.2, indorsing to the latter the corresponding policies; he shall keep the
1954. Then the policy was endorsed to PNB. However, 13 days after the mortgaged property in good condition, making repairs and
issuance of the policy, the insured cancelled the policy. Notice of cancellation protecting walls that may be necessary;
was given to the bank. Then on April 6, 1955, the building and its contents were
burned. Saura filed a claim against the Insurer and mortgagee bank. It was only 2.   Erected on the land mortgaged, was a building of strong materials owned by
then that it was discovered that the policy had been previously cancelled. The the mortgagor Saura Import & Export Co., Inc., which had always been
trial court dismissed the complaint. Insurance policies provide a provision on covered by insurance, many years prior to the mortgage contract.
cancellation of policy by the insurer upon prior notice. Where the policy contains
no provisions that a certain number of days notice shall be given, a reasonable 3.   Pursuant to the requirement, Saura insured the building and its contents
notice and opportunity to obtain other insurance must be given. The policy in with the Philippines International Surety, an insurance firm acceptable to
question, does not provide for the notice, its form or period. This being the case, mortgagee Bank, for P29,000.00 against fire for the period of one year from
it devolves upon the Court to apply the generally accepted principles of October 2, 1954. As required therefor, the insurance policy was endorsed to
insurance, regarding cancellation of the insurance policy by the insurer. Actual the mortgagee PNB, in a Memo which states that loss if any, is payable to
notice of cancellation in a clear and unequivocal manner, preferably in writing, Philippine National Bank as their interest may appear, subject to the terms,
in view of the importance of an insurance contract, should be given by the conditions and warranties of this policy.
insurer to the insured, so that the latter might be given an opportunity to obtain
other insurance for his own protection. The notice should be personal to the 4.   The policy was delivered to the mortgagee Bank by Saura.
insured and not to and/or through any unauthorized person by the policy. In this a.   On October 15, 1954, barely thirteen (13) days after the issuance
case, It was the primary duty of the insurance company to notify the insured, but of the fire insurance policy (October 2, 1954), the insurer cancelled
it did not. The house and its contents were burned on April 6, 1955, at the time the same, effective as of the date of issue.
when the policy was enforced, and it is evident that both the insurance company b.   Notice of the cancellation was given to appellee Bank in writing,
and the appellee bank failed, to notify the insured appellant Saura of the sent by Registered Mail and was received by the Bank on
cancellation made. While the insurance company gave notice to the mortgagee, it November 8,1954.
was not enough to constitute notice to Saura. Thus, as there is ineffective c.   On April 6, 1955, the building and its contents, worth P40,685.69
cancellation and a loss occurred while the policy was in effect, insurance were burned.
company shall pay the insured.
5.   Saura filed a claim with the Insurer and mortgagee Bank.
DOCTRINE: If a mortgage or lien exists against the property insured, and the a.   Upon the presentation of notice of loss with the PNB, Saura
policy contains a clause stating that loss, if any, shall be payable to such learned for the first time that the policy had previously been
mortgagee or the holder of such lien as his interest may appear, notice of cancelled on October 2, 1954, by the Insurer, when Saura's folder
cancellation to the mortgagee or lien holder alone is ineffective as a cancellation in the Bank's file was opened and the notice of cancellation
of the policy as to the owner of the property. Therefore, the liability of the (original and duplicate) sent by the Insurer to the Bank, was found.
insurance company becomes a fact.
6.   Upon refusal of the Insurer Philippines International Surety to pay the
amount of the Insurance, Civil Case No. 26847 was filed with the Manila
CFI against the Insurer, and the PNB was later included as party defendant,
after it had refused to prosecute the case jointly with Saura Import & Export to a cancellation of the policy by the insurer, and consequently a
Co., Inc. letter containing notice of cancellation which is mailed by the
insurer but not received by the insured, is ineffective as
7.   Trial Court: Complaint dismissed. neither the insurer or the mortgagee cancellation
bank informed the plaintiff Saura of the cancellation of the policy.
2.   The policy in question, does not provide for the notice, its form or period.
ISSUE: Whether or not proper notice of cancellation was effected by the The insurance Law, Act No. 2427, does not likewise provide for such
insurance company—NO. The notice was not given to the mortgagor but to the notice.
mortgagee, which should not be the case. a.   This being the case, it devolves upon the Court to apply the
generally accepted principles of insurance, regarding cancellation
RULING: of the insurance policy by the insurer.
WHEREFORE, the decision appealed from is hereby REVERSED, and b.   Actual notice of cancellation in a clear and unequivocal manner,
another is entered, condemning the defendant-appellee Philippines International preferably in writing, in view of the importance of an insurance
Surety Co., Inc., to pay Saura Import & Export Co., Inc., appellant herein, the sum of contract, should be given by the insurer to the insured, so that the
P29,000.00, the amount involved in Policy No. 429, subject-matter of the instant latter might be given an opportunity to obtain other insurance for
case. Without costs. his own protection. The notice should be personal to the insured
and not to and/or through any unauthorized person by the policy.

RATIO: 3.   In the case at bar, the defendant insurance company, must have realized the
paramount importance of sending a notice of cancellation, when it sent the
1.   Fire insurance policies and other contracts of insurance upon property, in notice of cancellation of the policy to the defendant bank (as mortgagee),
addition to the common provision for cancellation of the policy upon but not to the insured with which it (insurance company) had direct dealing.
request of the insured, generally provide for cancellation by the insurer by a.   It was the primary duty of the defendant-appellee insurance
notice to the insured for a prescribed period, which is usually 5 days, and company to notify the insured, but it did not.
the return of the unearned portion of the premium paid by the insured, such b.   It should be stated that the house and its contents were burned on
provision for cancellation upon notice being authorized by statutes in some April 6, 1955, at the time when the policy was enforced (October
jurisdiction, either specifically or as a provision of an adopted standard 2, 1954 to October 2, 1955); and it is evident that both the
form of policy. insurance company and the appellee bank failed, to notify the
a.   The purpose of provisions or stipulations for notice to the insured, insured appellant Saura of the cancellation made.
is to prevent the cancellation of the policy, without allowing the
insured ample opportunity to negotiate for other insurance in its 4.   Of course, the defendant insurance company contends that it gave notice to
stead. the defendant-appellee bank as mortgagee of the property, and that was
b.   The form and sufficiency of a notice of cancellation is determined already a substantial compliance with its duty to notify the insured of the
by policy provisions. cancellation of the policy.
c.   In order to form the basis of the cancellation of a policy, notice to a.   But notice to the bank, as far as appellant herein is concerned, is
the insured need not be in any particular form, in the absence of a not effective notice.
statute or policy provision prescribing such form, and it is
sufficient, so long as it positively and unequivocally indicates to 5.   "If a mortgage or lien exists against the property insured, and the policy
the insured, that it is the intention of the company that the policy contains a clause stating that loss, if any, shall be payable to such mortgagee
shall cease to be binding. or the holder of such lien as his interest may appear, notice of cancellation
d.   Where the policy contains no provisions that a certain number of to the mortgagee or lien holder alone is ineffective as a cancellation of the
days notice shall be given, a reasonable notice and opportunity to policy as to the owner of the property."
obtain other insurance must be given. Actual personal notice to the a.   therefore, the liability of the insurance company becomes a fact.
insured is essential to a cancellation under a provision for
cancellation by notice. The actual receipt by the insured of a notice 6.   It may be argued that, no error has been assigned against the insurance
of cancellation is universally recognized as a condition precedent company and no prayer is found therein asking that it be made liable.
a.   It must be noted, however, that the case was dismissed by the
lower court and the main object of the appeal is to secure a reversal
of the said judgment. This Court is clothed with ample authority to
review matters, even if they are not assigned as errors in the
appeal, if it finds that their consideration is necessary in arriving at
a just decision of the case.

7.   Although assigned errors apparently appear to be directed against the


appellee bank alone, they in essence, seek a reversal of the decision on
dismissal, entered by the lower court, which in the main has for its purpose
the finding of liability on the policy.
a.   In the course of our examination of the records of the case, court
found that liability attached principally upon the insurance
company, for its failure to give notice of the cancellation of the
policy to herein appellant itself.
002 MALAYAN INSURANCE INC. v. CRUZ ARNALDO (SEE) 4.   It must state (a) which of the grounds mentioned in Section 64 is relied upon
October 12, 1987|Cruz, J. | Right to rescind or cancel and (b) that upon written request of the insured, the insurer will furnish the facts
on which the cancellation is based.
PETITIONER: Malayan Insurance Inc.
RESPONDENTS: Gregoria Cruz Arnaldo, in her capacity as Insurance FACTS:
Commissioner and Coronacion Pinca 26.   On June 7, 1981, Malayan Insurance Inc. (MICO) issued to Coronacion
Pinca, a Fire Insurance Policy on her property for the amount of P14,000
SUMMARY: Pinca insured her house with Malayan Insurance (MICO) on June effective July 22, 1981 until July 22, 1982.
22, 1981. Mico allegedly cancelled the policy and sent notice on Oct 15, 1981 27.   On October 15, 1981, MICO allegedly cancelled the policy for non-
for non payment. Pinca then paid the premium to Adora, agent of MICO on Dec. payment of the premium and send the notice to Pinca
24, 1981 which was remitted to MICO on January 15. On January 18, 1982, the 28.   On December 24, 1981, Pinca paid the premium to Domingo Adora, agent
house was burned. On Feb. 5, 1982 MICO returned the premiums to Adora on of MICO.
the ground that the policy was already cancelled early on but Adora refused to 29.   On January 15, 1982, Adora remitted the payment to MICO, together with
accept the payment. Pinca then claimed on the policy which MICO rejected. other payments.
(read facts #9-11 for procedural events). The issue in this case is WoN the 30.   On January 18, the insured property was completely burned.
contract was validly rescinded by the alleged cancellation sent by MICO to 31.   On February 5, 1982, Pinca’s payment was returned by MICO to Adora on
Pinca.—No, it wasn’t validly cancelled. MICO banks on Sec 77 which states that the ground that the policy had been cancelled early on but Adora refused to
no premiums paid=no insurance contract but such provision is not applicable accept it.
since premiums were actually paid here. We do not share MICO's view that there 32.   Pinca then made the demand for payment of the insurance proceeds but
was no existing insurance at the time of the loss sustained by Pinca because her MICO rejected her claims on the ground that the policy was cancelled.
policy never became effective for non-payment of premium. Payment was in fact 33.   Pinca then went to the Insurance Commission where she won. So MICO
made, rendering the policy operative as of June 22, 1981, and removing it from brought the case to the courts.
the provisions of Sec 77. Thereafter, the policy could be cancelled on any of the 34.   Insurance Commission decided the case on April 5, 1982. Notice of which
supervening grounds enumerated in Article 64 (except "nonpayment of was received by MICO on April 10, 1982. It then filed an MR on April 25
premium") provided the cancellation was made in accordance therewith and with which was denied on June 4. Notice of denial was received by MICO on
Article 65. (insert doctrine). In this case, there is no proof that the notice, June 13, 1982. The present case was filed with the SC on July 2, 1982.
assuming it complied with the other requisites mentioned, was actually mailed to 35.   MICO claims that it has 30 days within which to appeal via certiorari under
and received by Pinca. All MICO offers to show that the cancellation was Sec. 416 of the Insurance Code. Alternatively, it invokes Rule 45 of the
communicated to the insured is its employee's testimony that the said ROC.
cancellation was sent "by mail through our mailing section." without more. On 36.   Arnaldo, as Insurance Commissioner and Pinca, insist that the applicable
the other hand, there is the flat denial of Pinca, who says she never received the law is BP 129 which provides for a 15 day period for appeal.
claimed cancellation and who, of course, did not have to prove such denial
considering the strict language of Section 64 that no insurance policy shall be ISSUE/s:
cancelled except upon prior notice, it behooved MICO to make sure that the 7.   WoN the case was filed out of time.—Yes. It doesn't matter which law was
cancellation was actually sent to and received by the insured. The presumption applied, under all 3, Sec 416 of the Insurance Code, Rule 45 of the ROC,
cited is unavailing against the positive duty enjoined by Section 64 upon MICO and BP 129, the case was filed out of time.
and the flat denial made by Pinca that she had received notice of the claimed 8.   WoN the insurance contract was validly cancelled.—No, there is no proof
cancellation. that the notice, assuming it complied with the other requisites mentioned
above, was actually mailed to and received by Pinca.
DOCTRINE: A valid cancellation must, therefore, require concurrence of the
following conditions: RULING: WHEREFORE, the petition is DENIED. The decision of the Insurance
1.   There must be prior notice of cancellation to the insured; Commission dated April 10, 1981, and its Order of June 4, 1981, are AFFIRMED in
2.   The notice must be based on the occurrence, after the effective date of the full, with costs against the petitioner. This decision is immediately executory.
policy, of one or more of the grounds mentioned;
3.   The notice must be (a) in writing, (b) mailed, or delivered to the named insured, RATIO:
(c) at the address shown in the policy; The appeal was filed out of time
1.   The pivotal date is the date the notice of denial of the MR was received by 2.   MICO banks on Sec 77 of the Insurance Code which states that:
MICO. a.   SEC. 77. An insurer is entitled to payment of the premium as soon as the
2.   MICO claims that it received the notice on June 18, 1982 and offered as thing is exposed to the peril insured against. Notwithstanding any
evidence a copy of the Order on June 14 with a rubber-stamped notation on agreement to the contrary, no policy or contract of insurance issued by an
the upper left hand corner that it was received on June 18, 1982 without insurance company is valid and binding unless and until the premium
thereof has been paid, except in the case of a life or an industrial life
indication from whom it was received. At the bottom of the copy, there is
policy whenever the grace period provision applies.
another signature under which “6-13-82” was written. 3.   The above provision is not applicable because payment of the premium
3.   Pinca presented an authenticated copy of the same Order with a rubber- was in fact eventually made in this case. Notably, the premium invoice
stamped notation at the bottom indicating that it was received for MICO by issued to Pinca at the time of the delivery of the policy on June 7, 1981 was
J. Gotladera on “6-13-82” stamped "Payment Received" of the amoung of P930.60 on "12-24-81" by
4.   Between the two dates, the court chooses to believe June 13, 1982, not only Domingo Adora. This is important because it suggests an understanding
because the numbers "6-13-82" appear on both annexes but also because it between MICO and the insured that such payment could be made later, as
is the date authenticated by the administrative division of the Insurance agent Adora had assured Pinca. In any event, it is not denied that this
Commission. The document presented by MICO is at worst self-serving; at payment was actually made by Pinca to Adora, who remitted the same to
best, it might only indicate that it was received on June 18, 1982, by the MICO.
legal department of MICO, after it had been received earlier by some other 4.   MICO’s theory is that the policy became effective only upon payment
of its personnel on June 13, 1982. Whatever the reason for the delay in which was on December 24, 1981 and valid only up to July 22, 1982. So the
transmitting it to the legal department need not detain us here. policy would only run for 8 months despite payment of a year’s worth of
5.   Under Section 416 of the Insurance Code, the period for appeal is thirty premiums.
days from notice of the decision of the Insurance Commission. MICO filed 5.   MICO also claims that Adora was not authorized to receive the premium
its motion for reconsideration on April 25, 1981, or fifteen days such notice, payment on its behalf but at the same time, MICO acknowledges Adora as
and the reglementary period began to run again after June 13, 1981, date of its agent. This acknowledgment of Adora as an agent defeats MICO’s claim
its receipt of notice of the denial of the said motion for reconsideration. As that Adora was not authorized to receive the premiums. It is a well known
the herein petition was filed on July 2, 1981, or nineteen days later, there is principle under the law of agency that payment to an agent has the same
no question that it is tardy by four days. effect as payment to the principal himself.
6.   Counted from June 13, the fifteen-day period prescribed under Rule 45, 6.   There is the MICO’s argument, however, that Adora was not authorized to
assuming it is applicable, would end on June 28, 1982, or also four accept the premium payment because six months had elapsed since the
days from July 2, when the petition was filed. issuance by the policy itself. It is argued that this prohibition was binding
7.   If it was filed under B.P. 129, then, considering that the motion for upon Pinca, who made the payment to Adora at her own riskl as she was
reconsideration was filed on the fifteenth day after MICO received notice of bound to first check his authority to receive it.
the decision, only one more day would have remained for it to appeal, to 7.   MICO is taking an inconsistent stand. While contending that acceptance of
wit, June 14, 1982. That would make the petition eighteen days late by July the premium payment was prohibited by the policy, it at the same time
2. insists that the policy never came into force because the premium had not
8.   Indeed, even if the applicable law were still R.A. 5434, governing appeals been paid. One surely, cannot have his cake and eat it too.
from administrative bodies, the petition would still be tardy. The law 8.   We do not share MICO's view that there was no existing insurance at the
provides for a fixed period of ten days from notice of the denial of a time of the loss sustained by Pinca because her policy never became
seasonable motion for reconsideration within which to appeal from the effective for non-payment of premium. Payment was in fact made,
decision. Accordingly, that ten-day period, counted from June 13, 1982, rendering the policy operative as of June 22, 1981, and removing it from the
would have ended on June 23, 1982, making the petition filed on July 2, provisions of Article 77, Thereafter, the policy could be cancelled on any of
1982, nine days late. the supervening grounds enumerated in Article 6437 (except "nonpayment of
9.   Whichever law is applicable, therefore, the petition can and should be
dismissed for late filing.                                                                                                                        
37
SEC. 64. No policy of insurance other than life shall be cancelled by the insurer except upon prior
The insurance contract WAS NOT validly cancelled (important) notice thereof to the insured, and no notice of cancellation shall be effective unless it is based on the
1.   MICO contends that the contract was cancelled before the fire because there occurrence, after the effective date of the policy, of one or more of the following: (a) non-payment of
was no payment of premiums and that the claim was without proof of loss. premium; (b) conviction of a crime arising out of acts increasing the hazard insured against; (c) discovery
of fraud or material misrepresentation; (d) discovery of willful, or reckless acts or commissions increasing
premium") provided the cancellation was made in accordance therewith and was willing to make her payment retroact to July 22, 1981, its stipulated
with Article 6538. commencement date. After all, agent Adora was very accomodating and had
9.   A valid cancellation must, therefore, require concurrence of the following earlier told her "to call him up any time" she was ready with her payment on
conditions: the policy earlier issued. She was obviously only reciprocating in kind when
a.   There must be prior notice of cancellation to the insured; she paid her premium for the period beginning July 22, 1981, and not
b.   The notice must be based on the occurrence, after the effective date of the December 24, 1981.
policy, of one or more of the grounds mentioned; 13.   MICO claims that Pinca knew of the cancellation but a close study of the
c.   The notice must be (a) in writing, (b) mailed, or delivered to the named transcript will show that Pinca meant to renew the policy if it had already
insured, (c) at the address shown in the policy;
been cancelled but not it if was still effective. It was all conditional. As it
d.   It must state (a) which of the grounds mentioned in Section 64 is relied
upon and (b) that upon written request of the insured, the insurer will has not been shown that there was a valid cancellation of the policy, there
furnish the facts on which the cancellation is based. was no need to renew it but merely to pay the premium. Payment was thus
10.   MICO claims it cancelled the policy in question on October 15, 1981, for made on the original transaction.
non-payment of premium. To support this assertion, it presented one of its
employees, who testified that "the original of the endorsement and credit Valuation of the loss
memo" — presumably meaning the alleged cancellation — "were sent the 14.   The last point raised by MICO should not pose much difficulty. The
assured by mail through our mailing section" However, there is no proof valuation fixed in fire insurance policy is conclusive in case of total loss in
that the notice, assuming it complied with the other requisites the absence of fraud, which is not shown here. Loss and its amount may be
mentioned above, was actually mailed to and received by Pinca. All determined on the basis of such proof as may be offered by the insured,
MICO offers to show that the cancellation was communicated to the insured which need not be of such persuasiveness as is required in judicial
is its employee's testimony that the said cancellation was sent "by mail proceedings. If, as in this case, the insured files notice and preliminary
through our mailing section." without more. MICO then says that its "stand proof of loss and the insurer fails to specify to the former all the defects
is enervated (sic) by the legal presumption of regularity and due thereof and without unnecessary delay, all objections to notice and proof of
performance of duty." (not realizing perhaps that "enervated" means loss are deemed waived under Section 90 of the Insurance Code.
"debilitated" not "strengthened"). 15.   The certification issued by the Integrated National Police, Lao-ang, Samar,
11.   On the other hand, there is the flat denial of Pinca, who says she never as to the extent of Pinca's loss should be considered sufficient. Notably,
received the claimed cancellation and who, of course, did not have to MICO submitted no evidence to the contrary nor did it even question the
prove such denial considering the strict language of Section 64 that no extent of the loss in its answer before the Insurance Commission. It is also
insurance policy shall be cancelled except upon prior notice, it worth observing that Pinca's property was not the only building bumed in
behooved MICO to make sure that the cancellation was actually sent to the fire that razed the commercial district of Lao-ang, Samar, on January 18,
and received by the insured. The presumption cited is unavailing against 1982.
the positive duty enjoined by Section 64 upon MICO and the flat denial 16.   There is nothing in the Insurance Code that makes the participation of an
made by Pinca that she had received notice of the claimed cancellation. adjuster in the assessment of the loss imperative or indespensable, as MICO
12.   It stands to reason that if Pinca had really received the said notice, she suggests. Section 325, which it cites, simply speaks of the licensing and
would not have made payment on the original policy on December 24, duties of adjusters.
1981. Instead, she would have asked for a new insurance, effective on that
date and until one year later, and so taken advantage of the extended period.
The Court finds that if she did pay on that date, it was because she honestly
believed that the policy issued on June 7, 1981, was still in effect and she
                                                                                                                                                                                                                                                                                       
the hazard insured against; (e) physical changes in the property insured which result in the property
becoming uninsurable;or (f) a determination by the Commissioner that the continuation of the policy
would violate or would place the insurer in violation of this Code.

38
  SEC.   65.   All   notices   of   cancellation   mentioned   in   the   preceding   section   shall   be   in   writing,   mailed   or  
delivered   to   the   named   insured   at   the   address   shown   in   the   policy,   and   shall   state   (a)   which   of   the  
grounds   set   forth   in   section   sixty-­‐four   is   relied   upon   and   (b)   that,   upon   written   request   of   the   named  
insured,  the  insurer  will  furnish  the  facts  on  which  the  cancellation  is  based.  
003 Areola v. CA (COSCOLLUELA revised by SIAPNO) for both insurer and insured Article 1191.
September 22, 1994 | Romero, J. | Right to Rescind

PETITIONER: Santos B. Areola and Lydia D. Areola FACTS:


RESPONDENTS: Court of Appeals, Prudential Guarantee and Assurance, Inc. 11.   Santos Areola (Areola), a lawyer from Dagupan bought from Prudential
Guarantee and Assurance, Inc. (Prudential) a personal accident insurance
SUMMARY: 7 months after the issuance of Santos Areola’s Personal Accident policy covering the one-year period between noon of November 28, 1984
Insurance Policy, Prudential unilaterally cancelled the same since company records and noon of November 28, 1985.
revealed that Areola failed to pay his premiums evidenced by the lack of Official 12.   Areola was allegedly not able to pay the total amount of P1,609.65 which
Receipt which was supposed to be issued within 7 days after the Provisional included the premium, documentary stamp and premium tax.
Receipt. Shocked by the cancellation of the policy, Santos approached Carlito Ang, 13.   On December 17, 1984, Prudential issued a collector’s provisional receipt
agent of Prudential and demanded the issuance of an official receipt. Ang told to Areola for the total amount. It noted thereon that the collector’s
Santos that it was a mistake and assured its rectification. Santos demanded the same provisional receipt will be confirmed by their official receipt.
terms and same rate increase as when he paid the provincial receipt but Malapit 14.   The Branch Manager of Baguio City sent Areola an Endorsement which
insisted that the partial payment he made was exhausted and that he should pay the “cancelled flat” his insurance policy for non-payment of premium. The
balance or his policy will cease to operate. Assistant Vice-President Ampil III same credited “a return premium of P1,609.65 plus documentary stamps
apologized but Areola still filed a complaint for breach of contract with damages and premium tax” to the account of Areola.
before the lower court. Areola received through Carlito Ang the leeter of Assistant 15.   Shocked, Areola confronted the representative of Prudential, Carlito Ang.
Vice-President Mariano M. Ampil III finding error on their part since premiums Areola still failed to receive any official receipt from Prudential
were not remitted Malapit, proposed to extend its lifetime. The RTC favored Areola 16.   Areola sent a letter demanding from Prudential that he be insured under the
stating that Prudential was in bad faith in the cancellation of the policy. The CA same terms and conditions of his previous policy commencing upon the
reversed the decision of the RTC stating that the cancellation was not motivated by receipt of his letter, or that the current commercial rate of increase on the
bad faith. payment he had made under the provisional receipt be return within 5 days.
17.   Areola received a letter from production manager Malapit, informing him of
The issue is WoN the Areolas can file against damages despite the effort to rectify the “partial payment” of P1,000 he had made on the policy had been
the cancellation. SC held YES. The decision of the RTC was correct. Malapit's exhausted pursuant to the provisions of the Short Period Rate Scale. He was
fraudulent act of misappropriating the premiums paid is beyond doubt directly also warned that failure to pay the balance, the company’s liability would
imputable to Prudential. Malapit’s receipt of said premiums is receipt by private cease to operate.
respondent insurance company who, by provision of law, under Article 1910 of the 18.   Prudential, in response to Areola’s letter, wrote a letter to the latter stating
Civil Code, is bound by the acts of its agent. “Art. 1910. The principal must comply that the company was verifying whether the payment had in fact been
with all the obligations which the agent may have contracted within the scope of his remitted to the company and why no official receipt had been issued
authority.” As for any obligation wherein the agent has exceeded his power, the therefor. Asst. Vice-President Mariano Ampil III emphasized that the OR
principal is not bound except when he ratifies it expressly or tacitly. Subsequent should have been issued 7 days from the issuance of the provisional receipt,
reinstatement could not possibly absolve Prudential there being an obvious breach and because there was no such OR, there was reason to believe that no
of contract a contract of insurance creates reciprocal obligations for both insurer payment had been made.
and insured Article 1191. There is a choice between fulfillment or rescission of the 1.   Ampil apologized for the inconvenience and said they will hold
obligation in case one of the obligors fails to comply with what is incumbent upon Areola covered under the terms of the policy until such time that
him. The circumstances entitle the injured party to payment of damages, regardless the matter is cleared.
of whether he demands fulfillment or rescission of the obligation. Nominal 2.   Ampil wrote another letter confirming the total amount was
damages are "recoverable where a legal right is technically violated and must be received and that Prudential was amenable to extending the policy
vindicated against an invasion that has produced no actual present loss of any kind, up to December 17, 1985.
or where there has been a breach of contract and no substantial injury or actual 19.   Unfortunately, as early as August 6, 1985 Areola and his wife already filed
damages whatsoever have been or can be shown. a breach of contract with damages.
20.   Prudential admitted that the cancellation was due to Malapit’s failure to turn
DOCTRINE: Subsequent reinstatement could not absolve the insurer there being over the premiums. However, it argued that by acknowledging the
an obvious breach of contract a contract of insurance creates reciprocal obligations
inconvenience and taking steps to rectify its omission, Prudential had 15.   It is beyond doubt that Malapit represented the interests of Prudential and
complied with its obligation. Hence, Areola has no longer a cause of action. acted in its behalf. His act of receiving the premiums collected is well
21.   Trial court ruled in favor of Areola ruling that Prudential was in bad faith in within the provice of his authority.
unilaterally cancelling the insurance policy. Had Areola met an accident, 16.   Malapit’s failure to remit the premiums he received cannot constitute as a
Prudential would certainly have disclaimed liability. It held that Prudential defense for Prudential. no exoneration from liability could result therefrom.
breached its contract with Areola. 17.   The fact that Prudential was itself defrauded due to the anomalies that took
22.   CA reversed the trial court. It held that Prudential was not motivated by bad place in its Baguio branch office, such as the non-accrual of said premiums
faith or malice and that the cancellation was based on what the existing to its account, does not free the same from its obligation to Areola.
records merely showed. In fact, it’s subsequent acts of rectifying the wrong Consequently, Prudential is liable by way of damages for the fraudulent acts
indicate that Prudential did not ct precipitately or willfully to inflict a wrong committed by Malapit that gave occasion to the erroneous cancellation of
on Areola. subject insurance policy.
23.   Hence, this appeal. 18.   Its earlier act of reinstating the insurance policy also can not obliterate the
24.   Areola argues that it was Malapit’s misappropriation of the premiums injury. Prudential should be reminded that a contract of insurance
which was the proximate cause of the cancellation of the insurance policy. creates reciprocal obligations for both insurer and insured. Reciprocal
Malapit’s act of signing and even sending the notice of cancellation himself, obligations are those which arise from the same cause and in which
notwithstanding his personal knowledge of Areola’s full payment of each party is both a debtor and a creditor of the other, such that the
premiums, further reinforces the allegation of bad faith. Subsequent obligation of one is dependent upon the obligation of the other.
reinstatement could not possibly absolve Prudential from liability, there 19.   Under the law governing reciprocal obligations the injured party is given a
being an obvious breach of contract. choice between fulfillment or rescission of the obligation. The law also
25.   Prudential however argues that where reinstatement, the equitable relief entitles Areola to payment of damages, regardless of whether he demands
sought by Areola was granted at an opportune moment i.e. prior to the filing fulfillment or rescission.
of the complaint, he was left without a cause of action 20.   Under the circumstances of the instant case, the relationship as creditor and
debtor between the parties arose from a common cause: i.e., by reason of
ISSUE: WoN the Areolas can file against damages despite the effort to rectify the their agreement to enter into a contract of insurance under whose terms,
cancellation – SC held YES Malapit's fraudulent act of misappropriating the respondent insurance company promised to extend protection to petitioner-
premiums paid is beyond doubt directly imputable to Prudential. insured against the risk insured for a consideration in the form of premiums
to be paid by the latter.
RULING: WHEREFORE, the petition for review on certiorari is 21.   Under the law governing reciprocal obligations, particularly the second
hereby GRANTED and the decision of the Court of Appeals in CA-G.R. No. 16902 paragraph of Article 1191, the injured party, petitioner-insured in this case,
on May 31, 1990, REVERSED. The decision of Branch 40, RTC Dagupan City, in is given a choice between fulfillment or rescission of the obligation in case
Civil Case No. D-7972 rendered on June 30, 1987 is hereby REINSTATED subject one of the obligors, such as respondent insurance company, fails to comply
to the following modifications: (a) that nominal damages amounting to P30,000.00 with what is incumbent upon him. However, said article entitles the injured
be awarded petitioner in lieu of the damages adjudicated by court a quo; and (b) that party to payment of damages, regardless of whether he demands fulfillment
in the satisfaction of the damages awarded therein, respondent insurance company is or rescission of the obligation. Untenable then is respondent insurance
ORDERED to pay the legal rate of interest computed from date of filing of company’s argument, namely, that reinstatement being equivalent to
complaint until final payment thereof. fulfillment of its obligation, divests petitioner-insured of a rightful claim for
payment of damages. Such a claim finds no support in our laws on
SO ORDERED. obligations and contracts.
22.   The nature of the damages is in the form of nominal damages contrary to
RATIO: what was awarded by the trial court because no actual or substantial damage
1.   Malapit’s fraudulent act of misappropriating the premiums paid by Areola or injury was inflicted on Areola at the time the insurance policy was
is beyond doubt directly imputable to Prudential. A corporation acts solely cancelled. (Areola was awarded actual, moral, exemplary, and atty’s fees)
thru its employees. The latter’s acts are considered as its own for which it
can be held to account.
004 Tan v. CA (SOLCO) years. Despite the death of the insured, the insurance company can still rescind the
June, 29, 1989 | Gutierrez, J. | Right to cancel or rescind contract so long as it is within the two period and prior to the commencement of the
action.
PETITIONER: Emilio Tan, et al.
RESPONDENTS: The Court of Appeals, et al.
FACTS:
26.   Petitioners appeal from the Decision of the Insurance Commissioner
SUMMARY: On November 6, 1973 Tan Lee Siong was issued an 80k life insurance
dismissing herein petitioners' complaint against respondent Philippine
policy from American Life Insurance Company. On April 26, 1975. Tan Lee Siong
American Life Insurance Company for the recovery of the proceeds of
died from hepatoma. His heirs then filed a claim with the insurance company, but
Policy No. 1082467 in the amount of P 80,000.00.
they rejected his claim, stating that they are rescinding the contract because of the
27.   On September 23,1973, Tan Lee Siong, father of herein petitioners, applied
concealment and misrepresentation of Tan Lee Siong. Apparently, he had diabetes,
for life insurance in the amount of P 80,000.00 with respondent company.
liver, problems, and hypertension, but he failed to disclose this in his application. So
Said application was approved and Policy No. 1082467 was issued effective
the heirs filed a case against the insurance company with the Insurance
November 6,1973, with petitioners the beneficiaries thereof.
Commissioner (IC) to claim the insurance proceeds. The IC dismissed the case. CA
28.   On April 26,1975, Tan Lee Siong died of hepatoma. Petitioners then filed
affirmed. Hence petitioners are now appealing to the SC. Their main contention is
with respondent company their claim for the proceeds of the life insurance
that the insurance company no longer has any right to rescind the contract because
policy. However, in a letter dated September 11, 1975, respondent company
they did so after Tan Lee Siong already died. According to them, Section 48 of the
denied petitioners' claim and rescinded the policy by reason of the alleged
code only allows rescission if the insured is still alive. They claim that it must be
misrepresentation and concealment of material facts made by the deceased
given this interpretation because insurance company’s exert “overwhelming
Tan Lee Siong in his application for insurance. The premiums paid on the
influence” on their clients. Furthermore, they also argue that there was no
policy were thereupon refunded.
misrepresentation or concealment on their part because the insurance company did
29.   Alleging that respondent company's refusal to pay them the proceeds of the
not adduce evidence that the application was sufficiently explained to Tan Lee Siong
policy was unjustified and unreasonable, petitioners filed on November 27,
in layman’s terms. Moreover, they additionally state that it was the insurance agent
1975, a complaint against the former with the Office of the Insurance
who filled up the application. The issue in this case is (1) WON an insurance
Commissioner.
company still rescind the policy despite the death of the insured? And (2) WON Tan
30.   After hearing the evidence of both parties, the Insurance Commissioner
Lee Siong was guilty of concealment. With regard to the 1st issue, the phrase “
rendered judgment on August 9, 1977, dismissing petitioners' complaint.
during the lifetime” found in section 48 simply means that the policy is no longer
31.   The Court of Appeals dismissed ' the petitioners' appeal from the Insurance
considered in force after the insured has died. It doesn’t state that the insurer loses
Commissioner's decision for lack of merit
his right to rescind after the death of the insured. The key phrase is “for a period of
32.   Hence, this petition
two years.” The petitioners' interpretation would give rise to the incongruous
situation where the beneficiaries of an insured who dies right after taking out
ISSUE/s:
and paying for a life insurance policy, would be allowed to collect on the policy
2.   Did the insured commit an act of misrepresentation/concealment? YES, it
even if the insured fraudulently concealed material facts. Addressing the 2nd
was established that he had diabetes, hypertension, and liver problems
issue, the court stated that Tan Lee Siong’s defense is without merit. First of all, he
5 years even before the insurance contract was obtained. He failed to
signed the insurance contract which meant that he clearly understood what was in the
disclose these material details.
contract. It can be reasonably presumed that his signature affirms the correctness of
all the entries therein. Being a businessman, Tan Lee Siong would not have signed
3.   Can the insurance company rescind the insurance contract AFTER the death
something that he did not understand. The presumption is that a person intends
of the insured party? YES, to hold that the insurance contract can only
the ordinary consequence of his voluntary act and takes ordinary care of his
be rescinded when the insured party is still alive would result in the
concerns. His defense that it wasn’t filled up by him, or that it wasn’t reasonably
incongruous situation that despite the concealment and fraud of the
explained, or that the insurance company waived its right by ratification is thus
insured, the beneficiaries can nonetheless collect from him.
untenable. Lastly, he had hypertension, diabetes, and liver problems for a period of at
least 5 years before his application . He clearly knew that he was sick but he didn’t
RULING: WHEREFORE, the petition is hereby DENIED for lack of merit. The
disclose these facts.
questioned decision of the Court of Appeals is AFFIRMED..
DOCTRINE: The action to rescind a life insurance contract lasts for a period of two
RATIO:
1.  The petitioners contend that the respondent company no longer had the right to their companies, concluding the transactions and otherwise smoothing out all
to rescind the contract of insurance as rescission must allegedly be done during difficulties.
the lifetime of the insured within two years and prior to the commencement of 8.   It would thus be unjust if having been subjected to the whirlwind pressure of
action insurance salesmanship this Court itself has long denounced, the assured who
2.  According to the petitioners, the Insurance Law was amended and the second dies within the two-year period, should stand charged of fraudulent concealment
paragraph of Section 4839 added to prevent the insurance company from and misrepresentation.
exercising a right to rescind after the death of the insured. 9.  The legislative answer to the arguments posed by the petitioners is the
3.  The so-called "incontestability clause" precludes the insurer from raising the "incontestability clause" added by the second paragraph of Section 48.
defenses of false representations or concealment of material facts insofar as 10.  The insurer has two years from the date of issuance of the insurance
health and previous diseases are concerned if the insurance has been in force for contract or of its last reinstatement within which to contest the policy,
at least two years during the insured's lifetime. The phrase "during the whether or not, the insured still lives within such period. After two years, the
lifetime" found in Section 48 simply means that the policy is no longer defenses of concealment or misrepresentation, no matter how patent or well
considered in force after the insured has died. The key phrase in the second founded, no longer lie. Congress felt this was a sufficient answer to the
paragraph of Section 48 is "for a period of two years." various tactics employed by insurance companies to avoid liability. The
4.  As noted by the Court of Appeals, to wit: The policy was issued on petitioners' interpretation would give rise to the incongruous situation
November 6,1973 and the insured died on April 26,1975. The policy was where the beneficiaries of an insured who dies right after taking out and
thus in force for a period of only one year and five months. Considering paying for a life insurance policy, would be allowed to collect on the policy
that the insured died before the two-year period had lapsed, respondent even if the insured fraudulently concealed material facts.
company is not, therefore, barred from proving that the policy is void ab 11.  The petitioners further argue that no evidence was presented to show that the
initio by reason of the insured's fraudulent concealment or medical terms were explained in a layman's language to the insured. They state
misrepresentation. Moreover, respondent company rescinded the contract that the insurer should have presented its two medical field examiners as
of insurance and refunded the premiums paid on September 11, 1975, witnesses. Moreover, the petitioners allege that the policy intends that the
previous to the commencement of this action on November 27,1975. medical examination must be conducted before its issuance otherwise the insurer
5.  The petitioners then contend that there could have been no concealment or "waives whatever imperfection by ratification."
misrepresentation by their late father because Tan Lee Siong did not have to buy 12.  We agree with the Court of Appeals which ruled:
insurance. He was only pressured by insistent salesmen to do so. 13.  On the other hand, petitioners argue that no evidence was presented by
6.  According to them, it was the insurance agents repeated visits and respondent company to show that the questions appearing in Part II of the
solicitations which caused their father to accept the contract. Their father wasn’t application for insurance were asked, explained to and understood by the
the one who “knocked on the door” of the insurance company. In addition, deceased so as to prove concealment on his part. The same is not well taken.
assured was a man of means. If he intended to defraud the insurance company The deceased, by affixing his signature on the application form, affirmed
he could’ve obtained a insurance policy way more than 80k. Moreover, it was the correctness of all the entries and answers appearing therein. It is but to
the insurance agent who filled out the forms. be expected that he, a businessman, would not have affixed his signature on
7.  Petitioners state further: This Honorable Supreme Court has had occasion to the application form unless he clearly understood its significance. For, the
denounce the pressure and practice indulged in by agents in selling insurance. At presumption is that a person intends the ordinary consequence of his
one time or another most of us have been subjected to that pressure, that voluntary act and takes ordinary care of his concerns.
practice. This court took judicial cognizance of the whirlwind pressure of 14.  The evidence for respondent company shows that on September 19,1972, the
insurance selling-especially of the agent's practice of 'supplying the deceased was examined by Dr. Victoriano Lim and was found to be diabetic and
information, preparing and answering the application, submitting the application hypertensive; that by January, 1973, the deceased was complaining of
progressive weight loss and abdominal pain and was diagnosed to be suffering
                                                                                                                        from hepatoma, Another physician, Dr. Wenceslao Vitug, testified that the
39
 Section  48.  Whenever  a  right  to  rescind  a  contract  of  insurance  is  given  to  the  insurer  by  any  provision  of  this   deceased came to see him on December 14, 1973 for consolation and
chapter,  such  right  must  be  exercised  previous  to  the  commencement  of  an  action  on  the  contract.   claimed to have been diabetic for five years. Because of the concealment
 
After  a  policy  of  life  insurance  made  payable  on  the  death  of  the  insured  shall  have  been  in  force  during  the  
made by the deceased of his consultations and treatments for hypertension,
lifetime   of   the   insured   for   a   period   of   two   years   from   the   date   of   its   issue   or   of   its   last   reinstatement,   the   diabetes and liver disorders, respondent company was thus misled into
insurer   cannot   prove   that   the   policy   is   void  ab   initio  or   is   rescindable   by   reason   of   the   fraudulent   accepting the risk and approving his application as medically standard and
concealment  or  misrepresentation  of  the  insured  or  his  agent.   dispensing with further medical investigation and examination. For as long
 
as no adverse medical history is revealed in the application form, an applicant
for insurance is presumed to be healthy and physically fit and no further medical
investigation or examination is conducted by respondent company.
15.  There is also no strong showing that we should apply the "fine print" or
"contract of adhesion" rule in this case.
16.  There is no showing that the questions in the application form for insurance
regarding the insured's medical history are in smaller print than the rest of the
printed form or that they are designed in such a way as to conceal from the
applicant their importance.
005 Great Pacific Life Assurance Corp. v. CA (Soriano) membership in the group life insurance plan. In an application form,
13 October 1999 | Quisumbing, J. | Right to Cancel or Rescind Dr. Leuterio answered questions concerning his health condition:
a.   Have you ever had, or consulted, a physician for a heart
PETITIONER: Great Pacific Life Assurance Corp condition, high blood pressure, cancer, diabetes, lung,
RESPONDENT: Court of Appeals kidney or stomach disorder or any other physical
impairment? – Dr. Leuterio answered No.
SUMMARY: Grepalife and DBP entered into a group life insurance to b.   Are you now, to the best of your knowledge, in good health?
insure the lives of eligible housing mortgagors of DBP. Dr. Leuterio, – Dr. Leuterio answered yes.
physician and a housing debtor of DBP applied for membership in the 39.   Nov 15, 1983 - Grepalife issued a certificate as insurance coverage
group life insurance plan, and indicated in his application form that he of Dr. Leuterio to the extent of his DBP mortbage indebtedness.
never consulted, a physician for a heart condition, high blood pressure, 40.   Aug 6, 1984 – Dr. Leuterio died due to massive cerebral hemmorage.
cancer, diabetes, lung, kidney or stomach disorder or any other physical 41.   DBP submitted a death claim to Grepalife.
impairment and that to the best of his knowledge he is in good health. 42.   Grepalife denied the claim alleging that Dr. Leuterio was not
Grepalife issued a certificate as insurance coverage of Dr. Leuterio to the physically healthy when he applied for an insurance coverage on
extent of his DBP mortbage indebtedness. Almost a year after, Dr. November 15, 1983. Grepalife insisted that Dr. Leuterio did not
Leuterio died due to massive cerebral hemmorage. DBP submitted its disclose he had been suffering from hypertension, which caused his
claim to Grepalife but was denied on the ground of concealment hence it death. Allegedly, such non-disclosure constituted concealment that
was entitled to rescind the contract. The issue in this case is whether justified the denial of the claim.
Grepalife is entitled to rescind the insurance contract. The Supreme Court 43.   The widow of Leuterio Merdarda, filed a complaint with the RTC
held No, Contrary to petitioner's allegations, there was no sufficient proof against Grepalife for Specific Performance with Damages.
that the insured had suffered from hypertension. Aside from the 44.   During the trial, Dr. Hernando Mejia, who issued the death
statement of the insured's widow who was not even sure if the medicines certificate, was called to testify. Dr. Mejias findings, based
taken by Dr. Leuterio were for hypertension, the petitioner had not partly from the information given by the respondent widow,
proven nor produced any witness who could attest to Dr. Leuterio's stated that Dr. Leuterio complained of headaches presumably
medical history. Clearly, it had failed to establish that there was due to high blood pressure. The inference was not conclusive
concealment made by the insured, hence it cannot refuse payment of the because Dr. Leuterio was not autopsied, hence, other causes
claim. were not ruled out.
45.   RTC rendered a decision in favor of respondent widow and
DOCTRINE: The fraudulent intent on the part of the insured must be
established to entitle the insurer to rescind the contract. Misrepresentation
against Grepalife. CA affirmed RTC’s decision.
as a defense of the insurer to avoid liability is an affirmative defense and 46.   Hence this petition.
the duty to establish such defense by satisfactory and convincing ISSUE/s:
evidence rests upon the insurer. In the case at bar, the petitioner failed to 9.   WoN Grepalife has the right to rescind the insurance contract based
clearly and satisfactorily establish its defense, and is therefore liable to on the alleged concealment of Dr. Leuterio? – No. The Supreme
pay the proceeds of the insurance. Court found the petition not meritorious. Contrary to petitioner's
allegations, there was no sufficient proof that the insured had
suffered from hypertension. Aside from the statement of the insured's
FACTS: widow who was not even sure if the medicines taken by Dr. Leuterio
37.   A contract of life insurance was executed between petitioner were for hypertension, the petitioner had not proven nor produced
Grepalife and Development bank of the Philippines (DBP). Grepalife any witness who could attest to Dr. Leuterio's medical history.
agreed to insure the lives if eligible housing loan mortgagors of Clearly, it had failed to establish that there was concealment made by
DBP. the insured, hence it cannot refuse payment of the claim.
38.   Dr. Leuterio, physician and a housing debtor of DBP applied for
defense of the insurer to avoid liability is an affirmative defense and
RULING: WHEREFORE, the petition is hereby DENIED. The Decision the duty to establish such defense by satisfactory and convincing
and Resolution of the Court of Appeals in CA-G.R. CV 18341 is evidence rests upon the insurer. In the case at bar, the petitioner
AFFIRMED with MODIFICATION that the petitioner is ORDERED to pay failed to clearly and satisfactorily establish its defense, and is
the insurance proceeds amounting to Eighty-six thousand, two hundred therefore liable to pay the proceeds of the insurance.
(P86,200.00) pesos to the heirs of the insured, Dr. Wilfredo Leuterio
(deceased), upon presentation of proof of prior settlement of mortgagors
indebtedness to Development Bank of the Philippines. Costs against
petitioner.

RATIO:
34.   Petitioner contends that Dr. Leuterio failed to disclose that he
had hypertension, which might have caused his
death. Concealment exists where the assured had knowledge of
a fact material to the risk, and honesty, good faith, and fair
dealing requires that he should communicate it to the assured,
but he designedly and intentionally withholds the same.
35.   On the contrary the medical findings were not conclusive because
Dr. Mejia did not conduct an autopsy on the body of the decedent.
As the attending physician, Dr. Mejia stated that he had no
knowledge of Dr. Leuterios any previous hospital confinement.
36.   Dr. Leuterios death certificate stated that hypertension was only the
possible cause of death. The private respondents statement, as to the
medical history of her husband, was due to her unreliable
recollection of events. Hence, the statement of the physician was
properly considered by the trial court as hearsay.
37.   Contrary to appellants allegations, there was no sufficient proof that
the insured had suffered from hypertension. Aside from the
statement of the insureds widow who was not even sure if the
medicines taken by Dr. Leuterio were for hypertension, the appellant
had not proven nor produced any witness who could attest to Dr.
Leuterios medical history.
38.   Aside from the statement of the insured's widow who was not even
sure if the medicines taken by Dr. Leuterio were for hypertension,
the appellant had not proven nor produced any witness who could
attest to Dr. Leuterio's medical history . . . Appellant insurance
company had failed to establish that there was concealment made by
the insured, hence, it cannot refuse payment of the claim.
39.   Grepalife had failed to establish that there was concealment made by
the insured, hence, it cannot refuse payment of the claim.
40.   The fraudulent intent on the part of the insured must be established
to entitle the insurer to rescind the contract. Misrepresentation as a
006 SUN LIFE OF CANADA (PH) INC vs SIBYA (STA. MARIA) DOCTRINE: Section 48 serves a noble purpose, as it regulates the actions of both
June 8, 2016 | Reyes, J. | Right to Rescind or Cancel (none in this case) the insurer and the insured. Under the provision, an insurer is given two years - from
the effectivity of a life insurance contract and while the insured is alive - to discover
PETITIONER: SUN LIFE OF CANADA (PHILIPPINES), INC. or prove that the policy is void ab initio or is rescindible by reason of the fraudulent
RESPONDENT: MA. DAISY'S. SIBYA, JESUS MANUEL S. SIBYA III, JAIME concealment or misrepresentation of the insured or his agent. After the two-year
LUIS S. SIBYA, AND THE ESTATE OF THE DECEASED ATTY. JESUS SIBYA, period lapses, or when the insured dies within the period, the insurer must make
JR. good on the policy, even though the policy was obtained by fraud, concealment,
or misrepresentation. This is not to say that insurance fraud must be rewarded, but
SUMMARRY: Atty. Jesus Jr. applied for life insurance with Sun Life. In his that insurers who recklessly and indiscriminately solicit and obtain business must be
Application, he indicated that he had sought advice for kidney problems. Sun Life penalized, for such recklessness and lack of discrimination ultimately work to the
approved the application and issued the Policy. The policy indicated the respondents detriment of bona fide takers of insurance and the public in general.
as beneficiaries and entitles them to a death benefit of P1,000,000.00 should Atty.
Jesus Jr. die on or before February 5, 2021, or a sum of money if Atty. Jesus Jr. is still FACTS:
living on the endowment date. On May 11, 2001, Atty. Jesus Jr. died as a result of a 1.   Atty. Jesus Sibya, Jr. (Atty. Jesus Jr.) applied for life insurance with Sun
gunshot wound. As such, Ma. Daisy filed a Claimant's Statement with Sun Life to Life. In his Application for Insurance, he indicated that he had sought
seek the death benefits. In a letter, however, Sun Life denied the claim on the ground advice for kidney problems.
that the details on Atty. Jesus Jr.'s medical history were not disclosed in his 2.   Atty. Jesus Jr. indicated the following in his application:
application. Sun Life tendered a check representing the refund of the premiums paid. "Last 1987, had undergone lithotripsy due to kidney stone under Dr. Jesus
Respondents reiterated their claim against Sun Life which still refused to pay. Sun Benjamin Mendoza at National Kidney Institute, discharged after 3 days, no
Life instead filed a Complaint for Rescission before the RTC and prayed for judicial recurrence as claimed."hanRoblesVirtualawlibrary
confirmation of Atty. Jesus Jr.'s rescission of insurance policy. Sun Life alleged that 3.   Sun Life approved Atty. Jesus Jr.'s application and issued Insurance Policy
Atty. Jesus Jr. did not disclose in his insurance application his previous medical No. 031097335. The policy indicated the respondents as beneficiaries and
treatment at the National Kidney Transplant Institute. This undisclosed fact suggested entitles them to a death benefit of P1,000,000.00 should Atty. Jesus Jr. die
that the insured was in "renal failure" and at a high risk medical condition, had it on or before February 5, 2021, or a sum of money if Atty. Jesus Jr. is still
known such, it would not have issued the insurance policy. For their defense, the living on the endowment date.
respondents claimed that Atty. Jesus Jr. did not commit misrepresentation in his 4.   On May 11, 2001, Atty. Jesus Jr. died as a result of a gunshot wound in San
application for insurance. They averred that Atty. Jesus Jr. was in good faith when he Joaquin, Iloilo. As such, Ma. Daisy filed a Claimant's Statement with Sun
signed the insurance application and even authorized Sun Life to inquire further into Life to seek the death benefits indicated in his insurance policy.
his medical history for verification purposes. RTC dismissed the complaint for lack 5.   In a letter, however, Sun Life denied the claim on the ground that the details
of merit. RTC held that Atty. Jesus Jr. did not commit material concealment and on Atty. Jesus Jr.'s medical history were not disclosed in his application.
misrepresentation when he applied for life insurance with Sun Life. CA ruled that the Simultaneously, Sun Life tendered a check representing the refund of the
evidence show that there was no fraudulent intent on the part of Atty. Jesus Jr. in premiums paid by Atty. Jesus Jr.
submitting his insurance application. The issue is WON Sun Life has the right to 6.   The respondents reiterated their claim against Sun Life thru a letter. Sun
rescind the policy? – No. It has already been held that when the insured dies within Life refused to heed the respondents' requests and instead filed a Complaint
the incontestability period, the insurer must make good on the policy, even though the for Rescission before the RTC and prayed for judicial confirmation of Atty.
policy was obtained by fraud, concealment, or misrepresentation. In the present case, Jesus Jr.'s rescission of insurance policy.
Sun Life issued Atty. Jesus Jr.'s policy on February 5, 2001. Thus, it has two years 7.   In its Complaint, Sun Life alleged that Atty. Jesus Jr. did not disclose in his
from its issuance, to investigate and verify whether the policy was obtained by fraud, insurance application his previous medical treatment at the National Kidney
concealment, or misrepresentation. Upon the death of Atty. Jesus Jr., however, on Transplant Institute in May and August of 1994. According to Sun Life, the
May 11, 2001, or a mere three months from the issuance of the policy, Sun Life loses undisclosed fact suggested that the insured was in "renal failure" and at a
its right to rescind the policy. As discussed in Manila Bankers, the death of the high risk medical condition. Consequently, had it known such fact, it would
insured within the two-year period will render the right of the insurer to rescind the not have issued the insurance policy in favor of Atty. Jesus Jr.
policy nugatory. As such, the incontestability period will now set in. Also, Sun Life 8.   For their defense, the respondents claimed that Atty. Jesus Jr. did not
failed to clearly and satisfactorily establish its allegations of concealment and commit misrepresentation in his application for insurance. They averred
misrepresentation, and is therefore liable to pay the proceeds of the insurance. that Atty. Jesus Jr. was in good faith when he signed the insurance
application and even authorized Sun Life to inquire further into his medical
history for verification purposes. According to them, the complaint is just a modified the RTC decision by absolving Sun Life from the charges of
ploy to avoid the payment of insurance claims. violation of Sections 241 and 242 of the Insurance Code.
9.   The RTC dismissed the complaint for lack of merit. It held that Sun Life 14.   The CA ruled that the evidence on records show that there was no
violated Sections 241, paragraph 1(b), (d), and (e)40 and 24241 of the fraudulent intent on the part of Atty. Jesus Jr. in submitting his insurance
Insurance Code when it refused to pay the rightful claim of the respondents. application. Instead, it found that Atty. Jesus Jr. admitted in his application
10.   Moreover, the RTC ordered Sun Life to pay the amounts of P1,000,000.00 that he had sought medical treatment for kidney ailment.
as death benefits, P100,000.00 as moral damages, P100,000.00 as 15.   Sun Life filed a Motion for Partial Reconsideration the same was denied
exemplary damages, and P100,000.00 as attorney's fees and costs of suit.
11.   The RTC held that Atty. Jesus Jr. did not commit material concealment and ISSUE:
misrepresentation when he applied for life insurance with Sun Life. It 1.   WON Sun Life has the right to rescind the policy? – No. It has already
observed that given the disclosures and the waiver and authorization to been held that when the insured dies within the period, the insurer
investigate executed by Atty. Jesus Jr. to Sun Life, the latter had all the must make good on the policy, even though the policy was obtained by
means of ascertaining the facts allegedly concealed by the applicant. fraud, concealment, or misrepresentation. Also, Sun Life failed to clearly
12.   Aggrieved, Sun Life elevated the case to the CA. and satisfactorily establish its allegations of concealment and
13.   The CA affirmed the RTC decision with respect to payment of death misrepresentation, and is therefore liable to pay the proceeds of the
benefits and damages in favor of the respondents. The CA, however, insurance.

RULING: WHEREFORE, the petition for review is DENIED. The Decision dated
                                                                                                                        November 18, 2013 and Resolution dated February 13, 2014 of the Court of Appeals
40
 Sec.  241.  (1)  No  insurance  company  doing  business  in  the  Philippines  shall  refuse,  without  
in CA-G.R. CV. No. 93269 are hereby AFFIRMED.
just  cause,  to  pay  or  settle  claims  arising  under  coverages  provided  by  its  policies,  nor  shall  
any  such  company  engage  in  unfair  claim  settlement  practices.  Any  of  the  following  acts  by  
an  insurance  company,  if  committed  without  just  cause  and  performed  with  such  frequency  
RATIO:
as  to  indicate  a  general  business  practice,  shall  constitute  unfair  claim  settlement  practices:   1.   In Manila Bankers Life Insurance Corporation v. Aban, the Court held that
x   xxx   if the insured dies within the two-year contestability period, the insurer is
(b)   failing   to   acknowledge   with   reasonable   promptness   pertinent   communications   with   bound to make good its obligation under the policy, regardless of the
respect   to   claims   arising   under   its   policies;   presence or lack of concealment or misrepresentation. The Court held:
x  x  x  x   Section 48 serves a noble purpose, as it regulates the actions of both the insurer and
(d)  not  attempting  in  good  faith  to  effectuate  prompt,  fair  and  equitable  settlement  of  claims   the insured. Under the provision, an insurer is given two years - from the effectivity
submitted   in   which   liability   has   become   reasonably   clear;   or   of a life insurance contract and while the insured is alive - to discover or prove that
(e)   compelling   policyholders   to   institute   suits   to   recover   amounts   due   under   its   policies   by   the policy is void ab initio or is rescindible by reason of the fraudulent concealment
offering  without  justifiable  reason  substantially  less  than  the  amounts  ultimately  recovered   or misrepresentation of the insured or his agent. After the two-year period lapses,
in  suits  brought  by  them.   or when the insured dies within the period, the insurer must make good on the
  policy, even though the policy was obtained by fraud, concealment, or
41
 Sec.  242.  The  proceeds  of  a  life  insurance  policy  shall  be  paid  immediately  upon  maturity  of   misrepresentation. This is not to say that insurance fraud must be rewarded, but that
the  policy,  unless  such  proceeds  are  made  payable  in  installments  or  as  an  annuity,  in  which   insurers who recklessly and indiscriminately solicit and obtain business must be
case  the  installments,  or  annuities  shall  be  paid  as  they  become  due:  Provided,  however,  That   penalized, for such recklessness and lack of discrimination ultimately work to the
in   the   case   of   a   policy   maturing   by   the   death   of   the   insured,   the   proceeds   thereof   shall   be   detriment of bona fide takers of insurance and the public in general.
paid  within  sixty  days  after  presentation  of  the  claim  and  filing  of  the  proof  of  the  death  of   2.   In the present case, Sun Life issued Atty. Jesus Jr.'s policy on February 5,
the  insured.  Refusal  or  failure  to  pay  the  claim  within  the  time  prescribed  herein  will  entitle   2001. Thus, it has two years from its issuance, to investigate and verify
the  beneficiary  to  collect  interest  on  the  proceeds  of  the  policy  for  the  duration  of  the  delay   whether the policy was obtained by fraud, concealment, or
at   the   rate   of   twice   the   ceiling   prescribed   by   the   Monetary   Board,   unless   such   failure   or   misrepresentation.
refusal   to   pay   is   based   on   the   ground   that   the   claim   is   fraudulent.   3.   Upon the death of Atty. Jesus Jr., however, on May 11, 2001, or a mere
  three months from the issuance of the policy, Sun Life loses its right to
The  proceeds  of  the  policy  maturing  by  the  death  of  the  insured  payable  to  the  beneficiary   rescind the policy. As discussed in Manila Bankers, the death of the
shall  include  the  discounted  value  of  all  premiums  paid  in  advance  of  their  due  dates,  but  are   insured within the two-year period will render the right of the insurer
not  due  and  payable  at  maturity.  
 
to rescind the policy nugatory. As such, the incontestability period will
now set in.
4.   Assuming, however, for the sake of argument, that the incontestability
period has not yet set in, the Court agrees, nonetheless, with the CA
when it held that Sun Life failed to show that Atty. Jesus Jr. committed
concealment and misrepresentation.
5.   As correctly observed by the CA, Atty. Jesus Jr. admitted in his application
his medical treatment for kidney ailment. Moreover, he executed an
authorization in favor of Sun Life to conduct investigation in reference with
his medical history. The decision in part states:
Records show that in the Application for Insurance, [Atty. Jesus Jr.]
admitted that he had sought medical treatment for kidney ailment. When
asked to provide details on the said medication, [Atty. Jesus Jr.] indicated
the following information: year ("1987"), medical procedure ("undergone
lithotripsy due to kidney stone"), length of confinement ("3 days"),
attending physician ("Dr. Jesus Benjamin Mendoza") and the hospital
("National Kidney Institute").

It appears that [Atty. Jesus Jr.] also signed the Authorization which gave
[Sun Life] the opportunity to obtain information on the facts disclosed by
[Atty. Jesus Jr.] in his insurance application.

Given the express language of the Authorization, it cannot be said that


[Atty. Jesus Jr.] concealed his medical history since [Sun Life] had the
means of ascertaining [Atty. Jesus Jr.'s] medical record.

With regard to allegations of misrepresentation, we note that [Atty. Jesus


Jr.] was not a medical doctor, and his answer "no recurrence" may be
construed as an honest opinion. Where matters of opinion or judgment are
called for, answers made in good faith and without intent to deceive will not
avoid a policy even though they are untrue.24 (Citations omitted and italics
in the original)
6.   Indeed, the intent to defraud on the part of the insured must be ascertained
to merit rescission of the insurance contract. Concealment as a defense for
the insurer to avoid liability is an affirmative defense and the duty to
establish such defense by satisfactory and convincing evidence rests upon
the provider or insurer.
7.   In the present case, Sun Life failed to clearly and satisfactorily establish
its allegations, and is therefore liable to pay the proceeds of the
insurance.
007 DEL ROSARIO v. EQUITABLE INSURANCE (TAN) passenger car or street railway car in or on which the Insured is travelling as a
Jun. 29, ’63 | Paredes, J. | In doubt, rule in favor of the insured farepaying passenger.................... Php 1,500
Section 3. Injury sustained by the burning of a church, theatre, public library
or municipal administration building while the Insured is therein at the
commencement of the fire.................... Php 2,000
PETITIONER: Simeon Del Rosario
Section 4. Injury sustained by the wrecking or disablement of a regular
RESPONDENTS: The Equitable Insurance and Casualty Co., Inc. passenger elevator car in which the Insured is being conveyed as a passenger
(Elevator in mines excluded).................... Php 2,500
SUMMARY: Equitable Insurance issued Personal Accident Policy No. 7136 on Section 5. Injury sustained by a stroke of lightning or by a cyclone..... Php
the life of Francisco del Rosario, alias Paquito Bolero, son Simeon Del Rosario, 3,000
binding itself to pay the sum of P1,000.00 to P3,000.00, as indemnity for the
death of the insured. Please refer to the facts for the provisions of the policy, but
Part VI. Exceptions
it basically provided different amounts to be paid for the different ways which
This policy shall not cover disappearance of the Insured nor shall it cover
Francisco may die. However, the policy did not state any specific amount in case Death, Disability, Hospital fees, or Loss of Time, caused to the insured:
Francisco dies due to drowning. While Francisco was on a ship, (well obvious x x x (h.) By drowning except as a consequence of the wrecking or
naman hahaha), he jumped off due to a fire which broke out, and he drowned. disablement in the Philippine waters of a passenger steam or motor vessel
His father, Simeon, then filed for a claim but Equitable only paid Php 1,000. in which the Insured is travelling as a farepaying passenger; x x x."
Atty. Francisco, lawyer of Simeon, then filed a claim asking that Equitable pay
Php 3,000. Equitable then asked for the opinion of the Insurance Commissioner 3.   A rider was then executed:
and the latter said that the amount to be paid was only Php 1,000. The issue is
whether or not Equitable should pay Php 1,000 or Php 3,000. The SC, agreeing "IV. DROWNING
with the RTC, said that where there is an ambiguity with respect to the terms It is hereby declared and agreed that exemption clause Letter- (h) embodied in PART VI of
and conditions of the policy, the same will be resolved against the one the policy is hereby waived by the company, and to form a part of the provision covered
responsible thereof. Since Equitable promised to pay Php 1,000 to Php 3,000, by the policy."
and it didn’t state a specific amount for drowning, the courts must rule in favor of
4.   On February 24, 1957, the insured Francisco del Rosario, alias Paquito Bolero,
the insured, and the maximum amount promised should be given.
while on board the motor launch "ISLAMA" together with 33 others, including
his beneficiary in the Policy, Remedios Jayme, were forced to jump off said
RATIO: Where two interpretations, equally fair, of languages used in an
launch on account of fire which broke out on said vessel, resulting in the death
insurance policy may be made, that which allows the greater indemnity will
of drowning, of the insured and beneficiary in the waters of Jolo.
prevail.
5.   On April 13, 1957, Simeon del Rosario, father of the Francisco, and as the sole
FACTS: heir, filed a claim for payment with defendant company, and on September 13,
1.   Equitable Insurance issued Personal Accident Policy No. 7136 on the life of 1957, Equitable paid to him the sum of P1,000.00, pursuant to Section 1 of Part
Francisco del Rosario, alias Paquito Bolero, son Simeon Del Rosario, I of the policy and was issued this receipt:
binding itself to pay the sum of P1,000.00 to P3,000.00, as indemnity for
RECEIVED of the EQUITABLE INSURANCE & CASUALTY CO., INC.,
the death of the insured. the sum of PESOS — ONE THOUSAND (P1,000.00) Philippine Currency,
2.   The policy is as follows: being settlement in full for all claims and demands against said Company as a
result of an accident which occurred on February 26, 1957, insured under out
Part I. Indemnity For Death ACCIDENT Policy No. 7136, causing the death of the Assured. In view of
If the insured sustains any bodily injury which is effected solely through the foregoing, this policy is hereby surrendered and CANCELLED.
violent, external, visible and accidental means, and which shall result,
independently of all other causes and within sixty (60) days from the
occurrence thereof, in the Death of the Insured, the Company shall pay the 6.   On the same day, Atty. Vicente J. Francisco, wrote defendant company
amount set opposite such injury: acknowledging receipt by his client (plaintiff herein), of the P1,000.00, but
informing said company that said amount was not the correct one. He said that it
Section 1. Injury sustained other than those specified below unless excepted should be Php 1,500 under sec. 2 of the policy because the circumstances of
hereinafter.................... Php 1,000 Francisco’s death was similar to what was provided therein.
Section 2. Injury sustained by the wrecking or disablement of a railroad 7.   Equitable then referred the matter to the Insurance Commissioner who rendered
an opinion stating that the former’s liability was only Php 1,000, pursuant to
Sec. 1 of the policy. Because of this, Equitable refused to pay more than the RATIO:
original Php 1,000. 1.   The SC stated that the findings of the lower court are supported by the generally
8.   However, Atty. Francisco then sent another letter, stating that the total amount accepted principles or rulings on insurance, which enunciate that where there is
that should be paid is Php 3,000. However, Equitable refused to pay the same. an ambiguity with respect to the terms and conditions of the policy, the same
9.   This led to a case filed in the RTC. Equitable then filed motion to dismiss, will be resolved against the one responsible thereof.
stating that the claim is already extinguished due to the Php 1,000 already being 2.   It should be recalled in this connection, that generally, the insured, has little, if
paid. any, participation in the preparation of the policy, together with the drafting
10.   The RTC then ruled that (Take note of this because the SC didn’t really add of its terms and Conditions.
much): 3.   The interpretation of obscure stipulations in a contract should not favor the party
a.   Under the terms of this policy, defendant company agreed to pay who cause the obscurity (Art. 1377, N.C.C.), which, in the case at bar, is the
P1,000.00 to P3,000.00 as indemnity for the death of the insured. The insurance company.
insured died of drowning. 4.   And so it has been generally held that the "terms in an insurance policy, which
b.   Death by drowning is covered by the policy the pertinent provisions of are ambiguous, equivocal or uncertain . . . are to be construed strictly against, the
which reads as follows: Part I of the policy fixes specific amounts as insurer, and liberally in favor of the insured so as to effect the dominant purpose
indemnities in case of death resulting from "bodily injury which is of indemnity or payment to the insured, especially where a forfeiture is
effected solely thru violence, external, visible and accidental means." involved.”
c.   However, Part I of the Policy is not applicable in case of death by 5.   The reason for this rule is that the "insured usually has no voice in the selection
drowning because death by drowning is not one resulting from "bodily or arrangement of the words employed and that the language of the contract is
injury which is affected solely thru violent, external, visible and selected with great care and deliberation by expert and legal advisers employed
accidental means" as "Bodily Injury" means a cut, a bruise, or a wound by, and acting exclusively in the interest of, the insurance company"
and drowning is death due to suffocation and not to any cut, bruise or 6.   Where two interpretations, equally fair, of languages used in an insurance
wound. policy may be made, that which allows the greater indemnity will prevail.
d.   Besides, on the face of the policy Exhibit "A" itself, death by drowning
is a ground for recovery apart from the bodily injury because death by
bodily injury is covered by Part I of the policy while death by drowning
is covered by Part VI thereof.
e.   But while the policy mentions specific amounts that may be recovered
for death for bodily injury, yet, there is not specific amount mentioned
in the policy for death through drowning although the latter is, under
Part VI of the policy, a ground for recovery thereunder.
f.   Since the defendant has bound itself to pay P1000.00 to P3,000.00 as
indemnity for the death of the insured but the policy does not positively
state any definite amount that may be recovered in case of death by
drowning, there is an ambiguity in this respect in the policy, which
ambiguity must be interpreted in favor of the insured and strictly
against the insurer so as to allow greater indemnity.
g.   Therefore, Php 3,000 should be paid. However, in the absence of bad
faith, the attys fees and expenses of litigation cannot be awarded.
11.   Equitable appealed to the CA it just forwarded it to the SC.

ISSUES:
2.   W/N Equitable should pay Php 1000 or Php 3000? Php 3000 because the
ambiguity should be construed in favor of the insured.

RULING: The judgment appealed from is hereby affirmed. Without costs.


008 SUN INSURANCE v. CA (TIMBOL) 2.   As she watched television, he stood in front of her and pointed the
July 17, 1992 | Cruz, J. | Liability of Insurer for Loss gun at her
3.   She pushed it aside and said it might be loaded
PETITIONER: Sun Insurance Office, LTD 4.   He assured her it was not and then pointed it to his temple
RESPONDENTS: CA and Nerissa Lim 5.   The next moment there was an explosion and Lim slumped to the
floor
SUMMARY: Felix Lim, Jr. obtained a Personal Accident Policy worth P200,000 6.   He was dead before he fell
with Sun Insurance Office, assigning Nerissa Lim, his wife, as the beneficiary. Two 36.   The widow sued Sun Insurance in the RTC of Zamboanga City and was
months later, after Felix’s mother’s birthday party, Lim died with a bullet wound in sustained. This decision was affirmed on appeal, and the MR was denied.
his head. According to Nalagon, Lim’s secretary, Lim was playing with his handgun, The Sun Insurance then came to this SC to fault the CA for approving the
from which he had previously removed the magazine. Nalagon asked him to put it payment of the claim and the award of damages
aside but Lim assured her that it was not loaded, and the next moment there was an
explosion. This prompted the wife, Nerissa, to recover from Sun Insurance, which ISSUE/s:
the RTC allowed and was affirmed by the CA. However, Sun Insurance claims that 4.   WoN Nerissa can claim– YES, there was an accident, and Felix Lim did not
there was no accident and that Sun Insurance should not be liable because under the willfully expose himself in peril. There is nothing in the policy that relieves
exception, the company should not be liable for bodily injury consequent upon the insurer of the responsibility to pay indemnity agreed upon if the insured
committing or attempting to commit suicide or willfully exposing himself to needless is shown to have contributed to his own accident.
peril except in an attempt to save human life. Hence, the issue before the court.
Whether or not Nerissa Lim may be entitled to receive the proceeds. RULING: WHEREFORE, the challenged decision of the Court of Appeals is
AFFIRMED in so far as it holds the petitioner liable to the private respondent in the
The SC held that Nerissa Lim is entitled to receive the proceeds of the Personal sum of P200,000.00 representing the face value of the insurance contract, with
Accident Policy. Lim was unquestionably negligent but this does not bar Nerissa interest at the legal rate from the date of the filing of the complaint until the full
from claimaing against an accident. There is nothing in the policy that relieves the amount is paid, but MODIFIED with the deletion of all awards for damages,
insurer of the responsibility to pay the indemnity agreed upon if the insured is shown including attorney's fees, except the costs of the suit.
to have contributed to his own accident. Neither does the circumstances relieve the
company, because Lim did not willfully expose himself to needless peril. The act RATIO:
was precisely intended to assure Nalagon that the gun was indeed harmless. 23.   The words “accident” and “accidental” have never acquired any technical
signification in law, and when used in an insurance contract are to be
DOCTRINE: There is nothing in the policy that relieves the insurer of the construed and considered according to the ordinary understanding and
responsibility to pay the indemnity agreed upon if the insured is shown to have common usage and speech of people generally
contributed to his own accident. Indeed, most accidents are caused by negligence. 24.   In-substance, the courts are practically agreed that the words “accident” and
There are only four exceptions expressly made in the contract to relieve the insurer “accidental” mean that which happens by chance or fortuitously, without
from liability, and none of these exceptions (Ratio 11) is applicable in the case at bar intention or design, and which is unexpected, unusual, and unforeseen
25.   The definition that has usually been adopted by the courts is that an accident
is an event that takes place without one’s foresight or expectation – an event
FACTS: that proceeds from an unknown cause, or is an unusual effect of a known
33.   Sun Insurance issued Personal Accident Policy No. 05687 to Felix Lim, Jr. case, and therefore not expected
wih a face value of P200,000 26.   An accident is an event which happens without any human agency or, if
1.   Two months later, he was dead with a bullet wound in his head happening through human agency, an event which, under the circumstances,
34.   As beneficiary, his wife Nerissa Lim sought payment on the policy but her is unusual to and not expected by the person to whom it happens
claim was rejected 1.   It has also been defined as an injury which happens by reason of
1.   Sun Insurance agreed that there was no suicide some violence or casualty to the injured without his design,
2.   It argued, however that there was no accident either consent, or voluntary co-operation
35.   Pilar Nalagon, Lim’s secretary, was the only eyewitness to his death 27.   In light of these definitions, the Court is convinced that the incident that
1.   According to Nalagaon, Lim was in a happy mood (but not drunk) resulted in Lim’s death was indeed an accident
and was playing with his handgun, from which he had previously
removed the magazine
1.   Sun Insurance invoking the case of De La Cruz v. Capital during whci such person attains the
Insurance, says that there is no accident when a deliberate act is Upper Age Limit
performed unless some additional, unexpected, independent and 2.   Consequent Upon
unforeseen happening occurs which produces or brings about their 1.   The Insured Person committing or
injury death attempting to commit suicide or
2.   There was such a happening. This was the firing of the gun which willfully exposing himself to needless
was the additional unexpected and independent and unforeseen peril except in an attempt to save
occurrence that led to the isnured person’s death human life
28.   Sun Insurance also cited an exception that the company shall not be liable in 2.   War, invasion, act of foreign enemy,
respect of bodily injury, consequent upon the insured person attempting to hostilities (whether war be declared
commit suicie or willfully exposing himself to needless peril except in an or not) civil war, rebellion,
attempt to save human life revolution, insurrection, or military
29.   Sun Insurance also maintains that by the mere act of pointing the gun to his or usurped power
temple, Lim had willfully exposed himself to needless peril and so came 2.   Bodily injury or death disablement or medical
under the exception. The theory that a gun is per se dangerous and should expenses consequent upon or contributed to by the
therefore be handled cautiously in every case Insured Person
1.   The SC held that as testified by the seceraty, im had removed 1.   Having taken a drug unless the Insured
the magazine from the gun and believe it as no longer proves that the drug was taken in accordance
danagerous with proper medical prescription and
2.   He expressly assuredher that the gun was not loaded directions and not for treatment of drug
3.   It is submitted that Lim did not wilfuly expose himself to addiction
needless peril when he pointed the gun to his temple because 2.   Suffering from pre-existing physical or
the fact is that he thought it was not unsafe to do so mental defect or infirmity which had not been
4.   The act was precisely intended to assure Nalagon that the gun declared to and accepted in wriitn gby the
was indeed harmless Company
30.   Lim was unquestionably negligent and that negligence cost him his own 3.   Death Disablement or Medical Expenses consequent
life. But it should not prevent his widow from recovering from the upon or contributed to by the Insured Person being
insurance policy he obtained precisely against accident pregnant or suffering from sickness or disease not
31.   There is nothing in the policy that relieves the insurer of the resulting from bodily injury due to a gradually
responsibility to pay the indemnity agreed upon if the insured is shown operating cause
to have contributed to his own accident 4.   Risks of Murder and Assault
32.   Indeed, most accidents are caused by negligence. There are only four 34.   It bears noting that insurance contracts are as a rule supposed to be
exceptions expressly made in the contract to relieve the insurer from interpreted liberally in favor of the assured. There is no reason to
liability, and none of these exceptions is applicable in the case at bar deviate from this rule, especially in view of the circumstances of this
33.   Exceptions: case as above analyzed
1.   The Company shall not be liable in respect of: 35.   It is evident that Sun Insurance was acting in good faith then it resisted
1.   Bodily Injury Nerissa Lim’s case on the ground that the death of the insured was covered
1.   Sustained by the exception
1.   While the Insured Person is engaging 36.   The issue was indeed debatable ad was clearly not raised only for the
in (or practicing for or taking part in purpose of evaiding a legitimate obligation. The Court therefore holds that
training peculiar to) any of the the award of mrla and exemplary damages and of attorney’s fees is unjust
Excluded Activities and so must be disapproved
2.   By any person before such person 37.   In order that a person may be liable to the payment of moral damages, the
attains the Lower Age Limit or after law requires that his act be wrongful
tehe xpiry of the Period of Insurance 1.   The adverse result of an action does not per se make the act
wrongful and subject the act or to the payment of moral damages
2.   The law could not have meant to impose a penalty on the right to
litigate; such right is so precious that moral damages may not be
charged on those who may exercise it erroneously. For these the
law taxes costs
3.   If a party wins, he cannot as a rule, recover attorney’s fees and
litigation expenses, since it is not the fact of winning alone that
entitles him to recover such damages of the exceptional
circumstnaces enumerated in Art. 2208
1.   Otherwise, every time a defendant wins, automatically the
plaintiff must pay attorney’s fees thereby putting a
premium on the right to litigate which should not be so
2.   For those expenses, the law deems the award of costs as
sufficient
009 GULF RESORT v. PH CHARTER (Escalona edited by Chiqui) swimming pools. Item 5 in those policies referred to the two
May 16, 2005 | Puno, J. | Liability of insurer for loss (2) swimming pools only; that subsequently AHAC issued in
Gulf Resort’s favor Policy No. 206-4182383-0 and in said
PETITIONER: Gulf Resort Inc. policy the earthquake endorsement clause was deleted and
RESPONDENTS: Philippine Charter Insurance Corporation
the entry under Endorsements/Warranties at the time of issue
SUMMARY: Gulf Resort, Inc. was insured with AHAC, which includes loss or read that Gulf Resort renewed its policy with AHAC for the
damage to shock to the two swimming pools insured by a policy occasioned by period of March 14, 1989 to March 14, 1990 under Policy No.
or through or in consequence of earthquakes. Gulf Resort then insured its 206-4568061-9, which carried the entry under
properties with PCIC based on the same insurance contract they had with Endorsement/Warranties at Time of Issue, which read
AHAC. Later on, an earthquake struck Central Luzon and Northern Luzon so the
properties and 2 swimming pools in its Agoo Playa Resort were damaged. Gulf Endorsement to Include Earthquake Shock in the amount
then claimed against PCIC for the damages the earthquake created on its of P10,700.00 and paid P42,658.14 as premium thereof.
properties. Gulf's claim was denied on the ground that its insurance policy only 8.   Gulf then insured its properties with PCIC using the exact same
afforded earthquake shock coverage to the two swimming pools of the resort. contract it used to have with AHAC. Gulf paid only P393.00 as
Gulf contends that pursuant to a rider, no qualifications were placed on the scope premium against earthquake shock.
of the earthquake shock coverage, thus the policy extended earthquake shock
coverage to all of the insured properties. The RTC ruled in favor of PCIC that 9.   In all six insurance policies prior to the one with PCIC, the
the policy only covered the two swimming pools. The CA affirmed the decision premium against peril of earthquake shock is the same.
of the RTC. W/N the insurance policy covers the other properties beyond the 10.  On July 16, 1990, an earthquake struck Central Luzon and
two swimming pools – NO, the Court ruled that the insurance policy did not Northern Luzon and Gulf Resort’s properties covered by Policy
cover the other properties. The Court first ruled that a stipulation in the contract No. 31944 issued by PCIC, including the two swimming pools
cannot be construed piecemeal, such that the contract should be construed as a
whole, including the provisions and riders. They should all be construed together in its Agoo Playa Resort were damaged.
in order to evidence the intention of the parties of whether they wanted to insure 11.  After the earthquake, Gulf Resort advised PCIC that it would
the other properties against earthquake shocks. The Court held that there was no be making a claim under its Insurance Policy No. 31944 for
intention of the parties to include such because there were no premiums paid for damages on its properties. PCIC instructed Gulf Resort to file a
the other properties against earthquake shocks, apart from the two swimming formal claim, then assigned the investigation of the claim to an
pools. This is in consonance with the history of the deletion of the other
properties from insurance against earthquake shocks in the insurance contracts independent claims adjuster, Bayne Adjusters and Surveyors,
that Gulf had with AHAC. Inc.
12.  On July 30, 1990, PCIC, through its adjuster, requested Gulf
DOCTRINE: All provisions of the insurance policy should be examined and Resort to submit various documents in support of its claim. On
interpreted in consonance with each other. All its parts are reflective of the true August 7, 1990, Bayne Adjusters and Surveyors, Inc., through
intent of the parties. The policy cannot be construed piecemeal. Certain
stipulations cannot be segregated and then made to control; neither do particular its Vice-President A.R. de Leon, rendered a preliminary
words or phrases necessarily determine its character. report finding extensive damage caused by the earthquake
to the clubhouse and to the two swimming pools. Mr. de
FACTS: Leon stated that except for the swimming pools, all affected
items have no coverage for earthquake shocks.
7.   Gulf Resort is the owner of the Plaza Resort situated at Agoo, 13.  On August 11, 1990, Gulf Resort filed its form demand for
La Union and had its properties in said resort insured originally settlement of the damage to all its properties in the Agoo Playa
with the American Home Assurance Company (AHAC). In the Resort. On August 23, 1990, PCIC denied Gulf Resort’s
first four insurance policies issued by AHAC, the risk of loss claim on the ground that its insurance policy only afforded
from earthquake shock was extended only to Gulf Resort’s two earthquake shock coverage to the two swimming pools of
the resort. Gulf Resort and PCIC failed to arrive at a insurance contract PCIC is said to have been based
settlement. Thus, on January 24, 1991, Gulf Resort filed a and copied, covered an extended earthquake shock
complaint with the regional trial court of Pasig. insurance on all the insured properties.
14.  RTC ruled in favor of the PCIC, stating: ISSUES:
a.   The above schedule clearly shows that plaintiff paid 2.   WoN the insurance policy covered only the two swimming pools, rather
than all the properties insured, against the risk of an earthquake shock –
only a premium of P393.00 against the peril of YES. Only the two swimming pools are covered because the contract that
earthquake shock, the same premium it paid against reflected the intent of the parties evidenced that only the two swimming
earthquake shock only on the two swimming pools in pools were insured against earthquake shocks. This was proven through the
all the policies issued by PCIC. From this fact the Court lack of payment of premiums and testimonies of witnesses.
must consequently agree with the position of PCIC that
RULING: The judgment of the Court of Appeals is affirmed. The petition
the endorsement rider means that only the two for certiorari is dismissed.
swimming pools were insured against earthquake
shock. RATIO:
b.   Gulf Resort correctly points out that a policy of 41.  Four key important points in the insurance policy are
insurance is a contract of adhesion hence, where the important.
language used in an insurance contract or application is a.   in the designation of location of risk, only the two
such as to create ambiguity the same should be resolved swimming pools were specified as included, viz:
against the party responsible therefor, i.e., the insurance i.   ITEM 3 — 393,000.00 — On the two (2)
company which prepared the contract. To the mind of swimming pools only (against the peril of
the Court, the language used in the policy in litigation is earthquake shock only)
clear and unambiguous hence there is no need for b.   Under the breakdown for premium payments, it was
interpretation or construction but only application of the stated that:
provisions therein. i.   PREMIUM RECAPITULATION
c.   From the above observations the Court finds that only ITEM NOS. AMNT RATES
the two (2) swimming pools had earthquake shock PREMIUM 393K 0.100%-
coverage and were heavily damaged by the earthquake, E/S 393.00
which struck on July 16, 1990. PCIC having admitted c.   Third, Policy Condition No. 6 stated:
that the damage to the swimming pools was appraised i.   This insurance does not cover any loss or
by PCIC’s adjuster at P386,000.00, PCIC must, by damage occasioned by or through or in
virtue of the contract of insurance, pay Gulf Resort said consequence, directly or indirectly of any of the
amount. following occurrences, namely: — Earthquake,
15.  Gulf Resort’s Motion for Reconsideration was denied. Thus, volcanic eruption or other convulsion of nature.
Gulf Resort filed an appeal with the Court of Appeals. After d.   The rider attached to the policy titled, “Extended
review, the appellate court affirmed the decision of the trial Coverage Endorsement (To Include the Perils of
court and ruled, thus: Explosion, Aircraft, Vehicle and Smoke) stated:
a.   However, after carefully perusing the documentary ANNUAL PAYMENT AGREEMENT ON
LONG TERM POLICIES
evidence of both parties, We are not convinced that THE INSURED UNDER THIS POLICY HAVING ESTABLISHED
the last two (2) insurance contracts, which the Gulf AGGREGATE SUMS INSURED IN EXCESS OF FIVE MILLION
PESOS, IN CONSIDERATION OF A DISCOUNT OF 5% OR 7 %
Resort had with PCIC and upon which the subject OF THE NET PREMIUM x x x POLICY HEREBY UNDERTAKES
TO CONTINUE THE INSURANCE UNDER THE ABOVE
NAMED x x x AND TO PAY THE PREMIUM.
c.   The insurer assumes the risk;
Earthquake Endorsement d.   Such assumption of risk is part of a general scheme to
In consideration of the payment by the Insured to the Company of the
sum of P. . . . . . . . . . . . . . . . . additional premium the Company
distribute actual losses among a large group of persons
agrees, notwithstanding what is stated in the printed conditions of this bearing a similar risk;
Policy to the contrary, that this insurance covers loss or damage
(including loss or damage by fire) to any of the property insured by
e.   In consideration of the insurer's promise, the
this Policy occasioned by or through or in consequence of insured pays a premium.
Earthquake. 46.  An insurance premium is the consideration paid an insurer for
Provided always that all the conditions of this Policy shall apply undertaking to indemnify the insured against a specified
(except in so far as they may be hereby expressly varied) and that any
reference therein to loss or damage by fire should be deemed to apply
peril. In fire, casualty, and marine insurance, the premium
also to loss or damage occasioned by or through or in consequence of payable becomes a debt as soon as the risk attaches. In the
Earthquake. subject policy, no premium payments were made with
regard to earthquake shock coverage, except on the two
42.  Gulf Resort contends that pursuant to this rider, no swimming pools. There is no mention of any premium
qualifications were placed on the scope of the earthquake payable for the other resort properties with regard to
shock coverage. Thus, the policy extended earthquake earthquake shock. This is consistent with the history of Gulf
shock coverage to all of the insured properties. Resort’s previous insurance policies from AHAC.
43.  It is basic that all the provisions of the insurance policy 47.  PCIC, in compliance with the condition set by Gulf Resort,
should be examined and interpreted in consonance with copied AIU Policy No. 206-4568061-9 in drafting its Insurance
each other. All its parts are reflective of the true intent of Policy No. 31944. It is true that there was variance in some
the parties. The policy cannot be construed piecemeal. terms, specifically in the replacement cost endorsement, but the
Certain stipulations cannot be segregated and then made to principal provisions of the policy remained essentially similar
control; neither do particular words or phrases necessarily to AHAC’s policy. Consequently, we cannot apply the "fine
determine its character. print" or "contract of adhesion" rule in this case as the
44.  Gulf Resort cannot focus on the earthquake shock endorsement parties intent to limit the coverage of the policy to the two
to the exclusion of the other provisions. All the provisions and swimming pools only is not ambiguous.
riders, taken and interpreted together, indubitably show the 48.   Gulf Resort also cited and relies on the attachment of the phrase "Subject to:
intention of the parties to extend earthquake shock coverage to Other Insurance Clause, Typhoon Endorsement, Earthquake Shock
the two swimming pools only. Endorsement, Extended Coverage Endorsement, FEA Warranty & Annual
45.  A careful examination of the premium recapitulation will Payment Agreement on Long Term Policies" to the insurance policy as
proof of the intent of the parties to extend the coverage for earthquake
show that it is the clear intent of the parties to extend shock.
earthquake shock coverage only to the two swimming a.   However, this phrase is merely an enumeration of the descriptive
pools. Section 2(1) of the Insurance Code defines a contract of titles of the riders, clauses, warranties or endorsements to which
insurance as an agreement whereby one undertakes for a the policy is subject, as required under Section 50, paragraph 2 of
consideration to indemnify another against loss, damage or the Insurance Code.
49.   SC also holds that no significance can be placed on the deletion of the
liability arising from an unknown or contingent event. Thus, an qualification limiting the coverage to the two swimming pools. The
insurance contract exists where the following elements concur: earthquake shock endorsement cannot stand alone.
a.   The insured has an insurable interest; 50.   The Court also rejects Gulf Resort’s contention that PCIC’s
b.   The insured is subject to a risk of loss by the happening contemporaneous and subsequent acts to the issuance of the insurance
of the designated peril; policy falsely gave the Gulf Resort’s assurance that the coverage of the
earthquake shock endorsement included all its properties in the resort. PCIC A. Yes, sir. The final action is still with us although they can recommend what insurance to
take.
only insured the properties as intended by the Gulf Resort.
Q. In the procurement of the insurance police (sic) from March 14, 1988 to March 14, 1989,
a.   The verbal assurances allegedly given by PCIC’s representative did you give written instruction to Forte Insurance Agency advising it that the earthquake
Atty. Umlas were not proved. Atty. Umlas categorically denied shock coverage must extend to all properties of Agoo Playa Resort in La Union?
having given such assurances. A. No, sir. We did not make any written instruction, although we made an oral instruction to
that effect of extending the coverage on (sic) the other properties of the company.
b.   But as testified to by the representative of Bayne Adjusters and
Q. And that instruction, according to you, was very important because in April 1987 there was
Surveyors, Inc., PCIC never meant to lead Gulf Resort to believe an earthquake tremor in La Union?
that the endorsement for earthquake shock covered properties other A. Yes, sir.
than the two swimming pools Q. And you wanted to protect all your properties against similar tremors in the [future], is that
51.   The case law will show that this Court will only rule out blind adherence to correct?
A. Yes, sir.
terms where facts and circumstances will show that they are basically one- Q. Now, after this policy was delivered to you did you bother to check the provisions with
sided. Thus, we have called on lower courts to remain careful in respect to your instructions that all properties must be covered again by earthquake shock
scrutinizing the factual circumstances behind each case to determine the e endorsement?
A. Are you referring to the insurance policy issued by American Home Assurance Company
cacy of the claims of contending parties. In Development Bank of the
marked Exhibit G?
Philippines v. National Merchandising Corporation, et al., the parties, who Atty. Mejia: Yes.
were acute businessmen of experience, were presumed to have assented to Witness:
the assailed documents with full knowledge. A. I examined the policy and seeing that the warranty on the earthquake shock endorsement
has no more limitation referring to the two swimming pools only, I was contented already that
52.   We cannot apply the general rule on contracts of adhesion to the case at bar.
the previous limitation pertaining to the two swimming pools was already removed.
Gulf Resort cannot claim it did not know the provisions of the policy. From Petitioner also cited and relies on the attachment of the phrase Subject to: Other Insurance Clause,
the inception of the policy, Gulf Resort had required the PCIC to copy Typhoon Endorsement, Earthquake Shock Endorsement, Extended Coverage Endorsement,
verbatim the provisions and terms of its latest insurance policy from FEA Warranty & Annual Payment Agreement on Long Term Policies to the insurance policy as
proof of the intent of the parties to extend the coverage for earthquake shock. However, this phrase
AHAC-AIU.
is merely an enumeration of the descriptive titles of the riders, clauses, warranties or endorsements
to which the policy is subject, as required under Section 50, paragraph 2 of the Insurance Code.
There were a lot of testimonies used as evidence of showing the intent We also hold that no significance can be placed on the deletion of the qualification limiting the
coverage to the two swimming pools. The earthquake shock endorsement cannot stand alone. As
of the parties to exclude the other properties from being insured explained by the testimony of Juan Baranda III, underwriter for AHAC-AIU:
against earthquake shocks. Here’s the transcript of those testimonies: DIRECT EXAMINATION OF JUAN BARANDA III
TSN, August 11, 1992
pp. 9-12
CROSS EXAMINATION OF LEOPOLDO MANTOHAC TSN, November 25, 1991 Atty. Mejia:
pp. 12-13 We respectfully manifest that the same exhibits C to H inclusive have been previously marked
Q. Now Mr. Mantohac, will it be correct to state also that insofar as your insurance policy by counsel for defendant as Exhibit[s] 1-6 inclusive. Did you have occasion to review of (sic)
during the period from March 4, 1984 to March 4, 1985 the coverage on earthquake shock these six (6) policies issued by your company [in favor] of Agoo Playa Resort?
was limited to the two swimming pools only? WITNESS:
A. Yes, sir. It is limited to the two swimming pools, specifically shown in the warranty, there Yes[,] I remember having gone over these policies at one point of time, sir.
is a provision here that it was only for item 5. Q. Now, wach (sic) of these six (6) policies marked in evidence as Exhibits C to H
Q. More specifically Item 5 states the amount of P393,000.00 corresponding to the two respectively carries an earthquake shock endorsement[?] My question to you is, on the basis
swimming pools only? on (sic) the wordings indicated in Exhibits C to H respectively what was the extent of the
A. Yes, sir. coverage [against] the peril of earthquake shock as provided for in each of the six (6)
CROSS EXAMINATION OF LEOPOLDO MANTOHAC TSN, November 25, 1991 policies?
pp. 23-26 xxx
Q. For the period from March 14, 1988 up to March 14, 1989, did you personally arrange for WITNESS:
the procurement of this policy? The extent of the coverage is only up to the two (2) swimming pools, sir.
A. Yes, sir. Q. Is that for each of the six (6) policies namely: Exhibits C, D, E, F, G and H?
Q. Did you also do this through your insurance agency? A. Yes, sir.
A. If you are referring to Forte Insurance Agency, yes. ATTY. MEJIA:
Q. Is Forte Insurance Agency a department or division of your company? What is your basis for stating that the coverage against earthquake shock as provided for in
A. No, sir. They are our insurance agency. each of the six (6) policies extend to the two (2) swimming pools only?
Q. And they are independent of your company insofar as operations are concerned? WITNESS:
A. Yes, sir, they are separate entity. Because it says here in the policies, in the enumeration Earthquake Shock Endorsement, in the
Q. But insofar as the procurement of the insurance policy is concerned they are of course Clauses and Warranties: Item 5 only (Earthquake Shock Endorsement), sir.
subject to your instruction, is that not correct? ATTY. MEJIA:
Witness referring to Exhibit C-1, your Honor. that we have issued with no specific attachments, premium rates and so on. It was inadvertent,
WITNESS: sir.
We do not normally cover earthquake shock endorsement on stand alone basis. For swimming The Court also rejects petitioners contention that respondents contemporaneous and subsequent acts
pools we do cover earthquake shock. For building we covered it for full earthquake coverage to the issuance of the insurance policy falsely gave the petitioner assurance that the coverage of the
which includes earthquake shock earthquake shock endorsement included all its properties in the resort. Respondent only insured the
COURT: properties as intended by the petitioner. Petitioners own witness testified to this agreement, viz:
As far as earthquake shock endorsement you do not have a specific coverage for other things CROSS EXAMINATION OF LEOPOLDO MANTOHAC
other than swimming pool? You are covering building? They are covered by a general TSN, January 14, 1992
insurance? pp. 4-5
WITNESS: Q. Just to be clear about this particular answer of yours Mr. Witness, what exactly did you tell
Earthquake shock coverage could not stand alone. If we are covering building or another we Atty. Omlas (sic) to copy from Exhibit H for purposes of procuring the policy from Philippine
can issue earthquake shock solely but that the moment I see this, the thing that comes to my Charter Insurance Corporation?
mind is either insuring a swimming pool, foundations, they are normally affected by A. I told him that the insurance that they will have to get will have the same provisions as this
earthquake but not by fire, sir. American Home Insurance Policy No. 206-4568061-9.
DIRECT EXAMINATION OF JUAN BARANDA III Q. You are referring to Exhibit H of course?
TSN, August 11, 1992 A. Yes, sir, to Exhibit H.
pp. 23-25 Q. So, all the provisions here will be the same except that of the premium rates?
Q. Plaintiffs witness, Mr. Mantohac testified and he alleged that only Exhibits C, D, E and F A. Yes, sir. He assured me that with regards to the insurance premium rates that they will be
inclusive [remained] its coverage against earthquake shock to two (2) swimming pools only charging will be limited to this one. I (sic) can even be lesser.
but that Exhibits G and H respectively entend the coverage against earthquake shock to all the CROSS EXAMINATION OF LEOPOLDO MANTOHAC
properties indicated in the respective schedules attached to said policies, what can you say TSN, January 14, 1992
about that testimony of plaintiffs witness? pp. 12-14
WITNESS: Atty. Mejia:
As I have mentioned earlier, earthquake shock cannot stand alone without the other half of it. Q. Will it be correct to state[,] Mr. Witness, that you made a comparison of the provisions and
I assure you that this one covers the two swimming pools with respect to earthquake shock scope of coverage of Exhibits I and H sometime in the third week of March, 1990 or
endorsement. Based on it, if we are going to look at the premium there has been no change thereabout?
with respect to the rates. Everytime (sic) there is a renewal if the intention of the insurer was A. Yes, sir, about that time.
to include the earthquake shock, I think there is a substantial increase in the premium. We are Q. And at that time did you notice any discrepancy or difference between the policy wordings
not only going to consider the two (2) swimming pools of the other as stated in the policy. As as well as scope of coverage of Exhibits I and H respectively?
I see, there is no increase in the amount of the premium. I must say that the coverage was not A. No, sir, I did not discover any difference inasmuch (sic) as I was assured already that the
broaden (sic) to include the other items. policy wordings and rates were copied from the insurance policy I sent them but it was only
COURT: when this case erupted that we discovered some discrepancies.
They are the same, the premium rates? Q. With respect to the items declared for insurance coverage did you notice any discrepancy
WITNESS: at any time between those indicated in Exhibit I and those indicated in Exhibit H respectively?
They are the same in the sence (sic), in the amount of the coverage. If you are going to do A. With regard to the wordings I did not notice any difference because it was exactly the
some computation based on the rates you will arrive at the same premiums, your Honor. same P393,000.00 on the two (2) swimming pools only against the peril of earthquake shock
CROSS-EXAMINATION OF JUAN BARANDA III which I understood before that this provision will have to be placed here because this
TSN, September 7, 1992 particular provision under the peril of earthquake shock only is requested because this is an
pp. 4-6 insurance policy and therefore cannot be insured against fire, so this has to be placed.
ATTY. ANDRES: The verbal assurances allegedly given by respondents representative Atty. Umlas were not proved.
Would you as a matter of practice [insure] swimming pools for fire insurance? Atty. Umlas categorically denied having given such assurances.
WITNESS: Finally, petitioner puts much stress on the letter of respondents independent claims adjuster, Bayne
No, we dont, sir. Adjusters and Surveyors, Inc. But as testified to by the representative of Bayne Adjusters and
Q. That is why the phrase earthquake shock to the two (2) swimming pools only was placed, Surveyors, Inc., respondent never meant to lead petitioner to believe that the endorsement for
is it not? earthquake shock covered properties other than the two swimming pools, viz:
A. Yes, sir. DIRECT EXAMINATION OF ALBERTO DE LEON (Bayne
ATTY. ANDRES: Adjusters and Surveyors, Inc.)
Will you not also agree with me that these exhibits, Exhibits G and H which you have pointed TSN, January 26, 1993
to during your direct-examination, the phrase Item no. 5 only meaning to (sic) the two (2) pp. 22-26
swimming pools was deleted from the policies issued by AIU, is it not? Q. Do you recall the circumstances that led to your discussion regarding the extent of
xxx coverage of the policy issued by Philippine Charter Insurance Corporation?
ATTY. ANDRES: A. I remember that when I returned to the office after the inspection, I got a photocopy of the
As an insurance executive will you not attach any significance to the deletion of the insurance coverage policy and it was indicated under Item 3 specifically that the coverage is
qualifying phrase for the policies? only for earthquake shock. Then, I remember I had a talk with Atty. Umlas (sic), and I relayed
WITNESS: to him what I had found out in the policy and he confirmed to me indeed only Item 3 which
My answer to that would be, the deletion of that particular phrase is inadvertent. Being a were the two swimming pools have coverage for earthquake shock.
company underwriter, we do not cover. . it was inadvertent because of the previous policies xxx
Q. Now, may we know from you Engr. de Leon your basis, if any, for stating that except for
the swimming pools all affected items have no coverage for earthquake shock?
xxx
A. I based my statement on my findings, because upon my examination of the policy I found
out that under Item 3 it was specific on the wordings that on the two swimming pools only,
then enclosed in parenthesis (against the peril[s] of earthquake shock only), and secondly,
when I examined the summary of premium payment only Item 3 which refers to the
swimming pools have a computation for premium payment for earthquake shock and all the
other items have no computation for payment of premiums.
In sum, there is no ambiguity in the terms of the contract and its riders. Petitioner cannot rely on the
general rule that insurance contracts are contracts of adhesion which should be liberally construed in
favor of the insured and strictly against the insurer company which usually prepares it. A contract of
adhesion is one wherein a party, usually a corporation, prepares the stipulations in the contract, while
the other party merely affixes his signature or his "adhesion" thereto. Through the years, the courts
have held that in these type of contracts, the parties do not bargain on equal footing, the weaker
party's participation being reduced to the alternative to take it or leave it. Thus, these contracts are
viewed as traps for the weaker party whom the courts of justice must protect. Consequently, any
ambiguity therein is resolved against the insurer, or construed liberally in favor of the insured.
The case law will show that this Court will only rule out blind adherence to terms where facts and
circumstances will show that they are basically one-sided. Thus, we have called on lower courts to
remain careful in scrutinizing the factual circumstances behind each case to determine the efficacy of
the claims of contending parties. In Development Bank of the Philippines v. National
Merchandising Corporation, et al., the parties, who were acute businessmen of experience, were
presumed to have assented to the assailed documents with full knowledge.
We cannot apply the general rule on contracts of adhesion to the case at bar. Petitioner cannot claim
it did not know the provisions of the policy. From the inception of the policy, petitioner had required
the respondent to copy verbatim the provisions and terms of its latest insurance policy from AHAC-
AIU. The testimony of Mr. Leopoldo Mantohac, a direct participant in securing the insurance policy
of petitioner, is reflective of petitioners knowledge, viz:
DIRECT EXAMINATION OF LEOPOLDO MANTOHAC
TSN, September 23, 1991
pp. 20-21
Q. Did you indicate to Atty. Omlas (sic) what kind of policy you would want for those
facilities in Agoo Playa?
A. Yes, sir. I told him that I will agree to that renewal of this policy under Philippine Charter
Insurance Corporation as long as it will follow the same or exact provisions of the previous
insurance policy we had with American Home Assurance Corporation.
Q. Did you take any step Mr. Witness to ensure that the provisions which you wanted in the
American Home Insurance policy are to be incorporated in the PCIC policy?
A. Yes, sir.
Q. What steps did you take?
A. When I examined the policy of the Philippine Charter Insurance Corporation I specifically told
him that the policy and wordings shall be copied from the AIU Policy No. 206-4568061-9.
010 Bonifacio Bros v. Mora (Valle) that Mora would insure the automobile with HS Reyes as beneficiary.
29 May 1967 | Castro, J. | Liability of insurer for loss 48.   The car was insured with State Bonding and Insurance Co and motor car
insurance policy was issued to Mora.42
PETITIONER: Bonifacio Bros. Inc, et al 49.   During effectivity of the contract, the car had an accident. The insurance
RESPONDENTS: Enrique Mora et al company assigned the accident to HH Bayne Adjustment Co for
investigation and apparaisal. Mora, without HS Reyes’ knowledge and
SUMMARY: consent, authorized Bonifacio Bros to furnish the labor and materials (some
Mora entered into a policy contract with State Bonding (Insurance company) supplied by Ayala Auto Parts).
covering an Oldsmobile Sedan Model 1956. This was mortgaged to HS Reyes 50.   For the cost of labor and materials, Mora was billed Php 2102.73 through
and HS Reyes was named as beneficiary in the policy. During the effectivity of HH Bayne Adjustment.
the contract, the car got into an accident. The insurance company had Bayne 51.   The insurane company after claiming a franchise of 100 pesos drew a check
Adjustment Co for the investigayion and appraisal of the damage done. Mora for php 2002.73 as proceeds payable to the order of Mora or HS Reyes and
also enlisted the help from Bonifacio Bros for the repair of the car. Bonifacio entrusted the check to HH Bayne for disposition and delivery to the proper
Bros used the parts of Ayala. Mora was billed 2102.73 for the cost of materials party.
and labor. The insurance company drew a check payable to Mora or to HS 52.   The car was delivered to Mora without the consent of HS Reyes and
Reyes. The car was delivered to Mora without paying Bonifacio and Ayala. Now without payment to Bonifacio Bros and Ayala.
Bonifacio and Ayala filed this case claiming that they are the ones entitled to the 53.   Bonifacio Bros and Ayala filed a complaint with the Municpal court of
proceeds of the insurance. Manila against Mora and State Bonding on the theory that the proceeds be
paid directly to them.
The issue in this case is whether or not Bonifacio and Ayala are entitled to the 54.   The insurance company filed an answer with counterclaim for interpleader,
proceeds. To answer this issue, we must also answer the question of whether or requiring Bonifacio and HS Reyes to interplead to determine who has a
nt Bonifacio and Ayala are parties to the insurance contract. In relation to the better rght to the proceeds. Mora was declared in default.
topic for class, the issue is whether or not the contention of Bonifacio and Ayala 55.   Municipal court: declared HS Reyes as having better right to the proceeds
that the right of HS Reyes as beneficiary to the proceeds arises only if tehre was and ordered State Bonding to pay
loss and not damage. They argue that damage is in this case and not loss. 56.   CFI: affirmed the lower court’s decision. Bonifacio and Ayala moved for
reconsideration but was denied. Hence this appeal.
Bonifacio and Ayala are not parties to the contract because if the parties truly 57.   Bonifacio and Ayala argue that the insurance company andMora are parties
intended to include them, it should have been shown in the policy. Also, an to the repair of the car as well as the wages due. They based this argument
insurance contract is between the insured and insurer and a third party has no on par 4 of the insurance contract (see end of digest). They claim that
rights to the proceeds of it. Bayne’s recommendation of payment of the bill of materials and repairs for
which Bonifacio and Ayala drew a check indicates that Mora and Bayne
The court held that there should be no distinction between loss and damage since acted and in representation of the insurance company.
loss necessarily includes damage. Loss is defined as the injury or damage 58.   Bonifacio and Aayla contend that the right of HS Reyes to the proceeds
sustained by the insured in consequence of the happening of one or ore if the arises only if there was loss and not where there is mere damage.
accidents or misfortune against which the insurer, in consideration of the
premium, has undertaen to indemnifty the insured. (Black Law’s dictionary). ISSUE/s:
10.   WoN there is privity of contract between Bonifacio and ayala on one hand
DOCTRINE: and insurance company on another– No. The contract is between the
Loss in insurance is defined as the injury or damage sustained by the insured in insurance ompany and Mora. If tehre was any intention to include them, it
consequence of the happening of one or ore if the accidents or misfortune against should have been stipulated in the contract. But there is none here.
which the insurer, in consideration of the premium, has undertaen to indemnifty 11.   WoN the insurance proceeds arises only if there was loss and not where
the insured. (Black Law’s dictionary). there is mere damage as in the instant case – The court held that there
shouldn’t be a distinction between the two words: loss and damage.
Since loss, in insurance law, embraces damage.
FACTS:
47.   Enrique Mora is the owner of an Oldsmobile Sedan Model 1956 with plate                                                                                                                        
number QC-8088. This was mortgaged to HS Reyes Inc with the condition See  end  for  the  contents  of  the  policy  
62.   Here, there is no trust, expressed or implied.
RULING: Accordingly, the judgment appealed from is hereby affirmed, at 63.   No cause of action exists in favor of Bonifacio and Ayala in so far as the
appellants' cost. proceeds are concerned. Their claim, if at all, is merely equitable in nature
and must be made through Mora who entered into a contract with them.
RATIO: 64.   The situation is also covered by sec. 50 of the insurance act which reads:
53.   The SC held that Bonifacio and Ayala’s argument is beside the “The insurance shall be applied exclusively to the proper intrest of the
pointbecause from the facts and pleadings, it will be seen that Bonifacio and person in whose name it is made unless otherwise specified in the policy.”
Ayala’s cause of action is upon the terms of the insurance contract. They 65.   The policy here has been framed that. “Loss if any is payable to HS REYES
seek to recover the insurance proceeds and for this purpose, they relyon par INC.” which unmistajably shows the intention of the parties.
4. They are not mentioned in the contract as parties nor is there any clause ISSUE ABOUT LOSS
or provision from which the court can infer there is an obligatioj on the part 66.   Any attempt to draw a distinction between loss and damage is uncalled for
of the insurance company to pay the cost of repairs directly to them. because the word loss in insurance embraces injury or damage.
54.   It is fundamental that contracts take effect only between the parties thereto. 67.   Loss in insurance is defined as the injury or damage sustained by the
Except in some specific instances provided by. Law where the contract insured in consequence of the happening of one or ore if the accidents or
contains some stipulation in favor of a third person or a stipulation pour misfortune against which the insurer, in consideration of the premium, has
autrui. undertaen to indemnifty the insured. (Black Law’s dictionary).
55.   Under this doctrine, a third person is allowed to avail himself of a benefit 68.   A loss may either be total or partial.
granted to him by the terms of the contract, provided that the contracting
parties have clearly and deliberately conferred a favor upon such person. "1. The Company (referring to the State Bonding & Insurance Co., Inc) will, subject
Consequently a third person not a party to the contract has no action against to the Limits of Liability, indemnify the Insured against loss of or damages to the
the parties thereto, and cannot generally demand the enforcement of the Motor Vehicle and its accessories and spare parts whilst thereon; (a) by accidental
same.
collision or overturning or collision or overturning consequently upon mechanical
56.   The fairest test to determine whether the interest of a third person in a
contract is a stipulation pour autrui or merely an incidental interest, is to breakdown or consequent upon wear and tear.
rely upon the intention of the parties as disclosed by their contract.
57.   Here, the contract does not contain any words or clauses to disclose the
intent to give any benefit to any repairmen or material men in case of repair.
The parties omitted such stipulation which is a circumstance that supports xxx xxx xxx
the conclusion.
58.   The “loss payable clause” in the policy stipulates that the loss if any is
payable to HS Reyes.
59.   If the insurance company had the intention to make itself liable to the repair 2. At its own option the Company may pay in cash the amount of the loss or damage
shop or material men, it could have easily inserted a stipulation to that or may repair, reinstate, or replace the Motor Vehicle or any part thereof or its
effect. accessories or spare parts. The liability of the Company shall not exceed to value of
60.   A perusal of par 4 would show that instead of establishing privity between the parts whichever is the less. The Insured's estimate of value stated in the schedule
the appellant and the insurance company, such stipulation merely will be the maximum amount payable by the Company in respect of any claim for
establishes the procedure that the insured has to follow in order to be loss or damage.
entitled to indemnity for repair. This paragraph therefore should not be
construed as bringing into existence in favor of the appellants a right of
action against the insurance company as such intention can never be
inferred therefrom.
xxx xxx xxx
61.   Also, a policy of insurance is a distinct and independent contract
between the insured and insurer, and third persons have no right either
in a court of equity, or in a court of law, to the proceeds of it, unless
there be some contract of trust, expressed or implied, by the insured and
4. The Insured may authorize the repair of the Motor Vehicle necessitated by
third person.
damage for which the Company may he liable under this Policy provided that: —
(a) The estimated cost of such repair does not exceed the Authorized Repair
Limit. (b) A detailed estimate of the cost is forwarded to the Company without
delay, subject to the condition that 'Loss, if any, is payable to H.S. Reyes, Inc.', by
virtue of the fact that said Oldsmobile sedan was mortgaged in favor of the said H.S.
Reyes, Inc. and that under a clause in said insurance policy, any loss was made
payable to the H.S. Reyes, Inc. as Mortgagee;
11 Finman General Assurance v. CA (VARGAS) Carlie Surposa – insured; deceased
1992 September 2 | Nocon, J. | Liability of insurer for loss Type of Policy: Personal accident insurance policy
10.   Carlie Surposa was insured with Finman under Finman General Teachers
PETITIONER: Finman General Assurance Corporation Protection Plan together with his parents, Sps. Julia and Carlos Surposa, and
RESPONDENTS: The Honorable Court of Appeals and Julia Surposa brothers.
SUMMARY: Carlie Surposa is insured with Finman under a Personal Accident 11.   While said insurance was in full force and effect, the insured Carlie, died a
Insurance Policy. He died as a result of a stab wound without provocation and result of the stab wound inflicted by one of the three unidentified men
warning on the part of Carlie as he was just waiting for a ride home after without provocation and warning on the part of Carlie as he and his cousin
attending the Maskarra Annual Festival. Julia Surposa filed a notice of claim were waiting for a ride home after attending the celebration of Maskarra
which denied the claim saying that murder and assault are not within the scope Annual Festival.
of the insurance policy since death resulting from murder/assault is not 12.   Thereafter, private respondent Surosa and other beneficiaries of said policy
accidental but rather a deliberate or intentional act. The issue in this case is filed a written notice of claim with Finman which denied said claim
WON Finman is liable to pay Julia Surposa the proceeds of personal accident contending that murder and assault are not within the scope of the coverage
insurance policy based on the murder of Carlie Surposa. The Court ruled in the of the insurance policy.
affirmative. An accident is an event that takes place without one's foresight or 13.   Both the Insurance Commission and the appellate court ruled against
expectation — an event that proceeds from an unknown cause, or is an unusual Finman and held that it is liable to pay the proceeds of the policy.
effect of a known cause and, therefore, not expected. The generally accepted rule 14.   Finman filed this present petition alleging grave abuse of discretion on the
is that, death or injury does not result from accident or accidental means within part of the appellate court in applying the principle of expresso unius
the terms of an accident-policy if it is the natural result of the insured's voluntary exclusion alterius in a personal accident insurance policy since death
act, unaccompanied by anything unforeseen except the death or injury. In this resulting from murder/assault are impliedly excluded in said insurance
case, the insured died from an event that took place without his foresight or policy considering that the cause of death was not accidental but rather a
expectation, an event that proceeded from an unusual effect of a known cause deliberate and intentional act.
and, therefore, not expected. Neither can it be said that where was a capricious
desire on the part of the accused to expose his life to danger considering that he ISSUE: WON Finman is liable to pay Julia Surposa the proceeds of personal
was just going home after attending a festival. Furthermore, the personal accident insurance policy based on the murder of Carlie Surposa – YES. Where the
accident insurance policy involved herein specifically enumerated only ten (10) death or injury is not the natural or probable result of the insured's voluntary act, or if
circumstances wherein no liability attaches to petitioner insurance company for something unforeseen occurs in the doing of the act which produces the injury, the
any injury, disability or loss suffered by the insured as a result of any of the resulting death is within the protection of the policies insuring against death or injury
stimulated causes. The principle of " expresso unius exclusio alterius" — the
from accident.
mention of one thing implies the exclusion of another thing — is therefore
applicable in the instant case since murder and assault, not having been expressly
RULING: WHEREFORE, finding no irreversible error in the decision of the
included in the enumeration of the circumstances that would negate liability in
said insurance policy cannot be considered by implication to discharge the respondent Court of Appeals, the petition for certiorari with restraining order and
petitioner insurance company from liability for, any injury, disability or loss preliminary injunction is hereby DENIED for lack of merit.
suffered by the insured. It is well settled that contracts of insurance are to be
construed liberally in favor of the insured and strictly against the insurer. Thus RATIO:
ambiguity in the words of an insurance contract should be interpreted in favor of 69.   The terms "accident" and "accidental" as used in insurance contracts have
its beneficiary. not acquired any technical meaning, and are construed by the courts in their
DOCTRINE: Where the death or injury is not the natural or probable result of ordinary and common acceptation. Thus, the terms have been taken to mean
the insured's voluntary act, or if something unforeseen occurs in the doing of the that which happen by chance or fortuitously, without intention and design,
act which produces the injury, the resulting death is within the protection of the and which is unexpected, unusual, and unforeseen. An accident is an event
policies insuring against death or injury from accident. that takes place without one's foresight or expectation — an event that
FACTS: proceeds from an unknown cause, or is an unusual effect of a known cause
9.   Characters: and, therefore, not expected.
70.   The generally accepted rule is that, death or injury does not result from
Finman General Assurance Corporation – insurance company
accident or accidental means within the terms of an accident-policy if it is
Julia Surposa – assured
the natural result of the insured's voluntary act, unaccompanied by anything
unforeseen except the death or injury. There is no accident when a
deliberate act is performed unless some additional, unexpected,
independent, and unforeseen happening occurs which produces or brings
about the result of injury or death. In other words, where the death or injury
is not the natural or probable result of the insured's voluntary act, or if
something unforeseen occurs in the doing of the act which produces the
injury, the resulting death is within the protection of the policies insuring
against death or injury from accident.
71.   In the case at bar, it cannot be pretended that Carlie Surposa died in the
course of an assault or murder as a result of his voluntary act considering
the very nature of these crimes. In the first place, the insured and his
companion were on their way home from attending a festival. They were
confronted by unidentified persons. The record is barren of any
circumstance showing how the stab wound was inflicted. Nor can it be
pretended that the malefactor aimed at the insured precisely because the
killer wanted to take his life. In any event, while the act may not exempt the
unknown perpetrator from criminal liability, the fact remains that the
happening was a pure accident on the part of the victim. The insured died
from an event that took place without his foresight or expectation, an
event that proceeded from an unusual effect of a known cause and,
therefore, not expected. Neither can it be said that where was a
capricious desire on the part of the accused to expose his life to danger
considering that he was just going home after attending a festival.
72.   Furthermore, the personal accident insurance policy involved herein
specifically enumerated only ten (10) circumstances wherein no liability
attaches to petitioner insurance company for any injury, disability or loss
suffered by the insured as a result of any of the stimulated causes. The
principle of " expresso unius exclusio alterius" — the mention of one thing
implies the exclusion of another thing — is therefore applicable in the
instant case since murder and assault, not having been expressly included in
the enumeration of the circumstances that would negate liability in said
insurance policy cannot be considered by implication to discharge the
petitioner insurance company from liability for, any injury, disability or loss
suffered by the insured. Thus, the failure of the petitioner insurance
company to include death resulting from murder or assault among the
prohibited risks leads inevitably to the conclusion that it did not intend to
limit or exempt itself from liability for such death.
73.   Article 1377 of the Civil Code of the Philippines provides that the
interpretation of obscure words or stipulations in a contract shall not favor
the party who caused the obscurity. Moreover, it is well settled that
contracts of insurance are to be construed liberally in favor of the insured
and strictly against the insurer. Thus ambiguity in the words of an insurance
contract should be interpreted in favor of its beneficiary.
012 FORTUNE INSURANCE v. CA (VICENCIO) FACTS:
May 23, 1995 | Davide, Jr., J. | Liability of insurer for loss 37.   Producers Bank of the Philippines (Producer) was insured by defendant
Fortune Insurance and Surety Co., Inc. (Fortune) and an insurance policy
PETITIONER: Fortune Insurance and Surety Co., Inc. was issued.
RESPONDENTS: Court of Appeals, and Producers Bank of the Philippines 38.   An armored car of the Producer, while in the process of transferring cash in
the sum of P725,000.00 under the custody of its teller, Maribeth Alampay,
SUMMARY: Producers Bank was insured by Fortune Insurance through a theft or from its Pasay Branch to its Head Office at Makati, Metro Manila on June
robbery insurance policy. During the term of the policy, an armored car of Producer, 29, 1987, was robbed of the said cash. The robbery took place while the
while in the process of transferring cash under the custoy of its teller, from its Pasay armored car was traveling along Taft Avenue in Pasay City;
Branch to its Head Office in Makati, was robbed of the said cash. The armored car 39.   The said armored car was driven by Benjamin Magalong Y de Vera,
was escorted by Security Guard Atiga (assigned by Unicorn Security Services), and escorted by Security Guard Saturnino Atiga Y Rosete. Driver Magalong
was driven by Driver Magalong (assigned by PRC Managament Systems). After an was assigned by PRC Management Systems with the Producers by virtue of
investigation, Guard Atiga and Driver Magalong, were charged, together with a few an Agreement.
others for violation of PD 532 (Anti-Highway Robbery Law). Fortune refused to pay 40.   The Security Guard Atiga was assigned by Unicorn Security Services, Inc.
as the loss is excluded from the coverage of the insurance policy which excludes: with the Producers by virtue of a contract of Security Service.
“any loss caused by any dishonest, fraudulent or criminal act of the insured or any 41.   After an investigation conducted by the Pasay police authorities, the driver
officer, employee, partner, director, trustee or authorized representative of the Magalong and guard Atiga were charged, together with Edelmer Bantigue
Insured whether acting alone or in conjunction with others.” The trial court ruled in Y Eulalio, Reynaldo Aquino and John Doe, with violation of P.D. 532
favor of Producers. It held that their services were merely offered by PRC (Anti-Highway Robbery Law) before the Fiscal of Pasay City.
Managament and Unicorn Services and assigned by them to Producers. Fortune 42.   The Fiscal of Pasay City then filed an information charging the aforesaid
however insists that under the four-fold test, it is Producers who exercises control persons with the said crime before RTC of Pasay City. The case is still
over the Atiga and Magalong, and that PRC Management Systems and Unicorn being tried as of this date;
Services are only labor-only contractors. The CA however, affirmed the TC decision. 43.   Demands were made by the Producers upon the defendant Fortune to pay
the amount of the loss of P725,000.00, but the latter refused to pay as the
The SC sides with Fortune. Although there is paucity in evidence in saying that PRC loss is excluded from the coverage of the insurance policy, specifically
and Unicorn are labor-only contractors, and hence it may not be said with clearness under page 1 thereof, "General Exceptions" Section (b),
that Atiga and Magalong were indeed “employees” of Producers, the Court held that 44.   The said section reads as follows: “any loss caused by any dishonest,
regardless, they are undoubtedly under the “authorized representatives” part of the fraudulent or criminal act of the insured or any officer, employee, partner,
exception clause. Producers entrusted the three (with the teller Alampay) with the director, trustee or authorized representative of the Insured whether acting
specific duty to safely transfer the money to its head office, with Alampay to be alone or in conjunction with others”
responsible for its custody in transit; Magalong to drive the armored vehicle which 45.   Producers opposes the contention of the Fortune and contends that Atiga
would carry the money; and Atiga to provide the needed security for the money, the and Magalong are not “its officer, employee, trustee or authorized
vehicle, and his two other companions. representative, at the time of the robbery.”
46.   The Trial Court rendered a decision in favor of Producers, ordering Fortune
DOCTRINE: It has been aptly observed that in burglary, robbery, and theft to pay the net amount of Php 540,000 as liability under the policy, with
insurance, the opportunity to defraud the insurer — the moral hazard — is so great special clause deductions.
that insurers have found it necessary to fill up their policies with countless 47.   The TC ruled that Magalong and Atiga were not employees or
restrictions, many designed to reduce this hazard. Persons frequently excluded under representatives of Producers.
such provisions are those in the insured’s service and employment. The purpose of 48.   The Court is satisfied that Producers may not be said to have selected and
the exception is to guard against liability should the theft be committed by one engaged Magalong and Atiga, their services as armored car driver and as
having unrestricted access to the property. In such cases, the terms specifying the security guard having been merely offered by PRC Management and by
excluded classes are to be given their meaning as understood in common speech. The Unicorn Security and which latter firms assigned them to Producers. The
terms “service” and “employment” are generally associated with the idea of wages and salaries of both Magalong and Atiga are presumably paid by
selection, control, and compensation. their respective firms, which alone wields the power to dismiss them.
49.   Neither is the Court prepared to accept the proposition that driver Magalong
and guard Atiga were the "authorized representatives" of Producers. They
were merely an assigned armored car driver and security guard, exempt from liability under the general exceptions clause of the insurance
respectively, for the June 29, 1987 money transfer from plaintiff's Pasay policy.
Branch to its Makati Head Office.
50.   Quite plainly — it was teller Maribeth Alampay who had "custody" of the RULING: WHEREFORE , the instant petition is hereby GRANTED. The decision
P725,000.00 cash being transferred along a specified money route, and of the Court of Appeals in CA-G.R. CV No. 32946 dated 3 May 1994 as well as that
hence plaintiff's then designated "messenger" adverted to in the policy. of Branch 146 of the Regional Trial Court of Makati in Civil Case No. 1817 are
51.   The CA affirmed the decision in toto, saying that the language used by REVERSED and SET ASIDE. The complaint in Civil Case No. 1817 is
defendant-appellant in the above quoted stipulation is plain, ordinary and DISMISSED.
simple. No other interpretation is necessary. The word "employee" must be
taken to mean in the ordinary sense. RATIO:
52.   Said driver and security guard cannot be considered as employees of 38.   It should be noted that the insurance policy entered into by the parties is a
Producers bank because it has no power to hire or to dismiss said driver and theft or robbery insurance policy which is a form of casualty insurance.
security guard under the contracts except only to ask for their replacements Section 174 of the Insurance Code provides:
from the contractors. 1.   Sec. 174. Casualty insurance is insurance covering loss or liability
53.   According to Fortune, when Producers commissioned a guard and a driver arising from accident or mishap, excluding certain types of loss
to transfer its funds from one branch to another, they effectively and which by law or custom are considered as falling exclusively
necessarily became its authorized representatives in the care and custody of within the scope of insurance such as fire or marine. It includes,
the money. Assuming that they could not be considered authorized but is not limited to, employer's liability insurance, public liability
representatives, they were, nevertheless, employees of Producers. insurance, motor vehicle liability insurance, plate glass
54.   It asserts that the existence of an employer-employee relationship "is insurance, burglary and theft insurance, personal accident and
determined by law and being such, it cannot be the subject of agreement." health insurance as written by non-life insurance companies, and
55.   Thus, if there was in reality an employer-employee relationship between other substantially similar kinds of insurance.
Producers, on the one hand, and Magalong and Atiga, on the other, the 39.   It has been aptly observed that in burglary, robbery, and theft insurance,
provisions in the contracts of Producers with PRC Management System for “the opportunity to defraud the insurer — the moral hazard — is so great
Magalong and with Unicorn Security Services for Atiga which state that that insurers have found it necessary to fill up their policies with countless
Producers is not their employer and that it is absolved from any liability as restrictions, many designed to reduce this hazard. Seldom does the insurer
an employer, would not obliterate the relationship. assume the risk of all losses due to the hazards insured against.”
56.   Fortune points out that an employer-employee relationship depends upon 40.   Persons frequently excluded under such provisions are those in the
four standards: (1) the manner of selection and engagement of the putative insured’s service and employment. The purpose of the exception is to guard
employee; (2) the mode of payment of wages; (3) the presence or absence against liability should the theft be committed by one having unrestricted
of a power to dismiss; and (4) the presence and absence of a power to access to the property. In such cases, the terms specifying the excluded
control the putative employee's conduct. classes are to be given their meaning as understood in common speech. The
57.   Of the four, the right-of-control test has been held to be the decisive factor. terms “service” and “employment” are generally associated with the idea of
It asserts that the power of control over Magalong and Atiga was vested in selection, control, and compensation.
and exercised by Producers. Fortune further insists that PRC Management 41.   A contract of insurance is a contract of adhesion, thus any ambiguity therein
System and Unicorn Security Services are but "labor-only" contractors should be resolved against the insurer, or it should be construed liberally in
under Article 106 of the Labor Code. favor of the insured and strictly against the insurer.
58.   Fortune thus contends that Magalong and Atiga were employees of 42.   Limitations of liability should be regarded with extreme jealousy and must
Producers, following the ruling in International Timber Corp. vs. NLRC that be construed in such a way, as to preclude the insurer from non-compliance
a finding that a contractor is a "labor-only" contractor is equivalent to a with its obligation.
finding that there is an employer-employee relationship between the owner 43.   It goes without saying then that if the terms of the contract are clear and
of the project and the employees of the "labor-only" contractor. unambiguous, there is no room for construction and such terms cannot be
enlarged or diminished by judicial construction.
ISSUE/s: 44.   With the foregoing principles in mind, it may now be asked whether
5.   WoN the Fortune Insurance is liable under the Money, Security and Payroll Magalong and Atiga qualify as employees or authorized representatives of
Robbery policy it issued to Producers Bank – NO, because Fortune is Producers under paragraph (b) of the general exceptions clause of the policy
which, for easy reference, is again quoted: “any loss caused by any vehicle which would carry the money; and Atiga to provide the needed
dishonest, fraudulent or criminal act of the insured or any officer, employee, security for the money, the vehicle, and his two other companions.
partner, director, trustee or authorized representative of the Insured whether 54.   In short, for these particular tasks, the three acted as agents of
acting alone or in conjunction with others.” Producers. A "representative" is defined as one who represents or
45.   There is marked disagreement between the parties on the correct meaning of stands in the place of another; one who represents others or another in
the terms "employee" and "authorized representatives." a special capacity, as an agent, and is interchangeable with "agent."
46.   It is clear to us that insofar as Fortune is concerned, it was its intention
to exclude and exempt from protection and coverage losses arising
from dishonest, fraudulent, or criminal acts of persons granted or
having unrestricted access to Producers' money or payroll.
47.   When it used then the term "employee," it must have had in mind any
person who qualifies as such as generally and universally understood,
or jurisprudentially established in the light of the four standards in the
determination of the employer-employee relationship, or as statutorily
declared even in a limited sense as in the case of Article 106 of the
Labor Code which considers the employees under a "labor-only"
contract as employees of the party employing them and not of the party
who supplied them to the employer.
48.   Fortune claims that Producers’ contracts with PRC Management Systems
and Unicorn Security Services are "labor-only" contracts.
49.   Producers, however, insists that by the express terms thereof, it is not the
employer of Magalong.
50.   Notwithstanding such express assumption of PRC Management Systems
and Unicorn Security Services that the drivers and the security guards each
shall supply to Producers are not the latter's employees, it may, in fact, be
that it is because the contracts are, indeed, "labor-only" contracts.
51.   Whether they are is, in the light of the criteria provided for in Article
106 of the Labor Code, a question of fact. Since the parties opted to
submit the case for judgment on the basis of their stipulation of facts
which are strictly limited to the insurance policy, the contracts with PRC
Management Systems and Unicorn Security Services, the complaint for
violation of P.D. No. 532, and the information therefor filed by the City
Fiscal of Pasay City, there is a paucity (insuffiency) of evidence as to
whether the contracts between Producers and PRC Management
Systems and Unicorn Security Services are "labor-only" contracts.
52.   But even granting for the sake of argument that these contracts were
not "labor-only" contracts, and PRC Management Systems and Unicorn
Security Services were truly independent contractors, we are satisfied that
Magalong and Atiga were, in respect of the transfer of Producer's
money from its Pasay City branch to its head office in Makati, its
"authorized representatives" who served as such with its teller
Maribeth Alampay.
53.   Howsoever viewed, Producers entrusted the three with the specific duty
to safely transfer the money to its head office, with Alampay to be
responsible for its custody in transit; Magalong to drive the armored
013 DBP POOL v. RADIO MINDANAO (Villavicencio) namely:
January 27, 2006 | Austria-Martinez, J. | Liability of insurer for loss (c)War, invasion, act of foreign enemy, hostilities, or warlike operations
(whether war be declared or not), civil war.
PETITIONER: DBP POOL OF ACCREDITED INSURANCE COMPANIES (d) Mutiny, riot, military or popular rising, insurrection, rebellion, revolution,
RESPONDENTS: RADIO MINDANAO NETWORK,
INC.
 military or usurped power.
7.   The insurance companies maintained that the evidence showed that the fire was
SUMMARY: Radio Mindanao was insured by fire insurance policies from caused by members of the Communist Party of the Philippines/New Peoples
Provident and DBP Pool for its equipment and transmitter facilities. Radio Mindanao Army (CPP/NPA); and consequently, denied the claims.
station located in SSS Building, Bacolod City, was razed by fire. Radio Mindanao 8.   Radio Mindanao filed Civil Case No. 90-602 against DBP Pool and Provident
sought recovery under the two insurance policies but the claims were denied on the for the proceeds of the insurance policies.
ground that the cause of loss was an excepted risk that the loss or damage was 9.   Regional Trial Court of Makati, Branch 138, rendered a decision in favor of
because of rebellion, specifically the insurance companies maintained that the Radio Mindanao, ordered insurance companies to pay.
evidence showed that the fire was caused by members of the Communist Party of the 10.   CA affirmed the decision, with the modification that the applicable interest rate
Philippines/New Peoples Army. Radio Mindanao filed Civil Case No. 90-602 was reduced to 6% per annum.
against DBP Pool and Provident for the proceeds of the insurance policies. RTC and 11.   The only evidence which the Court can consider to determine if the fire was due
CA ruled in favor of Radio Mindanao because there is no evidence to prove the to the intentional act committed by the members of the New Peoples Army
attack of the CPP/NPA. WoN the burden of proof rests upon the insured to prove (NPA), are the testimony [sic] of witnesses Lt. Col. Nicolas Torres and SPO3
that it is not part of the excepted risk – NO, an insurer seeking to defeat a claim Leonardo Rochar who were admittedly not present when the fire occurred.
because of an exception or limitation in the policy has the burden of proving that the 12.   They were informed by bystanders that heavily armed men entered the
loss comes within the purview of the exception or limitation set up. It is sufficient for transmitter house, poured gasoline in (sic) it and then lighted it. After that, they
Radio Mindanao to prove the fact of damage or loss. Once Radio Mindanao makes went out shouting Mabuhay ang NPA.
out a prima facie case in its favor, the duty or the burden of evidence shifts DBP 13.   The persons whom they investigated and actually saw the burning of the station
Pool to controvert Radio Mindanao prima facie case. It is only when DBP Pool has were not presented as witnesses.
sufficiently proven that the damage or loss was caused by an excepted risk does the 14.   The documentary evidence particularly Exhibits 5 and 5-C do not satisfactorily
burden of evidence shift back to Radio Mindanao who is then under a duty of prove that the author of the burning were members of the NPA.
producing evidence to show why such excepted risk does not release DBP Pool from 15.   Exhibit 5-B which is a letter released by the NPA merely mentions some
any liability. dissatisfaction with the activities of some people in the media in Bacolod. There
was no mention there of any threat on media facilities.
16.   Even SFO III Rochas admitted that he was not sure that the said armed men
DOCTRINE: An insurer seeking to defeat a claim because of an exception or
were members of the CPP-NPA
limitation in the policy has the burden of proving that the loss comes within the
17.   In fact the only person who seems to be so sure that that the CPP-NPA had a
purview of the exception or limitation set up.
hand in the burning of DYHB was Lt. Col. Nicolas Torres.
18.   We cannot nevertheless admit his testimony as conclusive proof that the CPP-
FACTS: NPA was really involved in the incident considering that he admitted that he did
1.   Radio Mindanao owns several broadcasting stations all over the country. not personally see the armed men even as he tried to pursue them.
2.   Provident covered Radio Mindanao’s transmitter equipment and generating set 19.   Anent the letter of a certain Celso Magsilang, who claims to be a member of
for the amount of P13,550,000.00 under Fire Insurance Policy No. 30354. NPA-NIROC, being an admission of person which is not a party to the present
3.   DBP Pool covered respondents transmitter, furniture, fixture and other action, is likewise inadmissibl
transmitter facilities for the amount of P5,883,650.00 under Fire Insurance 20.   The reason being that an admission is competent only when the declarant, or
Policy No. F-66860. someone identified in legal interest with him, is a party to the action.
4.   Radio Mindanao station located in SSS Building, Bacolod City, was razed by
fire causing damage in the amount of P1,044,040.00. ISSUE/s:
5.   Radio Mindanao sought recovery under the two insurance policies but the 6.   WoN the fire was caused by an excepted risk, thus the insurance companies are
claims were denied on the ground that the cause of loss was an excepted risk not liable – NO, because evidence failed to show that the fire was indeed caused
excluded under condition no. 6 (c) and (d), to wit: by NPA members as the evidence were mere hearsay.
6.   This insurance does not cover any loss or damage occasioned by or through or in
consequence, directly or indirectly, of any of the following consequences,
7.   WoN the burden of proof rests upon the insured to prove that it is not part of the obtain a favorable judgment.
excepted risk – NO, an insurer seeking to defeat a claim because of an exception 61.   an insurer seeking to defeat a claim because of an exception or limitation in
or limitation in the policy has the burden of proving that the loss comes within the policy has the burden of proving that the loss comes within the purview
the purview of the exception or limitation set up. It is sufficient for Radio of the exception or limitation set up. If a proof is made of a loss apparently
Mindanao to prove the fact of damage or loss. Once Radio Mindanao makes out within a contract of insurance, the burden is upon the insurer to prove that the
a prima facie case in its favor, the duty or the burden of evidence shifts DBP loss arose from a cause of loss which is excepted or for which it is not liable, or
Pool to controvert Radio Mindanao prima facie case. It is only when DBP Pool from a cause which limits its liability.
has sufficiently proven that the damage or loss was caused by an excepted risk 62.   Consequently, it is sufficient for Radio Mindanao to prove the fact of damage or
does the burden of evidence shift back to Radio Mindanao who is then under a loss. Once Radio Mindanao makes out a prima facie case in its favor, the duty or
duty of producing evidence to show why such excepted risk does not release the burden of evidence shifts DBP Pool to controvert Radio Mindanao prima
DBP Pool from any liability. facie case.
63.   It is only when DBP Pool has sufficiently proven that the damage or loss was
RULING: WHEREFORE, the petition is DISMISSED. The Court of Appeals caused by an excepted risk does the burden of evidence shift back to Radio
Decision dated November 16, 2000 and Resolution dated January 30, 2001 rendered Mindanao who is then under a duty of producing evidence to show why such
in CA-G.R. CV No. 56351 are AFFIRMED in toto. excepted risk does not release DBP Pool from any liability.
64.   Unfortunately for DBP Pool, it failed to discharge its primordial burden of
RATIO: proving that the damage or loss was caused by an excepted risk.
55.   Both the trial court and the CA were correct in ruling that petitioner failed to
prove that the loss was caused by an excepted risk. Res Gestae
56.   DBP Pool argues that Radio Mindanao is responsible for proving that the cause
of the damage/loss is covered by the insurance policy, as stipulated in the 1.   DBP Pool argues that the trial court and the CA erred in not appreciating the
insurance policy, to wit: reports of witnesses Lt. Col Torres and SFO II Rochar that the bystanders they
Any loss or damage happening during the existence of abnormal conditions interviewed claimed that the perpetrators were members of the CPP/NPA as an
(whether physical or otherwise) which are occasioned by or through in exception to the hearsay rule as part of res gestae.
consequence directly or indirectly, of any of the said occurrences shall be 2.   Res gestae, as an exception to the hearsay rule, refers to those exclamations and
deemed to be loss or damage which is not covered by the insurance, except to statements made by either the participants, victims, or spectators to a crime
the extent that the Insured shall prove that such loss or damage happened immediately before, during, or after the commission of the crime, when the
independently of the existence of such abnormal conditions. circumstances are such that the statements were made as a spontaneous reaction
In any action, suit or other proceeding where the Companies allege that by or utterance inspired by the excitement of the occasion and there was no
reason of the provisions of this condition any loss or damage is not covered by opportunity for the declarant to deliberate and to fabricate a false statement.
this insurance, the burden of proving that such loss or damage is covered shall 3.   The rule in res gestae applies when the declarant himself did not testify and
be upon the Insured. provided that the testimony of the witness who heard the declarant complies
57.   Limitations of liability should be regarded with extreme jealousy and must be with the following requisites: (1) that the principal act, the res gestae, be a
construed in such a way as to preclude the insurer from noncompliance with its startling occurrence; (2) the statements were made before the declarant had the
obligations. time to contrive or devise a falsehood; and (3) that the statements must concern
58.   The burden of proof contemplated by the aforesaid provision actually refers to the occurrence in question and its immediate attending circumstances.
the burden of evidence (burden of going forward).As applied in this case, it 4.   The Court is not convinced to accept the declarations as part of res gestae.
refers to the duty of the insured to show that the loss or damage is covered by 5.   it cannot be said however, that these utterances were made spontaneously by
the policy. the bystanders and before they had the time to contrive or devise a falsehood.
59.   The foregoing clause notwithstanding, the burden of proof still rests upon DBP 6.   Both SFO III Rochar and Lt. Col. Torres received the bystanders statements
Pool to prove that the damage or loss was caused by an excepted risk in order to while they were making their investigations during and after the fire. It is
escape any liability under the contract. reasonable to assume that when these statements were noted down, the
60.   Burden of proof is the duty of any party to present evidence to establish his bystanders already had enough time and opportunity to mill around, talk to one
claim or defense by the amount of evidence required by law, which is another and exchange information, not to mention theories and speculations, as
preponderance of evidence in civil cases. The party, whether plaintiff or is the usual experience in disquieting situations where hysteria is likely to take
defendant, who asserts the affirmative of the issue has the burden of proof to place. It cannot therefore be ascertained whether these utterances were the
products of truth.
7.   Furthermore, admissibility of evidence should not be equated with its weight
and sufficiency. Admissibility of evidence depends on its relevance and
competence
8.   These declarations should be calibrated vis--vis the other evidence on record.
9.   The documentary evidence presented by DBP Pool, i.e., (1) the police blotter;
(2) the certification from the Bacolod Police Station; and (3) the Fire
Investigation Report, none of these documents categorically stated that the
perpetrators were members of the CPP/NPA.
10.   All these documents show that indeed, the suspected executor of the fire were
believed to be members of the CPP/NPA.But suspicion alone is not sufficient,
preponderance of evidence being the quantum of proof.
014 FGU INSURANCE v. CA (Yap) FACTS:
Date | Justice, J. | Liability of Insurer for Loss 59.   Parties:
a.   Estate of Ang Gui – owner of ANCO (partnership engaged in the
PETITIONER: FGU Insurance shipping business)
RESPONDENTS: San Miguel Corporation (SMC), Estate of Ang Gui b.   San Miguel Corporation (SMC) – shipper-consignee
represented by Lucid, Julian and Jaime Ang c.   FGU Insurance – insurer of SMC
60.   SMC contracted ANCO, via its barge and tugboat, to deliver case of Pale
SUMMARY: SMC contracted with ANCO to carry, via its barge and tugboat, Pilsen and Cerveza Negra from Mandaue City to Iloilo and Antique.
cases of Pale Pilsen and Cerveza Negra from Mandaue City, Cebu to Iloilo and a.   When the boats arrived in Antique, the tugboat left the barge
Antique. When the boats arrived in Antique, the tugboat left the barge. At that (which did not have its own).
time, the clouds over the area were already dark and the waves were already big. b.   That afternoon, the clouds over the area were dark and the waves
The arrestre workers were likewise having a hard time unloading the cases onto were already big. The arrastre workers unloading the cargoes of
the dock. SMC’s district sales supervisor advised ANCO’s representative to SMS were already complainting about the difficulty in unloading
move the barge to a safer place, but the latter was confident that the barge could the cargo, so SMC’s district sales supervisor requested ANCO’s
withstand the waves. When the barge’s rope attached to the wharf was cut due to representative to transfer the barge to a safer place because the
the waves, the workers abandoned the vessel, which run aground and was broken vessel might not be able to withstand the big waves.
and the cargoes of beer were swept away. SMC filed a case for breach of c.   ANCO’s representative was, however, confident that the barge
contract against ANCO (now represented by the Estate of Ang Gui, one of the could withstand the waves.
partners). ANCO argues that the loss was caso fortuito, and that FGU Insurane is d.   At that time, only ANCO’s barge was left at the wharf, the rest of
the party liable to SMC for the lost cases. FGU counters that ANCO failed to the boats already left to seek shelter.
exercise the diligence of a good father of a family in the care and supervision of e.   Later on, the crew abandoned the vessel since the barge’s rope
the cases of beer. The RTC and CA ruled the ANCO was blatantly negligent, but attached to the wharf was cut off by the big waves – the barge run
that FGU Insurance is still liable for 53% due to partial loss. Hence, this petition. aground and was broken and the cargoes of beer in the barge were
swept away. Only 10,790 (out of 40,550) cases of beer were
The pertinent issue is whether FGU Insurance is liable. discharged into the custody of the arrastre operator.
61.   SMC filed a complaint for breach of contract against ANCO. The
The SC ruled in the negative. First, there is no res judicata (with the original partnership was dissolved due to Ang Gui’s death, so the Estate now
case) in this petition for certiorari from the third-party complaint since the issues represents ANCO in the case.
discussed in the original case did not touch on the issues herein: negligence of a.   ANCO, although admitting that the cases were indeed loaded on
ANCO and its liability, and the liability of FGU. Second, there was no caso the vessel, argues that it had agreed with SMC that the former
fortuito since the event herein was neither unforeseen (ie., signs of the would not be liable for any loss or damage resulting to the cargoes
impending storm were already present) nor unavoidable (ie., the barge could by reason of fortuitous event, and that FGU Insurance insured
have been moved to a safer place). Third, the records clearly show ANCO’s 20,000 cases under a Marine Insurance Policy. ANCO alleged that
failure to exercise the extraordinary diligence mandated by law for common to expect them to leave the wharf “was like advising the fish to jum
carriers. Fourth, the US Supreme Court’s ruling in Standard Marine Insurance from the frying pan into the fire and an advice that borders on
v. Nome Beach (ie., that ordinary negligence of the insured’s agents is part of the madness.” Hence, ANCO filed a third-party complaint against
risk assumed by the insurer, but the insurer is exonerated when such negligence FGU Insurance.
amounts to gross negligence, misconduct, or wrongful acts) is applicable herein. b.   FGU Insurance counters that it is liable to SMC for only 3
Here, both the trial court and the CA concluded from the evidence that ANCO situations: (1) total loss of the entire shipment; (2) loss of any case
was indeed blatantly (ie., grossly) negligent. asa result of the sinking of the vessel; or (3) loss as a result of the
vessel being on fire. FGU alleged that ANCO failed to exercise the
DOCTRINE: While mistake and negligence of the master or crew are incident diligence of a good father of the family in the care and supervision
to navigation and constitute part of the perils that the insurer is obliged to incur, of the cargoes insured to prevent its loss and/or destruction. Hence,
such negligence or recklessness must not be of such gross character as to amount FGU Insurance filed a motion to dismiss.
to misconduct or wrongful acts; otherwise, such negligence shall release the 62.   RTC found that the cargoes were lost due to fortuitous event, but there was
insurer from liability under the insurance contract. failure on ANCO’s part to observe the degree of diligence required that
would exonerate them from liability. However, FGU Insurance would be nor unavoidable (ie., the barge could have been moved to a safer
liable for 53% since there was partial loss. CA affirmed in toto. place).
63.   Hence, this petition. d.   While the loss of the cargoes was admittedly caused by Typhoon
Sisang, a natural disaster, ANCO could not escape liability to
ISSUE/s: respondent SMC – the records clearly show ANCO’s failure to
12.   Whether the doctrine of res judicata applies – NO, because the former exercise the extraordinary diligence mandated by law for common
decision did not pass upon the issues raised in the instance case. carriers.
13.   Whether FGU Insurance is liable to SMC – NO, because the gross e.   Regarding FGU Insurance’s liablity, the US Supreme Court’s
negligence of ANCO exonerated FGU Insurance’s liability. following decision is relevant:
i.   Standard Marine Insurance v. Nome Beach – the ordinary
RULING: CA decision is AFFIRMED with MODIFICATION dismissing the third- diligence of the insured and his agents has long been held
party complaint (against FGU Insurance). as a part of the risk which the insurer takes upon himself,
and the existence of which, where it is the proximate
RATIO: cause of the loss, does not absolve the insurer from
74.   The doctrine of res judicata DOES NOT apply herein. liability. But willful exposure, gross negligence,
a.   Since the case at bar arose from the same incident involved in Civil negligence amounting to misconduct, etc., have often been
Case No. R-19341, only findings with respect to matters passed held to release the insurer from such liability.
upon by the court in the former judgment are conclusive in the f.   From the above-mentioned decision, the United States Supreme
disposition of the instant case. Court has made a distinction between ordinary negligence and
b.   The former case ruled on the following matters: (1) that the barge gross negligence or negligence amounting to misconduct and its
before and during the voyage was seaworthy (2) that there was effect on the insured's right to recover under the insurance contract.
proper notice of loss made by ANCO within the reglementary According to the Court, while mistake and negligence of the
period and (3) that the barge was a constructive total loss. master or crew are incident to navigation and constitute a part of
c.   Said decision, however, did not pass upon the issue regarding the the perils that the insurer is obliged to incur, such negligence or
liability of ANCO for the loss of the cargoes. Neither did the court recklessness must not be of such gross character as to amount to
pass upon the issue of the alleged negligence of the crewmembers misconduct or wrongful acts; otherwise, such negligence shall
of the barge being the cause of the loss of the cargoes owned by release the insurer from liability under the insurance contract.
SMC. g.   Here, both the tril court and the CA concluded from the evidence
that ANCO was indeed blatantly negligent.
75.   FGU Insurance is exonerated from liability.
a.   ANCO admitted that they failed to deliver to the designated
consignee the cases of beer.
b.   The gross negligence of ANCO is found on the following facts:
i.   The barge did not have its own engine, yet the tugboat left
it to fend for itself notwithstanding the fact that as the
vessels arrived in Antique, signs of the impending storm
were already manifest.
ii.   SMC’s district sale supervisor already requested the barge
to transfer to a safer place, but ANCO’s representative
faied to do so.
iii.   Transferring to another place “was not like asking a fish
to jump out the frying pan and into the fire” since the
other boats were able to move safely to Malandong, 5km
from the wharf where the barge remained.
c.   There is no caso fortuito since the event herein was neither
unforeseen (ie., signs of the impending storm were already present)
015 TRAVELLERS INSURANCE v. CA (Adrias) amendment. Absent such written claim filed by the person suing under an insurance
May 22, 1997 | Hermosisima, Jr. J. | Notice of Loss contract, no cause of action accrues under such insurance contract.Considering that
it is the rejection of that claim that triggers the running of the one-year prescriptive
PETITIONER: Travellers Insurance & Surety Corporation period to bring suit in court, and there can be no opportunity for the insurer to even
RESPONDENTS: Court of Appeals and Vicente Mendoza reject a claim if none has been filed in the first place.

Insurer: Travellers Insurance & Surety Corporation (Travellers)


Insured: Lady Love Taxicab
SUMMARY:
At about 5:30am July 20, 1980, a 78-year old woman, Feliza Vineza
de Mendoza (Feliza) was on her way to hear mass at the Tayuman
Cathedral. While walking, she was hit by a taxi that was running fast.
As a result of the bumping, Fileza died. Defendant Armando Abello,
the registered owner of the taxi certified to the fact that Rodrigo
Dumlao drove the vehicle on July 20, 1980. Thus, the police
apprehended Dumlao; the State charged and convicted him of reckless
imprudence resulting to Homicide. The representatives of Fileza then
filed a complaint for damages against Armando Abello as the owner
and Dumlao as the driver. Subsequently, Travellers Insurance was
impleaded because it was the compulsory insurer of the taxicab. The
RTC ordered Abellon, Dumlao and Travellers to pay the respondents
Mendoza solidarily. CA affirmed.Travellers contention: it did not
issue a policy as compulsory insurer and assuming that it had indeed
covered the taxicab for third-party liability insurance, Mendoza failed
to file a written notice of claim with Travellers as required by Sec.
384 of the Insurance Code. The issue in this case is WoN Mendoza
can claim from Travellers? The SC held in the negative.Travellers
asserts that no written claim was filed by Mendoza and rejected by it,
and Mendoza does not dispute such assertion through a denial in his
pleadings, we are constrained to rule that the CA committed reversible
error in finding Travellers liable under an insurance contract the
existence of which had not at all been proven in court. Even if there
were such a contract, Mendoza’s cause of action cannot prevail
because he failed to file the written claim mandated by Section 384 of FACTS:
the Insurance Code. He is deemed, under this legal provision, to have 10.  The petition herein seeks the review and reversal of the
waived his rights as against Travellers. decision of respondent CA affirming in toto the judgment of
the RTC in an action for damages filed by private respondent
DOCTRINE: There exist an indispensable requirement of having filed the written Vicente Mendoza, Jr. as heir of his mother who was killed in a
claim mandated by Section 384 of the Insurance Code before and after its vehicular accident.
11.  At about 5:30am July 20, 1980, a 78-year old woman, Feliza
Vineza de Mendoza (Feliza) was on her way to hear mass at ISSUE/s:
the Tayuman Cathedral. While walking, she was hit by a taxi 2.   WoN Mendoza can claim from Travellers? NO because the insurance
contract was never presented in court and a written notice of claim was
that was running fast. never filed.
12.  Several persons witnessed the accident among who were
Rolando Marvilla, Ernesto Lopez and Eulogio Tabalno. RULING: WHEREFORE, the instant petition is HEREBY GRANTED. The
13.  Maravilla ran towards Feliza and at this moment, a private jeep decision of the Court of Appeals in CA-G.R. CV No. 09416 and the decision of the
stopped and brought Feliza to the Mary Johnston Hospital in Regional Trial Court in Civil Case No. 135486 are REVERSED and SET ASIDE
insofar as Travellers Insurance & Surety Corporation was found jointly and severally
Tondo. liable to pay actual, moral and exemplary damages, death indemnity, attorney’s fees
14.  One of the witnesses, Lopez saw Mendoza Jr. and his brother and litigation expenses in Civil Case No. 135486. The complaint against Travellers
crying near the scene of the accident and informed them of Insurance & Surety Corporation in said case is hereby ordered dismissed. No
what happened. The brothers then proceeded to the Mary pronouncement as to costs. SO ORDERED.
Johnston Hospital where they were informed that Feliza was
RATIO:
brought to UST hospital where she passed away. Third-party Liability Insurance Contract
15.  Evidently, as a result of the bumping, Fileza died.
16.  In a criminal case, evidence showed that the driver of the 11.   When Mendoza filed his amended complaint to implead Travellers as party
taxicab was driving in a careless, reckless and imprudent defendant and alleged Travellers as the third-party liability insurer of the
manner at a high speed. The driver fled from the scene of the Lady Love taxicab, Mendoza failed to attach a copy of the insurance
contract to the amended complaint. Mendoza does not deny this omission.
accident and did not render assistance to the victim. 12.   It is significant to point out that the right of a third person to sue the insurer
17.  The taxicab was identified to be a Lady Love Taxi with Plane depends on whether the contract of insurance is intended to benefit third
No. 438. persons also or only the insured.
18.  Defendant Armando Abello, the registered owner of the taxi 13.   And the test applied has been this:
certified to the fact that Rodrigo Dumlao drove the vehicle on a.   Where the contract provides for indemnity against liability to
third persons, then third persons to whom the insured is liable can
July 20, 1980. sue the insurer.
19.  Thus, the police apprehended Dumlao; the State charged and b.   Where the contract is for indemnity against actual loss or
convicted him of reckless imprudence resulting to Homicide. payment, then third persons cannot proceed against the insurer,
20.  The representatives of Fileza then filed a complaint for the contract being solely to reimburse the insured for liability
damages against Armando Abello as the owner and Dumlao as actually discharged by him thru payment to third persons, said
third persons’ recourse being thus limited to the insured alone.”
the driver. 14.   Since Mendoza failed to attach a copy of the insurance contract to his
21.  Subsequently, Travellers Insurance was impleaded because it complaint, the trial court could not possibly know the real nature and
was the compulsory insurer of the taxicab. pecuniary limits of Travellers liability.
22.  The RTC ordered Abellon, Dumlao and Travellers to pay the 15.   The trial court did not distinguish between Mendoza’s cause of action
respondents Mendoza solidarily. against the owner and the driver of the Lady Love taxicab and his cause of
action against Travellers. The former is based on torts and quasi-delicts
23.  The CA affirmed the decision. while the latter is based on contract.
24.  Travellers contention: it did not issue a policy as compulsory 16.   While it is true that where the insurance contract provides for indemnity
insurer and assuming that it had indeed covered the taxicab for against liability to third persons, such third persons can directly sue the
third-party liability insurance, Mendoza failed to file a written insurer, however, the direct liability of the insurer under indemnity
notice of claim with Travellers as required by Sec. 384 of the contracts against third-party liability does not mean that the insurer can be
held solidarily liable with the insured and/or the other parties found at fault.
Insurance Code.
17.   The liability of the insurer is based on contract; that of the insured is based 26.   Even if there were such a contract, Mendoza’s cause of action cannot
on tort. prevail because he failed to file the written claim mandated by Section 384
18.   The trial court proceeded to hold Travellers liable for an award of damages of the Insurance Code. He is deemed, under this legal provision, to have
exceeding its limited liability of P50,000.00. This only shows beyond doubt waived his rights as against Travellers.
that the trial court was under the erroneous presumption that Travellers
could be found liable absent proof of the contract and based merely on the
proof of reckless imprudence on the part of the driver of the Lady Love
taxicab that fatally hit Feliza.

Notice of Claim
19.   At the time of the vehicular incident which resulted in the death of
Mendoza’s mother, during which time the Insurance Code had not yet been
amended by Batas Pambansa (B.P.) Blg. 874, Section 384 provided as
follows:
Any person having any claim upon the policy issued pursuant to
this chapter shall, without any unnecessary delay, present to the
insurance company concerned a written notice of claim setting
forth the amount of his loss, and/or the nature, extent and duration
of the injuries sustained as certified by a duly licensed physician.
Notice of claim must be filed within six months from date of the
accident, otherwise, the claim shall be deemed waived. Action or
suit for recovery of damage due to loss or injury must be brought
in proper cases, with the Commission or the Courts within one year
from date of accident, otherwise the claimants right of action shall
prescribe.
20.   It is significant to note that Section 384 was amended by B.P. Blg. 874 to
categorically provide that “action or suit for recovery of damage due to loss
or injury must be brought in proper cases, with the Commissioner or the
Courts within one year from denial of the claim, otherwise the claimant’s
right of action shall prescribe.”
21.   There exist an indispensable requirement of having filed the written claim
mandated by Section 384 of the Insurance Code before and after its
amendment.
22.   Absent such written claim filed by the person suing under an insurance
contract, no cause of action accrues under such insurance contract.
23.   Considering that it is the rejection of that claim that triggers the
running of the one-year prescriptive period to bring suit in court, and
there can be no opportunity for the insurer to even reject a claim if
none has been filed in the first place.
24.   The one-year period should instead be counted from the date of rejection by
the insurer as this is the time when the cause of action accrues.
25.   Travellers therefore asserts that no written claim was filed by Mendoza and
rejected by it, and Mendoza does not dispute such assertion through a denial
in his pleadings, we are constrained to rule that the CA committed
reversible error in finding Travellers liable under an insurance contract the
existence of which had not at all been proven in court.
016 PACIFIC TIMBER EXPORT CORP. v. CA (ALCAZAR) INSURANCE COMPANY, INC. printed Marine Policy form as filed with
February 25, 1982 | De Castro, J. | Notice of Loss and approved by the Office of the Insurance Commissioner
3.   On April 2, 1963, two marine cargo insurance policies were issued: Policy
PETITIONER: Pacific Timber Export Corporation No. 53 H0 1033 for 542 pieces of logs equivalent to 499,950 board feet and
RESPONDENTS: Court of Appeals and Workmen’s Insurance Company Inc. Policy No. 53 H0 1033 was for 853 pieces of logs equivalent to 695,548
board feet. The total cargo insured under the two marine policies
SUMMARY: Plaintiff secured a temporary temporary insurance from the defendant accordingly consisted of 1,395 logs, or the equivalent of 1,195.498 bd. ft.
for its exportation of 1,250,000 board feet of Philippine Lauan and Apitong logs and 4.   Some of the logs intended to be exported were lost during loading
the defendant subsequently issued Cover Note No. 1010. Then on April 2, 1963, operations in the Diapitan Bay.
defendant issued two more marine insurance policies covering 1,195.498 bd. ft. of 5.   The logs were taken from the log pond of the plaintiff and from there were
logs. Some of the logs intended to be exported were lost during loading operations towed in rafts to the vessel. However, while the logs were alongside the
due to bad weather. So on April 4, 1963, Plaintiff filed its claims against insurer vessel, bad weather developed causing 75 pieces of logs which were rafted
under the 2 marine insurance policies. The defendant then asked an adjuster to together to break loose from each other. 45 pieces of logs were salvaged,
investigate the loss. The adjuster submitted that the 30 pieces of logs lost were not but 30 pieces were verified to have been lost or washed away as a result of
covered by the two policies issued on April 2, 1963 but were covered by Cover Note the accident.
No. 1010. The defendant denied the claims on the ground that Cover Note No. 1010 6.   The plaintiff then informed the defendant about the loss of 32 pieces of logs
has already become null and void by virtue of the issuance of Marine Policy Nos. 53 through a letter dated April 4, 1963. The defendant only received the letter
HO 1032 and 1033. The Insurance Commissioner and Trial Court ruled in favor of on April 15, 1963.
the plaintiff while the CA dismissed the case because there was allegedly no 7.   The plaintiff subsequently submitted a 'Claim Statement demanding
premium paid for the Cover Note. The issue is: Whether the Insurance Company was payment of the loss under Policies Nos. 53 HO 1032 and 53 HO 1033.
absolved from responsibility due to unreasonable delay in giving the notice of loss. 8.   The defendant then asked an adjuster to investigate the loss. The adjuster
NO. The ground of delay was not promptly and specifically asserted when plaintiff submitted that the 30 pieces of logs lost were not covered by the two
filed their claim on the insurance agreement. The law requires this ground of delay to policies issued on April 2, 1963 but were covered by Cover Note No. 1010
be promptly and specifically asserted when a claim on the insurance agreement is insuring 1,250,000 bd. feet for the amount of $70,000.00.
made. From April 15,1963 the date of receipt of the claim up to July 1963, enough 9.   The defendant denied liability on that its investigation revealed that the
time was available for the defendant to determine if plaintiff was guilty of delay in entire shipment of logs covered by the two marines policies No. 53 HO
communicating the loss. Defendant also had the opportunity to raise the ground of 1032 and 53 HO 1033 were received in good order at their point of
delay during the proceedings with the Insurance Commissioner. However, it did not destination.
do so. Even on the assumption that there was delay, waiver can be raised against the 10.   It was also stated that the loss may not be considered as covered under
Insurance Company because Section 84 of the Insurance Act provides: Delay in the Cover Note No. 1010 because the said Note had become 'null and void by
presentation to an insurer of notice or proof of loss is waived if caused by any act of virtue of the issuance of Marine Policy Nos. 53 HO 1032 and 1033'
his or if he omits to take objection promptly and specifically upon that ground. 11.   Insurance Commissioner: Ruled in favor of plaintiff stating that ‘it is only
fair and equitable to indemnify the insured under Cover Note No. 1010,' and
DOCTRINE: Section 84 of the Insurance Act requires that the ground of delay must advised early settlement of the said marine loss and salvage claim.
be promptly and specifically asserted when a claim on the insurance agreement is 12.   TC: Ruled in favor of Insured
made. The nature of this specific ground for resisting a claim places the insurer on 13.   CA: Dismissed the case and ruled that the cover note is void for lack of
duty to inquire when the loss took place, so that it could determine whether delay valuable consideration as it appeared that no premium payment therefor was
would be a valid ground upon which to object to a claim against it. made by PTEC.

ISSUE/s:
FACTS:
1.   Whether a separate premium is needed for cover notes. NO.
1.   On March 19, 1963 Plaintiff secured temporary insurance from the 65.   [IMPORTANT] Whether the Insurance Company was absolved from
defendant for its exportation of 1,250,000 board feet of Philippine Lauan
responsibility due to unreasonable delay in giving the notice of loss. NO.
and Apitong logs to be shipped from the Diapitan Bay, Quezon Province to
The ground of delay was not promptly and specifically asserted when
Okinawa and Tokyo, Japan.
plaintiff filed their claim on the insurance agreement.
2.   The defendant issued Cover Note No. 1010, insuring the said cargo of the
plaintiff "Subject to the Terms and Conditions of the WORKMEN'S
RULING: ACCORDINGLY, the appealed decision is set aside and the decision of waived if caused by any act of his or if he omits to take objection promptly
the Court of First Instance is reinstated in toto with the affirmance of this Court. No and specifically upon that ground.
special pronouncement as to costs.

RATIO:

Cover note is Valid

1.   The fact that no separate premium was paid on the Cover Note before the
loss insured against occurred, does not militate against the validity of
plaintiff’s contention, for no such premium could have been paid, since by
the nature of the Cover Note, it did not contain, as all Cover Notes do not
contain particulars of the shipment that would serve as basis for the
computation of the premiums. As a logical consequence, no separate
premiums are intended or required to be paid on a Cover Note.
2.   At any rate, it is not disputed that Plaintiff paid in full all the premiums as
called for by the statement issued by Defendant after the issuance of the two
regular marine insurance policies, thereby leaving no account unpaid by
Plaintiff due on the insurance coverage, which must be deemed to include
the Cover Note.
3.   If the Note is to be treated as a separate policy instead of integrating it to the
regular policies subsequently issued, the purpose and function of the Cover
Note would be set at naught or rendered meaningless, for it is in a real sense
a contract, not a mere application for insurance which is a mere offer.

Insurance Company cannot be absolved from liability

66.   The defense of delay as raised by private respondent in resisting the claim
cannot be sustained. The law requires this ground of delay to be promptly
and specifically asserted when a claim on the insurance agreement is made.
67.   The facts show that instead of invoking the ground of delay in objecting to
petitioner's claim of recovery on the cover note, it took steps indicative that
this particular ground for objection to the claim was never in its mind.
68.   The nature of this specific ground for resisting a claim places the insurer on
duty to inquire when the loss took place, so that it could determine whether
delay would be a valid ground upon which to object to a claim against it.
69.   From April 15,1963 the date of receipt of the claim up to July 1963, enough
time was available for the defendant to determine if plaintiff was guilty of
delay in communicating the loss.
70.   Defendant also had the opportunity to raise the ground of delay during the
proceedings with the Insurance Commissioner. However, tt did not do so.
71.   The SC stated that it must be because it did not find any delay, as this Court
fails to find a real and substantial sign thereof.
72.   But even on the assumption that there was delay, waiver can be raised
against the Insurance Company because Section 84 of the Insurance Act
provides: Delay in the presentation to an insurer of notice or proof of loss is
017 ANG v. FULTON (APASAN)
July, 31, 1961 | Labrador, J. | Statute of Limitations DOCTRINE: The condition contained in the insurance policy that claims must be
presented within one year after rejection is NOT merely a procedural requirement.
PETITIONER: Paulo Ang and Sally Ang The condition is an important matter, essential to a prompt settlement of claims
RESPONDENTS: Fulton Fire Insurance Co. against insurance companies, as it demands that insurance suits be brought by the
insured while the evidence as to the origin and cause of destruction have not yet
SUMMARY: On September 9, 1953, Fulton Fire Insurance Company (Fulton disappeared. It is in the nature of a condition precedent to the liability of the insurer,
Insurance) issued a fire insurance policy in favor of P&S Department Store owned or in other terms, a resolutory cause, the purpose of which is to terminate all
by the Spouses Ang (Spouses Ang) over stocks of general merchandise contained in liabilities in case the action is not filed by the insured within the period stipulated.
the building. On December 17, 1954, the store containing the goods insured was
destroyed by fire. Spouses Ang filed their claim and Fulton Insurance forwarded FACTS:
this to its adjuster, Manila Adjustment Company (Adjuster). In the meantime, Paulo Material dates in the case:
Ang and 10 other accused were charged with Arson, but Paulo Ang was acquitted. April 19, 1956 – receipt of the denial of the claim
Subsequently, the claim was denied and the Spouses Ang received the notice of May 11, 1956 – first civil case filed by Spouses Ang against Paramount Surety
denial on April 19, 1956. Hence, this present civil action was filed by the Spouses (Agent)
Ang on May 5, 1958 against Fulton Insurance and Paramount Surety and Insurance May 5, 1958 – second civil case filed by Spouses Ang against Fulton Insurance
Company Inc., (Paramount Surety). (principal)
CONTENTION OF FULTON: Fulton Insurance objected that the claim had already 1.   September 9, 1953 – Fulton Fire Insurance Company (Fulton Insurance)
been forfeited since under paragraph 13 of the policy, it provides that “if the claim is issued a fire insurance policy, in favor of P. & S Department Store (owned
made and rejected but no action is commenced within 12 months after such by Spouses Ang) over stocks of general merchandise, consisting principally
rejection, all benefits under the policy would be forfeited.” And since in the present of dry goods, contained in a building occupied by the Spouses Ang at
case, the notice of denial was received on April 19, 1956 but the action was filed Laoag, Ilocos Norte.
only on May 5, 1958, then the claim should be considered as already forfeited. a.   The premium is P500.00 annually.
CONTENTION OF SPOUSES ANG: Spouses Ang replied that they instituted a civil b.   The insurance was issued for one year, but the same was renewed
case on May 11, 1956 (the alleged case was really just mentioned in the reply), and for another year on September 31, 1954.
although the said case was dismissed without prejudice, the filing thereof tolled the 2.   December 17, 1954 – the store containing the goods insured was destroyed
12 month period. by fire.
The RTC ruled in favor of the Spouses Ang and held that since the dismissal was due 3.   December 30 – Spouses Ang executed the first claim form. The claim,
to a mere procedural lapse (see fact 11[a]) and the dismissal was without prejudice, together with all the necessary papers relating thereto, were forwarded to
then it effectively tolled the running of the period. Hence this petition. Issue is the Manila Adjustment Company, Fulton's adjusters.
whether or not the filing of the civil case tolled or suspended the running of the 4.   January 12, 1955 – The Manila Adjustment Company accepted receipt of
12-month period as stipulated in the policy – NO, the condition contained in the the claim and requested the submission of the books of accounts of the
insurance policy that claims must be presented within one year after rejection is insured for the year 1953-1954 and a clearance from the Philippine
NOT merely a procedural requirement. The condition is an important matter, Constabulary and the police.
essential to a prompt settlement of claims against insurance companies, as it 5.   January 13, 1955 – Paulo Ang and ten others (Paulo et al) were charged for
demands that insurance suits be brought by the insured while the evidence as to the arson in the Court of First Instance of Ilocos Norte. The court acquitted
origin and cause of destruction have not yet disappeared. It is in the nature of a plaintiff Paulo Ang of the crime of arson.
condition precedent to the liability of the insurer, or in other terms, a resolutory 6.   April 6, 1956 – the Fulton wrote the Spouses Ang that their claim was
cause, the purpose of which is to terminate all liabilities in case the action is not filed denied.
by the insured within the period stipulated. The bringing of the action against the 7.   April 19, 1956 – the above denial was received by the Spouses Ang.
Paramount Surety, the agent of Fulton cannot have any legal effect except that of 8.   May 5, 1958 – this present action was filed by the Spouses Ang. The action
notifying the agent of the claim. Beyond such notification, the filing of the action can was originally instituted against both Fulton and the Paramount Surety and
serve no other purpose. There is no law giving any effect to such action upon the Insurance Company, Inc., (Paramount Surety) but on June 16, 1958, upon
principal. Besides, there is no condition in the policy that the action must be filed motion of the Paramount Surety, the latter was dropped from the complaint.
against the agent, and this Court can not by interpretation, extend the clear scope of 9.   May 26, 1958 – Fulton filed an answer to the complaint, admitting the
the agreement beyond what is agreed upon by the parties. existence of the contract of insurance, its renewal and the loss by fire of the
department store and the merchandise contained therein, but denying that presented within one year after rejection is NOT merely a procedural
the loss by the fire was accidental, alleging that it was occasioned by the requirement.
willful act of the plaintiff Paulo Ang himself. It claims that under
paragraph 13 of the policy, if the loss or damage is occasioned by the
willful act of the insured, or if the claim is made and rejected but no action RULING: Judgment of the CFI was SET ASIDE.
is commenced within 12 months after such rejection, all benefits under the
policy would be forfeited, and that since the claim of the plaintiffs was RATIO:
denied and plaintiffs received notice of denial on April 19, 1956, and they 1.   The clause subject of the issue is paragraph 13 of the policy, which reads as
brought the action only on May 5, 1958, all the benefits under the policy follows:
have been forfeited. 13. If the claim be in any respect fraudulent, or if any false
10.   February 12, 1959 – Spouses Ang filed a reply to the above answer of the declaration is made or used in support thereof, or if any
Fulton, alleging that: fraudulent means or devices are used by the Insured or any
a.   on May 11, 1956, they had instituted Civil Case No. 2949 in the one acting on his behalf to obtain any benefit under this
Court of First Instance of Manila, to assert the claim (NOTE: this Policy, or, if the loss or damage be occasioned by the willful
alleged case was really not mentioned in the previous facts); act or with connivance of the Insured, or, if the claim be
b.   that this case was dismissed without prejudice on September 3, made and rejected and an action or suit be not
1957 and that deducting the period within which said action was commenced within twelve months after such rejection or
pending, the present action was still within the 12 month period (in case of arbitration place in pursuance of the 18th
from April 12, 1956. condition of this Policy) within twelve months after the
11.   The CFI held that the bringing of the civil action on May 11, 1956 (fact arbitrator or arbitrators or umpire shall have made their
10[a]), tolled the running of the 12 month period within which the action award, all benefits under this Policy shall be forfeited.
must be filed. Said the court on this point: 2.   Fulton cited in support of its contention the cases of (but only the second
a.   “True, indeed, plaintiffs (Spouses Ang) committed a procedural case was explained by the SC in ratio no. 6)
mistake in first suing the agent (Paramount Surety) instead of its a.   E. Macias & Co. vs. Warner, Barnes & Co., Ltd.,;
principal (Fulton), the herein defendant, as correctly pointed out by b.   E. Macias & Co. vs. China Fire Insurance Co., and
counsel for the defendant. But the mistake being merely c.   Castillo etc. vs. Metropolitan Insurance Co.,
procedural, and the defendant not having been misled by the error, 3.   Paulo et al contend that their action had not yet prescribed at the time of the
'There is nothing sacred about process or pleadings, their forms or bringing of the action, because the period of prescription was interrupted by
contents.” the filing of the first action against the Paramount Surety & Insurance Co.,
b.   “The complaint was dismissed by the Court without prejudice on in accordance with Article 1155 of the Civil Code.
September 3, 1957, and motion for reconsideration dated a.   Counsel further argues that the basis of prescription of an action is
September 21, 1957. The instant complaint was filed on May 8, the abandonment by a person of his right of action or claim, so that
1958. The Rules of Court is applicable in the computation of time. any act of said person tending to show his intention not to abandon
Now, as correctly pointed out by the plaintiffs' counsel, by simple his right of action or claim, as the filing of the previous action in
mathematical computation, the present action was filed leas thin the case at bar, interrupts the period of prescription.
nine (9) months after the notice of rejection received by plaintiffs b.   Furthermore, counsel argues, the dismissal of the previous action is
on April 19, 1956, because the filing of the original complaint without prejudice, which means that plaintiffs have the right to file
stopped the running of the period.” another complaint against the principal.
12.   In view of the reasons thus above quoted, the court rendered decision in 4.   The basic error committed by the trial court is its view that the filing of the
favor of the Spouses Ang. Hence, this petition. action against the agent of Fulton was "merely a procedural mistake of no
significance or consequence, which may be overlooked." The condition
ISSUE: contained in the insurance policy that claims must be presented within
1.   Whether or not the court below erred in holding that the filing of the one year after rejection is NOT merely a procedural requirement. The
previous suit tolled or suspended the running of the prescriptive period – condition is an important matter, essential to a prompt settlement of
YES, - The condition contained in the insurance policy that claims must be claims against insurance companies, as it demands that insurance suits
be brought by the insured while the evidence as to the origin and cause
of destruction have not yet disappeared. It is in the nature of a
condition precedent to the liability of the insurer, or in other terms, a
resolutory cause, the purpose of which is to terminate all liabilities in
case the action is not filed by the insured within the period stipulated.
5.   The bringing of the action against the Paramount Surety, the agent of
Fulton CANNOT have any legal effect except that of notifying the agent
of the claim. Beyond such notification, the filing of the action can serve
no other purpose. There is no law giving any effect to such action upon
the principal. Besides, there is no condition in the policy that the action
must be filed against the agent, and this Court can not by
interpretation, extend the clear scope of the agreement beyond what is
agreed upon by the parties.
6.   The case of E. Macias & Co. vs. China Fire Insurance Co. has settled the
issue presented by Paulo et al in the case at bar definitely against their
claim. In that case, the SC declared that the contractual station in an
insurance policy prevails over the statutory limitation, as well as over
the exceptions to the statutory limitations that the contract necessarily
supersedes the statute (of limitations) and the limitation is in all phases
governed by the former.
7.   As stated in said case and in accordance with the decision of the Supreme
Court of the United States in Riddlesbarger vs. Hartford Fire Insurance Co.,
the rights of the parties flow from the contract of insurance, hence they
are NOT bound by the statute of limitations nor by exemptions thereto.
In the words of our own law, their contract is the law between the parties,
and their agreement that an action on a claim denied by the insurer
must be brought within one year from the denial, governs, NOT the
rules on the prescription of actions.
018 SUN INSURANCE v. CA and TAN (Arcenas) 3.   August 20, 1983 - Tan filed his claim for fire loss with SUN INSURANCE,
March 13, 1991 | Paras, J. | Statute of Limitations but on February 29, 1984, SUN INSURANCE wrote to Tan denying his
claim.
4.   April 3, 1984 - Tan sought reconsideration of the denial of his claim.
PETITIONER: Sun Insurance Office, Ltd.
5.   September 3, 1985 - Tan's counsel wrote to SUN INSURANCE inquiring
RESPONDENTS: Court of Appeals and Emilio Tan
about the status of his April 3, 1984 request for reconsideration to which
SUN INSURNACE replied that the denial of Tan's claim remained
SUMMARY: Emilio Tan insured his electrical supply store with Sun Insurance (the
unchanged, enclosing copies of petitioners' letters of February 29, 1984 and
store was located inside the building of his brother in Iloilo City). Four days after the
May 17, 1985 (response to petition for reconsideration).
issuance of the policy (Aug 19 ’83), the building burned, including the insured store.
6.   November 20, 1985 - Tan filed a civil case with the RTC but SUN
Tan’s claimed for fire loss the day after (aug 20 ’83) but was denied by Sun
INSURANCE filed a motion to dismiss on the alleged ground that the
Insurance months later (Feb 29 ’84). Tan received the denial letter (apr 2, ’84) and
action had already prescribed.
sought reconsideration (apr 3 ’84). But there was no communication from Sun
7.   RTC RULING: MTD and the subsequent motion for reconsideration was
Insurance so Tan’s counsel inquired about the status a year later (Sep 3, ’85) to
denied
which Sun Insurance reiterated the denial. Hence, Tan filed a civil case against Sun
8.   CA RULING: SUN INSURNANCE’s petition was denied and the court
Insurnace with the RTC. Sun Insurance filed a motion to dismiss alleging that the
held that the court a quo may continue until its final termination. SUN
action had already prescribed. The MTD was denied by RTC which was also
INSURANCE’s MR was also denied.
affirmed by the CA. Hence, this petition. The issue before the SC is w/n the filing of
9.   Hence, the instant petition.
a motion for reconsideration interrupts the 12-month prescriptive period to contest
denial of claim. The Court held in the negative. The clear and unambigious wording
ISSUE/s:
of the insurance policy must be taken and understood in their plain, ordinary and
1.   w/n the filing of a motion for reconsideration interrupts the twelve (12)
popular sense; hence, since condition 27 of the insurance policy states that if no
months prescriptive period to contest the denial of the insurance claim –
action or suit is commenced within 12 months from receipt of rejection or in case of
NO. The clear and unambigious wording of the insurance policy must be
arbitration, within 12 months after due notice of the award made by the arbitrator,
taken and understood in their plain, ordinary and popular sense; hence,
the claim is deemed abandoned and shall not be recoverable. Since Tan admitted
since condition 27 of the insurance policy states that if no action or suit is
that he received a copy of the letter of rejection on April 2, 1984, the 12-month
commenced within 12 months from receipt of rejection or in case of
prescriptive period started to run from the said date of April 2, 1984, for such is the
arbitration, within 12 months after due notice of the award made by the
plain meaning and intention of Section 27 of the insurance policy. Furthermore, it is
arbitrator, the claim is deemed abandoned and shall not be recoverable.
apparent that Section 27 of the insurance policy was stipulated pursuant to Section
63 of the Insurance Code. The condition contained in an insurance policy that claims
RULING: the questioned decision of the Court of Appeals is REVERSED and SET
must be presented within one year after rejection is not merely a procedural
ASIDE, and Civil Case No. 16817 filed with the Regional Trial Court is hereby
requirement but an important matter essential to a prompt settlement of claims
DISMISSED
against insurance companies as it demands that insurance suits be brought by the
insured while the evidence as to the origin and cause of destruction have not yet
RATIO:
disappeared.
Filing of MR does not interrupt 12-month prescriptive period to contest denial of
insurance claim
DOCTRINE: Sec. 63. A condition, stipulation or agreement in any policy of
1.   While it is a cardinal principle of insurance law that a policy or contract of
insurance, limiting the time for commencing an action thereunder to a period of less
insurance is to be construed liberally in favor of the insured and strictly
than one year from the time when the cause of action accrues, is void.
against the insurer company, yet, contracts of insurance, like other
contracts, are to be construed according to the sense and meaning of the
FACTS: terms which the parties themselves have used.
1.   Aug 15, 1983- Emilio Tan (Tan) took from Sun Insurance Office, Ltd (SUN 2.   If such terms are clear and unambiguous, they must be taken and
INSURANCE) a P300,000.00 property insurance policy to cover his understood in their plain, ordinary and popular sense.
interest in the electrical supply store of his brother which is in a building in 3.   Condition 27 of the Insurance Policy, which is the subject of the conflicting
Iloilo City. contentions of the parties, reads:
2.   Aug 19 - Four (4) days after the issuance of the policy, the building was
burned including the insured store.
a.   27. Action or suit clause — If a claim be made and rejected and an i.   Since "cause of action" requires as essential elements
action or suit be not commenced either in the Insurance not only a legal right of the plaintiff and a correlated
Commission or in any court of competent jurisdiction within obligation of the defendant in violation of the said
twelve (12) months from receipt of notice of such rejection, or legal right, the cause of action does not accrue until the
in case of arbitration taking place as provided herein, within party obligated (surety) refuses, expressly or impliedly, to
twelve (12) months after due notice of the award made by the comply with its duty (in this case to pay the amount of the
arbitrator or arbitrators or umpire, then the claim shall for all bond).
purposes be deemed to have been abandoned and shall not 2.   Indisputably, the above-cited pronouncements of this Court may be taken to
thereafter be recoverable hereunder. mean that the insured's cause of action or his right to file a claim either
4.   As the terms are very clear and free from any doubt or ambiguity in the Insurance Commission or in a court of competent jurisdiction
whatsoever, it must be taken and understood in its plain, ordinary and commences from the time of the denial of his claim by the Insurer,
popular sense pursuant to the above-cited principle laid down by this Court. either expressly or impliedly.
5.   Tan, in his letter addressed to SUN INSURANCE, admitted that he received 3.   But as pointed out by the SUN INSURANCE, the rejection referred to
a copy of the letter of rejection on April 2, 1984. Thus, the 12-month should be construed as the rejection, in the first instance, for if what is being
prescriptive period started to run from the said date of April 2, 1984, for referred to is a reiterated rejection conveyed in a resolution of a petition for
such is the plain meaning and intention of Section 27 of the insurance reconsideration, such should have been expressly stipulated.
policy. a.   Thus, to allow the filing of a motion for reconsideration to suspend
6.   While the question of whether or not the insured was definitely advised of the running of the prescriptive period of twelve months, a whole
the rejection of his claim through the letter dated February 29, 1984, may new body of rules on the matter should be promulgated so as to
arise, the certainty of the denial of Tan's claim was clearly manifested in avoid any conflict that may be brought by it, such as:
said letter (see end of digest) i.   whether the mere filing of a plea for reconsideration of a
7.   In Ang v. Fulton Fire Insurance Co, the Court had settled the rationale for denial is sufficient or must it be supported by
the necessity of bringing suits against the Insurer within one year from the arguments/affidavits/material evidence;
rejection of the claim. The condition contained in an insurance policy that ii.   how many petitions for reconsideration should be
claims must be presented within one year after rejection is not merely a permitted?
procedural requirement but an important matter essential to a prompt 4.   Contrary to Tan’s view, SC held that the use of the phrase "final rejection"
settlement of claims against insurance companies as it demands that cannot be taken to mean the rejection of a petition for reconsideration
insurance suits be brought by the insured while the evidence as to the as insisted by CA and Tan. Such was clearly not the meaning
origin and cause of destruction have not yet disappeared. contemplated by this Court. The Insurance policy in said case provides that
Applying to this case: the insured should file his claim, first, with the carrier and then with the
8.   It is apparent that Section 27 of the insurance policy was stipulated pursuant insurer. The "final rejection" being referred to in said case is the rejection
to Section 63 of the Insurance Code, which states that: by SUN INSURANCE
a.   Sec. 63. A condition, stipulation or agreement in any policy of DENIAL LETTER
insurance, limiting the time for commencing an action thereunder We refer to your claim for fire loss of 20th August, 1983 at Huervana St., La Paz, Iloilo City.
to a period of less than one year from the time when the cause of We now have the report of our adjusters and after a thorough and careful review of the same
action accrues, is void. and the accompanying documents at hand, we are rejecting, much to our regrets, liability for
the claim under our policies for one or more of the following reasons:
When does the cause of action accrue?
1. xxx xxx xxx
1.   Tan used two cases to prove his point: 2. xxx xxx xxx
a.   Eagle Star Insurance Co. vs. Chia Yu where the Court held: For your information, we have referred all these matters to our lawyers for their opinion as to
i.   The right of the insured to the payment of his loss accrues the compensability of your claim, particularly referring to the above violations. It is their
from the happening of the loss. However, the cause of opinion and in fact their strong recomendation to us to deny your claim. By this letter, we do
action in an insurance contract does not accrue until not intend to waive or relinquish any of our rights or defenses under our policies of insurance.
the insured's claim is finally rejected by the insurer.
This is because before such final rejection there is no real
necessity for bringing suit;
b.   ACCFA vs. Alpha Insurance & Surety Co., Inc.
019 Coastwise Lighterage v. Court of Appeals (Linds) Coastwise.
July 12, 1995 | Francisco, R., J. | Affreightment; Subrogation 65.   Upon reaching Manila Bay, while approaching Pier 18, one of the barges,
"Coastwise 9", struck an unknown sunken object. The forward buoyancy
PETITIONER: Coastwise Lighterage Corporation compartment was damaged, and water gushed in through a hole "two inches
RESPONDENTS: Court of Appeals, and Phil Gen Insurance wide and twenty-two inches long.
66.   As a consequence, the molasses at the cargo tanks were contaminated and
SUMMARY: Pag-asa entered into a contract to transport molasses from Negros rendered unfit for the use it was intended. This prompted the consignee,
to Manila with Coastwise using latter’s dumb barges which were towed in Pag-asa Sales, Inc. to reject the shipment of molasses as a total loss.
tandem by MT Marica, owned by Coastwise. Upon reaching Pier 18 in Manila Thereafter, Pag-asa Sales, Inc. filed a formal claim with the insurer of its
bay, one of the barges hit an unknown object, which caused a hole and the lost cargo, herein private respondent, Philippine General Insurance
molasses inside were contaminated. Pag-asa rejected the shipment as a total loss. Company (PhilGen, for short) and against the carrier, herein petitioner,
Pag-asa filed formal claims against PhilGen (Pag-asa’s insurer) and Coastwise. Coastwise Lighterage.
Coastwise rejected, forcing Philgen to pay the claim amounting to P700T. 67.   Coastwise Lighterage denied the claim and it was PhilGen which paid the
Philgen filed a claim against Coastwise by virtue of subrogation. RTC granted. consignee, Pag-asa Sales, Inc., the amount of P700,000.00, representing the
CA affirmed. value of the damaged cargo of molasses.
68.   In turn, PhilGen then filed an action against Coastwise Lighterage before
The issues before the Court are whether Coastwise was acting as common carrier the Regional Trial Court of Manila, seeking to recover the amount of
under the agreement, and whether Philgen was subrogated upon payment of Pag- P700,000.00 which it paid to Pag-asa Sales, Inc. for the latter's lost cargo.
asa’s goods. 69.   PhilGen now claims to be subrogated to all the contractual rights and claims
which the consignee may have against the carrier, which is presumed to
Anent the first issue, Coastwise was a common carrier. Coastwise Lighterage, by have violated the contract of carriage.
the contract of affreightment, was not converted into a private carrier, but remained a 70.   The RTC awarded the amount prayed for by PhilGen. On Coastwise
common carrier and was still liable as such. the presumption of negligence that attaches to Lighterage's appeal to the Court of Appeals, the award was affirmed.
common carriers, once the goods it transports are lost, destroyed or deteriorated, applies
to the petitioner. This presumption, which is overcome only by proof of the exercise of
extraordinary diligence, remained unrebutted in this case. Coastwise was negligent since
the patron of the vessel was unlicensed.
ISSUE/s:
Anent the second issue, Undoubtedly, upon payment by respondent insurer PhilGen of the
1.   Whether petitioner Coastwise Lighterage was transformed into a private
amount of P700,000.00 to Pag-asa Sales, Inc., the consignee of the cargo of molasses
totally damaged while being transported by petitioner Coastwise Lighterage, the former
carrier, by virtue of the contract of affreightment which it entered into with
was subrogated into all the rights which Pag-asa Sales, Inc. may have had against the the consignee, Pag-asa Sales, Inc. Corollarily, if it were in fact transformed
carrier, herein petitioner Coastwise Lighterage. into a private carrier, did it exercise the ordinary diligence to which a
private carrier is in turn bound.
DOCTRINE: A common carrier, by the contract of affreightment, was not converted a.   No, still common carrier, which needs to exercise
into a private carrier, but remained a common carrier and was still liable as such. extraordinary diligence in the vigilance over cargoes.
2.   Whether the insurer was subrogated into the rights of the consignee against
Payment by the insurer to the assured operated as an equitable assignment to the former the carrier, upon payment by the insurer of the value of the consignee's
of all remedies which the latter may have against the third party whose negligence or goods lost while on board one of the carrier's vessels.
wrongful act caused the loss. The right of subrogation is not dependent upon, nor does it
grow out of, any privity of contract or upon written assignment of claim. It accrues simply
upon payment of the insurance claim by the insurer.
RULING: WHEREFORE, premises considered, this petition is DENIED and the
appealed decision affirming the order of Branch 35 of the Regional Trial Court of
FACTS: Manila for petitioner Coastwise Lighterage to pay respondent Philippine General
64.   Pag-asa Sales, Inc. entered into a contract to transport molasses from the Insurance Company the "principal amount of P700,000.00 plus interest thereon at the
province of Negros to Manila with Coastwise Lighterage Corporation legal rate computed from March 29, 1989, the date the complaint was filed until fully
(Coastwise for brevity), using the latter's dumb barges. The barges were paid and another sum of P100,000.00 as attorney's fees and costs"10 is likewise
towed in tandem by the tugboat MT Marica, which is likewise owned by
hereby AFFIRMED petitioner. This presumption, which is overcome only by proof of the exercise of
extraordinary diligence, remained unrebutted in this case.
RATIO: 6.   Based on the records, the patron of Coastwise’ vessel was not even licensed.
Clearly, petitioner Coastwise Lighterage's embarking on a voyage with an
unlicensed patron cannot safely claim to have exercised extraordinary
Arguments before the Court re: first issue
diligence, by placing a person whose navigational skills are questionable, at the
helm of the vessel which eventually met the fateful accident. It may also
1.   On the first issue, petitioner (Coastwise) contends that the RTC and the logically, follow that a person without license to navigate, lacks not just the skill to
Court of Appeals erred in finding that it was a common carrier. It stresses do so, but also the utmost familiarity with the usual and safe routes taken by
the fact that it contracted with Pag-asa Sales, Inc. to transport the shipment seasoned and legally authorized ones. Had the patron been licensed, he could be
of molasses from Negros Oriental to Manila and refers to this contract as a presumed to have both the skill and the knowledge that would have prevented the
"charter agreement". It then proceeds to cite the case of Home Insurance vessel's hitting the sunken derelict ship that lay on their way to Pier 18.
Company vs. American Steamship Agencies, Inc. wherein this Court held:
". . . a common carrier undertaking to carry a special cargo or chartered to a Ruling re: second issue
7.   On the issue of subrogation, which petitioner contends as inapplicable in this case,
special person only becomes a private carrier."
we once more rule against the petitioner. We have already found petitioner liable for
breach of the contract of carriage it entered into with Pag-asa Sales, Inc. However,
Ruling re: first issue
for the damage sustained by the loss of the cargo which petitioner-carrier was
2.   Petitioner's (Coastwise’) reliance on the aforementioned case is misplaced. In its
transporting, it was not the carrier which paid the value thereof to Pag-asa Sales, Inc.
entirety, the conclusions of the court are as follows:
but the latter's insurer, herein private respondent PhilGen.
a.   Accordingly, the charter party contract is one of affreightment over the
8.   Article 2207 of the Civil Code is explicit on this point:
whole vessel, rather than a demise. As such, the liability of the shipowner
a.   Art. 2207. If the plaintiffs property has been insured, and he has received
for acts or negligence of its captain and crew, would remain in the absence
indemnity from the insurance company for the injury or loss arising out of
of stipulation.
the wrong or breach of contract complained of, the insurance company
3.   The distinction between the two kinds of charter parties (i.e. bareboat or
shall be subrogated to the rights of the insured against the wrongdoer or
demise and contract of affreightment) is more clearly set out in the case of
the person who violated the contract.
Puromines, Inc. vs. Court of Appeals, wherein we ruled:
9.   Article 2207 of the Civil Code is founded on the well-settled principle of
a.   Under the demise or bareboat charter of the vessel, the charterer will
subrogation. If the insured property is destroyed or damaged through the fault or
generally be regarded as the owner for the voyage or service stipulated.
negligence of a party other than the assured, then the insurer, upon payment to the
The charterer mans the vessel with his own people and becomes the owner
assured will be subrogated to the rights of the assured to recover from the wrongdoer
pro hac vice, subject to liability to others for damages caused by
to the extent that the insurer has been obligated to pay. Payment by the insurer to the
negligence. To create a demise, the owner of a vessel must completely and
assured operated as an equitable assignment to the former of all remedies which the
exclusively relinquish possession, command and navigation thereof to the
latter may have against the third party whose negligence or wrongful act caused the
charterer, anything short of such a complete transfer is a contract of
loss. The right of subrogation is not dependent upon, nor does it grow out of, any
affreightment (time or voyage charter party) or not a charter party at all.
privity of contract or upon written assignment of claim. It accrues simply upon
b.   On the other hand a contract of affreightment is one in which the owner of
payment of the insurance claim by the insurer.
the vessel leases part or all of its space to haul goods for others. It is a
10.   Undoubtedly, upon payment by respondent insurer PhilGen of the amount of
contract for special service to be rendered by the owner of the vessel and
P700,000.00 to Pag-asa Sales, Inc., the consignee of the cargo of molasses totally
under such contract the general owner retains the possession, command
damaged while being transported by petitioner Coastwise Lighterage, the former
and navigation of the ship, the charterer or freighter merely having use of
was subrogated into all the rights which Pag-asa Sales, Inc. may have had against
the space in the vessel in return for his payment of the charter hire.
the carrier, herein petitioner Coastwise Lighterage.
c.   An owner who retains possession of the ship though the hold is the
property of the charterer, remains liable as carrier and must answer for any
breach of duty as to the care, loading and unloading of the cargo
4.   Although a charter party may transform a common carrier into a private one, the
same however is not true in a contract of affreightment on account of the
aforementioned distinctions between the two. Pursuant therefore to the ruling in
the aforecited Puromines case, Coastwise Lighterage, by the contract of
affreightment, was not converted into a private carrier, but remained a
common carrier and was still liable as such.
5.   It follows then that the presumption of negligence that attaches to common carriers,
once the goods it transports are lost, destroyed or deteriorated, applies to the
015 TRAVELLERS INSURANCE v. CA (Adrias) damages filed by private respondent Vicente Mendoza, Jr. as heir of his
May 22, 1997 | Hermosisima, Jr. J. | Notice of Loss mother who was killed in a vehicular accident.
26.   At about 5:30am July 20, 1980, a 78-year old woman, Feliza Vineza de
PETITIONER: Travellers Insurance & Surety Corporation Mendoza (Feliza) was on her way to hear mass at the Tayuman Cathedral.
RESPONDENTS: Court of Appeals and Vicente Mendoza While walking, she was hit by a taxi that was running fast.
27.   Several persons witnessed the accident among who were Rolando Marvilla,
Insurer: Travellers Insurance & Surety Corporation (Travellers) Ernesto Lopez and Eulogio Tabalno.
Insured: Lady Love Taxicab 28.   Maravilla ran towards Feliza and at this moment, a private jeep stopped and
brought Feliza to the Mary Johnston Hospital in Tondo.
SUMMARY: 29.   One of the witnesses, Lopez saw Mendoza Jr. and his brother crying near
At about 5:30am July 20, 1980, a 78-year old woman, Feliza Vineza de Mendoza the scene of the accident and informed them of what happened. The
(Feliza) was on her way to hear mass at the Tayuman Cathedral. While walking, she brothers then proceeded to the Mary Johnston Hospital where they were
was hit by a taxi that was running fast. As a result of the bumping, Fileza died. informed that Feliza was brought to UST hospital where she passed away.
Defendant Armando Abello, the registered owner of the taxi certified to the fact that 30.   Evidently, as a result of the bumping, Fileza died.
Rodrigo Dumlao drove the vehicle on July 20, 1980. Thus, the police apprehended 31.   In a criminal case, evidence showed that the driver of the taxicab was
Dumlao; the State charged and convicted him of reckless imprudence resulting to driving in a careless, reckless and imprudent manner at a high speed. The
Homicide. The representatives of Fileza then filed a complaint for damages against driver fled from the scene of the accident and did not render assistance to
Armando Abello as the owner and Dumlao as the driver. Subsequently, Travellers the victim.
Insurance was impleaded because it was the compulsory insurer of the taxicab. The 32.   The taxicab was identified to be a Lady Love Taxi with Plane No. 438.
RTC ordered Abellon, Dumlao and Travellers to pay the respondents Mendoza 33.   Defendant Armando Abello, the registered owner of the taxi certified to the
solidarily. CA affirmed.Travellers contention: it did not issue a policy as fact that Rodrigo Dumlao drove the vehicle on July 20, 1980.
compulsory insurer and assuming that it had indeed covered the taxicab for third- 34.   Thus, the police apprehended Dumlao; the State charged and convicted him
party liability insurance, Mendoza failed to file a written notice of claim with of reckless imprudence resulting to Homicide.
Travellers as required by Sec. 384 of the Insurance Code. The issue in this case is 35.   The representatives of Fileza then filed a complaint for damages against
WoN Mendoza can claim from Travellers? The SC held in the negative.Travellers Armando Abello as the owner and Dumlao as the driver.
asserts that no written claim was filed by Mendoza and rejected by it, and Mendoza 36.   Subsequently, Travellers Insurance was impleaded because it was the
does not dispute such assertion through a denial in his pleadings, we are constrained compulsory insurer of the taxicab.
to rule that the CA committed reversible error in finding Travellers liable under an 37.   The RTC ordered Abellon, Dumlao and Travellers to pay the respondents
insurance contract the existence of which had not at all been proven in court. Even if Mendoza solidarily.
there were such a contract, Mendoza’s cause of action cannot prevail because he 38.   The CA affirmed the decision.
failed to file the written claim mandated by Section 384 of the Insurance Code. He 39.   Travellers contention: it did not issue a policy as compulsory insurer and
is deemed, under this legal provision, to have waived his rights as against assuming that it had indeed covered the taxicab for third-party liability
Travellers. insurance, Mendoza failed to file a written notice of claim with Travellers
as required by Sec. 384 of the Insurance Code.
DOCTRINE: There exist an indispensable requirement of having filed the written
claim mandated by Section 384 of the Insurance Code before and after its ISSUE/s:
amendment. Absent such written claim filed by the person suing under an insurance 3.   WoN Mendoza can claim from Travellers? NO because the insurance
contract, no cause of action accrues under such insurance contract.Considering that contract was never presented in court and a written notice of claim was
it is the rejection of that claim that triggers the running of the one-year prescriptive never filed.
period to bring suit in court, and there can be no opportunity for the insurer to even
reject a claim if none has been filed in the first place. RULING: WHEREFORE, the instant petition is HEREBY GRANTED. The
decision of the Court of Appeals in CA-G.R. CV No. 09416 and the decision of the
Regional Trial Court in Civil Case No. 135486 are REVERSED and SET ASIDE
FACTS: insofar as Travellers Insurance & Surety Corporation was found jointly and severally
25.   The petition herein seeks the review and reversal of the decision of liable to pay actual, moral and exemplary damages, death indemnity, attorney’s fees
respondent CA affirming in toto the judgment of the RTC in an action for and litigation expenses in Civil Case No. 135486. The complaint against Travellers
Insurance & Surety Corporation in said case is hereby ordered dismissed. No amended by Batas Pambansa (B.P.) Blg. 874, Section 384 provided as
pronouncement as to costs. SO ORDERED. follows:
Any person having any claim upon the policy issued pursuant to
RATIO: this chapter shall, without any unnecessary delay, present to the
Third-party Liability Insurance Contract insurance company concerned a written notice of claim setting
forth the amount of his loss, and/or the nature, extent and duration
27.   When Mendoza filed his amended complaint to implead Travellers as party of the injuries sustained as certified by a duly licensed physician.
defendant and alleged Travellers as the third-party liability insurer of the Notice of claim must be filed within six months from date of the
Lady Love taxicab, Mendoza failed to attach a copy of the insurance accident, otherwise, the claim shall be deemed waived. Action or
contract to the amended complaint. Mendoza does not deny this omission. suit for recovery of damage due to loss or injury must be brought
28.   It is significant to point out that the right of a third person to sue the insurer in proper cases, with the Commission or the Courts within one year
depends on whether the contract of insurance is intended to benefit third from date of accident, otherwise the claimants right of action shall
persons also or only the insured. prescribe.
29.   And the test applied has been this: 36.   It is significant to note that Section 384 was amended by B.P. Blg. 874 to
a.   Where the contract provides for indemnity against liability to categorically provide that “action or suit for recovery of damage due to loss
third persons, then third persons to whom the insured is liable can or injury must be brought in proper cases, with the Commissioner or the
sue the insurer. Courts within one year from denial of the claim, otherwise the claimant’s
b.   Where the contract is for indemnity against actual loss or right of action shall prescribe.”
payment, then third persons cannot proceed against the insurer, 37.   There exist an indispensable requirement of having filed the written claim
the contract being solely to reimburse the insured for liability mandated by Section 384 of the Insurance Code before and after its
actually discharged by him thru payment to third persons, said amendment.
third persons’ recourse being thus limited to the insured alone.” 38.   Absent such written claim filed by the person suing under an insurance
30.   Since Mendoza failed to attach a copy of the insurance contract to his contract, no cause of action accrues under such insurance contract.
complaint, the trial court could not possibly know the real nature and 39.   Considering that it is the rejection of that claim that triggers the
pecuniary limits of Travellers liability. running of the one-year prescriptive period to bring suit in court, and
31.   The trial court did not distinguish between Mendoza’s cause of action there can be no opportunity for the insurer to even reject a claim if
against the owner and the driver of the Lady Love taxicab and his cause of none has been filed in the first place.
action against Travellers. The former is based on torts and quasi-delicts 40.   The one-year period should instead be counted from the date of rejection by
while the latter is based on contract. the insurer as this is the time when the cause of action accrues.
32.   While it is true that where the insurance contract provides for indemnity 41.   Travellers therefore asserts that no written claim was filed by Mendoza and
against liability to third persons, such third persons can directly sue the rejected by it, and Mendoza does not dispute such assertion through a denial
insurer, however, the direct liability of the insurer under indemnity in his pleadings, we are constrained to rule that the CA committed
contracts against third-party liability does not mean that the insurer can be reversible error in finding Travellers liable under an insurance contract the
held solidarily liable with the insured and/or the other parties found at fault. existence of which had not at all been proven in court.
33.   The liability of the insurer is based on contract; that of the insured is based 42.   Even if there were such a contract, Mendoza’s cause of action cannot
on tort. prevail because he failed to file the written claim mandated by Section 384
34.   The trial court proceeded to hold Travellers liable for an award of damages of the Insurance Code. He is deemed, under this legal provision, to have
exceeding its limited liability of P50,000.00. This only shows beyond doubt waived his rights as against Travellers.
that the trial court was under the erroneous presumption that Travellers
could be found liable absent proof of the contract and based merely on the
proof of reckless imprudence on the part of the driver of the Lady Love
taxicab that fatally hit Feliza.

Notice of Claim
35.   At the time of the vehicular incident which resulted in the death of
Mendoza’s mother, during which time the Insurance Code had not yet been
PACIFIC TIMBER EXPORT CORP. v. CA (ALCAZAR) INSURANCE COMPANY, INC. printed Marine Policy form as filed with
February 25, 1982 | De Castro, J. | Notice of Loss and approved by the Office of the Insurance Commissioner
16.   On April 2, 1963, two marine cargo insurance policies were issued: Policy
PETITIONER: Pacific Timber Export Corporation No. 53 H0 1033 for 542 pieces of logs equivalent to 499,950 board feet and
RESPONDENTS: Court of Appeals and Workmen’s Insurance Company Inc. Policy No. 53 H0 1033 was for 853 pieces of logs equivalent to 695,548
board feet. The total cargo insured under the two marine policies
SUMMARY: Plaintiff secured a temporary temporary insurance from the defendant accordingly consisted of 1,395 logs, or the equivalent of 1,195.498 bd. ft.
for its exportation of 1,250,000 board feet of Philippine Lauan and Apitong logs and 17.   Some of the logs intended to be exported were lost during loading
the defendant subsequently issued Cover Note No. 1010. Then on April 2, 1963, operations in the Diapitan Bay.
defendant issued two more marine insurance policies covering 1,195.498 bd. ft. of 18.   The logs were taken from the log pond of the plaintiff and from there were
logs. Some of the logs intended to be exported were lost during loading operations towed in rafts to the vessel. However, while the logs were alongside the
due to bad weather. So on April 4, 1963, Plaintiff filed its claims against insurer vessel, bad weather developed causing 75 pieces of logs which were rafted
under the 2 marine insurance policies. The defendant then asked an adjuster to together to break loose from each other. 45 pieces of logs were salvaged,
investigate the loss. The adjuster submitted that the 30 pieces of logs lost were not but 30 pieces were verified to have been lost or washed away as a result of
covered by the two policies issued on April 2, 1963 but were covered by Cover Note the accident.
No. 1010. The defendant denied the claims on the ground that Cover Note No. 1010 19.   The plaintiff then informed the defendant about the loss of 32 pieces of logs
has already become null and void by virtue of the issuance of Marine Policy Nos. 53 through a letter dated April 4, 1963. The defendant only received the letter
HO 1032 and 1033. The Insurance Commissioner and Trial Court ruled in favor of on April 15, 1963.
the plaintiff while the CA dismissed the case because there was allegedly no 20.   The plaintiff subsequently submitted a 'Claim Statement demanding
premium paid for the Cover Note. The issue is: Whether the Insurance Company was payment of the loss under Policies Nos. 53 HO 1032 and 53 HO 1033.
absolved from responsibility due to unreasonable delay in giving the notice of loss. 21.   The defendant then asked an adjuster to investigate the loss. The adjuster
NO. The ground of delay was not promptly and specifically asserted when plaintiff submitted that the 30 pieces of logs lost were not covered by the two
filed their claim on the insurance agreement. The law requires this ground of delay to policies issued on April 2, 1963 but were covered by Cover Note No. 1010
be promptly and specifically asserted when a claim on the insurance agreement is insuring 1,250,000 bd. feet for the amount of $70,000.00.
made. From April 15,1963 the date of receipt of the claim up to July 1963, enough 22.   The defendant denied liability on that its investigation revealed that the
time was available for the defendant to determine if plaintiff was guilty of delay in entire shipment of logs covered by the two marines policies No. 53 HO
communicating the loss. Defendant also had the opportunity to raise the ground of 1032 and 53 HO 1033 were received in good order at their point of
delay during the proceedings with the Insurance Commissioner. However, it did not destination.
do so. Even on the assumption that there was delay, waiver can be raised against the 23.   It was also stated that the loss may not be considered as covered under
Insurance Company because Section 84 of the Insurance Act provides: Delay in the Cover Note No. 1010 because the said Note had become 'null and void by
presentation to an insurer of notice or proof of loss is waived if caused by any act of virtue of the issuance of Marine Policy Nos. 53 HO 1032 and 1033'
his or if he omits to take objection promptly and specifically upon that ground. 24.   Insurance Commissioner: Ruled in favor of plaintiff stating that ‘it is only
fair and equitable to indemnify the insured under Cover Note No. 1010,' and
DOCTRINE: Section 84 of the Insurance Act requires that the ground of delay must advised early settlement of the said marine loss and salvage claim.
be promptly and specifically asserted when a claim on the insurance agreement is 25.   TC: Ruled in favor of Insured
made. The nature of this specific ground for resisting a claim places the insurer on 26.   CA: Dismissed the case and ruled that the cover note is void for lack of
duty to inquire when the loss took place, so that it could determine whether delay valuable consideration as it appeared that no premium payment therefor was
would be a valid ground upon which to object to a claim against it. made by PTEC.

ISSUE/s:
FACTS:
2.   Whether a separate premium is needed for cover notes. NO.
14.   On March 19, 1963 Plaintiff secured temporary insurance from the 73.   [IMPORTANT] Whether the Insurance Company was absolved from
defendant for its exportation of 1,250,000 board feet of Philippine Lauan
responsibility due to unreasonable delay in giving the notice of loss. NO.
and Apitong logs to be shipped from the Diapitan Bay, Quezon Province to
The ground of delay was not promptly and specifically asserted when
Okinawa and Tokyo, Japan.
plaintiff filed their claim on the insurance agreement.
15.   The defendant issued Cover Note No. 1010, insuring the said cargo of the
plaintiff "Subject to the Terms and Conditions of the WORKMEN'S
RULING: ACCORDINGLY, the appealed decision is set aside and the decision of waived if caused by any act of his or if he omits to take objection promptly
the Court of First Instance is reinstated in toto with the affirmance of this Court. No and specifically upon that ground.
special pronouncement as to costs.

RATIO:

Cover note is Valid

4.   The fact that no separate premium was paid on the Cover Note before the
loss insured against occurred, does not militate against the validity of
plaintiff’s contention, for no such premium could have been paid, since by
the nature of the Cover Note, it did not contain, as all Cover Notes do not
contain particulars of the shipment that would serve as basis for the
computation of the premiums. As a logical consequence, no separate
premiums are intended or required to be paid on a Cover Note.
5.   At any rate, it is not disputed that Plaintiff paid in full all the premiums as
called for by the statement issued by Defendant after the issuance of the two
regular marine insurance policies, thereby leaving no account unpaid by
Plaintiff due on the insurance coverage, which must be deemed to include
the Cover Note.
6.   If the Note is to be treated as a separate policy instead of integrating it to the
regular policies subsequently issued, the purpose and function of the Cover
Note would be set at naught or rendered meaningless, for it is in a real sense
a contract, not a mere application for insurance which is a mere offer.

Insurance Company cannot be absolved from liability

74.   The defense of delay as raised by private respondent in resisting the claim
cannot be sustained. The law requires this ground of delay to be promptly
and specifically asserted when a claim on the insurance agreement is made.
75.   The facts show that instead of invoking the ground of delay in objecting to
petitioner's claim of recovery on the cover note, it took steps indicative that
this particular ground for objection to the claim was never in its mind.
76.   The nature of this specific ground for resisting a claim places the insurer on
duty to inquire when the loss took place, so that it could determine whether
delay would be a valid ground upon which to object to a claim against it.
77.   From April 15,1963 the date of receipt of the claim up to July 1963, enough
time was available for the defendant to determine if plaintiff was guilty of
delay in communicating the loss.
78.   Defendant also had the opportunity to raise the ground of delay during the
proceedings with the Insurance Commissioner. However, tt did not do so.
79.   The SC stated that it must be because it did not find any delay, as this Court
fails to find a real and substantial sign thereof.
80.   But even on the assumption that there was delay, waiver can be raised
against the Insurance Company because Section 84 of the Insurance Act
provides: Delay in the presentation to an insurer of notice or proof of loss is
017 ANG v. FULTON (APASAN)
July, 31, 1961 | Labrador, J. | Statute of Limitations DOCTRINE: The condition contained in the insurance policy that claims must be
presented within one year after rejection is NOT merely a procedural requirement.
PETITIONER: Paulo Ang and Sally Ang The condition is an important matter, essential to a prompt settlement of claims
RESPONDENTS: Fulton Fire Insurance Co. against insurance companies, as it demands that insurance suits be brought by the
insured while the evidence as to the origin and cause of destruction have not yet
SUMMARY: On September 9, 1953, Fulton Fire Insurance Company (Fulton disappeared. It is in the nature of a condition precedent to the liability of the insurer,
Insurance) issued a fire insurance policy in favor of P&S Department Store owned or in other terms, a resolutory cause, the purpose of which is to terminate all
by the Spouses Ang (Spouses Ang) over stocks of general merchandise contained in liabilities in case the action is not filed by the insured within the period stipulated.
the building. On December 17, 1954, the store containing the goods insured was
destroyed by fire. Spouses Ang filed their claim and Fulton Insurance forwarded FACTS:
this to its adjuster, Manila Adjustment Company (Adjuster). In the meantime, Paulo Material dates in the case:
Ang and 10 other accused were charged with Arson, but Paulo Ang was acquitted. April 19, 1956 – receipt of the denial of the claim
Subsequently, the claim was denied and the Spouses Ang received the notice of May 11, 1956 – first civil case filed by Spouses Ang against Paramount Surety
denial on April 19, 1956. Hence, this present civil action was filed by the Spouses (Agent)
Ang on May 5, 1958 against Fulton Insurance and Paramount Surety and Insurance May 5, 1958 – second civil case filed by Spouses Ang against Fulton Insurance
Company Inc., (Paramount Surety). (principal)
CONTENTION OF FULTON: Fulton Insurance objected that the claim had already 13.   September 9, 1953 – Fulton Fire Insurance Company (Fulton Insurance)
been forfeited since under paragraph 13 of the policy, it provides that “if the claim is issued a fire insurance policy, in favor of P. & S Department Store (owned
made and rejected but no action is commenced within 12 months after such by Spouses Ang) over stocks of general merchandise, consisting principally
rejection, all benefits under the policy would be forfeited.” And since in the present of dry goods, contained in a building occupied by the Spouses Ang at
case, the notice of denial was received on April 19, 1956 but the action was filed Laoag, Ilocos Norte.
only on May 5, 1958, then the claim should be considered as already forfeited. a.   The premium is P500.00 annually.
CONTENTION OF SPOUSES ANG: Spouses Ang replied that they instituted a civil b.   The insurance was issued for one year, but the same was renewed
case on May 11, 1956 (the alleged case was really just mentioned in the reply), and for another year on September 31, 1954.
although the said case was dismissed without prejudice, the filing thereof tolled the 14.   December 17, 1954 – the store containing the goods insured was destroyed
12 month period. by fire.
The RTC ruled in favor of the Spouses Ang and held that since the dismissal was due 15.   December 30 – Spouses Ang executed the first claim form. The claim,
to a mere procedural lapse (see fact 11[a]) and the dismissal was without prejudice, together with all the necessary papers relating thereto, were forwarded to
then it effectively tolled the running of the period. Hence this petition. Issue is the Manila Adjustment Company, Fulton's adjusters.
whether or not the filing of the civil case tolled or suspended the running of the 16.   January 12, 1955 – The Manila Adjustment Company accepted receipt of
12-month period as stipulated in the policy – NO, the condition contained in the the claim and requested the submission of the books of accounts of the
insurance policy that claims must be presented within one year after rejection is insured for the year 1953-1954 and a clearance from the Philippine
NOT merely a procedural requirement. The condition is an important matter, Constabulary and the police.
essential to a prompt settlement of claims against insurance companies, as it 17.   January 13, 1955 – Paulo Ang and ten others (Paulo et al) were charged for
demands that insurance suits be brought by the insured while the evidence as to the arson in the Court of First Instance of Ilocos Norte. The court acquitted
origin and cause of destruction have not yet disappeared. It is in the nature of a plaintiff Paulo Ang of the crime of arson.
condition precedent to the liability of the insurer, or in other terms, a resolutory 18.   April 6, 1956 – the Fulton wrote the Spouses Ang that their claim was
cause, the purpose of which is to terminate all liabilities in case the action is not filed denied.
by the insured within the period stipulated. The bringing of the action against the 19.   April 19, 1956 – the above denial was received by the Spouses Ang.
Paramount Surety, the agent of Fulton cannot have any legal effect except that of 20.   May 5, 1958 – this present action was filed by the Spouses Ang. The action
notifying the agent of the claim. Beyond such notification, the filing of the action can was originally instituted against both Fulton and the Paramount Surety and
serve no other purpose. There is no law giving any effect to such action upon the Insurance Company, Inc., (Paramount Surety) but on June 16, 1958, upon
principal. Besides, there is no condition in the policy that the action must be filed motion of the Paramount Surety, the latter was dropped from the complaint.
against the agent, and this Court can not by interpretation, extend the clear scope of 21.   May 26, 1958 – Fulton filed an answer to the complaint, admitting the
the agreement beyond what is agreed upon by the parties. existence of the contract of insurance, its renewal and the loss by fire of the
department store and the merchandise contained therein, but denying that presented within one year after rejection is NOT merely a procedural
the loss by the fire was accidental, alleging that it was occasioned by the requirement.
willful act of the plaintiff Paulo Ang himself. It claims that under
paragraph 13 of the policy, if the loss or damage is occasioned by the
willful act of the insured, or if the claim is made and rejected but no action RULING: Judgment of the CFI was SET ASIDE.
is commenced within 12 months after such rejection, all benefits under the
policy would be forfeited, and that since the claim of the plaintiffs was RATIO:
denied and plaintiffs received notice of denial on April 19, 1956, and they 8.   The clause subject of the issue is paragraph 13 of the policy, which reads as
brought the action only on May 5, 1958, all the benefits under the policy follows:
have been forfeited. 13. If the claim be in any respect fraudulent, or if any false
22.   February 12, 1959 – Spouses Ang filed a reply to the above answer of the declaration is made or used in support thereof, or if any
Fulton, alleging that: fraudulent means or devices are used by the Insured or any
a.   on May 11, 1956, they had instituted Civil Case No. 2949 in the one acting on his behalf to obtain any benefit under this
Court of First Instance of Manila, to assert the claim (NOTE: this Policy, or, if the loss or damage be occasioned by the willful
alleged case was really not mentioned in the previous facts); act or with connivance of the Insured, or, if the claim be
b.   that this case was dismissed without prejudice on September 3, made and rejected and an action or suit be not
1957 and that deducting the period within which said action was commenced within twelve months after such rejection or
pending, the present action was still within the 12 month period (in case of arbitration place in pursuance of the 18th
from April 12, 1956. condition of this Policy) within twelve months after the
23.   The CFI held that the bringing of the civil action on May 11, 1956 (fact arbitrator or arbitrators or umpire shall have made their
10[a]), tolled the running of the 12 month period within which the action award, all benefits under this Policy shall be forfeited.
must be filed. Said the court on this point: 9.   Fulton cited in support of its contention the cases of (but only the second
a.   “True, indeed, plaintiffs (Spouses Ang) committed a procedural case was explained by the SC in ratio no. 6)
mistake in first suing the agent (Paramount Surety) instead of its a.   E. Macias & Co. vs. Warner, Barnes & Co., Ltd.,;
principal (Fulton), the herein defendant, as correctly pointed out by b.   E. Macias & Co. vs. China Fire Insurance Co., and
counsel for the defendant. But the mistake being merely c.   Castillo etc. vs. Metropolitan Insurance Co.,
procedural, and the defendant not having been misled by the error, 10.   Paulo et al contend that their action had not yet prescribed at the time of the
'There is nothing sacred about process or pleadings, their forms or bringing of the action, because the period of prescription was interrupted by
contents.” the filing of the first action against the Paramount Surety & Insurance Co.,
b.   “The complaint was dismissed by the Court without prejudice on in accordance with Article 1155 of the Civil Code.
September 3, 1957, and motion for reconsideration dated a.   Counsel further argues that the basis of prescription of an action is
September 21, 1957. The instant complaint was filed on May 8, the abandonment by a person of his right of action or claim, so that
1958. The Rules of Court is applicable in the computation of time. any act of said person tending to show his intention not to abandon
Now, as correctly pointed out by the plaintiffs' counsel, by simple his right of action or claim, as the filing of the previous action in
mathematical computation, the present action was filed leas thin the case at bar, interrupts the period of prescription.
nine (9) months after the notice of rejection received by plaintiffs b.   Furthermore, counsel argues, the dismissal of the previous action is
on April 19, 1956, because the filing of the original complaint without prejudice, which means that plaintiffs have the right to file
stopped the running of the period.” another complaint against the principal.
24.   In view of the reasons thus above quoted, the court rendered decision in 11.   The basic error committed by the trial court is its view that the filing of the
favor of the Spouses Ang. Hence, this petition. action against the agent of Fulton was "merely a procedural mistake of no
significance or consequence, which may be overlooked." The condition
ISSUE: contained in the insurance policy that claims must be presented within
2.   Whether or not the court below erred in holding that the filing of the one year after rejection is NOT merely a procedural requirement. The
previous suit tolled or suspended the running of the prescriptive period – condition is an important matter, essential to a prompt settlement of
YES, - The condition contained in the insurance policy that claims must be claims against insurance companies, as it demands that insurance suits
be brought by the insured while the evidence as to the origin and cause
of destruction have not yet disappeared. It is in the nature of a
condition precedent to the liability of the insurer, or in other terms, a
resolutory cause, the purpose of which is to terminate all liabilities in
case the action is not filed by the insured within the period stipulated.
12.   The bringing of the action against the Paramount Surety, the agent of
Fulton CANNOT have any legal effect except that of notifying the agent
of the claim. Beyond such notification, the filing of the action can serve
no other purpose. There is no law giving any effect to such action upon
the principal. Besides, there is no condition in the policy that the action
must be filed against the agent, and this Court can not by
interpretation, extend the clear scope of the agreement beyond what is
agreed upon by the parties.
13.   The case of E. Macias & Co. vs. China Fire Insurance Co. has settled the
issue presented by Paulo et al in the case at bar definitely against their
claim. In that case, the SC declared that the contractual station in an
insurance policy prevails over the statutory limitation, as well as over
the exceptions to the statutory limitations that the contract necessarily
supersedes the statute (of limitations) and the limitation is in all phases
governed by the former.
14.   As stated in said case and in accordance with the decision of the Supreme
Court of the United States in Riddlesbarger vs. Hartford Fire Insurance Co.,
the rights of the parties flow from the contract of insurance, hence they
are NOT bound by the statute of limitations nor by exemptions thereto.
In the words of our own law, their contract is the law between the parties,
and their agreement that an action on a claim denied by the insurer
must be brought within one year from the denial, governs, NOT the
rules on the prescription of actions.
018 SUN INSURANCE v. CA and TAN (Arcenas) 12.   August 20, 1983 - Tan filed his claim for fire loss with SUN INSURANCE,
March 13, 1991 | Paras, J. | Statute of Limitations but on February 29, 1984, SUN INSURANCE wrote to Tan denying his
claim.
13.   April 3, 1984 - Tan sought reconsideration of the denial of his claim.
PETITIONER: Sun Insurance Office, Ltd.
14.   September 3, 1985 - Tan's counsel wrote to SUN INSURANCE inquiring
RESPONDENTS: Court of Appeals and Emilio Tan
about the status of his April 3, 1984 request for reconsideration to which
SUN INSURNACE replied that the denial of Tan's claim remained
SUMMARY: Emilio Tan insured his electrical supply store with Sun Insurance (the
unchanged, enclosing copies of petitioners' letters of February 29, 1984 and
store was located inside the building of his brother in Iloilo City). Four days after the
May 17, 1985 (response to petition for reconsideration).
issuance of the policy (Aug 19 ’83), the building burned, including the insured store.
15.   November 20, 1985 - Tan filed a civil case with the RTC but SUN
Tan’s claimed for fire loss the day after (aug 20 ’83) but was denied by Sun
INSURANCE filed a motion to dismiss on the alleged ground that the
Insurance months later (Feb 29 ’84). Tan received the denial letter (apr 2, ’84) and
action had already prescribed.
sought reconsideration (apr 3 ’84). But there was no communication from Sun
16.   RTC RULING: MTD and the subsequent motion for reconsideration was
Insurance so Tan’s counsel inquired about the status a year later (Sep 3, ’85) to
denied
which Sun Insurance reiterated the denial. Hence, Tan filed a civil case against Sun
17.   CA RULING: SUN INSURNANCE’s petition was denied and the court
Insurnace with the RTC. Sun Insurance filed a motion to dismiss alleging that the
held that the court a quo may continue until its final termination. SUN
action had already prescribed. The MTD was denied by RTC which was also
INSURANCE’s MR was also denied.
affirmed by the CA. Hence, this petition. The issue before the SC is w/n the filing of
18.   Hence, the instant petition.
a motion for reconsideration interrupts the 12-month prescriptive period to contest
denial of claim. The Court held in the negative. The clear and unambigious wording
ISSUE/s:
of the insurance policy must be taken and understood in their plain, ordinary and
2.   w/n the filing of a motion for reconsideration interrupts the twelve (12)
popular sense; hence, since condition 27 of the insurance policy states that if no
months prescriptive period to contest the denial of the insurance claim –
action or suit is commenced within 12 months from receipt of rejection or in case of
NO. The clear and unambigious wording of the insurance policy must be
arbitration, within 12 months after due notice of the award made by the arbitrator,
taken and understood in their plain, ordinary and popular sense; hence,
the claim is deemed abandoned and shall not be recoverable. Since Tan admitted
since condition 27 of the insurance policy states that if no action or suit is
that he received a copy of the letter of rejection on April 2, 1984, the 12-month
commenced within 12 months from receipt of rejection or in case of
prescriptive period started to run from the said date of April 2, 1984, for such is the
arbitration, within 12 months after due notice of the award made by the
plain meaning and intention of Section 27 of the insurance policy. Furthermore, it is
arbitrator, the claim is deemed abandoned and shall not be recoverable.
apparent that Section 27 of the insurance policy was stipulated pursuant to Section
63 of the Insurance Code. The condition contained in an insurance policy that claims
RULING: the questioned decision of the Court of Appeals is REVERSED and SET
must be presented within one year after rejection is not merely a procedural
ASIDE, and Civil Case No. 16817 filed with the Regional Trial Court is hereby
requirement but an important matter essential to a prompt settlement of claims
DISMISSED
against insurance companies as it demands that insurance suits be brought by the
insured while the evidence as to the origin and cause of destruction have not yet
RATIO:
disappeared.
Filing of MR does not interrupt 12-month prescriptive period to contest denial of
insurance claim
DOCTRINE: Sec. 63. A condition, stipulation or agreement in any policy of
9.   While it is a cardinal principle of insurance law that a policy or contract of
insurance, limiting the time for commencing an action thereunder to a period of less
insurance is to be construed liberally in favor of the insured and strictly
than one year from the time when the cause of action accrues, is void.
against the insurer company, yet, contracts of insurance, like other
contracts, are to be construed according to the sense and meaning of the
FACTS: terms which the parties themselves have used.
10.   Aug 15, 1983- Emilio Tan (Tan) took from Sun Insurance Office, Ltd (SUN 10.   If such terms are clear and unambiguous, they must be taken and
INSURANCE) a P300,000.00 property insurance policy to cover his understood in their plain, ordinary and popular sense.
interest in the electrical supply store of his brother which is in a building in 11.   Condition 27 of the Insurance Policy, which is the subject of the conflicting
Iloilo City. contentions of the parties, reads:
11.   Aug 19 - Four (4) days after the issuance of the policy, the building was
burned including the insured store.
a.   27. Action or suit clause — If a claim be made and rejected and an i.   Since "cause of action" requires as essential elements
action or suit be not commenced either in the Insurance not only a legal right of the plaintiff and a correlated
Commission or in any court of competent jurisdiction within obligation of the defendant in violation of the said
twelve (12) months from receipt of notice of such rejection, or legal right, the cause of action does not accrue until the
in case of arbitration taking place as provided herein, within party obligated (surety) refuses, expressly or impliedly, to
twelve (12) months after due notice of the award made by the comply with its duty (in this case to pay the amount of the
arbitrator or arbitrators or umpire, then the claim shall for all bond).
purposes be deemed to have been abandoned and shall not 6.   Indisputably, the above-cited pronouncements of this Court may be taken to
thereafter be recoverable hereunder. mean that the insured's cause of action or his right to file a claim either
12.   As the terms are very clear and free from any doubt or ambiguity in the Insurance Commission or in a court of competent jurisdiction
whatsoever, it must be taken and understood in its plain, ordinary and commences from the time of the denial of his claim by the Insurer,
popular sense pursuant to the above-cited principle laid down by this Court. either expressly or impliedly.
13.   Tan, in his letter addressed to SUN INSURANCE, admitted that he received 7.   But as pointed out by the SUN INSURANCE, the rejection referred to
a copy of the letter of rejection on April 2, 1984. Thus, the 12-month should be construed as the rejection, in the first instance, for if what is being
prescriptive period started to run from the said date of April 2, 1984, for referred to is a reiterated rejection conveyed in a resolution of a petition for
such is the plain meaning and intention of Section 27 of the insurance reconsideration, such should have been expressly stipulated.
policy. a.   Thus, to allow the filing of a motion for reconsideration to suspend
14.   While the question of whether or not the insured was definitely advised of the running of the prescriptive period of twelve months, a whole
the rejection of his claim through the letter dated February 29, 1984, may new body of rules on the matter should be promulgated so as to
arise, the certainty of the denial of Tan's claim was clearly manifested in avoid any conflict that may be brought by it, such as:
said letter (see end of digest) i.   whether the mere filing of a plea for reconsideration of a
15.   In Ang v. Fulton Fire Insurance Co, the Court had settled the rationale for denial is sufficient or must it be supported by
the necessity of bringing suits against the Insurer within one year from the arguments/affidavits/material evidence;
rejection of the claim. The condition contained in an insurance policy that ii.   how many petitions for reconsideration should be
claims must be presented within one year after rejection is not merely a permitted?
procedural requirement but an important matter essential to a prompt 8.   Contrary to Tan’s view, SC held that the use of the phrase "final rejection"
settlement of claims against insurance companies as it demands that cannot be taken to mean the rejection of a petition for reconsideration
insurance suits be brought by the insured while the evidence as to the as insisted by CA and Tan. Such was clearly not the meaning
origin and cause of destruction have not yet disappeared. contemplated by this Court. The Insurance policy in said case provides that
Applying to this case: the insured should file his claim, first, with the carrier and then with the
16.   It is apparent that Section 27 of the insurance policy was stipulated pursuant insurer. The "final rejection" being referred to in said case is the rejection
to Section 63 of the Insurance Code, which states that: by SUN INSURANCE
a.   Sec. 63. A condition, stipulation or agreement in any policy of DENIAL LETTER
insurance, limiting the time for commencing an action thereunder We refer to your claim for fire loss of 20th August, 1983 at Huervana St., La Paz, Iloilo City.
to a period of less than one year from the time when the cause of We now have the report of our adjusters and after a thorough and careful review of the same
action accrues, is void. and the accompanying documents at hand, we are rejecting, much to our regrets, liability for
the claim under our policies for one or more of the following reasons:
When does the cause of action accrue?
1. xxx xxx xxx
5.   Tan used two cases to prove his point: 2. xxx xxx xxx
a.   Eagle Star Insurance Co. vs. Chia Yu where the Court held: For your information, we have referred all these matters to our lawyers for their opinion as to
i.   The right of the insured to the payment of his loss accrues the compensability of your claim, particularly referring to the above violations. It is their
from the happening of the loss. However, the cause of opinion and in fact their strong recomendation to us to deny your claim. By this letter, we do
action in an insurance contract does not accrue until not intend to waive or relinquish any of our rights or defenses under our policies of insurance.
the insured's claim is finally rejected by the insurer.
This is because before such final rejection there is no real 019 Coastwise Lighterage v. Court of Appeals (Linds)
necessity for bringing suit; July 12, 1995 | Francisco, R., J. | Affreightment; Subrogation
b.   ACCFA vs. Alpha Insurance & Surety Co., Inc.
PETITIONER: Coastwise Lighterage Corporation compartment was damaged, and water gushed in through a hole "two inches
RESPONDENTS: Court of Appeals, and Phil Gen Insurance wide and twenty-two inches long.
73.   As a consequence, the molasses at the cargo tanks were contaminated and
SUMMARY: Pag-asa entered into a contract to transport molasses from Negros rendered unfit for the use it was intended. This prompted the consignee,
to Manila with Coastwise using latter’s dumb barges which were towed in Pag-asa Sales, Inc. to reject the shipment of molasses as a total loss.
tandem by MT Marica, owned by Coastwise. Upon reaching Pier 18 in Manila Thereafter, Pag-asa Sales, Inc. filed a formal claim with the insurer of its
bay, one of the barges hit an unknown object, which caused a hole and the lost cargo, herein private respondent, Philippine General Insurance
molasses inside were contaminated. Pag-asa rejected the shipment as a total loss. Company (PhilGen, for short) and against the carrier, herein petitioner,
Pag-asa filed formal claims against PhilGen (Pag-asa’s insurer) and Coastwise. Coastwise Lighterage.
Coastwise rejected, forcing Philgen to pay the claim amounting to P700T. 74.   Coastwise Lighterage denied the claim and it was PhilGen which paid the
Philgen filed a claim against Coastwise by virtue of subrogation. RTC granted. consignee, Pag-asa Sales, Inc., the amount of P700,000.00, representing the
CA affirmed. value of the damaged cargo of molasses.
75.   In turn, PhilGen then filed an action against Coastwise Lighterage before
The issues before the Court are whether Coastwise was acting as common carrier the Regional Trial Court of Manila, seeking to recover the amount of
under the agreement, and whether Philgen was subrogated upon payment of Pag- P700,000.00 which it paid to Pag-asa Sales, Inc. for the latter's lost cargo.
asa’s goods. 76.   PhilGen now claims to be subrogated to all the contractual rights and claims
which the consignee may have against the carrier, which is presumed to
Anent the first issue, Coastwise was a common carrier. Coastwise Lighterage, by have violated the contract of carriage.
the contract of affreightment, was not converted into a private carrier, but remained a 77.   The RTC awarded the amount prayed for by PhilGen. On Coastwise
common carrier and was still liable as such. the presumption of negligence that attaches to Lighterage's appeal to the Court of Appeals, the award was affirmed.
common carriers, once the goods it transports are lost, destroyed or deteriorated, applies
to the petitioner. This presumption, which is overcome only by proof of the exercise of
extraordinary diligence, remained unrebutted in this case. Coastwise was negligent since
the patron of the vessel was unlicensed.
ISSUE/s:
Anent the second issue, Undoubtedly, upon payment by respondent insurer PhilGen of the
amount of P700,000.00 to Pag-asa Sales, Inc., the consignee of the cargo of molasses
3.   Whether petitioner Coastwise Lighterage was transformed into a private
totally damaged while being transported by petitioner Coastwise Lighterage, the former carrier, by virtue of the contract of affreightment which it entered into with
was subrogated into all the rights which Pag-asa Sales, Inc. may have had against the the consignee, Pag-asa Sales, Inc. Corollarily, if it were in fact transformed
carrier, herein petitioner Coastwise Lighterage. into a private carrier, did it exercise the ordinary diligence to which a
private carrier is in turn bound.
DOCTRINE: A common carrier, by the contract of affreightment, was not converted a.   No, still common carrier, which needs to exercise
into a private carrier, but remained a common carrier and was still liable as such. extraordinary diligence in the vigilance over cargoes.
4.   Whether the insurer was subrogated into the rights of the consignee against
Payment by the insurer to the assured operated as an equitable assignment to the former the carrier, upon payment by the insurer of the value of the consignee's
of all remedies which the latter may have against the third party whose negligence or goods lost while on board one of the carrier's vessels.
wrongful act caused the loss. The right of subrogation is not dependent upon, nor does it
grow out of, any privity of contract or upon written assignment of claim. It accrues simply
upon payment of the insurance claim by the insurer.
RULING: WHEREFORE, premises considered, this petition is DENIED and the
appealed decision affirming the order of Branch 35 of the Regional Trial Court of
FACTS: Manila for petitioner Coastwise Lighterage to pay respondent Philippine General
71.   Pag-asa Sales, Inc. entered into a contract to transport molasses from the Insurance Company the "principal amount of P700,000.00 plus interest thereon at the
province of Negros to Manila with Coastwise Lighterage Corporation legal rate computed from March 29, 1989, the date the complaint was filed until fully
(Coastwise for brevity), using the latter's dumb barges. The barges were paid and another sum of P100,000.00 as attorney's fees and costs"10 is likewise
towed in tandem by the tugboat MT Marica, which is likewise owned by hereby AFFIRMED
Coastwise.
72.   Upon reaching Manila Bay, while approaching Pier 18, one of the barges, RATIO:
"Coastwise 9", struck an unknown sunken object. The forward buoyancy
Clearly, petitioner Coastwise Lighterage's embarking on a voyage with an
Arguments before the Court re: first issue unlicensed patron cannot safely claim to have exercised extraordinary
diligence, by placing a person whose navigational skills are questionable, at the
11.   On the first issue, petitioner (Coastwise) contends that the RTC and the helm of the vessel which eventually met the fateful accident. It may also
logically, follow that a person without license to navigate, lacks not just the skill to
Court of Appeals erred in finding that it was a common carrier. It stresses
do so, but also the utmost familiarity with the usual and safe routes taken by
the fact that it contracted with Pag-asa Sales, Inc. to transport the shipment seasoned and legally authorized ones. Had the patron been licensed, he could be
of molasses from Negros Oriental to Manila and refers to this contract as a presumed to have both the skill and the knowledge that would have prevented the
"charter agreement". It then proceeds to cite the case of Home Insurance vessel's hitting the sunken derelict ship that lay on their way to Pier 18.
Company vs. American Steamship Agencies, Inc. wherein this Court held:
". . . a common carrier undertaking to carry a special cargo or chartered to a Ruling re: second issue
special person only becomes a private carrier." 17.   On the issue of subrogation, which petitioner contends as inapplicable in this case,
we once more rule against the petitioner. We have already found petitioner liable for
Ruling re: first issue breach of the contract of carriage it entered into with Pag-asa Sales, Inc. However,
12.   Petitioner's (Coastwise’) reliance on the aforementioned case is misplaced. In its for the damage sustained by the loss of the cargo which petitioner-carrier was
entirety, the conclusions of the court are as follows: transporting, it was not the carrier which paid the value thereof to Pag-asa Sales, Inc.
a.   Accordingly, the charter party contract is one of affreightment over the but the latter's insurer, herein private respondent PhilGen.
whole vessel, rather than a demise. As such, the liability of the shipowner 18.   Article 2207 of the Civil Code is explicit on this point:
for acts or negligence of its captain and crew, would remain in the absence a.   Art. 2207. If the plaintiffs property has been insured, and he has received
of stipulation. indemnity from the insurance company for the injury or loss arising out of
13.   The distinction between the two kinds of charter parties (i.e. bareboat or the wrong or breach of contract complained of, the insurance company
demise and contract of affreightment) is more clearly set out in the case of shall be subrogated to the rights of the insured against the wrongdoer or
Puromines, Inc. vs. Court of Appeals, wherein we ruled: the person who violated the contract.
a.   Under the demise or bareboat charter of the vessel, the charterer will 19.   Article 2207 of the Civil Code is founded on the well-settled principle of
generally be regarded as the owner for the voyage or service stipulated. subrogation. If the insured property is destroyed or damaged through the fault or
The charterer mans the vessel with his own people and becomes the owner negligence of a party other than the assured, then the insurer, upon payment to the
pro hac vice, subject to liability to others for damages caused by assured will be subrogated to the rights of the assured to recover from the wrongdoer
negligence. To create a demise, the owner of a vessel must completely and to the extent that the insurer has been obligated to pay. Payment by the insurer to the
exclusively relinquish possession, command and navigation thereof to the assured operated as an equitable assignment to the former of all remedies which the
charterer, anything short of such a complete transfer is a contract of latter may have against the third party whose negligence or wrongful act caused the
affreightment (time or voyage charter party) or not a charter party at all. loss. The right of subrogation is not dependent upon, nor does it grow out of, any
b.   On the other hand a contract of affreightment is one in which the owner of privity of contract or upon written assignment of claim. It accrues simply upon
the vessel leases part or all of its space to haul goods for others. It is a payment of the insurance claim by the insurer.
contract for special service to be rendered by the owner of the vessel and 20.   Undoubtedly, upon payment by respondent insurer PhilGen of the amount of
under such contract the general owner retains the possession, command P700,000.00 to Pag-asa Sales, Inc., the consignee of the cargo of molasses totally
and navigation of the ship, the charterer or freighter merely having use of damaged while being transported by petitioner Coastwise Lighterage, the former
the space in the vessel in return for his payment of the charter hire. was subrogated into all the rights which Pag-asa Sales, Inc. may have had against
c.   An owner who retains possession of the ship though the hold is the the carrier, herein petitioner Coastwise Lighterage.
property of the charterer, remains liable as carrier and must answer for any
breach of duty as to the care, loading and unloading of the cargo
14.   Although a charter party may transform a common carrier into a private one, the
same however is not true in a contract of affreightment on account of the
aforementioned distinctions between the two. Pursuant therefore to the ruling in
the aforecited Puromines case, Coastwise Lighterage, by the contract of
affreightment, was not converted into a private carrier, but remained a
common carrier and was still liable as such.
15.   It follows then that the presumption of negligence that attaches to common carriers,
once the goods it transports are lost, destroyed or deteriorated, applies to the
petitioner. This presumption, which is overcome only by proof of the exercise of
extraordinary diligence, remained unrebutted in this case.
16.   Based on the records, the patron of Coastwise’ vessel was not even licensed.
001 GEAGONIA v. CA (BALISONG) 8.   Geagonia declared in the policy under the subheading entitled CO-
6 Feb. 1995 | Davide, Jr., J. | “Other Insurance” Clause; Insurable Interest in INSURANCE that Mercantile Insurance Co., Inc. was the co-insurer for
Mortgage P50,000.00.

PETITIONER: Armando Geagonia 9.   The policy contained the following condition:


RESPONDENTS: Court of Appeals and Country Bankers Insurance Corporation
Condition 3. The insured shall give notice to the Company of any
insurance or insurances already effected, or which may
SUMMARY: Geagonia insured his stocks-in-trade with Country Bankers for
subsequently be effected, covering any of the property or
P100,000. Pursuant to Condition 3 requiring disclosure of other insurance policies
properties consisting of stocks in trade, goods in process and/or
covering the same subject matter for more than P200,000, Geagonia declared that
inventories only hereby insured, and unless notice be given and the
Mercantile Insurance was a co-insurer for P50,000. The goods were razed by fire.
particulars of such insurance or insurances be stated therein or
Geagonia filed a claim, Country Bankers denied the claim on the ground that the
endorsed in this policy pursuant to Section 50 of the Insurance
Condition 3 was violated when Geagonia did not declare that the goods were also
Code, by or on behalf of the Company before the occurrence of
covered by two insurance policies issued by PFIC for P100,000 each. The PFIC
any loss or damage, all benefits under this policy shall be deemed
policies contained loss-payable clauses in favor of mortgagee Cebu Tesing. The
forfeited, provided however, that this condition shall not apply
issue is whether Geagonia may claim from Country Bankers. The Court held that he
when the total insurance or insurances in force at the time of the
may. The “Other Insurance” clause is not proscribed by law. It is a provision which
loss or damage is not more than P200,000.00.
invariably appears in fire insurance policies and is intended to prevent an increase in
the moral hazard. However, in order to constitute a violation, the other insurance 10.   On 27 May 1990 a fire broke out in the public market. The insured stocks-
must be upon the same subject matter, the same interest therein, and the same in-trade were completely destroyed. Geagonia filed a claim against Country
risk. Condition 3 in the Country Banker’s policy does not absolutely declare void Bankers, but the claim was denied because Country Bankers found that at
any violation thereof. It expressly provides that the condition "shall not apply when the time of the loss, Geagonia’s stocks-in-trade were likewise insured by
the total insurance or insurances in force at the time of the loss or damage is not two insurance policies worth P100,000 each with Philippines First
more than P200,000.00. Provisions, conditions or exceptions in policies which Insurance, Co., Inc. (PFIC) in violation of the condition in the policy.
tend to work a forfeiture of insurance policies should be construed most strictly
against those for whose benefits they are inserted, and most favorably toward 11.   The PFIC policies indicate that the insured was Discount Mart under the
those against whom they are intended to operate. The prohibition applies only to proprietorship of Geagonia, and with a mortgage clause making the loss
double insurance, and the nullity of the policy shall only be to the extent exceeding payable to Cebu Tesing Textiles, as their interest may appear. Hence, this
P200,000.00 of the total policies obtained. petition for collection of sum of money for the recovery of P100,000.

DOCTRINE: In order to constitute a violation, the other insurance must be upon the 12.   The Insurance Commission ruled in favor of Geagonia saying that he had
same subject matter, the same interest therein, and the same risk. no knowledge of the PFIC policies because it was Cebu Tesing which
procured them. The CA reversed the IC, holding that Geagonia knew of the
Provisions, conditions or exceptions in policies which tend to work a forfeiture of PFIC policies when it procured insurance from Country Bankers as
insurance policies should be construed most strictly against those for whose benefits admitted in his letter dated 18 January 1991.
they are inserted, and most favorably toward those against whom they are intended
to operate. ISSUE/s:
2.   Whether Geagonia may claim from the policy. YES — Condition 3 in the
FACTS: Country Banker’s policy does not absolutely declare void any violation
7.   Geagonia is the owner of Norman's Mart located in the public market of San thereof. It expressly provides that the condition "shall not apply when the
Francisco, Agusan del Sur. On 22 December 1989, he obtained from the total insurance or insurances in force at the time of the loss or damage is not
Country Bankers a fire insurance policy for P100,000.00. The period of the more than P200,000.00.
policy was from 22 December 1989 to 22 December 1990 and covered the
following: "Stock-in-trade consisting principally of dry goods such as RULING: WHEREFORE, the instant petition is hereby GRANTED. The decision
RTW's for men and women wear and other usual to assured's business." of the Court of Appeals in CA-G.R. SP No. 31916 is SET ASIDE and the decision of
the Insurance Commission in Case No. 3340 is REINSTATED.
RATIO:
4.   Geagonia knew of the PFIC policies when he applied for insurance with
Country Bankers. It was incredible that he did not know about the prior
policies since these policies were not new or original. Policy No. GA-28144
was a renewal of Policy No. F-24758, while Policy No. GA-28146 had been
renewed twice, the previous policy being F-24792.

5.   The “Other Insurance” clause is not proscribed by law. It is a provision


which invariably appears in fire insurance policies and is intended to
prevent an increase in the moral hazard. However, in order to constitute a
violation, the other insurance must be upon the same subject matter,
the same interest therein, and the same risk.

6.   As to mortgaged property, the mortgagor and mortgagee have independent


interests therein. The mortgagor’s interest covers the full value of the
property, the mortgagee’s interest is to the extent of the debt to which the
property is attached as security.

7.   In the policy obtained by the mortgagor with loss payable clause in favor of
the mortgagee as his interest may appear, the mortgagee is only a
beneficiary under the contract, and recognized as such by the insurer but not
made a party to the contract itself. Hence, any act of the mortgagor which
defeats his right will also defeat the right of the mortgagee. This kind of
policy covers only such interest as the mortgagee has at the issuing of the
policy. The PFIC policies contain a loss payable clause. This would have
violated the “Other Insurance” clause.

8.   Be that as it may, Condition 3 in the Country Banker’s policy does not


absolutely declare void any violation thereof. It expressly provides that the
condition "shall not apply when the total insurance or insurances in force at
the time of the loss or damage is not more than P200,000.00. Provisions,
conditions or exceptions in policies which tend to work a forfeiture of
insurance policies should be construed most strictly against those for
whose benefits they are inserted, and most favorably toward those
against whom they are intended to operate.

9.   Condition 3 of the subject policy is not totally free from ambiguity and
must, perforce, be meticulously analyzed. Such analysis leads us to
conclude that (a) the prohibition applies only to double insurance, and (b)
the nullity of the policy shall only be to the extent exceeding P200,000.00
of the total policies obtained.
002 PIONEER INSURANCE v. YAP (Buenaventura) DOCTRINE:
December 19, 1974 | Fernandez, J. | Double Insurance The validity of a clause in a fire insurance policy to the effect that the
procurement of additional insurance WITHOUT the consent of the insurer
PETITIONER: Pioneer Insurance and Surety Corporation renders ipso facto the policy void
RESPONDENTS: Oliva Yap The obvious purpose of the aforesaid requirement in the policy is to prevent
SUMMARY: over-insurance and thus avert the perpetration of fraud. The public, as well as
the insurer, is interested in preventing the situation in which a fire would be
Oliva Yap was the owner of a store in a two-storey building where she sold profitable to the insured.
shopping bags and footwear.
Yap took out a Fire Insurance Policy No. 4216 from Pioneer Insurance with a
value of P25,000.00 covering her stocks, office furniture, fixtures and fittings.
Among the conditions in the policy executed by the parties are the following: FACTS:
The Insured shall give notice to the Company of any insurance or insurances 1.   Respondent Oliva Yap was the owner of a store in a two-storey building
already effected, or which may subsequently be effected and unless such notice where she sold shopping bags and footwear, such as shoes, sandals and
be given and the particulars of such insurance or insurances be stated in, or step-ins. Chua Soon Poon, Oliva Yap's son-in-law, was in charge of the
endorsed on this Policy by or on behalf of the Company before the occurrence store.
of any loss or damage, all benefits under this Policy shall be forfeited 2.   On April 19, 1962, respondent Yap took out Fire Insurance Policy No.
Another insurance policy for P20,000.00 issued by Great American covering the 4216 from petitioner Pioneer Insurance & Surety Corporation with a
same properties. The endorsement recognized co-insurance by Northwest for face value of P25,000.00 covering her stocks, office furniture, fixtures and
the same value. fittings of every kind and description. Among the conditions in the policy
Oliva Yap took out another fire insurance policy for P20,000.00 covering the executed by the parties are the following:
same properties from the Federal Insurance Company, Inc., which was procured i.   The Insured shall give notice to the Company of any
WITHOUT notice to and the written consent of Pioneer. insurance or insurances already effected, or which may
A fire broke out in the building, and the store was burned. Yap filed an subsequently be effected, covering any of the property
insurance claim, but the same was denied for a breach. hereby insured, and unless such notice be given and the
Oliva Yap filed a case for payment of the face value of her fire insurance policy. particulars of such insurance or insurances be stated in,
The insurance company refused to pay because she never informed Pioneer of or endorsed on this Policy by or on behalf of the
another insurer. The trial court decided in favor of Yap. The CA affirmed saying Company before the occurrence of any loss or damage,
there was a substitution of insurance policies. all benefits under this Policy shall be forfeited.
ii.   It is understood that, except as may be stated on the face
The issue is Whether or not Pioneer should be absolved from liability on the of this policy there is no other insurance on the property
policy on account of any violation of the co-insurance clause. SC held yes, there hereby covered and no other insurance is allowed except
was a violation. by the consent of the Company endorsed hereon. Any
The insurance policy for P20,000.00 issued by the Great American, ceased to be false declaration or breach or this condition will render
recognized by them as a co-insurance policy. this policy null and void.
The endorsement shows the clear intention of the parties to recognize on the 3.   At the time of the insurance on April 19, 1962 of Policy No. 4219 in favor
date the endorsement was made, the existence of only one co-insurance, the of respondent Yap, an insurance policy for P20,000.00 issued by the
Northwest one. The finding of the Court of Appeals that the Great American Great American Insurance Company covering the same properties was
Insurance policy was substituted by the Federal Insurance policy is indeed noted on said policy as co-insurance. Later, on August 29, 1962, the
contrary to said stipulation. parties executed as an endorsement on Policy No. 4219, stating:
Other insurance without the consent of Pioneer would avoid the contract. It i.   It is hereby declared and agreed that the co-insurance
required no affirmative act of election on the part of the company to make existing at present under this policy is as follows:
operative the clause avoiding the contract, wherever the specified conditions P20,000.00 — Northwest Ins., and not as originally
should occur. Its obligations ceased, unless, being informed of the fact, it stated. (emphasis supplied)
consented to the additional insurance. ii.   Except as varied by this endorsement, all other terms and
conditions remain unchanged.
4.   Still later, or on September 26, 1962, respondent Oliva Yap took out policy, it could only be the Northwest Insurance policy for the same amount
another fire insurance policy for P20,000.00 covering the same of P20,000.00. The endorsement quoted above shows the clear intention of
properties, this time from the Federal Insurance Company, Inc., which the parties to recognize on the date the endorsement was made the existence
new policy was, however, procured without notice to and the written of only one co-insurance, and that is the Northwest Insurance policy, which
consent of petitioner Pioneer Insurance & Surety Corporation and, according to the stipulation of the parties during the hearing and endorsed
therefore, was not noted as a co-insurance in Policy No. 4219. only on August 20, 1962. The finding of the Court of Appeals that the Great
5.   At dawn on December 19, 1962, a fire broke out in the building housing American Insurance policy was substituted by the Federal Insurance policy
respondent Yap's above-mentioned store, and the said store was burned. is unsubstantiated by the evidence of record.
Respondent Yap filed an insurance claim, but the same was denied in on the 3.   The Court of Appeals would consider petitioner to have waived the formal
ground of "breach and/or violation of any and/or all terms and conditions" requirement of endorsing the policy of co-insurance "since there was
of Policy No. 4219. absolutely no showing that it was not aware of said substitution and
6.   Yap filed with the CFI of Manila the present complaint, asking, among preferred to continue the policy." The fallacy of this argument is that,
others, for payment of the face value of her fire insurance policy. In its contrary to Section 1, Rule 131 of the Revised Rules of Court, which
answer, Pioneer alleged that no property belonging to plaintiff Yap and requires each party to prove his own allegations, it would shift to petitioner
covered by the insurance policy was destroyed by the fire; that Yap's claim Pioneer, respondent Yap's burden of proving her proposition that Pioneer
was filed out of time; and that Yap took out an insurance policy from was aware of the alleged substitution, and with such knowledge preferred to
another insurance company without petitioner's knowledge and/or continue the policy. Respondent Yap cites Gonzales La O vs. Yek Tong Lin
endorsement, in violation of the express stipulations in Policy No. 4219, Fire and Marine Insurance Co., Ltd. to justify the assumption but in that
hence, all benefits accruing from the policy were deemed forfeited. case, unlike here, there was knowledge by the insurer of violations of the
7.   The trial court decided for plaintiff Oliva Yap; and its judgment was contract, to wit: "If, with the knowledge of the existence of other insurances
affirmed in full by the Court of Appeals. which the defendant deemed violations of the contract, it has preferred to
ISSUE: continue the policy, its action amounts to a waiver of the annulment of the
1.   whether or not Pioneer should be absolved from liability on account of any contract ..." A waiver must be express. If it is to be implied from conduct
violation by Yap of the co-insurance clause – YES, The validity of a clause mainly, said conduct must be clearly indicative of a clear intent to
in a fire insurance policy to the effect that the procurement of additional waive such right. Especially in the case at bar where petitioner is assumed
insurance without the consent of the insurer renders ipso facto the policy to have waived a valuable right, nothing less than a clear, positive waiver,
void made with full knowledge of the circumstances, must be required.
RULING: WHEREFORE, the appealed judgment of the Court of Appeals is 4.   By the plain terms of the policy, other insurance without the consent of
reversed and set aside, and the petitioner absolved from all liability under the policy. petitioner would ipso facto avoid the contract. It required no
Costs against private respondent. SO ORDERED. affirmative act of election on the part of the company to make
operative the clause avoiding the contract, wherever the specified
conditions should occur. Its obligations ceased, unless, being informed of
RATIO: the fact, it consented to the additional insurance.
5.   The validity of a clause in a fire insurance policy to the effect that the
1.   There was a violation by respondent Oliva Yap of the co-insurance clause procurement of additional insurance without the consent of the insurer
contained in Policy No. 4219 that resulted in the avoidance of petitioner's renders ipso facto the policy void is well-settled:
liability. The insurance policy for P20,000.00 issued by the Great American 6.   In Milwaukee Mechanids' Lumber Co., vs. Gibson, a substantially identical
Insurance Company covering the same properties of respondent Yap and clause was sustained and enforced, the court saying: "The rule in this state
duly noted on Policy No. 4219 as c-insurance, ceased, by agreement of the and practically all of the states is to the effect that a clause in a policy to the
parties, to be recognized by them as a co-insurance policy. effect that the procurement of additional insurance without the consent of
2.   The Court of Appeals says that the Great American Insurance policy was the insurer renders the policy void is a valid provision. The earlier cases of
substituted by the Federal Insurance policy for the same amount, and Planters Mutual Insurance Co., vs. Green are to the same effect.
because it was a mere case of substitution, there was no necessity for its 7.   Where a policy contains a clause providing that the policy shall be void if
endorsement on Policy No. 4219. This finding, as well as reasoning, suffers insured has or shall procure any other insurance on the property, the
from several flaws. There is no evidence to establish and prove such a procurement of additional insurance without the consent of the insurer
substitution. If anything was substituted for the Great American Insurance avoids the policy." (Planters' Mut. Ins. Ass'n vs. Green)
8.   The policy provided that it should be void in case of other insurance
"without notice and consent of this company. ..." It also authorized the
company to terminate the contract at any time, at its option, by giving notice
and refunding a ratable proportion of the premium. Held, that additional
insurance, unless consented to, or unless a waiver was shown, ipso
facto avoided the contract, and the fact that the company had not, after
notice of such insurance, cancelled the policy, did not justify the legal
conclusion that it had elected to allow it to continue in force." (Johnson vs.
American Fire Ins., Co.)
9.   The aforecited principles have been applied in this jurisdiction in General
Insurance & Surety Corporation vs. Ng Hua. There, the policy issued by the
General Insurance & Surety Corporation in favor of respondent Ng Hua
contained a provision identical with the provisions in Policy No. 4219
quoted above.
10.   “The annotation then, must be deemed to be a warranty that the property
was not insured by any other policy. Violation thereof entitled the insurer to
rescind. Furthermore, even if the annotations were overlooked the defendant
insurer would still be free from liability because there is no question that the
policy issued by General Indemnity has not been stated in nor endorsed on
Policy No. 471 of defendant. The obvious purpose of the aforesaid
requirement in the policy is to prevent over-insurance and thus avert the
perpetration of fraud where a fire would be profitable to the insured
11.   The obvious purpose of the aforesaid requirement in the policy is to
prevent over-insurance and thus avert the perpetration of fraud. The
public, as well as the insurer, is interested in preventing the situation in
which a fire would be profitable to the insured. According to Justice
Story: "The insured has no right to complain, for he assents to comply with
all the stipulation on his side, in order to entitle himself to the benefit of the
contract, which, upon reason or principle, he has no right to ask the court to
dispense with the performance of his own part of the agreement, and yet to
bind the other party to obligations, which, but for those stipulation would
not have been entered into."
Malayan Insurance Co., Inc. v. Phil. First Insurance Co., Inc. (CABUSORA) from any external cause, if by land, with a limit of P6M per any one
July 11, 2012 | Reyes, J. | Double/Overinsurance land vehicle.
61.   (1 Dec 1993) Wyeth executed its annual contract of carriage with
PETITIONER: Malayan Insurance Co., Inc. Reputable. It was signed by Reputable’s representative however not by
RESPONDENTS: Philippines First Insurance Co., Inc. and Reputable Forwarder Wyeth’s but the terms thereof still faithfully observed by the parties.
62.   Reputable undertook to answer for "all risks with respect to the goods and shall be
Services, Inc.
liable to the Wyeth, for the loss, destruction, or damage of the goods/products due to
any and all causes whatsoever, including theft, robbery, flood, storm, earthquakes,
SUMMARY: Wyeth and Reputable entered into a contract of carriage where lightning, and other force majeure while the goods/products are in transit and until
Reputable is to deliver Wyeth’s products and that Reputable is to required to secure actual delivery to the customers, salesmen, and dealers” of Wyeth.
said products with an insurance. Wyeth secured an insurance (Marine Insurance 63.   (11 Feb 1994) Reputable signed a Special Risk Insurance Policy with
Policy) on its own products from Philippines First and Reputable secured said petitioner Malayan for the amount of P1M since the contract of carriage
products as well with a Special Risk Insurance Policy with Malayan as required by required Reputable to secure an insurance policy on Wyeth’s goods.
the contract of carriage. Truck carrying the said cargo was hijacked. Philippines First 64.   (6 Oct 1994) Reputable received from Wyeth 1,000 boxes of Promil infant
paid Wyeth and asked from Reputable for reimbursement. Reputable refused to pay formula worth P2,357,582.70 to be delivered by Reputable to Mercury
and impleaded Malayan for the payment in its own insurance policy. Malayan denies Drug Corporation in Libis, Quezon City. However, the truck carrying
liability according to Secs. 5 and 12 of the SR Insurance Policy with respect to Wyeth’s products was hijacked by about 10 armed men. They threatened to
having double insurance policy. kill the truck driver and two helpers if they refuse to turn over the truck.
(Issue) WON Malayan should be liable. YES. There was no double insurance in this The hijacked truck was recovered two weeks later without its cargo.
case. Under Sec. 93, Insurance Code, double insurance exists where the same 65.   (8 Mar 1995) Philippines First paid Wyeth P2,133,257.00 as indemnity.
person is insured by several insurers separately in respect to the same subject and 66.   Philippines First demanded reimbursement from Reputable who ignored it.
interest (see elements in Ratio #9). While it is true that the Marine Policy and the SR 67.   Philippines First instituted an action for sum of money against Reputable.
Policy were both issued over the same subject matter, i.e. goods belonging to Complaint stated that Reputable is a "private corporation engaged in the business of
Wyeth, and both covered the same peril insured against, it is, however, beyond a common carrier."
cavil that the said policies were issued to two different persons or entities. It is 68.   Reputable claimed (Answer) that it is a private carrier, and it cannot be liable under
undisputed that Wyeth is the recognized insured of Philippines First under its Marine the contract of carriage since it was not signed by Wyeth’s representative (Fact #3)
Policy, while Reputable is the recognized insured of Malayan under the SR Policy. and that the cause of the loss was force majeure (hijacking).
69.   Reputable impleaded Malayan as third-party defendant to collect under the SR
Policy which covered the risk of robbery or hijacking.
DOCTRINE: The requirements of double insurance are: 70.   Malayan disclaimed liability stating that Sec. 5, SR Policy43, the insurance
1.   The person insured is the same; does not cover any loss or damage to property which at the time of the
2.   Two or more insurers insuring separately; happening of such loss or damage is insured by any marine policy and that
3.   There is identity of subject matter; the SR Policy expressly excluded third-party liability.
4.   There is identity of interest insured; and 71.   RTC: Reputable liable to Philippines First for the amount of indemnity it
5.   There is identity of the risk or peril insured against. paid to Wyeth. In turn, Malayan is liable to Reputable to the extent of the
In this case, the two insurance policies (Marine and Special Risk) were acquired by policy coverage.
two different entities (i.e. Marine by Wyeth and Special Risk by Reputable). 72.   Malayan appealed. It further contended that it should be liable for no more
than P468,766.70, its alleged pro-rata share of the loss based on the amount
FACTS: covered by the policy according to Sec. 12, SR Policy44.
59.   Since 1989, Wyeth Philippines, Inc. (Wyeth) and respondent Reputable
Forwarder Services, Inc. (Reputable) had been annually executing a                                                                                                                        
contract of carriage, whereby the latter undertook to transport and deliver 43
Section 5. INSURANCE WITH OTHER COMPANIES. The insurance does not cover any loss or
the former’s products to its customers, dealers or salesmen. damage to property which at the time of the happening of such loss or damage is insured by or would but
for the existence of this policy, be insured by any Fire or Marine policy or policies except in respect of any
60.   (18 Nov 1993) Wyeth procured Marine Policy No. MAR 13797 from excess beyond the amount which would have been payable under the Fire or Marine policy or policies had
respondent Philippines First Insurance Co., Inc. (Philippines First) to this insurance not been effected.
secure its products (nutritional, pharmaceutical and other products 44
12. OTHER INSURANCE CLAUSE. If at the time of any loss or damage happening to any property
usual or incidental to the insured’s business) while in transit to the hereby insured, there be any other subsisting insurance or insurances, whether effected by the insured or
by any other person or persons, covering the same property, the company shall not be liable to pay or
Philippines. The policy covers all risks of direct physical loss or damage contribute more than its ratable proportion of such loss or damage.
73.   CA45: Affirmed with modification RTC Decision. Attorney’s fee awarded to 4.   Sec. 5, SR Policy is actually the other insurance clause (also called
Reputable is deleted. "additional insurance" and "double insurance").
74.   Malayan filed an MR which was denied. Hence, this petition. 5.   In a similar case (Geagonia v. CA) the Court upheld Condition No. 3
(similar to Sec. 5) which was a warranty that no other insurance exists and
ISSUE/s: in case of a violation the condition shall not apply when the total
8.   WON Reputable is a private carrier; – NO. The judicial admission that Reputable is a insurance/s in force is not more than P200K. Such condition is not
common carrier which would be conclusive as to the pleader was made by Philippines proscribed by law as its incorporation in the policy is allowed by Sec. 75,
First and not Reputable, thus, such is not conclusive as to Reputable. Reputable actually
Insurance Code. In this case, Sec. 5 does not provide for the nullity of the
consistently claimed that it is a private carrier.
9.   WON Reputable is strictly bound by the stipulations in its contract of carriage with SR Policy but simply limits the liability of Malayan only up to the excess of
Wyeth, such that it should be liable for any risk of loss or damage, for any cause the amount that was not covered by the other insurance policy.
whatsoever, including that due to theft or robbery and other force majeure; – YES. The 6.   In interpreting the "other insurance clause" in Geagonia, the Court ruled
contract provides for such and a private carrier is bound by the stipulations of such that the prohibition applies only in case of double insurance. The Court
contract. ruled that in order to constitute a violation of the clause, the other
10.   WON the RTC and CA erred in rendering "nugatory" Secs. 5 and 12, insurance must be upon same subject matter, the same interest therein,
SR Policy; – YES. Court found that from the intention of the parties was and the same risk. Thus, even though the multiple insurance policies
that both provisions were to apply in case there is double insurance, involved were all issued in the name of the same assured, over the same
however, Court did not found that this was a case of double insurance, the subject matter and covering the same risk, it was ruled that there was no
two insurances being secured by two different entities. violation of the "other insurance clause" since there was no double
11.   WON Reputable should be held solidarily liable with Malayan for the amount of
P998,000.00 due to Philippines First.
insurance.
7.   Sec. 12, SR Policy, on the other hand, is the over insurance clause. More
RULING: WHEREFORE, premises considered, the petition is DENIED. The particularly, it covers the situation where there is over insurance due to
Decision dated February 29, 2008 and Resolution dated August 28, 2008 of the double insurance. In such case, Sec. 15 provides that Malayan shall "not be
Court of Appeals in CA-G.R. CV No. 71204 are hereby AFFIRMED. Cost against liable to pay or contribute more than its ratable proportion of such loss or
petitioner Malayan Insurance Co., Inc. SO ORDERED. damage." This is in accord with the principle of contribution provided under
Sec. 94(e), Insurance Code, which states that "where the insured is over
RATIO: insured by double insurance, each insurer is bound, as between himself and
On the third issue other insurance vis-a-vis over insurance. the other insurers, to contribute ratably to the loss in proportion to the
1.   Malayan refers to Sec. 5, SR Policy as an "over insurance clause" and to amount for which he is liable under his contract."
Sec. 12 as a "modified other insurance clause." 8.   Clearly, both Secs. 5 and 12 presuppose the existence of a double insurance.
2.   Malayan claims that Secs. 5 and 12 are separate provisions applicable under The pivotal question that now arises is whether there is double insurance.
distinct circumstances. Sec. 5 operates in the case were the assured itself 9.   Under Sec. 93, Insurance Code, double insurance exists where the same
who obtained additional insurance coverage on the same property and the person is insured by several insurers separately in respect to the same
loss incurred by Wyeth’s cargo was more than that insured by Philippines subject and interest. Requirements are:
First’s marine policy. While, Sec. 12 will not completely absolve Malayan 1.   The person insured is the same;
if additional insurance coverage on the same cargo were obtained by 2.   Two or more insurers insuring separately;
someone besides Reputable, in which case Malayan’s SR policy will 3.   There is identity of subject matter;
contribute or share ratable proportion of a covered cargo loss." 4.   There is identity of interest insured; and
3.   Malayan’s position cannot be countenanced. 5.   There is identity of the risk or peril insured against.
10.   In the present case, while it is true that the Marine Policy and the SR Policy
were both issued over the same subject matter, i.e. goods belonging to
                                                                                                                        Wyeth, and both covered the same peril insured against, it is, however,
45
CA findings: (1) Reputable is estopped from assailing the validity of the contract of carriage on the beyond cavil that the said policies were issued to two different persons or
ground of lack of signature of Wyeth s representative/s; (2) Reputable is liable under the contract for the entities. It is undisputed that Wyeth is the recognized insured of Philippines
value of the goods even if the same was lost due to fortuitous event; and (3) Section 12 of the SR Policy First under its Marine Policy, while Reputable is the recognized insured of
prevails over Section 5, it being the latter provision; however, since the ratable proportion provision
of Section 12 applies only in case of double insurance, which is not present, then it should not be
Malayan under the SR Policy.
applied and Malayan should be held liable for the full amount of the policy coverage, that is,
P1,000,000.00.
11.   The fact that Reputable procured Malayan’s SR Policy over the goods 21.   According to the contract of carriage, Reputable shall be liable to Wyeth for the loss of
of Wyeth pursuant merely to the stipulated requirement under its the goods/products due to any and all causes whatsoever, including theft, robbery and
other force majeure while the goods/products are in transit and until actual delivery to
contract of carriage with the latter does not make Reputable a mere Wyeth’s customers, salesmen and dealers.
agent of Wyeth in obtaining the said SR Policy. On the fourth issue Reputable is not solidarily liable with Malayan.
12.   The interest of Wyeth over the property subject matter of both 22.   There is solidary liability only when the obligation expressly so states, when the law so
insurance contracts is also different and distinct from that of provides or when the nature of the obligation so requires.
Reputable’s. The policy issued by Philippines First was in consideration of 23.   Heirs of George Y. Poe v. Malayan lnsurance Company., lnc.: Where the insurance
the legal and/or equitable interest of Wyeth over its own goods. On the contract provides for indemnity against liability to third persons, the liability of the
insurer is direct and such third persons can directly sue the insurer. The direct
other hand, what was issued by Malayan to Reputable was over the latter’s liability of the insurer under indemnity contracts against third party-liability does
insurable interest over the safety of the goods, which may become the not mean, however, that the insurer can be held solidarily liable with the insured
basis of the latter s liability in case of loss or damage to the property and and/or the other parties found at fault, since they are being held liable under
falls within the contemplation of Sec. 15, Insurance Code. different obligations. The liability of the insured carrier or vehicle owner is based on
13.   There arises no double insurance since they were issued to two different tort, in accordance with the provisions of the Civil Code; while that of the insurer arises
from contract, particularly, the insurance policy:
persons/entities having distinct insurable interests. Necessarily, over
1.   Malayan's obligation is based on the SR Policy while Reputable's is based on the contract
insurance by double insurance cannot likewise exist. of carriage.
14.   Hence, as correctly ruled by the RTC and CA, neither Sec. 5 nor Sec. 12 of
the SR Policy can be applied.
On the first issue Reputable is a private carrier.
15.   Malayan relies on the alleged judicial admission of Philippines First in its complaint that
Reputable is a common carrier. Invoking Sec. 4, Rule 129, Rules on Evidence that "an
admission verbal or written, made by a party in the course of the proceeding in the same
case, does not require proof."
16.   Judicial admissions, such as matters alleged in the pleadings do not require proof, and
need not be offered to be considered by the court however, such allegation, statement, or
admission is conclusive as against the pleader, and that the facts alleged in the
complaint are deemed admissions of the plaintiff and binding upon him.
17.   In this case, the pleader or the plaintiff who alleged that Reputable is a common carrier
was Philippines First. Thus, such cannot be conclusive as to Reputable.
18.   Records show that the alleged judicial admission of Philippines First was essentially
disputed by Reputable in its answer and claimed that latter is actually a private or special
carrier. Also, Malayan in it’s answer admitted such fact. RTC and CA findings that
Reputable is a special or private carrier is warranted by the evidence on record (e.g.
unrebutted testimony of Reputable’s Vice President and General Manager, Mr. William
Ang Lian Suan, who expressly stated in open court that Reputable serves only one
customer, Wyeth).
19.   Article 1732, Civil Code: Common carriers are persons, corporations, firms, or
associations engaged in the business of carrying or transporting passenger or goods, or
both by land, water or air for compensation, offering their services to the public. On the
other hand, a private carrier is one wherein the carriage is generally undertaken by special
agreement and it does not hold itself out to carry goods for the general public. A common
carrier becomes a private carrier when it undertakes to carry a special cargo or chartered
to a special person only. For all intents and purposes, therefore, Reputable operated as a
private/special carrier with regard to its contract of carriage with Wyeth.
On the second issue Reputable is bound by the terms of the contract of carriage.
20.   The extent of a private carrier’s obligation is dictated by the stipulations of a contract it
entered into, provided its stipulations, clauses, terms and conditions are not contrary to
law, morals, good customs, public order, or public policy. "The Civil Code provisions on
common carriers should not be applied where the carrier is not acting as such but as a
private carrier. Public policy governing common carriers has no force where the public at
large is not involved."
004 PHILAM v. AUDITOR GENERAL (Callueng) with another to protect the first insurer from a risk it has already assumed. In
January 18, 1968 | Sanchez, J. | Reinsurance contradistinction a reinsurance treaty is merely an agreement between two
insurance companies whereby one agrees to cede and the other to
PETITIONER: The Philippine American Life Insurance Company accept reinsurance business pursuant to provisions specified in the treaty.
RESPONDENTS: The Auditor General
FACTS:
SUMMARY: On January 1950, Philamlife and Airco entered into a reinsurance 1.   On January 1, 1950, Philippine American Life Insurance Company
treaty where Philamlife agreed to reinsure with Airco the excess of life insurance (Philamlife), a domestic life insurance corporation, and American
on the lives of persons written by Philamlife. In their agreement it is also International Reinsurance Company (Airco) of Pembroke, Bermuda, a
stipulated that even though Philamlife is already on a risk for its maximum corporation organized under the laws of the Republic of Panama, entered
retention under policies previously issued, when new policies are applied for and into an agreement — reinsurance treaty — which provides in its
issued they can cede automatically any amount, within the limits specified. No paragraph 1, Article I, the following:
question ever arose with respect to the remittances made by Philamlife to Airco a.   Art. I. On and after the 1st day of January 1950, the Ceding
before July 16, 1959, the date of approval of the Margin Law. Subsequently, the Company [Philamlife] agrees to reinsure with AIRCO the
Central Bank of the Philippines collected the sum of P268,747.48 as foreign entire first excess of such life insurance on the lives of persons
exchange margin on Philamlife remittances to Airco made subsequent to July 16, as may be written by the Ceding Company under direct
1959. Philamlife then filed with the Central Bank a claim for refund for the same application over and above its maximum limit of retention for life
amount arguing that the reinsurance premiums remitted were paid on January insurance, and AIRCO binds itself, subject to the terms and
1950 and is therefore exempt from the 25% foreign exchange margin fee. The provisions of this agreement, to accept such reinsurances on the
Acting legal counsel of the Monetary board resolved that reinsurance contracts same terms and for an amount not exceeding its maximum limit for
entered into and approved by the Central Bank before July 17, 1959 are exempt automatic acceptance of life reinsurance. . . .
from the payment of the 25% foreign exchange margin, even if remittances 2.   By the third paragraph of the same Article I, it is also stipulated that even
thereof are made after July 17, 1959. Still the Auditor of the CB denied though Philamlife "is already on a risk for its maximum retention
Philamlife’s claim for refund and reconsideration was denied, hence the petition. under policies previously issued, when new policies are applied for and
ISSUE: WoN Philamlife’s claim was covered by the exemption. NO. The Court issued [Philamlife] can cede automatically any amount, within the limits .
stated that for an exemption to come into play, there must be a reinsurance . . specified, on the same terms on which it would be willing to accept the
policy or, as in the reinsurance treaty provided, a "reinsurance cession" which risk for its own account, if it did not already have its limit of retention."
may be automatic or facultative. To distinguish, a reinsurance policy is a 3.   Reinsurances under said reinsurance treaty of January 1, 1950 may also be
contract of indemnity one insurer makes with another to protect the first insurer had facultatively upon other cases pursuant to Article II thereof,
from a risk it has already assumed. On the other hand, a reinsurance treaty is whereby Airco's liability begins from acceptance of the risk. These cases
merely an agreement between two insurance companies whereby one agrees to include those set forth in paragraph 2 of the treaty's Article I which
surrender and the other to accept reinsurance business pursuant to provisions expressly excludes from automatic reinsurance the following: (a) any
specified in the treaty. Treaties are contracts for insurance; reinsurance policies application for life insurance with Philamlife which, together with other
or cessions are contracts of insurance. Although the reinsurance treaty precedes papers containing information as to insurability of the risk, shows that "the
the Margin Law by over nine years nothing in that treaty obligates Philamlife to total amount of life insurance (including accidental death benefit) applied
remit to Airco a fixed, certain, and obligatory sum by way of reinsurance for to or already issued by all companies [other life insurance companies
premiums. All that the reinsurance treaty provides on this point is that Philamlife which had previously accepted the risk] exceeds the equivalent of Five
"agrees to reinsure." The treaty speaks of a probability; not a reality. Philamlife’s Hundred Thousand Dollars ($500,000) United States currency," and (b) any
obligation to remit reinsurance premiums becomes fixed and definite upon the life on which Philamlife 'retains for its own account less than its regular
execution of the reinsurance cession. Because, for every life insurance policy maximum limit of retention for the age, sex, plan, rating and occupation of
surrendered to Airco, Philamlife agrees to pay premium. It is only after a the risk.'
reinsurance cession is made that payment of reinsurance premium may be 4.   Every life insurance policy reinsured under the aforecited agreement "shall
exacted, as it is only after Philamlife seeks to remit that reinsurance premium be upon the yearly renewable term plan for the amount at risk under
that the obligation to pay the margin fee arises. the policy reinsured."
5.   Philamlife agrees to pay premiums for all reinsurances "on an annual
DOCTRINE: A reinsurance policy is a contract of indemnity one insurer makes premium basis."
6.   It is conceded that no question ever arose with respect to the remittances foreign exchange "issued, approved and outstanding as of the date this Act
made by Philamlife to Airco before July 16, 1959, the date of approval of [Republic Act 2609] takes effect."
the Margin Law. 2.   For an exemption to come into play, there must be a reinsurance
7.   The Central Bank of the Philippines collected the sum of P268,747.48 as policy or, as in the reinsurance treaty provided, a "reinsurance
foreign exchange margin on Philamlife remittances to Airco purportedly cession" which may be automatic or facultative.
totalling $610,998.63 and made subsequent to July 16, 1959. 3.   There should not be any misapprehension as to the distinction between
8.   Philamlife subsequently filed with the Central Bank a claim for the a reinsurance treaty, on the one hand, and a reinsurance policy or
refund of the above sum of P268,747.48. The ground therefor was that the a reinsurance cession, on the other. The concept of one and the other is well
reinsurance premiums so remitted were paid pursuant to the January expressed thus:
1, 1950 reinsurance treaty, and, therefore, were pre-existing obligations a.   . . . A reinsurance policy is thus a contract of indemnity one
expressly exempt from the margin fee. insurer makes with another to protect the first insurer from a
9.   The Monetary Board resolved that "reinsurance contracts entered into risk it has already assumed. . . . In contradistinction a
and approved by the Central Bank before July 17, 1959 are exempt reinsurance treaty is merely an agreement between two
from the payment of the 25% foreign exchange margin, even if insurance companies whereby one agrees to cede and the
remittances thereof are made after July 17, 1959," because such other to accept reinsurance business pursuant to provisions
remittances "are only made in the implementation of a mother specified in the treaty. The practice of issuing policies by
contract, a continuing contract, which is the reinsurance treaty." insurance companies includes, among other things, the issuance of
10.   The foregoing resolution notwithstanding, the Auditor of the Central reinsurance policies on standard risks and also on substandard risks
Bank, on April 19, 1961, refused to pass in audit Philamlife's claim for under special arrangements. The lumping of the different
refund. agreements under a contract has resulted in the term known to the
11.   Philamlife sought reconsideration with the Auditor General. insurance world as "treaties." Such a treaty is, in fact, an agreement
12.   The request for reconsideration was denied. The Auditor General in between insurance companies to cover the different situations
effect expressed the view that the existence of the reinsurance treaty of described. Reinsurance treaties and reinsurance policies are not
January 1, 1950 did not place reinsurance premia — on reinsurance synonymous. Treaties are contracts for insurance; reinsurance
effected on or after the approval of the Margin Law on July 17, 1959 — policies or cessions . . . are contracts of insurance.
out of the reach of said statute. 4.   Philamlife's obligation to remit reinsurance premiums becomes fixed
and definite upon the execution of the reinsurance cession. Because, for
ISSUE/s: every life insurance policy ceded to Airco, Philamlife agrees to pay
1.   WoN Philamlife’s claim was covered by the exemption. NO, because for premium. It is only after a reinsurance cession is made that payment of
an exemption to come into play, there must be a reinsurance policy or, reinsurance premium may be exacted, as it is only after Philamlife seeks to
as in the reinsurance treaty provided, a "reinsurance cession" which remit that reinsurance premium that the obligation to pay the margin fee
may be automatic or facultative. arises.
RULING: For the reasons given, the petition for review is hereby denied, and the 5.   Upon the premise that the margin fee of P268,747.48 was collected on
ruling of the Auditor General of October 24, 1961 denying refund is hereby affirmed. remittances made on reinsurance effected on or after the Margin Law took
effect, refund thereof does not come within the coverage of the
RATIO: exemption circumscribed in Section 3 of the said law.
1.   True, the reinsurance treaty precedes the Margin Law by over nine years. 6.   Philamlife’s point is that if the Margin Law were, applied, it "would have
Nothing in that treaty, however, obligates Philamlife to remit to Airco a paid much more to have the continuing benefit of reinsurance of its risks
fixed, certain, and obligatory sum by way of reinsurance premiums. All that than it has been required to do so by the reinsurance treaty in question" and
the reinsurance treaty provides on this point is that Philamlife "agrees to that "the theoretical equality between the contracting parties . . . would be
reinsure." The treaty speaks of a probability; not a reality. For, without disturbed and one of them placed at a distinct disadvantage in relation to the
reinsurance, no premium is due. Of course, the reinsurance treaty lays down other."
the duty to remit premiums — if any reinsurance is effected upon the 7.   This pose at once loses potency on the face of the rule long recognized that,
covenants in that treaty written. So it is that the reinsurance treaty per existing laws form part of the contract "as the measure of the
se cannot give rise to a contractual obligation calling for the payment of obligation to perform them by the one party and the right acquired by
the other." Stated otherwise, "[t]he obligation does not inhere and subsist
in the contract itself, propio vigore, but in the law applicable to the Law, we are constrained to state, is at war with the State's economic policy
contract." Indeed, Article 1315 of the Civil Code gives out the precept that of preserving the stability of our currency. Philamlife may not, in the
parties to a perfected contract "are bound . . . to all the consequences which, words of the Solicitor General, "tie the hands of the State and render it
according to their nature, may be in keeping with . . . law." powerless to impose certain margin or cost restrictions on its
8.   Accordingly, when Philamlife entered into the reinsurance treaty of remittances of reinsurance premia in foreign exchange to fall due as
January 1, 1950 with Airco, it did so with the understanding that the policies become reinsurable under said treaty, whenever such
municipal laws of the Philippines at the time said treaty was executed, remittances would constitute an excessive demand on our international
became an unwritten condition thereof. Such municipal laws constitute reserves."
part of the obligations of contract. It is in this context that we say that 12.   Viewed from this focal point, there cannot be an impairment of the
Republic Act 265, the Central Bank Act, enacted on June 15, 1948 — obligation of contracts. For, the State may, through its police power,
previous to the date of the reinsurance treaty — became a part of the adopt whatever economic policy may reasonably be deemed to promote
obligations of contract created by the latter. And under Republic Act 265, public welfare, and to enforce that policy by legislation adapted to its
reasonable restrictions may be imposed by the State through the purpose.
Central Bank on all foreign exchange transactions "in order to protect
the international reserve of the Central Bank during an exchange SEPARATE CONCURRING, FERNANDO, J.
crisis." The Margin Law is nothing more than a supplement to the 1.   This opinion deals solely with the possible unconstitutional application of
Central Bank Act; it is a reasonable restriction on transactions in Section 3 of the Law in view of the command of the non-impairment clause.
foreign exchange. It, too, is an additional arm given, the Central Bank to It is undeniable that the claim made by petitioner Philamlife as to its
attain its objectives, to wit: (1) "to maintain monetary stability in the applicability cannot be sustained. It is equally accurate to affirm that "the
Philippines;" and (2) "to preserve the international value of the peso and the State may, through its police power, adopt whatever economic policy may
convertibility of the peso into other freely convertible currencies." On top of reasonably be deemed to promote public welfare, and to enforce that policy
all these is that that statute was enacted in a background of "dangerously by legislation adapted to its purpose." In that sense necessarily, the
low international reserves." guarantee against non-impairment as the majority opinions so aptly state
9.   That some such law as Republic Act 2609 was envisioned by the "does not bar a proper exercise of the police power."
contracting parties, Philamlife and Airco, when the January 1, 1950 2.   Such a statement provokes further thought. It cannot be said without
reinsurance treaty was executed, may be gleaned from the provisions of rendering nugatory the constitutional guarantee of non-impairment, and for
Article VI of said treaty whereunder "[e]xcept in those instances where that matter both the equal protection and due process clauses which equally
AIRCO is taxed directly and independently on premiums collected by it serve to protect property rights, that at the mere invocation of the police
from the Ceding Company, AIRCO shall reimburse the Ceding Company power, the objection on non-impairment grounds automatically loses force.
for the tax paid on reinsurance premiums paid AIRCO by the Ceding Here, as in other cases where governmental authority may trench upon
Company which are not allowed the Ceding Company, as a deduction in the property rights, the process of balancing, adjustment or harmonization is
statement of the Ceding Company." called for.
10.   Philamlife complains that reinsurance contracts abroad would be made 3.   It is not then the formulation of the applicable constitutional principle
impractical by the imposition of the 25% margin fee. Reasons there are which, as above stated, has been set forth with clarity and accuracy that
which should deter us from giving in to this view. First, there is no concrete invites further scrutiny. It is rather the process by which the disposition of a
evidence that such imposition of the 25% margin fee is unreasonable. controversy whenever the protection of the contract clause is sought that, to
Second, if really continuance of the existing reinsurance treaty becomes my mind, needs additional emphasis.
unbearable that contract itself provides that Philamlife may potestatively
write finis thereto on ninety days' written notice. In truth, Philamlife is not
forced to continue its reinsurance treaty indefinitely with Airco.
11.   The Margin Law, Republic Act 2609, it is well to remember, is a
remedial currency measure. It was thus passed to reduce as far as is
practicable the excessive demand for foreign exchange. Philamlife's
stand that because it had a continuing — though revocable — reinsurance
treaty with Airco, all remittances of reinsurance premia made by it to its
foreign reinsurer should be withdrawn from the operation of the Margin
005 - PHILAMGEN V CA & FELMAN (CANDELARIA) be transported from Zamboanga to Cebu to consignee Coca-Cola PH, Cebu
June 11, 1997 | Bellosillo, J. | Topic (Coca-Cola Cebu). The shipment was insured with Philamgen under a
Marine Open Policy.
PETITIONER: PHILIPPINE AMERICAN GENERAL INSURANCE COMPANY 2.   MV Asilda left Zamboanga in fine weather, but it sank in Zamboanga del
RESPONDENTS: COURT OF APPEALS AND FELMAN SHIPPING LINES Norte, bringing down her entire cargo.
3.   Coca-Cola Cebu filed a claim with Felman for recovery of damages for the
SUMMARY: Coca-Cola loaded on board MV Asilda, a vessel owned and operated loss of the soft drink bottles. Filamgen denied prompting Coca-Cola
by respondent Felman 7,500 cases of 1L Coca-Cola soft drink bottles to be Cebu to file an insurance claim with Philamgen which paid its claim of
transported from Zamboanga to Cebu to Coca-Cola Cebu. The shipment was insured Php 750K.
with Philamgen. MV Asilda left Zamboanga in fine weather, but sank by Zamboanga 4.   Under its right of subrogation, Philamgen claimed against Felman which
del Norte, bringing down her entire cargo. Philamgen was subrogated to Coca-Cola disclaimed liability. Philamgen sued the ship owner for sum of money
Cebu’s rights after payment to Coca-Cola due to the damages. Philamgen claimed and damages. Philamgen alleged that the sinking and loss of MV Asilda
from Felman. and its cargo were due to the unseaworthiness as she was put to sea in an
unstable condition, and that it was improperly manned and that its
The trial court ruled in favor of Felman, stating that it was seaworthy, and that the officers were grossly negligent in failing to take appropriate measures to
damage was due to fortuitous event. In addition, Coca-Cola breached its implied proceed a nearby port or beach after the vessel started to list.
warranty on the vessel’s seaworthiness. The CA ruled that the MV Asilda was 5.   Felman filed a Motion to Dismiss alleging there was no right of
unseaworthy, but retained the TC ruling that Coca-Cola breached its implied subrogation in favor of Philamgen, and that Felman had abandoned its
warranty. rights and interests over MV Asilda together with her freight and
appurtenances for the purpose of limiting and extinguishing its liability
The issue in this case is whether or not Philamgen was properly subrogated to the under Article 587 of the Code of Commerce.
rights of Felman, in light of the breach of warranty.
6.   TC: ruled in favor of Felman. MV Asilda was seaworthy when it left the
The Court granted the petition, stating that Felman was equally negligent because port of Zamboanga. The loss of the vessel and its entire shipment could
the unseaworthiness arose from the point when the ship was on deck. As the shipper, only be attributed to either a fortuitous event, in which case, no liability
Felman was negligent in allowing the excessive amount of cargo. The Court also should attach unless there was a stipulation to the contrary, or to the
ruled that there was a breach of warranty of seaworthiness that removed the negligence of the captain and his crew, in which case, Art. 587 of the Code
entitlement of Coca-Cola from payment of its claim in the policy. When Philamgen of Commerce should apply.The lower court further ruled that assuming MV
paid the claim, there was in effect a voluntary payment and no right of subrogation Asilda was unseaworthy, still PHILAMGEN could not recover from
accrued in its favor. Philamgen paid it at its own risk. It is generally held that in FELMAN since Coca-Cola has breached its implied warranty on the
every marine insurance policy, the assured impliedly warrants to the assurer that vessels seaworthiness. The payment made by Philamgen to the assured was
the vessel is seaworthy and such warranty is as much a term of the contract as if undue, wrong and mistaken payment. . Since it was not legally owing, it did
expressly written on the face of the policy. However, since under Article 2207 of not give PHILAMGEN the right of subrogation so as to permit it to bring an
the Civil Code gives by equitable assignment to the insurer the right to claim against action in court as a subrogee.
a third-party offender, precluded only by a contract of insurance, Philamgen is 7.   CA: MV Asilda was unseaworthy, and that 2,500 cases of Coca-Cola
entitled to claim from Felman. softdrink bottles were improperly stowed on deck. While the vessel
possessed the necessary Coast Guard certification indicating its
DOCTRINE: It is generally held that in every marine insurance policy, the assured seaworthiness with respect to the structure of the ship itself, it was not
impliedly warrants to the assurer that the vessel is seaworthy and such seaworthy with respect to the cargo. However, the CA still denied
warranty is as much a term of the contract as if expressly written on the face of Philamgen’s claim because Coca-Cola’s implied warranty was not complied
the policy. with.

ISSUE/s: W/N Philamgen was properly subrogated to the rights of Felman


FACTS:
1.   1983: Coca-Cola Bottlers Philippines, Inc. (Coca-Cola), loaded on RULING: SC affirmed the lower courts decision. Pwede rin wherefore.
board MV Asilda, a vessel owned and operated by respondent Felman
Shipping Lines (Felman), 7,500 cases of 1L Coca-Cola soft drink bottles to
RATIO: seaworthiness cannot be raised by the assurer without showing concealment
Preliminary Issue #1: Unseaworthiness of the vessel or misrepresentation by the assured.
81.   The vessel MV Asilda was unseaworthy for being top heavy. 90.   The marine policy issued by Philamgen to Coca-Cola had disposed of usual
1.   While the vessel may not have been overloaded, yet the distribution or stowage of warranty of worthiness. The seaworthiness of the vessel as between the
the cargo on board was done in such a manner that the vessel was in top-heavy
Coca-Cola and Philamgen is admitted.
condition at the time of her departure and which condition rendered her unstable
and unseaworthy for that particular voyage. The vessel was designed for fishing, 91.   The result of the admission of seaworthiness by the assurer PHILAMGEN
and was not designed to carry a substantial amount or quantity of cargo on deck. If may mean one or two things: (a) that the warranty of the seaworthiness is to
the cargo had been confined to those that could have been accommodated under be taken as fulfilled; or, (b) that the risk of unseaworthiness is assumed by
deck, her stability would not have been affected, and would not have been in danger the insurance company.
of capsizing.
2.   From the moment that the vessel was utilized to load heavy cargo on its deck, the 92.   The insertion of such waiver clauses in cargo policies is in recognition of
vessel was rendered unseaworthy for the purpose of carrying the type of cargo the realistic fact that cargo owners cannot control the state of the
because the weight of the deck cargo so decreased the vessels metacentric height as vessel. Thus it can be said that with such categorical waiver, PHILAMGEN
to cause it to become unstable.
has accepted the risk of unseaworthiness so that if the ship should sink by
82.   Approximately 2,500 cases of softdrink bottles were stowed on unseaworthiness, as what occurred in this case, PHILAMGEN is liable.
deck. Several days after MV Asilda sank, an estimated 2,500 empty Coca-
Cola plastic cases were recovered near the vicinity of the
sinking. Considering that the ships hatches were properly secured, the
empty Coca-Cola cases recovered could have come only from the vessels
deck cargo.
83.   Carrying a deck cargo raises the presumption of unseaworthiness unless it
can be shown that the deck cargo will not interfere with the proper
management of the ship. It was established that MV Asilda was not designed
to carry substantial amount of cargo on deck.
Preliminary Issue #2: Art. 587 of Code of Commerce not applicable
84.   The ship agent is liable for the negligent acts of the captain in the care of
goods loaded on the vessel. This liability however can be limited through abandonment of the
vessel, its equipment and freightage as provided in Art. 587
85.   The right of abandonment of vessels, as a legal limitation of a shipowners liability, does not
apply to cases where the injury or average was occasioned by the shipowners own fault. Art.
587 applies only to situations where the fault or negligence is committed solely by the
captain. Where the shipowner is likewise to be blamed, Art. 587 will not apply, and such
situation will be covered by the provisions of the Civil Code on common carrier.
86.   In addition, Felman was equally negligent because the unseaworthiness arose from the point
when the ship was on deck. As the shipper, Felman was negligent in allowing the excessive
amount of cargo.
SUBROGATION ISSUE:
87.   Due to the unseaworthiness of the vessel, the Court ruled that there was a
breach of warranty of seaworthiness that removed the entitlement of Coca-
Cola from payment of its claim in the policy. When Philamgen paid the
claim, there was in effect a voluntary payment and no right of subrogation
accrued in its favor. Philamgen paid it at its own risk.
88.   It is generally held that in every marine insurance policy, the assured
impliedly warrants to the assurer that the vessel is seaworthy and such
warranty is as much a term of the contract as if expressly written on
the face of the policy.
89.   In policies where the law will generally imply a warranty of seaworthiness,
it can be excluded by terms in writing in the policy in the clearest language.
And where the policy stipulated that the seaworthiness of the vessel
between the assured and the assurer is admitted, the question of
006 MALAYAN INSURANCE v. CA and TKC MARKETING(Castillo) M/LP 97800305 amounting to P18,986,902.45 and M/LP 97800306
March 20, 1997 | Romero, J. | Perils of the sea amounting to P1,195,005.45, both dated September 1989.
3.   While the vessel was docked in Durban, South Africa on September 11,
1989 enroute to Manila, the civil authorities arrested and detained it because
PETITIONER: Malayan Insurance Corporation
of a lawsuit on a question of ownership and possession.
RESPONDENTS: CA, TKC Marketing Corporations
4.   As a result, private respondent TKC notified petitioner on October 4, 1989
of the arrest of the vessel and made a formal claim for the amount of
SUMMARY: Respondent TKC was the owner/consignee of some metric tons US$916,886.66, representing the dollar equivalent on the policies, for non-
of soya bean meal which was loaded on board the ship MV Al Kaziema. Said delivery of the cargo. Private respondent likewise sought the assistance of
cargo was insured against the risk of loss by petitioner Malayan Insurance petitioner on what to do with the cargo.
Corporation for which it issued two (2) Marine Cargo Policies. While vessel 5.   Petitioner Malayan replied that the arrest of the vessel by civil authority was
was docked in South Africa, the civil authorities arrested and detained it
not a peril covered by the policies. Private respondent TKC advised
because of a lawsuit on a question of ownership and possession. TKC notified
petitioner Malayan that it might tranship the cargo and requested an
Malayan and made a claim. Malayan replied that the arrest of the vessel by civil
extension of the insurance coverage until actual transhipment, which
authority was not a peril covered by the policies. Private respondent TKC
extension was approved upon payment of additional premium.
advised petitioner Malayan that it might tranship the cargo and requested an
6.   The insurance coverage was extended under the same terms and conditions
extension of the insurance coverage until actual transhipment, which extension
embodied in the original policies while in the process of making
was approved upon payment of additional premium. However, cargo was sold
arrangements for the transhipment of the cargo from Durban to Manila,
in South Africa because of its perishable nature, it cannot withstand voyage of covering the period October 4-December 19, 1989.
20 days to Manila and another 20 for discharge. TKC reduced its claim, 7.   However, on December 11, 1989, the cargo was sold in Durban, South
deducting the proceeds from the sale. Malayan maintains its position that the Africa, for US $154.40 per metric ton or a total of P10,304,231.75 due to its
arrest of the vessel by civil authorities on a question of ownership was an perishable nature which could no longer stand a voyage of twenty days to
excepted risk under the marine insurance policies. Hence, TKC filed for
Manila and another twenty days for the discharge thereof.
damages. Lower court ruled in favor of TKC. CA. affirmed. Issue is WoN the
8.   On January 5, 1990, private respondent TKC forthwith reduced its claim to
arrest of the vessel was a risk covered under the subject insurance policies. The
US$448,806.09 (or its peso equivalent of P9,879,928.89 at the exchange
Court ruled in the affirmative saying that the “arrest" in the instant case is
rate of P22.0138 per $1.00) representing private respondent’s loss after the
covered under “perils” under the marine policy, and not just arrest caused solely
proceeds of the sale were deducted from the original claim of $916,886.66
by executive or political acts. The Court held that with the deletion of Clause 12
or P20,184,159.55.
in the F.C. & S Clause, the Institute War Clauses was deemed incorporated in
9.   Petitioner Malayan maintained its position that the arrest of the vessel by
subsection 1of Section 1. With the incorporation of subsection 1.1 of Section civil authorities on a question of ownership was an excepted risk under the
1 of the Institute War Clauses, however, this Court agrees with the Court of marine insurance policies.
Appeals and the private respondent that “arrest” caused by ordinary 10.   This prompted private respondent TKC to file a complaint for damages
judicial process is deemed included among the covered risks. praying that aside from its claim, it be reimbursed the amount of
P128,770.88 as legal expenses and the interest it paid for the loan it
DOCTRINE: By way of a historical background, marine insurance developed
obtained to finance the shipment totalling P942,269.30. Private respondent
as an all-risk coverage, using the phrase “perils of the sea” to encompass the
TKC also asked for moral damages, exemplary damages, and attorney’s
wide and varied range of risks that were covered. The subject policies contain
fees.
the “Perils” clause which is a standard form in any marine insurance policy.
11.   Lower court decided in favor of private respondent TKC, and required
Malayan to pay insurance claim, and additional consequential and
liquidated damages, exemplary damages, reimbursement, attorney’s fees,
FACTS: and cost of the suit. Plus interest of 12%.
1.   Private respondent TKC Marketing Corp. was the owner/consignee of some 12.   CA affirmed the decision of lower court stating that with the deletion of
3,189.171 metric tons of soya bean meal which was loaded on board the Clause 12 of the policies issued to private respondent, the same became
ship MV Al Kaziemah on or about September 8, 1989 for carriage from the automatically covered under subsection 1.1 of Section 1 of the Institute War
port of Rio del Grande, Brazil, to the port of Manila. Clauses.
2.   Said cargo was insured against the risk of loss by petitioner Malayan a.   The arrests, restraints or detainments contemplated in the former
Insurance Corporation for which it issued two (2) Marine Cargo Policy Nos.
clause were those effected by political or executive acts. Losses
occasioned by riot or ordinary judicial processes were not covered Arrests included in “perils” covered by policy
therein. In other words, arrest, restraint or detainment within the
meaning of Clause 12 (or F.C. & S. Clause) rules out detention by 1.   Petitioner Malayan submits the following: (a) an arrest by civil authority is
ordinary legal processes. not compensable since the term “arrest” refers to “political or executive
b.   Hence, arrests by civil authorities, such as what happened in the acts” and does not include a loss caused by riot or by ordinary judicial
instant case, is an excepted risk under Clause 12 of the Institute process as in this case; (b) the deletion of the Free from Capture or Seizure
Cargo Clause or the F.C. & S. Clause. Clause would leave the assured covered solely for the perils specified by the
c.   However, with the deletion of Clause 12 of the Institute Cargo wording of the policy itself; (c) the rationale for the exclusion of an arrest
Clause and the consequent adoption or institution of the Institute pursuant to judicial authorities is to eliminate collusion between
War Clauses (Cargo), the arrest and seizure by judicial processes unscrupulous assured and civil authorities.
which were excluded under the former policy became one of the 2.   The resolution of this controversy hinges on the interpretation of the
covered risks. “Perils” clause of the subject policies in relation to the excluded risks or
13.   The appellate court added that the failure to deliver the consigned goods in warranty specifically stated therein.
the port of destination is a loss compensable, not only under the Institute 3.   By way of a historical background, marine insurance developed as an all-
War Clause but also under the Theft, Pilferage, and Non-delivery Clause risk coverage, using the phrase “perils of the sea” to encompass the wide
(TNPD) of the insurance policies, as read in relation to Section 130 of the and varied range of risks that were covered. The subject policies contain the
Insurance Code and as held in Williams v. Cole. “Perils” clause which is a standard form in any marine insurance policy.
14.   The appellate court contended that since the vessel was prevented at an Said clause reads:
intermediate port from completing the voyage due to its seizure by civil a.   “Touching the adventures which the said MALAYAN
authorities, a peril insured against, the liability of petitioner continued until INSURANCE CO., are content to bear, and to take upon them in
the goods could have been transhipped. this voyage; they are of the Seas; Men-of-War, Fire, Enemies,
a.   But due to the perishable nature of the goods, it had to be promptly Pirates, Rovers, Thieves, Jettisons, Letters of Mart and Counter
sold to minimize loss. Accordingly, the sale of the goods being Mart, Suprisals, Takings of the Sea, Arrests, Restraints and
reasonable and justified, it should not operate to discharge Detainments of all Kings, Princess and Peoples, of what Nation,
petitioner from its contractual liability. condition, or quality soever, Barratry of the Master and Mariners,
and of all other Perils, Losses, and Misfortunes, that have come to
ISSUES: hurt, detriment, or damage of the said goods and merchandise or
1.   WoN the arrest of the vessel was a risk covered under the subject insurance any part thereof. And in case of any loss or misfortune it shall be
policies – YES, the “arrest" in the instant case is covered under “perils” lawful to the ASSURED, their factors, servants and assigns, to sue,
under the marine policy, and not just arrest caused solely by executive or labour, and travel for, in and about the defence, safeguards, and
political acts. By way of a historical background, marine insurance recovery of the said goods and merchandises, and ship, & c., or
developed as an all-risk coverage, using the phrase “perils of the sea” to any part thereof, without prejudice to this INSURANCE; to the
encompass the wide and varied range of risks that were covered. The charges whereof the said COMPANY, will contribute according to
subject policies contain the “Perils” clause which is a standard form in any the rate and quantity of the sum herein INSURED. And it is
marine insurance policy. [MAIN ISSUE] expressly declared and agreed that no acts of the Insurer or Insured
2.   WoN there was undue reliance on the doctrine that insurance policies are in recovering, saving, or preserving the Property insured shall be
construed strictly against insurer - NO, any construction of a marine policy considered as a Waiver, or Acceptance of Abandonment…”
rendering it void should be avoided. Such policies will, therefore, be 4.   The exception or limitation to the “Perils” clause and the “All other perils”
construed strictly against the company in order to avoid a forfeiture, unless clause in the subject policies is specifically referred to as Clause 12 called
no other result is possible from the language used. the “Free from Capture & Seizure Clause” or the F.C. & S. Clause
which reads, thus:
RULING: WHEREFORE, the petition for review is DENIED and the decision of a.   “Warranted free of capture, seizure, arrest, restraint or detainment,
the Court of Appeals is AFFIRMED. and the consequences thereof or of any attempt thereat; also from
the consequences of hostilities and warlike operations, whether
RATIO: there be a declaration of war or not; but this warranty shall not
exclude collision, contact with any fixed or floating object (other Clauses.”
than a mine or torpedo), stranding, heavy weather or fire unless 13.   This Court agrees with the Court of Appeals when it held that “x x x.
caused directly (and independently of the nature of the voyage or Although the F.C. & S. Clause may have originally been inserted in marine
service which the vessel concerned or, in the case of a collision, policies to protect against risks of war, its interpretation in recent years to
any other vessel involved therein is performing) by a hostile act by include seizure or detention by civil authorities seems consistent with the
or against a belligerent power and for the purpose of this warranty general purposes of the clause, x x x.”
‘power’ includes any authorities maintaining naval, military or air 14.   In this regard, since what was also excluded in the deleted F.C. & S.
forces in association with power. Clause was “arrest” occasioned by ordinary judicial process, logically,
b.   Further warranted free from the consequences of civil war, such “arrest” would now become a covered risk under subsection 1.1 of
revolution, insurrection, or civil strike arising therefrom or piracy. Section 1 of the Institute War Clauses, regardless of whether or not
c.   Should Clause 12 be deleted, the relevant current institute war said “arrest” by civil authorities occurred in a state of war.
clauses shall be deemed to form part of this insurance.”
5.   However, the F.C. & S. Clause was deleted from the policies. Insurance contracts are strictly construed against the company
Consequently, the Institute War Clauses (Cargo) was deemed incorporated
in subsection 1.1 of Section 1. 15.   Respondent TKC argued that Malayan, being the sole author of the policies,
6.   According to petitioner Malayan, the automatic incorporation of subsection “arrests” should be strictly interpreted against it because the rule is that any
1.1 of Section 1 of the Institute War Clauses (Cargo), among others, means ambiguity is to be taken contra proferentum.
that any “capture, arrest, detention, etc.” pertained exclusively to warlike a.   Risk policies should be construed reasonably and in a manner as to
operations if this Court strictly construes the heading of the said Clauses. make effective the intentions and expectations of the parties.
7.   It further claims that on the strength of jurisprudence on the matter, the b.   It added that the policies clearly stipulate that they cover the risks
term “arrests” would only cover those arising from political or of non-delivery of an entire package and that it was petitioner itself
executive acts, concluding that whether private respondent’s claim is that invited and granted the extensions and collected premiums
anchored on subsection 1.1 of Section 1 of the Institute War Clauses thereon.
(Cargo) or the F.C. & S. Clause, the arrest of the vessel by judicial 16.   It must be borne in mind that such contracts are invariably prepared by the
authorities is an excluded risk. companies and must be accepted by the insured in the form in which they
8.   This Court cannot agree with petitioner’s assertions, particularly when it are written.
alleges that in the “Perils” Clause, it assumed the risk of arrest caused solely 17.   Any construction of a marine policy rendering it void should be avoided.
by executive or political acts of the government of the seizing state and Such policies will, therefore, be construed strictly against the company in
thereby excludes “arrests” caused by ordinary legal processes, such as in the order to avoid a forfeiture, unless no other result is possible from the
instant case. language used.
9.   With the incorporation of subsection 1.1 of Section 1 of the Institute War 18.   Exceptions to the general coverage are construed most strongly against the
Clauses, however, this Court agrees with the Court of Appeals and the company.
private respondent that “arrest” caused by ordinary judicial process is 19.   A contract of insurance, being a contract of adhesion, par excellence, any
deemed included among the covered risks. ambiguity therein should be resolved against the insurer; in other words, it
10.   This interpretation becomes inevitable when subsection 1.1 of Section 1 of should be construed liberally in favor of the insured and strictly against the
the Institute War Clauses provided that “this insurance covers the risks insurer.
excluded from the Standard Form of English Marine Policy by the
clause ‘Warranted free of capture, seizure, arrest, etc. x x x’ ” or the
F.C. & S. Clause. Subsection 1.1 of Section 1 of Institute of War Clauses:
11.   Jurisprudentially, “arrests” caused by ordinary judicial process is also a
risk excluded from the Standard Form of English Marine Policy by the “1. This insurance covers:
F.C. & S. Clause.
12.   Petitioner cannot adopt the argument that the “arrest” caused by ordinary 1.1. The risks excluded from the standard form of English Marine Policy by the
judicial process is not included in the covered risk simply because the F.C. clause warranted free of capture, seizure, arrest, restraint or detainment, and the
& S. Clause under the Institute War Clauses can only be operative in case of consequences thereof of hostilities or warlike operations, whether there be a
hostilities or warlike operations on account of its heading “Institute War declaration of war or not; but this warranty shall not exclude collision, contact with
any fixed or floating object (other than a mine or torpedo), stranding, heavy weather
or fire unless caused directly (and independently of the nature on voyage or service
which the vessel concerned or, in the case of a collision any other vessel involved
therein is performing) by a hostile act by or against a belligerent power; and for the
purpose of this warranty ‘power’ includes any authority maintaining naval, military
or air forces in association with a power. Further warranted free from the
consequences of civil war, revolution, rebellion, insurrection, or civil strike arising
therefrom, or piracy.”
007 CATHAY INSURANCE CO. v. CA (Surname) Cathay is unmistakable.
June 30, 1987 | Paras, J. | Perils of the Sea/Ship b.   Alleged contractual limitations contained in insurance policies are
regarded with extreme caution by courts and are to be strictly
PETITIONER: CATHAY INSURANCE CO. construed against the insurer; obscure phrases and exceptions
RESPONDENTS: HON. COURT OF APPEALS AND REMINGTON should not be allowed to defeat the very purpose for which the
INDUSTRIAL SALES CORPORATION policy was procured.
c.   Rust is not an inherent vice of the seamless steel pipes without
SUMMARY: Remington filed a claim against Cathay Insurance amounting to interference of external factors.
868,339 representing Remington’s losses and damages incurred in a shipment of d.   No matter how Cathay might want it otherwise, the 15-day clause
seamless steel pipes under an insurance contract in favor of it as the insured, of the policy had been foreclosed in the pre-trial order and it was
consignee or importer of the aforesaid merchandise while in transit from Japan not even raised in petitioner's answer to private respondent's
to Philippines on board vessel SS Eadtern Mariner. It was said that during the complaint.
voyage, there some rusting of the steel pipes on board. Cathay disclaims liability e.   The decision was correct in not holding that the heavy rusting of
on the ground that the rusting of the steel pipe is not considered “perils of the the seamless steel pipes did not occur during the voyage of 7 days
sea” The issue is WoN the rusting of the steel pipe is a risk covered under the from July 1 to July 7, 1981.
phrase “peril of the sea? The supreme court rule in the affirmative because f.   The alleged lack of supposed bad order survey from the arrastre
rusting of the steel occurs in the course of the voyage in view of the toll on the capitalized on by Cathay was more than clarified by no less than 2
cargo of wind, water and salt conditions witnesses.
g.   The placing of notation "rusty" in the way bills is not only private
DOCTRINE: respondent's right but a natural and spontaneous reaction of
At any rate if the insurer cannot be held accountable therefor, the court would whoever received the seamless steel pipes in a rusty condition at
fail to observe a cardinal rule in the interpretation of contracts, namely, that any private respondent's bodega.
ambiguity therein should be construed against the maker/issuer/drafter thereof, h.   The Court of Appeals did not engage in any guesswork or
namely, the insurer. speculation in concluding a loss allowance of 30% in the amount
of P868,339.15.
i.   The rate of 34% per annum double the ceiling prescribed by the
FACTS: Monetary Board is the rate of interest fixed by the Insurance Policy
78.   This case was originated from a complaint filed by Remington corporation itself and the Insurance Code.
against Cathay insurance seeking collection off the sum of 868,339 82.   While Cathay in its petition maintains that:
representing Remington’s losses and damages incurred in a shipment of a.   Remington does not dispute the fact that, contrary to the finding of
seamless steel pipes under an insurance contract in favor of it as the insured, the respondent Court (Cathay has failed "to present any evidence
consignee or importer of the aforesaid merchandise while in transit from of any viable exeption to the application of the policy") there is in
Japan to Philippines on board vessel SS Eadtern Mariner. The total value of fact an express exeption to the application of the policy.
the shipment was 2.8 million at the prevailing rate of 7.95 pesos to a dollar b.   As adverted to in the Petition for Review, Remington has admitted
in the June and July of 1984 when the shipment was made that the question shipment in not covered by a " square provision
79.   RTC Decision: In favor of Remington Corporation ordering Cathay of the contract," but Remington claims implied coverage from
insurance to pay it the sum of P866,339.15 as its recoverable insured loss the phrase " perils of the sea" mentioned in the opening
equivalent to 30% of the value of the seamless steel pipes; ordering Cathay sentenced of the policy.
to pay Remington interest on the aforecited amount at the rate of 34% or c.   The insistence of Remington that rusting is a peril of the sea is
double the ceiling prescribed by the Monetary Board per annum from erroneous.
February 3, 1982 or 90 days from Remington’s submission of proof of loss d.   Remington inaccurately invokes the rule of strict construction
to Cathay until paid as provided in the settlement of claim provision of the against insurer under the guise of construction in order to impart a
policy; and ordering Cathay to pay Remington certain amounts for marine non-existing ambiguity or doubt into the policy so as to resolve it
surveyor's fee, attorney's fees and costs of the suit. against the insurer.
80.   Cathay insurance brought the case before the Supreme Court e.   Remington while impliedly admitting that a loss occasioned by an
81.   Remington in its comment to the petition contends that: inherent defect or vice in the insured article is not within the terms
a.   Coverage of Remington’s loss under the insurance policy issued by
of the policy, erroneously insists that rusting is not an inherent vice
or in the nature of steel pipes.
f.   Rusting is not a risk insured against, since a risk to be insured
against should be a casualty or some casualty, something which
could not be foreseen as one of the necessary incidents of
adventure.
g.   A fact capable of unquestionable demonstration or of public
knowledge needs no evidence. This fact of unquestionable
demonstration or of public knowledge is that heavy rusting of steel
or iron pipes cannot occur within a period of a seven (7) day
voyage. Besides, Cathay had introduced the clear cargo receipts or
tally sheets indicating that there was no damage on the steel pipes
during the voyage.
h.   The evidence of Remington betrays the fact that the account of
P868,339.15 awarded by the respondent Court is founded on
speculation, surmises or conjectures and the amount of less has not
been proven by competent, satisfactory and clear evidence.

ISSUE/s:
14.   WoN the rusting of the steel pipe is a risk covered under the phrase “peril of
the sea”– YES, because rusting of the steel occurs in the course of the
voyage in view of the toll on the cargo of wind, water and salt conditions

RULING: WHEREFORE, this petition is hereby DENIED, and the assailed


decision of the Court of Appeals is hereby AFFIRMED.

RATIO:
76.   There is no question that the rusting of steel pipes in the course of a
voyage is a "peril of the sea" in view of the toll on the cargo of wind,
water, and salt conditions.
77.   At any rate if the insurer cannot be held accountable therefor, the court
would fail to observe a cardinal rule in the interpretation of contracts,
namely, that any ambiguity therein should be construed against the
maker/issuer/drafter thereof, namely, the insurer.
78.   Besides the precise purpose of insuring cargo during a voyage would be
rendered fruitless. Be it noted that any attack of the 15-day clause in the
policy was foreclosed right in the pre-trial conference.
79.   Finally, it is a cardinal rule that save for certain exceptions, findings of facts
of the appellate tribunal are binding on the SC. Not one of said exceptions
can apply to this case.
008 Filipino Merchants Insurance Co v. CA (COSCOLLUELA) 76.   The goods described as 600 metric tons of fishmeal in new gunny bags of
November 28, 1989 | Regalado, J. | Insurable Interest (Marine Insurance) 90 kilos each from Bangkok, Thailand were insured by Filipino Merchants
Insurance Co. (Filipino Merchants) against all risks under warehouse to
PETITIONER: Filipino Merchants Insurance Co., Inc. warehouse terms.
RESPONDENTS: Court of Appeals, Choa Tiek Seng 77.   The fishmeal in 666 new gunny bags were unloaded unto Razon and the
surveyor of Filipino Merchant ascertained and certified that in such
discharge 105 bags were in bad order condition as jointy surveyed by
SUMMARY: Seng was the consignee of a shipment of fishmeal from Bangkok. Reunis’ agent.
Upon arrival at the Port of Manila, it was certified by the arrastre operator Razon, 78.   The condition of the bad order was reflected in the turn over survey report.
Filipino Merchants’ surveyor and Reunis that the same was in bad condition. Seng The cargo was again surveyed by Razon before delivery to Send and such
brought a formal claim with the insurance company Filipino Marchants, but it condition was reflected in a certificate.
refused to pay. Thus, Seng brought suit against the Filipino Merchants. The trial 79.   Filipino Merchants’ surveyor conducted a final and detailed survey of the
court and CA ruled that it was liable to Seng. Filipino Merchants, however, argued cargo in the warehouse for which a survey report was prepared with the
that Seng had no insurable interest in the cargo, thus the marine insurance policy is findings on the extent of shortage or loss on the bad order bags totaling 227
null and void. bags.
80.   Seng made a formal claim against Filipino Merchants for P51,568.62. A
The issue is WoN Seng has insurable interest in the goods. formal claim statement was also presented by Seng against Reunis, but
Filipino Merchants refused to pay the claim. Consequently, Seng brought an
YES, as consignee of the goods in transit under an invoice containing the terms action against Filipino Merchants, and then Filipino Merchants presented a
under “C & F Manila”, Seng has an insurable interest in the goods. His interest over third-party complaint against Reunis and Razon.
the goods is based on the perfected contract of sale. The perfected contract of sale 81.   The trial cour ruled in favor of Seng. On the third-party complaint, Reunis
between him and the shipper of the goods operates to vest in him an equitable title and Razon were ordered to pay Filipino Merchant jointly and severally of
even before delivery or before he performed the conditions of the sale. The contract the amounts paid.
of shipment, whether under F.O.B., C.I.F., or C. & F. as in this case, is immaterial in 82.   On appeal, the CA affirmed the decision of the trial court. It modified the
the determination of whether the vendee has an insurable interest or not in the goods same with regard to the adjudication of the third-party complaint, absolving
in transit. The perfected contract of sale even without delivery vests in the vendee an Reunis.
equitable title, an existing interest over the goods sufficient to be the subject of 83.   Hence this petition.
insurance. The delivery of the goods on board the carrying vessels partake of the 84.   Filipino Merchants argue that the CA erred:
nature of actual delivery since, from that time, the foreign buyers assumed the risks 1.   In its interpretation and application of the all risks clause of the
of loss of the goods and paid the insurance premium covering them. C & F contracts maritime insurance policy when it helf that Filipino Merchant was
are shipment contracts. The term means that the price fixed includes in a lump sum liable to Seng for the partial loss of the cargo, notwithstanding the
the cost of the goods and freight to the named destination. It simply means that the clear absence of proof of some fortuitous event, casualty, or
seller must pay the costs and freight necessary to bring the goods to the named accidental cause to which the loss is attributable
destination but the risk of loss or damage to the goods is transferred from the seller 2.   In not holding that Seng had no insurable interest in the
to the buyer when the goods pass the ship’s rail in the port of shipment. subject cargo, hence, the marine insurance policy taken out is
null and void
DOCTRINE: In principle, anyone has an insurable interest in property who derives 3.   In not holding that Send was guilty of fraud in not disclosing
a benefit from its existence or would suffer loss from its destruction whether he has the fact, it being bound out of utmost good faith to do so, that it
or has not any title in, or lien upon or possession of the property. had no insurable interest in the subject cargo, which bars its
recovery on the policy
FACTS:
ISSUE/s:
75.   Choa Tiek Seng (Seng) was the consignee of a shipment of fishmeal loaded
WoN Seng has insurable interest in the goods – YES, As consignee of the goods in
on board the vessel SS Bougainville owned by Compagnie Maritime Des
transit under an invoice containing the terms under “C & F Manila”, Seng has an
Chargeurs Reunis (Reunis). The same was unloaded at the Port of Manila
insurable interest in the goods.
unto the arrastre contractor E. Razon, Inc. (Razon)
RULING: WHEREFORE, the instant petition is DENIED and the assailed decision
of the respondent Court of Appeals is AFFIRMED in toto. SO ORDERED.

RATIO:
93.   As consignee of the goods in transit under an invoice containing the terms
under “C & F Manila”, Seng has an insurable interest in the goods.
94.   Sec. 13 of the Insurance Code defines insurable interest in property as every
interest in property, whether real or personal, or any relation thereto, or
liability in respect thereof, of such nature that a contemplated peril might
directly damnify the insured.
95.   In principle, anyone has an insurable interest in property who derives a
benefit from its existence or would suffer loss from its destruction whether
he has or has not any title in, or lien upon or possession of the property.
96.   Seng, as vendee/consignee of the goods in transit has such existing interest
therein as may be the subject of a valid contract of insurance.
97.   His interest over the goods is based on the perfected contract of sale. The
perfected contract of sale between him and the shipper of the goods operates
to vest in him an equitable title even before delivery or before he performed
the conditions of the sale.
98.   The contract of shipment, whether under F.O.B., C.I.F., or C. & F. as in this
case, is immaterial in the determination of whether the vendee has an
insurable interest or not in the goods in transit. The perfected contract of
sale even without delivery vests in the vendee an equitable title, an existing
interest over the goods sufficient to be the subject of insurance.
99.   Further, Article 1523 of the Civil Code provides that where, in pursuance of
a contract of sale, the seller is authorized or required to send the goods to
the buyer, delivery of the goods to a carrier, whether named by the buyer or
not, for, the purpose of transmission to the buyer is deemed to be a delivery
of the goods to the buyer, the exceptions to said rule not obtaining in the
present case.
100.  The delivery of the goods on board the carrying vessels partake of the
nature of actual delivery since, from that time, the foreign buyers assumed
the risks of loss of the goods and paid the insurance premium covering
them.
101.  C & F contracts are shipment contracts. The term means that the price fixed
includes in a lump sum the cost of the goods and freight to the named
destination. It simply means that the seller must pay the costs and freight
necessary to bring the goods to the named destination but the risk of loss or
damage to the goods is transferred from the seller to the buyer when the
goods pass the ship’s rail in the port of shipment
102.  Moreover, the issue of lack of insurable interest was not among the defenses
averred in Filipino Merchants’ answer. It is a settled rule that an issue
which has not been raised in the court a quo cannot be raised for the first
time on appeal as it would be offensive to the basic rules of fair play, justice
and due process.
009 DELSAN TRANSPORT v. CA (CASTRO edited by CRUZ)  
15 November 2001 | De Leon, J. | Implied Warranties FACTS:
83.   Caltex entered into a contract of affreightment (legal term used in shipping)
PETITIONER: DELSAN TRANSPORT LINES, INC with Delsan Transport for a period of one year whereby the latter agreed to
RESPONDENTS: THE HON. COURT OF APPEALS and AMERICAN transport Caltex’s indusrial fuel oil from Batangas-Bataan refinery to other
HOME ASSURANCE CORPORATION parts of the country
84.   Under the said contract, Delsan took on board its vessel, MT Maysun the
SUMMARY: industrial fuel oil of Caltex to be delivered to Caltex Oil Terminal in
Common carrier company: Delsan Transport Zamboanga
Vessel proper: MT Maysun 85.   The shipment was insured with American Home Assurance
Shipper: Caltex 86.   MT Maysun set sail from Batangas for Zamboanga. Unfortunately, the
Insurance company: American Home Assurance vessel sank near Panay Gulf in the Visayas taking with it the entire cargo of
fuel il
Caltex entered into a shipping contract with Delsan Transport. The latter took on 87.   Subsequently, American Home Assurance paid Caltex, the sum of 5,096,
board in its vessel (MT Maysun) the industial fuel oil of Caltex from the 635.57 representing the insured value of the lost cargo
Batangas-Bataan Refinery to Zamboanga. The said cargo is insured to American 88.   Exercising its rght of subrogation under Art. 220746 of the New Civil Code,
Home Assurance. Unfortunately, MT Maysun sank and American Home paid Ameican Home demanded from Delsan the same amount it paid to Caltex
Caltex the amount of the lost cargo. After the said payment, on the basis of 89.   Due to its failure to collect from Delsan despite prior demand, American
Subrogation under Art. 2207, American Home asked Delsan to pay them for the Home filed a complaint before the RTC of Makati for collection of sum of
amount of the insured cargo. However, Delsan refused to pay. American Home money.
sued Delsan for collection of sum of money. RTC dismissed the complaint. CA 90.   RTC dismissed the complaint. The trial court found that the vessel, was
reversed the decision and ordered Delsan to pay American Home. seaworthy to undertake the voyage as determined by the PH Coast Guard
per survey certificate upon inspection during its annual dry-docking and
Delsan argues that American Home’s payment of insurance policy is an implied that the incident was caused by unexpected inclement weather condition or
force majeure, thus exempting Delsan from liability for the loss of its cargo
admission of MT Maysun’s seaworthiness. The issue is WoN said payment of
91.   The decision of the trial court, however, was reversed, on appeal, by the
insurance policy is an implied admission of MT Maysun’s seaworthiness Court of Appeals. The appellate court gave credence to the weather report
and therefore not liable for payment. The SC ruled on the negative saying that issued by the Philippine Atmospheric, Geophysical and Astronomical
though payment made by American Home for the insured value of the lost cargo Services Administration (PAGASA for brevity) which showed that from
operates as waiver of its right to enforce the term of implied warranty against 2:00 oclock to 8:00 oclock in the morning on August 16, 1986, the wind
Caltex under the marine insurance, this cannot be validly interpreted as an speed remained at 10 to 20 knots per hour while the waves measured
automatic admission of the sea vessels seaworthiness as to foreclose recourse for from .7 to two (2) meters in height only in the vicinity of the Panay Gulf
where the subject vessel sank, in contrast to herein Delsan’s allegation that
any obligation as a common carrier. The fact of payment by American Home
the waves were twenty (20) feet high.
grants it the subrogatory right which enables it to exercise legal remedies that 92.   PETITIONER DELSAN’S ARGUMENT: It invokes the provision of
would be available to Caltex as owner of the lost cargo against the common
carrier.                                                                                                                        
46
  Art.   2207.   If   the   plaintiffs   property   has   been   insured,   and   he   has  
received   indemnity   from   the   insurance   company   for   the   injury   or   loss  
DOCTRINE:
Section 113 of the Insurance Code of the Philippines states that in every marine
arising   out   of   the   wrong   or   breach   of   contract   complained   of,   the  
insurance upon a ship or freight, or freightage, or upon any thing which is the insurance   company   shall   be   subrogated   to   the   rights   of   the   insured  
subject of marine insurance there is an implied warranty by the shipper that the against  the  wrongdoer  or  the  person  who  has  violated  the  contract.  If  
ship is seaworthy. Consequently, the insurer will not be liable to the assured for the  amount  paid  by  the  insurance  company  does  not  fully  cover  the  
any loss under the policy in case the vessel would later on be found as not
seaworthy at the inception of the insurance. injury   or   loss,   the   aggrieved   party   shall   be   entitled   to   recover   the  
deficiency  from  the  person  causing  the  loss  or  injury.  
Section 113 of the Insurance Code of the Philippines, which states that in cargo operates as waiver of its right to enforce the term of implied warranty
every marine insurance upon a ship or freight, or freightage, or upon against Caltex under the marine insurance, this cannot be validly
any thing which is the subject of marine insurance there is an implied interpreted as an automatic admission of the sea vessels seaworthiness
warranty by the shipper that the ship is seaworthy. Consequently, the as to foreclose recourse for any obligation as a common carrier
insurer will not be liable to the assured for any loss under the policy in 81.   The fact of payment by American Home grants it the subrogatory right
case the vessel would later on be found as not seaworthy at the which enables it to exercise legal remedies that would be available to
inception of the insurance. Caltex as owner of the lost cargo against the common carrier.
a.   It theorized that when American Home paid Caltex the value of its 82.   The right of subrogation has its roots in equity. It is designed to promote
lost cargo, its act is equivalent to a tacit recognition that the ill- and to accomplish justice and is the mode which equity adopts to compel
fated vessel was seaworthy; otherwise, American Home was not the ultimate payment of a debt by one who in justice and good conscience
legally liable to Caltex due to the latters breach of implied ought to pay. It is not dependent upon, nor does it grow out of, any privity
warranty under the marine insurance policy that the vessel was of contract or upon written assignment of claim. It accrues simply upon
seaworthy. payment by the insurance company of the insurance claim.
93.   It also alleges that the Court of Appeals erred in ruling that MT Maysun 83.   From the nature of their business and for reasons of public policy, common
was not seaworthy on the ground that the marine officer who served as the carriers are bound to observe extraordinary diligence in the vigilance over
chief mate of the vessel, Francisco Berina, was allegedly not qualified. the goods and for the safety of passengers transported by them, according to
Under Section 116 of the Insurance Code of the Philippines, the implied all the circumstances of each case.
warranty of seaworthiness of the vessel, which the private respondent 84.   In the event of loss, destruction or deterioration of the insured goods,
admitted as having been fulfilled by its payment of the insurance common carriers shall be responsible unless the same is brought about,
proceeds to Caltex of its lost cargo, extends to the vessels complement. among others, by flood, storm, earthquake, lightning or other natural
Besides, petitioner avers that although Berina had merely a 2nd officers disaster or calamity.
license, he was qualified to act as the vessels chief officer 85.   In order to escape liability for the loss of its cargo of industrial fuel oil
belonging to Caltex, Delsan attributes the sinking of MT Maysun to
fortuitous event or force majeure. From the testimonies of Jaime Jarabe and
ISSUE/s: Francisco Berina, captain and chief mate, respectively of the ill-fated vessel,
15.   WoN the payment of American Home to Calex for the insured value of the it appears that a sudden and unexpected change of weather condition
lost cargo amounted to an admission that the vessel was seaworthy – No, occurred in the early morning of August 16, 1986; that at around 3:15
there is no provision in the contract of insurance that said payment is an oclock in the morning a squall (unos) carrying strong winds with an
admission that such vessel is seaworthy besides it would run contrary to the approximate velocity of 30 knots per hour and big waves averaging
nature of an insurance policy itself eighteen (18) to twenty (20) feet high, repeatedly buffeted MT Maysun
16.   WoN the non-presentation of the arine insurance policy bars the complaint causing it to tilt, take in water and eventually sink with its cargo
for recovery of sum of lack of money for lack of cause of action – No, mere 86.   This tale of strong winds and big waves by the said officers of the petitioner
failure to present its insurance policy is not a ground to dismiss the case for however, was effectively rebutted and belied by the weather report[15] from
lack of cause of action the Philippine Atmospheric, Geophysical and Astronomical Services
Administration (PAGASA), the independent government agency charged
RULING: WHEREFORE, the instant petition is DENIED. The Decision dated with monitoring weather and sea conditions, showing that from 2:00 oclock
June 17, 1996 of the Court of Appeals in CA-G.R. CV No. 39836 is AFFIRMED. to 8:00 oclock in the morning on August 16, 1986, the wind speed remained
Costs against the petitioner. at ten (10) to twenty (20) knots per hour while the height of the waves
ranged from .7 to two (2) meters in the vicinity of Cuyo East Pass and
Panay Gulf where the subject vessel sank. Thus, as the appellate court
correctly ruled, petitioners vessel, MT Maysun, sank with its entire cargo
SO ORDERED for the reason that it was not seaworthy. There was no squall or bad weather
or extremely poor sea condition in the vicinity when the said vessel sank.
RATIO: 87.   The appellate court also correctly opined that the Delsan’s witnesses, Jaime
NOT AN ADMISSION OF VESSEL’S SEAWORTHINESS Jarabe and Francisco Berina, ship captain and chief mate, respectively, of
80.   Though payment made by American Home for the insured value of the lost the said vessel, could not be expected to testify against the interest of their
employer, the herein common carrier.
88.   Neither may Delsan escape liability on the basis of certificates at the time of
dry-docking and inspection, because at that time the ship may have
appeared fit, however it does not negate the presumption of
unseaworthiness triggered by an unexplained sinking.
89.   Additionally, the exoneration of MT Maysuns officers and crew by the
Board of Marine Inquiry merely concerns their respective administrative
liabilities. It does not in any way operate to absolve the petitioner common
carrier from its civil liability arising from its failure to observe
extraordinary diligence in the vigilance over the goods it was transporting
and for the negligent acts or omissions of its employees, the determination
of which properly belongs to the courts.

NON-PRESENTATION OF INSURANCE POLICY


90.   Presentation in evidence of the marine insurance policy is not indispensable
in this case before the insurer may recover from the common carrier the
insured value of the lost cargo in the exercise of its subrogatory right. The
subrogation receipt, by itself, is sufficient to establish not only the
relationship of herein American Home as insurer and Caltex, as the
assured shipper of the lost cargo of industrial fuel oil, but also the
amount paid to settle the insurance claim. The right of subrogation
accrues simply upon payment by the insurance company of the insurance
claim
010 Oriental Assurance Corp. v. CA (Daguman) Voyage or Period of Insurance:
From Palawan-ETD January 16, 1986
August 9, 1991 | Melencio-Herrera, J. | Loss To: Manila

PETITIONER: Oriental Assurance Corporation Subject matter Insured:


2,000 cubic meters apitong Logs
RESPONDENTS: Court of Appeals and Panama Saw Mill Co. Agreed Value
Amount Insured Hereunder:
SUMMARY: Panama Sawmill Co bought apitong logs to be shipped from Pesos: One Million Only (P1,000,000.00)
Palawan to Manila vila Transpacific Towage. These logs were insured against Philippine Currency
Premium — P2,500.00 rate — 0.250%
loss with Oriental Assurance corporation. The logs were loaded on two barges. Doc. stamps 187.60 Invoice No. 157862
On the way to Manila, one of the barges was damages causing the loss of 497 l % P/tax 25.00
out of the 598 logs loaded on it. When Panama demanded payment for the loss, TOTAL P2,712.50
Oriental refused to pay since not all the shipment were lost (as stipulated in their CLAUSES, ENDORSEMENTS, SPECIAL CONDITIONS and WARRANTIES
Warranted that this Insurance is against TOTAL LOSS ONLY. Subject to the following clauses:
insurance contract). Both the RTC and CA ruled in favor of Panama saying that — Civil Code Article 1250 Waiver clause
the insurance contract should be liberally construed in order to avoid a denial of — Typhoon warranty clause
substantial justice and that the logs loaded in the two barges should be treated — Omnibus clause.
separately. Issue WoN Oriental Assurance can be held liable under its marine
insurance policy based on the theory of a divisible contract of insurance, and 96.   The logs were loaded on two (2) barges: (1) on barge PCT-7000,610 pieces
consequently, a constructive loss? – NO, because the fact that the logs were of logs with a volume of 1,000 cubicmeters; and (2) on Barge TPAC-1000,
loaded on two different barges did not make the contract several and divisible as 598 pieces of logs, also with a volume of 1,000 cubic meters. On 28
to the items insured. The logs on the two barges were not separately valued or January 1986, the two barges were towed by one tug-boat, the MT
separately insured. Only one premium was paid for the entire shipment, making 'Seminole' But, as fate would have it, during the voyage, rough seas and
for only one cause or consideration. The insurance contract must, therefore, be strong winds caused damage to Barge TPAC-1000 resulting in the loss
considered indivisible. of 497 pieces of logs out of the 598 pieces loaded thereon.
97.   Panama demanded payment for the loss but Oriental Assurance refuse on
DOCTRINE: The terms of the contract constitute the measure of the insurer the ground that its contracted liability was for "TOTAL LOSS ONLY." The
liability and compliance therewith is a condition precedent to the insured's right rejection was upon the recommendation of the Tan Gatue Adjustment
to recovery from the insurer. Whether a contract is entire or severable is a Company
question of intention to be determined by the language employed by the parties 98.   Unable to convince Oriental Assurance to pay its claim, Panama filed a
Complaint for Damages against Ever Insurance Agency (allegedly, also
FACTS: liable), Benito Sy Lee Yong and Oriental Assurance, before the Regional
94.   Panama Sawmill Co. (Panama) bought in Palawan, 1208 pieces of Apitong Trial Court,  Kalookan
Logs, with a volume of 2000 cubic meters. It hired Transpacific Towage, to 99.   The trial court ruled in favor of Panama.
transport the logs to Manila and insured it against loss for 1M pesos with 100.  CA affirmed the judgment.
Oriental Assurance. 101.  Both Courts shared the view that the insurance contract should be liberally
95.   There is a claim by Panama that the insurance coverage could have been for construed in order to avoid a denial of substantial justice; and that the logs
3M pesos were it not for the fraudulent act of Benito Ling to whom it had loaded in the two barges should be treated separately such that the loss
entrusted the amount of 6k for the payment of the premium for a 3M policy. sustained by the shipment in one of them may be considered as
"constructive total loss" and correspondingly compensable.
Oriental Assurance issued Marine Insurance Policy No. OACM 86/002, which stipulated, among others: 102.  In this Petition for Review on Certiorari, Oriental Assurance challenges the
aforesaid dispositions. In its Comment, Panama, in turn, maintains that the
Name of Insured: constructive total loss should be based on a policy value of P3-M and not
Panama Sawmill, Inc.
Karuhatan, Valenzuela
P1-M, and prays that the award to Ever Insurance Agency or Antonio Sy
Metro Manila Lee Yong of damages and attorney's fees be set aside.
ISSUE/s:
Vessel: 1.   WoN   Oriental   Assurance   can   be   held   liable   under   its   marine   insurance  
MT. 'Seminole' Barge PCT 7,000-1,000 cubic meter apitong Logs
Barge Transpac 1,000-1,000 cubic meter apitong Logs policy   based   on   the   theory   of   a   divisible   contract   of   insurance,   and  
consequently,  a  constructive  loss?  –  NO,  because  the fact that the logs were Thus, it concluded that the loss of 497 pieces of logs from barge TPAC-
loaded on two different barges did not make the contract several and 1000, mathematically speaking, is more than three-fourths (¾) of the 598
divisible as to the items insured. The logs on the two barges were not pieces of logs loaded in that barge and may, therefore, be considered as
separately valued or separately insured. Only one premium was paid for the constructive total loss.
entire shipment, making for only one cause or consideration. The insurance 7.   The basis thus used is, in our opinion, reversible error. The
contract must, therefore, be considered indivisible. requirements for the application of Section 139 of the Insurance Code,
quoted above, have not been met. The logs involved, although placed in
RULING: Judgment in question is dismissed. two barges, were not separately valued by the policy, nor separately
insured. Resultantly, the logs lost in barge TPAC-1000 in relation to the
RATIO: total number of logs loaded on the same barge cannot be made the basis
2.   Whether a contract is entire or severable is a question of intention to be for determining constructive total loss.
determined by the language employed by the parties. The policy in question 8.   The logs having been insured as one inseparable unit, the correct basis for
shows that the subject matter insured was the entire shipment of 2,000 cubic determining the existence of constructive total loss is the totality of the
meters of apitong logs. shipment of logs. Of the entirety of 1,208, pieces of logs, only 497 pieces
3.   The fact that the logs were loaded on two different barges did not make thereof were lost or 41.45% of the entire shipment. Since the cost of those
the contract several and divisible as to the items insured. The logs on 497 pieces does not exceed 75% of the value of all 1,208 pieces of logs, the
the two barges were not separately valued or separately insured. Only shipment cannot be said to have sustained a constructive total loss under
one premium was paid for the entire shipment, making for only one Section 139(a) of the Insurance Code.
cause or consideration. The insurance contract must, therefore, be 9.   In the absence of either actual or constructive total loss, there can be no
considered indivisible. recovery by the insured Panama against the  insurer,  Oriental  Assurance.
4.   More importantly, the insurer's liability was for "total loss only." A total
loss may be either actual or constructive (Sec. 129, Insurance Code). An  
actual total loss is caused by:

(a) A total destruction of the thing insured;


(b) The irretrievable loss of the thing by sinking, or by being broken up;
(c) Any damage to the thing which renders it valueless to the owner for the purpose
for which he held it; or
(d) Any other event which effectively deprives the owner of the possession, at the
port of destination, of the thing insured. (Section 130, Insurance Code).

5.   A constructive total loss is one which gives to a person insured a right to


abandon, under Section 139 of the Insurance Code. This provision reads:

SECTION 139. A person insured by a contract of marine insurance may abandon the
thing insured, or any particular portion thereof separately valued by the policy, or
otherwise separately insured, and recover for a total loss thereof, when the cause of
the loss is a peril injured against,

(a)   If more than three-fourths thereof in value is actually lost, or would have to
be expended to recover it from the peril; (b) If it is injured to such an extent
as to reduce its value more than three-fourths.

6.   The CA treated the loss as a constructive total loss, and for the purpose of
computing the more than three-fourths value of the logs actually lost,
considered the cargo in one barge as separate from the logs in the other.
011 KEPPEL CEBU SHIPYARD v. PIONEER INSURANCE (Dim) to abandon under Section 139.
September 25, 2009 | J. Nachura | Loss or Abandonment

PETITIONER: Keppel Cebu Shipyard Inc. FACTS:


RESPONDENT: Pioneer Insurance and Surety Corporation
103.  In 2000, Keppel Cebu Shipyard Inc. (Keppel), and WG&A Jebsens
SUMMARY: Keppel and WG&A entered into a shiprepair agreement to Shipmanagement, Inc., executed a shiprepair agreement, where Keppel
renovate the latter’s superferry using Keppel’s dry docking facilities. WG&A would renovate WG&A’s M/V Superferry 3 using its dry docking facilities.
had the ship insured with Pioneer. During the course of the repair, while the ship Prior to the execution of the shiprepair agreement, the M/V Superferry 3
was dry docked in Keppel’s shipyard, the ship was gutted by fire. WG&A was insured by WG&A with Pioneer Insurance and Surety Corporation
declared a total constructive loss and filed an insurance claim with Pioneer. (Pioneer) for $8,472,581.
104.  During the course of its repair, M/V Superferry 3, which was dry docked in
Pioneer paid the total constructive loss value and tried to collect from Keppel.
Keppel’s shipyward, was gutted by fire. WG&A declared the vessels
Keppel refused so Pioneer filed a claim with CIAC. Keppel counters that a total damage as a total constructive loss and, hence, filed an insurance claim with
constructive loss was not adequately proven. Pioneer adduced evidence the Pioneer. Pioneer paid the insurance claim of WG&A. WG&A, in turn,
estimates made by three (3) disinterested and qualified shipyards for the cost of executed a Loss and Subrogation Receipt in favor of Pioneer.
the repair of the vessel. All the estimates showed that the repair expense would 105.  Pioneer tried to collect from Keppel, but the latter denied any responsibility
exceed P270,000,000.00, the amount equivalent to of the vessel’s insured value for the loss of the subject vessel. As Keppel continuously refused to pay
of P360,000,000. Keppel denies the liability because, aside from its claim that it despite repeated demands. Pioneer filed a request for arbitration before the
Construction Industry Arbitration Commission (CIAC).
cannot be held culpable for negligence resulting in the destructive fire, there was
106.  Pioneer argues that Keppel had custody of and control over the M/V
no constructive total loss, as the amount of damage was only P170,611,260.00, Superferry 3 while said vessel was in Keppel’s premises. Sevillejo (who did
the amount of repair expense quoted by a different assessor. The issue is WoN welding works on the ship) was an employee under the control of Keppel.
there exists a total constructive loss, thus entitling Pioneer to be subrogated to 107.  Keppel argues that the proximate cause of the fire was not the negligence of
the rights of the shipowner – YES. The SC ruled Pioneer is entitled to 360 Sevillejo, but the shipowner’s own negligence in allowing Sevillejo to do
million pesos less the salvage value of the ship from Keppel. In the seeming welding work in such extremely hazardous circumstances (apparently, there
were flammable lifevests left by the vessel’s crew under the place where the
legal conflict between the insurance policy’s provisions and the Insurance Code,
welder was working).
the SC held that Section 139 of the Insurance Code should govern, because (1) CIAC’s decision
Philippine law is deemed incorporated in every locally executed contract; and (2) 108.  The CIAC held both WG&A and Keppel negligent, with the latter being
the marine insurance policies in question expressly provided that Philippine laws held to pay Pioneer 50 million pesos. Both parties appealed to the CA. The
will govern in conflicts with English law. Considering the extent of the damage, CA ultimately affirmed the decision of the CIAC. Both parties appealed to
WG&A opted to abandon the ship and claimed the value of its policies. Pioneer, the SC.
Insurance issue brought up to the SC
finding the claim compensable, paid the claim, with WG&A issuing a Loss and
109.  Pioneer claims that there existed a total constructive loss so that it had
Subrogation Receipt evidencing receipt of the payment of the insurance proceeds to pay WG&A the full amount of the insurance coverage and, by
from Pioneer. operation of law, it was entitled to be subrogated to the rights of
WG&A to claim the amount of the loss. It further argues that the
DOCTRINE: Sec. 139 - A person insured by a contract of marine insurance limitation of liability clause found in the Shiprepair Agreement is null and
may abandon the thing insured, or any particular portion hereof separately void for being iniquitous and against public policy.
valued by the policy, or otherwise separately insured, and recover for a total loss 110.  Keppel counters that a total constructive loss was not adequately
thereof.. proven by Pioneer, and that there is no proof of payment of the insurance
proceeds. KCSI insists on the validity of the limited-liability clause up to
Sec. 131. A constructive total loss is one which gives to a person insured a right P50,000,000.00, because WG&A acceded to the provision when it executed
the Shiprepair Agreement. Keppel also claims that the salvage value of the
vessel should be deducted from whatever amount it will be made to pay to
Pioneer. 93.   It appears, however, that in the execution of the insurance policies over
M/V Superferry 3, WG&A and Pioneer incorporated by reference the
ISSUES: American Institute Hull Clauses 2/6/77, the Total Loss Provision of which
(1)   [not so important] WoN negligence over the fire breaking out in the
reads:
ship can be attributed to Keppel
(2)   WoN there exists a total constructive loss, thus entitling Pioneer to be Total Loss
subrogated to the rights of the shipowner
In ascertaining whether the Vessel is a constructive Total Loss the Agreed
RULING: WHEREFORE, the Petition of Pioneer is PARTIALLY GRANTED. Value shall be taken as the repaired value and nothing in respect of the
Accordingly, KCSI is ordered to pay Pioneer the amount of P360,000,000.00 less damaged or break-up value of the Vessel or wreck shall be taken into
P30,252,648.09, equivalent to the salvage value recovered by Pioneer from M/V account.
Superferry 3, or the net total amount of P329,747,351.91, with 6% interest per
annum reckoned from the time the Request for Arbitration was filed until this There shall be no recovery for a constructive Total Loss hereunder
Decision becomes final and executory, plus 12% interest per annum on the said unless the expense of recovering and repairing the Vessel would
amount or any balance thereof from the finality of the Decision until the same will exceed the Agreed Value in policies on Hull and Machinery. In making
have been fully paid. The arbitration costs shall be borne by both parties on a pro rata this determination, only expenses incurred or to be incurred by reason of a
basis. Costs against Keppel.
single accident or a sequence of damages arising from the same accident
shall be taken into account, but expenses incurred prior to tender of
RATIO:
abandonment shall not be considered if such are to be claimed separately
[Torts Issue: Negligence]
under the Sue and Labor clause. x x x.
91.   Sevillejo, being one of the specially trained welders specifically authorized
by KCSI to do the hot works on M/V Superferry 3 to the exclusion of other 94.   In the course of the arbitration proceedings, Pioneer adduced in evidence
workers, failed to comply with the strict safety standards of KCSI, not only the estimates made by three (3) disinterested and qualified shipyards
because he worked without the required permit, fire watch, fire buckets, and for the cost of the repair of the vessel. All the estimates showed that the
extinguishers, but also because he failed to undertake other precautionary repair expense would exceed P270,000,000.00, the amount equivalent to
measures for preventing the fire. The greater the danger, the greater the of the vessels insured value of P360,000,000.00.
degree of care required. Extraordinary risk demands extraordinary care.
Similarly, the more imminent the danger, the higher degree of care 95.   Thus, WG&A opted to abandon M/V Superferry 3 and claimed from
warranted. Keppel failed to exercise the necessary degree of caution and Pioneer the full amount of the policies. Pioneer paid WG&As claim, and
foresight called for by the circumstances. now demands from KCSI the full amount of P360,000,000.00, by virtue of
subrogation.
[Total Constructive Loss] [Main Topic]
96.   Keppel denies the liability because, aside from its claim that it cannot
92.   In marine insurance, a constructive total loss occurs under any of the be held culpable for negligence resulting in the destructive fire, there
conditions set forth in Section 139 of the Insurance Code47 was no constructive total loss, as the amount of damage was only
US$3,800,000.00 or P170,611,260.00, the amount of repair expense
                                                                                                                        quoted by Simpson, Spence & Young.
47
  Sec. 139. A person insured by a contract of marine insurance may 97.   In the face of this apparent conflict, we hold that Section 139 of the
abandon the thing insured, or any particular portion hereof separately Insurance Code should govern, because (1) Philippine law is deemed
valued by the policy, or otherwise separately insured, and recover for a
total loss thereof, when the cause of the loss is a peril insured against:                                                                                                                                                                                                                                                                                        
(a) If more than three-fourths thereof in value is actually lost, or (b) If it is injured to such an extent as to reduce its value more than
would have to be expended to recover it from the peril; three-fourths; x x x.  
incorporated in every locally executed contract; and (2) the marine 102.  Likewise, Clause 20 is a stipulation that may be considered contrary to
insurance policies in question expressly provided the following: public policy. To allow KCSI to limit its liability to only P50,000,000.00,
notwithstanding the fact that there was a constructive total loss in the
a.   “This insurance is subject to English jurisdiction, except in the amount of P360,000,000.00, would sanction the exercise of a degree of
event that loss or losses are payable in the Philippines, in which diligence short of what is ordinarily required. It would not be difficult for a
case if the said laws and customs of England shall be in conflict negligent party to escape liability by the simple expedient of paying an
with the laws of the Republic of the Philippines, then the laws of amount very much lower than the actual damage or loss sustained by the
the Republic of the Philippines shall govern.” other.

98.   The CA held that Section 139 of the Insurance Code is merely permissive
on account of the word may in the provision. This is incorrect. Properly
considered, the word may in the provision is intended to grant the
insured (WG&A) the option or discretion to choose the abandonment
of the thing insured (M/V Superferry 3), or any particular portion
thereof separately valued by the policy, or otherwise separately
insured, and recover for a total loss when the cause of the loss is a peril
insured against.

99.   This option or discretion is expressed as a right in Section 131 of the same
Code, to wit:

a.   Sec. 131. A constructive total loss is one which gives to a person


insured a right to abandon under Section 139.

100.  It cannot be denied that M/V Superferry 3 suffered widespread damage
from the fire that occurred on February 8, 2000, a covered peril under the
marine insurance policies obtained by WG&A from Pioneer. These
estimates constituted credible and acceptable proof of the extent of the
damage sustained by the vessel. It is significant that these estimates were
confirmed by the Adjustment Report submitted by the average adjuster that
Pioneer had enlisted to verify and confirm the extent of the damage. The
Adjustment Report verified and confirmed that the damage to the vessel
amounted to a constructive total loss and that the claim for P360,000,000.00
under the policies was compensable

101.  Considering the extent of the damage, WG&A opted to abandon the ship
and claimed the value of its policies. Pioneer, finding the claim
compensable, paid the claim, with WG&A issuing a Loss and Subrogation
Receipt evidencing receipt of the payment of the insurance proceeds from
Pioneer.

[Limited Liability Clause] [not so important]


012 PAN MALAYAN vs. COURT OF APPEALS (Eleazar)
September 5, 1991 | Regalado, J. | Loss/Abandonment FACTS:
17.   This case had its origin in a shipment of 1,500 metric petitions of IR-36
PETITIONERS: Pan Malayan Insurance Corporation certified rice seeds which private respondent, The Food and Agricultural
RESPONDENTS: Court of Appeals and The Food and Agricultural Organization of the United Nations (FAO) to send to Kampuchea to be
Organization of the United Nations distributed to the people for seedling purposes. Respondent court, CA,
affirms the factual findings therein of the court a quo as chronologized
SUMMARY: The Food and Agricultural Organization of the United Nations hereunder.
(FAO), intended and made arrangements to send to Kampuchea 1,500 metric 18.   On May 22, 1980, FAO received a formal offer from the Luzon Stevedoring
petitions of IR-36 certified rice seeds to be distributed to the people for Corporation (LUZTEVECO) whereby the latter offered to ship the former's
seedling purposes. LUZTEVECO was to ship the cargo amounting to aforesaid cargo, consisting of 3,000 metric petitions in two lots of rice
US$83,325.92. The cargo was loaded on board LUZTEVECO Barge No. LC- seeds, to Vietnam Ocean Shipping Industry in Vaung Tau, Vietnam for
3000 and consisted of 34,122 bags of IR-36 certified rice seeds purchased by freight fees of $55.50/MT, subject to the terms and conditions indicated in
FAO from the Bureau of Plant Industry for P4,602,270.00. FAO secured the corresponding communication.
insurance coverage in the amount of P5,250,000.00 from Pan Malayan 19.   On May 28, 1980, FAO wrote LUZTEVECO formally confirming its
Insurance Corporation. On June 16, 1980, FAO gave instructions to acceptance of the foregoing offer amounting to US$83,325.92 in respect of
LUZTEVECO to leave for Vaung Tau, Vietnam to deliver the cargo which, by one lot of 1,500 metric petitions winch is the subject of the present action.
its nature, could not withstand delay because of the inherent risks of 20.   The cargo was loaded on board LUZTEVECO Barge No. LC-3000 and
termination and/or spoilage. On the same date, the insurance premiums on the consisted of 34,122 bags of IR-36 certified rice seeds purchased by FAO
shipment was paid by FAO petitioner. On June 26, 1980, FAO was advised of from the Bureau of Plant Industry for P4,602,270.00.
the sinking of the barge in the China Sea, hence it informed petitioner thereof 21.   On June 12, 1980, the loading was completed and LUZTEVECO issued its
and, later, formally filed its claim under the marine insurance policy. On July Bill of Lading No. 01 in favor of FAO. The latter then secured insurance
29, 1980, FAO was informed by LUSTEVECO of the recovery of the lost coverage in the amount of P5,250,000.00 from petitioner, Pan Malayan
shipment, for which reason FAO formally filed its claim with LUZTEVECO Insurance Corporation, as evidenced by the latter's Marine Cargo Policy
for compensation of damage to its cargo. LUZTEVECO failed and refused to No. B-11474A and Premium Invoice No. 78615, dated June 16, 1980.
pay. Pan Malayan likewise failed to pay for the losses and damages sustained 22.   On June 16, 1980, FAO gave instructions to LUZTEVECO to leave for
by FAO by reason of its inability to recover the value of the shipment from Vaung Tau, Vietnam to deliver the cargo which, by its nature, could not
LUZTEVECO. Pan Malayan claims that part of the cargo was recovered and withstand delay because of the inherent risks of termination and/or spoilage.
thus the claim by FAO was unwarranted. This is evidenced by two surveys On the same date, the insurance premiums on the shipment was paid by
upon the cargo wherein it was found that only around 78% was lost. FAO filed FAO to Pan Malayan.
a civil case against both LUZTEVECO and Pan Malayan. Trial court found in 23.   On June 23, 1980, FAO was informed by LUZTEVECO that the tugboat
favor of FAO and ordered both to pay jointly and severally the full amount of and barge carrying FAO's shipment returned to Manila after leaving on June
the claim. This was affirmed by CA. The issue is WoN respondent court 16, 1980 and that the shipment again left Manila for Vaung Tau Vietnam on
committed a reversible error in holding that the trial court is correct in holding June 21, 1980 with the barge being towed by a different tugboat. Since
that there is a total loss of the shipment. The SC said no, the law classifies loss this was an unauthorized deviation, FAO demanded an explanation on
into either total or partial. As said and proven, the seeds were of fragile nature. June 25, 1980.
And the wetting of said seeds affected the state of seeds. Thus rendering them 24.   On June 26, 1980, FAO was advised of the sinking of the barge in the
useless for FAO. Although there were bags which were recovered, these were China Sea, hence it informed Pan Malayan thereof and, later, formally
“stained” and not in the same condition it was brought in. In addition to this, filed its claim under the marine insurance policy. On July 29, 1980, FAO
FAO did not receive any compensation for said recovered bags as the same was informed by LUSTEVECO of the recovery of the lost shipment, for
were distributed by LUZVETECO without authorization of FAO. which reason FAO formally filed its claim with LUZTEVECO for
compensation of damage to its cargo.
DOCTRINE: Under Sections 129 and 130 of the New Insurance Code, a total 25.   Thereafter, despite repeated demands to replace the same or to pay for the
loss may either be actual or constructive. In case of total loss in Marine total insured value in the sum of P5,250,000.00, LUSTEVECO failed and
Insurance, the assured isentitled to recover from the underwriter the whole refused to do so. Pan Malayan likewise failed to pay for the losses and
amount of his subscription damages sustained by FAO by reason of its inability to recover the value of
the shipment from LUZTEVECO. court is correct in holding that there is a total loss of the shipment – NO, as
26.   Pan Malayan claims that on July 31, 1980 it supposedly engaged the said and proven, the seeds were of fragile nature. And the wetting of said
services of Pan Asiatic Adjustment and Marine Surveying Corporation to seeds affected the state of seeds. Thus rendering them useless for FAO.
investigate and examine the shipment. On August 4, 1980, J.A. Barroso, Jr. Although there were bags which were recovered, these were “stained” and
of said corporation reportedly conducted a survey on the shipment and not in the same condition it was brought in
found that 9,629 bags of rice seeds were in good order, 23,510 bags
sustained wattage of 10% to 15%, and 983 bags were shorthanded or RULING: WHEREFORE, the assailed judgment and resolution of respondent Court
missing. of Appeals are hereby AFFIRMED in toto.
27.   After the alleged survey, Barroso, Jr. made a report recommending to Pan
Malayan the denial of FAO's claim because the partial damage suffered by RATIO:
the shipment is not compensable under the policy. On the basis of said 12.   The law classifies loss into either total or partial. Total loss may be actual or
recommendation, Pan Malayan denied FAO's claim. absolute, or it may otherwise be constructive or technical. Pan Malayan
28.   Pan Malayan further avers that upon the request of counsel of FAO, a submits that the CA erred in ruling that there was total loss of the shipment
survey of the shipment was conducted on September 26, 27 and 29, 1980 by despite the fact that only 27,922 bags of rice seeds out of 34,122 bags were
Conrado Catalan, Jr. of Manila Adjusters & Surveyors Company and he rendered valueless to FAO and the shipment sustained only a loss of 78%.
found 6,200 bags in good order condition. At the time of his survey, 23,510 13.   FAO, however, claims that, for all intents and purposes, it has practically
bags of the shipment had allegedly already been sold by LUZTEVECO. lost its total or entire shipment in this case, inclusive of expenses, premium
29.   Pan Malayan further asserts that on September 29, 1980, FAO wrote a fees, and so forth, despite the alleged recovery by defendant LUZTEVECO.
letter to Pan Malayan signifying its willingness to abandon the proceeds 14.   As found by the court below and reproduced with approval by respondent
of the sale of the 23,510 bags and the remaining good order bags, but court, FAO "has never been compensated for this total loss or damage, a
that on October 6, 1980 Pan Malayan rejected FAO's proposed fact which is not denied nor controverted. If there were some cargoes saved,
abandonment. by LUZTEVECO, private respondent abandoned it and the same was sold
30.   FAO then instituted Civil Case No. 41716 against LUZTEVECO and/or or used for the benefit of LUZTEVECO or Pan Malayan Corporation.
Pan Malayan, as defendants, with the Regional Trial Court of Pasig, Metro Under Sections 129 and 130 of the New Insurance Code, a total loss may
Manila which, on December 14, 1987, rendered judgment in favor of FAO either be actual or constructive. In case of total loss in Marine Insurance,
with the following decretal portion: the assured is entitled to recover from the underwriter the whole amount of
a.   WHEREFORE, by virtue of preponderance of evidence and in his subscription.
consideration of justice and equity, this Court hereby renders 15.   It is a fact that on July 9, 1980, FAO formally filed its claim under the
judgment in favor of the plaintiff against the defendant Luzon marine insurance policy issued by Pan Malayan. FAO thus claims actual
Stevedoring Corporation and defendant Pan Malayan Insurance loss under paragraphs (c) and (d) of Section 130 of the Insurance Code
Corporation, ordering both the defendants, to pay jointly and which provides:
severally, the plaintiff, to wit: a.   SEC. 130. An actual total loss is caused by:
i.   The sum of P5,250,000.00 with interest thereon, at legal i.   (a) A total destruction of the thing insured;
rate from September 29, 1980 until fully paid; ii.   (b) The irretrievable loss of the thing by sinking, or by
ii.   The sum of P250,000.00 by way of attorney's fees and being broken up;
expenses of litigation; and iii.   (c) Any damage to the thing which renders it valueless to
iii.   The cost of this suit. the owner for the purpose for which he held it; or
31.   Pan Malayan alone appealed the said decision to respondent Court of iv.   (d) Any other event which effectively deprives the owner
Appeals, docketed therein as CA-G.R. CV No. 22114, and on July 20, 1990 of the possession, at the port of destination of the thing
respondent court, CA, affirmed the decision of the trial court except for the insured.
award of attorney's fees which was reduced to P25,000.00. 16.   Respondent court affirmed the ruling of the trial court to the effect that there
32.   Pan Malayan's motion for reconsideration was denied in respondent court's was indeed actual total loss, painstakingly explaining therein the following
resolution of September 3, 1990. grounds for holding petitioner liable for the entire amount of the insurance
coverage:
ISSUE/s a.   ... The lower court was not incorrect in holding that there is a total
1.   WoN respondent court committed a reversible error in holding that the trial or entire loss of shipment in the case at bar.
b.   First, the fact of the sinking of Barge LC-3000 as the occurrence of
the risk insured against under the marine insurance was proved
17.   Pan Malayan, on the other hand, claims that respondent court gravely erred
in sustaining the ruling of the trial court that there was total loss of the
shipment since from the evidence on record and the findings of respondent
court itself, only 27,922 bags of rice seeds out of 34,122 bags were rendered
valueless to FAO and the shipment sustained only a loss of 78%.
18.   Thus, Pan Malayan concludes that the findings of the court a quo, as
affirmed by the Court of Appeals, are contrary to the evidence. Upon an
examination, however, of the records presented before this Court, it is quite
clear that there was indeed actual total loss.
19.   While this Court is not a trier of facts, yet, when the findings of the Court of
Appeals are alleged to be without citation of specific evidence on which
they are based, there is sufficient reason for us to review the appellate
court's decision. Under the factual milieu of this case, we find that there is
abundant evidence to support the conclusion of respondent court.
20.   It will be recalled that said rice seeds were treated and would germinate
upon mere contact with water. The rule is that where the cargo by the
process of decomposition or other chemical agency no longer remains the
same kind of thing as before, an actual total loss has been suffered.
a.   ... However, the complete physical destruction of the subject
matter is not essential to constitute an actual total loss.
b.   Such a loss may exist where the form and specie of the thing is
destroyed, although the materials of which it consisted still exist,
as where the cargo by the process of decomposition or other
chemical agency no longer remains the same kind of thing as
before.
21.   Moreover, it is undisputed that no replacement whatsoever or any payment,
for that matter, of the value of said lost cargo was made to FAO by Pan
Malayan or LUZTEVECO. It is thus clear that FAO suffered actual total
loss under Section 130 of the Insurance Code, specifically under paragraphs
(c) and (d) thereof, recompense for which it has been denied up to the
present
22.   In view of our aforestated holding that there was actual total loss of the
goods insured in this case, it is no longer necessary to pass upon the issue of
the validity of the abandonment made by FAO.
23.   Section 135 of the Insurance Code explicitly provides that "(u)pon an actual
total loss, a person insured is entitled to payment without notice of
abandonment."
24.   This is a statutory adoption of a long standing doctrine in maritime
insurance law that in case of actual total loss, the right of the insured to
claim the whole insurance is absolute, without need of a notice of
abandonment.
013 ROQUE v. IAC (Escalona) of extraordinary nature, or arise from some overwhelming power, which cannot
November 11, 1985 | Gutierrez, J. | Loss or Abandonment be guarded against by the ordinary exertion of human skill and prudence. It is
also the general rule that everything which happens thru the inherent vice of the
PETITIONER: Isabela Roque under the name of Isabela Roque Timber thing, or by the act of the owners, master or shipper, shall not be reputed a peril,
Enterprises if not otherwise borne in the policy. It must be considered to be settled,
RESPONDENTS: Hon. Intermediate Appellate Court and Pioneer Insurance furthermore, that a loss which, in the ordinary course of events, results from the
And Surety Corporation natural and inevitable action of the sea, from the ordinary wear and tear of the
ship, or from the negligent failure of the ship's owner to provide the vessel with
SUMMARY: Manila Bay Lighterage Corporation, a common carrier, entered proper equipment to convey the cargo under ordinary conditions, is not a peril of
into a contract with Isabela Roque Timber Enterprises, to load and carry about the sea. Such a loss is rather due to what has been aptly called the "peril of the
422.18 cubic meters of logs from Malampaya Sound, Palawan to North Harbor, ship." The insurer undertakes to insure against perils of the sea and similar
Manila. Timber Enterprises insured the logs for P100,000 with Pioneer perils, not against perils of the ship. Neither barratry can be used as a ground by
Insurance and Surety Corporation. Timber Enterprises loaded 811 pieces of logs Roque. Barratry as defined in American Insurance Law is "any willful
on the barge, but never reached the destination because the barge sank. Timber misconduct on the part of master or crew in pursuance of some unlawful or
Enterprises alleged that the barge was not seaworthy and developed a leak, one fraudulent purpose without the consent of the owners, and to the prejudice of the
of the hatches were open causing water to enter, and that the barge did not have owner's interest." Barratry necessarily requires a willful and intentional act in its
sufficient tarpaulin allowing the ordinary splash of sea waves to bring in more commission. No honest error of judgment or mere negligence, unless criminally
water in the barge. Timber Enterprises demanded Manila Bay payment for the gross, can be barratry. In the case at bar, there is no finding that the loss was
loss of the shipment and unrealized profits, as well as demanding from Pioneer occasioned by the willful or fraudulent acts of the vessel's crew. There was only
the P100,000 from the insurance. Pioneer refused to pay on the ground that its simple negligence or lack of skill.
liability was only on “Total loss by the total loss of vessel only”. Hence Timber
Enterprises commenced a civil case against Manila Bay and Pioneer, rendering DOCTRINE: Since the law provides for an implied warranty of seaworthiness
judgment in favor of Timber Enterprises. Pioneer was the only one who in every contract of ordinary marine insurance, it becomes the obligation of a
appealed. The appellate court absolved the liability of Pioneer on the ground that cargo owner to look for a reliable common carrier, which keeps its vessels in
there was a breach of implied warranty of seaworthiness by Timber Enterprises seaworthy condition. The shipper of cargo may have no control over the vessel
and that the loss was due to the perils of the ship, not the perils of the sea. but he has full control in the choice of the common carrier that will transport his
goods.
The relevant issues are WoN there is a warranty of seaworthiness by the cargo
owner in cases of Marine Cargo Insurance and WoN the loss of the cargo was In marine cases, the risks insured against are "perils of the sea". The purpose of
caused by “perils of the ship” and not “perils of the sea”. such insurance is protection against contingencies and against possible damages
and such a policy does not cover a loss or injury, which must inevitably take
On the first issue, the liability of the insurance company is governed by law. place in the ordinary course of things. There is no doubt that the term 'perils of
Section 113 of the Insurance Code provides that “In every marine insurance the sea' extends only to losses caused by sea damage, or by the violence of the
upon a ship or freight, or freightage, or upon anything which is the subject of elements, and does not embrace all losses happening at sea. They insure against
marine insurance, a warranty is implied that the ship is seaworthy.” Hence, there losses from extraordinary occurrences only, such as stress of weather, winds and
can be no mistaking the fact that the term "cargo" can be the subject of marine waves, lightning, tempests, rocks and the like.
insurance and that once it is so made, the implied warranty of seaworthiness  
immediately attaches to whoever is insuring the cargo whether he be the FACTS:
shipowner or not. Moreover, the fact that the unseaworthiness of the ship was
unknown to the insured is immaterial in ordinary marine insurance and may not
16.   On February 19, 1972, the Manila Bay Lighterage Corporation (Manila
be used by him as a defense in order to recover on the marine insurance policy.
Bay), a common carrier, entered into a contract with the petitioners whereby
the former would load and carry on board its barge Mable 10 about 422.18
As to the second issue, by applying Sec. 113 of the Insurance Code, there is no
cubic meters of logs from Malampaya Sound, Palawan to North Harbor,
doubt that the term 'perils of the sea' extends only to losses caused by sea
Manila. The petitioners insured the logs against loss for P100,000.00 with
damage, or by the violence of the elements, and does not embrace all losses
respondent Pioneer Insurance and Surety Corporation (Pioneer).
happening at sea; it is said to include only such losses as are
17.   On February 29, 1972, the petitioners loaded on the barge, 811 pieces of vi.   The sum of P150,000.00 award to plaintiffs, shall bear
logs at Malampaya Sound, Palawan for carriage and delivery to North interest of six per cent (6%) from March 25, 1975, until
Harbor, Port of Manila, but the shipment never reached its destination amount is fully paid.
because Mable 10 sank with the 811 pieces of logs somewhere off Cabuli 21.   Respondent Pioneer appealed to the Intermediate Appellate Court. Manila
Point in Palawan on its way to Manila. As alleged by the petitioners in their Bay did not appeal. According to the petitioners, the transportation
complaint and as found by both the trial and appellate courts, the barge company is no longer doing business and is without funds.
where the logs were loaded was not seaworthy such that it developed a leak. 22.   During the initial stages of the hearing, Manila Bay informed the trial court
The appellate court further found that one of the hatches was left open that it had salvaged part of the logs. The court ordered them to be sold to the
causing water to enter the barge and because the barge was not provided highest bidder with the funds to be deposited in a bank in the name of Civil
with the necessary cover or tarpaulin, the ordinary splash of sea waves Case No. 86599.
brought more water inside the barge. 23.   On January 30, 1984, the appellate court modified the trial court's decision
18.   On March 8, 1972, the petitioners wrote a letter to Manila Bay demanding and absolved Pioneer from liability after finding that there was a breach of
payment of P150,000.00 for the loss of the shipment plus P100,000.00 as implied warranty of seaworthiness on the part of the petitioners and that the
unrealized profits but the latter ignored the demand. Another letter was sent loss of the insured cargo was caused by the "perils of the ship" and not by
to respondent Pioneer claiming the full amount of P100,000.00 under the the "perils of the sea". It ruled that the loss is not covered by the marine
insurance policy but respondent refused to pay on the ground that its insurance policy.
liability depended upon the "Total loss by Total Loss of Vessel only".
19.   Hence, petitioners commenced Civil Case No. 86599 against Manila Bay ISSUES:
and respondent Pioneer. 1.   WoN there is a warranty of seaworthiness by the cargo owner in cases of
20.   After hearing, the trial court found in favor of the petitioners. The Marine Cargo Insurance – YES. The fact that the term "cargo" can be the
dispositive portion of the decision reads: subject of marine insurance and that once it is so made, the implied
a.   FOR ALL THE FOREGOING, the Court hereby rendered warranty of seaworthiness immediately attaches to whoever is insuring the
judgment as follows: cargo whether he be the shipowner or not.
i.   Condemning defendants Manila Bay Lighterage 1.   WoN the loss of the cargo was caused by “perils of the ship” and not “perils
Corporation and Pioneer Insurance and Surety of the sea” – The loss was caused by the perils of the ship because there was
Corporation to pay plaintiffs, jointly and severally, the lack of evidence of extreme weather and the petitioner’s complaint included
sum of P100,000.00; allegations of negligence by the ship crew, attributing to the perils of the
ii.   Sentencing defendant Manila Bay Lighterage Corporation ship.
to pay plaintiff, in addition, the sum of P50,000.00, plus 2.   WoN the IAC erred in not ordering the return to petitioner the amount of
P12,500.00, that the latter advanced to the former as down P8,000, which was deposited in the trial court as salvage value of the logs
payment for transporting the logs in question; that were recovered – YES. The IAC should have ordered to return the
iii.   Ordering the counterclaim of defendant Insurance against salvaged value of the logs to the petitioner.
plaintiffs, dismissed, for lack of merit, but as to its cross-
claim against its co-defendant Manila Bay Lighterage RULING: WHEREFORE, the decision appealed from is AFFIRMED with the
Corporation, the latter is ordered to reimburse the former modification that the amount of P8,000.00 representing the value of the salvaged
for whatever amount it may pay the plaintiffs as such logs which was ordered to be deposited in the Manila Banking Corporation in the
surety; name of Civil Case No. 86599 is hereby awarded and ordered paid to the petitioners.
iv.   Ordering the counterclaim of defendant Lighterage The liability adjudged against Manila Bay Lighterage Corporation in the decision of
against plaintiffs, dismissed for lack of merit; the trial court is accordingly reduced by the same amount.
v.   Plaintiffs' claim of not less than P100,000.00 and
P75,000.00 as exemplary damages are ordered dismissed, RATIO:
for lack of merits; plaintiffs' claim for attorney's fees in
the sum of P10,000.00 is hereby granted, against both
103.  In their first assignment of error, the petitioners contend that the implied
defendants, who are, moreover ordered to pay the costs;
warranty of seaworthiness provided for in the Insurance Code refers only to
and
the responsibility of the shipowner who must see to it that his ship is
reasonably fit to make in safety the contemplated voyage.
104.  The Timber Enterprises states that a mere shipper of cargo, having no should not be so occasioned, and whether the fact of
control over the ship, has nothing to do with its seaworthiness. They unseaworthiness were known or unknown would be immaterial.
argue that a cargo owner has no control over the structure of the ship, 111.  Since the law provides for an implied warranty of seaworthiness in
its cables, anchors, fuel and provisions, the manner of loading his cargo every contract of ordinary marine insurance, it becomes the obligation
and the cargo of other shippers, and the hiring of a sufficient number of a cargo owner to look for a reliable common carrier, which keeps its
of competent officers and seamen. The petitioners' arguments have no vessels in seaworthy condition. The shipper of cargo may have no
merit. control over the vessel but he has full control in the choice of the
105.  There is no dispute over the liability of the common carrier Manila Bay. In common carrier that will transport his goods. Or the cargo owner may
fact, it did not bother to appeal the questioned decision. However, the enter into a contract of insurance, which specifically provides that the
petitioners state that Manila Bay has ceased operating as a firm and nothing insurer answers not only for the perils of the sea but also provides for
may be recovered from it. They are, therefore, trying to recover their losses coverage of perils of the ship.
from the insurer. 112.  We are constrained to apply Section 113 of the Insurance Code to the facts
106.  The liability of the insurance company is governed by law. Section 113 of of this case. As stated by the private respondents:
the Insurance Code provides: a.   In marine cases, the risks insured against are "perils of the
a.   In every marine insurance upon a ship or freight, or freightage, or sea" (Chute v. North River Ins. Co., Minn—214 NW 472, 55 ALR
upon any thing which is the subject of marine insurance, a 933). The purpose of such insurance is protection against
warranty is implied that the ship is seaworthy. contingencies and against possible damages and such a policy
107.  Section 99 of the same Code also provides in part. does not cover a loss or injury which must inevitably take place
a.   Marine insurance includes: in the ordinary course of things. There is no doubt that the
i.   Insurance against loss of or damage to: term 'perils of the sea' extends only to losses caused by sea
1.   Vessels, craft, aircraft, vehicles, goods, freights, damage, or by the violence of the elements, and does not
cargoes, merchandise, ... embrace all losses happening at sea. They insure against losses
b.   From the above-quoted provisions, there can be no mistaking the from extraordinary occurrences only, such as stress of weather,
fact that the term "cargo" can be the subject of marine insurance winds and waves, lightning, tempests, rocks and the like. These
and that once it is so made, the implied warranty of seaworthiness are understood to be the "perils of the sea" referred in the policy,
immediately attaches to whoever is insuring the cargo whether he and not those ordinary perils which every vessel must encounter.
be the shipowner or not. "Perils of the sea" has been said to include only such losses as are
108.  As we have ruled in the case of Go Tiaoco y Hermanos v. Union Insurance of extraordinary nature, or arise from some overwhelming power,
Society of Canton (40 Phil. 40): which cannot be guarded against by the ordinary exertion of
a.   The same conclusion must be reached if the question be discussed human skill and prudence. Damage done to a vessel by perils of
with reference to the seaworthiness of the ship. It is universally the sea includes every species of damages done to a vessel at
accepted that in every contract of insurance upon anything which is sea, as distinguished from the ordinary wear and tear of the
the subject of marine insurance, a warranty is implied that the ship voyage, and distinct from injuries suffered by the vessel in
shall be seaworthy at the time of the inception of the voyage. This consequence of her not being seaworthy at the outset of her
rule is accepted in our own Insurance Law (Act No. 2427, sec. voyage (as in this case). It is also the general rule that everything
106). ... which happens thru the inherent vice of the thing, or by the act of
109.  Moreover, the fact that the unseaworthiness of the ship was unknown to the owners, master or shipper, shall not be reputed a peril, if not
the insured is immaterial in ordinary marine insurance and may not be otherwise borne in the policy. (14 RCL on Insurance, Sec. 384, pp.
used by him as a defense in order to recover on the marine insurance 1203- 1204; Cia. de Navegacion v. Firemen's Fund Ins. Co., 277
policy. US 66, 72 L. ed. 787, 48 S. Ct. 459).
110.  As was held in Richelieu and Ontario Nav. Co. v. Boston Marine, Inc.,
Co. (136 U.S. 406): 113.  With regard to the second assignment of error, petitioners maintain, that
a.   There was no look-out, and both that and the rate of speed were the loss of the cargo was caused by the perils of the sea, not by the
contrary to the Canadian Statute. The exception of losses perils of the ship because as found by the trial court, the barge was
occasioned by unseaworthiness was in effect a warranty that a loss turned loose from the tugboat east of Cabuli Point "where it was
buffeted by storm and waves." Moreover, petitioners also maintain that
barratry, against which the cargo was also insured, existed when the insurer liable, be some casualty, something which could not be foreseen as
personnel of the tugboat and the barge committed a mistake by turning one of the necessary incidents of the adventure. The purpose of the policy is
loose the barge from the tugboat east of Cabuli Point. The trial court also to secure an indemnity against accidents which may happen, not against
found that the stranding and foundering of Mable 10 was due to improper events which must happen.
loading of the logs as well as to a leak in the barge which constituted 118.  In the present case the entrance of the sea water into the ship's hold through
negligence. the defective pipe already described was not due to any accident which
114.  On the contention of the petitioners that the trial court found that the loss happened during the voyage, but to the failure of the ship's owner properly
was occasioned by the perils of the sea characterized by the "storm and to repair a defect of the existence of which he was apprised. The loss was
waves" which buffeted the vessel, the records show that the court ruled therefore more analogous to that which directly results from simple
otherwise. It stated: unseaworthiness than to that which result from the perils of the sea.
a.   ... The other affirmative defense of defendant Lighterage, 'That the 119.  Suffice it to say that upon the authority of those cases there is no room to
supposed loss of the logs was occasioned by force majeure... "was doubt the liability of the shipowner for such a loss as occurred in this case.
not supported by the evidence. At the time Mable 10 sank, there By parity of reasoning the insurer is not liable; for generally speaking, the
was no typhoon but ordinary strong wind and waves, a condition shipowner excepts the perils of the sea from his engagement under the bill
which is natural and normal in the open sea. The evidence shows of lading, while this is the very perils against which the insurer intends to
that the sinking of Mable 10 was due to improper loading of the give protection. As applied to the present case it results that the owners of
logs on one side so that the barge was tilting on one side and for the damaged rice must look to the shipowner for redress and not to the
that it did not navigate on even keel; that it was no longer insurer.
seaworthy that was why it developed leak; that the personnel of the 120.  Neither can petitioners allege barratry on the basis of the findings
tugboat and the barge committed a mistake when it turned loose showing negligence on the part of the vessel's crew.
the barge from the tugboat east of Cabuli point where it was 121.  Barratry as defined in American Insurance Law is "any willful
buffeted by storm and waves, while the tugboat proceeded to west misconduct on the part of master or crew in pursuance of some
of Cabuli point where it was protected by the mountain side from unlawful or fraudulent purpose without the consent of the owners, and
the storm and waves coming from the east direction. ..." to the prejudice of the owner's interest." (Sec. 171, U.S. Insurance Law,
115.  In fact, in the petitioners' complaint, it is alleged that "the barge Mable quoted in Vance, Handbook on Law of Insurance, 1951, p. 929.)
10 of defendant carrier developed a leak which allowed water to come 122.  Barratry necessarily requires a willful and intentional act in its commission.
in and that one of the hatches of said barge was negligently left open by No honest error of judgment or mere negligence, unless criminally gross,
the person in charge thereof causing more water to come in and that can be barratry. (See Vance on Law of Insurance, p. 929 and cases cited
"the loss of said plaintiffs' cargo was due to the fault, negligence, therein.)
and/or lack of skill of defendant carrier and/or defendant carrier's 123.  In the case at bar, there is no finding that the loss was occasioned by the
representatives on barge Mable 10." willful or fraudulent acts of the vessel's crew. There was only simple
116.  It is quite unmistakable that the loss of the cargo was due to the perils negligence or lack of skill. Hence, the second assignment of error must
of the ship rather than the perils of the sea. The facts clearly negate the likewise be dismissed.
petitioners' claim under the insurance policy. In the case of Go Tiaoco y
Hermanos v. Union Ins. Society of Canton, supra, we had occasion to 124.  Anent the third assignment of error, we agree with the petitioners that the
elaborate on the term "perils of the ship." We ruled: amount of P8,000.00 representing the amount of the salvaged logs should
117.  It must be considered to be settled, furthermore, that a loss which, in the have been awarded to them. However, this should be deducted from the
ordinary course of events, results from the natural and inevitable amounts which have been adjudicated against Manila Bay Lighterage
action of the sea, from the ordinary wear and tear of the ship, or from Corporation by the trial court.
the negligent failure of the ship's owner to provide the vessel with
proper equipment to convey the cargo under ordinary conditions, is not
a peril of the sea. Such a loss is rather due to what has been aptly called the
"peril of the ship." The insurer undertakes to insure against perils of the sea
and similar perils, not against perils of the ship. As was well said by Lord
Herschell in Wilson, Sons & Co. v. Owners of Cargo per the
Xantho ([1887], 12 A. C., 503, 509), there must, in order to make the
014 PHILHOME ASSURANCE v. CA (Fordan) there was no showing, and none was alleged by the parties, that the fire was
June 20, 1996 | Kapunan, J. | Measure of indemnity caused by a natural disaster or calamity not attributable to human agency. On the
contrary, there is strong evidence indicating that the acetylene cylinder caught
PETITIONER: Philippine Home Assurance Corporation fire because of the fault and negligence of ESLI, its captain and its crew.
RESPONDENTS: Court of Appeals and Eastern Shipping Lines, Inc.
b. Whether or not PHAC is liable for the additional charges or expenses
SUMMARY: ESLI loaded on board SS Eastern Explorer in Kobe, Japan, the incurred by the owner of the ship in the salvage operations and in the
following shipment for carriage to Manila and Cebu, freight pre-paid and in transshipment of the goods via a different carrier. NO. As a rule, general or
good order and condition: 2 boxes internal combustion engine parts, 334 bags gross averages include all damages and expenses which are deliberately caused
ammonium chloride, 200 bags Glue 300, and garments. While the vessel was off in order to save the vessel, its cargo, or both at the same time, from a real and
Okinawa, Japan, a small flame was detected on the acetylene cylinder located in known risk. While the instant case may technically fall within the purview of the
the accommodation area near the engine room on the main deck level. As the said provision, the formalities prescribed under Arts. 813 and 814 of the Code
crew was trying to extinguish the fire, the acetylene cylinder suddenly exploded of Commerce in order to incur the expenses and cause the damage corresponding
sending a flash of flame throughout the accommodation area, thus causing death to gross average were not complied with. Consequently, ESLI’s claim for
and severe injuries to the crew and instantly setting fire to the whole contribution from the consignees of the cargo at the time of the occurrence of the
superstructure of the vessel. The incident forced the master and the crew to average turns to naught. Prescinding from the foregoing premises, it indubitably
abandon the ship. Several hours later, a tugboat under the control of Fukuda follows that the cargo consignees cannot be made liable to ESLI for additional
Salvage Co. arrived near the vessel and commenced to tow the vessel for the port freight and salvage charges. Consequently, ESLI must refund to PHAC the
of Naha, Japan. Fire fighting operations were again conducted at the said port. amount it paid under protest for additional freight and salvage charges in behalf
After the fire was extinguished, the cargoes which were saved were loaded to of the consignees.
another vessel for delivery to their original ports of destination. ESLI then
charged the consignees several amounts corresponding to additional freight and DOCTRINE: In our jurisprudence, fire may not be considered a natural disaster
salvage charges (see fact 7). The charges were all paid by PHAC under protest or calamity since it almost always arises from some act of man or by human
for and in behalf of the consignees and the former, as subrogee, filed a complaint means. It cannot be an act of God unless caused by lightning or a natural disaster
before the RTC of Manila against ESLI to recover the sum paid under protest on or casualty not attributable to human agency.
the ground that the same were actually damages directly brought about by the
fault, negligence, illegal act and/or breach of contract of ESLI. On the other As a rule, general or gross averages include all damages and expenses which are
hand, ESLI contended that it exercised the diligence required by law in the deliberately caused in order to save the vessel, its cargo, or both at the same
handling, custody and carriage of the shipment; that the fire was caused by an time, from a real and known risk. Furthermore, the formalities prescribed under
unforeseen event; that the additional freight charges are due and demandable Arts. 813 and 814 of the Code of Commerce in order to incur the expenses and
pursuant to the BOL; and that salvage charges are properly collectible under the cause the damage corresponding to gross average should be complied with.
Salvage Law. The RTC dismissed PHAC’s complaint and ruled in favor of
ESLI. This was also affirmed by the CA. Hence, the current petition. The FACTS:
following are the issues and ratio: 111.  Eastern Shipping Lines, Inc. (ESLI) loaded on board SS Eastern Explorer in
Kobe, Japan, the following shipment for carriage to Manila and Cebu,
a. Whether or not the fire on the vessel which was caused by the explosion freight pre-paid and in good order and condition:
of an acetylene cylinder loaded on the same was the fault or negligence of a.   2 boxes internal combustion engine parts consigned to William Lines,
ESLI. YES. The SC held that, in our jurisprudence, fire may not be considered a Inc. under Bill of Lading (BOL) No. 042283;
natural disaster or calamity since it almost always arises from some act of man b.   10 metric tons (334 bags) ammonium chloride consigned to Orca’s
or by human means. It cannot be an act of God unless caused by lightning or a Company under BOL No. KCE-12;
natural disaster or casualty not attributable to human agency. In the case at bar, it c.   200 bags Glue 300 consigned to Pan Oriental Match Company under
is not disputed that a small flame was detected on the acetylene cylinder and that BOL No. KCE-8; and
by reason thereof, the same exploded despite efforts to extinguish the fire. d.   garments consigned to Ding Velayo under BOL Nos. KMA-73 and
Neither is there any doubt that the acetylene cylinder, obviously fully loaded, KMA-74.
was stored in the accommodation area near the engine room and not in a storage 112.  While the vessel was off Okinawa, Japan, a small flame was detected on the
area considerably far, and in a safe distance, from the engine room. Moreover, acetylene cylinder located in the accommodation area near the engine room
on the main deck level. ship to save their lives and were saved from drowning by passing vessels
113.  As the crew was trying to extinguish the fire, the acetylene cylinder in the vicinity.
suddenly exploded sending a flash of flame throughout the accommodation c.   The burning of the vessel rendering it a constructive total loss and
area, thus causing death and severe injuries to the crew and instantly setting incapable of pursuing its voyage to the Philippines was, therefore, not
fire to the whole superstructure of the vessel. The incident forced the master the fault or negligence of defendant but a natural disaster or calamity
and the crew to abandon the ship. which nobody would like to happen.
114.  Thereafter, SS Eastern Explorer was found to be a constructive total loss d.   The salvage operations conducted by Fukuda Salvage Company was
and its voyage was declared abandoned. perfectly a legal operation and charges made on the goods recovered
115.  Several hours later, a tugboat under the control of Fukuda Salvage Co. were legitimate charges.
arrived near the vessel and commenced to tow the vessel for the port of e.   The Salvage Law48 is thus applicable to the case at bar.
Naha, Japan. f.   With respect to the additional freight charged by ESLI from the
116.  Fire fighting operations were again conducted at the said port. After the fire consignees of the goods, the same are also validly demandable as
was extinguished, the cargoes which were saved were loaded to another provided by Art. 117449 and Art. 126650 of the Civil Code.
vessel for delivery to their original ports of destination. g.   The burning of SS Eastern Explorer while off Okinawa rendered it
117.  ESLI charged the consignees several amounts corresponding to additional physically impossible for ESLI to comply with its obligation of
freight and salvage charges, as follows: delivering the goods to their port of destination pursuant to the contract
a.   P1,927.65, representing salvage charges assessed against the goods for of carriage.
the goods covered by BOL No. 042283; h.   Under Art. 1266 of the Civil Code, the physical impossibility of the
b.   P2,980.64 for additional freight and P826.14 for salvage charges against prestation extinguished ESLI’s obligation.
the goods for BOL No. KCE-12; i.   It is but legal and equitable for ESLI therefore, to demand additional
c.   P3,292.26 for additional freight and P4,130.68 for salvage charges freight from the consignees for forwarding the goods from Naha, Japan
against the goods covered by BOL No. KCE-8; and to Manila and Cebu City on board another vessel, the “EASTERN
d.   P8,337.06 for salvage charges against the goods for BOL Nos. KMA-73 MARS”. This also finds support under Art. 84451 of the Code of
and KMA-74. Commerce.
118.  The charges were all paid by Philippine Home Assurance Corporation j.   Furthermore, the terms and conditions of the Bill of Lading authorize the
(PHAC) under protest for and in behalf of the consignees. imposition of additional freight charges in case of forced interruption or
119.  PHAC, as subrogee of the consignees, thereafter filed a complaint before abandonment of the voyage.
the RTC of Manila against ESLI to recover the sum paid under protest on 122.  On appeal, the CA affirmed the findings of the RTC. Hence, the current
the ground that the same were actually damages directly brought about by petition.
the fault, negligence, illegal act and/or breach of contract of ESLI.
120.  In its answer, ESLI contended that it exercised the diligence required by law
in the handling, custody and carriage of the shipment; that the fire was                                                                                                                        
48
Section 1. When in case of shipwreck, the vessel or its cargo shall be beyond the control of
caused by an unforeseen event; that the additional freight charges are due
the crew, or shall have been abandoned by them, and picked up and conveyed to a safe place
and demandable pursuant to the Bill of Lading; and that salvage charges are by other persons, the latter shall be entitled to a reward for the salvage.
properly collectible under Act No. 2616 (Salvage Law). Those who, not being included in the above paragraph, assist in saving a vessel or its cargo
121.  The RTC dismissed PHAC’s complaint and ruled in favor of ESLI. It held from shipwreck, shall be entitled to like reward.
that the question to be resolved is whether or not the fire on the vessel 49
Article 1174. Except in cases expressly specified by law, or when it is otherwise declared by
which was caused by the explosion of an acetylene cylinder loaded on the stipulation, or when the nature of the obligation require the assumption of risk, no person shall
same was the fault or negligence of ESLI. be responsible for those events which could not be foreseen, or which though foreseen, were
a.   Based on evidence, SS Eastern Explorer was a seaworthy vessel and inevitable.
50
before the ship loaded the Acetylene Cylinder No. NCW 875, the same Article 1266. The debtor in obligations to do shall also be released when the prestation
becomes legally or physically impossible without the fault of the obligor.
has been tested, checked and examined and was certified to have 51
Article 844. A captain who may have taken on board the goods saved from the wreck shall
complied with the required safety measures and standards. continue his course to the port of destination; and on arrival should deposit the same, with
b.   When the fire was detected by the crew, fire fighting operations was judicial intervention at the disposal of their legitimate owners. xxx
immediately conducted but due to the explosion of the acetylene The owners of the cargo shall defray all the expenses of this arrival as well as the payment of
cylinder, the crew were unable to contain the fire and had to abandon the the freight which, after taking into consideration the circumstances of the case, may be fixed
by agreement or by a judicial decision.
ISSUES: cylinder caught fire because of the fault and negligence of ESLI, its captain
1.   Whether or not the fire on the vessel which was caused by the explosion and its crew.
of an acetylene cylinder loaded on the same was the fault or negligence a.   First, the acetylene cylinder which was fully loaded should not have
of ESLI. – YES, since fire is not considered a natural disaster or calamity been stored in the accommodation area near the engine room where the
and there is strong evidence indicating that the acetylene cylinder caught heat generated therefrom could cause the acetylene cylinder to explode
fire because of the fault and negligence of ESLI, its captain and its crew. by reason of spontaneous combustion. ESLI should have easily foreseen
2.   Whether or not PHAC is liable for the additional charges or expenses that the acetylene cylinder, containing highly inflammable material, was
incurred by the owner of the ship in the salvage operations and in the in real danger of exploding because it was stored in close proximity to
transshipment of the goods via a different carrier. – NO, since the engine room.
formalities prescribed under Arts. 813 and 814 of the Code of Commerce b.   Second, ESLI should have known that by storing the acetylene cylinder
in order to incur the expenses and cause the damage corresponding to gross in the accommodation area supposed to be reserved for passengers, it
average were not complied with. unnecessarily exposed its passengers to grave danger and injury. Curious
passengers, ignorant of the danger the tank might have on humans and
RULING: The judgment appealed from is hereby reversed and set aside. ESLI is property, could have handled the same or could have lighted and smoked
ordered to return to PHAC the amount it paid under protest in behalf of the cigarettes while repairing in the accommodation area.
consignees. c.   Third, the fact that the acetylene cylinder was checked, tested and
examined and subsequently certified as having complied with the safety
RATIO: measures and standards by qualified experts before it was loaded in the
First Issue: vessel only shows to a great extent that negligence was present in the
125.  It is worthy to note at the outset that the goods subject of the present handling of the acetylene cylinder after it was loaded and while it was
controversy were neither lost nor damaged in transit by the fire that razed on board the ship. Indeed, had the ESLI and its agents not been
the carrier. In fact, the said goods were all delivered to the consignees, even negligent in storing the acetylene cylinder near the engine room, then the
if the transshipment took longer than necessary. same would not have leaked and exploded during the voyage.
126.  What is at issue therefore is not whether or not the carrier is liable for the 132.  Verily, there is no merit in the finding of the RTC to which CA erroneously
loss, damage, or deterioration of the goods transported by them but who, agreed that the fire was not the fault or negligence of ESLI but a natural
among the carrier, consignee or insurer of the goods, is liable for the disaster or calamity. The records are simply wanting in this regard.
additional charges or expenses incurred by the owner of the ship in the 133.  On PHAC’s objection to the admissibility of the Statement of Facts and the
salvage operations and in the transshipment of the goods via a different Marine Note Protest issued by Capt. Tiburcio A. Licaylicay, said documents
carrier. are hearsay evidence.
127.  In absolving ESLI of any liability, CA sustained the RTC’s finding that the a.   Capt. Licaylicay, Master of SS Eastern Explorer who issued the said
fire that gutted the ship was a natural disaster or calamity. PHAC takes documents, was not presented in court to testify to the truth of the facts
exception to this conclusion and the SC agree. he stated therein; instead, ESLI presented Junpei Maeda, its Branch
128.  In our jurisprudence, fire may not be considered a natural disaster or Manager in Tokyo and Yokohama, Japan, who evidently had no
calamity since it almost always arises from some act of man or by human personal knowledge of the facts stated in the documents at issue.
means. It cannot be an act of God unless caused by lightning or a natural b.   It is clear from Sec. 36, Rule 130 of the Rules of Court that any
disaster or casualty not attributable to human agency. evidence, whether oral or documentary, is hearsay if its probative value
129.  In the case at bar, it is not disputed that a small flame was detected on the is not based on the personal knowledge of the witness but on the
acetylene cylinder and that by reason thereof, the same exploded despite knowledge of some other person not on the witness stand.
efforts to extinguish the fire. Neither is there any doubt that the acetylene c.   Consequently, hearsay evidence, whether objected to or not, has no
cylinder, obviously fully loaded, was stored in the accommodation area near probative value unless the proponent can show that the evidence falls
the engine room and not in a storage area considerably far, and in a safe within the exceptions to the hearsay evidence rule. It is excluded
distance, from the engine room. because the party against whom it is presented is deprived of his right
130.  Moreover, there was no showing, and none was alleged by the parties, that and opportunity to cross-examine the persons to whom the statements or
the fire was caused by a natural disaster or calamity not attributable to writings are attributed.
human agency.
131.  On the contrary, there is strong evidence indicating that the acetylene
Second Issue:
134.  On the issue on the expenses incurred in saving the cargo which was based
on general average.
a.   As a rule, general or gross averages include all damages and expenses
which are deliberately caused in order to save the vessel, its cargo, or
both at the same time, from a real and known risk.
b.   While the instant case may technically fall within the purview of the said
provision, the formalities prescribed under Arts. 81352 and 81453 of the
Code of Commerce in order to incur the expenses and cause the damage
corresponding to gross average were not complied with.
c.   Consequently, ESLI’s claim for contribution from the consignees of the
cargo at the time of the occurrence of the average turns to naught.
d.   Prescinding from the foregoing premises, it indubitably follows that the
cargo consignees cannot be made liable to ESLI for additional freight
and salvage charges.
e.   Consequently, ESLI must refund to PHAC the amount it paid under
protest for additional freight and salvage charges in behalf of the
consignees.

                                                                                                                       
52
ARTICLE 813. In order to incur the expenses and cause the damages corresponding to gross
average, there must be a resolution of the captain, adopted after deliberation with the sailing
mate and other officers of the vessel, and after hearing the persons interested in the cargo who
may be present.
If the latter shall object, and the captain and officers or a majority of them, or the
captain, if opposed to the majority, should consider certain measures necessary, they may be
executed under his responsibility, without prejudice to the right of the shippers to proceed
against the captain before the competent judge or court, if they can prove that he acted with
malice, lack of skill, or negligence.
If the persons interested in the cargo, being on board the vessel, have not been heard,
they shall not contribute to the gross average, their share being chargeable against the captain,
unless the urgency of the case should be such that the time necessary for previous
deliberations was wanting.
53
ARTICLE 814. The resolution adopted to cause the damages which constitute general
average must necessarily be entered in the log book, stating the motives and reasons for the
dissent, should there be any, and the irresistible and urgent causes which impelled the captain
if he acted of his own accord.
In the first case the minutes shall be signed by all the persons present who could do
so before taking action, if possible; and if not, at the first opportunity. In the second case, it
shall be signed by the captain and by the officers of the vessel.
In the minutes, and after the resolution, shall be stated in detail all the goods
jettisoned, and mention shall be made of the injuries caused to those kept on board. The
captain shall be obliged to deliver one copy of these minutes to the maritime judicial authority
of the first port he may make, within twenty-four hours after his arrival, and to ratify it
immediately under oath.
001 MALAYAN INSURANCE v. CRUZ ARNALDO (GALINDEZ) 3.   On June 7, 1981, Malayan Insurance Co (MICO) issued to Coronacion
October 12, 1987 | Cruz, J. | Measure of Indemnity Pinca (Pinca) a fire insurance policy on her property for P14,000 effective
July 22, 1981 until July 22, 1982.
4.   On October 15, 1981, MICO allegedly cancelled the policy for non-
PETITIONER: Malayan Insurance Co. Inc.
payment of the premium and sent a corresponding notice to Pinca.
RESPONDENTS: Gregoria Cruz Arnaldo, in her capacity as the Insurance
5.   On December 24, 1981, payment of the premium for Pinca was received by
Commissioner, and Coronacion Pinca
Domingo Adora (Adora), MICO’s agent.
6.   On January 15, 1981, Adora remitted this payment to MICO, with the other
SUMMARY: MICO issued to Pinca a fire insurance policy for P14,000,
payments.
effective from July 22, 1981 to July 22, 1982. MICO allegedly cancelled the
7.   On January 18, 1982, Pinca’s property was completely burned.
policy for non-payment of premium on October 15. However, Pinca’s payment
8.   On February 5, 1982, Pinca’s payment was returned by MICO to Adora on
for the premium was received by Adora, MICO’s agent on December 24.
the ground that the policy had been cancelled earlier, but Adora refused to
accept it.
On January 15, 1982, Pinca’s property was burned. On February 5 of the same
9.   Pinca made demands for payment, which MICO rejected. Hence, she went
year, her payment was returned by MICO to Adora on the ground that the policy
to the Insurance Commission. It is because Pinca was ultimately sustained
had been cancelled earlier. Pinca still demanded for payment from MICO but
by Insurance Commissioner Gregoria Cruz Arnaldo (Gregoria) which is
MICO refused hence Pinca brought the matter to the Insurance Commissioner,
why MICO comes to the Court for relief.
which sustained Pinca. Hence, MICO filed the present case before the Court.
ISSUE/s:
ISSUE RELATED TO SYLLABUS TOPIC: WoN the valuation fixed in the
2.   WoN the petition is procedurally infirm – YES. It was filed out of time,
policy is conclusive – YES. Valuation fixed in the fire insurance policy is
hence for this alone, it must be dismissed.
conclusive in case of total loss in the absence of fraud, which is not shown here.
3.   WoN there was valid notice of cancellation – NO. There is flat denial by
Pinca who says she never received the cancellation and who did not have to
(SEE DOCTRINE)
prove such denial as Sec. 64 states that no insurance policy shall be
cancelled except upon prior notice, it behooved MICO's to make sure that
The certification issued by the Integrated National Police, Lao-ang, Samar, as to
the cancellation was actually sent to and received by the insured.
the extent of Pinca's loss should be considered sufficient. Notably, MICO
4.   (RELEVANT) WoN the valuation fixed in the policy is conclusive – YES.
submitted no evidence to the contrary nor did it even question the extent of the
Valuation fixed in the fire insurance policy is conclusive in case of total loss
loss in its answer before the Insurance Commission. It is also worth observing
in the absence of fraud, which is not shown here.
that Pinca's property was not the only building bumed in the fire that razed the
commercial district of Lao-ang, Samar, on January 18, 1982.
RULING: WHEREFORE, the petition is DENIED. The decision of the Insurance
DOCTRINE: Loss and its amount may be determined on the basis of such Commission dated April 10, 1981, and its Order of June 4, 1981, are AFFIRMED in
proof as may be offered by the insured, which need not be of such full, with costs against the petitioner. This decision is immediately executory.
persuasiveness as is required in judicial proceedings. If, as in this case, the
insured files notice and preliminary proof of loss and the insurer fails to specify RATIO:
to the former all the defects thereof and without unnecessary delay, all PROCEDURAL
objections to notice and proof of loss are deemed waived under Section 90 of 7.   The Court referred to the petition as stillborn (Hofs might ask) on a
the Insurance Code. procedural viewpoint alone.
8.   The notice of Gregoria dated April 5, 1982 was received by MICO on April
 
10, 1982. On April 25, it filed an MR which was denied on June 4. Notice
When a person's house is razed, the fire usually burns down the efforts of a lifetime was received by MICO on June 13.
and forecloses hope for the suddenly somber future. The vanished abode becomes a 9.   MICO’s position is that the petition is governed by Section 416 of the
charred and painful memory. Where once stood a home, there is now, in the sighing Insurance Code giving it 30 days within which to appeal by certiorari to the
wisps of smoke, only a gray desolation. The dying embers leave ashes in the heart.
SC. Alternatively, it invokes Rule 45 of the ROC.
(Dean Hofi likes these things….) 10.   Gregoria and Pinca insist that the applicable law is BP 129, which states
that the period for appeal is 15 days, as in the ROC.
FACTS:
11.   The pivotal date is the date the notice of the denial of the MR was received 21.   The payment was made on December 24 and the fire occurred on January
by MICO. 18.
12.   MICO avers it’s June 18, but Pinca states it’s June 13, because an 22.   Suppose the payment was made an accepted in August 1981, would the
authenticated copy of the Order with a rubber-stamped notation at its commencement of the policy have been changed to the date of the payment,
bottom indicates that it was received for MICO by J. Gotladera on 6/13/82. or would the payment have retroacted to July 22, 1981? If MICO accepted
13.   Between the two dates, the Court believes the latter because it is the date the payment in December 1981 and the insured property had not been
authenticated by the administrative division of the Insurance Commission. burned, would that policy not have expired just the same on July 22, 1982,
14.   Under Section 416 of the Insurance Code, the period for appeal is thirty pursuant to its original terms, and not on December 24, 1982?
days from notice of the decision of the Insurance Commission. MICO filed 23.   In MICO’s theory, the policy would have been effective only upon payment
its MR on April 25, 1981, or fifteen days such notice, and the reglementary if accepted. Hence, it would’ve been valid only from December 24, 1981 to
period began to run again after June 13, 1981, date of its receipt of notice of July 22, 1981. In other words, the policy would have run for only eight
the denial of the said motion for reconsideration. As the herein petition was months although the premium paid was for one whole year.
filed on July 2, 1981, or nineteen days later, there is no question that it is 24.   MICO’s acknowledgment of Adora as its agent defeats its contention that
tardy by four days. he was not authorized to receive the premium payment on its behalf.
15.   Counted from June 13, the fifteen-day period prescribed under Rule 45, 25.   Under the insurance code and law on agency, payment to Adora is payment
assuming it is applicable, would end on June 28, 1982, or also four days to MICO.
from July 2, when the petition was filed. 26.   The Court moreover went on to discuss the grounds of cancellation and the
16.   If it was filed under B.P. 129, then, considering that the motion for proper method56.
reconsideration was filed on the fifteenth day after MICO received notice of 27.   A valid cancellation must therefore require the following:
the decision, only one more day would have remained for it to appeal, to a.   There must be prior notice of cancellation to the insured
wit, June 14, 1982. That would make the petition eighteen days late by July b.   The notice must be based on the occurrence, after the effective date of the policy, of
one or more of the grounds mentioned
2. c.   The notice must be (a) in writing, (b) mailed, or delivered to the name insured, (c)
17.   Even if the applicable law were still R.A. 5434, governing appeals from at the address shown in the policy
administrative bodies, the petition would still be tardy. The law provides for d.   It must state (a) which of the grounds mentioned under Sec. 64 is relied upon and
(b) upon the written request of the insured, the insurer will furnish the facts on
a fixed period of ten days from notice of the denial of a seasonable motion
which the cancellation is based.
for reconsideration within which to appeal from the decision. Accordingly, 28.   MICO’s claims that it cancelled the policy on October 15 for non-payment
that ten-day period, counted from June 13, 1982, would have ended on June of premiums. It presented one of its employees who testified that the alleged
23, 1982, making the petition filed on July 2, 1982, nine days late. cancellation was sent the assured by mail. However, there is no proof that
the notice was actually mailed to and received by Pinca.
ON THE MERITS 29.   On the other hand, there is flat denial by Pinca who says she never received
18.   MICO’s arguments that there was no payment of premium and that the the cancellation and who did not have to prove such denial as Sec. 64 states
policy had been cancelled before the occurence of the loss are not
acceptable. Its contention that the claim was allowed without proof of loss                                                                                                                        
56
is also untenable.  SEC.  64.   No   policy   of   insurance   other   than   life   shall   be   cancelled   by   the   insurer   except   upon   prior  
notice   thereof  to   the   insured,   and   no   notice   of   cancellation   shall   be   effective   unless   it   is   based   on   the  
19.   MICO relies on Section 77 of the Insurance Code54. However, the provision
occurrence,  after  the  effective  date  of  the  policy,  of  one  or  more  of  the  following:  
is not applicable because the payment of premium was made. (a)   non-­‐payment  of  premium;  
20.   The premium invoice issued to Pinca at the time of delivery of the policy on (b)   conviction  of  a  crime  arising  out  of  acts  increasing  the  hazard  insured  against;  
June 7 was stamped with “PAYMENT RECEIVED” of the amount of (c)   discovery  of  fraud  or  material  misrepresentation;  
P930.60 by Domingo Adora55. (d)   discovery  of  willful,  or  reckless  acts  or  commissions  increasing  the  hazard  insured  against;  
(e)   physical   changes   in   the   property   insured   which   result   in   the   property   becoming  
uninsurable;or  
                                                                                                                       
54 (f)   a   determination   by   the   Commissioner   that   the   continuation   of   the   policy   would   violate   or  
 SEC.  77.  An  insurer  is  entitled  to  payment  of  the  premium  as  soon  as  the  thing  is  exposed  to  the  peril  
would  place  the  insurer  in  violation  of  this  Code.  
insured   against.   Notwithstanding   any   agreement   to   the   contrary,   no   policy   or   contract   of   insurance  
SEC.  65.   All   notices   of   cancellation   mentioned   in   the   preceding   section   shall   be   in   writing,   mailed   or  
issued  by  an  insurance  company  is  valid  and  binding  unless  and  until  the  premium  thereof  has  been  paid,  
delivered   to   the   named   insured   at   the   address   shown   in   the   policy,   and   shall   state   (a)   which   of   the  
except  in  the  case  of  a  life  or  an  industrial  life  policy  whenever  the  grace  period  provision  applies.  
55 grounds   set   forth   in   section   sixty-­‐four   is   relied   upon   and   (b)   that,   upon   written   request   of   the   named  
  This   is   important   because   it   suggests   an   understanding   between   MICO   and   the   insured   that   such  
insured,  the  insurer  will  furnish  the  facts  on  which  the  cancellation  is  based.  
payment  could  be  made  later,  as  agent  Adora  had  assured  Pinca.  In  any  event,  it  is  not  denied  that  this  
payment  was  actually  made  by  Pinca  to  Adora,  who  remitted  the  same  to  MICO.    
that no insurance policy shall be cancelled except upon prior notice, it 39.   The certification issued by the Integrated National Police, Lao-ang, Samar,
behooved MICO's to make sure that the cancellation was actually sent to as to the extent of Pinca's loss should be considered sufficient.
and received by the insured. 40.   Notably, MICO submitted no evidence to the contrary nor did it even
30.   The presumption cited is unavailing against the positive duty enjoined by question the extent of the loss in its answer before the Insurance
Section 64 upon MICO and the flat denial made by Pinca that she had Commission. It is also worth observing that Pinca's property was not the
received notice of the claimed cancellation. only building bumed in the fire that razed the commercial district of Lao-
31.   If Pinca really received the notice, she wouldn’t have made payment on the ang, Samar, on January 18, 1982.
original policy on December 24. Instead, she would have asked for a new 41.   There is nothing in the Insurance Code that makes the participation of an
insurance effective on that date until one year later, to take advantage of the adjuster in the assessment of the loss imperative or indispensable, as MICO
extended period. suggests. Section 325, which it cites, simply speaks of the licensing and
32.   The Court finds that if she paid on that date, it’s because she honestly duties of adjusters.
believed the policy issued on June 7 was still in effect and she was willing
to make her payment retroact to July 22, its stipulated commencement date.    
33.   MICO suggests that Pinca knew the policy had already been cancelled and
that when she paid the premium on December 24, 1981, her purpose was "to
renew it." As this could not be done by the agent alone under the terms of
the original policy, the renewal thereof did not legally bind MICO.
34.   However, a study of the transcript57 will show that Pinca meant to renew the
policy if it had really been already cancelled but not if it was still
effective—it was conditional.
35.   Since it was not shown that there was a valid cancellation of the policy,
there was no need to renew it but to pay the premium thereon. Payment was
legally made on the original transaction and was validly received on behalf
of MICO by Adora. Adora was not informed of the cancellation either and
saw no reason not to accept the payment.

RELEVANT TO SYLLABUS TOPIC


36.   The valuation fixed in the fire insurance policy is conclusive in case of total
loss in the absence of fraud, which is not shown here.
37.   Loss and its amount may be determined on the basis of such proof as may
be offered by the insured, which need not be of such persuasiveness as is
required in judicial proceedings.
38.   If, as in this case, the insured files notice and preliminary proof of loss and
the insurer fails to specify to the former all the defects thereof and without
unnecessary delay, all objections to notice and proof of loss are deemed
waived under Section 90 of the Insurance Code.
                                                                                                                       
57
  Q:   Now,   Madam   Witness,   on   December   25th   you   made   the   alleged   payment.   Now,   my   question   is  
that,  did  it  not  come  to  your  mind  that  after  the  lapse  of  six  (6)  months,  your  policy  was  cancelled?  
A:  I  have  thought  of  that  but  the  agent  told  me  to  call  him  up  at  anytime.  
Q:  So  if  you  thought  that  your  policy  was  already  intended  to  revive  cancelled  policy?  
A:  Misleading,  Your  Honor.  
Hearing  Officer:  The  testimony  of  witness  is  that,  she  thought  of  that.  
Q:  I  will  revise  the  question.  Now,  Mrs.  Witness,  you  stated  that  you  thought  the  policy  was  cancelled.  
Now,  when  you  made  the  payment  of  December  24,  1981,  your  intention  was  to  revive  the  policy  if  it  
was  already  cancelled?  
A:  Yes,  to  renew  it.  
 
002  NEW  LIFE  ENTERPRISES  v.  CA  (Gonzales)   5.   However, the three insurance companies denied his claim for payment for
March  31,  1992  |  Regalado,  J.  |  Other  Insurance  Clause   breach of Conditions No. 358 and 27.59 Condition No. 3 requires the insured
  to give notice of any insurance or insurances already effected covering the
stocks in trade.
PETITIONERS:  New  Life  Enterprises  and  Julian  Sy   6.   New Life then filed separate civil actions against the insurance companies.
RESPONDENT:   Court   of   Appeals,   Equitable   Insurance   Corporation,   Reliance   Surety   7.   RTC ruled in favor of New Life.
and  Insurance,  Western  Guaranty  Corporation   8.   CA reversed the RTC decision
  9.   Hence this petition.
   
SUMMARY: Julian Sy, a partner in New Life, insured against fire New Life’s ISSUE:  
stocks in trade with Western, Reliance, and Equitable. The building occupied by 2.   WoN Conditions No. 3 and 27 of the insurance contracts were violated
New Life and the stocks in trade therein were gutted by fire. Sy failed a claim thereby resulting in their forfeiture of all the benefits thereunder – YES.
against the three insurance companies. The latter denied the claim for breach of New Life failed to give notice of any insurance already effected covering
Condition 3, which requires the insured to give notice of any insurance or insurances the stocks in trade.
already effected covering the stocks in trade. New Life filed a case against the three
insurance companies. RTC ruled in favor of New Life. CA reversed. The issue is RULING:  CA  Decision  affirmed.    
WoN Condition 3 was violated – YES. New Life failed to give notice of any  
insurance already effected covering the stocks in trade. RATIO:  
1.   Condition No. 3, otherwise known as the "Other Insurance Clause," is
  uniformly contained in all the-aforestated insurance contracts.
DOCTRINE: Violation of the Other Insurance Clause entitled the insurer to rescind. 2.   New Life admits that the insurance policies did not state or endorse
  thereon the other insurance coverage obtained or subsequently effected
FACTS:   on the same stocks in trade for the loss of which compensation is claimed
1.   Julian Sy and Jose Sy Bang formed a business partnership under the by New Life. (aka the insurance companies are not co-insurers)
business name of New Life Enterprises. The partnership engaged in the sale 3.   In other words, the coverage by other insurance or co-insurance effected
of construction materials at a two storey building in Iyam, Lucena City. or subsequently arranged by New Life were neither stated nor endorsed
Julian Sy insured against fire the stocks in trade of New Life with in the policies of the three insurance companies, warranting forfeiture of
a.   Western in the amount of P350,000 (May 15, 1981; renewed on all benefits thereunder if we are to follow the express stipulation in the
May 13, 1982) aforequoted Policy Condition No. 3.
b.   Reliance in the amount of P300,000 (July 30, 1981, renewed) and
P700,000 (November 12, 1981)                                                                                                                        
c.   Equitable in the amount of P200,000 (February 8, 1982)
d.   Notice that when Sy renewed the insurance with Western, the 58
"3. The insured shall give notice to the Company of any insurance or insurances already effected, or
other insurances were subsisting. That is why even though which may subsequently be effected, covering any of the property or properties consisting of stocks in
Western’s policy is the first, there is still a violation of the trade, goods in process and/or inventories only hereby insured, and unless such notice be given and the
particulars of such insurance or insurances be stated therein or endorsed on this policy pursuant to Section
condition. 50 of the Insurance Code, by or on behalf of the Company before the occurrence of any loss or damage,
2.   The building occupied by New Life was gutted by fire. all benefits under this policy shall be deemed forfeited, provided however, that this condition shall not
3.   According to the police, the cause of fire was electrical in nature. apply when the total insurance or insurances in force at the time of loss or damage is not more than
4.   After the fire, Julian Sy went to the agent of Reliance whom he asked to
accompany him to the office of the company so that he can file his claim. P200,000.00."
He submitted the fire clearance, the insurance policies and inventory of
stocks. The three insurance companies are sister companies, and when he 59
 27.  Action  or  suit  clause.  —  If  a  claim  be  made  and  rejected  and  an  action  or  suit  be  not  commenced  
was following-up his claim with Equitable, the Claims Manager told him to either   in   the   Insurance   Commission   or   any   court   of   competent   jurisdiction   of   notice   of   such   rejection,   or  
go first to Reliance and if said company agrees to pay, they would also pay. in  case  of  arbitration  taking  place  as  provided  herein,  within  twelve  (12)  months  after  due  notice  of  the  
award  made  by  the  arbitrator  or  abitrators  or  umpire,  then  the  claim  shall  for  all  purposes  be  deemed  to  
The same treatment was given him by the other insurance companies. have  been  abandoned  and  shall  not  thereafter  be  recoverable  hereunder."  
 
4.   New Life contends that they are not to be blamed for the omissions, not insured by any other policy. Violation thereof entitled the insurer to
alleging that insurance agent Leon Alvarez (for Western) and Yap Kam rescind.”
Chuan (for Reliance and Equitable) knew about the existence of the 13.   Pacific Banking Corporation vs. Court of Appeals: “had the insurer
additional insurance coverage and that they were not informed about the known that there were many co-insurances, it could have hesitated or
requirement that such other or additional insurance should be stated in plainly desisted from entering into such contract. Hence, the insured was
the policy, as they have not even read said policies. guilty of clear fraud”
5.   These contentions cannot pass judicial muster. 14.   To further warrant and justify the forfeiture of the benefits under the
6.   The insured is specifically required to disclose to the insurer any other insurance contracts involved, we need merely to turn to Policy Condition
insurance and its particulars which he may have effected on the same No. 15 thereof: "… if any false declaration be made or used in support
subject matter. thereof, … all benefits under this Policy shall be forfeited…"
7.   The knowledge of such insurance by the insurer's agents, even assuming 15.   Additionally, insofar as the liability of Reliance is concerned, the
the acquisition thereof by the former, is not the "notice" that would stop complaint for recovery was filed in court only on January 31, 1984, or
the insurers from denying the claim. Besides, the so-called theory of after more than one (1) year had elapsed from New Life’s receipt of the
imputed knowledge, that is, knowledge of the agent is knowledge of the insurers' letter of denial. Thus, the claim has prescribed.
principal, aside from being of dubious applicability here has likewise 16.   Assuming arguendo that New Life felt the legitimate need to be clarified
been roundly refuted by the CA. as to the policy condition violated, there was a considerable lapse of time
8.   Thus, it points out that while Julian Sy claimed that he had informed from their receipt of the insurer's clarificatory letter dated March 30,
agent Alvarez regarding the co-insurance on the property, he contradicted 1983, up to the time the complaint was filed in court on January 31,
himself by inexplicably claiming that he had not read the terms of the 1984. The one-year prescriptive period was yet to expire on November
policies; that Yap Dam Chuan could not likewise have obtained such 29, 1983, or about eight (8) months from the receipt of the clarificatory
knowledge for the same reason, aside from the fact that the insurance letter, but New Life let the period lapse without bringing their action in
with Western was obtained before those of Reliance and Equitable; and court. We accordingly find no "peculiar circumstances" sufficient to
that the conclusion of the trial court that Reliance and Equitable are relax the enforcement of the one-year prescriptive period and we,
"sister companies" is an unfounded conjecture drawn from the mere fact therefore, hold that New Life’s claim was definitely filed out of
that Yap Kam Chuan was an agent for both companies which also had time. LLjur
the same insurance claims adjuster. Availment of the services of the same
agents and adjusters by different companies is a common practice in the
insurance business and such facts do not warrant the speculative
conclusion of the trial court.
   
9.   When the words and language of documents are clear and plain or readily
understandable by an ordinary reader thereof, there is absolutely no room
for interpretation or construction anymore.
10.   It may be true that the majority rule is that insured persons may accept
policies without reading them, and that this is not negligence per se. But,
this is not without any exception.
11.   It is and was incumbent upon Sy to read the insurance contracts, and this
can be reasonably expected of him considering that he has been a
businessman since 1965 and the contract concerns indemnity in case of
loss in his money-making trade of which important consideration he
could not have been unaware as it was precisely the reason for his
procuring the same.
12.   Pioneer Insurance vs. Yap: “considering the terms of the policy which
required the insured to declare other insurances, the statement in question
must be deemed to be a statement (warranty) binding on both insurer and
insured, that there were no other insurance on the property… The
annotation then, must be deemed to be a warranty that the property was
003  SUN  INSURANCE  v.  CA  (TIMBOL  edited  by  GUECO)    
July  17,  1992  |  Cruz,  J.  |  Casualty  Insurance    
  FACTS:  
PETITIONER:  Sun  Insurance  Office,  LTD   1.   Sun   Insurance   issued   Personal   Accident   Policy   No.   05687   to   Felix   Lim,   Jr.  
RESPONDENTS:  CA  and  Nerissa  Lim   (“Felix”)  with  a  face  value  of  P200,000.  
  i.   Two  months  later,  he  was  dead  with  a  bullet  wound  in  his  head  
SUMMARY:   Felix   obtained   a   Personal   Accident   Policy   from   Sun   Insurance,   assigning   2.   As  beneficiary,  his  wife  Nerissa  Lim  sought  payment  on  the  policy,  but  her  
his   wife,   Nerissa,   as   his   beneficiary.   Two   months   later,   after   his   mother’s   birthday   claim  was  rejected  
party,   Felix   died   with   a   bullet   wound   in   his   head.   According   to   Nalagon,   his   i.   Sun  Insurance  agreed  that  there  was  no  suicide  
secretary,  Felix  pointed  the  gun  at  her  (Nalagon)  and  when  she  pushed  it  aside,  he   ii.   It  argued,  however  that  there  was  no  accident  either    
assured   her   that   he   had   removed   the   magazine   (ammunition   storage).   To   prove   his   3.   Pilar  Nalagon,  Lim’s  secretary,  was  the  only  eyewitness  to  his  death    
point,  he  placed  the  gun  to  his  temple  and  fired  it.  There  was  then  an  explosion  and   i.   According   to   Nalagon,   Lim   was   in   a   happy   mood   (but   not   drunk)  
Felix  fell  to  the  floor.     and  was  playing  with  his  handgun,  from  which  he  had  previously  
Nerissa   sought   to   recover   the   insurance   proceeds   from   Sun   Insurance.   However,   removed  the  magazine  
Sun   Insurance   refused,   arguing   that   when   Felix   placed   the   gun   to   his   head,   he   ii.   As   she   watched   television,   he   stood   in   front   of   her   and   pointed  
willfully   exposed   himself   to   needless   peril;   thus,   removing   himself   from   the   the  gun  at  her  
coverage  of  the  policy  (which  absolves  Sun  Insurance  from  liability  for  bodily  injury   iii.   She  pushed  it  aside  and  said  it  might  be  loaded    
caused   by   a   commission   or   an   attempt   to   commit   suicide   or   when   the   insured   iv.   He  assured  her  it  was  not  and  then  pointed  it  to  his  temple  
willfully  exposes  himself  to  needless  peril).     v.   The   next   moment   there   was   an   explosion   and   Lim   slumped   to   the  
ISSUE:   WON   the   death   of   Felix   was   an   accident—YES.   RULING:   (See   doctrine   for   floor  
definition  of  accident).  In  De  la  Cruz  v.  Capital  Insurance,  the  Court  stated  that  there   vi.   He  was  dead  before  he  fell  
is   no   accident   when   a   deliberate   act   is   performed.   This   is   unless   some   additional,   4.   The   widow   sued   Sun   Insurance   in   the   RTC   of   Zamboanga   City   and   was  
unexpected,   independent   and   unforeseen   happening   occurs   which   produces   or   sustained.  This  decision  was  affirmed  on  appeal,  and  the  MR  was  denied.  
brings   about   their   injury   or   death.   There   was   such   a   happening   in   this   case.   The   The  Sun  Insurance  then  came  to  this  SC  to  fault  the  CA  for  approving  the  
firing   of   the   gun   was   the   additional,   unexpected,   independent,   and   unforeseen   payment  of  the  claim  and  the  award  of  damages    
occurrence  that  led  to  Felix’s  death.  His  act  of  pointing  the  gun  to  his  temple  was    
NOT   deliberate.   Although   he   was   negligent,   he   did   not,   however,   willfully   expose   ISSUE/s:  
himself   to   needless   peril   because   he   believed   in   good   faith   that   it   was   safe   to   do   so   1.   WoN   the   death   of   Felix   was   an   accident—YES.   In   De   la   Cruz   v.   Capital  
(the  act  being  intended  to  assure  Nalagon  that  the  gun  was  indeed  harmless).     Insurance,   the   Court   stated   that   there   is   no   accident   when   a   deliberate   act  
  is  performed.  This  is  unless  some  additional,  unexpected,  independent  and  
DOCTRINE:  Casualty  insurance  is  insurance  covering  loss  or  liability  arising  from  an   unforeseen  happening  occurs  which  produces  or  brings  about  their  injury  
accident   or   mishap,   excluding   certain   types   of   loss   which   by   law   or   custom   are   or   death.   There   was   such   a   happening   in   this   case.   The   firing   of   the   gun  
considered  as  falling  exclusively  within  the  scope  of  other  types  of  insurance  such  as   was   the   additional,   unexpected   and   independent,   and   unforeseen  
fire  or  marine.  It  includes,  but  is  not  limited  to,  employer’s  liability  insurance,  motor   occurrence  that  led  to  Felix’s  death.  
vehicle   liability   insurance,   plate   glass   insurance,   burglary   and   theft   insurance,    
personal   accident   and   health   insurance   as   written   by   non-­‐life   insurance   companies,   RULING:   WHEREFORE,   the   challenged   decision   of   the   Court   of   Appeals   is   AFFIRMED  
and  other  substantially  similar  kinds  of  insurance  (Sec.  176  of  the  Insurance  Code).   in   so   far   as   it   holds   the   petitioner   liable   to   the   private   respondent   in   the   sum   of  
  P200,000.00  representing  the  face  value  of  the  insurance  contract,  with  interest  at  
An  accident  is  an  event  which  happens  without  any  human  agency  or,  if  happening   the   legal   rate   from   the   date   of   the   filing   of   the   complaint   until   the   full   amount   is  
through  human  agency,  an  event  which,  under  the  circumstances,  is  unusual  to  and   paid,   but   MODIFIED   with   the   deletion   of   all   awards   for   damages,   including  
not   expected   by   the   person   to   whom   it   happens.   It   has   also   been   defined   as   an   attorney's  fees,  except  the  costs  of  the  suit.    
injury  which  happens  by  reason  of  some  violence  or  casualty  to  the  insured  without    
his  design,  consent,  or  voluntary  cooperation.     RATIO:  
1.   The  words  “accident”  and  “accidental”  have  never  acquired  any  technical   iv.   The   act   was   precisely   intended   to   assure   Nalagon   that   the   gun  
signification   in   law,   and   when   used   in   an   insurance   contract   are   to   be   was  indeed  harmless  
construed   and   considered   according   to   the   ordinary   understanding   and   8.   Lim  was  unquestionably  negligent,  and  that  negligence  costed  him  his  own  
common  usage  and  speech  of  people  generally   life.   But   it   should   not   prevent   his   widow   from   recovering   from   the  
2.   In-­‐substance,  the  courts  are  practically  agreed  that  the  words  “accident”   insurance  policy  he  obtained  precisely  against  accident    
and   “accidental”   mean   that   which   happens   by   chance   or   fortuitously,   9.   There  is  nothing  in  the  policy  that  relieves  the  insurer  of  the  responsibility  
without   intention   or   design,   and   which   is   unexpected,   unusual,   and   to   pay   the   indemnity   agreed   upon   if   the   insured   is   shown   to   have  
unforeseen   contributed  to  his  own  accident    
3.   The   definition   that   has   usually   been   adopted   by   the   courts   is   that   an   10.   Indeed,   most   accidents   are   caused   by   negligence.   There   are   only   four  
accident  is  an  event  that  takes  place  without  one’s  foresight  or  expectation   exceptions   expressly   made   in   the   contract   to   relieve   the   insurer   from  
–  an  event  that  proceeds  from  an  unknown  cause,  or  is  an  unusual  effect   liability,  and  none  of  these  exceptions  is  applicable  in  the  case  at  bar    
of  a  known  case,  and  therefore  not  expected     11.   Exceptions:  
4.   An   accident   is   an   event   which   happens   without   any   human   agency   or,   if   i.   The  Company  shall  not  be  liable  in  respect  of:  
happening   through   human   agency,   an   event   which,   under   the   1.   Bodily  Injury  
circumstances,   is   unusual   to   and   not   expected   by   the   person   to   whom   it   a)   Sustained  
happens   i.   While   the   Insured   Person   is   engaging   in  
i.   It  has  also  been  defined  as  an  injury  which  happens  by  reason  of   (or   practicing   for   or   taking   part   in  
some   violence   or   casualty   to   the   injured   without   his   design,   training   peculiar   to)   any   of   the  
consent,  or  voluntary  co-­‐operation   Excluded  Activities  
5.   In   light   of   these   definitions,   the   Court   is   convinced   that   the   incident   that   ii.   By   any   person   before   such   person  
resulted  in  Lim’s  death  was  indeed  an  accident     attains   the   Lower   Age   Limit   or   after   the  
i.   Sun  Insurance  invoking  the  case  of  De  La  Cruz  v.  Capital  Insurance,   expiry  of  the  Period  of  Insurance  during  
says  that  there  is  no  accident  when  a  deliberate  act  is  performed   which   such   person   attains   the   Upper  
unless  some  additional,  unexpected,  independent  and  unforeseen   Age  Limit  
happening   occurs   which   produces   or   brings   about   their   injury   b)   Consequent  Upon  
death     i.   The   Insured   Person   committing   or  
ii.   There  was  such  a  happening.  This  was  the  firing  of  the  gun  which   attempting   to   commit   suicide   or  
was   the   additional   unexpected   and   independent   and   unforeseen   willfully   exposing   himself   to   needless  
occurrence  that  led  to  the  insured  person’s  death     peril   except   in   an   attempt   to   save  
6.   Sun  Insurance  also  cited  an  exception  that  the  company  shall  not  be  liable   human  life  
in  respect  of  bodily  injury,  consequent  upon  the  insured  person  attempting   ii.   War,   invasion,   act   of   foreign   enemy,  
to  commit  suicide  or  willfully  exposing  himself  to  needless  peril  except  in   hostilities  (whether  war  be  declared  or  
an  attempt  to  save  human  life     not)   civil   war,   rebellion,   revolution,  
7.   Sun  Insurance  also  maintains  that  by  the  mere  act  of  pointing  the  gun  to   insurrection,   or   military   or   usurped  
his  temple,  Lim  had  willfully  exposed  himself  to  needless  peril  and  so  came   power  
under  the  exception.  The  theory  that  a  gun  is  per  se  dangerous  and  should   2.   Bodily   injury   or   death   disablement   or   medical   expenses  
therefore  be  handled  cautiously  in  every  case   consequent   upon   or   contributed   to   by   the   Insured  
i.   The   SC   held   that   as   testified   by   the   secretary,   Lim   had   removed   Person  
the  magazine  from  the  gun  and  believe  it  as  no  longer  dangerous   a)   Having   taken   a   drug   unless   the   Insured   proves  
ii.   He  expressly  assured  her  that  the  gun  was  not  loaded   that   the   drug   was   taken   in   accordance   with  
iii.   It   is   submitted   that   Lim   did   not   willfully   expose   himself   to   proper   medical   prescription   and   directions   and  
needless  peril  when  he  pointed  the  gun  to  his  temple  because  the   not  for  treatment  of  drug  addiction  
fact  is  that  he  thought  it  was  not  unsafe  to  do  so  
b)   Suffering   from   pre-­‐existing   physical   or   mental  
defect  or  infirmity  which  had  not  been  declared  
to  and  accepted  in  wriitn  gby  the  Company  
3.   Death   Disablement   or   Medical   Expenses   consequent  
upon   or   contributed   to   by   the   Insured   Person   being  
pregnant   or   suffering   from   sickness   or   disease   not  
resulting  from  bodily  injury  due  to  a  gradually  operating  
cause  
4.   Risks  of  Murder  and  Assault  
12.   It   bears   noting   that   insurance   contracts   are   as   a   rule   supposed   to   be  
interpreted  liberally  in  favor  of  the  assured.  There  is  no  reason  to  deviate  
from  this  rule,  especially  in  view  of  the  circumstances  of  this  case  as  above  
analyzed    
13.   It   is   evident   that   Sun   Insurance   was   acting   in   good   faith   then   it   resisted  
Nerissa   Lim’s   case   on   the   ground   that   the   death   of   the   insured   was  
covered  by  the  exception  
14.   The   issue   was   indeed   debatable   ad   was   clearly   not   raised   only   for   the  
purpose   of   evaiding   a   legitimate   obligation.   The   Court   therefore   holds   that  
the  award  of  mrla  and  exemplary  damages  and  of  attorney’s  fees  is  unjust  
and  so  must  be  disapproved  
15.   In  order  that  a  person  may  be  liable  to  the  payment  of  moral  damages,  the  
law  requires  that  his  act  be  wrongful  
i.   The   adverse   result   of   an   action   does   not   per   se   make   the   act  
wrongful  and  subject  the  act  or  to  the  payment  of  moral  damages  
ii.   The   law   could   not   have   meant   to   impose   a   penalty   on   the   right   to  
litigate;  such  right  is  so  precious  that  moral  damages  may  not  be  
charged  on  those  who  may  exercise  it  erroneously.  For  these  the  
law  taxes  costs  
iii.   If   a   party   wins,   he   cannot   as   a   rule,   recover   attorney’s   fees   and  
litigation   expenses,   since   it   is   not   the   fact   of   winning   alone   that  
entitles   him   to   recover   such   damages   of   the   exceptional  
circumstances  enumerated  in  Art.  2208  
1.   Otherwise,   every   time   a   defendant   wins,   automatically  
the   plaintiff   must   pay   attorney’s   fees   thereby   putting   a  
premium  on  the  right  to  litigate  which  should  not  be  so  
2.   For   those   expenses,   the   law   deems   the   award   of   costs   as  
sufficient  
 
   
004  Biagtan  v  Insular  Life  Assurance  Company  Ltd.  (GUSTILO)   is   controlling.  If   the   injuries   suffered   by   the   insured   clearly   resulted   from   the  
                             March  29,  1972|  Makalintal,  J  |  Accident  Policy   intentional  act  of  a  third  person  the  insurer  is  relieved  from  liability  as  stipulated.  
 
 
PETITIONER:  Emilia  Biagtan,  Juan  T.  Biagtan  Jr.,  Miguel  Biagtan,  Gil  Biagtan  &  Gracia   FACTS:  
Biagtan   85.   Juan   S.   Biagtan   (J.   Bigtan)   was   insured   with   InsularLife   Assurance   Company  
RESPONDENTS:  Insular  Life  Assurance  Company  Ltd   (Insular   Life)   under   Policy   No.   398075   for   the   sum   of   P5,000.00   and,   under  
SUMMARY:  Juan  Bigtan  (J.  Bigtan)  was  insured  with  Insular  Life  Assurance  Company   a  supplementary  contract  denominated  "Accidental  Death  Benefit  Clause,  
(Insular   Life)   for   the   sum   of   P5,000.00   and,   under   a   supplementary   contract   for   an   additional   sum   of   P5,000.00   if   "the   death   of   the   Insured   resulted  
denominated  "Accidental  Death  Benefit  Clause,  for  an  additional  sum  of  P5,000.00   directly   from   bodily   injury   effected   solely   through   external   and   violent  
if   "the   death   of   the   Insured   resulted   directly   from   bodily   injury   effected   solely   means  sustained  in  an  accident  ...  and  independently  of  all  other  causes."  
through  external  and  violent  means  sustained  in  an  accident  ...  and  independently   The   clause,   however,expressly   provided   that   it   would   not   apply   where  
of  all  other  causes."  The  clause,  however,expressly  provided  that  it  would  not  apply   death  resulted  from  an  injury"intentionally  inflicted  by  another  party.  
where   death   resulted   from   an   injury"intentionally   inflicted   by   another   party.   The   86.   On  the  night  of  May  20,  1964,  or  during  the  first  hours  of  the  following  day  
house  of  insured  J.  Biagtan  was  robbed  by  a  band  of  robbers  who  were  charged  in   a   band   of   robbers   entered   the   house   of   J.   Biagtan.   What   transpired   is   as  
and   convicted   by   the   CFI   of   Pangasinan   for   robbery   with   homicide;   that   in   follows:   “that   on   the   night   of   May   20,   1964   or   the   first   hours   of   May   21,  
committing   the   robbery,   the   robbers,   on   reaching   the   staircase   landing   on   the   1964,   while   the   said   life   policy   and   supplementary   contract   were   in   full  
second   floor,   rushed   towards   the   door   of   the   second   floor   room,   where   they   force  and  effect,  the  house  of  insured  J.  Biagtan  was  robbed  by  a  band  of  
suddenly  met  a  person  near  the  door  of  one  of  the  rooms  who  turned  out  to  be  J.   robbers   who   were   charged   in   and   convicted   by   the   CFI   of   Pangasinan   for  
Biagtan   who   received   thrusts   from   their   sharp-­‐pointed   instruments,   causing   robbery   with   homicide;   that   in   committing   the   robbery,   the   robbers,   on  
wounds  on  the  body  of  said  J.  Biagtan  resulting  in  his  death.  Emilia  Bigtan  (E  Bigtan)   reaching   the   staircase   landing   on   the   second   floor,   rushed   towards   the  
et   al   as   beneficiaries   of   the   insured,   filed   a   claim   under   the   policy.   The   insurance   door  of  the  second  floor  room,  where  they  suddenly  met  a  person  near  the  
company  paid  the  basic  amount  of  P5,000.00  but  refused  to  pay  the  additional  sum   door   of   one   of   the   rooms   who   turned   out   to   be   J.   Biagtan   who   received  
of   P5,000.00   under   the   accidental   death   benefit   clause,   on   the   ground   that   the   thrusts  from  their  sharp-­‐pointed  instruments,  causing  wounds  on  the  body  
insured's   death   resulted   from   injuries   intentionally   inflicted   by   third   parties   and   of   said   J.   Biagtan   resulting   in   his   death   at   about   7   a.m.   on   the   same   day,  
therefore  was  not  covered.  E  Bigtan  et  al,  filed  suit  to  recover.  The  trial  court  ruled   May  21,  1964.”  
in  favor  of  E  Bigtan  et  al.  Hence,  this  appeal.  Issue  is  WoN  the  wounds  received  by   87.   Emilia  Bigtan  (E.  Bigtan),  et  al  as  beneficiaries  of  the  insured,  filed  a  claim  
the   insured   at   the   hands   of   the   robbers   (9   in   all,   5   mortal,   4   non   mortal)   were   under   the   policy.   The   insurance   company   paid   the   basic   amount   of  
inflicted  intentionally?  YES.   P5,000.00   but   refused   to   pay   the   additional   sum   of   P5,000.00   under   the  
The  SC  held  that  9  wounds  were  inflicted  upon  the  insured,  all  by  means  of  thrusts   accidental   death   benefit   clause,   on   the   ground   that   the   insured's   death  
with  sharp-­‐pointed  instruments  wielded  by  the  robbers.  This  is  a  physical  fact  as  to   resulted  from  injuries  intentionally  inflicted  by  third  parties  and  therefore  
which  there  is  no  dispute.  So  is  the  fact  that   5  of  those  wounds  caused  the  death  of   was   not   covered.  E  Bigtan  et  al,   filed  suit  to  recover.  After   due  hearing  the  
the  insured.  Whether  the  robbers  had  the  intent  to  kill  or  merely  to  scare  the  victim   court  a   quo  rendered   judgment   in   their   favor.     Hence,   the   present   appeal  
or  to  ward  off  any  defense  he  might  offer,  it  cannot  be  denied  that  the  act  itself  of   by  Insular  Life.    
inflicting   the   injuries   was   intentional.   Where   a   gang   of   robbers   enter   a   house   and   ISSUE/s:  
coming   face   to   face   with   the   owner,   even   if   unexpectedly,   stab   him   repeatedly,   it   is   4.  WoN  under  the  facts  stipulated  and  found  by  the  trial  court  the  wounds  received  
contrary  to  all  reason  and  logic  to  say  that  his  injuries  are  not  intentionally  inflicted,   by  the  insured  at  the  hands  of  the  robbers  (9  in  all,  5  mortal,  4  non  mortal)  were  
regardless  of  whether  they  prove  fatal  or  not.  As  it  was,  in  the  present  case  they  did   inflicted   intentionally?   YES   because   where   a   gang   of   robbers   enter   a   house   and  
prove   fatal,   and   the   robbers   have   been   accused   and   convicted   of   the   crime   of   coming   face   to   face   with   the   owner,   even   if   unexpectedly,   stab   him   repeatedly   (9  
robbery  with  homicide.   times),   it   is   contrary   to   all   reason   and   logic   to   say   that   his   injuries   are   not  
DOCTRINE:   Intentional   as   used   in   an   accident   policy   excepting   intentional   injuries   intentionally   inflicted,   regardless   of   whether   they   prove   fatal   or   not.   As   it   was,   in  
inflicted   by   the   insured   or   any   other   person,   etc.,   implies   the   exercise   of   the   the  present  case  they  did  prove  fatal.  
reasoning   faculties,   consciousness   and   volition.   Where   a   provision   of   the   policy    
excludes  intentional  injury,  it  is  the  intention  of  the  person  inflicting  the  injury  that  
RULING:   WHEREFORE,   the   decision   appealed   from   is   reversed   and   the   complaint   them,   together   with   the   owner   of   the   house,   approached   and   stood   in  
dismissed,  without  pronouncement  as  to  costs.   front  of  the  main  gate,  a  shot  was  fired  and  it  turned  out  afterwards  that  
RATIO:   the   watchman   was   hit   in   the   abdomen,   the   wound   causing   his   death.  
103.  The   trial   court   stated   that   since   the   parties   presented   no   evidence   and   Under   those   circumstances   this   Court   held   that   it   could   not   be   said   that  
submitted   the   case   upon   stipulation,   there   was   no   "proof   that   the   act   of   the  killing  was  intentional  for  there  was  the  possibility  that  the  malefactor  
receiving   thrust   from   the   sharp-­‐pointed   instrument   of   the   robbers   was   had  fired  the  shot  to  scare  people  around  for  his  own  protection  and  not  
intended   to   inflict   injuries   upon   the   person   of   the   insured   or   any   other   necessarrily  to  kill  or  hit  the  victim.  
person  or  merely  to  scare  away  any  person  so  as  to  ward  off  any  resistance   107.  However,   in   this   case,   9   wounds   inflicted   with   bladed   weapons   at   close  
or   obstacle   that   might   be   offered   in   the   pursuit   of   their   main   objective   range  cannot  conceivably  be  considered  as  innocent  insofar  as  such  intent  
which  was  robbery.   is   concerned.   The   manner   of   execution   of   the   crime   permits   no   other  
104.  The   SC   stated   that   the   trial   court   committed   a   plain   error   in   drawing   the   conclusion.  Thus,  it  has  been  held  that  "intentional"  as  used  in  an  accident  
conclusion   it   did   from   the   admitted   facts.   9   wounds   were   inflicted   upon   policy  excepting  intentional  injuries  inflicted  by  the  insured  or  any  other  
the   insured,   all   by   means   of   thrusts   with   sharp-­‐pointed   instruments   person,   etc.,   implies   the   exercise   of   the   reasoning   faculties,  
wielded   by   the   robbers.   This   is   a   physical   fact   as   to   which   there   is   no   consciousness   and   volition.   Where   a   provision   of   the   policy   excludes  
dispute.   So   is   the   fact   that   5   of   those   wounds   caused   the   death   of   the   intentional   injury,   it   is   the   intention   of   the   person   inflicting   the   injury  
insured.  Whether  the  robbers  had  the  intent  to  kill  or  merely  to  scare  the   that  is  controlling.  If  the  injuries  suffered  by  the  insured  clearly  resulted  
victim  or  to  ward  off  any  defense  he  might  offer,  it  cannot  be  denied  that   from   the   intentional   act   of   a   third   person   the   insurer   is   relieved   from  
the  act  itself  of  inflicting  the  injuries  was  intentional.   liability  as  stipulated.  
105.  It   should   be   noted   that   the   exception   in   the   accidental   benefit   clause   108.  The  SC  cited  2  American  cases.  The  first  Hutchcraft's  Ex'r  v.  Travelers'  Ins.  
invoked   by   E.   Bigtan   et   al   does   not   speak   of   the   purpose   whether   Co,    the  insured  was  waylaid  and  assassinated  for  the  purpose  of  robbery.  
homicidal  or  not  of  a  third  party  in  causing  the  injuries,  but  only  of  the  fact   The  Court  held  that  while  the  assassination  of  the  insured  was  as  to  him  an  
that   such   injuries   have   been   "intentionally"   inflicted,   this   obviously   to   unforeseen  event  and  therefore  accidental,  "the  clause  of  the  proviso  that  
distinguish  them  from  injuries  which,  although  received  at  the  hands  of  a   excludes   the   insurer's   liability,   in   case   death   or   injury   is   intentionally  
third   party,   are   purely   accidental.   This   construction   is   the   basic   idea   inflicted  by  another  person,  applies  to  this  case.The  other  one  was  Butero  
expressed  in  the  coverage  of  the  clause  itself,  namely,  that  "the  death  of   v.   Travelers'   Acc.   Ins.   Co,    the   insured   was   shot   three   times   by   a   person  
the   insured   resulted   directly   from   bodily   injury   effected   solely   through   unknown  late  on  a  dark  and  stormy  night,  while  working  in  the  coal  shed  of  
external   and   violent   means   sustained   in   an  accident  ...   and   a   railroad   company.   The   policy   did   not   cover   death   resulting   from  
independently   of   all   other   causes."   A   gun   which   discharges   while   being   "intentional   injuries   inflicted   by   the   insured   or   any   other   person."  The  
cleaned   and   kills   a   bystander;   a   hunter   who   shoots   at   his   prey   and   hits   a   Court   found   that   under   the   facts,   showing   that   the   murderer   knew   his  
person  instead;  an  athlete  in  a  competitive  game  involving  physical  effort   victim   and   that   he   fired   with   intent   to   kill,   there   could   be   no   recovery  
who  collides  with  an  opponent  and  fatally  injures  him  as  a  result:  these  are   under   the   policy   which   excepted   death   from   intentional   injuries   inflicted  
instances   where   the   infliction   of   the   injury   is   unintentional   and   therefore   by  any  person  
would  be  within  the  coverage  of  an  accidental  death  benefit  clause  such   as    
that  in  question  in  this  case.  But  where  a  gang  of  robbers  enter  a  house   Separate  Opinions:  
Concurring,  Barredo,  J.    
and  coming  face  to  face  with  the  owner,  even  if  unexpectedly,  stab  him  
1.  Examining  the  unequivocal  language  of  the  provision  in  controversy  and  considering  that  
repeatedly,  it  is  contrary  to  all  reason  and  logic  to  say  that  his  injuries  are  
the  insured  accepted  the  policy  without  asking  that  it  be  made  clear  that  the  phrase  "injury  
not  intentionally  inflicted,  regardless  of  whether  they  prove  fatal  or  not.   intentionally  inflicted  by  a  third  party"  should  be  understood  to  refer  only  to  injuries  inflicted  
As  it  was,  in  the  present  case  they  did  prove  fatal,  and  the  robbers  have   by  a  third  party  without  any  wilful  intervention  on  his  part  of  the  insured  or,  in  other  words,  
been  accused  and  convicted  of  the  crime  of  robbery  with  homicide.   without   any   connivance   with   him   the   insured   in   order   to   augment   the   proceeds   of   the   policy  
106.  The   trial   court   relied   on   the   case   of   Calanoc   v   CA   in   support   of   its   decision.   for   his   benificiaries,   I   am   inclined   to   agree   that   death   caused   by   criminal   assault   is   not  
The   facts   in   that   case,   however,   are   different   from   those   obtaining   here.   covered   by   the   policies   of   the   kind   here   in   question,   specially   if   the   assault,   as   a   matter   of  
The  insured  there  was  a  watchman  in  a  certain  company,  who  happened  to   fact,   could   have   been   more   or   less   anticipated,   as   when   the   insured   happens   to   have   violent  
be   invited   by   a   policeman   to   come   along   as   the   latter   was   on   his   way   to   enemies  or  is  found  in  circumstances  that  would  make  his  life  fair  game  of  third  parties.  
investigate  a  reported  robbery  going  on  in  a  private  house.  As  the  two  of  
2.  As  to  the  rest,  I  have  no  doubt  that  the  killing  of  the  insured  in  this  case  is  as  intentional  as  
any  intentional  act  can  be,  hence  this  concurrence  
Teehankee,  dissenting:  
1.  It   is   respectfully   submitted   that   the   lower   court   committed   no   error   in   law   in   holding  
defendant   insurance   company   liable   to   plaintiffs-­‐beneficiaries   under   its   accidental   death  
benefit   clause,   by   virtue   of   the   following   considerations:   The   case   of  Calanoc  cited   by   the  
lower  court  is  indeed  controlling  here.  This  Court,  there  construing  a  similar  clause,  squarely  
ruled   that   fatal   injuries   inflicted   upon   an   insured   by   a   malefactors   during   the   latter's  
commission   of   a   crime   are   deemed   accidental   and   within   the   coverage   of   such   accidental  
death  benefit  clauses  and  the  burden  of  proving  that  the  killing  was  intentional  so  as  to  have  
it  fall  within  the  stipulated  exception  of  having  resulted  from  injuries  "intentionally  inflicted  
by  a  third  party"  must  be  discharged  by  the  insurance  company.  This  Court  there  clearly  held  
that  in  such  cases  where  the  killing  does  not  amount  to  murder,  it  must  be  held  to  be  a  "pure  
accident"   on   the   part   of   the   victim,   compensable   with   double-­‐indemnity,   even   though   the  
malefactor  is  criminally  liable  for  his  act.  
2.  E.  Bigtan,  et  al  properly  assert  in  their  brief  that  the  sole  error  assigned  by  Insular  Life,  to  
wit,  that  the  fatal  injuries  were  not  accidental  as  held  by  the  lower  court  but  should  be  held  
to  have  been  intentionally  inflicted,  raises  a  question  of  fact  which  Insular  Life  is  now  barred  
from  raising,  since  it  expressly  limited  its  appeal  to  this  Court  purely  "on  questions  of  law",  
per  its  noitice  of  appeal.  
3.  It  has  long  been  an  established  rule  of  construction  of  so-­‐called  contracts  of  adhesion  such  
as  insurance  contracts,  where  the  insured  is  handed  a  printed  insurance  policy  whose  fine-­‐
print   language   has   long   been   selected   with   great   care   and   deliberation   by   specialists   and  
legal  advisers  employed  by  and  acting  exclusively  in  the  interest  of  the  insurance  company,  
that  the  terms  and  phraseology  of  the  policy,  particularly  of  any  exception  clauses,  must  be  
clearly  expressed  so  as  to  be  easily  understood  by  the  insured  and  any  "ambiguous,  equivocal  
or  uncertain  terms"  are  to  be  "construed  strictly  and  most  strongly  against  the  insurer  and  
liberally   in   favor   of   the   insured   so   as   to   effect   the   dominant   purpose   of   indemnity   or  
payment  to  the  insured,  especially  where  a  forfeiture  is  involved.  
4.  It  is  obvious  from  the  very  classification  of  the  exceptions  and  applying  the  rule  of  noscitus  
a   sociis  that   the   double-­‐indemnity   policy   covers   the   insured   against   accidental   death,  
whether   caused   by   fault,   negligence   or   intent   of   a   third   party   which   is   unforeseen   and  
unexpected   by   the   insured.   All   the   associated   words   and   concepts   in   the   policy   plainly  
exclude  the  accidental  death  from  the  coverage  of  the  policy  only  where  the  injuries  are  self-­‐
inflicted  or  attended  by  some  proscribed  act  of  the  insured  or  are  incurred  in  some  expressly  
excluded   calamity   such   as   riot,   war   or   atomic   explosion.   Finally,   the   untenability   of   herein  
Insular   Life’s   claim   that   the   insured's   death   fell   within   the   exception   is   further   heightened   by  
the   stipulated   fact   that   two   other   insurance   companies   which   likewise   covered   the   insured  
for   which   larger   sums   under   similar   accidental   death   benefit   clauses   promptly   paid   the  
benefits  thereof  to  E.  Bigtan  et  al.    
   
005  Malayan  Insurance  Co.  v.  CA  (Hilario) respondent   Vallejos,   but   it   cannot,   as   incorrectly   held   by   the   trial   court,   be   made  
September  26,  1998  |  Padilla,  J.  |  Casualty  Insurance "solidarily"   liable   with   the   two   principal   tortfeasors   namely   respondents   Sio   Choy  
and  San  Leon  Rice  Mill,  Inc.  For  if  Malayan  Insurance  were  solidarily  liable  with  Sio  
PETITIONER:  Malayan  Insurance  Co.  Inc. Choy  and  San  Leon  Rice  Mill  by  reason  of  the  indemnity  contract  against  third  party  
RESPONDENTS:  CA  (Third  Division,  Martin  C.  Vallejos,  Sio  Choy,  San  Leon  Rice  Mill   liability-­‐under   which   an   insurer   can   be   directly   sued   by   a   third   party   —   this   will  
Inc.,  Pangasinan  Transportation  Co.,  Inc. result   in   a   violation   of   the   principles   underlying   solidary   obligation   and   insurance  
contracts.   To   recapitulate   then:   We   hold   that   only   respondents   Sio   Choy   and   San  
SUMMARY:   Sio   Choy   owns   a   jeep,   and   had   it   insured   with   Malayan   Insurance.   A   Leon  Rice  Mill,  Inc.  are  solidarily  liable  to  the  respondent  Martin  C.  Vallejos  for  the  
Private  Car  Comprehensive  Policy  was  issued,  with  coverage  for  “own  damage”  not   amount   of   P29,103.00.   Vallejos   may   enforce   the   entire   obligation   on   only   one   of  
to   exceed   P600,   and   “third   party   liability”   in   the   amount   of   P   20,000.   During   the   said   solidary   debtors.   If   Sio   Choy   as   solidary   debtor   is   made   to   pay   for   the   entire  
effectivity  of  the  policy,  the  insured  jeep  was  being  driven  by  an  employee  of  San   obligation  (P29,103.00)  and  Malayan  Insurance,  as  insurer  of  Sio  Choy,  is  compelled  
Leon   Rice   Mill,   and   it   collided   with   a   passenger   bus   belonging   to   Pangasinan   to  pay  P20,000.00  of  said  entire  obligation,  Malayan  Insurance  would  be  entitled,  as  
Transportation   Co.   (PANTRANCO),   causing   damage   to   the   insured   vehicle   and   subrogee  of  Sio  Choy  as  against  San  Leon  Rice  Mills,  Inc.,  to  be  reimbursed  by  the  
injuries   to   the   driver   and   a   passenger   of   the   jeep,   Vallejos.   So,   Valles   filed   an   action   latter  in  the  amount  of  P14,551.50  (which  is  1/2  of  P29,103.00  )  
for   damages   against   Sio   Choy,   Malayan   Insurance,   and   PANTRANCO.   Malayan  
Insurance  on  the  other  hand,  tried  to  evade  liability  by  claiming  that  the  driver  of   DOCTRINE:   While   it   is   true   that   where   the   insurance   contract   provides   for  
the   insured   car   at   the   time   of   the   accident   was   not   Sio   Choy’s   employee,   but   of   San   indemnity  against  liability  to  third  persons,  such  third  persons  can  directly  sue  the  
Leon  Rice  Mill,  and  that  the  real  party  which  should  be  held  liable  is  San  Leon  Rice   insurer,     however,   the   direct   liability   of   the   insurer   under   indemnity   contracts  
Mill,  not  Malayan  Insurance.  RTC  ruled  in  favor  of  Vallejos,  saying  that  the  parties   against   third   party   liability   does   not   mean   that   the   insurer   can   be   held   solidarily  
are   solidarily   liable   to   Vallejos,   and   the   CA   affirmed   but   absolved   San   Leon   Rice   Mill   liable   with   the   insured   and/or   the   other   parties   found   at   fault.   The   liability   of   the  
of  liability,  saying  that  it  was  not  liable  to  pay  Malayan  Insurance,  because  it  is  not   insurer  is  based  on  contract;  that  of  the  insured  is  based  on  tort.
privy   to   the   insurance   policy   issued   by   Malayan   Insurance   in   favor   of   Sio   Choy.  
Malayn   Insurance   appeals   before   the   SC,   claiming   that   San   Leon   Rice   Mill   should   FACTS:
reimburse   Malayan   Insurance   any   amount   in   excess   of   ½   of   the   entire   amount   of   88.   Malayan   Insurance   issued   a   Private   Car   Comprehensive   Policy   covering   a  
damages  it  may  be  ordered  to  pay  jointly  and  severally  with  Sio  Choy.   Willys  jeep  in  favor  of  Sio  Choy.  The  coverage  was  for  “own  damage”  not  
  to  exceed  P600,  and  “third-­‐party  liability”  in  the  amount  of  P20,000.  
Issues:     89.   During  the  effectivity  of  the  policy,  the  insured  jeep  was  being  driven  by  an  
1.   Whether   the   RTC   was   correct   in   holding   Malayan   Insurance,   Sio   Choy,   and   San   employee   of   San   Leon   Rice   Mill,   and   it   collided   with   a   passenger   bus  
Leon  Rice  Mill  solidarily  liable  to  Vallejos  –  NO,  only  Sio  Choy  and  San  Leon  Rice  Mill   belonging   to   Pangasinan   Transportation   Co.   (PANTRANCO),   causing  
are  solidarily  liable  to  Vallejos  for  damages  awarded  in  his  favor.   damage  to  the  insured  vehicle  and  injuries  to  the  driver  and  a  passenger  of  
2.   Whether   Malayan   Insurance   is   entitled   to   be   reimbursed   by   San   Leon   Rice   Mill   the  jeep,  Vallejos.  
for   whatever   amount   Malayan   Insurance   has   been   adjudged   to   pay   respondent   90.   So,  Valles  filed  an  action  for  damages  against  Sio  Choy,  Malayan  Insurance,  
Vallejos   on   its   insurance   policy—   NO,   Malayan   Insurance   is   entitled   to   be   and  PANTRANCO.  
reimbursed  by  San  Leon  Rice  Mill  because  of  subrogation.  Malayan  Insurance,  upon   91.   In   its   answer,   PANTRANCO   claimed   that   the   jeep   of   Sio   Choy   was  operated  
paying  respondent  Vallejos  the  amount  of  riot  exceeding  P20,000.00,  shall  become   at   an   excessive   speed   and   bumped   the   PANTRANCO   bus   which   moved   to  
the  subrogee  of  the  insured,  the  respondent  Sio  Choy;  as  such,  it  is  subrogated  to   avoid   the   jeep,   that   it   had   observed   the   diligence   of   a   good   father   of   a  
whatever  rights  the  latter  has  against  respondent  San  Leon  Rice  Mill,  Inc.     family   to   prevent   damage,   especially   in   the   selection   and   supervision   of   its  
  employees  and  in  the  maintenance  of  its  motor  vehicles,  trying  to  absolve  
Sio   Choy   and   San   Leon   Rice   Mill   are   the   principal   tortfeasors   who   are   primarily   itself  of  liability.  
liable   to   Vallejos,   thus   their   liability   is   solidary.   On   the   other   hand,   Malayan   92.   Sio  Choy  filed  an  answer  with  counterclaim,  and  said  that  he  had  already  
Insurance’s   liability   is   its   insurance   contract   with   Sio   Choy.   It   is   only   liable   for   paid  Vallejos  for  his  hospitalization  and  other  expenses,  and  another  cross  
P20,000  because  it  is  the  insurer  of  Sio  Choy  under  the  third  party  liability  clause.   claim  against  Malayan  Insurance  invoking  the  policy  covering  the  jeep.  Sio  
(see   doctrine)   In   the   case   at   bar,   petitioner   as   insurer   of   Sio   Choy,   is   liable   to   Choy   alleged   that   the   insurance   company   obliged   itself   to   indemnify   Sio  
Choy,   as   insured,   for   the   damage   to   his   motor   vehicle,   as   well   as   for   any  
liability  to  third  persons  arising  out  of  any  accident  during  the  effectivity  of   RATIO:
such  insurance  contract.   Issue  1  
93.   Malayan   Insurance   on   the   other   hand,   tried   to   evade   liability   by   claiming   109.  Only   Sio   Choy   and   San   Leon   Rice   Mill   are   solidarily   liable   to   Vallejos   for  
that   the   driver   of   the   insured   car   at   the   time   of   the   accident   was   not   Sio   damages  awarded  in  his  favor.  
Choy’s  employee,  but  of  San  Leon  Rice  Mill,  and  that  the  real  party  which   110.   Sio  Choy  was  made  liable  to  Vallejos  as  owner  of  the  jeep  pursuant  to  Art.  
should  be  held  liable  is  San  Leon  Rice  Mill,  not  Malayan  Insurance.   2184   of   the   CC:   Art.  2184.  In  motor  vehicle  mishaps,  the  owner  is  solidarily  liable  with  his  
94.   RTC   ruled   in   favor   of   Vallejos   and   ordered   Sio   Choy,   Malayan   Insurance,   driver,   if   the   former,   who   was   in   the   vehicle,   could   have,   by   the   use   of   due   diligence,  
prevented   the   misfortune   it   is   disputably   presumed   that   a   driver   was   negligent,   if   he   had  
and  San  Leon  Rice  Mill  to  pay  damages,  their  liability  being  solidary.   been  found  guilty  of  reckless  driving  or  violating  traffic  regulations  at  least  twice  within  the  
1.   The  trial  court  held  petitioner  together  with  respondents  Sio  Choy   next  preceding  two  months.    
and  San  Leon  Rice  Mills  Inc.  solidarily  liable  to  respondent  Vallejos   If  the  owner  was  not  in  the  motor  vehicle,  the  provisions  of  article  2180  are  applicable.  
for   a   total   amount   of   P29,103.00,   with   the   qualification   that   111.  San   Leon   Rice   Mill   as   adjudged   liable   to   Vallejos   because   it   is   the   employer  
Malayn  Insurance’s  liability  is  only  up  to  P20,000.00.   of  the  driver  of  the  insured  jeep  at  the  time  of  the  mishap,  in  accordance  
95.   The   CA   affirmed   the   ruling   of   the   RTC,   except   that   it   ruled   that   San   Leon   with  Art.  2180  of  the  CC:    
Rice   Mill   has   no   obligation   to   indemnify   or   reimburse   the   Malayan   Art.  2180.  The  obligation  imposed  by  article  2176  is  demandable  not  only  for  one's  own  acts  
or  omissions,  but  also  for  those  of  persons  for  whom  one  is  responsible.    
Insurance  since  San  Leon  Rice  Mill  is  not  privy  to  the  contract  of  insurance   xxx  xxx  xxx  
between  Sio  Choy  and  Malayan  Insurance.   Employers  shall  be  liable  for  the  damages  caused  by  their  employees  and  household  helpers  
96.   Malayn  Insurance  appeals  before  the  SC,  claiming  that  San  Leon  Rice  Mill   acting   within   the   scope   of   their   assigned   tasks,   even   though   the   former   are   not   engaged   ill  
should   reimburse   Malayan   Insurance   any   amount   in   excess   of   ½   of   the   any  business  or  industry.    
xxx  xxx  xxx  
entire   amount   of   damages   it   may   be   ordered   to   pay   jointly   and   severally   The   responsibility   treated   in   this   article   shall   cease   when   the   persons   herein   mentioned  
with  Sio  Choy.   proved  that  they  observed  all  the  diligence  of  a  good  father  of  a  family  to  prevent  damage.  
112.  Sio   Choy   and   San   Leon   Rice   Mill   are   the   principal   tortfeasors   who   are  
ISSUE/s: primarily  liable  to  Vallejos,  thus  their  liability  is  solidary.  
12.   Whether  the  RTC  was  correct  in  holding  Malayan  Insurance,  Sio  Choy,  and   113.  On   the   other   hand,   Malayan   Insurance’s   liability   is   its   insurance   contract  
San  Leon  Rice  Mill  solidarily  liable  to  Vallejos  –  NO,  only  Sio  Choy  and  San   with   Sio   Choy.   It   is   only   liable   for   P20,000   because   it   is   the   insurer   of   Sio  
Leon   Rice   Mill   are   solidarily   liable   to   Vallejos   for   damages   awarded   in   his   Choy  under  the  third  party  liability  clause.  
favor.   114.  While   it   is   true   that   where   the   insurance   contract   provides   for   indemnity  
13.   Whether  Malayan  Insurance  is  entitled  to  be  reimbursed  by  San  Leon  Rice   against  liability  to  third  persons,  such  third  persons  can  directly  sue  the  
Mill   for   whatever   amount   Malayan   Insurance   has   been   adjudged   to   pay   insurer,   however,   the   direct   liability   of   the   insurer   under   indemnity  
respondent   Vallejos   on   its   insurance   policy—   NO,   Malayan   Insurance   is   contracts  against  third  party  liability  does  not  mean  that  the  insurer  can  
entitled   to   be   reimbursed   by   San   Leon   Rice   Mill   because   of   subrogation.   be  held  solidarily  liable  with  the  insured  and/or  the  other  parties  found  
Malayan   Insurance,   upon   paying   respondent   Vallejos   the   amount   of   riot   at   fault.   The   liability   of   the   insurer   is   based   on   contract;   that   of   the  
exceeding   P20,000.00,   shall   become   the   subrogee   of   the   insured,   the   insured  is  based  on  tort.    
respondent  Sio  Choy;  as  such,  it  is  subrogated  to  whatever  rights  the  latter   115.  In  the  case  at  bar,  petitioner  as  insurer  of  Sio  Choy,  is  liable  to  respondent  
has  against  respondent  San  Leon  Rice  Mill,  Inc.   Vallejos,   but   it   cannot,   as   incorrectly   held   by   the   trial   court,   be   made  
"solidarily"   liable   with   the   two   principal   tortfeasors   namely   respondents  
RULING:   To   recapitulate   then:   We   hold   that   only   respondents   Sio   Choy   and   San   Sio   Choy   and   San   Leon   Rice   Mill,   Inc.   For   if   Malayan   Insurance   were  
Leon  Rice  Mill,  Inc.  are  solidarily  liable  to  the  respondent  Martin  C.  Vallejos  for  the   solidarily   liable   with   Sio   Choy   and   San   Leon   Rice   Mill   by   reason   of   the  
amount   of   P29,103.00.   Vallejos   may   enforce   the   entire   obligation   on   only   one   of   indemnity  contract  against  third  party  liability-­‐under  which  an  insurer  can  
said   solidary   debtors.   If   Sio   Choy   as   solidary   debtor   is   made   to   pay   for   the   entire   be   directly   sued   by   a   third   party   —   this   will   result   in   a   violation   of   the  
obligation  (P29,103.00)  and  Malayan  Insurance,  as  insurer  of  Sio  Choy,  is  compelled   principles  underlying  solidary  obligation  and  insurance  contracts.  
to  pay  P20,000.00  of  said  entire  obligation,  Malayan  Insurance  would  be  entitled,  as   Issue  2  
subrogee  of  Sio  Choy  as  against  San  Leon  Rice  Mills,  Inc.,  to  be  reimbursed  by  the   116.  Malayan  Insurance  is  entitled  to  be  reimbursed  by  San  Leon  Rice  Mill.  
latter  in  the  amount  of  P14,551.50  (which  is  1/2  of  P29,103.00  )
117.  Subrogation   is   a   normal   incident   of   indemnity   insurance.   Upon   payment   of  
the  loss,  the  insurer  is  entitled  to  be  subrogated  pro  tanto  to  any  right  of  
action   which   the   insured   may   have   against   the   third   person   whose  
negligence  or  wrongful  act  caused  the  loss.  
118.  It   follows,   therefore,   that   Malayan   Insurance,   upon   paying   respondent  
Vallejos   the   amount   of   riot   exceeding   P20,000.00,   shall   become   the  
subrogee  of  the  insured,  the  respondent  Sio  Choy;  as  such,  it  is  subrogated  
to   whatever   rights   the   latter   has   against   respondent   San   Leon   Rice   Mill,  
Inc.  
   
006 FAR EASTERN SURETY VS DE MISA (HIRANG) taxicab passengers. The insurer, likewise, denied responsibility.
26 October 1968 |Reyes, J. | 3rd party Claim 4.   CFI ruled in favor of Socorro Dancel Vda. De Misa, Araceli Maria Pinto
and awarded them actual, moral, and exemplary damages and attorney's
PETITIONER: FAR EASTERN SURETY & INSURANCE COMPANY, INC fees, payable by La Mallorca; and sentenced the insurance company to pay
to La Mallorca P10,000.00 on its third-party liability insurance
RESPONDENTS: SOCORRO DANCEL VDA. DE MISA, ARACELI MARIA 5.   Upon appeal to the CA, Far Eastern interposes the defense that it was not
PINTO and LA MALLORCA answerable except for whatever amounts the insured might be legally liable
for in the event of accident caused by, or arising out of, the use of the motor
SUMMARY: Socorro Dancel Vda. de Misa and Araceli Pinto hired a taxicab vehicle, however the CA said that while the collision was due to the fault
operated by respondent La Mallorca in Quezon City. While proceeding south of the driver of the sand truck, CA nevertheless held the taxicab
toward the Archbishop's Palace in Shaw Boulevard, the taxicab collided with a operator liable in damages to the passengers of its motor vehicle on the
gravel and sand truck. The two passengers of the La Mallorca taxicab were strength of its representation that the passengers were insured against
injured and filed suit for damage. La Mallorca denied liability but but instituted accidents(in view of its third party liability insurance contract), as shown
a third-party complaint against herein appellant, Far Eastern Surety and Insurance by the sticker affixed to the taxicab – Basically the CA reasoned that even
Company, to recoup from the latter, based on its Common Carrier's Accident if it was found out that Far Eastern is not liable to pay since as per the
Insurance No. CCA 106, any damages that might be recovered by the contract it is only liable if the insured is legally liable to pay, Far Eastern
plaintiff’s taxicab passengers. CFI ruled in favor of the passengers and also still needs to pay because of the representation La Mallorca did.
ordered Far Eastern to pay to La Mallorca P10,000.00 on its third-party liability (representation that the passengers were insured against, as shown by
insurance. Upon appeal to the CA, Far Eastern Interposes the defense that it cannot the sticker affixed to the taxicab)
be held liable because La Mallorca was not legally liable to pay the passengers ISSUE/s: WoN Far Eastern Surety is liable to the La Mallorca on its policy of
since it was found out the that the accident was because of the fault of the Sand insurance. – No, while the decision correctly held that La Mallorca was in estoppel,
truck driver but CA still held them liable. (PLESE SEE FACT 5 IMPORTANT). and could not be heard to deny that its passengers were insured, it does not
The issue is whether Far Eastern is liable and the SC held in the negative. The SC necessarily follow that the estoppel, likewise, applied to the appellant insurer
stated that indeed La Mallorca was estopped from asserting that it was not liable
because of the fact that it placed stickers in its taxicabs showing that the RULING: WHEREFORE, the decision of the Court of Appeals is modified, by
passengers were insured against accidents however, it does not necessarily eliminating therefrom the award against the appellant, Far Eastern Insurance Co.,
follow that the estoppel, likewise, applied to the appellant insurer. There was Inc., in favor of the taxicab operator, La Mallorca, including the sharing of the costs
no showing that Far Eastern consented to, or even knew of, the representation of litigation, which shall be exclusively borne by the latter entity. Without costs in
made by the taxicab company to its passengers. this instance.
DOCTRINE: Estoppel of the insured does not necessarily bind the insurer without RATIO:
any evidence that insurer consented to, or even knew of, the representation made 135.  SC held that the appealed decision itself shows that the indemnity awarded
to the passengers of the La Mallorca taxicab was not because of the accident
FACTS: but was exclusively predicated on the representation made by the
1.   On 3 September 1957 the two respondents, Socorro Dancel Vda. de Misa taxicab company to its passengers that the latter were insured against
and Araceli Pinto hired a taxicab operated by respondent La Mallorca accidents
in Quezon City. While proceeding south toward the Archbishop's Palace in a.   This is plain from the consideranda made in the appealed decision
Shaw Boulevard, the taxicab collided with a gravel and sand truck, driven “... indeed, the notice in the sticker evidently being intended in
by one Faustino Nabor. order to court the riding public into patronizing La Mallorca,
2.   As a result, the two passengers of the La Mallorca taxicab were injured and being placed there right in the taxi, the only meaning that
and filed suit for damages against the taxicab company in the Court of can be given to it — and certainly it must have a meaning for
First Instance. it could not have been there placed if intended to be useless —
was that La Mallorca bound itself, in its contract of carriage,
3.   The operator denied liability, but instituted a third-party complaint against
with that additional stipulation therein indicated, that the
herein appellant, Far Eastern Surety and Insurance Company, to recoup passengers were "Insured", and if there be any ambiguity in its
from the latter, based on its Common Carrier's Accident Insurance No. meaning, such ambiguity must be construed most strongly
CCA 106, any damages that might be recovered by the plaintiff’s against the party causing the ambiguity”
136.  Having that as a basis, this Court must find that La Mallorca had indeed,
insured its pasengers and since such a stipulation was not at all illegal, it
must bind La Mallorca, and would be enough to render it liable for injuries
to the passengers thereof, even though it had not been at fault, i.e., that the
damage had come from & fortuitous event coming from the fault of a third
party for which it was not responsible, since the Law also dictates that:
a.   ART. 1174. Except in cases expressly specified by the law, or when it is
otherwise declared by stipulation, or when the nature of the obligation
requires the assumption of risk, no person shall be responsible for those
events which could not be foreseen, or which, though foreseen, are
inevitable.
137.   While the decision correctly held that la Mallorca was in estoppel and
could not be heard to deny that its passengers were insured, it does not
necessarily follow that the estoppel, likewise, applied to the appellant
insurer. As it does not appear that the insurance company authorized or
consented to, or even knew of, the representation made by the taxicab
company to its passengers, it follows that the source of the award of
damages against the taxicab company was beyond, or outside of, the
contemplation of the parties to the contract of Accident Insurance No. CCA
106, and that the insurer may not be held liable for such damages.
Contract

Liability  of  the  Far  Eastern  Surety  is  defined  as  follows:  —  

"l.  The  Company  will  subject  to  the  Limits  of  Liability  indemnify  the  Insured  in  the  
event   of   accident   caused   by   or   arising   out   of   the   use   of   the   Motor   Vehicle   or   in  
connection   with   the   loading   or   unloading   of   the   Motor   Vehicle   against   all   sums  
including   claimant's   costs   and   expense   which   the   Insured   shall   become   legally  
liable  to  pay  in  respect  of:  

(a)  death  of  or  bodily  injury  to  any  person  

(b)  damage  to  property  

"The   above-­‐quoted   stipulation   exempts   the   Far   Eastern   Surety   from   paying  
damages  other  than  actual  bodily  injuries  sustained  by  third  parties  

   
007  Dela  Cruz  v.  Capital  Insurance  (HORTALEZA)   Additionally,  Furthermore,  the  policy  involved  herein  specifically  excluded  from  its  
June,  30,  1966  |  Barrera,  J.  |  Casualty  Insurance   coverage,  Death  or  disablement  resulting  from  engagement  in  boxing  contests  was  
  not   declared   outside   of   the   protection   of   the   insurance   contract.   Failure   of   the  
PETITIONER:Simon  Dela  Cruz   defendant  insurance  company  to  include  death  resulting  from  a  boxing  match  or  
RESPONDENTS:  The  Capital  Insurance  and  Surety  co.,  inc.   other  sports  among  the  prohibitive  risks  leads  inevitably  to  the  conclusion  that  it  
SUMMARY:  Eduardo  de  la  Cruz,  employed  as  a  mucker  in  the  Itogon-­‐Suyoc  Mines,   did  not  intend  to  limit  or  exempt  itself  from  liability  for  such  death(  read  ratio,  to  
Inc.   in   Baguio,   was   the   holder   of   an   accident   insurance   policy   (No.   ITO-­‐BFE-­‐170)   see  the  list  of  excluded  risks)  
underwritten   by   the   Capital   Insurance   &   Surety   Co.,   Inc.,(Capital)   for   the   period    
beginning   November   13,   1956   to   November   12,   1957.   On   January   1,   1957   in   DOCTRINE  1:  The  fact  that  boxing  is  attended  with  some  risks  of  external  injuries  
connection   with   an   Itogon   festival,   Eduardo   joined   an   organized   amateur   boxing   does  not  make  any  injuries  received  in  the  course  of  the  game  not  accidental.  In  
match,   to   which   he   was   paired   with   an   individual   with   the   same   general   height   and   boxing   as   in   other   equally   physically   rigorous   sports,   such   as   basketball   or  
weight.   During,   the   fight   Eduardo   slipped   simultaneously   his   opponent   threw   a   baseball,   death   is   not   ordinarily   anticipated   to   result.   If,   therefore,   it   ever   does,  
punch   to   which   resulted   in   him   getting   punched   at   the   back   of   the   head.   After   the   injury   or   death   can   only   be   accidental   or   produced   by   some   unforeseen  
which  he  was  brought  to  the  hospital  and  died  the  very  next  day.     happening  or  event  as  what  occurred  in  this  case  
   
Simon  de  la  Cruz,  the  father  of  the  insured  and  who  was  named  beneficiary  under   DOCTRINE   2:   Failure   of   the   defendant   insurance   company   to   include   death  
the   policy,   thereupon   filed   a   claim   with   Capital   for   payment   of   the   indemnity   under   resulting   from   a   boxing   match   or   other   sports   among   the   prohibitive   risks   leads  
the   insurance   policy.   As   the   claim   was   denied,Simon   instituted   the   action   in   the   inevitably   to   the   conclusion   that   it   did   not   intend   to   limit   or   exempt   itself   from  
Court   of   First   Instance   of   Pangasinan   for   specific   performance.   The   defense   of   liability  for  such  death  
Capital  was  based  on  the  fact  that  the  death  of  Eduardo  was  not  accidental  as  the    
death  was  caused  by  his  participation  in  the  boxing  match.     DOCTRINE   3:   In   other   words,   where   the   death   or   injury   is   not   the   natural   or  
  probable  result  of  the  insured's  voluntary  act,  or  if  something  unforeseen  occurs  
The   terms   "accident"   and   "accidental",   as   used   in   insurance   contracts,   have   not   in  the  doing  of  the  act  which  produces  the  injury,  the  resulting  death  is  within  the  
acquired   any   technical   meaning,   and   are   construed   by   the   courts   in   their   ordinary   protection  of  policies  insuring  against  death  or  injury  from  accident.  
and   common   acceptation.   Thus,   the   terms   have   been   taken   to   mean   that   which    
happen   by   chance   or   fortuitously,   without   intention   and   design,   and   which   is   FACTS:  
unexpected,   unusual,   and   unforeseen.   An   accident   is   an   event   that   takes   place   1.   This  is  an  appeal  by  the  Capital  Insurance  &  Surety  Company,  Inc.,  from  the  
without   one's   foresight   or   expectation   —   an   event   that   proceeds   from   an   unknown   decision   of   the   Court   of   First   Instance   of   Pangasinan   (in   Civ   Case   No.   U-­‐
cause,  or  is  an  unusual  effect  of  a  known  cause  and,  therefore,  not  expected   265),   ordering   it   to   indemnify   therein   plaintiff   Simon   de   la   Cruz   for   the  
  death  of  the  latter's  son,  to  pay  the  burial  expenses,  and  attorney's  fees.  
WoN  Eduardo’s  participation  in  the  boxing  match  which  led  to  his  death  was  not   2.   Eduardo  de  la  Cruz,  employed  as  a  mucker  in  the  Itogon-­‐Suyoc  Mines,  Inc.  
accidental?   Capital   is   wrong,   it   was   accidental.   It   is   not   denied,   that   Eduardo’s   in  Baguio,  was  the  holder  of  an  accident  insurance  policy  (No.  ITO-­‐BFE-­‐170)  
slipping   during   the   boxing   match,   was   accidental.   In   the   present   case,   Eduardo’s   underwritten   by   the   Capital   Insurance   &   Surety   Co.,   Inc.,   for   the   period  
participation   in   the   boxing   match   was   voluntary   the   injury   that   caused   Eduardo’s   beginning  November  13,  1956  to  November  12,  1957.  
death  was  accidental.  Without  this  unfortunate  incident,  that  is,  the  unintentional   3.    On  January  1,  1957,  in  connection  with  the  celebration  of  the  New  Year,  
slipping   of   the   deceased,   perhaps   he   could   not   have   received   that   blow   in   the   head   the   Itogon-­‐Suyoc   Mines,   Inc.   sponsored   a   boxing   contest   for   general  
and   would   not   have   died.   The   fact   that   boxing   is   attended   with   some   risks   of   entertainment  wherein  the  insured  Eduardo  de  la  Cruz,  a  non-­‐professional  
external   injuries   does   not   make   any   injuries   received   in   the   course   of   the   game   boxer  participated.  
not   accidental.   In   boxing   as   in   other   equally   physically   rigorous   sports,   such   as   4.   In  the  course  of  his  bout  with  another  person,  likewise  a  non-­‐professional,  
basketball  or  baseball,  death  is  not  ordinarily  anticipated  to  result.  If,  therefore,  it   of   the   same   height,   weight,   and   size,   Eduardo   slipped   and   was   hit   by   his  
ever   does,   the   injury   or   death   can   only   be   accidental   or   produced   by   some   opponent  on  the  left  part  of  the  back  of  the  head,  causing  Eduardo  to  fall,  
unforeseen  happening  or  event  as  what  occurred  in  this  case.   with   his   head   hitting   the   rope   of   the   ring.   He   was   brought   to   the   Baguio  
   
General   Hospital   the   following   day.   The   cause   of   death   was   reported   as   received  in  the  course  of  the  game  not  accidental.  In  boxing  as  in  other  
hemorrhage,  intracranial,  left.   equally  physically  rigorous  sports,  such  as  basketball  or  baseball,  death  is  
5.   Simon  de  la  Cruz,  the  father  of  the  insured  and  who  was  named  beneficiary   not  ordinarily  anticipated  to  result.  If,  therefore,  it  ever  does,  the  injury  
under  the  policy,  thereupon  filed  a  claim  with  the  insurance  company  for   or   death   can   only   be   accidental   or   produced   by   some   unforeseen  
payment   of   the   indemnity   under   the   insurance   policy.   As   the   claim   was   happening   or   event   as   what   occurred   in   this   case.   And   considering   that   it  
denied,   De   la   Cruz   instituted   the   action   in   the   Court   of   First   Instance   of   was   not   an   accident   Capital   failed   to   exclude   such   sport   event   from   its  
Pangasinan  for  specific  performance.  Defendant  insurer  set  up  the  defense   policy.     Failure   of   the   defendant   insurance   company   to   include   death  
that   the   death   of   the   insured,   caused   by   his   participation   in   a   boxing   resulting  from  a  boxing  match  or  other  sports  among  the  prohibitive  risks  
contest,  was  not  accidental  and,  therefore,  not  covered  by  insurance.  After   leads  inevitably  to  the  conclusion  that  it  did  not  intend  to  limit  or  exempt  
due  hearing  the  court  rendered  the  decision  in  favor  of  the  plaintiff  which   itself  from  liability  for  such  death  
is  the  subject  of  the  present  appeal.    
6.   It  is  not  disputed  that  during  the  ring  fight  with  another  non-­‐professional   RULING:   Wherefore,   in   view   of   the   foregoing   considerations,   the   decision  
boxer,   Eduardo   slipped,   which   was   unintentional.   At   this   opportunity,   his   appealed  from  is  hereby  affirmed,  with  costs  against  appellant.  so  ordered.  
opponent   landed   on   Eduardo's   head   a   blow,   which   sent   the   latter   to   the    
ropes.   That   must   have   caused   the   cranial   injury   that   led   to   his   death.   RATIO:  
Eduardo   was   insured   "against   death   or   disability   caused   by   accidental   1.   even  if  we  take  appellant's  theory,  the  death  of  the  insured  in  the  case  at  
means".   Appellant   insurer   now   contends   that   while   the   death   of   the   bar   would   still   be   entitled   to   indemnification   under   the   policy.   The  
insured   was   due   to   head   injury,   said   injury   was   sustained   because   of   his   generally   accepted   rule   is   that,   death   or   injury   does   not   result   from  
voluntary  participation   in  the  contest.  It   is   claimed  that  the  participation  in   accident   or   accidental   means   within   the   terms   of   an     accident-­‐policy   if   it   is  
the   boxing   contest   was   the   "means"   that   produced   the   injury   which,   in   the   natural   result   of   the   insured's   voluntary   act,   unaccompanied   by  
turn,  caused  the  death  of  the  insured.  And,  since  his  inclusion  in  the  boxing   anything  unforeseen  except  the  death  or  injury.  
card  was  voluntary  on  the  part  of  the  insured,  he  cannot  be  considered  to   2.   There   is   no   accident   when   a   deliberate   act   is   performed   unless   some  
have  met  his  death  by  "accidental  means"   additional,   unexpected,   independent,   and   unforeseen   happening   occurs  
7.   The   terms   "accident"   and   "accidental",   as   used   in   insurance   contracts,   which  produces  or  brings  about  the  result  of  injury  or  death.  
have  not  acquired  any  technical  meaning,  and  are  construed  by  the  courts   3.   In  other  words,  where  the  death  or  injury  is  not  the  natural  or  probable  
in   their   ordinary   and   common   acceptation.   Thus,   the   terms   have   been   result  of  the  insured's  voluntary  act,  or  if  something  unforeseen  occurs  in  
taken   to   mean   that   which   happen   by   chance   or   fortuitously,   without   the   doing   of   the   act   which   produces   the   injury,   the   resulting   death   is  
intention   and   design,   and   which   is   unexpected,   unusual,   and   unforeseen.   within   the   protection   of   policies   insuring   against   death   or   injury   from  
An   accident   is   an   event   that   takes   place   without   one's   foresight   or   accident.  
expectation   —   an   event   that   proceeds   from   an   unknown   cause,   or   is   an   4.   In   the   present   case,   while   the   participation   of   the   insured   in   the   boxing  
unusual  effect  of  a  known  cause  and,  therefore,  not  expected.   contest   is   voluntary,   the   injury   was   sustained   when   he   slid,   giving  
8.   Appellant  however,  would  like  to  make  a  distinction  between  "accident  or   occasion  to  the  infliction  by  his  opponent  of  the  blow  that  threw  him  to  
accidental"   and   "accidental   means",   which   is   the   term   used   in   the   the   ropes   of   the   ring.   Without   this   unfortunate   incident,   that   is,   the  
insurance   policy   involved   here.   It   is   argued   that   to   be   considered   within   unintentional  slipping  of  the  deceased,  perhaps  he  could  not  have  received  
the  protection  of  the  policy,  what  is  required  to  be  accidental  is  the  means   that   blow   in   the   head   and   would   not   have   died.   The   fact   that   boxing   is  
that   caused   or   brought   the   death   and   not   the   death   itself.   It   may   be   attended  with  some  risks  of  external  injuries  does  not  make  any  injuries  
mentioned  in  this  connection,  that  the  tendency  of  court  decisions  in  the   received  in  the  course  of  the  game  not  accidental.  In  boxing  as  in  other  
United   States   in   recent   years   is   to   eliminate   the   fine   distinction   between   equally  physically  rigorous  sports,  such  as  basketball  or  baseball,  death  is  
the   terms   "accidental"   and   "accidental   means"   and   to   consider   them   as   not  ordinarily  anticipated  to  result.  If,  therefore,  it  ever  does,  the  injury  
legally  synonymous   or   death   can   only   be   accidental   or   produced   by   some   unforeseen  
ISSUE/s:   happening  or  event  as  what  occurred  in  this  case.    
1.   WoN  Eduardo’s  participation  in  the  boxing  match  which  led  to  his  death   5.   Furthermore,   the   policy   involved   herein   specifically   excluded   from   its  
was   not   accidental?   No,   it   was   accidental.     The   fact   that   boxing   is   coverage  —  
attended  with  some  risks  of  external  injuries  does  not  make  any  injuries  
a.   (e)   Death   or   disablement   consequent   upon   the   Insured  engaging  
in   football,   hunting,   pigsticking,   steeplechasing,   polo-­‐playing,  
racing  of  any  kind,  mountaineering,  or  motorcycling.  
6.   Death   or   disablement   resulting   from   engagement   in   boxing   contests   was  
not   declared   outside   of   the   protection   of   the   insurance   contract.   Failure   of  
the  defendant  insurance  company  to  include  death  resulting  from  a  boxing  
match   or   other   sports   among   the   prohibitive   risks   leads   inevitably   to   the  
conclusion  that  it  did  not  intend  to  limit  or  exempt  itself  from  liability  for  
such  death.  
         
       
     
 
   
008  BIAGTAN  v.  INSULAR  LIFE  (LAGUILLES)   resulted directly from bodily injury effected solely through external and
March  29,  1972  |  Makalintal,  J.  |  Casualty  Insurance   violent means sustained in an accident, and independently of all other
  causes.”
PETITIONER:   Emilia   Biagtan,   Juan   Biagtan,   Miguel   Biagtan,   Gil   Biagtan   and   24.   The clause, however, expressly provided that it would not apply where
Gracia  Biagtan   death resulted from an injury intentionally inflicted by a third party.
RESPONDENTS:  Insular  Life  Assurance  Company     25.   On the night of May 1964, a band of robbers entered the house of Biagtan
where they committed robbery upon reaching the staircase landing of the
 
second floor, rushed towards the doors of the second floor room where they
SUMMARY:  Biagtan  was  insured  with  Insular  Life  for  P5,000,  which  included  an  
suddenly met Biagtan who received thrusts from their sharp-pointed
accidental  death  benefit  clause  for  an  additional  P5,000  which  can  be  claimed  
instruments, resulting to his death.
if   the   death   of   the   insured   was   caused   solely   through   external   and   violent   26.   Plaintiffs as beneficiaries of Biagtan, filed a claim under the policy. Insular
means  sustained  in  an  accident.  The  accidental  death  benefit  clause  provided   Life paid the basic amount of P5,000, but refused to pay the additional sum
that   the   clause   will   not   apply   if   the   death   was   due   to   an   injury   intentionally   of P5,000 under the accidental death benefit clause, on the ground that
inflicted  by  a  third  party.  Biagtan’s  house  was  then  robbed,  and  in  the  course   Biagtan’s death resulted from injuries intentionally inflicted by third parties
of   the   robbery,   the   band   of   robbers   stabbed   him,   which   caused   Biagtan’s   and therefore was not covered.
death.   Biagtan’s   beneficiaries   then   claimed   from   Insular   Life,   and   the   latter   27.   Plaintiffs filed suit to recover, and the court rendered a decision in their
paid   the   basic   amount   of   P5,000,   without   the   additional   P5,000   from   the   favor.
accidental   death   benefit.   Insular   Life   contends   that   Biagtan’s   death   resulted   ISSUE:  
from   injuries   intentionally   inflicted   by   third   parties,   and   therefore   was   not   1.   WoN the wounds received by the insured at the hands of the robbers were
covered.   The   lower   court   rendered   a   decision   in   favor   the   beneficiaries.   The   inflicted intentionally, so as to bar recovery under the accidental death
issue  is  WoN  the  wounds  received  by  the  insured  were  intentionally  inflicted,   benefit clause – Yes, because the case was decided based on stipulation, and
so  as  to  bar  recovery  under  the  accidental  death  benefit  clause.     there was no proof that the act was intentionally done or merely to scare
  away Biagtan.
The   SC   held   in   the   affirmative.   The   parties   presented   no   evidence   and    
submitted   the   case   upon   stipulation,   so   there   was   no   proof   that   the   injuries   RULING:   WHEREFORE,   the   decision   appealed   from   is   reversed   and   the   complaint  
were  intentionally  inflicted,  were  done  merely  to  scare  away  Biagtan  to  ward   dismissed,  without  pronouncement  as  to  costs.    
off   any   resistance   on   his   part.   Whether   the   robbers   had   the   intent   to   kill   or    
merely   to   scare   away   the   victim,   it   cannot   be   denied   that   the   act   itself   of   RATIO:    
inflicting   the   injuries   was   intentional.   The   exception   in   the   accidental   death   14.   Since the parties presented no evidence and submitted the case upon
benefit   clause   does   not   speak   of   the   purpose   of   a   third   party   causing   the   stipulation, there was no proof that the act of receiving thrusts from the
sharp-pointed instrument of the robbers was intended to inflict injuries upon
injuries,  but  only  of  the  fact  that  such  injuries  have  been  intentionally  inflicted.  
the person of Biagtan, or merely to scare away any person so as to ward off
Where   a   gang   of   robbers   enter   a   house   and   coming   face   to   face   with   the  
any resistance or obstacle that might be offered in the pursuit of their main
owner,   even   if   unexpectedly,   stab   him   repeatedly,   it   is   contrary   to   all   reason   objective, which was robbery.
and  logic  to  say  that  his  injuries  are  not  intentionally  inflicted.     15.   Nine wounds were inflicted upon the deceased, all by means of thrusts with
  sharp pointed instruments. This is a physical fact as to which there is no
DOCTRINE:   The   exception  in  the  accidental  benefit  clause  invoked  by  the  heirs   dispute. So is the fact that five of those wounds caused the death of the
of   Biagtan   does   not   speak   of   the   purpose   –   whether   homicidal   or   not   –   of   a   insured.
third  party  in  causing  the  injuries,  but  only  of  the  fact  that  such  injuries  have   16.   Whether the robbers had the intent to kill or merely to scare the victim or to
been  “intentionally”  inflicted  –  this  obviously  to  distinguish  them  from  injuries   ward off any defense he might offer, it cannot be denied that the act itself of
which,  although  received  at  the  hands  of  a  third  party,  are  purely  accidental.     inflicting the injuries was intentional.
  17.   It should be noted that the exception in the accidental benefit clause
FACTS:     invoked by the heirs of Biagtan does not speak of the purpose – whether
23.   Juan S. Biagtan was insured with Insular Life Assurance Company for homicidal or not – of a third party in causing the injuries, but only of the
P5,000 and, under a supplementary contract denominated “Accidental fact that such injuries have been “intentionally” inflicted – this obviously to
Death Benefit Clause,” for an additional P5,000 if the “death of the insured distinguish them from injuries which, although received at the hands of a
third party, are purely accidental.
18.   Where a gang of robbers enter a house and coming face to face with the
owner, even if unexpectedly, stab him repeatedly, it is contrary to all reason
and logic to say that his injuries are not intentionally inflicted.
 
Dissenting  opinion,  Teehankee:  
1.   The insurance company is liable to the beneficiaries under its accidental
death benefit clause, because the case of Calanoc cited by the lower court is
indeed controlling. The Court held that fatal injuries inflicted upon an
insured by a malefactor during the latter’s commission of a crime are
deemed accidental and the burden of proving that the killing was intentional
must be discharged by the insurance company.
2.   The insurance company failed to discharge its burden of proving that the
injuries were inflicted intentionally. The lower court correctly held that
since the case was submitted upon the parties’ stipulation of facts which did
not cover the malefactors’ intent at all, there was an utter absence of
evidence as to the real intention of the malefactors.
3.   Also, the beneficiaries properly assert in their brief the sole error assigned
by the insurance company, that the fatal injuries were not accidental but
should be held to have been intentionally inflicted, raises a question of fact
– which insurance company is now barred from raising, since it expressly
limited its appeal to the SC purely on questions of law.
4.   The accidental death benefit clause assuring the insured’s beneficiaries of
double indemnity, upon payment of an extra premium, in the event that the
insured meets violent accidental death is contractually stipulated in the
policy.
5.   The only exception which is NOT susceptible of classification is provided
also in the policy, which excepts injuries inflicted intentionally by a third
patty, either with or without provocation on the part of the insure, and
whether or not the attack or the defense by the third party was caused by a
violation of the law by the insured.
 
   
009 PHIL-NIPPON KYOEI, CORP. v. GUDELOSAO (Marcos) such as fire or marine. It includes, but is not limited to, employer's liability
July 13, 2016 | Jardeleza, J. | Casualty Insurance insurance, motor vehicle liability insurance, plate glass insurance, burglary and
theft insurance, personal accident and health insurance as written by non-life
PETITIONER: Phil-Nippon Kyoei, Corp. insurance companies, and other substantially similar kinds of insurance.
RESPONDENTS: Rosalia T. Gudelosao, on her behalf and in behalf of minor
children Christy Mae T. Gudelosao and Rose Elden T. Gudelosao, Carmen  
Tancontian, on her behalf and in behalf of the children Camela B. Tancontian, FACTS:
Beverly B. Tancontian, And Ace B. Tancontian 123.  Phil-Nippon purchased a “Ro-Ro” passenger/cargo vessel named “MV
Mahlia” in Japan.
SUMMARY: Phil-Nippon purchased a “Ro-Ro” passenger/cargo vessel named 124.  For the vessel’s one month conduction voyage from Japan to the PH, Phil-
“MV Mahlia” in Japan. For its conduction voyage, Phil-Nippon and TMLC hired Nippon and Top Ever Marine Management Maritime Co, Ltd. (TMLC)
crew members through the latter’s local manning agency, TEMMPC. Phil- hired crew members through the latter’s local manning agency, Top Ever
Nippon secured a Marine Insurance Policy from SSSICI over the vessel against Marine Management Philippine Corporation (TEMMPC).
loss, damage, and third party liability or expense arising from the occurrence of 125.  Edwin Gudelasao and Virgilio A. Tancontian, and six other crewmembers
the perils of the sea for the voyage of the vessel. The insurance policy included were hired.
Personal Accident Policies for the 8 crewmembers in case of accidental death or 126.  TEMMPC, through Capt. Orbeta, and the 8 crewmembers signed separate
injury. While still within Japanese waters, the vessel sank due to extreme bad contracts of employment.
weather conditions. Only one survived the incident while the rest of the 127.  Phil-Nippon secured a Marine Insurance Policy from South Sea Surety &
crewmembers perished. The heirs and beneficiaries of Gudelasao and Insurance Co., Inc. (SSSICI) over the vessel against loss, damage, and third
Tancontian filed separate complaints for death benefits and other damages party liability or expense arising from the occurrence of the perils of the sea
against Phil-Nippon, TEMMPC, Capt. Orbeta, TMCL, and SSSICI with the for the voyage of the vessel.
NLRC. The Labor Arbiter rendered a Decision finding solidary liability among 128.  The insurance policy included Personal Accident Policies for the 8
Phil-Nippon, TEMMPC, TMCL, and Capt. Orbeta. It also found SSSICI liable crewmembers in case of accidental death or injury.
to the heirs for the proceeds of the Personal Accident Policies and attorney’s 129.  On Feb. 24, 2003, while still within Japanese waters, the vessel sank due to
fees. On appeal, the NLRC modified the LA Decision. The heirs then filed a extreme bad weather conditions. Only one survived the incident while the
petition for certiorari before the CA and the CA granted the petition. The CA rest of the crewmembers perished.
held that TEMMPC and TMCL are liable under POEA-SEC, while Phil-Nippon 130.  The heirs and beneficiaries of Gudelasao and Tancontian filed separate
is not liable under the POEA-SEC, but by virtue of its being a shipowner. complaints for death benefits and other damages against Phil-Nippon,
Further, SSSICIS’s liability as Phil-Nippon’s insurer directly arose from the TEMMPC, Capt. Orbeta, TMCL, and SSSICI with the NLRC.
contract of insurance against liability. The CA then ordered that Phil-Nippon’s 131.  The Labor Arbiter rendered a Decision finding solidary liability among
liability will only be extinguished upon payment by SSSICI of the insurance Phil-Nippon, TEMMPC, TMCL, and Capt. Orbeta. It also found SSSICI
proceeds. The issue before the Court is WoN the CA erred in ruling that the liable to the heirs for the proceeds of the Personal Accident Policies and
liability of Phil-Nippon is extinguished only upon SSSICI's payment of attorney’s fees.
insurance proceeds. – YES. Because although the Personal Accident Policies are 132.  The LA, however, ruled that the liability of Phil-Nippon shall be deemed
casualty insurance, they do not answer for petitioner's liabilities arising from the extinguished only upon SSSICI’s payment of the insurance proceeds.
sinking of the vessel. It is an indemnity insurance procured by petitioner for the 133.  On appeal, the NLRC modified the LA Decision. It absolved Phil-Nippon,
benefit of the seafarers. As a result, petitioner is not directly liable to pay under TEMMPC, TMLC, and Capt. Orbeta from any liability based on the limited
the policies because it is merely the policyholder of the Personal Accident liability rule. It however, affirmed SSSICI’s liability after finding that the
Policies. Based on the pleadings submitted by the parties, the insurer itself Personal Accident Policies answer for the death benefit claims under the
admits that what is being insured against is not the liability of the shipowner for Philippine Overseas Employment Administration Standard Employment
death or injuries to passengers but the death of the seafarers arising from Contract (POEA-SEC)
accident. 134.  The heirs filed a Parital MR but the NLRC denied it in a Resolution.
135.  The heirs then filed a petition for certiorari before the CA where they
DOCTRINE: Casualty insurance is insurance covering loss or liability arising argued that the NLRC gravely abused its discretion.
from accident or mishap, excluding certain types of loss which by law or custom 136.  The CA granted the petition and set aside the decision of the NLRC.
are considered as falling exclusively within the scope of other types of insurance 137.  The CA found that the NLRC erred when it ruled that the obligation of Phil-
Nippon, TEMMPC and TMCL for the payment of death benefits under the 18.   [INSURANCE] WoN the CA erred in ruling that the liability of Phil-
POEA-SEC was ipso facto transferred to SSSICI upon the death of the Nippon is extinguished only upon SSSICI's payment of insurance proceeds.
seafarers. Its liability under the POEA-SEC is separate and distinct from the – YES. Because although the Personal Accident Policies are casualty
liability of the shipowner. (liable daw TEMMPC and TMCL under POEA- insurance, they do not answer for petitioner's liabilities arising from the
SEC) sinking of the vessel. It is an indemnity insurance procured by petitioner for
138.  However, the CA ruled that Phil-Nippon is not liable under the POEA-SEC, the benefit of the seafarers. As a result, petitioner is not directly liable to
but by virtue of its being a shipowner. (dito naman liable Phil-Nippon as a pay under the policies because it is merely the policyholder of the Personal
shipowner pero hindi under POEA-SEC) Accident Policies.
139.  Thus, it is liable for the injuries to the passengers even without a
determination of its fault or negligence. SSSICIS’s liability as Phil- RULING: WHEREFORE, the petition is PARTLY GRANTED . The CA Decision
Nippon’s insurer directly arose from the contract of insurance against dated October 4, 2007 and the Resolution dated January 11, 2008 of the Court of
liability. The CA then ordered that Phil-Nippon’s liability will only be Appeals are AFFIRMED WITH THE FOLLOWING MODIFICATIONS:
extinguished upon payment by SSSICI of the insurance proceeds. (1)   The death benefits are limited to the amount granted under the Release of
140.  Phil-Nippon filed an MR which was denied by the CA. SSSICI did not file All Rights and Full Satisfaction of Claim dated December 14, 2007
a MR therefore the CA decision is final and executory as to it. executed between respondents and Top Ever Marine Management
141.  Hence, this petition. Company Ltd., Top Ever Marine Management Philippine Corporation, and
142.  PETITIONER’S ARGUMENT: Phil-Nippon claims that the CA erred in Captain Oscar Orbeta;
ignoring the fundamental rule in Maritime Law that the shipowner may (2)   As a solidary co-debtor, petitioner's liability to respondents under the
exempt itself from liability by abandoning the vessel and freight it may POEA-SEC is also extinguished by virtue of the Release of All Rights and
have earned during the voyage, and the proceeds of the insurance of any Full Satisfaction of Claim dated December 14, 2007; and
that it may have earned during the voyage, and the proceeds of the (3)   The last paragraph of the dispositive portion of the CA Decision dated
insurance if any. Since the liability of the shipowner is limited to the value October 4, 2007 stating: "Nevertheless, upon payment of said proceeds
of the vessel unless there is insurance, any claim against petitioner is limited to said widows by respondent SOUTH SEA SURETY & INSURANCE
to the proceeds arising from the insurance policies procured from SSSICI. CO., INC., respondent PHIL-NIPPON CORPORATION's liability to
Thus, there is no reason in making petitioner's exoneration from liability all the complainants is deemed extinguished. . ." is DELETED.
conditional on SSSICI's payment of the insurance proceeds.
143.  SSSICI filed a comment alleging that the NLRC has no jurisdiction over the RATIO:
insurance claim because claims on the Personal Accident Policies did not
arise from employer-employee relations. It also alleged that a separate case First Issue:
was filed against it for a sum of money in the RTC praying for the payment LIABILITY UNDER THE POEA STANDARD EMPLOYMENT CONTRACT
of proceeds under the individual Marine Insurance Policy with a Personal 138.  At the outset, the CA erred in absolving Phil-Nippon from the liabilities
Accident Policy covering the crewmembers of MV Mahlia. This case was under the POEA-SEC.
eventually dismissed and is now subject of an appeal before the CA. SSSICI 139.  It was the local principal of the deceased seafarers for the conduction trip of
prays that this matter be considered in resolving the present case. MV Mahlia. It hired them through TMCL, which also acted through its
144.  TEMMPC filed its Manifestation informing us of TEMMPC and TMCL's agent, TEMMPC. It admitted its role as a principal of its agents TMCL,
Joint Motion to Dismiss the Petition and the CA's Resolution granting it. TEMMPC and Capt. Orbeta in their Joint Partial Appeal before the NLRC.
The dismissal is based on the execution of the Release of All Rights and As such, it is solidarily liable with TEMMPC and TMCL for the benefits
Full Satisfaction Claim (Release and Quitclaim) between respondents and under the POEA-SEC.
TEMMPC, TMCL, and Capt. Orbeta. 140.  The doctrine of limited liability is not applicable to claims under POEA-
SEC.
ISSUE/s: 141.  The claim for death benefits under the POEA-SEC is the same species as
17.   WoN the doctrine of real and hypothecary nature of maritime law (also the workmen's compensation claims under the Labor Code — both of which
known as the limited liability rule) applies in favor of Phil-Nippon. – NO belong to a different realm from that of Maritime Law. Therefore, the
because the claim for death benefits under the POEA-SEC is the same limited liability rule does not apply to petitioner's liability under the POEA-
species as the workmen's compensation claims under the Labor Code — SEC.
both of which belong to a different realm from that of Maritime Law. 142.  Nevertheless, the Release and Quitclaim benefit Phil-Nippon as a solidary
debtor. that the liabilities insured against are all monetary claims, excluding the
143.  We have consistently applied the Civil Code provisions on solidary bene ts under the POEA-SEC, of respondents in connection with the sinking
obligations, specifcally Articles 1217 and 1222, to labor cases. of the vessel.
144.  Considering that Phil-Nippon is solidarily liable with TEMMPC and 10.   We rule that while the Personal Accident Policies are casualty insurance,
TMCL, we hold that the Release and Quitclaim executed by the heirs in they do not answer for petitioner's liabilities arising from the sinking of the
favor of TEMMPC and TMCL redounded to the former's benefit. vessel. It is an indemnity insurance procured by petitioner for the
Accordingly, the liabilities of Phil-Nippon under the POEA-SEC to the benefit of the seafarers. As a result, Phil-Nippon is not directly liable to
heirs are now deemed extinguished. We emphasize, however, that this pay under the policies because it is merely the policyholder of the
pronouncement does not foreclose the right of reimbursement of the Personal Accident Policies.
solidary debtors who paid (i.e., TEMMPC and TMCL) from Phil-Nippon as 11.   Section 176 (formerly Sec. 174) of The Insurance Code defines casualty
their co-debtor. insurance as follows:
SEC. 174. Casualty insurance is insurance covering loss or liability arising
from accident or mishap, excluding certain types of loss which by law or
Second Issue: [INSURANCE TOPIC] custom are considered as falling exclusively within the scope of other types of
LIABILITY UNDER THE PERSONAL ACCIDENT POLICIES insurance such as fire or marine. It includes, but is not limited to, employer's
1.   First, the CA correctly upheld the NLRC’s jurisdiction to order SSSICI to liability insurance, motor vehicle liability insurance, plate glass insurance, burglary
pay the heirs the value of the proceeds of the Personal Accident Policies. and theft insurance, personal accident and health insurance as written by non-life
insurance companies, and other substantially similar kinds of insurance.
2.   The Migrant Workers and Overseas Filipinos Act of 1995 gives the Labor
12.   Notably, the parties did not submit the Personal Accident Policies with the
Arbiters of the NLRC the original and exclusive jurisdiction over claims
NLRC or the CA. However, based on the pleadings submitted by the
arising out of an employer-employee relationship or by virtue of any law or
parties, SSSICI admitted that the crewmembers of MV Mahlia are insured
contract involving Filipino workers for overseas deployment, including
for the amount of P3,240,000.00, payable upon the accidental death of the
claims for actual, moral, exemplary and other forms of damage. It further
crewmembers. It further admitted that the insured risk is the loss of life or
creates a joint and several liability among the principal or employer, and the
bodily injury brought about by the violent external event or accidental
recruitment/placement agency, for any and all claims
means. Based on the foregoing, the insurer itself admits that what is
3.   Here, the award of the insurance proceeds arose out of the personal accident
being insured against is not the liability of the shipowner for death or
insurance procured by petitioner as the local principal over the deceased
injuries to passengers but the death of the seafarers arising from
seafarers who were Filipino overseas workers.
accident.
4.   The premiums paid by Phil-Nippon were, in actuality, part of the total
13.   The liability of SSSICI to the beneficiaries is direct under the insurance
compensation paid for the services of the crewmembers.
contract.
5.   Put differently, the labor of the employees is the true source of the benefitts
14.   Under the contract, petitioner is the policyholder, with SSSICI as the
which are a form of additional compensation to them. Undeniably, such
insurer, the crewmembers as the cestui que vie or the person whose life
claim on the personal accident cover is a claim under an insurance contract
is being insured with another as beneficiary of the proceeds, and the
involving Filipino workers for overseas deployment within the jurisdiction
latter's heirs as beneficiaries of the policies.
of the NLRC.
15.   Upon Phil-Nippon's payment of the premiums intended as additional
6.   In any event, SSSICI can no longer assail its liability under the Personal
compensation to the crewmembers, SSSICI as insurer undertook to
Accident Policies. SSSICI failed to file a motion for reconsideration on the
indemnify the crewmembers' beneficiaries from an unknown or
CA Decision. In a Resolution, the CA certified in a Partial Entry of
contingent event.
Judgment that the CA Decision with respect to SSSICI has become final
16.   Thus, when the CA conditioned the extinguishment of petitioner's
and executory and is recorded in the Book of Entries of Judgments.
liability on SSSICI's payment of the Personal Accident Policies'
7.   Thus, SSSICI's liability on the Personal Accident Policies can no longer be
proceeds, it made a finding that petitioner is subsidiarily liable for the
disturbed in this petition.
face value of the policies. To reiterate, however, there is no basis for
8.   [IMPORTANT UNDER CASUALTY INSURANCE] The CA
such finding; there is no obligation on the part of Phil-Nippon to pay
determined that the Personal Accident Policies are casualty insurance,
the insurance proceeds because it is, in fact, the obligee or policyholder
specifically one of liability insurance.
in the Personal Accident Policies.
9.   The CA determined that Phil-Nippon, as insured, procured from SSSICI the
17.   Since it is not the party liable for the value of the insurance proceeds, it
Personal Accident Policies in order to protect itself from the consequences
follows that the limited liability rule does not apply as well.
of the total loss of the vessel caused by the perils of the sea. The CA found
   
010 Mercantile Insurance v. Felipe Ysmael, Jr & Co (Matsumura) FACTS:
January 13, 1989 | Bidin, J. | Suretyship 145.  Felipe Ysmael Jr., & Co., Inc (Felipe Co) filed an application for an
overdraft line of 1 million pesos, and a credit line of 1 million pesos with
PETITIONER: Mercantile Insurance Co., Inc Philippine National Bank (PNB).
RESPONDENTS: Felipe Ysmael, Jr, & Co, Inc. 146.  PNB was willing to grant the credit accommodation provided that Felipe Co
file a bond in the sum of P140,000 to guarantee the payment of the credit.
SUMMARY: Felipe Ysmael Jr., & Co (Felipe Co) applied for an overdraft line 147.  Felipe Co, represented by Felipe Ysmael, filed a surety bond (Surety bond
and credit line totaling to 2 million pesos with PNB. PNB granted the same upon 1) with Mercantile Insurance in the sum of P100,000.
the condition that Felipe Co would file a surety bond worth P140,000 –which 148.  Felipe Co and Mercantile Insurance executed another surety bond (Surety
was later decreased to P100,000. Hence, Felipe Co executed 2 surety bonds with bond 2) in the sum of P40,000.
Mercantile Insurance totaling to P100,000. As surety and in consideration of the 149.  Under both surety agreements, it was stipulated that if Felipe Co performs
surety bonds, Felipe Co and Magdalena Estate (represented by Felipe Ysmael in and fulfills its undertakings with PNB, then the surety bonds would be null
his capacity as President and in his personal capacity) executed an indemnity and void.
agreement per surety bond. Under the said agreement, it was stipulated that that 150.  As security and in consideration of the surety bonds, Felipe Co and
indemnification can be claimed by Mercantile Insurance irrespective of whether Magdalena Estate –represented by Felipe Ysmael as president and in his
or not Mercantile actually paid to PNB. personal capacity –executed an indemnity agreement. In this agreement, it
was stated that Felipe Co and Felipe Ysmael bound themselves jointly and
Felipe Co failed to pay PNB, so PNB sent a demand letter to Mercantile severally to indemnify Mercantile Insurance for any and all payments it
Insurance asking it to pay the P1000,000 surety. Mercantile Insurance then sent a shall incur or become liable to pay as surety in surety bond 1.
letter to Felipe Co demanding that it pay in accordance with the indemnity 151.  Gabriel Daza Jr, Edgardo Trodesillas, and Augusto Torres in their official
agreement. Felipe Co didn’t want to pay, so Mercantile Insurance filed a civil capacities, and Felipe Co executed another indemnity agreement with
case against Felipe Co and Felipe Ysmael. The RTC ordered Felipe Co and Mercantile Insurance in consideration of surety bond 2. This agreement had
Felipe Ysmael to pay Mercantile, and that upon payment Mercantile should pay the same stipulation as the first indemnity agreement (See Fact #6).
PNB. This decision was appealed to the CA, but because it was a question of law 152.  In September 5, 1957, the bond of P140,000 (in fact #2) was reduced to
it was directed to the SC. P100,000
a.   P60,000 on surety bond 1
b.   P40,000 on surety bond 2
The  issue  is  W/N  Mercantile  can  be  allowed  indemnification  upon  Felipe  
153.  Felipe Co failed to pay the overdraft and credit line. Thus, PNB demanded
Co   and   Felipe   Ysmael’s   default   even   if   the   former   hasn’t   paid   PNB   yet.   Mercantile Insurance to pay P100,000 pursuant to the surety bonds -60K
The   SC   ruled   YES.   It   is   settled   that   the   parties   may   establish   such   from surety bond 1; 40k from surety bond 2.
stipulations,  clauses,  terms,  and  conditions  as  they  may  want  to  include,   154.  Thereafter the Legal Department of Mercantile Insurance wrote a letter of
as   long   as   they   are   not   contrary   to   law,   morals,   good   customs,   public   demand to Felipe Co
policy,   or   public   order.   In   the   case   at   bar,   the   stipulation   was   freely   a.   inviting their attention to the letter of demand of PNB; and
b.   demanding that Felipe Co settle the account.
entered   by   the   parties.   The   next   question   then   is   whether   this  
155.  When Felipe Co failed to settle their obligation with PNB, Mercantile
stipulation  is  valid.  Jurisprudence  has  provided  that  a  stipulation  in  the   Insurance filed a civil case against Felipe Co. and Felipe Ysmael (the case
indemnity  agreement  allowing  the  surety  to  recover  even  before  it  paid   didn’t say what exactly the civil case was)
the  creditor  is  enforceable.  Thus,  the  indemnity  agreement  has  the  force   156.  [Fact #12 –15 are civpro aspects, so you can skip if you want] Instead of
and  effect  of  law   filing an answer, Felipe Co and Felipe Ysmael filed a motion to dismiss
which was denied.
157.  Thereafter Felipe Co and Felipe Ysmael filed an answer, and the case was
DOCTRINE:   The   stipulation   in   the   indemnity   agreement   allowing   the   set for pre-trial.
surety   to   recover   even   before   it   paid   the   creditor   is   enforceable.   In   158.  During the pre-trial, Felipe Co and Felipe Ysmael and its counsel failed to
accordance   therewith,   the   surety   may   demand   from   the   indemnitors   appear, hence upon motion, it was declared in default. Mercantile Insurance
presented its evidence ex parte.
even  before  paying  the  creditors.  
159.  Upon a MR by Felipe Co and Felipe Ysmael, the case was re-opened and
was scheduled for the reception of Felipe Co’s evidence. Thereafter both 4.   Contracts are respected as the law between the contracting parties. It
parties were required to submit a memorandum. is settled that the parties may establish such stipulations, clauses,
160.  The RTC ruled that Felipe Co and Felipe Ysmael are solidarily liable to pay terms, and conditions as they may want to include, as long as they
Mercantile Insurance P100,000, plus 15% for attorney’s fees and costs. are not contrary to law, morals, good customs, public policy, or
Moreover, that upon payment by Felipe Co and Felipe Ysmael, Mercantile
public order.
Insurance shall deliver the money to PNB to satisfy Felipe Co’s obligation
to the bank. 5.   It is a basic and fundamental rule in the interpretation of contracts
161.  On appeal to the CA, the CA saw it as a question of law and directed the that if the terms thereof are clear and leave no doubt as to the
case to the SC. intention of the contracting parties, the literal meaning of the
162.  Hence, this petition. stipulation shall control.
163.   Felipe Co and Felipe Ysmael argue that 6.   In the case at bar, there is no dispute as to meaning of the terms of
a.   The stipulation that indemnification can be claimed by the Indemnity Agreement. The only bone of contention is whether
Mercantile Insurance irrespective of whether or not or not such terms are null and void.
Mercantile actually paid to PNB is void for being contrary to 7.   A careful analysis of the contract in question will show that the
public policy and good morals because to receive an provisions therein do not contravene any law or public policy
indemnity for something it has not paid in its capacity as much less do they militate against the public good.
surety would be tantamount to unjust enrichment. 8.   In fact, such stipulation is sanctioned by well-established
b.   A cause of action needs to have a wrongful violation and jurisprudence.
damages. That in this case, these are not present for 9.   Having voluntarily entered into such contract, the Felipe Co and
Mercantile Insurance suffered no injury. Felipe Ysmael cannot now be heard to complain. Their indemnity
agreement has the force and effect of law.
ISSUE/s: 10.   The indemnity agreement was not executed for the benefit of the
1.   W/N Mercantile Insurance can be allowed indemnification from creditors; it was rather for the benefit of the surety and if the latter
Felipe Co and Felipe Ysmael, upon the latter’s default even before thought it necessary in its own interest to impose this stipulation, and
Mercantile Insurnace has paid to the creditor PNB –YES. the indemnitors voluntarily agreed to the same, the court should
Jurisprudence has provided that such a stipulation is valid. Thus, respect the agreement of the parties and require them to abide by
when such is stated in a contract entered between the parties, it has their contract.
the force and effect of law between the parties. 11.   Thus, the lower court did not err in ordering Felipe Co and Felipe
Ysmael to pay jointly and severally the sum of P100,000.00 plus
RULING: WHEREFORE, the decision appealed from is hereby AFFIRMED.
15% as attorney's fees.
RATIO: 12.   It must be stressed that in the case at bar, the principal debtors are
1.   It has been held that in previous cases that "The stipulation in the simultaneously the same persons who executed the Indemnity
indemnity agreement allowing the surety to recover even before Agreement. Thus, the position occupied by them is that of a
it paid the creditor is enforceable. In accordance therewith, the principal debtor and indemnitor at the same time, and their
surety may demand from the indemnitors even before paying the liability being joint and several with the Mercantile Insurance,
creditors." PNB may proceed against either for fulfillment of the obligation
2.   Hence, the case is not premature, nor fails to state a cause of action. as covered by the Surety bonds.
3.   It is apparent that the cause of action was derived from the terms of 13.   There is, therefore, no principle of guaranty involved and, therefore,
the Indemnity agreement the provision of Article 2071 of the Civil Code does not apply.
a.   That Mercantile Insurnace may proceed against Felipe Co Otherwise stated, there is no more need for PNB to exhaust all the
and Felipe Ysmael to impose payment, even prior to making properties of the Felipe Co before it may proceed against Mercantile
payments to the PNB. Insurance.
14.   As to the attorney's fees, it has been squarely ruled by this Court that
the award of fifteen (15) per cent for cases of this nature is not
unreasonable
   
011  FIRST  LEPANTO-­‐TAISHO  INSURANCE  v.  CHEVRON  PH  (CHIQUI)   97.   Chevron Philippines, Inc., (Chevron) sued First Lepanto-Taisho Insurance
January  18,  2012  |  Villarama,  Jr.  J.  |  Contract  of  Surety  Strict  Interpretation   Corporation (First Lepanto) for the payment of unpaid oil and petroleum
  purchases made by its distributor Fumitechniks Corporation
(Fumitechniks).
PETITIONER:   First   Lepanto-­‐Taisho   Insurance   Corporation   (FLT   Prime   Insurance  
98.   Fumitechniks, represented by Ma. Lourdes Apostol, had applied for and
Corporation)    
was issued Surety Bond FLTICG (16) No. 01012 by First Lepanto for the
RESPONDENTS:  Chevron  Philippines,  Inc  (formerly  known  as  CALTEX)   amount of P15,700,000.00. As stated in the attached rider, the bond was in
compliance with the requirement for the grant of a credit line with Chevron
SUMMARY: Fumitechniks purchased oil and petroleum from Chevron. Because of "to guarantee payment/remittance of the cost of fuel products withdrawn
this, Fumitechniks applied for a surety bond from First Lepanto. Fumitechniks then within the stipulated time in accordance with the terms and conditions of
defaulted on its obligation since the check it issued was dishonored. Chevron the agreement." The surety bond was executed on October 15, 2001 and
demanded payment from First Lepanto. First Lepanto refused to pay and argued that will expire on October 15, 2002.
the principal agreement between Fumitechniks and Chevron does not exist and 99.   Fumitechniks defaulted on its obligation. The check it issued to Chevron in
since First Lepanto’s contract with Fumitechniks is merely an accessory contract, no the amount of P11,461,773.10, when presented for payment, was
demand can be made. RTC ruled in favor of First Lepanto. CA reversed. W/N First dishonored for reason of "Account Closed." Chevron notified First Lepanto
Lepanto is liable as surety for the obligation of Fumitechniks – NO, the surety bond of Fumitechniks' unpaid purchases in the total amount of P15,084,030.30.
specifically makes reference to a written agreement. It is basic that if the terms of a 100.  First Lepanto through its counsel, requested that it be furnished copies of
contract are clear and leave no doubt upon the intention of the contracting parties, the documents such as delivery receipts. Chevron complied by sending
the literal meaning of its stipulations shall control. A reading of the Surety Bond copies of invoices showing deliveries of fuel and petroleum products
shows that it secures the payment of purchases on credit by Fumitechniks in between November 11, 2001 and December 1, 2001.
accordance with the terms and conditions of the "agreement" it entered into with 101.  Simultaneously, a letter was sent to Fumitechniks demanding that the latter
Chevron. The word "agreement" has reference to the distributorship agreement, the submit to First Lepanto the following: (1) its comment on Chevron notice
principal contract and by implication included the credit agreement mentioned in letter; (2) copy of the agreement secured by the Bond, together with copies
the rider. However, it turned out that Chevron has executed written agreements only of documents such as delivery receipts; and (3) information on the
with its direct customers but not distributors like Fumitechniks and it also never particulars, including "the terms and conditions, of any arrangement that
relayed the terms and conditions of its distributorship agreement to the First [Fumitechniks] might have made or any ongoing negotiation with Caltex in
Lepanto after the delivery of the bond. Moreover, being an onerous undertaking, a connection with the settlement of the obligations subject of the Caltex
surety agreement is strictly construed against the creditor, and every doubt is letter."
resolved in favor of the solidary debtor. Having accepted the bond, Chevron as 102.  Fumitechniks through its counsel wrote First Lepanto counsel informing
creditor must be held bound by the recital in the surety bond that the terms and that it cannot submit the requested agreement since no such agreement was
conditions of its distributorship contract be reduced in writing or at the very least executed between Fumitechniks and Chevron. Fumitechniks also enclosed a
communicated in writing to the surety. Chevron is charged with notice of the copy of another surety bond issued by CICI General Insurance Corporation
specified form of the agreement or at least the disclosure of basic terms and in favor of Chevron to secure the obligation of Fumitechniks and/or Prime
conditions of its distributorship and credit agreements with its client Fumitechniks Asia Sales and Services, Inc. in the amount of P15,000,000.00.
after its acceptance of the bond delivered by the latter. Also, it never made any 103.  Consequently, First Lepanto advised Chevron of the non-existence of the
effort to relay those terms and conditions of its contract with Fumitechniks upon the principal agreement as confirmed by Fumitechniks. First Lepanto explained
commencement of its transactions with said client, which obligations are covered by that being an accessory contract, the bond cannot exist without a principal
the surety bond issued by First Lepanto. agreement as it is essential that the copy of the basic contract be submitted
to the proposed surety for the appreciation of the extent of the obligation to
DOCTRINE: Section 176 of the Insurance Code states: The liability of the surety be covered by the bond applied for.
or sureties shall be joint and several with the obligor and shall be limited to the 104.  Chevron formally demanded from First Lepanto the payment of its claim
amount of the bond. It is determined strictly by the terms of the contract of under the surety bond. However, First Lepanto reiterated its position that
suretyship in relation to the principal contract between the obligor and the without the basic contract subject of the bond, it cannot act on Chevron
obligee. claim.
  105.  Alleging that First Lepanto unjustifiably refused to heed its demand for
payment, Chevron prayed for judgment ordering First Lepanto to pay the
FACTS:  
sum of P15,080,030.30, plus interest, costs and attorney's fees equivalent to RATIO:  
ten percent of the total obligation.
106.  First Lepanto, in its Answer with Counterclaim, asserted that the Surety 119.  The main issue to be resolved is one of first impression: whether a surety is
Bond was issued for the purpose of securing the performance of the liable to the creditor in the absence of a written contract with the principal.
obligations embodied in the Principal Agreement stated therein, which
contract should have been attached and made part thereof.
1.   Section 175 of the Insurance Code defines a suretyship as a contract or
107.  RTC rendered judgment dismissing the complaint as well as First Lepanto
agreement whereby a party, called the surety, guarantees the performance
counterclaim
by another party, called the principal or obligor, of an obligation or
1.   Terms and conditions of the oral credit line agreement between
undertaking in favor of a third party, called the obligee.
Chevron and Fumitechniks have not been relayed to First Lepanto
a.   It includes official recognizances, stipulations, bonds or
and neither were the same conveyed even during trial
undertakings issued under Act 536, as amended. Suretyship arises
2.   Since the surety bond is a mere accessory contract, the bond cannot
upon the solidary binding of a person — deemed the surety — with
stand in the absence of the written agreement secured thereby.
the principal debtor, for the purpose of fulfilling an obligation.
3.   RTC noted the practice of First Lepanto, as testified on by its
b.   Although the contract of a surety is in essence secondary only to a
witnesses, to attach a copy of the written agreement (principal
valid principal obligation, the surety becomes liable for the debt or
contract) whenever it issues a surety bond, or to be submitted later
duty of another although it possesses no direct or personal interest
if not yet in the possession of the assured, and in case of failure to
over the obligations nor does it receive any benefit therefrom. And
submit the said written agreement, the surety contract will not be
notwithstanding the fact that the surety contract is secondary to the
binding despite payment of the premium.
principal obligation, the surety assumes liability as a regular party
108.  CA ruled in favor of Chevron
to the undertaking.
1.   First Lepanto cannot insist on the submission of a written
2.   The extent of a surety's liability is determined by the language of the
agreement to be attached to the surety bond considering that
suretyship contract or bond itself. It cannot be extended by implication,
Chevron was not aware of such requirement and unwritten
beyond the terms of the contract. Thus, to determine whether First Lepanto
company policy.
is liable to Chevron under the surety bond, it becomes necessary to examine
2.   First Lepanto is estopped from assailing the oral credit line
the terms of the contract itself. (Terms and conditions of the contract can be
agreement, having consented to the same upon presentation by
seen at the end of the digest)
Fumitechniks of the surety bond it issued.
3.   A reading of Surety Bond FLTICG (16) No. 01012 shows that it secures
3.   Considering that such oral contract between Fumitechniks and
the payment of purchases on credit by Fumitechniks in accordance
Chevron has been partially executed, the CA ruled that the
with the terms and conditions of the "agreement" it entered into with
provisions of the Statute of Frauds do not apply.
Chevron. The word "agreement" has reference to the distributorship
agreement, the principal contract and by implication included the
ISSUE/s:   credit agreement mentioned in the rider. However, it turned out that
1.   W/N First Lepanto is liable as surety for the obligation of Fumitechniks – Chevron has executed written agreements only with its direct
NO, the surety bond specifically makes reference to a written agreement. It customers but not distributors like Fumitechniks and it also never
is basic that if the terms of a contract are clear and leave no doubt upon the relayed the terms and conditions of its distributorship agreement to the
intention of the contracting parties, the literal meaning of its stipulations First Lepanto after the delivery of the bond. This was clearly admitted by
shall control. Chevron Marketing Coordinator, Alden Casas Fajardo.
4.   Chevron, however, maintains that the delivery of the bond and acceptance
RULING: WHEREFORE, the petition for review on certiorari is PARTLY of premium payment by First Lepanto binds the latter as surety,
GRANTED . The Decision dated November 20, 2006 and Resolution dated May 8, notwithstanding the non- submission of the oral distributorship and credit
2007 of the Court of Appeals in CA-G.R. CV No. 86623, are REVERSED and SET agreement which understandably cannot be attached to the bond.
ASIDE. The Decision dated August 5, 2005 of the Regional Trial Court of Makati 5.   The law is clear that a surety contract should be read and interpreted
City, Branch 59 in Civil Case No. 02-857 dismissing respondent's complaint as well together with the contract entered into between the creditor and the
as petitioner's counterclaim, is hereby REINSTATED and UPHELD. No principal.
pronouncement as to costs.SO ORDERED. 6.   Section 176 of the Insurance Code states: The liability of the surety or
sureties shall be joint and several with the obligor and shall be limited to the
amount of the bond. It is determined strictly by the terms of the contract ends to which a motion for reconsideration is addressed is precisely to
of suretyship in relation to the principal contract between the obligor convince the court that its ruling is erroneous and improper, contrary to the
and the obligee. law or evidence; the movant has to dwell of necessity on issues already
a.   A surety contract is merely a collateral one, its basis is the passed upon.
principal contract or undertaking which it secures. Necessarily, the
stipulations in such principal agreement must at least be Damages
communicated or made known to the surety particularly in this
case where the bond expressly guarantees the payment of Chevron
2.   Finally, we hold that the RTC correctly dismissed First Lepanto
fuel products withdrawn by Fumitechniks in accordance with the
counterclaim for moral damages and attorney's fees. The filing alone of a
terms and conditions of their agreement.
civil action should not be a ground for an award of moral damages in the
b.   The bond specifically makes reference to a written agreement.
same way that a clearly unfounded civil action is not among the grounds for
It is basic that if the terms of a contract are clear and leave no
moral damages. Besides, a juridical person is generally not entitled to moral
doubt upon the intention of the contracting parties, the literal
damages because, unlike a natural person, it cannot experience physical
meaning of its stipulations shall control.
suffering or such sentiments as wounded feelings, serious anxiety, mental
c.   Moreover, being an onerous undertaking, a surety agreement is
anguish or moral shock.
strictly construed against the creditor, and every doubt is
a.   This is so because moral damages, though incapable of pecuniary
resolved in favor of the solidary debtor. Having accepted the estimation, are in the category of an award designed to compensate
bond, Chevron as creditor must be held bound by the recital in the the claimant for actual injury suffered and not to impose a penalty
surety bond that the terms and conditions of its distributorship
on the wrongdoer There is no evidence presented to establish the
contract be reduced in writing or at the very least communicated in
factual basis of First Lepanto claim for moral damages.
writing to the surety.
3.   First Lepanto is likewise not entitled to attorney's fees. The settled rule is
7.   The creditor is generally held bound to a faithful observance of the rights of
that no premium should be placed on the right to litigate and that not every
the surety and to the performance of every duty necessary for the protection
winning party is entitled to an automatic grant of attorney's fees. In pursuing
of those rights. Moreover, in this jurisdiction, obligations arising from
its claim on the surety bond, Chevron was acting on the belief that it can
contracts have the force of law between the parties and should be complied
collect on the obligation of Fumitechniks notwithstanding the non-
with in good faith. Chevron is charged with notice of the specified form
submission of the written principal contract.
of the agreement or at least the disclosure of basic terms and conditions
of its distributorship and credit agreements with its client Fumitechniks
Surety  Bond  FLTICG  (16)  No.  01012  is  a  standard  form  used  by  First  Lepanto,  which  states:    
after its acceptance of the bond delivered by the latter. However, it
never made any effort to relay those terms and conditions of its
That we, FUMITECHNIKS CORP. OF THE PHILS. of#154 Anahaw St., Project 7, Quezon City as
contract with Fumitechniks upon the commencement of its transactions principal and First Lepanto-Taisho Insurance Corporation a corporation duly organized and existing under
with said client, which obligations are covered by the surety bond and by virtue of the laws of the Philippines as Surety, are held firmly bound unto CALTEX
issued by First Lepanto. Contrary to Chevron’s assertion, there is no PHILIPPINES, INC. of ______ in the sum of FIFTEEN MILLION SEVEN HUNDRED THOUSAND
indication in the records that First Lepanto had actual knowledge of its ONLY PESOS (P15,700,000.00), Philippine Currency, for the payment of which sum, well and truly to be
made, we bind ourselves, our heirs, executors, administrators, successors, and assigns, jointly and
alleged business practice of not having written contracts with severally, firmly by these presents:
distributors; and even assuming First Lepanto was aware of such
practice, the bond issued to Fumitechniks and accepted by Chevron The conditions of this obligation are as follows:
specifically referred to a "written agreement."
WHEREAS, the above-bounden principal, on 15th day of October, 2001 entered into [an] agreement with
Motion for reconsideration CALTEX PHILIPPINES, INC. of ________________ to fully and faithfully

1.   As to the contention of First Lepanto that Chevron’s motion for a copy of which is attached hereto and made a part hereof:
reconsideration filed before the trial court should have been deemed not
filed for being pro forma, the Court finds it to be without merit. The mere WHEREAS, said Obligee__ requires said principal to give a good and sufficient bond in the above stated
sum to secure the full and faithful performance on his part of said agreement__. aAHDIc
fact that a motion for reconsideration reiterates issues already passed upon
by the court does not, by itself, make it a pro forma motion. Among the
NOW THEREFORE, if the principal shall well and truly perform and fulfill all the undertakings,
covenants, terms, conditions, and agreements stipulated in said agreement__ then this obligation shall be
null and void; otherwise it shall remain in full force and effect.

The liability of First Lepanto-Taisho Insurance Corporation under this bond will expire on October 15,
2002__.

xxx xxx xxx (Emphasis supplied.) The rider attached to the bond sets forth the following:

WHEREAS, the Principal has applied for a Credit Line in the amount of P E S O S : Fifteen Million
Seven Hundred thousand only (P15,700,000.00), Philippine Currency with the Obligee for the purchase of
Fuel Products;

WHEREAS, the obligee requires the Principal to post a bond to guarantee payment/remittance of the cost
of fuel products withdrawn within the stipulated timein accordance with terms and conditions of the
agreement;

IN NO CASE, however, shall the liability of the Surety hereunder exceed the sum of PESOS: Fifteen
million seven hundred thousand only (P15,700,000.00), Philippine Currency.

NOW THEREFORE, if the principal shall well and truly perform and fulfill all the undertakings,
covenants, terms and conditions and agreements stipulated in said undertakings, then this obligation shall
be null and void; otherwise, it shall remain in full force and effect.

The liability of FIRST LEPANTO-TAISHO INSURANCE CORPORATION, under this Bond will expire
on 10.15.01_. Furthermore, it is hereby understood that FIRST LEPANTO-TAISHO INSURANCE
CORPORATION will not be liable for any claim not presented to it in writing within fifteen (15) days
from the expiration of this bond, and that the Obligee hereby waives its right to claim or le any court
action against the Surety after the termination of fifteen (15) days from the time its cause of action
accrues.

 
   
012 PHILIPPINE PRYCE v. CA (MERILLES) 3.   On June 16, 1988, summons, together with the copy of the complaint, was
February 21, 1994 | Nocon, J. | Suretyship served on Philippine Pryce. Within the reglementary period, two successive
motions were filed by Philippine Pryce praying for a total of thirty (30) days
extention within which to file a responsible pleading.
PETITIONER: Philippine Pryce Assurance Corp
4.   In its Answer, dated July 29, 1988, but filed only on August 4, 1988,
RESPONDENTS: Court of Appeals and Gegroco, Inc.
petitioner admitted having executed the said bonds, but denied liability
because allegedly:
SUMMARY: Gegroco alleges that Phil. Pryce issued two surety bonds to a.   the checks which were to pay for the premiums bounced and were
Gegroco, in behalf of its Principal Sagum. Phil. Pryce admitted having executed dishonored hence there is no contract to speak of between Phil.
the two bonds, but denies liability. (To clarify, Phil. Pryce is the surety of Pryce and its supposed principal; and
Sagum) It alleges that the checks issued by Sagum, which were to pay for the b.   that the bonds were merely to guarantee payment of its principal's
premiums bounced and were dishonored, hence there is no contract to speak of
obligation, thus, excussion is necessary.
between it and Sagum (principal). Thefore, as surety, it is not liable for the
5.   After the issues had been joined, the case was set for pre-trial conference on
bonds it executed as the contract with Sagum was not perfected.
September 29, 1988.
6.   Philippine Pryce received its notice on September 9, 1988, while the notice
The issue before the SC is whether Phil. Pryce is liable for the bonds it issued to
addressed to its counsel was returned to the trial court with the notation
Gegroco. The SC held that Phil. Pryce is laible.
"Return to Sender, Unclaimed."
7.   On the scheduled date for pre-trial conference, only the counsel for
Under the Insurance Code: The surety is entitled to payment of the premium as Philippine Pryce appeared while both the representative of Gegroco and its
soon as the contract of suretyship or bond is perfected and delivered to the counsel were present.
obligor. No contract of suretyship or bonding shall be valid and binding unless 8.   The counsel for Philippine Pryce manifested that he was unable to contract
and until the premium therefor has been paid, except where the obligee has the Vice-President for operations of petitioner, although his client intended
accepted the bond, in which case the bond becomes valid and enforceable
to file a third party complaint against its principal.
irrespective of whether or not the premium has been paid by the obligor to the
a.   Hence, the pre-trial was re-set to October 14, 1988.
surety.
9.   Philippine Pryce filed a "Motion with Leave to Admit Third-Party
Complaint" with the Third-Party Complaint attached. On the same day, in
From the records of the case, it appears that Phil. Pryce actually received surety
the presence of the representative for both Philippine Pryce and Gegroco
bonds issued by Sagum. By accepting the bonds, the suretyship becomes valid
and their counsel, the pre-trial conference was re-set to December 1, 1988.
and enforceable irrespective of whether or not the premium has beed paid. As
10.   Meanwhile on November 29, 1988, the court admitted the Third Party
surety, Phil. Pryce is liable to Gegroco. Complaint and ordered service of summons on third party defendants.
11.   On the scheduled conference, Philippine Pryce and its counsel did not
DOCTRINE: No contract of suretyship or bonding shall be valid and binding appear notwithstanding their notice in open court.
unless and until the premium therefor has been paid, except where the obligee a.   The case again was re-set to February 1.
has accepted the bond, in which case the bond becomes valid and enforceable
12.   However, on February 1, Philippine Pryce and counsel was also absent
irrespective of whether or not the premium has been paid by the obligor to the
despite being duly notified.
surety.
13.   Hence, Philippine Pryce was considered as in default and Gegroco was
permitted to present evidence ex-parte.
  14.   When Philippine Pryce received the notice of Order of Default, it field a
FACTS: Motion for Reconsideration and New Trial
1.   Gegroco filed a complaint for collection of sum of money against Philippine 15.   The motion was denied. Philippine Pryce elevated the case to the CA. The
Pryce before the Makati RTC CA affirmed the RTC deicsion.
2.   The complaint alleged that Phi. Pryce issued two surety bonds (No. 0029, 16.   Hence, this case.
dated July 24, 1987 and No. 0037, dated October 7, 1987) in behalf of its
principal Sagum General Merchandise for FIVE HUNDRED THOUSAND ISSUE/s:
(P500,000.00) PESOS and ONE MILLION (1,000,000.00) PESOS, 1.   WON Phil. Pryce is liable for the bonds it issued to Gegroco - YES, it is
respectively. liable because by accepting the bonds, the suretyship becomes valid and
enforceable irrespective of whether or not the premium has beed paid. b.   The same rule applies to permissive counterclaims, third-party
claims and similar pleadings, which shall not be considered filed
RULING: WHEREFORE, in view of the foregoing, the decision of the Court of until and unless the filing fee prescribed therefor is paid. The court
Appeals dismissing the petition before them and affirming the decision of the trial may also allow payment of said fee within a prescriptive or
court and its order denying petitioner's Motion for Reconsideration are hereby reglementary period.
AFFIRMED. The present petition is DISMISSED for lack of merit. c.   Where the trial court acquires jurisdiction over a claim by the
filing of the appropriate pleading and payment of the prescribed
RATIO: (SKIP TO RATIO #4 FOR INSURANCE RELATED DISCUSSION) filing fee, but subsequently, the judgment awards a claim nor
1.   Philippine Pryce argues that that since the last pleading, which was specified in the pleading, or if specified the same has not been left
supposed to be the third-party defendant's answer has not been filed, the for determination by the court, the additional filing fee therefor
case is not yet ripe for pre-trial. shall constitute a lien on the judgment. It shall be the responsibility
2.   The SC does not agree and cites 3 main points: of the clerk of court or his duly authorized deputy to enforce said
a.   First, the trial court asserted, and the SC agrees, that no answer to lien and assess and collect the additional fee.
the third party complaint is forthcoming as Philippine Pryce never 4.   [INSURANCE RELATED TOPIC] Finally, there is reason to believe that
initiated the service of summons on the third party defendant. Philippine Pryce does not really have a good defense. It hinges its defense
b.   Second, in the regular course of events, the third-party defendant's on two arguments, namely:
answer would have been regarded as the last pleading referred to in a.   that the checks issued by its principal which were supposed to pay
Sec. 1, Rule 20. However, Philippine Pryce cannot just disregard for the premiums, bounced, hence there is no contract of surety to
the court's order to be present during the pre-trial and give a flimsy speak of; and
excuse, such as that the answer has yet to be filed. b.   that as early as 1986 and covering the time of the Surety Bond,
i.   The pre-trial is mandatory in any action, the main Phil. Pryce was not yet authorized by the insurance Commission to
objective being to simplify, abbreviate and expedite trial, issue such bonds.
if not to fully dispense with it. Hence, consistent with its 5.   The Insurance Code states that:
mandatory character the Rules oblige not only the a.   Sec. 177. The surety is entitled to payment of the premium as soon
lawyers but the parties as well to appear for this purpose as the contract of suretyship or bond is perfected and delivered to
before the Court and when a party fails to appear at a the obligor. No contract of suretyship or bonding shall be valid and
pre-trial conference he may be non-suited or considered binding unless and until the premium therefor has been paid,
as in default. except where the obligee has accepted the bond, in which case the
ii.   Records show that even at the very start, Philippine bond becomes valid and enforceable irrespective of whether or not
Pryce could have been declared as in default since it was the premium has been paid by the obligor to the surety. . . .
not properly presented during the first scheduled pre-trial (emphasis added)
on September 29, 1988. Nothing in the record is attached 6.   The above provision outrightly negates Philippine Pryce’s first defense. In a
which would show that petitioner's counsel had a special desperate attempt to escape liability, Phil. Pryce further asserts that the
authority to act in behalf of his client other than as its above provision is not applicable because allegedly had not accepted the
lawyer. surety bond, hence could not have delivered the goods to Sagum
c.   Third, the court of Appeals properly considered the third-party Enterprises. This statement clearly intends to muddle the facts as found by
complaint as a mere scrap of paper due to Philippine Pryce’s the trial court and which are on record.
failure to pay the requisite docket fees. 7.   Phil. Pryce it its answer admitted to have issued the bonds subject matter of
3.   The SC laid down the rules as follows: the original action.
a.   It is not simply the filing of the complaint or appropriate initiatory 8.   Also, the testimony of Mr. Leonardo T. Guzman, witness of Gegroco
pleading, but the payment of the prescribed docket fee, that vests a reveals that Phil. Pryce accepted the two surety bonds issued by Sagum.
trial court with jurisdiction over the subject-matter or nature of the 9.   Also attached to the records are exhibits consisting of delivery invoices
action. Where the filing of the initiatory pleading is not addressed to Sagum providing that parts were purchased, delivered, and
accompanied by payment of the docket fee, the court may allow received.
payment of the fee within a reasonable time, but in no case beyond 10.   On the other hand, Phil. Pryce’s defense that it did not have authority to
the applicable prescriptive or reglamentary period. issue a Surety Bond when it did is an admission of fraud committed against
Gegroco.
11.   No person can claim benefit from the wrong he himself committed. A
representation made is rendered conclusive upon the person making it and
cannot be denied or disproved as against the person relying thereon.
   
013 CAPITAL INSURANCE v. RONQUILLO TRADING (PELIÑO) Company, Inc (PMSCI).
July 25, 1983 | Gutierrez, J. | Surety b.   In consideration for the issuance of Capital of the surety bond,
Ronquillo executed an indemnity agreement, where they solidarily
PLAINTIFF-APPELLANT: Capital Insurance & Surety Co., Inc., herein represented by promised to pay Capital the sum of Php 1,827.00 in advance as
its General Agent, the Pan American Insurance Agencies, Inc. premium and documentary stamps for each period of 12 mos while the
DEFENDANTS-APPELLEES: Ronquillo Trading and Jose L. Bautista surety bond was in effect.
165.  On 30 April 1963, or 5 days before the expiration of liability on the bond,
SUMMARY: Capital through its general agent executed and issued a surety bond for P.D. Marchessini and Co, Ltd. and Delgado Shipping Agencies filed a case
$14,800 in behalf of Ronquillo Trading and in favor of Delgado Shipping. The bond was against PMSCI and Bautista, doing business under Ronquillo Trading, and
a guarantee for any additional freight which may be determined to be due on a cargo for Capital for $14,800 or its equivalent in Philippine currency for the loss they
258 surplus army vehicles consigned by PMSCI. Ronquillo executed an indemnity
suffered as a direct consequence of the failure to load the stipulated quantity
agreement where they solidarily promised to pay Capital Php 1,827 in advance as
premium and docu stamps while the surety bond was in effect. 5 days before the
of 406 US surplus army vehicles.
expiration of the liability on the bond, Delgado and P.D. Marchessini filed a case against a.   Capital was made party because of the bond it posted on behalf of
PMSCI, Bautista (Ronquillo), and Capital for $14,800 for the loss that they suffered since Ronquillo Trading and Bautista.
Ronquillo failed to load the stipulated quantity of 406 US surplus army vehicles. Capital 166.  Upon the expiration of the 12 mos life of the bond, Capital made a formal
was made a defendant because of the bond it posted on behalf of Ronquillo. Upon demand for the payment of the renewal premiums and cost of DST for
expiration of the 12 mos life of the bond, Capital made a formal demand or payment of another year in the amount of Php 1,827, but Ronquillo refused to pay,
renewal of premiums. But Ronquillo refused to pay, saying that Ronquillo’s liability saying that the liability of Capital under the surety bond accrued during the
under the surety bond accrued during the 12 mo period the bond was originally in force period of 12 mos the bond was originally in force and before its expiration
and before its expiration, so Ronquillo was under no obligation to renew the surety bond.
and that Ronquillo was under no obligation to renew the surety bond.
Capital filed a complaint in the city court, absolved Ronquillo. On appeal to CFI,
affirmed. Hence, this appeal. The issue in this case is whether or not Ronquillo still has
167.  Capital filed a complaint to recover the said sum in the City Court of
the obligation to pay the premiums and costs for the succeeding period once the surety’s Manila.
liability under the bond has accrued. The SC held in the negative. In the surety bond, it is a.   The court absolved Ronquillo from the complaint.
stipulated that the liability of the surety will expire on 5 May 1963, unless surety is 168.  Capital appealed the judgment to the CFI where the decision of the city
notified of any existing oblgiations thereunder. Under this stipulation, the bond expired court was affirmed and the complaint was dismissed.
on the stated date, and the phrase “unless surety… thereunder” refers to obligations a.   Capital filed a MR but was denied.
incurred during the term of the bond. Under the Indemnity Agreement, the duration of the 169.  Hence, this appeal.
bond is for “every 12 mos or fraction thereof, while this bond or any renewal or
substistution is in effect”. Since Ronquillo opted not to renew the contract, they can’t be
ISSUE/s:
obliged to pay the premiums. Where a contract of surety is terminated under its terms, the
liability of the principal for premiums after such termination ceases notwithstanding the
1.   WON Ronquillo still has the obligation to pay the premiums and costs for
pendency of the lawsuit to enforce a liability that accrued during its stipulated lifetime. the succeeding period once the surety’s liability under the bond has accrued.
– NO, where a contract of surety is terminated under its terms, the liability
DOCTRINE: Where a contract of surety is terminated under its terms, the liability of the of the principal for premiums after such termination ceases notwithstanding
principal for premiums after such termination ceases notwithstanding the pendency of the the pendency of the lawsuit to enforce a liability that accrued during its
lawsuit to enforce a liability that accrued during its stipulated lifetime. stipulated lifetime.
 
FACTS: RULING: WHEREFORE, the appeal is DISMISSED for alck of merit. The
164.  Capital Insurance & Surety Insurance Co., Inc. (Capital) through its general decision of the court a quo is AFFIRMED.
agent, executed and issued a surety bond in the amount of $14,800.00 or its
peso equivalent in behalf of Ronquillo Trading and Jose Bautista RATIO:
(collectively will be called Ronquillo) and in favor of S.S. Eurygenes, its On whether Ronquillo still has the obligation to pay the premiums and costs for the
master, and/or its agents, Delgado Shipping Agencies. succeeding period once the surety’s liability under the bond has accrued.
a.   The bond was a guarantee for any additional freight which may be 1.   Capital: the trial court’s finding that once the surety’s liability has accrued,
determined to be due on a cargo for 258 surplus army vehicles the defendants are under no obligation to pay the premiums for the
consigned from Pusan, Korea to Ronquillo on board the S.S. Eurygenes succeeding period that it is in effect by reason of existing obligation of the
and booked on said vessel by Philippine Merchants Steamship surety bond, is erroneous since it contradicts the provision in the indemnity
agreement.60
a.   It can be deduced that the payment of renewal premiums should depend
upon the life and effectivity of the bond and not on the accrual of its
liability.
b.   As long as the bond is in full force and effect, the principal should pay
the renewal premium and continue to do so even if the liability on the
bond has accrued, otherwise, surety companies will be at the mercy of
their principals because while their liability continues to subsist as long
as their accrued liability is not determined, the principals pay no
consideration for the use of their bond.
2.   Ronquillo: the only purpose of the civil case was to enforce liability which
existed even before the bond was executed.
a.   The bond was given to secure payment by Ronquillo of additional
freight as would already be due on the cargo when it actually arrived in
Manila. It wasn’t executed to secure obligation or liability which was
still to arise after the 12 mo life.
b.   While it’s true that the lower court held that the bond was still in effect
after expiry date, the effectivity was not because of a renewal made by
Ronquillo but because the surety bond provides that the “liaibility of
the surety will not expire if, as in this case, it is notified of an existing
obligation there under.” The meaning of the bond’s still being in effect
is that, suit on the bond instituted by the obliges prior to the expiration
of the “liability” thereunder was only for purpose of enforcing that
liability and amounter to notice to capital of an already existing or
accrued liability so as not to let that liability lapse or expire and bar
enforcement.
3.   SC: Agrees with Ronquillo.
a.   In the surety bond, it is stipulated that the liability of the surety will
expire on 5 May 1963, unless surety is notified of any existing
oblgiations thereunder.
b.   Under this stipulation, the bond expired on the stated date, and the
phrase “unless surety… thereunder” refers to obligations incurred
during the term of the bond.
c.   Under the Indemnity Agreement, the duration of the bond is for “every
12 mos or fraction thereof, while this bond or any renewal or
substistution is in effect”. Since Ronquillo opted not to renew the
contract, they can’t be obliged to pay the premiums.
d.   Where a contract of surety is terminated under its terms, the
liability of the principal for premiums after such termination
ceases notwithstanding the pendency of the lawsuit to enforce a
liability that accrued during its stipulated lifetime.

                                                                                                                       
60
 PREMIUMS  –  As  consideration  for  the  surety,  the  undersigned  jointly  and  severally,  agree  to  pay  the  
company  the  sum  of  1,800  pesos,  Philippine  currency,  in  advance  as  premium  thereof  for  every  12  mos  
or  fraction  thereof,  while  this  bond  or  any  renewal  or  substitution  is  in  effect.  
001 VDA. DE GABRIEL v. CA (PLEYTO)
Nov. 14, 1996 | Vitug, J. | Life Insurance DOCTRINE: An "accident insurance" is not thus to be likened to an ordinary life
PETITIONER: Jacqueline Jimenez Vda. De Gabriel insurance where the insured's death, regardless of the cause thereof, would normally
RESPONDENTS: Hon. Court of Appeals and Fortune Insurance & Surety be compensable. The latter is akin in property insurance to an "all risk" coverage
Company, Inc. where the insured, on the aspect of burden of proof, has merely to show the condition
SUMMARY: Gabriel was employed by ECDC at its construction project in Iraq. He of the property insured when the policy attaches and the fact of loss or damage
was covered by a personal accident insurance in the amount of P100k, under a group during the period of the policy and where, thereafter, the burden would be on the
policy procured from Fortune Insurance & Surety Company, Inc. by ECDC for its insurer to show any "excluded peril." When, however, the insured risk is specified,
overseas workers. The insured risk was for bodily injury caused by violent accidental like in the case, it lies with the claimant of the insurance proceeds to initially prove
external and visible means which injury would solely and independently of any other that the loss is caused by the covered peril.
cause result in death or disability. Within the life of the policy, Gabriel died. More
than a year later, ECDC reported Gabriel’s death to Fortune by telephone. Among
the documents submitted to Fortune were a copy of the the death certificate issued by FACTS:
the Ministry of Health of the Republic of Iraq, which stated that the reason of death 109.  Marcelino Gabriel, the insured, was employed by Emerald Construction &
is not yet known and that it is still under examination; and an autopsy report of the Development Corporation (ECDC) at its construction project in Iraq.
NBI to the effect that “due to advanced state of postmortem decomposition, cause of 110.  He was covered by a personal accident insurance in the amount of P100k
death could not be determined. Fortune denied the claim on the ground of under a group policy procured from Fortune Insurance & Surety Company,
prescription. So Jacqueline filed a case before the RTC. She averred that Gabriel Inc. (Fortune) by ECDC for its overseas workers
died of electrocution while in the performance of his work and prayed for the 111.  The insured risk was for bodily injury caused by violent accidental external
recovery of the 100k and other damages. Fortune filed its answer alleging that both and visible means which injury would solely and independently of any other
the death cert and the autopsy report failed to disclose the cause of death. It also cause result in death or disability
raised the defense of prescription. RTC ruled in favor of Jacqueline. Both Jacqueline 112.  May 22, 1982: within the life of the policy, Gabriel died in Iraq.
and Fortune appealed to the CA. CA reversed the decision. The issue in this case, 113.  A year later or on July 12, 1983, ECDC reported Gabriel’s death to Fortune
with regard to life insurance, WoN there is sufficient evidence to prove that the by telephone.
insured suffered from an accidental death, the risk covered by the policy. The 114.  Among the documents submitted to Fortune were a copy of the death
affidavit of Jacqueline and the letter of Gabriel’s co-working saying that Gabriel died certificate issued by the Ministry of Health of the Republic of Iraq, which
of electrocution were all ruled to be hearsay as both were not testified to by the stated that the reason of death is not yet known and that it is still under
affiants. Not one of the other documents submitted, to wit, the POEA decision, the examination; and an autopsy report of the NBI to the effect that “due to
death certificate issued by the Ministry of Health of Iraq and the NBI autopsy report, advanced state of postmortem decomposition, cause of death could not be
could give any probative value to the claim. The POEA decision did not make any determined
categorical holding on the specific cause of Gabriel's death. Neither did the death 115.  Fortune referred the insurance claim to Mission Adjustment Service, Inc.
certificate issued by the health authorities in Iraq nor the NBI autopsy report provide 116.  Following a series of communications between Jacqueline Jimenez vda. De
any clue on the cause of death. Evidence is utterly wanting to establish that the Gabriel (Jacqueline) and Fortune, the latter, on Sept. 22, 1983 ultimately
insured suffered from an accidental death, the risk covered by the policy. In an denied the claim of ECDC on the ground of prescription
accident insurance, the insured's beneficiary has the burden of proof in 117.  So Jacqueline went to the RTC of Manila. In her complaint against ECDC
demonstrating that the cause of death is due to the covered peril. Once the fact is and Fortune, she averred that her husband died of electrocution while in the
established, the burden then shifts to the insurer to show any excepted peril that may performance of his work and prayed for the recovery of P100k for insurance
have been stipulated by the parties. An "accident insurance" is not thus to be likened indemnification and of various other sums by way of actual, moral, and
to an ordinary life insurance where the insured's death, regardless of the cause exemplary damages, plus attorney’s fees and costs
thereof, would normally be compensable. The latter is akin in property insurance to 118.  Fortune filed its answer, which was not verified, admitting the genuineness
an "all risk" coverage where the insured, on the aspect of burden of proof, has merely and due execution of the insurance policy
to show the condition of the property insured when the policy attaches and the fact of 1.   It alleged, however, that since both the death certificate issued by
loss or damage during the period of the policy and where, thereafter, the burden the Iraqi Ministry of Health and the autopsy report of the NBI
would be on the insurer to show any "excluded peril." When, however, the insured failed to disclose the cause of Gabriel’s death, it denied liability
risk is specified, like in the case, it lies with the claimant of the insurance proceeds to under the policy
initially prove that the loss is caused by the covered peril.
2.   In addition, it raised the defense of prescription, invoking Sec. 384 concerned a written notice of claim setting forth the nature, extent and duration
of the Insurance Code of the injuries sustained as certified by a duly licensed physician. Notice of
3.   Later, it filed an amended answer, still unverified, reiterating its claim must be filed within six months from date of the accident, otherwise, the
original defenses but, it added a counterclaim and a cross claim claim shall be deemed waived. Action or suit for recovery of damage due to loss
119.  RTC: dismissed the case against ECDC for the failure to take steps to cause or injury must be brought, in proper cases, with the Commissioner or the Courts
the service of the fourth alias summons on ECDC. The dismissal was within one year from denial of the claim, otherwise, the claimant's right of
without prejudice. The case proceeded against Fortune alone. action shall prescribe.
120.  On May 28, 1987, the trial court rendered its decision in favor (partly) of 123.  The notice of death was given to Fortune, concededly, more than a year
Jacqueline’s claim. It held that Fortune was deemed to have waived the after the death of Jacqueline’s husband.
defense, i.e., that the cause of death was not covered by the policy, when the 124.  Fortune, in invoking prescription, was not referring to the one-year period
latter failed to impugn by evidence Jacqueline’s averment on the matter from the denial of the claim within which to file an action against an insurer
121.  With regard to the defense of prescription, the court considered the but obviously to the written notice of claim that had to be submitted within
complaint to have been timely filed or within 1 year from Fortune’s denial six months from the time of the accident
of claim 125.  Jacqueline arguest that Fortune must be deemed to have waived its right to
122.  Jacqueline and Fortune both appealed to the CA. controvert the claim, that is, to show that the cause of death is an excepted
1.   Jacqueline contended that the lower court should have awarded all peril, by failing to have its answers (to the Request for Admission sent by
the claims she had asked for. Jacqueline) duly verified
2.   Fortune asserted that the lower ocurt erred in ruling (i) that the 126.  It is true that a matter of which a written request for admission is made shall
insurer had waived the defense that Gabriel’s death was not caused be deemed impliedly admitted "unless, within a period designated in the
by the insured peril (“violent accidental external and visible request, which shall not be less than ten (10) days after service thereof, or
means”) specified in the policy and (ii) that the cause of action had within such further time as the court may allow on motion and notice, the
not prescribed party to whom the request is directed serves upon the party requesting the
123.  CA: reversed the decision. admission a sworn statement either denying specifically the matters of
1.   It held that Jacqueline failed to substantiate her allegation that her which an admission is requested or setting forth in detail the reasons why he
husband’s death was caused by a risk insured against. cannot truthfully either admit or deny those matters"
2.   It observed that the only evidence presented by Jacqueline, in her 127.  However, the verification, like in most cases required by the rules of
attempt to show the circumstances that led to the death of the procedure, is a formal, not jurisdictional, requirement, and mainly intended
insured, were her own affidavit and a letter alleged written by a co- to secure an assurance that matters which are alleged are done in good faith
worker of the deceased in Iraq which, unfortunately for her, were or are true and correct and not of mere speculation.
held to be both hearsay. 128.  When circumstances warrant, the court may simply order the correction of
unverified pleadings or act on it and waive strict compliance with the rules
ISSUE/s: in order that the ends of justice may thereby be served.
120.  (Issue of prescription) WoN Fortune correctly invoked Section 384 of the 129.  In the case of answers to written requests for admission particularly, the
Insurance Code – YES. Fortune, in invoking prescription, was not referring court can allow the party making the admission, whether made expressly or
to the one-year period from the denial of the claim within which to file an deemed to have been made impliedly, "to withdraw or amend it upon such
action against an insurer but obviously to the written notice of claim that terms as may be just
had to be submitted within six months from the time of the accident 130.  The CA acted neither erroneously nor with grave abuse of discretion when
121.  (Life insurance) WoN there is sufficient evidence to prove that the insured it seconded the court a quo and ruled that there is absolutely no basis in fact
suffered from an accidental death, the risk covered by the policy - and in law for the lower court to hold that Fortune waived the defense, that
the death of Gabriel was not caused by violent accidental external and
RULING: SC affirmed the CA’s decision visible means61

RATIO:                                                                                                                        
61
122.  Section 384 of the Insurance Code states: CA’s ruling:
Sec. 384. Any person having any claim upon the policy issued pursuant to this "As to the allegation of the plaintiff-appellant that the matters requested by her to be
chapter shall, without any unnecessary delay, present to the insurance company admitted by the defendant-appellant under the Request for Admission were already
deemed admitted by the latter for its failure to answer it under oath, has already been
131.  The insurance policy expressly provided that to be compensable, the injury 139.  Evidence is utterly wanting to establish that the insured suffered from
or death should be caused by “violent accidental external and visible an accidental death, the risk covered by the policy.
means.” 140.  In an accident insurance, the insured's beneficiary has the burden of
132.  In attempting to prove the cause of Gabriel’s death, all that petitioner proof in demonstrating that the cause of death is due to the covered
submitted were a letter sent to her by Gabriel’s co-worker, stating that peril.
Gabriel died when he tried to haul water out of a tank while its submerged 141.  Once the fact is established, the burden then shifts to the insurer to
motor was still functioning, and Jacqueline’s sinupaang salaysay show any excepted peril that may have been stipulated by the parties.
133.  The sinumpaang salaysay merely confirmed the receipt and stated contents 142.  An "accident insurance" is not thus to be likened to an ordinary life
of the letter insurance where the insured's death, regardless of the cause thereof,
134.  With this the CA said that the affidavit suffers from procedural infirmity as would normally be compensable.
it was not even testified to or identified by the affiant (Jacqueline) herself. 143.  The latter is akin in property insurance to an "all risk" coverage where
This self-serving affidavit therefore is a mere hearsay under the rules the insured, on the aspect of burden of proof, has merely to show the
135.  Similar with the letter which was never identified to in court by the condition of the property insured when the policy attaches and the fact
supposed author, suffers from the same defect as the affidavit of loss or damage during the period of the policy and where, thereafter,
136.  Not one of the other documents submitted, to wit, the POEA decision, dated the burden would be on the insurer to show any "excluded peril."
06 June 1984, the death certificate issued by the Ministry of Health of Iraq 144.  When, however, the insured risk is specified, like in the case, it lies with
and the NBI autopsy report, could give any probative value to the claim. the claimant of the insurance proceeds to initially prove that the loss is
137.  The POEA decision did not make any categorical holding on the specific caused by the covered peril.
cause of Gabriel's death. Neither did the death certificate issued by the 145.  While petitioner did fail in substantiating her allegation that the death of her
health authorities in Iraq nor the NBI autopsy report provide any clue on the husband was due to an accident, considering, however, the uncertainty on
cause of death. the real cause of death, Fortune might find its way clear into still taking a
138.  All that appeared to be clear was the fact of Gabriel's demise on 22 May second look on the matter and perhaps help ease the load of petitioner's loss.
1982 in Iraq.

                                                                                                                                                                                                                                                                                       
properly laid to rest when the lower court in its Order of May 28, 1987 correctly
ruled:
"At the outset, it must be stressed that the defendant indeed led a written
answer to the request for admission, sans veri cation. The case of Motor
Service Co., Inc. vs. Yellow Taxicab Co., Inc., et al.may not therefore be
controlling, or actually opposite. In said case, there was an absolute failure
on the part of the defendant to answer the request for admission, and thus
the court was justi ed in rendering a summary judgment. Here, however, as
clearly intimated elsewhere above, the defendant answered in writing
practically every question posed in the request for admission. The Court
believes, under the peculiar circumstance, that the more controlling
jurisprudence on the matter would be those cited by the defendant in its
memorandum, particularly the case of Quimpo vs. de la Victoria, 46 SCRA
139.'
"Prescinding from the foregoing, there is absolutely no basis in fact and in law for
the lower court to hold that the appellant insurance company was deemed to have
waived the defense, that the death of plaintiff-appellant's husband was not caused by
violent accidental external and visible means' as contemplated in the insurance
policy. The Death Certi cate (Exh. 9) and the Autopsy Report (Exh. 10), more than
controverted the allegation of the plaintiff-appellant as to the cause of death of her
husband."
002 INSULAR LIFE v. FELICIANO ET AL. (PUNSALAN)
Dec. 29, 1943 | Ozaeta, J. | Life Insurance; GG sa TB FACTS:
170.  This case previously had a 4-3 decision promulgated on Sept. 13, 1941
PETITIONER: The Insular Life Assurance Co., Ltd. where the SC affirmed the CA decision in favor of Serafin Feliciano et al.
RESPONDENTS: Serafin D. Feliciano et al and against The Insular Life Assurance Co., Ltd. (Insular) for P25k which
represents the value of 2 insurance policies issued by Insular on the life of
SUMMARY: Evaristo Feliciano died from advanced pulmonary tuberculosis. Evaristo Felciano.
Before dying though, he signed 2 applications for insurance with Insular Life. a.   An MR has been filed by Insular which is the subject of this case.
According to doctors who informed his brother, Evaristo was “in a very serious 171.  Evaristo Feliciano died on Sept. 29, 1935 from advanced pulmonary
and practically hopeless condition.” (sad) He disclosed this to Insular Life’s tuberculosis.
soliciting agent Romulo David and Medical Examiner Dr. Gregorio Valdez 172.  Before dying though, Evaristo signed his applications for insurance with
verbally and that he also had x-rays taken because of coughing and fever. Insular Life on Oct. 12, 1934.
Nevertheless, Agent David and Dr. Valdez falsified the answers to the questions a.   On that same date, Doctor Trepp (who took X-ray pictures of his
of the applications and stated that he appears to be in good health and that he is a lungs) informed Dr. Serafin Feliciano (brother of Evaristo) that
proper subject for life insurance. (IMPT: pls check fact 3, 7, and 8 to have an “Evaristo was already in a very serious and practically
idea of the contents of the application and the policy). The beneficiaries were hopeless condition.”
able to get the value of the insurance policies upon his death worth P25k (policy b.   Nevertheless, the question contained in the application – “Have
1 – P20k and policy 2 – P5k). The SC decided this 4-3 in favor of Felicianos and you ever suffered from any ailment or disease of the lungs,
now Insular Life files this MR. pleurisy, pneumonia or asthma?” – appears to have been answered,
Issue: WoN Felicianos are only entitled to the refund of the premiums(P1,389) “No”
based on the facts – YES, because insured Evaristo was found to be a c.   Lastly, above the signature of the applicant, following the answers
coparticipant and coresponsible with Agent David and Medical Examiner Valdez to the various questions asked of him, is the ff. printed statement:
in the fraudulent procurement of said policies in question and that by reason “I declare on behalf of myself and of any person who shall have or
thereof said policies are void ab initio. claim any interest in any policy issued hereunder, that each of the
When Evaristo signed the application in blank and authorized Agent David above answers is full, complete and true, and that to the best of my
and/or Dr. Valdez of Insular to write the answers for him, he made them his knowledge and belief I am a proper subject for life insurance.
own agents for that purpose, and was responsible for their acts in that 173.  One of Insular’s soliciting agent Romulo David (Agent David) wrote the
connection. He could not evade responsibility for: (1) He was not supposed false answer (in collusion with Medical Examiner Dr. Gregorio Valdez) in
to sign the application in blank (2) He knew that the answers to the order to secure Insular’s approval of the application so that the policy to be
questions would be the “basis of the policy” and for that, every reason he issued might be credited to Agent David in connection with the inter-
was required with his signature to vouch for the truth thereof. Evaristo not provincial contest which Insular was then holding among its soliciting
only accepted the first policy of P20k but then and there applied for another agents to boost the sales of its policies. (competitive af)
one and later accepted another P5k. He must also have had notice that the 174.  CA found that Agent David bribed Dr. Valdez with money which the
answers to the questions asked concerning his clinical history were false, and yet former got from Evaristo’s mother by way of advanced payment on the
he accepted the first policy and applied for another. Based on the policy [ratio premium.
6(a)], By accepting the policy, he became charged with knowledge of its 175.  CA also found that before he signed the application, Evaristo and his family
contents, whether or not he actually read it or not. Insured Evaristo then had no told Agent David and Dr. Valdez that he had been sick and coughing for
right to rely in good faith upon the oral representation of the agent and medical some time and had gone 3 times to the Santol Sanatorium and had taken X-
examiner as well as the oral representation that he was a “fit subject for ray pics of his lungs
insurance” notwithstanding that he had been and was still suffering with a.   Despite the information, Agent David and Dr. Valdez told
advanced pulmonary tuberculosis. them that applicant Evaristo was a fit subject for insurance
176.  The two policies subject of the case contains the ff. stipulations: (note
DOCTRINE: When the applicant signs the application for insurance in blank italicized part)
and authorized the soliciting agents to write answers for him, he made them his a.   “This policy and the application herefor constitute the entire
own agents for that purpose and was responsible for their acts. contract between the parties hereto… Only the President, or the
Fradulent procurement of the insurance policy amounts to it being void ab initio. Manager, acting jointly with the Secretary or Assistant Secretary
(and then only in writing signed by them) have power in behalf of
the Company to issue permits, or to modify this or any contract, or RATIO:
to extend the same time for making any premium payment, and the 145.  When Evaristo signed the application in blank and authorized Agent David
Company shall not be bound by any promise or representation and/or Dr. Valdez of Insular to write the answers for him, he made them
heretofore or hereafter given by any person other than the above- his own agents for that purpose, and was responsible for their acts in
named officials, and by them only in writing and signed conjointly that connection.
as stated.” 146.  If they falsified the answers for him, he could not evade the responsibility
177.  The application contains the ff. statements: (note italicized parts) for the falsification.
a.   18. — I [the applicant] hereby declare that all the above statements a.   He was not supposed to sign the application in blank
and answers as well as all those that I may make to the Company's b.   He knew that the answers to the questions would be the “basis
Medical Examiner in continuation of this application, to be of the policy”
complete, true and correct to the best of my knowledge and belief, c.   For that, every reason he was required with his signature to
and I hereby agree as follows: vouch for the truth thereof.
1. That his declaration, with the answers to be given by me to the 147.  SC also cannot escape from the conclusion that the insured Evaristo acted in
Medical Examiner, shall be the basis of the policy and form part of connivance with the soliciting agent and the medical examiner of the
same. Company in accepting the policies in question. (IMPT)
a.   Above the signature of applicant is the printed statement of the
xxx xxx xxx representation: “… I am a proper subject for life insurance.”
3. That the said policy shall not take effect until the first premium b.   In another sheet of the same application and above the signature of
has been paid and the policy has been delivered to and accepted by the applicant was also printed this statement: “That the said
me, while I am in good health. policy shall not take effect until the first premium has been
4. That the agent taking this application has no authority to make, paid and the policy has been delivered to and accepted by me,
modify or discharge contracts, or to waive any of the Company's while I am in good health.”
rights or requirements. c.   (IRONIC) When the applicant signed the application he was
having difficulty in breathing with a very high fever.
5. My acceptance of any policy issued on this application will d.   Stated in Fact#6 that he went to Santol Sanatorium for x-rays
constitute a ratification by me of any corrections in or additions to which proves that he knows that he is not a proper subject for
this application made by the Company in the space provided "For life insurance.
Home Office Corrections or Additions Only." I agree that e.   When he accepted the policy, he knew that he was not in good
photographic copy of this applications as corrected or added to health.
shall constitute sufficient notice to me of the changes made. 148.  Nevertheless, he not only accepted the first policy of P20k but then and
(Emphasis added.) there applied for another one and later accepted another P5k.
178.  Insular insists based on the facts of the case that the policies are null and 149.  SC cannot believe that the insured did not take the trouble to read the
void ab initio and that all that Felicianos are entitled to is the refund of the answers in the copy of the application before he accepted it and paid the
premiums paid. (P1,389) premium thereon.
a.   He must also have had notice that the answers to the questions
ISSUE/s: asked concerning his clinical history were false, and yet he
19.   WoN Felicianos are only entitled to the refund of the premiums(P1,389) accepted the first policy and applied for another.
based on the facts – YES, because insured Evaristo was found to be a 150.  In any event, he obligated himself to read the policy when he subscribed to
coparticipant and coresponsible with Agent David and Medical Examiner this statement:
Valdez in the fraudulent procurement of said policies in question and that a.   "My acceptance of any policy issued on this application will
by reason thereof said policies are void ab initio. constitute a ratification by me of any corrections in or additions to
this application made by the Company . . ."
RULING: MR is sustained and CA judgment reversed. Another judgment entered in b.   By accepting the policy, he became charged with knowledge of
favor of Feliciano and against Insular for the refund of the premiums amounting to its contents, whether or not he actually read it or not.
P1,389. c.   He could not ostrich-like hide his head from it in order to avoid his
part of the bargain and at the same time claim the benefit thereof. (nice)
151.  He knew (or was chargeable with knowledge) from the very terms of the 2
policies (policies printed: one in English and one in Spanish) that the agent
and medical examiner cannot bind Insular by any verbal promise or oral
representation.
152.  Insured Evaristo then had no right to rely in good faith upon the oral
representation of the agent and medical examiner as well as the oral
representation that he was a “fit subject for insurance” notwithstanding that
he had been and was still suffering with advanced pulmonary tuberculosis.

Separate and dissenting opinions


153.  CJ Yulo, separate: To serve the ends of justice, the case of Felicianos
should be removed from the protection of such rules.
154.  The beneficiaries contend that they took the policies on the basis of a life
expectancy of a person gravely stricken by tuberculosis. They consistently
stated that they so informed the agents of the insurance company but the
policies issued on the life of Evaristo were as a perfectly normal and healthy
person. The error is vital and is based on the contract itself. Who is
responsible for the error?
a.   The direct cause: false recitals in the application for insurance.
b.   While true that it was agents who filled out the application, it was
Evaristo who made it possible to procure such by signing the
application in blank.
155.  In justice and in equity, the responsibility for the falsifications made by
the insurance agents should be laid at the door of the assured and his
beneficiaries.

156.  J. Hontiveros, dissenting: The mere fact that insured Evaristo signed at
the bottom of the application for insurance when some of its lines
intended for answers to certain questions were still in blank, answers
which, accdg to evidence and findings of the lower courts, he had
grounds to believe wil be made in accordance which he and his family
had given to Agent David and Dr. Valdez, does not convert these two
persons into agents of the insured in a way as to make the Evaristo
liable for the former’s acts.
a.   The fact that copies of the forms which are attached to the policies
are almost illegible should have been taken into account together
with the fact that Evaristo does not know English, the language in 003 SUNLIFE ASSURANCE v. CA (RAYOS)
which those documents are written. June 22, 1995| Quiason J.. | Life Insurance
b.   Certain authorities cited supports this dissent that insured is not PETITIONER: Sunlife assurance Company of Canada
bound to know the contents of the application if practically RESPONDENTS: CA & Spouses Rolando and Bernarda
illegible and that insured may also rely on agent’s assurances that SUMMARY:Robert Bacani procured a life insurance contract with Sunlife the
his answers have been correctly written. This has special force beneficiary was his mother, Bernarda. A year later, Robert died in a plane crash.
where insured is illiterate and unable to read, or is ignorant of the Bernarda filed a claim seeking payment from the insurance policy. Sunlife
language. REFUSED on the ground that Robert did not disclose material facts relevant to the
157.  J. Imperial, dissenting: I concur in the foregoing dissenting opinion. issuance of the policy, thus making the policy voidable. Sunlife contended that
Robert gave false statements in his application; he did not disclose that he was iv.   Have you ever had or sought advice for?
diagnosed with renal (kidney) failure. Bernarda filed an action for specific v.   Urine, kidney or bladder disorder?
performance with the RTC. RTC ruled in favor of Bernarda. RTC concluded that 5.   The deceased answered question No. 5(a) in the affirmative but limited his
the Facts concealed were made in ‘good faith’ and under a belief that they need answer to a consultation with a certain Dr. Reinaldo D. Raymundo of the
not be disclosed, and other has no means of finding out. CA affirmed the decision Chinese General Hospital on February 1986, for cough and flu
and conclusion of the RTC. CA added that the cause of death was unrelated to the complications. The other questions were answered in the negative.
facts concealed. Issue: Whether or not the CA erred in upholding the ruling of 6.   Sunlife discovered that 2 weeks prior to his application for insurance, the
the RTC? YES, the facts concealed by Robert were material. The materiality insured (ROBERT) was examined and confined at the Lung Center of the
of the fact to the insurance contract is not determined by the event, but solely by Philippines, where he was diagnosed for renal failure. During his
the probable and reasonable influence of it to the other party in weighing his confinement, the deceased was subjected to urinalysis, ultra-sonography
disadvantages in entering in the contract. (IF IT AFFECTS the decision-making and hematology tests.
capacity of a person in entering the contract then it is a material fact). In the terms 7.   On November 17, 1988, respondent Bernarda Bacani and her husband,
of the contract Robert had to disclose all matters relating to his health. SC respondent Rolando Bacani, filed an action for specific performance against
established that good faith is not a defense in concealment. It was also established Sunlife Assurance.
that Robert found about the sickness 2 weeks before applying for the insurance 8.   Sunlife filed its answer with counterclaim and a list of exhibits consisting of
policy. It appeared that the concealment was deliberate. The waiver of medical medical records furnished by the Lung Center of the Philippines.
examination by Sunlife is of no moment because the medical results will still 9.   Bernarda filed a "Proposed Stipulation with Prayer for Summary Judgment"
affect Sunlife’s decision in approving the insurance policy or even giving a higher where they manifested that they "have no evidence to refute the
premium to pay. Sunlife properly rescinded the insurance contract within the 2 documentary evidence of concealment/misrepresentation by the decedent of
year period and returned to Bernarda the premium paid by Robert. his health condition.
DOCTRINE: The information withheld by Robert were material and relevant to 10.   Sunlife Assurance filed its Request for Admissions relative to the
the approval and issuance of the insurance policy. Section 26 provides: “A neglect authenticity and due execution of several documents as well as allegations
to communicate that which a party knows and ought to communicate, is called regarding the health of the insured. Bernarda failed to oppose said request
concealment.” Sec. 26 of the Insurance Code explicitly requires that parties of an or reply thereto, thereby rendering an admission of the matters alleged.
insurance contract have to communicate to each other, in good faith, all facts 11.   Sunlife Assurance then moved for a summary judgment and the trial court
within their knowledge which are material to the contract and which the other has decided in favor of Bernarda.
no means of finding out. 12.   Defendant’s counterclaim is Dismissed.
FACTS: 13.   Trial Court: concluded that the facts concealed by the insured were made in
good faith and under a belief that they need not be disclosed. Moreover, it
1.   Robert Bacani procured a LIFE INSURANCE for himself from Sunlife. He held that the health history of the insured was immaterial since the
was issued a policy valued at 100 thousand, with double indemnity in case insurance policy was "non-medical".
of accidental death. The designated BENEFICIARY was his mother, 14.   Sunlife appealed to the CA, which affirmed the decision of the Trial Court.
BERNARDA BACANI. 15.   CA ruled that Sunlife cannot avoid its obligation by claiming concealment
2.   BACANI died in a plane crash. Bernarda Bacani filed a claim with Sunlife, because the cause of death was unrelated to the facts concealed by the
seeking the benefits of the insurance policy taken by her son. Sunlife insured. It also sustained the finding of the trial court that matters relating to
conducted an investigation and its findings prompted it to reject the claim. the health history of the insured were irrelevant since SUNLIFE waived the
3.   Sunlife informed Bernarda Bacani, that the insured did not disclose material medical examination prior to the approval and issuance of the insurance
facts relevant to the issuance of the policy, thus rendering the contract of policy. Moreover, the appellate court agreed with the trial court that the
insurance voidable. A check representing the total premiums paid in the policy was "non-medical”
amount of P10,172.00 was attached to said letter. 16.   Petitioner’s MR was denied; hence this petition.
4.   Sunlife claimed that the insured gave false statements in his application 17.   This is a petition for review for certiorari under Rule 45 of the Revised
when he answered the following questions: Rules of Court to reverse and set aside the Decision dated February 21,
a.   Within the past 5 years have you: 1992 of the Court of Appeals in CA-G.R. CV No. 29068, and its Resolution
i.   Consulted any doctor or other health practitioner? dated April 22, 1992, denying reconsideration thereof.
ii.   Submitted to EGG? X-Rays? Blood tests? Other tests? ISSUE: Whether or not the CA erred in upholding the ruling of the RTC? YES,
iii.   Attended or been admitted to any hospital or other the facts concealed by Robert were material. The rule that factual findings of the
medical facility?
lower court and the appellate court are binding on this Court is not absolute and 8.   The facts concealed had no bearing to the cause of death of the insured, it is
admits of exceptions, such as when the judgment is based on a misappreciation of the well settled that the insured need not die of the disease he had failed to
facts. We, therefore, rule that SUNLIFE properly exercised its right to rescind the disclose to the insurer. It is sufficient that his non-disclosure misled the
contract of insurance by reason of the concealment employed by the insured. It must insurer in forming his estimates of the risks of the proposed insurance
be emphasized that rescission was exercised within the two-year contestability policy or in making inquiries.
period as recognized in Section 48 of The Insurance Code. 9.   We, therefore, rule that SUNLIFE properly exercised its right to
rescind the contract of insurance by reason of the concealment
HELD: WHEREFORE, the petition is GRANTED and the Decision of the Court of employed by the insured. It must be emphasized that rescission was
Appeals is REVERSED and SET ASIDE. SO ORDERED. exercised within the two-year contestability period as recognized in
RATIO: Section 48 of The Insurance Code.

1.   Sec. 26 of The Insurance Code is explicit in requiring a party to a contract


of insurance to communicate to the other, in good faith, all facts within his
knowledge which are material to the contract and as to which he makes no
warranty, and which the other has no means of ascertaining. Said Section
provides: A neglect to communicate that which a party knows and ought to
communicate, is called concealment.
2.   Materiality is to be determined not by the event, but solely by the probable
and reasonable influence of the facts upon the party to whom
communication is due, in forming his estimate of the disadvantages of the
proposed contract or in making his inquiries.
3.   The terms of the contract are clear. The insured is specifically required to
disclose to the insurer matters relating to his health.
4.   The information which the insured failed to disclose were material and
relevant to the approval and issuance of the insurance policy. The matters
concealed would have definitely affected petitioner's action on his
application, either by approving it with the corresponding adjustment for a
higher premium or rejecting the same. Moreover, a disclosure may have
warranted a medical examination of the insured by petitioner in order for it
to reasonably assess the risk involved in accepting the application.
5.   Important doctrines:
a.   Materiality of the information withheld does not depend on the
state of mind of the insured. Neither does it depend on the 004 Del Val v. Del Val (Rosales)
actual or physical events which ensue. Thus, "good faith" is no February 16, 1915 | Moreland, J. | Life Insurance
defense in concealment. The insured's failure to disclose the
fact that he was hospitalized for two weeks prior to filing his
application for insurance, raises grave doubts. It appears that
such concealment was deliberate on his part.
6.   The waiver of a medical examination [in a non-medical insurance contract]
renders even more material the information required of the applicant
concerning previous condition of health and diseases suffered, for such
information necessarily constitutes an important factor which the insurer
takes into consideration in deciding whether to issue the policy or not.
7.   Such argument of private respondents would make Section 27 of the
Insurance Code, which allows the injured party to rescind a contract of
insurance where there is concealment, ineffective.
PETITIONER: Francisco Del Val, et al and, after a partial administration, it was closed and the administrator
RESPONDENTS: Andres Del Val discharged by order of the Court of First Instance dated December 9, 1911;
that during the lifetime of the deceased he took out insurance on his life for
SUMMARY: Gregorio Nacianceno del Val had a life insurance of P40,000 the sum of P40,000 and made it payable to Andres del Val as sole
naming as sole beneficiary his son Andres Del Val who used the insurance beneficiary; that after his death the Andres collected the face of the policy;
money to repurchase his estate for P18,365.20 and keeping the balance of the that of said policy he paid the sum of P18,365.20 to redeem certain real
insurance of P21,634.80. Andres also did the same to the personal properties in estate which the decedent had sold to third persons with a right to
his possession. Francisco Del Val, et al., brothers and sisters, contended that the repurchase; that the redemption of said premises was made by the attorney
insurance claim as well as the personal properties should be given to the estate of Andres in the name of Francisco et al and Andres as heirs of the deceased
and not to Andres. Francisco et al contend that the amount of the insurance vendor; that the redemption of said premises they have had the use and
policy belonged to the estate of the deceased and not to Andres personally; and benefit thereof; that during that time the Francisco paid no taxes and made
they are entitled to a partition not only of the real and personal property, but also no repairs.
of the P40,000 life insurance. The trial court dismissed the action stating that it is 2.   It further appears from the pleadings that Andres on the death of the
an action for partition between co-heirs and the complaint fails to comply with deceased, took possession of most of his personal property, which he still
Code Civ., Pro. sec. 183, in that it does not 'contain an adequate description of has in his possession, and that he has also the balance on said insurance
the real property of which partition is demanded. Since the estate was finally policy amounting to P21,634.80.
closed, the matter of the personal property at least must be considered res 3.   Francisco et al contend that the amount of the insurance policy belonged to
judicata. Issue is WoN the proceeds of the life-insurance policy belongs the estate of the deceased and not to Andres personally; that, therefore, they
exclusively to Andres? SC held YES, because the proceeds of an insurance are entitled to a partition not only of the real and personal property, but also
policy belong exclusively to the beneficiary and not to the estate of the person of the P40,000 life insurance. The complaint prays a partition of all the
whose life was insured, and that such proceeds are the separate and individual property, both real and personal, left by the deceased; that Andres account
property of the beneficiary, and not of the heirs of the person whose life was for P21,634.80, and that that sum be divided equally among Francisco et al
insured. The proceeds of the life-insurance policy being the exclusive property and Andres along with the other property of deceased.
of Andres and he having used a portion thereof in the repurchase of the real 4.   Andres denies the material allegations of the complaint and sets up as
estate sold by the decedent prior to his death with right to repurchase, and such special defense and counterclaim that the redemption of the real estate sold
repurchase having been made and the conveyance taken in the names of all of by his father was made in the name of Francisco et al and himself instead of
the heirs instead of the Andres alone, Francisco et al’s claim that the property in his name alone without his knowledge or consent; and that it was not his
belongs to the heirs in common and not to Andres alone. If it is established by intention to use the proceeds of the insurance policy for the benefit of any
the evidence that that was his intention and that the real estate was delivered person but himself, he alleging that he was and is the sole owner thereof and
Francisco et al with that understanding, then it is probable that their contention is that it is his individual property. He, therefore, asks that he be declared the
correct and that they are entitled to share equally with Andres therein. If, owner of the real estate redeemed by the payment of the P18,365.20, the
however, it appears from the evidence in the case that the conveyances were owner of the remaining P21,634.80, the balance of the insurance policy, and
taken in the name of Francisco et al without his knowledge or consent, or that it that Francisco’s account for the use and occupation of the premises so
was not his intention to make a gift to them of the real estate, then it belongs to redeemed since the date of the redemption.
him. *The case was remanded to the trial court 5.   The learned trial court refused to give relief to either party and dismissed
the action.
DOCTRINE: That the proceeds of an insurance policy belong exclusively to the 6.   It says in its opinion: "This purports to be an action for partition, brought
beneficiary and not to the estate of the person whose life was insured, and that against an heir by his coheirs. The complaint, however, fails to comply with
such proceeds are the separate and individual property of the beneficiary, and not Code Civ., Pro. sec. 183, in that it does not 'contain an adequate
of the heirs of the person whose life was insured. description of the real property of which partition is demanded.'
Because of this defect (which has not been called to our attention and was
discovered only after the cause was submitted) it is more than doubtful
FACTS:
whether any relief can be awarded under the complaint, except by
1.   The pleadings set forth that the Francisco et al and Andres del Val are
agreement of all the parties." This alleged defect of the complaint was made
brother and sisters; that they are the only heirs at law and next of kin of
one of the two bases for the dismissal of the action.
Gregorio Nacianceno del Val, who died in Manila on August 4, 1910,
ISSUE/s:
intestate; that an administrator was appointed for the estate of the deceased,
20.   WoN the proceeds of the life-insurance policy belongs exclusively to 4.   We cannot agree with these contentions. The contract of life insurance is a
Andres – YES, because the proceeds of an insurance policy belong special contract and the destination of the proceeds thereof is determined by
exclusively to the beneficiary and not to the estate of the person whose life special laws which deal exclusively with that subject. The Civil Code has
was insured, and that such proceeds are the separate and individual property no provisions which relate directly and specifically to life- insurance
of the beneficiary, and not of the heirs of the person whose life was insured. contracts or to the destination of life insurance proceeds. That subject is
21.   WoN the defect of the complaint should dismiss the action – NO, because regulated exclusively by the Code of Commerce which provides for the
if, on the trial of the cause, evidence is offered which establishes the cause terms of the contract, the relations of the parties and the destination of the
of action which the complaint intended to allege, and such evidence is proceeds of the policy.
received without objection, the defect is thereby cured and cannot be made 5.   The proceeds of the life-insurance policy being the exclusive property of
the ground of a subsequent objection. Andres and he having used a portion thereof in the repurchase of the real
estate sold by the decedent prior to his death with right to repurchase, and
RULING: The judgment appealed from is set aside and the cause returned to the such repurchase having been made and the conveyance taken in the names
Court of First Instance whence it came for the purpose hereinabove stated. So of all of the heirs instead of the Andres alone, Francisco et al’s claim that
ordered. the property belongs to the heirs in common and not to Andres alone.
6.   We are not inclined to agree with this contention unless the fact appear or
RATIO: be shown that Andres acted as he did with the intention that the other heirs
1.   ISSUE 1: With the finding of the trial court that the proceeds of the life- should enjoy with him the ownership of the estate — in other words, that he
insurance policy belong exclusively to Andres as his individual and separate proposed, in effect, to make a gift of the real estate to the other heirs. If it is
property, we agree. That the proceeds of an insurance policy belong established by the evidence that that was his intention and that the real
exclusively to the beneficiary and not to the estate of the person whose estate was delivered Francisco et al with that understanding, then it is
life was insured, and that such proceeds are the separate and individual probable that their contention is correct and that they are entitled to share
property of the beneficiary, and not of the heirs of the person whose life equally with Andres therein. If, however, it appears from the evidence in the
was insured, is the doctrine in America. We believe that the same doctrine case that the conveyances were taken in the name of Francisco et al without
obtains in these Islands by virtue of section 428 of the Code of Commerce, his knowledge or consent, or that it was not his intention to make a gift to
which reads: them of the real estate, then it belongs to him. If that facts are as stated, he
a.   The amount which the underwriter must deliver to the person has two remedies. The one is to compel Francisco et al to reconvey to him
insured, in fulfillment of the contract, shall be the property of the and the other is to let the title stand with them and to recover from them the
latter, even against the claims of the legitimate heirs or creditors of sum he paid on their behalf.
any kind whatsoever of the person who effected the insurance in 7.   ISSUE 2: We do not regard this as sufficient reason for dismissing the
favor of the former. action. It is the doctrine of this court, set down in several decisions,
2.   It is claimed by the attorney for Francisco et al that the section just quoted is Lizarraga Hermanos vs. Yap Tico, 24 Phil. Rep., 504, that, even though the
subordinate to the provisions of the Civil Code as found in article 1035. complaint is defective to the extent of failing in allegations necessary to
This article reads: constitute a cause of action, if, on the trial of the cause, evidence is offered
a.   An heir by force of law surviving with others of the same character which establishes the cause of action which the complaint intended to
to a succession must bring into the hereditary estate the property or allege, and such evidence is received without objection, the defect is
securities he may have received from the deceased during the life thereby cured and cannot be made the ground of a subsequent objection. If,
of the same, by way of dowry, gift, or for any good consideration, therefore, evidence was introduced on the trial in this case definitely and
in order to compute it in fixing the legal portions and in the clearly describing the real estate sought to be partitioned, the defect in the
account of the division. complaint was cured in that regard and should not have been used to
3.   Counsel also claim that the proceeds of the insurance policy were a dismiss the action. We do not stop to inquire whether such evidence was or
donation or gift made by the father during his lifetime to Andres and that, as was not introduced on the trial, inasmuch as this case must be turned for a
such, its ultimate destination is determined by those provisions of the Civil new trial with opportunity to both parties to present such evidence as is
Code which relate to donations, especially article 819. This article provides necessary to establish their respective claims.
that "gifts made to children which are not betterments shall be considered as 8.   The court in its decision further says: "It will be noticed that the provision
part of their legal portion." above quoted refers exclusively to real estate. . . . It is, in other words, an
exclusive real property action, and the institution thereof gives the court no
jurisdiction over chattels. . . . But no relief could possibly be granted in this subsequent action to require a division of either the real or personal
action as to any property except the last (real estate), for the law property. If, on the other hand, an order had been made in the
contemplated that all the personal property of an estate be distributed before administration proceedings dividing the personal or the real property, or
the administration is closed. Indeed, it is only in exceptional cases that the both, among the heirs, then it is quite possible that, to a subsequent action
partition of the real estate is provided for, and this too is evidently intended brought by one of the heirs for a partition of the real or personal property, or
to be effected as a part of the administration, but here the complaint alleges both, there could have been interposed a plea of res judicata based on such
that the estate was finally closed on December 9, 1911, and we find upon order. As the matter now stands, however, there is no ground on which to
referring to the record in that case that subsequent motion to reopen the base such a plea. Moreover, no such plea has been made and no evidence
same were denied; so that the matter of the personal property at least must offered to support it.
be considered res judicata (for the final judgment in the administration 13.   For the complete and proper determination of the questions at issue in this
proceedings must be treated as concluding not merely what was case, we are of the opinion that the cause should be returned to the trial
adjudicated, but what might have been). So far, therefore, as the personal court with instructions to permit the parties to frame such issues as will
property at least is concerned, Francisco et al's only remedy was an appeal permit the settlement of all the questions involved and to introduce such
from said order." evidence as may be necessary for the full determination of the issues
9.   We do not believe that the law is correctly laid down in this quotation. The framed. Upon such issues and evidence taken thereunder the court will
courts of the Islands have jurisdiction to divide personal property between decide the questions involved according to the evidence, subordinating his
the common owners thereof and that power is as full and complete as is the conclusions of law to the rules laid down in this opinion.
power to partition real property. If an actual partition of personal property 14.   We do not wish to be understood as having decided in this opinion any
cannot be made it will be sold under the direction of the court and the question of fact which will arise on the trial and be there in controversy.
proceeds divided among the owners after the necessary expenses have been The trial court is left free to find the facts as the evidence requires. To the
deducted. facts as so found he will apply the law as herein laid down.
10.   The administration of the estate of the decedent consisted simply, so far as
the record shows, in the payment of the debts. No division of the property,
either real or personal, seems to have been made. On the contrary, the
property appears, from the record, to have been turned over to the heirs in
bulk. The failure to partition the real property may have been due either to
the lack of request to the court by one or more of the heirs to do so, as the
court has no authority to make a partition of the real estate without such
request; or it may have been due to the fact that all the real property of
decedent had been sold under pacto de retro and that, therefore, he was not
the owner of any real estate at the time of his death.
11.   As to the personal property, it does not appear that it was disposed of in the
manner provided by law. (Sec. 753, Code of Civil Procedure.) So far as this
action is concerned, however, it is sufficient for us to know that none of the
property was actually divided among the heirs in the administration
proceeding and that they remain coowners and tenants-in- common thereof
at the present time. To maintain an action to partition real or personal
property it is necessary to show only that it is owned in common.
12.   The order finally closing the administration and discharging the
administrator, referred to in the opinion of the trial court, has nothing to do
with the division of either the real or the personal property. The heirs have
the right to ask the probate court to turn over to them both the real and
personal property without division; and where that request is unanimous it
is the duty of the court to comply with it, and there is nothing in section 753
of the Code of Civil Procedure which prohibits it. In such case an order
finally settling the estate and discharging the administrator would not bar a
005 PINEDA v. CA (Gueco c/o Daguman edited by Sabaupan) non-contributory plan, the payment by the employer of the entire premium is a
September 27, 1993 | Davide Jr., J. | Life Insurance part of the total compensation paid for the services of the employee. Put
differently, the labor of the employees is the true source of the benefits, which
PETITIONERS: LUZ PINEDA, MARILOU MONTENEGRO, VIRGINIA are a form of additional compensation to them.
ALARCON, DINA LORENA AYO, CELIA CALUMBAG and LUCIA
LONTOK, FACTS:
RESPONDENTS: HON. COURT OF APPEALS and THE INSULAR LIFE 179.  In 1983, Prime Marine Services Inc. (PMSI), a manning agency, procured a
ASSURANCE COMPANY, LIMITED Group Policy from Insular Life Assurance Co. (Insular Life) to provide life
insurance coverage to its sea-based employees enrolled under the plan.
SUMMARY: Prime Marine Services, Inc. (PMSI), a crewing/manning outfit, 180.  During the effectivity of the policy, six covered employees of the PMSI
procured a Group Policy from Insular Life Assurance Co., Ltd. to provide life perished at sea when their vessel sunk in Morocco. They were survived by
insurance coverage to its sea-based employees. During the effectivity of the Pineda et.al., the beneficiaries under the policy.
policy, six covered employees perished at sea when their vessel sunk. They were 181.  Pineda et. al, sought to claim death benefits so they approached the
survived by the petitioners in this case, the beneficiaries under the policy. The President and General Manager of PMSI, Capt. Nuval who evinced
beneficiaries, except the spouses Alarcon, executed SPAs authorizing Capt. willingness to assist Pineda to recover Overseas Workers Welfare
Nuval, President and General Manager of PMSI, to “follow-up, ask, demand, Administration (OWWA) benefits from the POEA arising from the deaths
collect and receive” for their benefit indemnities of sums of money due them of their husbands/sons.
relative to the sinking of the vessel. By virtue of these written powers of 182.  They executed, with the exception of spouses Alarcon, a special power of
attorney, beneficiaries were able to receive their respective death benefits (from attorney authorizing Capt. Nuval to “follow up, ask, demand, collect and
OWWA). Unknown to them, however, PMSI, in its capacity as employer and receive” for the benefit indemnities of sums due to them relative to the
policyholder of the life insurance of its deceased workers, filed with Insular Life sinking of the vessel. By virtue of this authority, they were able to receive
formal claims for and in behalf of the beneficiaries, through Capt. Nuval. On the death benefits.
basis of SPAs, Insular Life drew against its account 6 checks payable to the 183.  Unknown to them, PMSI, in its capacity as employer and policyholder of
order of the beneficiaries. Capt. Nuval, upon receipt of these checks endorsed the life insurance of the deceased workers, filed formal claims with Insular
and deposited them in his own account. When Pineda et.al. learned that they Life in behalf of the beneficiaries through Capt. Nuval who presented the
were entitled, as beneficiaries, to life insurance benefits under a group policy, SPAs executed by Pineda et.al.
they sought to recover these benefits from Insular Life but the latter denied their 184.  Insular Life drew against its bank six checks four for 200k each, one for 50k
claim on the ground that their liability was already extinguished. and another for 40k, payable to the order of Pineda et al. Upon receipt of the
checks, Capt. Nuval deposited them in his own account with Boston Bank.
The issue is whether Pineda et al. are entitled to the insurance proceeds despite 185.  Upon learning that they were entitled as beneficiaries under the group
the fact that Insular Life already issued checks payable to the beneficiaries. The policy with Insular Life, Pineda et. al. sought to recover the benefits but
Court ruled in the affirmative. The employer-policyholder is the agent of the Insular Life denied their claim on the ground that the liability of Insular Life
insurer. The most persuasive rationale for this is that the employee has no was already extinguished upon deliver to PMSI of the six checks issued
knowledge of or control over the employer's actions in handling the policy or its under their names.
administration. The insurer-employer relationship meets the agency test (element 186.  Pineda et.al. filed an administrative complaint against Insular Life before
of control) with regard to the administration of the policy, whereas that between the Insurance Commission. It ruled in favor of Pineda et. al and Insular Life
the employer and its employees fails to reflect true agency. The insurer directs was ordered to pay. Specifically, it found that:
the performance of the employer’s administrative acts, and if these duties are not a.   Insular Life erred in appreciating that the powers of attorney
undertaken properly, the insurer is in a position to exercise more constricted executed by the beneficiaries convey absolute authority to Capt.
control over the employer’s conduct. Since PMSI acted as the agent of Insular Nuval, to demand, receive, receipt and take delivery of the
Life, the insurance company is bound by the misconduct of its agent. Hence, its insurance proceeds. The SPAs do not contain in unequivocal terms
liability to pay the beneficiaries was not extinguished. authority to Capt. Nuval to obtain, receive, receipt from the
insurance company insurance proceeds arising from the death of
DOCTRINE: Although the employer may be the titular or named insured, the the insured. On the contrary, the said SPAs are couched in terms
insurance is actually related to the life and health of the employee. Indeed, the which could easily arouse suspicion of an ordinary man.
employee is in the position of a real party to the master policy, and even in a
b.   The testimony of complainant’s rebuttal witness who declared that 2.   Certainly, it would be highly imprudent to read into the SPAs in question
neither she nor her husband, executed a SPA in favor of Capt. the power to collect and receive the insurance proceeds due to Pineda et.al
Nuval, authorizing him to claim and receive of any insurance from Group Policy No. G-004694. Insular Life knew that a power of
proceeds from Insular Life arising out of the death of their attorney in favor of Capt. Nuval for the collection and receipt of such
insured/seaman son, is not convincingly refuted. proceeds was a deviation from its practice with respect to group policies.
c.   Insular Life did not observe Section 18062 of the Insurance Code The Insular Life’s Assistant Manager of the Group Administrative
when it issued two checks for the 3 minor children of complainant Department testified to the following facts:
Ayo and another check for minor beneficiary Marissa, there being a.   The company practice when it comes to claims pertaining to group
no showing of any court authorization presented or the requisite insurance is that the policyholder-employer is the one who files the
bond posted. claim for the beneficiaries of the deceased. The reason for this is
187.  On appeal to CA, the appellate court ruled that the SPAs relied upon by that the group insurance is normally taken by the employer as an
Insular Life were sufficient to authorize Capt. Nuval to receive the proceeds employee-benefit program and as such, the benefit should be
of the insurance pertaining to the beneficiaries. It held that the fact that there awarded by the policyholder to make it appear that the benefit
was a verbal agreement between Pineda et al. and Capt. Nuval limiting the really is given by the employer.
latter’s authority to claiming specified death benefits cannot prejudice the b.   Group insurance is a contract where a group of individuals are
insurance company which relied on the terms of the SPAs which on their covered under one master contract. The contract is between the
fac do not disclose such limitation. policyholder-employer and the insurance company. Insular Life do
188.  Hence, this petition. not have contractual obligations with the individual employees; it
is between Prime Marine and Insular Life. Hence, all inquiries,
ISSUE/s: follow-up, payment of claims, premium billings, etc. are always
2.   Whether Pineda et al. are entitled to the insurance proceeds despite the fact coursed through the policyholder and an SPA is not necessary for
that Insular Life already issued checks payable to the beneficiaries. YES, the payment of claims. The payment is coursed through the
because PMSI (acting through Capt. Nuval) is the agent of the insurance policyholder-employer but is paid ultimately to the beneficiary.
company, the latter is bound by the acts of Capt. Nuval. Since Capt. Nuval c.   The checks were given to the policyholder, in this case, PMSI
misappropriated the proceeds, Insular Life is still liable to pay the through Capt. Nuval, but such was only coursing them through.
beneificiaries.
3.   This practice is usual in the group insurance business and is consistent with
RULING: Petition GRANTED. Decision of CA is SET ASIDE and the decision of the jurisprudence thereon in the State of California — from whose laws the
the INSURANCE COMMISSION is REINSTATED. Philippine Insurance Code has been mainly patterned — which holds that
the employer-policyholder is the agent of the insurer.
RATIO: 4.   Group insurance is essentially a single insurance contract that provides
coverage for many individuals. In its original and most common form,
1.   The Court agreed with the Insurance Commission that the SPAs the special group insurance provides life or health insurance coverage for the
powers of attorney "do not contain in unequivocal and clear terms authority employees of one employer.
to Capt. Nuval to obtain, receive, receipt from respondent company a.   The coverage terms for group insurance are usually stated in a
insurance proceeds arising from the death of the seaman-insured. On the master agreement or policy that is issued by the insurer to a
contrary, the said powers of attorney are couched in terms which could representative of the group or to an administrator of the insurance
easily arouse suspicion of an ordinary man.” program, such as an employer.
b.   The employer acts as a functionary in the collection and payment
                                                                                                                       
of premiums and in performing related duties. Likewise falling
62
within the ambit of administration of a group policy is the
Section 180. x x x In the absence of a judicial guardian, the father, or in the latter's absence or
incapacity, the mother of any minor, who is an insured or a beneficiary under a contract of life, health or
disbursement of insurance payments by the employer to the
accident insurance, may exercise, in behalf of said minor, any right, under the policy, without necessity of employees.
court authority or the giving of a bond where the interest of the minor in the particular act involved does c.   Most policies, such as the one in this case, require an employee to
not exceed twenty thousand pesos . . . . pay a portion of the premium, which the employer deducts from
wages while the remainder is paid by the employer. This is known
as a contributory plan as compared to a non-contributory plan 9.   Insular Life, however, likewise recognized Capt. Nuval as the attorney-in-fact of Pineda et al.
Insular Life acted imprudently and negligently in the premises by relying without question on
where the premiums are solely paid by the employer.
the SPA. It is an established principle that that third persons deal with agents at their peril and
5.   Although the employer may be the titular or named insured, the are bound to inquire as to the extent of the power of the agent with whom they contract.
insurance is actually related to the life and health of the employee. 10.   Even granting for the sake of argument that the SPAs were in due form,
Indeed, the employee is in the position of a real party to the master Insular Life was grossly negligent in delivering the checks, drawn in favor
policy, and even in a non-contributory plan, the payment by the of the Pineda et al., to a party who is not the agent mentioned in the SPA.
employer of the entire premium is a part of the total compensation paid 11.   The Court also did not agree with the opinion of the CA that since the
for the services of the employee. Put differently, the labor of the shares of the minors in the insurance proceeds are less than P50,000.00,
employees is the true source of the benefits, which are a form of then under Article 225 of the Family Code their mothers could receive such
additional compensation to them. shares without need of either court appointments as guardian or the posting
6.   In Elfstrom vs. New York Life Insurance Company, the California Supreme of a bond. It is of the view that Article 225 of the Family Code had repealed
Court explicitly ruled that in group insurance policies, the employer is the the third paragraph of Section 180 of the Insurance Code. The pertinent
agent of the insurer. Thus: We are convinced that the employer is the agent portion of Article 22563 of the Family Code reads as follows:
of the insurer in performing the duties of administering group insurance 12.   It is clear from the said Article that regardless of the value of the
policies. It cannot be said that the employer acts entirely for its own unemancipated common child's property, the father and mother ipso
benefit or for the benefit of its employees in undertaking administrative jure become the legal guardian of the child's property. However, if the
functions. While a reduced premium may result if the employer relieves the market value of the property or the annual income of the child exceeds
insurer of these tasks, and this, of course, is advantageous to both the P50,000.00, a bond has to be posted by the parents concerned to guarantee
employer and the employees, the insurer also enjoys significant advantages the performance of the obligations of a general guardian.
from the arrangement. The reduction in the premium which results from 13.   It must, however, be noted that the second paragraph of Article 225 of the
employer-administration permits the insurer to realize a larger volume of Family Code speaks of the "market value of the property or the annual
sales, and at the same time the insurer’s own administrative costs are income of the child," which means, therefore, the aggregate of the child's
markedly reduced. x x x The most persuasive rationale for adopting the property or annual income; if this exceeds P50,000.00, a bond is required.
view that the employer acts as the agent of the insurer, however, is that There is no evidence that the share of each of the minors in the proceeds of
the employee has no knowledge of or control over the employer’s the group policy in question is the minor's only property. Without such
actions in handling the policy or its administration. An agency evidence, it would not be safe to conclude that, indeed, that is his only
relationship is based upon consent by one person that another shall act property.
in his behalf and be subject to his control. It is clear from the evidence
regarding procedural techniques here that the insurer-employer 006 VILLACORTA v. INSURANCE COMMISSION (Saldua)
relationship meets this agency test with regard to the administration of Oct. 28, 1980 | Teehankee J. | Compulsory Motor Vehicle Insurance
the policy, whereas that between the employer and its employees fails to
reflect true agency. The insurer directs the performance of the PETITIONER: Jewel Villacorta, assisted by her husband, Guerrero Villacorta
employer’s administrative acts, and if these duties are not undertaken RESPONDENTS: The Insurance Commission, and Empire Insurance Company
properly the insurer is in a position to exercise more constricted control
over the employer’s conduct. SUMMARY:
Jewel Villacorta owns a 1976 Colt Lancer. It was insured with Empire Insurance
Company for Own Damage, Theft, and 3rd Party Liability. The car was brought to the
7.   In Neider v. Continental Assurance Company, the Court held that the “the
Sunday Machine Works Inc. for general check-up and repairs. While the car was in the
employer owes to the employee the duty of good faith and due care in
attending to the policy, and that the employer should make clear to the                                                                                                                        
employee anything required of him to keep the policy in effect, and the time 63
Art. 225. The father and the mother shall jointly exercise legal guardianship over the property of their
that the obligations are due. In its position as administrator of the policy, x x unemancipated common child without the necessity of a court appointment. In case of disagreement, the
x the employer should be considered as the agent of the insurer, and any father's decision shall prevail, unless there is judicial order to the contrary.
omission of duty to the employees in its administration should be Where the market value of the property or the annual income of the child exceeds P50,000, the parent
attributable to the insurer. concerned shall be required to furnish a bond in such amount as the court may determine, but not less than
8.   In the light of the above disquisitions and after an examination of the facts of this case, the ten per centum (10%) of the value of the property or annual income, to guarantee the performance of the
Court held that PMSI, through its President and General Manager, Capt. Nuval, acted as the obligations prescribed for general guardians.
agent of Insular Life. The latter is thus bound by the misconduct of its agent.
custody of the car repair shop and during the period of insurance, the car was allegedly 4.   While the car was cruising along Mabini St., going north at Montalban,
taken by 6 persons who drove it out to Montalban, Rizal. Then, the car got into an Rizal, the car got into an accident. It hit and bumped a gravel & sand truck
accident. The car’s driver at the time of the accident, Benito Mabasa and one of the that was parked at the right side of the road going south. As a consequence,
passengers died; the other 4 sustained physical injuries; and the car suffered extensive the gravel & sand truck veered to right side of the pavement going south,
damage. Villacorta filed a claim for total loss with Empire Insurance, but the claim was and the car veered to the right side of the pavement going north.
denied by Empire Insurance arguing that the accident did not fall either within the Own 5.   The car’s driver, Benito Mabasa and one of the passengers died; the other 4
Damage coverage under the “Authorized Driver” clause, and within the Theft coverage sustained physical injuries; and the car suffered extensive damage.
beause the car could not be considered as stolen. Villacorta filed a complaint with the 6.   The Comprohensive Motor Car Insurance policy for P35,000 issued by
Insurance Commission but it was dismissed as the arguments of Empire Insurance was Empire Insurance undertook to indeminify Villacorta against loss/damage
upheld by the Commission (see Fact #10). Issue is WON the accident was within the to car:
coverage of the insurance policy. SC held YES. “Authorized Driver” clause - that a A.   By accidental collision/overturning; collision/overturning
person other than the insured owner, who drives the car on the insured’s order (e.g. his consequent upon mechanical breakdown, or consequent upon wear
regular driver) or w/ his permission (e.g. friend/family member/employees of a car & tear;
service/repair shop) must be duly licensed drivers and are not disqualified to drive a B.   By fire, external explosion, self-ignition, lightning, burglary,
motor vehicle. The mere coincidence that the employee/s of the shop diverts the use of housebreaking, theft; and
the car to his own illicit/unauthorized purpose does not mean that the “authorized C.   By malicious act.
driver” clause has been violated to bar recovery, provided that such employee is duly 7.   Jewel Villacorta filed a claim for total loss with Empire Insurance, but the
qualified to drive under a valid driver’s license. Theft Clause - where a car is claim was denied by Empire Insurance.
admittedly unlawfully & wrongfully taken by some people (as in this case), such taking 8.   Empire Insurance’s Arguments:
constitutes the nature of theft. Where a car is admittedly unlawfully & wrongfully taken A.   That the accident did not fall w/in the provisions of the policy
by some people (as in this case), such taking constitutes the nature of theft. By taking either under the Own Damage, or Theft coverage, invoking the
possession of the personal property belonging to another and using it, his intent to gain policy provision on “Authorized Driver” clause. 64
is evident since he derives therefrom utility, satisfaction, enjoyment and pleasure. The B.   That the car was not stolen and is therefore not covered by the
use of a thing constitutes gain. Theft Clause.
9.   So Villacorta filed a complaint w/ the Insurance Commission. However, it
DOCTRINE: dismissed Villacorta’s complaint for recovery of total loss of the vehicle.
Main Purpose of the “Authorized Driver” Clause is that a person other than the insured 10.   Insurance Commission upheld the arguments of Empire Insurance:
owner, who drives the car on the insured’s order (e.g. his regular driver) or w/ his A.   “Authorized Driver” clause – that the policy limits the use of the
permission (e.g. friend/family member/employees of a car service/repair shop) must be insured vehicle to 2 persons only i.e. 1) Insured himself; or 2)
duly licensed drivers and are not disqualified to drive a motor vehicle. Any person on the insured’s permission. That under the 2nd
By taking possession of the personal property belonging to another and using it, his category, the words “any person” is qualified by the phrase “…on
intent to gain is evident since he derives therefrom utility, satisfaction, enjoyment and the insured’s order/ with his permission”. Both Jewel Villacorta
pleasure. The use of a thing constitutes gain. and her husband admitted that they did not know the person who
drove the car at the time of the accident, and did not consent to the
use of the same. Thus, Benito Mabasa (driver) is not an authorized
driver of Villacorta, in violation of the said clause of the policy.

FACTS:                                                                                                                        
1.   Jewel Villacorta is the owner of a 1976 Colt Lancer. The car was insured 64
"AUTHORIZED DRIVER”:
with Empire Insurance Company for Own Damage, Theft, and 3rd Party Any of the following:
Liability effective May 16, 1977 – May 16, 1978. (a) The insured
2.   On May 9, 1978, the car was brought to the Sunday Machine Works Inc. for (b) Any person driving on the Insured's Order, or with his permission: Provided,
general check-up and repairs. that the person driving is permitted, in accordance with the licensing or other laws or
3.   On May 11, 1978 (well within the period coverage of the insurance), while regulations, to drive the Scheduled Vehicle, or has been permitted and is not
it was in the custody of Sunday Machine Works, the car was allegedly taken disqualified by order of a Court of Law or by reason or any enactment or regulation
by 6 persons who drove it out to Montalban, Rizal. in that behalf."
B.   Theft Clause – That for there to be a “taking” w/in the purview of THEFT CLAUSE:
the law, there must have been a felonious intent on the part of the 1.   Applying the Theft Clause, where a car is admittedly unlawfully &
taker of the car, and the intent must be to permanently deprive the wrongfully taken by some people (as in this case), such taking constitutes
insured of his car. This was not the case here since the the nature of theft as defined in Art. 308, RPC.65
withholding of the car for a joyride by one of the residents of 2.   For purposes of recovering the loss under the subject insurance policy, it
Sunday Machine Works should not be construed as “taking” doesn’t matter whether the people who took the car were employees of the
(and if ever there was “taking”, that it was only temporary which car shop or not.
was not the kind of taking insured against). 3.   Furthermore, the evidence does not warrant that it was a mere “joy ride”:
11.   Hence, this petition. A.   The police found 1 Cal. 45 Colt, and 1 apple-type grenade from
the waist of Benito Mabasa (driver).
ISSUE/s: B.   That the taking was permanent rather than temporary since the car
1.   WON the accident was within the coverage of the insurance policy = YES. was totally smashed and was never returned in serviceable &
Main Purpose of the “Authorized Driver” Clause is that a person other than useful condition.
the insured owner, who drives the car on the insured’s order (e.g. his regular 4.   Assuming arguendo that taking was “temporary” and was a mere “joyride”,
driver) or w/ his permission (e.g. friend/family member/employees of a car SC held that when a person takes possession of a vehicle belonging to
service/repair shop) must be duly licensed drivers and are not disqualified to another, without the consent of its owner, he is guilty of theft
drive a motor vehicle. regardless of his purpose/object.
Theft Clause means that by taking possession of the personal property 5.   By taking possession of the personal property belonging to another and
belonging to another and using it, his intent to gain is evident since he using it, his intent to gain is evident since he derives therefrom utility,
derives therefrom utility, satisfaction, enjoyment and pleasure. The use of a satisfaction, enjoyment and pleasure. The use of a thing constitutes
thing constitutes gain. gain.

RULING: ACCORDINGLY, the appealed decision is set aside and judgment is


hereby rendered sentencing private respondent to pay petitioner the sum of The insurer must therefore indemnify the petitioner owner for the total loss of the
P35,000.00 with legal interest from the filing of the complaint until full payment is insured car in the sum of P35,000.00 under the theft clause of the policy, subject to
made and to pay the costs of suit. SO ORDERED. the filing of such claim for reimbursement or payment as it may have as subrogee
RATIO: against the Sunday Machine Works, Inc
“AUTHORIZED DRIVER’S” CLAUSE:
4.   To hold that the driver at the time of the accident, who is an employee of 007 PALERMO V. PYRAMID (SANTOS)
the auto repair shop, is too restrictive and contrary to the established May 31, 1988 | Griño-Aquino, J. | Compulsory Motor Vehicle Insurance
principle that insurance contracts, as contracts of adhesion, are construed in PLAINTIFF-APPELLE: Andrew Palermo
favor of the insured. DEFENDANT-APPELLANT: Pyramid Insurance Co., Inc.
5.   Main Purpose of the “Authorized Driver” Clause – that a person other SUMMARY: Palermo insured his car with Pyramid. Pyramid issued a private car
than the insured owner, who drives the car on the insured’s order (e.g. his comprehensive policy. Palermo met a violent accident while driving the car where
regular driver) or w/ his permission (e.g. friend/family member/employees it was totally wrecked; he sustained injuries and his father died. The insurance
of a car service/repair shop) must be duly licensed drivers and are not policy gives the insurer the option to either indemnify the insured for the loss or
disqualified to drive a motor vehicle. replace the damaged car but Pyramid refused to do any; alleging that Palermo, by
6.   A car owner who entrusts his car to an established car service & repair shop driving without a license, violated the policy provision stating that authorized
necessarily entrusts his car key to the shop owner & employees who are drivers are: (1) the insured and (2) any person driving on the Insured's order or
presumed to have the insured owner’s permission to drive the car for with his permission. Provided that the person driving is permitted in accordance
legitimate purposes e.g. road-testing/checking the car. with the licensing or other laws or regulations to drive the Motor Vehicle and is
7.   The mere coincidence that the employee/s of the shop diverts the use of the
car to his own illicit/unauthorized purpose does not mean that the                                                                                                                        
“authorized driver” clause has been violated to bar recovery, provided that 65
Art. 308. "(W)ho are liable for theft. — Theft is committed by any person who,
such employee is duly qualified to drive under a valid driver’s license. with intent to gain but without violence against or intimidation of persons nor force
upon things, shall take personal property of another without the latter's consent
not disqualified from driving such motor vehicle by order of a Court of law or by 9.   Mar 7, 1969: Insured Palermo, filed a complaint in the CFI of Negros
reason of any enactment or regulation in that behalf. TC ordered pyramid to pay Occidental against Pyramid Insurance Co., Inc., for payment of his claim under
Palermo. The issue is WON Palermo was authorized to drive the insured motor a Private Car Comprehensive Policy MV-1251 Pyramid issued.
vehicle despite his driver's license’s expiration. The SC said NO because the 10.   Pyramid’s answer alleged that it disallowed the claim because at the time of the
driver of the insured motor vehicle at the time of the accident was, the insured accident, the insured was driving his car with an expired driver's license.
himself, hence an "authorized driver" under the policy. (doctrine) 11.   Oct 29: TC ordered Pyramid "to pay Palermo P20k, value of the insurance of the
DOCTRINE: The requirement that the driver be "permitted in accordance with motor vehicle in question and to pay the costs."
the licensing or other laws or regulations to drive the Motor Vehicle and is not 12.   Nov 26, Palermo filed a "Motion for Immediate Execution Pending Appeal." It
disqualified from driving such motor vehicle by order of a Court of Law or by was opposed by Pyramid, but was granted by the TC on Dec 15, 1969.
reason of any enactment or regulation in that behalf," applies only when the 13.   Pyramid: TC erred in interpreting the following provision of the Private Car
driver" is driving on the insured's order or with his permission." It does not Comprehensive Policy MV-1251:
apply when the person driving is the insured himself. AUTHORIZED DRIVER:
In short, requirements in (b) (fact 13) does not apply when the driver is the Any of the following:
insured because he is automatically considered as authorized driver by virtue of (a) The Insured.
(a). (b) Any person driving on the Insured's order or with his permission. Provided that
FACTS: the person driving is permitted in accordance with the licensing or other laws or
1.   Oct 12,1968: after purchasing a brand new Nissan Cedric de Luxe Sedan from regulations to drive the Motor Vehicle and is not disqualified from driving such
Ng Sam Bok Motors in Bacolod, Palermo insured the car with Pyramid against motor vehicle by order of a Court of law or by reason of any enactment or regulation
a.   Any loss or damage for P 20k and in that behalf.
b.   Third party liability for P10k.
2.   Palermo paid Pyramid P 361.34 premium for 1y, Mar 12, 1968-9, for which ISSUE: WON Palermo was authorized to drive the insured motor vehicle despite
Pyramid issued a Private Car Comprehensive Policy his driver's license’s expiration – NO. The driver of the insured motor vehicle at the
3.   The car was mortgaged by Palermo with the vendor, Ng Sam Bok Motors, to time of the accident was, the insured himself, hence an "authorized driver" under the
secure the payment of the balance of the purchase price, which explains why the policy.
registration certificate in the name of Palermo remains in the hands of the
mortgagee, Ng Sam Bok Motors. RULING: Appealed decision is affirmed with costs against Pyramid.
4.   April 17: while driving the car, Palermo met a violent accident.
a.   La Carlota City fire engine crashed head on, RATIO:
b.   Palermo sustained physical injuries, 1.   The driver of the insured motor vehicle at the time of the accident was, the
c.   Cesar Palermo (father), was seriously injured and died shortly insured himself, hence an "authorized driver" under the policy.
thereafter, 2.   While the Motor Vehicle Law prohibits a person from operating a motor vehicle
d.   The car was totally wrecked. on the highway without a license or with an expired license, an infraction of the
5.   Pyramid was immediately notified of the occurrence, and upon its orders, the Motor Vehicle Law on the part of the insured, is not a bar to recovery under the
damaged car was towed from the scene of the accident to the compound of Ng insurance contract. It however renders him subject to the penal sanctions of the
Sam Bok Motors in Bacolod where it remains deposited up to the present time. Motor Vehicle Law.
6.   The insurance policy grants an option unto Pyramid, in case of accident either 3.   The requirement that the driver be "permitted in accordance with the licensing or
to: other laws or regulations to drive the Motor Vehicle and is not disqualified from
a.   Indemnify Palermo for loss or damage to the car in cash or driving such motor vehicle by order of a Court of Law or by reason of any
b.   To replace the damaged car. enactment or regulation in that behalf," applies only when the driver" is driving
7.   Pyramid refused to take either of the above-mentioned alternatives for the on the insured's order or with his permission." It does not apply when the person
reason as that the insured himself had violated the terms of the policy when driving is the insured himself.
he drove the car with an expired driver's license. 4.   Villacorta s. Insurance Commission: The main purpose of the "authorized
8.   CA certified this case to the SC for proper disposition as the only question driver" clause, is that a person other than the insured owner, who drives the car
involved is the interpretation of the provision of the insurance contract regarding on the insured's order, such as his regular driver, or with his permission, such as
the "authorized driver" of the insured motor vehicle. a friend or member of the family or the employees of a car service or repair
shop, must be duly licensed drivers and have no disqualification to drive a motor
vehicle.
5.   McMahon v. Pearlma (U.S.): Insured herself was personally operating her
automobile but without a license to operate it, her license having expired prior to
the issuance of the policy, SC of Massachusetts:... Operating an automobile on
a public highway without a license, which act is a statutory crime is not
precluded by public policy from enforcing a policy indemnifying her
against liability for bodily injuries The inflicted by use of the automobile."
008 First Integrated Bonding v. Hernando (Sarmiento) FACTS:
July 31, 1991 | Medialdea, J. | Compulsory Motor Vehicle Insurance
1.   Silverio Blanco was the owner of a passenger jeepney which he insured
PETITIONER: First Integrated Bonding and Insurance Company Inc. against liabilities for death and injuries to third persons with First Integrated
Bonding and Insurance Company, Inc. (First Insurance) under Motor
RESPONDENTS: Hon. Hernando, Victorino Advincula, Romana, Advincula, Vehicle Policy No. V-05-63751 with the face value of P30,000.00
Silverio Blanco
2.   Said jeepney driven by Blanco himself bumped a five-year old
SUMMARY: child, Deogracias Advincula, causing the latter's death.

Silverio owned a jee insured against liabilities for death and injuries to third persons 3.   A complaint for damages was brought by the child's parents,
with First Integrated Bonding and Insurance Company, Inc. under Motor Vehicle the Advincula spouses, against Silverio Blanco. First Insurance was
Policy. Said jeepney driven by Blanco himself bumped a five-year old also impleaded in the complaint as the insurer.
child, Deogracias Advincula, causing the latter's death. A complaint for damages was
brought by the child's parents, the Advincula spouses, against Silverio Blanco. First 4.   The Spouses Advincula presented exhibits (Marriage Certificate, Exhibit B
Insurance was also impleaded in the complaint as the insurer. First Insurance was - Birth Certificate, Exhibit B-1 - the Certificate of the Local Civil Registrar,
made to pay Blanco and the Spouses Advincula. Hence, the insurance company Exhibit C - Certificate of Death, Exhibit C-1 - the official receipt of the
appealed. First Insurance contends that the Advincula spouses have no cause of burial permit, Exhibit C-2 - the autopsy report, Exhibit D - filing fee under
action against it. As parents of the victim, they may proceed against the official receipt in the amount of P80.00, Exhibit D-1 - list of actual
driver, Silverio Blanco on the basis of the provisions of the New Civil expenses in connection with the death and burial of the
Code. However, they have no cause of action against First Insurance, because they deceased Advincula)
are not parties to the insurance contract. Court held that it is settled that where the
insurance contract provides for indemnity against liability to a third party, such third 5.   On the basis of the evidence presented by Spouses Advincula, RTC
party can directly sue the insurer (The liability of the insurer to such third person is rendered judgment which adjudicates to the Advincula spouses, moral
based on contract while the liability of the insured to the third party is based on tort. damages and expenses for the funeral, and attorney’s fees.
The injured for whom the contract of insurance is intended can sue directly the a.   First Insurance received a copy of the decision on March 14,
insurer. "Compulsory Motor Vehicle Liability Insurance (third party liability, or 1978.
TPL) is primarily intended to provide compensation for the death or bodily injuries b.   Upon motion of the Advincula spouses, the decision was amended
suffered by innocent third parties or passengers as a result of a negligent operation on March 27, 1978 which, in addition to the damages granted in
and use of motor vehicles. The victims and/or their dependents are assured of the original decision, awarded damages in the amount of
immediate financial assistance, regardless of the financial capacity of the motor P6,336.50 to Silverio Blanco.
vehicle owners. The insurer's liability accrues immediately upon the occurrence of c.   The amended decision was received by First Insurance on April
the injury or event upon which the liability 11, 1978.
depends, and does notdepend on the recovery of judgment by the injured party again d.   On May 11, 1978, entry of judgment was made, a copy of which
st the insured was furnished First Insurance on June 27, 1978. Upon motion of
the Advincula spouses, an order granting execution was issued by
DOCTRINE: The insurer's liability in a Compulsory Motor Vehicle Insurance the court on June 14, 1978, which was received by First Insurance
policy accrues immediately upon the occurrence of the injury or event upon which on August 1, 1978
the liability
depends, and does notdepend on the recovery of judgment by the injured party again 6.   On September 5, 1978, First Insurance filed a petition for relief from
st the insured judgment in the same case.
a.   the trial court issued an order, denying the petition for relief from
judgment, a copy of which was received by First Insurance
on October 10, 1978.
7.   The plaintiffs moved for execution of judgment and the same was granted
pursuant to an Order of this Court dated June 14, 1978. 3.   "In the event that the injured fails or refuses to include the insurer as party
defendant in his claim for indemnity against the insured, the latter is not
8.   First Insurance filed a motion for reconsideration of the order denying the prevented by law to avail of the procedural rules intended to avoid
petition for relief on May 14, 1979, but the same was denied. multiplicity of suits.
a.   Not even a 'no action' clause under the policy
ISSUE: Whether or not the parents of the victim has can directly sue the which requires that a final judgment be first obtained against the in
insurance company in a Compulsory Motor Vehicle Policy—YES. When an sured and that only thereafter can the person insured recover on the
insurance contract provides for indemnity against liability to a third party, such third policy
party can directly sue the insurer. can prevail over the Rules of Court provisions aimed at avoiding m
ultiplicity of suits."
RULING:
4.   First Insurance cannot evade its liability as insurer by hiding under the
ACCORDINGLY, finding respondent judge to have acted within his cloak of the insured. Its liability is primary and not dependent on the
jurisdiction in denying the petition for relief from judgment, the petition is recovery of judgment from the insured.
DISMISSED. The questioned decision of the trial court in Civil Case No. 1104
having become final and executory, is AFFIRMED. The temporary restraining order 5.   "Compulsory Motor Vehicle Liability Insurance (third party liability, or
issued on August 20, 1979 is hereby lifted. TPL) is primarily intended to provide compensation for the death or bodily
Costs against petitioner. injuries suffered by innocent third parties or passengers as a result of a
negligent operation and use of motor vehicles. The victims and/or their
dependents are assured of immediate financial assistance, regardless of the
RATIO: financial capacity of the motor vehicle owners.

1.   It is the contention of the First Insurance that the Advincula spouses have 6.   the insurer's liability accrues immediately upon the occurrence of the injury
no cause of action against it. As parents of the victim, they may proceed or event upon which the liability
against the driver, Silverio Blanco on the basis of the provisions of the New depends, and does notdepend on the recovery of judgment by the injured pa
Civil Code. However, they have no cause of action against First Insurance, rty against the insured
because they are not parties to the insurance contract.
7.   It is true that Blanco denied that he was negligent when the incident
2.   It is settled that where the insurance contract provides for indemnity against occurred. However, during the pre-trial conference, when respondent judge
liability to a third party, such third party can directly sue the insurer (The admitted all the exhibits of the plaintiffs to abbreviate the proceedings, no
liability of the insurer to such third person is based on contract while the objection was interposed by Blanco.
liability of the insured to the third party is based on tort a.   When a decision was rendered based only on the exhibits of the
a.   "The injured for whom the contract of insurance is intended can plaintiffs, Blanco likewise did not object. No motion for
sue directly the insurer. The general purpose of statutes enabling reconsideration was filed by either Blanco or First Insurance.
an injured person to proceed directly against the insurer is to b.   Hence, the decision became final and may no longer be attacked.
protect injured persons against the insolvency of the insured who c.   It should be noted also that First Insurance was declared in default
causes such injury, and to give such injured person a certain because of its failure to file an answer. As far as it was concerned,
beneficial interest in the proceeds of the policy, and statutes are to it failed to raise any triable issue. It lost its standing in court and
be liberally construed so that their intended purpose may be judgment may be rendered against it on the basis only of the
accomplished. evidence of the Advincula spouses.
b.   It has even been held that such a provision creates a contractual rel
ation which inures to the benefit 8.   Petitioner had been given its day in court. Despite its having been declared
of any and every person who may be negligently injured by the na in default and its failure to file a motion to lift the order of default, it was
med insured as if such injured person were specifically named in th still notified of the subsequent proceedings in the trial court. But no
e policy.
positive step was taken by it on time to vacate the order of default, the
decision nor the amended decision.

9.   Instead, it chose to file a petition for relief from judgment on September 1,


1978 almost five (5) months from its receipt of a copy of the amended
decision on April 11, 1978.
a.   Clearly, the said petition for relief from judgment was filed out of
time. The rules require that such petitions must be filed within
sixty (60) days after the petitioner learns of the judgment and not
more than six (6) months after such judgment was entered (Rule
38, Section 3).
b.   The period fixed by Rule 38 of the Rules of Court is non-
extendible and never interrupted. It is not subject to any condition
or contingency, because it is itself devised to meet a condition or
contingency. The remedy allowed by Rule 38 is an act of grace, as
it were, designed to give the aggrieved party another and last
chance.

10.   It appears that the award of damages in favor of Blanco has no basis. The
complaint in Civil Case 1104 was for damages brought by the spouses
against Blanco and First Insurance. Blanco did not put up any claim against
the latter.
a.   However, since the said decision had already become final
and executory, it can no longer be corrected or amended.
b.   In the same vein, the claim of petitioner that its liability to third
parties under the insurance policy is limited to P20,000.00 only can
no longer be given consideration at this late stage, when the
decision of the trial court awarding damages had already become
final and executory.
009 PERLA COMPANIA DE SEGUROS INC. v. ANCHETA (SEE) claim shall lie against the insurer of the directly offending vehicle.
August 8, 1988 |Cortes, J. | Compulsory Motorvehicle insurance (4)   In all cases, the right of the party paying the claim to recover against the
owner of the vehicle responsible for the accident shall be maintained.
PETITIONER: Perla Compania De Seguros, Inc.
RESPONDENTS: Hon. Constante A. Ancheta, Presiding Judge Of The Court FACTS:
Of First Instance Of Camarines Norte, Branch Iii, Ernesto A. Ramos And 189.  On December 27, 1977, in a collision between the IH Scout in which
Goyena Zenarosa-Ramos, For Themselves And As Guardian Ad Litem For private respondents (Ramos et al) were riding and a Superlines bus along
Minors Jobet, Banjo, David And Grace All Surnamed Ramos, Fernando M. the national highway in Sta. Elena, Camarines Norte, Ramos et al sustained
Abcede, Sr., For Himself And Guardian Ad Litem For Minor Fernando G. physical injuries in varying degrees of gravity.
Abcede, Jr., Miguel Jerez Mago As Guardian Ad Litem For Minors Arleen R. 190.  They filed with the CFI of Camarines Norte a complaint for damages
Mago, And Anacleta J. Zenarosa. against Superlines, the bus driver and Perla Companie de Seguros, the
insurer of the bus. The bus was insured with Perla for the amount of
SUMMARY: A collision between an IH Scout car, vehicle where Ramos et al P50,000.00 as for passenger liability and P50,000.00 as for third party
were riding, and a Superlines bus occurred along the national highwat in Sta. liability. The vehicle in which Ramos et al were riding was insured with
Elena, Camarines Norte. Ramos et al sustained physical injuries of varying Malayan Insurance Co.
degrees. They filed with the CFI of Camarines Norte a complaint for damages 191.  Before summons could be served, Judge Ancheta, issued an order requiring
against Superlines, the bus driver and Perla Companie de Seguros, the insurer of Perla to pay immediately (within 5 days from receipt of the order) the
the bus. The bus was insured with Perla for the amount of P50,000.00 as for P5000 under the “no fault clause” as provided for under Sec 378 of the
passenger liability and P50,000.00 as for third party liability. The vehicle in Insurance Code to Ramos et al. order also required the requisite documents
which Ramos et al were riding was insured with Malayan Insurance Co. Before to be attached to the record.
summons could be served, Judge Ancheta, issued an order requiring Perla to pay 192.  Perla denied in its Answer its alleged liability under the "no fault
immediately (within 5 days from receipt of the order) the P5000 under the “no indemnity" provision and likewise moved for the reconsideration of the
fault clause” as provided for under Sec 378 of the Insurance Code to Ramos et order.
al. order also required the requisite documents to be attached to the record. Perla 193.  Perla held the position that under Sec. 378 of the Insurance Code, the
denied liability claiming that the insurer that should be is that insurer of the insurer liable to pay the P5,000.00 is the insurer of the vehicle in which
vehicle in which the occupant is riding, mounting, or dismounting from. Judge private Ramos et al were riding, not Perla, as the provision states that "[i]n
Ancheta denied this and his second MR was also denied. the case of an occupant of a vehicle, claim shall lie against the insurer of the
vehicle in which the occupant is riding, mounting or dismounting from."
WoN Perla is the insurer liable to indemnify Ramos et al under Sec. 378 of the Respondent judge, however, denied reconsideration.
Insurance Code—NO, the insurer liable is the insurer of the vehicle that Ramos 194.  A second motion for reconsideration was filed by Perla. However,
et al rode, which is Malayan Insurnce Co. A perusal of Sec 378(iii) will show respondent judge denied the second motion for reconsideration and ordered
that, “Claim may be made against one motor vehicle only. In the case of an the issuance of a writ of execution.
occupant of a vehicle, claim shall lie against the insurer of the vehicle in which 195.  Hence, the instant petition praying principally for the annulment and setting
the occupant is riding, mounting or dismounting from. In any other case, claim aside of respondent judge's orders.
shall lie against the insurer of the directly offending vehicle. In all cases, the 196.  The court pending the case, issued a TRO.
right of the party paying the claim to recover against the owner of the vehicle 197.  
responsible for the accident shall be maintained.” ISSUE/s:
22.   WoN Perla is the insurer liable to indemnify Ramos et al under Sec. 378 of
DOCTRINE: The following rules on claims under the “no fault indemnity” the Insurance Code—NO, the insurer liable is the insurer of the vehicle that
provision, where proof of fault or negligence is not necessary for the payment of Ramos et al rode, which is Malayan Insurance Co.
any claim for death or injury to a passenger of a third party are as follows:
(1)   A claim may be made against one motor vehicle only. RULING: WHEREFORE, the petition is GRANTED and respondent judge's order
(2)   If the victim is an occupant of a vehicle, the claim shall lie against the dated March 1, 1978, requiring petitioner to pay private respondents the amount of
insurer of the vehicle in which he is riding, mounting or dismounting P5,000.00 as "no fault indemnity' under Sec. 378 of the Insurance Code, and that of
from. January 3, 1979, denying the second motion for reconsideration and issuing a writ of
(3)   In any other case (i.e. if the victim is not an occupant of a vehicle), the
execution, are ANNULLED and SET ASIDE. The temporary restraining order the claim be made against the insurer of the vehicle in which the occupant is
issued by the Court on January 24, 1979 is made permanent. riding, mounting, or dismounting from.
10.   That said vehicle might not be the one that caused the accident is of no
RATIO: moment since the law itself provides that the party paying the claim under
Sec. 378. Any claim for death or injury to any passenger or third party pursuant to Sec. 378 may recover against the owner of the vehicle responsible for the
the provision of this chapter shall be paid without the necessity of proving fault or accident. This is precisely the essence of "no fault indemnity" insurance
negligence of any kind. Provided, That for purposes of this section — which was introduced to and made part of our laws in order to provide
victims of vehicular accidents or their heirs immediate compensation,
(i) The indemnity in respect of any one person shall not exceed five thousand pesos; although in a limited amount, pending final determination of who is
responsible for the accident and liable for the victims'injuries or death. In
turn, the "no fault indemnity" provision is part and parcel of the Insurance
(ii) The following proofs of loss, when submitted under oath, shall be sufficient
Code provisions on compulsory motor vehicle ability insurance [Sec. 373-
evidence to substantiate the claim:
389] and should be read together with the requirement for compulsory
passenger and/or third party liability insurance [Sec. 377] which was
(a) Police report of accident, and mandated in order to ensure ready compensation for victims of vehicular
accidents.
(b) Death certificate and evidence sufficient to establish the proper 11.   Irrespective of whether or not fault or negligence lies with the driver of the
payee, or Superlines bus, as Ramos et al were not occupants of the bus, they cannot
claim the "no fault indemnity" provided in Sec. 378 from Perla. The claim
(c) Medical report and evidence of medical or hospital should be made against the insurer of the vehicle they were riding. This
disbursement in respect of which refund is claimed; is very clear from the law. Undoubtedly, in ordering Perla to pay Ramos et
al the 'no fault indemnity,' respondent judge gravely abused his discretion in
(iii) Claim may be made against one motor vehicle only. In the case of an occupant a manner that amounts to lack of jurisdiction. The issuance of the corrective
of a vehicle, claim shall lie against the insurer of the vehicle in which the occupant writ of certiorari is therefore warranted.
is riding, mounting or dismounting from. In any other case, claim shall lie against
the insurer of the directly offending vehicle. In all cases, the right of the party paying
the claim to recover against the owner of the vehicle responsible for the accident
shall be maintained.

8.   The provision is couched in clear terms. The following rules on claims


under the “no fault indemnity” provision, where proof of fault or negligence
is not necessary for the payment of any claim for death or injury to a
passenger of a third party are as follows:
a.   A claim may be made against one motor vehicle only.
b.   If the victim is an occupant of a vehicle, the claim shall lie against the
insurer of the vehicle in which he is riding, mounting or dismounting
from.
c.   In any other case (i.e. if the victim is not an occupant of a vehicle), the
claim shall lie against the insurer of the directly offending vehicle.
d.   In all cases, the right of the party paying the claim to recover against the
owner of the vehicle responsible for the accident shall be maintained.
9.   The law is very clear — the claim shall lie against the insurer of the
vehicle in which the "occupant" is riding, and no other. The claimant is
not free to choose from which insurer he will claim the "no fault
indemnity," as the law, by using the word "shall, makes it mandatory that
010 Summit Guaranty vs. De Guzman (Siapno) not find it necessary to immediately bring suit. In violation of its duties to adopt
June 30, 1987 | Gancayco, J. | Sec. 384 Application; Claim Settlement and implement reasonable standards for the prompt investigation of claims and
to effectuate prompt, fair and equitable settlement of claims, and with manifest
PETITIONER: SUMMIT GUARANTY AND INSURANCE COMPANY, bad faith, petitioner company devised means and ways of stalling the settlement
INC. (all cases) proceedings.
RESPONDENT: HON. JOSE C. DE GUZMAN (1st case), HONORABLE
GREGORIA C. ARNALDO (2nd case), and HONORABLE RAMON V. The one-year period should be counted from the date of rejection by the insurer
JABSON (3rd case) as this is the time when the cause of action accrues. Since in these cases there
has yet been no accrual of cause of action, We hold that prescription has not yet
SUMMARY: The 3 consolidated cases arose from three separate complaints set in.
filed against Summit for the payment of insurance on insurance policies issued
by it. DOCTRINE: The six-month period for filing the notice of claim and the one-
1st case: Ledesma’s tractor was bumped by a minibus insured by Summit (Third year period for bringing an action or suit do not need to concur. The one-year
Party Liability). period is only required “in proper cases.” The condition contained in an
2nd case: Geronima Pulmano was the owner of a jeep insured with Summit. Ariel insurance policy that claims must be presented within one year after rejection is
Pulmano this jeep got involved in a vehicular accident which resulted in the not merely a procedural requirement but an important matter essential to a
death of one of the victims. prompt settlement of claims against insurance companies as it demands that
3rd case: Amelia Generao owned a passenger jeepney that was insured with insurance suits be brought by the insured while the evidence as to the origin and
Summit under a Vehicle Comprehensive Police, while being driven by private cause of destruction have not yet disappeared.
respondent Pagkalinawan, this jeepney struck the van of a certain Mr. Hahn.
They all immediately made a notice of claim with Summit for their respective FACTS:
policies. However, Summit only made assurances but he never acted on it. In the G.R. No. L-48679
first case, assurances of payment were constantly given and Summit even said 1.   Private respondent Jose Ledesma was the owner of a tractor which was
that a check was ready for release. In the second case, no steps were taken to bumped by a minibus insured with Summit for purposes of Third Party
process the claim and no rejection of said claim was ever made even if the Liability.
Pulmanos had already complied with all the requirements. In the third case, 2.   Immediately thereafter, Ledesma made a notice of claim with Summit
Summit even provided legal assistance to one of the private respondents in the Guaranty for the damage and loss suffered by the tractor. Summit Guaranty
criminal case filed against him leading private respondents to believe that it was advised Ledesma to have the tractor repaired at GA Machineries for an
ready to pay. Because their notices were unheeded, they filed the complaints for estimated amount of P21,000.00. Later, Summit through its officials, made
the insurance with the Insurance Commission. an assurance of payment of the said amount.
3.   When G.A. Machineries was finally through with the repair, Ledesma made
Summit argued that the complaints, having been filed beyond the one-year several demands on Summit because of the repair shop's warning that
period provided in Section 384 of the Insurance Code, can no longer prosper. failure to pay would result in the auctioning of the tractor to cover the
Summit contends that the two periods prescribed in the aforementioned law that mechanic's lien. However, Ledesma only received additional assurances of
is, the six-month period for filing the notice of claim and the one-year period payment.
for bringing an action or suit are mandatory and must always concur. Issue is 4.   Due to the failure of Summit to settle his claim, Ledesma submitted a letter-
WoN the causes of action of private respondents have already prescribed – complaint to the Insurance Commission. The Commission, in turn, wrote
No, there is absolutely nothing in the law which mandates that the 6-month Summit to inquire about the status of the claim. Again, Summit promised to
and the 1-year must always concur. The one-year period is only required pay but did not deliver such promise.
“in proper cases.” It is very obvious that petitioner company is trying to use 5.   Ledesma filed a formal complaint with the Insurance Commission which
Section 384 of the Insurance Code as a cloak to hide itself from its liabilities. Summit moved to dismiss on the ground of prescription. The Commission,
The facts of these cases evidently reflect the deliberate efforts of petitioner deferred the resolution of the MTD causing Summit to file an MR which
company to prevent the filing of a formal action against it. Bearing in mind that was later denied. Hence, this petition for certiorari and prohibition.
if it succeeds in doing so until one year lapses from the date of the accident it
could set up the defense of prescription, petitioner company made private G.R. No. 50997
respondents believe that their claims would be settled in order that the latter will
6.   Private respondent Geronima Pulmano was the owner of a jeep insured with sustained as certified by a duly licensed physician. Notice of claim must be
Summit in the amount of P120,000.00, while being driven by private filed within six months from date of the accident, otherwise, the claim shall
respondent Ariel Pulmano this jeep got involved in a vehicular accident be deemed waived Action or suit for recovery of damage due to loss or
which resulted in the death of one of the victims. injury must be brought, in proper cases, with the Commission or the Courts
7.   Pulmanos immediately filed a notice of accident and claim with Summit within one year from date of accident, otherwise the claimant's right of
and diligently submitted all the required documents with it. However, action shall be prescribe.
Summit did not take any steps to process the claim. 16.   Summit contends that the two periods prescribed in the aforementioned
8.   Because of this, Pulmanos brought their claim to the Insurance Commission law-that is, the six-month period for filing the notice of claim and the one-
and the latter wrote Summit three letters and thereafter, filed a letter- year period for bringing an action or suit are mandatory and must
complaint with the Insurance Commission a copy of which was sent to always concur.
Summit by registered mail. Still Summit failed to settle the claim but to no 17.   Summit argues that even if the notice of claim was timely filed with the
avail. insurance company within the six-month period, as what happened in the
9.   Consequently, Pulmanos were constrained to file a complaint with the CFI three cases, the action or suit that follows, if filed beyond the one-year
of Tarlac. Summit moved to dismiss on the ground of prescription but period should necessarily be dismissed on the ground of prescription.
respondent Judge de Guzman denied the motion. Hence, this petition for
certiorari and prohibition. ISSUE: WoN the causes of action of private respondents have already prescribed –
No, there is absolutely nothing in the law which mandates that the 6-month and the
G.R. No. L-48758 1-year must always concur.
10.   Private respondent Amelia Generao owned a passenger jeepney that was
insured with Summit under a Vehicle Comprehensive Police, while being RULING: WHEREFORE, the instant petitions are hereby dismissed for lack of
driven by private respondent Pagkalinawan, this jeepney struck the van of a merit. The temporary restraining order dated July 18, 1979 issued in G.R. No. 50997
certain Mr. Hahn. is hereby lifted. With costs against Summit. Let the records of these cases be
11.   Two days after the accident, Generao notified Summit of the vehicular immediately remanded for prompt determination of the claims. This decision is
accident and demanded from it payment of damages on both vehicles. immediately executory.
12.   Thereafter, Generao submitted to Summit all the necessary papers in
support of the claim and required of her by the latter. The two had a RATIO:
dialogue at the office of Summit to settle the claim. Then, in the initial 1.   We find no merit in the contention of Summit. There is absolutely nothing
hearing of the criminal case that arose out of the incident, accused in the law which mandates that the two periods must always concur. On the
Pagkalinawan was represented by a lawyer of Summit. contrary, it is very clear that the one-year period is only required "in
13.   Nonetheless, time passed without Summit taking any final action on proper cases."
Generao's claim. 2.   Summit disregarded this very significant phrase when it made its own
14.   Mr. Hahn filed a complaint for damages against Generao and Pagkalinawan interpretation of the law. Had the lawmakers intended it to be the way
with CFI of Rizal. The latter, on the other hand, filed a third party Summit assumes it to be, then the phrase "in proper cases" would not have
complaint against Summit which in turn filed a MTD on the ground of been inserted.
prescription. Respondent Judge Ramon V. Jabson, however, denied the 3.   Aisporna vs. Court of Appeals: Legislative intent must be ascertained from
said motion. Subsequently, Summit filed a motion for reconsideration a consideration of the statute as a whole. A statute must be so construed as
which again was denied. Hence, this petition for certiorari and prohibition to harmonize and give effect to all its provisions whenever possible.
4.   It is very obvious that summit is trying to use Section 384 of the Insurance
Summit’s arguments Code as a cloak to hide itself from its liabilities. The facts of these cases
15.   According to Summit, the complaints of private respondents, having been evidently reflect the deliberate efforts of Summit to prevent the filing of a
filed beyond the one-year period provided in Section 384 of the formal action against it. Summit made private respondents believe that their
Insurance Code, can no longer prosper. Said law reads as follows: claims would be settled in order that the latter will not find it necessary to
SECTION 384. Any person having any claim upon the policy immediately bring suit.
issued pursuant to this chapter shall, without any unnecessary delay, present 5.   In violation of its duties to adopt and implement reasonable standards for
to the insurance company concerned a written notice of claim setting forth the prompt investigation of claims and to effectuate prompt, fair and
the amount of his loss, and/or the nature, extent and duration of the injuries
equitable settlement of claims, and with manifest bad faith, Summit cause of action does not accrue until the party obligated refuses, expressly
devised means and ways of stalling the settlement proceedings. or impliedly, to comply with its duty.
6.   In the second case, no steps were taken to process the claim and no 16.   Finally, We are pleased to note that the now defunct Batasang Pambansa,
rejection of said claim was ever made even if the Pulmanos had already after having recognized that Section 384 of the Insurance Code, has created
complied with all the requirements. so many problems for the insured amended the law to read as follows:
7.   In the third case, Summit even provided legal assistance to one of the SEC. 384. Any person having any claim upon the policy issued pursuant to
private respondents in the criminal case filed against him leading private this chapter shall, without any unnecessary delay, present to the insurance
respondents to believe that it was ready to pay. In the same case, Summit company concerned a written notice of claim setting forth the nature, extent
admits that it took no final action or adjudication of the claim. and duration of the injuries sustained as certified by a duly licensed
8.   Worse still, in the first case, assurances of payment were constantly given physician. Notice of claim must be filed within six months from date of the
and Summit even said that a check was ready for release. accident otherwise, the claim shall be deemed waived. Action or suit for
9.   This Court has made the observation that some insurance companies have recovery of damage due to loss or injury must be brought in proper cases,
been inventing excuses to avoid their just obligations and it is only the State with the Commissioner or the Courts within one year from denial of the
that can give the protection which the insuring public needs from possible claim, otherwise the claimant's right of action shall prescribe.
abuses of the insurers.
10.   Thus, the three cases do not fall within the meaning of "proper cases" as
contemplated in Section 384 of the Insurance Code. To hold otherwise
would enable Summit to evade its responsibility through its clever scheme.
11.   Summit cited the following principle in the case of Ang vs. Fulton Fire
Insurance: The condition contained in an insurance policy that claims must
be presented within one year after rejection is not merely a procedural
requirement but an important matter essential to a prompt settlement of
claims against insurance companies as it demands that insurance suits be
brought by the insured while the evidence as to the origin and cause of
destruction have not yet disappeared. It is in the nature of a condition
precedent to the liability of the insurer, or in other terms, a resolutory
clause, the purpose of which is to terminate all liabilities in case the action
is not filed by the insured within the period stipulated.
12.   Suffice it to say that the aforementioned case has no application to the
present cases as in that case the claim of the plaintiffs was denied as early as
April 18, 1956 and the action was brought only on May 5, 1958 or almost 2
years after. As we have already noted earlier, in the cases at bar, no denial
of the claims was ever made and on the contrary, private respondents
were made to believe that they will be paid by Summit. The alleged
delay, which is quite insignificant compared to the length of time that the
plaintiffs took in the Ang case in bringing suit, was not caused by herein
private respondents but by the Summit itself.
13.   The one-year period should instead be counted from the date of
rejection by the insurer as this is the time when the cause of action
accrues.
14.   Since in these cases there has yet been no accrual of cause of action, we
hold that prescription has not yet set in. This is because, before such final
rejection, there was no real necessity for bringing suit.
15.   Since a "cause of action" requires, as essential elements, not only a legal
right of the plaintiff and a correlative obligation of the defendant but also
"an act or omission of the defendant in violation of said legal right," the
011 STRONGHOLD INSURANCE CO., INC., v. PANAMA ISLAND RESORT FACTS:
HOTEL AND MARINA, INC. (SOLCO) 124.  The case stems from an action for sum of money filed by Pamana Island
June, 01, 2016 | Reyes, J. | Fire Insurance Resort Hotel and Marina Club, Inc. (Pamana) and Flowtech Construction
Corporation (Flowtech) against Stronghold on the basis of a Contractor's
PETITIONER: STRONGHOLD INSURANCE CO., INC. All Risk Bond of P9,047,960.14 obtained by Flowtech in relation to the
RESPONDENTS: PAMANA ISLAND RESORT HOTEL AND MARINA CLUB, construction of Pamana's project in Pamana Island, Subic Bay. On January
INC. 27, 1992, a fire in the project burned down cottages being built by
Flowtech, resulting in losses to Pamana.
SUMMARY: Pamana filed an action for a sum of money against Stronghold 125.  In a Decision dated October 14, 1999, the Regional Trial Court (RTC) of
Insurance on the basis of a Contractor’s All Risk Bond obtained by Flowtech in Makati City, Branch 135 declared Stronghold liable for the claim. Besides
relation to the construction of a certain project in Subic Bay. In 1992, A fire in the the award of insurance proceeds, exemplary damages and attorney's fees,
project burned down certain cottages being built by Flowtech, resulting in losses to the trial court ordered the payment of interest at double the applicable rate,
Panama. In 1999, the RTC held stronghold liable for the claim and awarded double following Section 243 of the Insurance Code which Stronghold was
the amount of interest on the principal amount of the insurance on the basis of Sec. declared to have violated66
243 (it did not however state in its judgement whether the applicable rate of interest 126.  RTC held them liable for: P4,728,297.82 with double the interest rate
was 6% or 12%). CA and SC affirmed. A writ of execution was thus issued. from the date of demand until fully paid, 500k exemplary and 100k
However, Stronghold filed an Urgent Motion to Suspend Execution and to attorney’s fees.
Rationalize Enforcement of the Decision. It argued that the award granted by the 127.  Stronghold's appeal seeking the reversal of the RTC judgment was denied
RTC was unconscionable and iniquitous. RTC granted the said motion and modified by the CA and thereafter, by the SC. On March 4, 2005, Flowtech filed with
the writ of execution. It reduced the award from 13.3m (Panama’s claim) to 8.2m on the RTC a motion for execution, which was granted7 on May 10, 2005. A
the basis that the computation of interest should be reckoned from the date of Writ of Execution was issued on May 12, 2005.
promulgation of judgement and not from the date of demand since the amount of the 128.  Thereafter, Stronghold filed an Urgent Motion to Suspend Execution and to
award only became certain after judgement was rendered. It also further argued that Rationalize Enforcement of the Decision, dated August 16, 2005,
the applicable rate of interest is 6% because the obligation of Stronghold does not contending that the interest penalty being demanded from it through the
arise from a loan or forbearance of money. CA reversed the RTC and stated that the Sheriff was unconscionable and iniquitous. The motion was opposed by
judgement is final and executory thus immutable. Pamana, which contended that the RTC decision had become final and thus,
could no longer be amended, altered and modified. Furthermore, the double
The issues in this case are (1) WoN the RTC can change the computation of interest interest rate being imposed upon the award was argued to be supported by
to be reckoned from the date of promulgation instead of the date of demand. NO Section 243 of the Insurance Code.
final judgments are immutable. Moreover, the RTC has no power to issue a writ of 129.  On November 22, 2005, the RTC rendered its Order granting Stronghold's
execution which is at variance with the judgement it seeks to promulgate. The motion. Interest was substantially reduced following the court's
judgement expressly states that it should be computed from the date of demand. pronouncement that its computation should be reckoned from the date of
promulgation of judgment until its finality and not from the date of
(2)  Should the applicable rate of interest be 6% per annum or 12% per annum? It
should be 12% because Sec. 243 of the code expressly states that the rate shall
be "twice the ceiling prescribed by the Monetary Board.” Hence, it should be                                                                                                                        
66
the 12% rate of interest as provided by the BSP. (It would have been 6% if Sec. 243. The amount of any loss or damage for which an insurer may be liable,
decided now because BSP Circular No. 799 reduced the interest to 6%. under any policy other than life insurance policy, shall be paid within thirty days
However, the decision was promulgated before said circular was issued and the after proof of loss is received by the insurer and ascertainment of the loss or damage
rate can only be applied prospectively). is made either by agreement between the insured and the insurer or by arbitration;
but if such ascertainment is not had or made within sixty days after such receipt by
DOCTRINE: Given the provisions of the Insurance Code, which is a special law, the insurer of the proof of loss, then the loss or damage shall be paid within ninety
the applicable rate of interest referred to in Sec. 243 shall be that imposed in a days after such receipt. Refusal or failure to pay the loss or damage within the time
loan or forbearance of money as imposed by the Bangko Sentral ng Pilipinas prescribed herein will entitle the assured to collect interest on the proceeds of the
(BSP), even irrespective of the nature of liability. policy for the duration of the delay at the rate of twice the ceiling prescribed by the
Monetary Board, unless such failure or refusal to pay is based on the ground that the
claim is fraudulent.
demand until full payment as enunciated in the Decision dated October interest."Instead of "double the rate of interest [on the proceeds of insurance]
14, 1999. from the date of demand until fully paid," the RTC's computation for purposes
130.  The trial court reasoned: The claim of (Pamana and Flowtech) for the of execution was limited to an interest rate of 6% per annum, resulting in a
interest of the principal amount in the sum of P7,528,774.05, docs not double rate of only 12% per annum, to be reckoned from the date of the trial
appear to be accurate. The principal amount of P4,728,297.82 demanded court's judgment until it became final and executory.
was ascertained only after the trial of the case on its merits. The obligation 2. Clearly, the RTC's issuances contravened a settled principle affecting
of Stronghold is not a loan or forbearance of money. The interest on the execution of judgments. Time and again, courts have emphasized that a
obligation shall begin to run from the time the claim is made judicially and writ of execution must conform substantially to every essential particular of
extrajudicially when the demand was established with certainty. But when the judgment promulgated. An execution that is not in harmony with the
such certainty cannot be so reasonably established at the time of the judgment is bereft of validity. This applies because "once a judgment becomes
demand, the interest shall begin only from the date of judgment of the court. final and executory, all that remains is the execution of the decision which is a
131.  The Decision was promulgated on October 14, 1999. The interest on the matter of right. The prevailing party is entitled to a writ of execution, the
principal amount should be reckoned from this date up to December 15, issuance of which is the trial court's ministerial duty."
2004, when the judgment became final and executory. 3.Although some arguments advanced by Stronghold appeal to the substantive
132.  This resulted in the amount awarded being reduced from P13,342,547.87 issues or merits of the RTC's main judgment that favored Pamana, such matters
(Pamana’s Claim) to P8,261,418.46 by the RTC. have long been settled via the RTC decision that had become final and
133.  Pamana thus appealed to the CA which reversed the RTC’s judgement executory. Anent the computation of interest on Stronghold's liability, it was
stating that it should be computed from the date of demand. Hence, this explained that the notice of loss was promptly served upon Stronghold, but it
petition by Stronghold took more than a year to reject the claim in violation of Section 243 of the
Insurance Code. Thus, double the applicable rate of interest on the principal
ISSUE/s: award should be imposed.
14.   WoN the RTC can change the computation of interest to be reckoned from 4.A disagreement, however, concerns the question of whether an interest rate of 6%
the date of promulgation instead the date of demand. NO final judgements or 12% per annum should apply in the computation, as this subject was not
are immutable. Moreover, the RTC has no power to issue a writ of specifically defined in the RTC judgment in the main case. The RTC, in the Order
execution which is at variance with the judgement it seeks to promulgate. dated November 22, 2005, pegged the interest rate at 6% per annum by explaining
15.   Should the applicable rate of interest be 6% per annum or 12% per annum? that Stronghold's obligation did not equate to a loan or forbearance of money. On the
It should be 12% because Sec. 243 of the code expressly states that the rate other hand, the CA explained that the double rate should be based on 12% per
shall be "twice the ceiling prescribed by the Monetary Board.” Hence, it annum, as the Insurance Code pertained to a rate "twice the ceiling prescribed by
should be the 12% rate of interest as provided by the BSP. (It would have the Monetary Board" and thus could only refer to the rate applicable to
been 6% if decided now because BSP Circular No. 799 reduced the interest obligations constituting a loan or forbearance of money.
to 6%. However, the decision was promulgated before said circular was 5. The Court agrees with the CA that given the provisions of the Insurance
issued). Code, which is a special law, the applicable rate of interest shall be that imposed
in a loan or forbearance of money as imposed by the Bangko Sentral ng
RULING: WHEREFORE, the petition is DENIED. The Decision dated July 20, Pilipinas (BSP), even irrespective of the nature of Stronghold's liability. In the
2006 and Resolution dated September 26, 2006 of the Court of Appeals in CA-G.R. past years, this rate was at 12% per annum. However, in light of Circular No. 799
SP No. 94313 are AFFIRMED with MODIFICATION in that beginning July 1, issued by the BSP on June 21, 2013 decreasing interest on loans or forbearance of
2013, the applicable interest shall be computed pursuant to Section 243 of the money, the CA's declared rate of 12% per annum shall be reduced to 6% per annum
Insurance Code at double the rate of six percent (6%) per annum. from the time of the circular's effectivity on July 1, 2013. The Court explained
in Nacar v. Gallery Frames that the new rate imposed under the circular could
only be applied prospectively, and not retroactively.
RATIO:
1.  The Court denies the petition. As correctly pointed out by the CA, the RTC's 012 LUMIBAO v. IAC (Soriano)
order to implement carried substantial changes in a judgment that had September 13, 1990 | Cortes, J. | Adjuster/ Sales Agencies and Technical Services
become final and executory. These variations pertained to "(1) the date from
which the double rate of interest on the principal amount of the claim, shall be
computed; (2) up to when such interest shall run; and (3) the applicable rate of
PETITIONER: Rosita Lumibao would be subversive of the very public policy which the law was designed and
RESPONDENT: Intermediate Appellate Court and Eugenio Trinidad intended to uphold. True, the statutes, like Sections 361 and 363 of Pres. Decree
No. 961, in terms, are addressed to the insurance companies, agents and brokers,
SUMMARY: Lumibao, an insurance agentrepresenting Manuka Bankers, was and is enacted for the protection of policyholders, but this is for the general body
able to convince private respondent Eugenio Trinidad, the Vice-President and of policyholders who would suffer by the enforcement of the prohibited
General Manager and principal stockholder of Victory Liner Inc., to take out a agreements, and not for those who have entered into such agreements and are
life insurance policy with Manila Bankers. Premium was fixed at P93,180 seeking to profit by its terms.
(considering the result of the medical exam showing he is diabetic). In order to
persuade Trinidad to take out the policy at the computed premium, Lumibao
offered to return to him (Trinidad) the amount corresponding to her (Lumibao)
commission out of the first premium payment, which is equivalent to 50%
(P46,590) of the 1st premium payment. Trinidad agreed to take the policy thus,
he issued 2 checks in favor of the insurance company for a total of P93,180.
Lumibao received P51,249 from as her commission. Yet, petitioner failed to
comply with her commitment to pay private Trinidad P46,590. Trinidad filed an FACTS:
action for specific performance and damages. Lumibao denied that she had made 198.  Lumibao is a life insurance underwriter/agent and a member of a group of
a verbal promise to return the 50% of the premium. The issues in this case are: insurance underwriters known as Bescon Insurance Agencies, Inc.,
1.   WoN Lumibao violated Section 361 (now Section 370) of the insueance representing the Manila Bankers Life Insurance Corporation (Manila
code and therefore be held liable—YES. A preponderance of evidence Bankers)
on record supports the findings of the courts that Lumibao had induced 199.  Lumibao was able to convince private respondent Eugenio Trinidad, the
Trinidad to take out a life insurance policy from Manila by promising Vice-President and General Manager and principal stockholder of Victory
him a rebate equivalent to 50% of the first annual premium payment. It Liner Inc., to take out a life insurance policy with Manila Bankers
is evident that petitioner's promise to pay private respondent an amount 200.  As a result of a medical examination conducted on Trinidad showing that he
equivalent to 50% of the first premium payment, which would be taken was a diabetic, the annual insurance premium was fixed at P93,180.00 for a
out of her commission on the insurance policy, is covered squarely by life insurance policy with a face value of PlM
the express provisions of Section 361. 201.  In order to persuade Trinidad to take out the policy at the computed
2.   WoN the rebate agreement between Lumibao and Trinidad is premium, Lumibao offered to return to him (Trinidad) the amount
enforeceable.—NO. Trinidad’s checks were issued for the account of corresponding to her (Lumibao) commission out of the first premium
Manila Bankers and so encashed by the insurance company as the payment, which is equivalent to 50% of the 1st premium payment. Trinidad
payee. Trinidad thus cannot "demand the return of what he has given" agreed to take the policy thus, on April 30, 1975, he issued 2 checks in
from Lumibao because he did not, strictly speaking, pay the amount of favor of the insurance company for P46,590 each or a total of P93,180.
P93,180.00 to Lumibao. Public policy considerations serve to 202.  Both checks were postdated May 30, 1975 so as to enable Lumibao to make
underscore further the Court's foregoing ruling that Lumibao's promise arrangements for the return to Trinidad of one check corresponding to the
of rebate, which is expressly prohibited by law, may not be enforced for amount of her commission.
compliance by the courts. 203.  Lumibao received P51,249 from Bescon Insurance Agencies, Inc. as her
commission. Yet, petitioner failed to comply with her commitment to pay
DOCTRINE: Section 361 of Pres. Decree No. 612 is similar to the so-called private Trinidad P46,590.00.
"anti-discrimination" statutes found in other jurisdictions which regulate the 204.  Trinidad, through his counsel, sent a demand. Lumibao denied that she had
activities in the insurance industry. The purpose of these statutes is the entered into such an arrangement with Trinidad.
prevention of unfair discriminatory practices by insurance companies, agents and 205.  Trinidad filed an action for specific performance and damages. Lumibao
brokers in order to ensure that equal terms are fixed for policyholders of the denied that she had made a verbal promise to return the 50% of the
same insurable class and equal expectation of life. In aid and furtherance of this premium.
desirable policy, the statutes prohibit such practices involving rebates or 206.  TRIAL COURT: The trial court made a categorical finding that petitioner
preferential treatment with respect to the cost of the policy or the benefits had induced private respondent to take out a life insurance policy with
allowed for the premium. It follows that to enforce contracts or agreements Manila Bankers Life Insurance Corporation by promising a rebate of 50%
directly forbidden under these statutes, thereby allowing recovery thereunder, of his first annual premium payment on said policy. However, despite such
finding, the trial court ordered the dismissal of Trinidad’s complaint, be issued as evidence thereof, or shall directly or shall indirectly, by
holding that it could not grant any relief because the agreement entered into giving or sharing a commission or in any manner whatsoever, pay or
between the parties was void for being contrary to the provisions Insurance allow or offer to pay or allow to the insured or to any employee of such
Code and public policy. insured, either as an inducement to the making of such insurance or
207.  IAC: Not satisfied with the decision, Lumibao interposed an appeal with after such insurance has been effected, any rebate from the premium
IAC. Appellate court affirmed the factual findingsof the trial court and which is specified in the policy, or any special favor or advantage in the
sustained the dismissal of Lumibao’s counterclaim. But in a split decision, dividends or other benefits to accrue thereon, or shall give or offer to give
*IAC reversed the trial court's judgment in so far as it dismissed the any valuable consideration or inducement of any kind, directly or indirectly,
complaint, and instead ordered Lumibao to Trinidad the sum of P46,590.00, which is not specified in such policy or contract of insurance; nor shall any
with interest thereon. such company, or any agent thereof, as to any policy or contract of
208.  Hence this petition. insurance issued, make any discrimination against any Filipino in the sense
ISSUE/s: that he is given less advantageous rates, dividends or other policy
158.  WoN Lumibao violated Section 361 (now Section 370) of the insueance conditions or privileges than are accorded to other nationals because of his
code and therefore be held liable—YES. A preponderance of evidence on race
record supports the findings of the courts that Lumibao had induced 161.  A preponderance of evidence on record supports the findings of the courts
Trinidad to take out a life insurance policy from Manila by promising him a that Lumibao had induced Trinidad to take out a life insurance policy from
rebate equivalent to 50% of the first annual premium payment. It is evident Manila by promising him a rebate equivalent to 50% of the first annual
that petitioner's promise to pay private respondent an amount equivalent to premium payment. These factual findings are, therefore, final and binding
50% of the first premium payment, which would be taken out of her upon the Court.
commission on the insurance policy, is covered squarely by the express 162.  Petitioner, however, argues that in view of the last paragraph of Article
provisions of Section 361. 1358 of the New Civil Code, such contract must be in writing (exceeding
159.  WoN the rebate agreement between Lumibao and Trinidad is P500), and since it was not, it should not be given credence by the court.
enforeceable.—NO. Trinidad’s checks were issued for the account of 163.  This contention is patently erroneous reliance on Article 1358 is misplaced
Manila Bankers and so encashed by the insurance company as the payee. for the apparent reason that this article does not lay down any evidentiary
Trinidad thus cannot "demand the return of what he has given" from rule which precludes oral testimony as a means of proving that parties have
Lumibao because he did not, strictly speaking, pay the amount of entered into a contract or agreement involving an amount of more than P
P93,180.00 to Lumibao. Public policy considerations serve to underscore 500.
further the Court's foregoing ruling that Lumibao's promise of rebate, which 164.  Neither can it be gainsaid that Libunao, an insurance agent, is enjoined by
is expressly prohibited by law, may not be enforced for compliance by the law from inducing prospective clients to take out insurance by offering
courts. rebates from the premiums specified in the insurance policies. (Check
Section 361 above)
RULING: WHEREFORE, the assailed decision of respondent appellate court in 165.  Violation of Sec 361 constitutes a ground for the immediate revocation
AC-G.R. CV No. 61200 is SET ASIDE, and the trial court's decision in Civil Case of the license issued to the erring insurance company, agent or broker
No. 3653 is hereby REINSTATED. Let the Insurance Commissioner be furnished a and the imposition of a fine not exceeding five hundred pesos.
copy of this decision for appropriate administrative action against petitioner pursuant 166.  It is evident that petitioner's promise to pay private respondent an amount
to Section 363 of Pres. Decree No. 961. equivalent to 50% of the first premium payment, which would be taken out
of her commission on the insurance policy, is covered squarely by the
RATIO: express provisions of Section 361.
160.  Section 361 of Pres. Decree No. 612 states: (Now Sec 370 of Insurance 167.  On the second issue: SC ruled that IAC committed reversible error of law in
Code) ordering petitioner to pay private respondent the promised rebate of
P46,590.00.
No insurance company doing business in the Philippines or any agent 168.  By virtue of Article 1409 (7) of the New Civil Code, the rebate agreement
thereof, no insurance broker, and no employee or other representative between the petitioner and private respondent is deemed a contract void ab
of any such insurance company, agent, or broker, shall make, procure or initio, and, consequently, does not give rise to enforceable rights and
negotiate any contract of insurance or agreement as to policy contract, other obligations as between the parties thereto.
than is plainly expressed in the policy or other written contract issued or to 169.  IAC opined that since the prohibition against rebate agreements under
Section 361, and the penalty imposed therefor under Section 363, refer only
to insurance agents, brokers or companies, and that Article 1412 (2) of the
New Civil Code provides the legal basis for allowing Trinidad, the party
who is not at fault, to recover from Lumibao.
170.  The SC disagrees. Although there are instances where the law recognizes
the right of an innocent party to recover what he has paid or delivered under
the agreement but contrary to the ruling of IAC, the case at bar does not fall
under any of the legal exceptions.
171.  IAC misapplied Art. 1412 (2). The article contemplates of a situation where
the party who is not at fault, in the performance of his undertaking with the
party who is at fault, has paid or delivered property to the latter. This is not
the case between private respondent and petitioner.
172.  The agreement between the parties consists of an undertaking on the part of
Trinidad to take out a life insurance policy with Manila Bankers and, on the
part of Lumibao, to give a rebate on the first premium payment. There is no
indication that Lumibao, when she made the promise, had acted with the
knowledge and under the authority of the insurance company. But it is clear
that the premium of P93,180 was actually paid to the insurance company in
consideration of the policy taken out.
173.  Trinidad’s checks were issued for the account of Manila Bankers and so
encashed by the insurance company as the payee. Trinidad thus cannot
"demand the return of what he has given" from Lumibao because he did not,
strictly speaking, pay the amount of P93,180.00 to Lumibao
174.  Public policy considerations serve to underscore further the Court's
foregoing ruling that Lumibao's promise of rebate, which is expressly
prohibited by law, may not be enforced for compliance by the courts.
175.  Section 361 of Pres. Decree No. 612 is similar to the so-called "anti-
discrimination" statutes found in other jurisdictions which regulate the
activities in the insurance industry. The purpose of these statutes is the
prevention of unfair discriminatory practices by insurance companies,
agents and brokers in order to ensure that equal terms are fixed for
policyholders of the same insurable class and equal expectation of life.
In aid and furtherance of this desirable policy, the statutes prohibit
such practices involving rebates or preferential treatment with respect
to the cost of the policy or the benefits allowed for the premium. It follows
that to enforce contracts or agreements directly forbidden under these
statutes, thereby allowing recovery thereunder, would be subversive of
the very public policy which the law was designed and intended to
uphold. True, the statutes, like Sections 361 and 363 of Pres. Decree No.
961, in terms, are addressed to the insurance companies, agents and brokers,
and is enacted for the protection of policyholders, but this is for the general
body of policyholders who would suffer by the enforcement of the
prohibited agreements, and not for those who have entered into such
agreements and are seeking to profit by its terms.
013 SMITH, BELL & CO Inc. vs CA (STA. MARIA) The Insurance Code (Sec. 190) is quite clear as to the purpose and role of a resident
February 6, 1997 | Panganiban, J. | Adjuster agent. Such agent, as a representative of the foreign insurance company, is tasked
only to receive legal processes on behalf of its principal and not to answer personally
PETITIONER: Smith, Bell & Co, Inc. for any insurance claims.
RESPONDENTS: Court of Appeals and Joseph Bengzon Chua
FACTS:
SUMMARRY: Joseph Chua doing business under the style of Tic Hin Chiong 1.   Joseph Chua doing business under the style of Tic Hin Chiong , Importer,
Importer, bought and imported to the Philippines 50 metric tons of Dicalcium bought and imported to the Philippines from the firm Chin Gact Co. Ltd. of
Phospate, Feed Grade. These were contained in 1,250 bags. This shipment was Taipei ,Taiwan, 50 metric tons of Dicalcium Phospate, Feed Grade F - 15%
insured by First Insurance Co against all risks at port of departure under Marine valued at US$13,000 . 00 CIF Manila .
Policy, with the note `Claim,if any, payable in US currency at Manila and with Smith, 2.   These were contained in 1,250 bags and shipped from the Port of
Bell & Co. Inc stamped at the lower left side of the policy as `Claim Agent. The Kaohsiung, Taiwan on Board SS GOLDEN WEALTH for the Port in
cargo arrived which was discharged to the local arrastre contractor, Metroport Manila .
Services Inc. with a number of the cargo in apparent bad order condition. 600 of the 3.   This shipment was insured by First Insurance Co . for US$19, 500`against
1,250 bags of the imported material were damaged by tearing at the sides of the all risks at port of departure under Marine Policy, with the note `Claim , if
container bags and the contents partly empty. Upon weighing, the contents of the any, payable in US currency at Manila and with Smith, Bell & Co. Inc
damaged bags were found to be 18,546 kg short . Joseph Chua filed with Smith, Bell stamped at the lower left side of the policy as `Claim Agent
& Co. Inc., a formal statement of claim with proof of loss and a demand for 4.   The cargo arrived at the Port of Manila and the entire cargo was discharged
settlement of the corresponding value of the losses. After purportedly conveying the to the local arrastre contractor, Metroport Services Inc. with a number of the
claim to its principal, Smith, Bell & Co. Inc., informed Joseph Chua by letter that its cargo in apparent bad order condition .
principal, First Insurance, offered only 50% of the claim as redress on the alleged 5.   Joseph Chua secured the services of a cargo surveyor to conduct a survey of
ground of discrepancy. The offer not being acceptable to Joseph Chua, he filed a case the damaged cargo which delivered by Joseph Chua’s broker to his
against Smith, Bell & Co. and First Insurance. Smith, Bell & Co. Inc denied liability premises at 12th Avenue,Grace Park, Caloocan City .
and said that it is merely a settling or claim agent of First Insurance Company and as 6.   The surveyor’s report showed that 600 of the 1,250 bags of the imported
such agent it is not personally liable under the policy in which it has not even taken material were damaged by tearing at the sides of the container bags and the
part of. The First Insurance Co. Ltd did not file an Answer, hence it was declared in contents partly empty. Upon weighing, the contents of the damaged bags
default. The lower court ruled that Joseph Chua has fully established the liability of were found to be 18,546 kg short.
First Insurance Co. As regards Smith, Bell & Co. Inc, it held that since it is 7.   Joseph Chua filed with Smith, Bell & Co. Inc., a formal statement of
admittedly a claim agent of the foreign insurance firm doing business in the claim with proof of loss and a demand for settlement of the corresponding
Philippines justice is better served if said agent is made liable without prejudice to its value of the losses.
right of action against its principal, First Insurance. The issue is WON a local settling 8.   After purportedly conveying the claim to its principal, Smith, Bell & Co.
or claim agent (Smith, Bell & Co) of a disclosed principal (First Insurance)-- a Inc., informed Joseph Chua by letter that its principal offered only 50% of
foreign insurance company-- can be held jointly and severally liable with said the claim as redress on the alleged ground of discrepancy between the
principal under the latter’s marine cargo insurance policy, given that the agent is not a amounts contained in the shipping agent’s reply to the claimant of only
party to the insurance contract? – NO. Undisputedly a settling agent acting within the US$90.48 with that of Metroport’s .
scope of its authority, cannot be held personally and/or solidarily liable for the 9.   The offer not being acceptable to Joseph Chua , the latter wrote Smith, Bell
obligations of its disclosed principal merely because there is allegedly a need for a & Co. Inc., expressing his refusal to the `redress offer , contending that the
speedy settlement of the claim of Joseph Chua. (see doctrine) Thus, Smith, Bell & Co discrepancy was a result of loss from vessel to arrastre to
Inc cannot be held personally liable. consignees warehouse which losses were still within the `all
risk insurance cover. No settlement of the claim having been made, Joseph
DOCTRINE: The scope and extent of the functions of an adjustment and settlement Chua then caused the instant case to be filed.
agent do not include personal liability. His functions are merely to settle and adjusts 10.   Denying any liability, Smith, Bell & Co. Inc., averred that it is merely a
claims in behalf of his principal if those claims are proven and undisputed, and if the settling or claim agent of First Insurance Company and as such agent it is
claim is disputed or is disapproved by the principal, like in the instant case, the agent not personally liable under the policy in which it has not even taken part of
does not assume any personal liability. The recourse of the insured is to press his . It then alleged Joseph Chua has no cause of action against it. The First
claim against the principal."
Insurance Co . Ltd . did not file an Answer, hence it was declared in the employer's interest, the employer is responsible so long as the acts are done
default while the agent is acting within the scope of his employment.'
11.   The lower court ruled that Joseph Chua has fully established the liability of
First Insurance Co. on the subject insurance contract as the former It, therefore, clearly appears that the scope and extent of the functions of an
presented concrete evidence of the amount of losses resulting from the risks adjustment and settlement agent do not include personal liability. His functions
insured against which were supported by reliable report and assessment of are merely to settle and adjusts claims in behalf of his principal if those claims are
professional cargo surveyor. proven and undisputed, and if the claim is disputed or is disapproved by the
12.   As regards Smith, Bell & Co. Inc, the lower court held that since it is principal, like in the instant case, the agent does not assume any personal liability.
admittedly a claim agent of the foreign insurance firm doing business in the The recourse of the insured is to press his claim against the principal."
Philippines justice is better served if said agent is made liable without 3.   Joseph Chua’s contention that Salonga does not apply simply because only
prejudice to its right of action against its principal , the insurance firm . the agent was sued therein while here both agent and principal were
impleaded and found solidarily liable is without merit. Such distinction is
ISSUE: immaterial. The agent can not be sued nor held liable whether singly or
solidarily with its principal.
1.   WON a local settling or claim agent (Smith, Bell & Co) of a disclosed 4.   Every cause of action ex contractu must be founded upon a contract, oral or
principal (First Insurance)-- a foreign insurance company-- can be held written, either express or implied. The only "involvement" of Smith, Bell
jointly and severally liable with said principal under the latter’s marine & Co. Inc., in the subject contract of insurance was having its name
cargo insurance policy, given that the agent is not a party to the insurance stamped at the bottom left portion of the policy as "Claim Agent."
contract? – NO. Undisputedly a settling agent acting within the scope of its Without anything else to back it up, such stamp cannot even be deemed
authority, cannot be held personally and/or solidarily liable for the by the remotest interpretation to mean that Smith, Bell & Co. Inc.,
obligations of its disclosed principal merely because there is allegedly a participated in the preparation of said contract. Hence, there is no
need for a speedy settlement of the claim of Joseph Chua. privity of contract, and correspondingly there can be no obligation or
RULING: WHEREFORE ,in view of the foregoing considerations ,the Petition liability, and thus no cause of action against Smith, Bell & Co. Inc.,
is GRANTED and the Decision appealed from is REVERSED and SET ASIDE .No attaches.
costs . 5.   Under Article 1311 of the Civil Code, contracts are binding only upon the
parties (and their assigns and heirs) who execute them. The subject cargo
RATIO: insurance was between the First Insurance Company, Ltd. and the Chin
1.   There are three reasons why we find for petitioner. Gact Co., Ltd., both of Taiwan, and was signed in Taipei, Taiwan by the
2.   First Reason: Existing Jurisprudence. Smith, Bell & Co. Inc., president of the First Insurance Company, Ltd. and the president of the Chin
undisputedly a settling agent acting within the scope of its authority, cannot Gact Co., Ltd. There is absolutely nothing in the contract which mentions
be held personally and/or solidarily liable for the obligations of its disclosed the personal liability of Smith, Bell & Co. Inc.,
principal merely because there is allegedly a need for a speedy settlement of 6.   Second Reason: Absence of Solidary Liability. May then Smith, Bell &
the claim of private respondent. In the leading case of Salonga vs. Warner, Co. Inc.,, in its capacity as resident agent (as found in the case cited by the
Barnes & Co., Ltd. this Court ruled in this wise: CA be held solidarily liable with the foreign insurer? Article 1207 of the
"xxx An adjustment and settlement agent is no different from any other agent from Civil Code clearly provides that "(t)here is a solidary liability only when the
the point of view of his responsibilty (sic), for he also acts in a representative obligation expressly so states, or when the law or the nature of the
capacity. Whenever he adjusts or settles a claim, he does it in behalf of his principal, obligation requires solidarity."
and his action is binding not upon himself but upon his principal. And here again, the 7.   The well-entrenched rule is that solidary obligation cannot lightly be
ordinary rule of agency applies. The following authorities bear this out: inferred. It must be positively and clearly expressed. The contention that, in
the end, it would really be First Insurance Company, Ltd. which would be
'An insurance adjuster is ordinarily a special agent for the person or company held liable is specious and cannot be accepted. Such a stance would inflict
for whom he acts, and his authority is prima facie coextensive with the business injustice upon Smith, Bell & Co. Inc., which would be made to advance the
intrusted to him. . .' funds to settle the claim without any assurance that it can collect from the
principal which disapproved such claim, in the first place. More
'An adjuster does not discharge functions of a quasi-judicial nature, but importantly, such position would have absolutely no legal basis.
represents his employer, to whom he owes faithful service, and for his acts, in
8.   The Insurance Code is quite clear as to the purpose and role of a resident 11.   The cause of action of Joseph Chua is based on a contract of insurance
agent. Such agent, as a representative of the foreign insurance which as already shown was not participated in by Smith, Bell & Co.
company, is tasked only to receive legal processes on behalf of its Inc.,. It is not a "person who claim(s) an interest adverse to the plaintiff"
principal and not to answer personally for any insurance claims. We nor is said respondent "necessary to a complete determination or settlement
quote: of the questions involved" in the controversy. Smith, Bell & Co. Inc., is
"SEC. 190. The Commissioner must require as a condition precedent to the improperly impleaded for not being a real-party-interest. It will not benefit
transaction of insurance business in the Philippines by any foreign insurance or suffer in case the action prospers.
company, that such company file in his office a written power of attorney 12.   Resort to Equity Misplaced. Finally, the CA also contends that "the interest
designating some person who shall be a resident of the Philippines as its general of justice is better served by holding the settling agent jointly and severally
agent, on whom any notice provided by law or by any insurance policy, proof of loss, liable with its principal." As no law backs up such pronouncement, the
summons and other legal processes may be served in all actions or other legal appellate Court is thus resorting to equity. However, equity which has been
proceedings against such company, and consenting that service upon such general aptly described as "justice outside legality," is availed of only in the absence
agent shall be admitted and held as valid as if served upon the foreign company at of, and never against, statutory law or judicial pronouncements. Upon the
its home office. Any such foreign company shall, as further condition precedent to other hand, the liability of agents is clearly provided for by our laws and
the transaction of insurance business in the Philippines, make and file with the existing jurisprudence.
Commissioner an agreement or stipulation, executed by the proper authorities of said |||
company in form and substance as follows:
'The (name of company) does hereby stipulate and agree in consideration of the
permission granted by the Insurance Commissioner to transact business in the
Philippines, that if at any time such company shall leave the Philippines, or cease to
transact business therein, or shall be without any agent in the Philippines on whom
any notice, proof of loss, summons, or legal process may be served, then in any
action or proceeding arising out of any business or transaction which occurred in the
Philippines, service of any notice provided by law, or insurance policy, proof of loss,
summons, or other legal process may be made upon the Insurance Commissioner
shall have the same force and effect as if made upon the company.'
Whenever such service of notice, proof of loss, summons, or other legal process shall
be made upon the Commissioner he must, within ten days thereafter, transmit by
mail, postage paid, a copy of such notice, proof of loss, summons, or other legal
process to the company at its home or principal office. The sending of such copy of
the Commissioner shall be necessary part of the service of the notice, proof of loss,
or other legal process."
9.   Further, we note that in the case cited by the CA, Smith, Bell & Co. Inc.,
was found to be a resident agent of First Insurance Co. Ltd. In the instant
case however, the trial court had to order the service of summons upon First
Insurance Co., Ltd. which would not have been necessary if Smith, Bell &
Co. Inc., was its resident agent. Indeed, from our reading of the records of
this case, we find no factual and legal bases for the finding of the CA that
Smith, Bell & Co. Inc., is the resident agent of First Insurance Co., Ltd. Cdt
10.   Third Reason: Not Real Party-In-Interest. Lastly, being a mere agent and
representative, Smith, Bell & Co. Inc., is also not the real party-in-
interest in this case. An action is brought for a practical purpose, that is, to
obtain actual and positive relief. If the party sued is not the proper party,
any decision that may be rendered against him would be futile, for the
decision cannot be enforced or executed.
014 VALENZUELA v. CA (TAN edited from VARGAS) 17.   From 1973-1975, Valenzuela solicited marine insurance from one of his
October 19, 1990 | Gutierrez, Jr., J. | Parties to the Contract clients Delta Motors amounting to P4.4M from which he was entitled to a
commission of 32%. However, Valenzuela did not receive his commission
PETITIONER: Arturo P. Valenzuela and Hospitalita N. Valenzuela amounting to P1.6M. During 1976-1978, premium payments were paid
RESPONDENTS: Hon. Court of Appeals, Bienvenido M. Aragon, Robert E. directly to Philamgen.
Parnell, Carlos K. Catolico, and the Philippine American General Insurance 18.   In 1977, Philamgen started to become interested in and expressed its intent
Company, Inc. to share in the commission due Valenzuela on a fifty-fifty basis.
SUMMARY: Valenzuela is the General Agent for Philamgen (insurance Valenzuela refused. Philamgen insisted on the sharing of commission with
company). Valenzuela solicited marine insurance from Delta Motors from which Valenzuela. Valenzuela firmly reiterated his objection to the proposals of
he was entitled to a commission of 32.5% from Philamgen. In 1977, Philamgen Philamgen.
started to become interested in and expressed its intent to share in the 19.   Because of the refusal of Valenzuela, Philamgen took drastic action against
commission due Valenzuela on a 50-50 basis thus giving Valenzuela an agent’s Valenzuela:
commission of 16.25%. Despite several attempts to convince Valenzuela, he still e.   Reversed the commission due him by not crediting in his account the
refused to accept such proposals to share commission with Philamgen. commission earned from the Delta Motors, Inc.
Consequently, Philamgen did some acts of harassment against Valenzuela which f.   Placed agency transactions on a cash and carry basis
led to the decline of Valenzuela’s business as insurance agent. Eventually, g.   Threatened the cancellation of policies issued by his agency
Philamgen terminated the General Agency Agreement with Valenzuela. The h.   Started to leak out news that Valenzuela has a substantial account with
issues in this case are Philamgen.
3.   WON Philamgen and/or its officers can be held liable for damages All of these acts resulted in the decline of his business as insurance agent.
due to the termination of the General Agency Agreement it entered Then on December 27, 1978, Philamgen terminated the General Agency
into with Valenzuela – YES. The agency involving Valenzuela and Agreement of Valenzuela.
Philamgen is one "coupled with an interest," and, therefore, should 20.   Vaenzuela filed a complaint against Philamgen.
not be freely revocable at the unilateral will of the latter. The principal 21.   Trial Court ruled in favor of Valenzuela stating that the principal cause of
cause of the termination of Valenzuela as General Agent of Philamgen the termination of Valenzuela as General Agent of Philamgen was based on
arose from his refusal to share his Delta Commission. Such termination, his refusal to share his Delta Commission.
as held by the Trial Court, is tainted with bad faith. As such, 22.   The acts of harassment done by Philamgen to force Valenzuela to agree to
4.   WON Valenzuela is liable to Philamgen for the unpaid and the sharing of his Delata commission do not justify the termination of the
uncollected premiums – NO. The remedy for the non-payment of General Agency Agreement entered into by Philamgen and Valenzuela.
premiums is to put an end to and render the insurance policy not 23.   CA ruled in favor of Philamgen stating that the principal’s power to revoke
binding. an agency at will is so pervasive that termination may be effected even if
the principal acts in bad faith, subject only to the principal’s liability for
DOCTRINE: Valenzuela, being a mere agent of Philamgen, is not a party to the damages.
contract of insurance. The contract is essentially between Philamgen and Delta
Motors. Consequently, Valenzuela is under no obligation to pay Philamgen for ISSUE:
the unpaid and uncollected premiums. The remedy for the non-payment of WON Philamgen and/or its officers can be held liable for damages due to the
premiums is to put an end to and render the insurance policy not binding. termination of the General Agency Agreement it entered into with Valenzuela –
YES. The agency involving Valenzuela and Philamgen is one "coupled with an
interest," and, therefore, should not be freely revocable at the unilateral will of the
FACTS: latter. The principal cause of the termination of Valenzuela as General Agent of
15.   Characters: Philamgen arose from his refusal to share his Delta Commission. Such termination,
Valenzuela – General Agent of Philamgen as held by the Trial Court, is tainted with bad faith.
Philamgen – Insurance Company WON Valenzuela is liable to Philamgen for the unpaid and uncollected
Delta Motors – Insured premiums – NO. The remedy for the non-payment of premiums is to put an end to
16.   Valenzuela is a General Agent of Philamgen since 1965. As such, he was and render the insurance policy not binding.
authorized to solicit and sell in behalf of Philamgen all kinds of non-life
insurance, and in sconsideration of services rendered was entitled to receive RULING: ACCORDINGLY, the petition is GRANTED. The impugned decision of
the full agent’s commission of 32.5% from Philamgen.
January 29, 1988 and resolution of April 27, 1988 of respondent court are hereby that the Philamgen started to covet a share of the insurance business that
SET ASIDE. The decision of the trial court dated January 23, 1986 in Civil Case No. Valenzuela had built up, developed and nurtured to profitability through
121126 is REINSTATED with the MODIFICATIONS that the amount of FIVE over thirteen (13) years of patient work and perseverance. When Valenzuela
HUNDRED TWENTY ONE THOUSAND NINE HUNDRED SIXTY-FOUR AND refused to share his commission in the Delta account, the boom suddenly
16/100 PESOS (P521,964.16) representing the petitioners Delta commission shall fell on him.
earn only legal interests without any adjustments under Article 1250 of the Civil 182.  With the termination of the General Agency Agreement, Valenzuela would
Code and that the contractual relationship between Arturo P. Valenzuela and no longer be entitled to commission on the renewal of insurance policies of
Philippine American General Insurance Company shall be deemed terminated upon clients sourced from his agency. Worse, despite the termination of the
the satisfaction of the judgment as modified. agency, Philamgen continued to hold Valenzuela jointly and severally
liable with the insured for unpaid premiums. Under these circumstances,
RATIO: it is clear that Valenzuela had an interest in the continuation of the
176.  The principal cause of the termination of Valenzuela as General Agent of agency when it was unceremoniously terminated not only because of the
Philamgen arose from his refusal to share his Delta commission. The commissions he should continue to receive from the insurance business he
records sustain the conclusions of the trial court on the apparent bad faith of has solicited and procured but also for the fact that by the very acts of the
the Philamgen in terminating the General Agency Agreement of respondents, he was made liable to Philamgen in the event the insured fail
Valenzuela. In the case at bar, the records show that the findings and to pay the premiums due. They are estopped by their own positive
conclusions of the trial court are supported by substantial evidence and averments and claims for damages. Therefore, the respondents cannot state
there appears to be no cogent reason to disturb them. that the agency relationship between Valenzuela and Philamgen is not
177.  As early as September 30,1977, Philamgen told the petitioners of its desire coupled with interest.
to share the Delta Commission with them. It stated that should Delta back
out from the agreement, Valenzuela would be charged interests through a As to Valenzuela’s liability to pay Philamgen for the unpaid and uncollected
reduced commission after full payment by Delta. premiums
178.  On January 23, 1978 Philamgen proposed reducing the Valenzuela’s 183.   We rule that the respondent court erred in holding Valenzuela liable. We find no factual and
legal basis for the award. Under Section 7767 of the Insurance Code, the remedy for the non-
commissions by 50% thus giving them an agent's commission of 16.25%.
payment of premiums is to put an end to and render the insurance policy not binding.
On February 8, 1978, Philamgen insisted on the reduction scheme. There 184.   Philippine Phoenix Surety and Insurance, Inc. v. Woodworks, Inc.: Non- payment of premium
were other pressures: does not merely suspend but puts an end to an insurance contract since the time of the payment
a.   The petitioners were told that the Delta commissions would not be is peculiarly of the essence of the contract.
credited to their account 185.  Since admittedly the premiums have not been paid, the policies issued have
b.   They were informed that the Valenzuela agency would be placed on a lapsed. The insurance coverage did not go into effect or did not continue
cash and carry basis thus removing the 60-day credit for premiums due and the obligation of Philamgen as insurer ceased. Hence, for Philamgen
c.   Existing policies were threatened to be cancelled which had no more liability under the lapsed and inexistent policies to
d.   The Valenzuela business was threatened with diversion to other agencies demand, much less sue Valenzuela for the unpaid premiums would be the
e.   Rumors were also spread about alleged accounts of the Valenzuela height of injustice and unfair dealing. In this instance, with the lapsing of
agency the policies through the nonpayment of premiums by the insured there
179.  It is also evident from the records that the agency involving Valenzuela and were no more insurance contracts to speak of.
Philamgen is one "coupled with an interest," and, therefore, should not be
freely revocable at the unilateral will of the latter.
180.  In the insurance business in the Philippines, the most difficult and
frustrating period is the solicitation and persuasion of the prospective clients
to buy insurance policies. Normally, agents would encounter much
embarrassment, difficulties, and oftentimes frustrations in the solicitation
and procurement of the insurance policies. To sell policies, an agent exerts
                                                                                                                       
great effort, patience, perseverance, ingenuity, tact, imagination, time and 67
Sec. 77 ... [N]otwithstanding any agreement to the contrary, no policy or contract of insurance is valid
money. and binding unless and until the premiums thereof have been paid except in the case of a life or industrial
181.  In the case of Valenzuela, he was able to build up an Agency from scratch life policy whenever the grace period provision applies (P.D. 612, as amended otherwise known as the
in 1965 to a highly productive enterprise. The records sustain the finding Insurance Code of 1974)
015 PHILAMLIFE v. ANSALDO (TIMBOL) supervisors, managers and public consumers of the Philippine American
July 26, 1994| Quiason, J. | Insurance Commissioner Life Insurance Company (PHILAMLIFE) as a result of certain practices
by said company
PETITIONER: Philippine American Life Insurance Company and Rodrigo De Los 135.  ANSALDO requested Rodrigo de los Reyes (DE LOS REYES), in his
Reyes capacity as PHILAMLIFE’s president, to comment on PATERNO’s letter
RESPONDENTS: Hon. Armando Ansaldo, Commissioner, and Ramon Montilla 136.  DE LOS REYES suggested that PATERNO submit some sort of a bill of
Paterno particulars listing and citing actual cases, facts, dates, figures, provisions
of law, rules and regulations, and all other pertinent data which are
SUMMARY: PATERNO sent a letter-complaint to COMM. ANSALDO alleging necessary to enable him to prepare an intelligent reply
certain problems encountered by agents, supervisors, managers and public 137.  ANSALDO received a letter from PATERNO maintaining that his letter-
consumers of PHILAMLIFE, particularly regarding the charges and fees in the complaint was sufficient in form and substance, and requested that a
CONTRACT OF AGENCY. DE LOS REYES, President of PHILAMLIFE, hearing thereon be conducted
answered to PATERNO’s letter stating that it was not sufficient in form and 138.  DE LOS REYES reiterated his claim that PATERNO’s letter did not supply
substance, and that since the COMMISIONER’s Quasi Judicial Power was being the information he needed to enable him to answer the letter-complaint
invoked, PATERNO should file a verified formal complaint. Upon submission, 139.  A hearing on the letter-complaint was held ANSALDO on the validity of
COMM. ANSALDO sent a notice of hearing to both parties. PHILAMLIFE (through the CONTRACT OF AGENCY complained of by PATERNO
Ortega its AVP) filed a Motion to Quash Subpoena/Notice on two grounds: (1) 140.  PATERNO was required by ANSALDO to specify the provisions of the
Subpoena/Notice has no legal basis and is premature; and (2) The Insurance agency contract which he claimed to be illegal
Commision has no jurisdiction over the subject matter nad the parties involved. 1.   PATERNO submitted a letter of specification and praying that
COMM. ANSALDO denied the Motion to Quash. Hence this petition. the provisions on charges and fees stated in the CONTRACT
OF AGENCY executed between PHILAMLIFE and its agents,
Whether or not COMM. ANSALDO has jurisdiction to rule over the CONTRACT as well as the implementing provisions as published in the
OF AGENCY. agent’s handbook, agency bulletins and circulars, be declared
as NULL AND VOID
The SC held that the Insurance Commissioner has no jurisdiction. First, the 2.   He also asked that the amounts of such charges and fees
Insurance Code under Sec. 415 provides that the COMMISIONER may only already deducted and collected by PHILAMLIFE in
regulate the business of insurance. The Contract of Agency is not considered as connection therewith be reimbursed to the agents, with interest
doing business of insurance. Second, althought Sec. 416 provides QUASI- at the prevailing rate reckoned from the date when they were
JUDICIAL POWERS to the COMMISSIONER, such powers are limited only to deducted
claims and complaints involving any loss, damage or liability, for which an insurer 141.  DE LOS REYES submitted an Answer:
may be answerable under any kind of policy or contract of insurance. Third, the 1.   PATERNO’s letter does not contain any of the particular
Insurance Code does not have provisions governing the relations between insurance information which PHILAMLIFE was seeking from him and
companies and their agents. Lastly, there are two categories of employees in an which he promised to submit
insurance company: (1) Salaried employees through a Contract of Employment, 2.   Since the Commission’s QUASI-JUDICIAL POWER was
which are governed by the Labor Code; and (2) Registered Representatives paid by being invoked with regard to the complaint, PATERNO must
commission, through a Contract of Agency. And disputes involving the latter are file a VERIFIED formal complaint before any further
within the jurisdiction of the regular courts. proceedings
142.  PATERNO executed an affidavit, verifying his letters
DOCTRINE: The Insurance Code does not have provisions governing the relations 143.  Manuel Ortega, PHILAMLIFE’s Senioer AVP and Executive Assistant to
between insurance companies and their agents. It follows that the Insurance the President, asked that Commission first rule on the questions of the
Commissioner cannot, in the exercise of its quasi-judicial power, assume jurisdiction jurisdiction of the Insurance Commissioner over the subject matter of the
over controversies between the insurance companies and their agents letters-complaint and the legal standing of PATERNO
144.  ANSALDO notified both parties of the hearing of the case
1.   Ortega filed a MOTION TO QUASH Subpoena/Notice on the
FACTS:
following grounds:
134.  A letter-complaint of Ramon M. Paterno, Jr. (PATERNO) to Commissioner
Ansaldo (ANSALDO), alleging certain problems encountered by agents,
1.   The Subpoena/Notice has no legal basis and is premature 2.   Suspension, or after due hearing, removal of directors
because: and/or officers and/or agents
a.   No complaint sufficient in form and contents has been 148.  A plain reading of the above-quoted provisions show that the Insurance
filed; Commissioner has the authority to regulate the business of insurance,
b.   No summons has been issued nor received by DE LOS which is defined as follows:
REYES, and hence no jurisdiction has been acquired 1.   The term “doing an insurance business” or “transacting an
over his person insurance business,” within the meaning of this Code, shall
c.   No answer has been filed, hence, Subpoena/Notice is include:
premature and lacks legal basis 1.   Making or proposing to make, as insurer, any insurance
2.   The Insurance Commission has no jurisdiction over: contract;
1.   The subject matter or nature of the action; AND 2.   Making or proposing to make, as surety, any contract of
2.   Over the parties involved suretyship as a vocation and not as merely incidental to
145.  ANSALDO denied the Motion to Quash. Hence this appeal. any other legitimate business or activity of the surety;
3.   Doing any kind of business, including a reinsurance
ISSUE/s: business, specifically recognized as constituting the
16.   WoN the resolution of the legality of the Contract of Agency falls within doing of an insurance business within the meaning of this
the jurisdiction of the Insurance Commissioner – NO, because disputes Code;
regarding the Contract of Agency are cognizable by the regular courts. A 4.   Doing or proposing to do any business in substance
Contract of Agency, moreover, is not considered an insurance business, equivalent to any of the foregoing in a manner designed
which the Commissioner may regulate. Also, the quasi-judicial power of the to evade the provisions of this Code
Commissioner may not be invoked because it is limited to claims and 149.  Since the CONTRACT OF AGENCY entered into between
damages for which an insurer may be liable under any kind of policy or a PHILAMLIFE and its agents is NOT INCLUDED within the meaning
contract of insurance of an insurance business, Sec.2 of the Insurance Code CANNOT be
invoked to give jurisdiction over the same to the Insurance
RULING: WHEREFORE, the petition is GRANTED. The Order dated November 6, Commissioner (Expressio Unius est exclusion alterius)
1986 of the Insurance Commission is SET ASIDE. 150.  Sec. 416 of the Insurance Code pertaining to the QUASI-JUDICIAL
POWERS of the COMMISSIONER is also not applicable:
RATIO: 1.   The Commissioner shall have the power to adjudicate claims and
146.  The general regulatory authority of the Insurance Commissioner is complaints involving any loss, damage or liability for which an
described in Sec. 414 of the Insurance Code, to wit: insurer may be answerable under any kind of policy or contract of
1.   The Insurance Commissioner shall have the duty to see that all insurance, or for which such insurer may be liable under a contract
laws relating to insurance, insurance companies and other of suretyship, or for whicha reinsurer may be used under any
insurance matters, mutual benefit associations and trusts for contract or reinsurance it may have entered into, or for which a
charitable uses are faithfully executed and to perform the duties mutual benefit association may be held liable under the
imposed upon him by this Code membership certificates it has issued to its members, where the
147.  On the other hand, Sec. 415 provides: amount of any such loss, damage or liability, excluding interest,
1.   In addition to the administrative sanctions provide elsewhere in costs and attorney’s fees, being claimed or sued upon any kind of
this Code, the Insurance Commissioner is hereby authorized, at his insurance, bond reinsurance contract, or membership certificate
discretion, to impose upon insurance companies, their directors does not exceed in any single claim P100,000
and/or officer and/or agents, for any willful failure or refusal to 151.  A reading of the said section shows that the quasi-judicial power of the
comply with, or violation of any provision of this Code, or any Insurance Commissioner is limted by law “to claims and complaints
order, instruction, regulation or ruling of the Insurance involving any loss, damage or liability for which an insurer may be
Commissioner, or any commission of irregularities, and/or answerable under any kind of policy or contract of insurance,”
conducting business in an unsafe and unsound manner as may be 152.  Hence, this power does not cover the relationship affecting the
determined by the Insurance Commissioner, the following: insurance company and its agents but is limited to adjudicating claims
1.   Fines not in excess of P500/day; AND and complaints filed by the insured against the insurance company
153.  While the subject of Insurance Agents and Brokers is discussed under
Chapter IV, Title I of the Insurance Code, the provisions of said Chapter
speak only of the licensing requirements an limitations imposed on
insurance agents and brokers
154.  The Insurance Code does not have provisions governing the relations
between insurance companies and their agents. It follows that the Insurance
Commissioner cannot, in the exercise of its quasi-judicial power, assume
jurisdiction over controversies between the insurance companies and their
agents
155.  We have held in the cases of GREPALIFE v. Judico, and Investment
Planning Corp., v. Social Security Commission, that an insurance company
may have two classes of agents who sell its insurance policies:
1.   Salaried employees who keep definited hours and work under the
control and supervision of the company; AND
2.   Registered representatives, who work on commission basis
156.  Under the first category, the relationship between the insurance company
and its agents is governed by the Contract of Employment and the
provisions of the Labor Code, while udner the second category, the same is
governed by the Contract of Agency and the provisions of the Civil Code on
the Agency
1.   Disputes involving the latter are cognizable by the regular courts

You might also like